Download as pdf or txt
Download as pdf or txt
You are on page 1of 452

Cases in Pediatric Acute Care

Downloaded from https://onlinelibrary.wiley.com/doi/ by National Institute Of Standard, Wiley Online Library on [06/03/2024]. See the Terms and Conditions (https://onlinelibrary.wiley.com/terms-and-conditions) on Wiley Online Library for rules of use; OA articles are governed by the applicable Creative Commons License
Cases in Pediatric Acute Care

Strengthening Clinical Decision Making

Edited by

Andrea M. Kline-Tilford, PhD, CPNP-AC/PC, FCCM, FAAN


C.S. Mott Children’s Hospital
Michigan Medicine
Ann Arbor, MI, USA

Catherine M. Haut, DNP, CPNP-AC/PC, FAANP, CCRN


Nemours/Alfred I. duPont Hospital for Children
Wilmington, DE, USA;
Mednax Pediatrix Medical Group
Baltimore, MD, USA
Downloaded from https://onlinelibrary.wiley.com/doi/ by National Institute Of Standard, Wiley Online Library on [06/03/2024]. See the Terms and Conditions (https://onlinelibrary.wiley.com/terms-and-conditions) on Wiley Online Library for rules of use; OA articles are governed by the applicable Creative Commons License
This edition first published 2020
© 2020 John Wiley & Sons Ltd

All rights reserved. No part of this publication may be reproduced, stored in a retrieval system, or transmitted, in any form or by any means,
electronic, mechanical, photocopying, recording or otherwise, except as permitted by law. Advice on how to obtain permission to reuse material
from this title is available at http://www.wiley.com/go/permissions.

The right of Andrea M. Kline-Tilford and Catherine M. Haut to be identified as the author(s) of this work has been asserted in accordance with law.

Registered Office(s)
John Wiley & Sons, Inc., 111 River Street, Hoboken, NJ 07030, USA
John Wiley & Sons Ltd, The Atrium, Southern Gate, Chichester, West Sussex, PO19 8SQ, UK

Editorial Office
9600 Garsington Road, Oxford, OX4 2DQ, UK

For details of our global editorial offices, customer services, and more information about Wiley products visit us at www.wiley.com.

Wiley also publishes its books in a variety of electronic formats and by print-on-demand. Some content that appears in standard print versions of this
book may not be available in other formats.

Limit of Liability/Disclaimer of Warranty

The contents of this work are intended to further general scientific research, understanding, and discussion only and are not intended and should
not be relied upon as recommending or promoting scientific method, diagnosis, or treatment by physicians for any particular patient. In view
of ongoing research, equipment modifications, changes in governmental regulations, and the constant flow of information relating to the use of
medicines, equipment, and devices, the reader is urged to review and evaluate the information provided in the package insert or instructions for
each medicine, equipment, or device for, among other things, any changes in the instructions or indication of USAge and for added warnings
and precautions. While the publisher and authors have used their best efforts in preparing this work, they make no representations or warranties
with respect to the accuracy or completeness of the contents of this work and specifically disclaim all warranties, including without limitation
any implied warranties of merchantability or fitness for a particular purpose. No warranty may be created or extended by sales representatives,
written sales materials or promotional statements for this work. The fact that an organization, website, or product is referred to in this work as a
citation and/or potential source of further information does not mean that the publisher and authors endorse the information or services the
organization, website, or product may provide or recommendations it may make. This work is sold with the understanding that the publisher is
not engaged in rendering professional services. The advice and strategies contained herein may not be suitable for your situation. You should
consult with a specialist where appropriate. Further, readers should be aware that websites listed in this work may have changed or disappeared
between when this work was written and when it is read. Neither the publisher nor authors shall be liable for any loss of profit or any other
commercial damages, including but not limited to special, incidental, consequential, or other damages.

Library of Congress Cataloging-in-Publication Data

Library of Congress Cataloging-in-Publication Data


Names: Kline-Tilford, Andrea M., editor. | Haut, Catherine, editor.
Title: Cases in pediatric acute care : strengthening clinical decision
making / edited by Andrea M. Kline-Tilford, Catherine Haut.
Description: Hoboken, NJ : Wiley-Blackwell 2020. | Includes bibliographical
references.
Identifiers: LCCN 2019056731 (print) | LCCN 2019056732 (ebook) |
ISBN 9781119568223 (paperback) | ISBN 9781119568209 (adobe pdf) |
ISBN 9781119568216 (epub)
Subjects: MESH: Critical Care–methods | Child | Critical
Illness–diagnosis | Critical Illness–therapy | Diagnosis, Differential |
Infant | Adolescent | Case Reports
Classification: LCC RJ370 (print) | LCC RJ370 (ebook) | NLM WS 366 | DDC
618.92/0028–dc23
LC record available at https://lccn.loc.gov/2019056731
LC ebook record available at https://lccn.loc.gov/2019056732

Cover Design: Wiley


Cover Images: Female doctor examining kid little child boy © Oksana Kuzmina/Shutterstock, Smiling doctor holding baby © JGI/Jamie Grill/
Getty Images, Pretty woman holding a newborn baby in her arms © Trendsetter Images/Shutterstock, Baby in mother’s arms in white blanket
© Herlanzer/Shutterstock, Paediatrician checking baby boys heart using stethoscope © Emely/Getty Images

Set in 9.5/12.5pt STIXTwoText by SPi Global, Pondicherry, India


Printed and bound by CPI Group (UK) Ltd, Croydon, CR0 4YY

10 9 8 7 6 5 4 3 2 1
Downloaded from https://onlinelibrary.wiley.com/doi/ by National Institute Of Standard, Wiley Online Library on [06/03/2024]. See the Terms and Conditions (https://onlinelibrary.wiley.com/terms-and-conditions) on Wiley Online Library for rules of use; OA articles are governed by the applicable Creative Commons License
v

Contents

Preface xi
List of Contributors xiii

1 Complication of Pneumonia 1
Ji-Yeon Kim
2 Infant with Diaphoresis during Feedings 5
Kristen Brown and Kristen Nelson McMillan
3 Hypotension in the Operating Room 7
Ann Marie Felauer
4 Toddler with Vomiting and Diarrhea 11
Kimberly L. DiMaria
5 Infant with Fever and Cold Symptoms 15
Cheryl Bartke
6 Toddler with Pineoblastoma and Bone Marrow Transplant 19
Cathryn Freeman
7 A 5-Year-Old with Altered Mental Status 23
Yu-shan Tseng and Richard Garcia
8 An 8-Year-Old with Abdominal Trauma 29
Julie Resler
9 Child with Left Eye Swelling 33
Amy Graf
10 Billing and Coding 37
Dawn M. Silverman
11 Language Delay 47
Jolene Dickerson
12 A 6-Month-Old Infant with RSV and Respiratory Failure 51
Catherine M. Haut
13 Adolescent Abdominal Pain after Cardiac Surgery 55
Cathy Woodward
14 Young Infant with Vomiting 57
Sarah Martin
15 School-age Child with Bruising 61
Lisa Hovingh and Candie Ritsema
16 Infant with Shunted Hydrocephalus 65
Mark Weber
Downloaded from https://onlinelibrary.wiley.com/doi/ by National Institute Of Standard, Wiley Online Library on [06/03/2024]. See the Terms and Conditions (https://onlinelibrary.wiley.com/terms-and-conditions) on Wiley Online Library for rules of use; OA articles are governed by the applicable Creative Commons License
vi Contents

17 Newborn with Poor Oral Intake and Irritability 69


Elaine Geary
18 An 8-Week-Old Infant with a Cough 73
Sarah Jane Garraty
19 Desaturation in Postoperative Cardiac Surgery Infant 77
Lisa Kohr
20 Pre-school Child with Seizures 81
Heather Herrera and Rachel Luthen
21 Child with Blood in Stool 85
Brittany Cook and Darby Tozer
22 Osteomyelitis Case Study 89
Kim Steanson
23 A 2-Year-Old with Liver Transplant and Fever 93
Louise Flynn
24 Respiratory Decompensation in Trached Infant 97
Jennifer Helman
25 Neonatal Hypoglycemia 101
Janice L. Wilson
26 Chronic Otitis Media 105
Kimberly Young-Conner
27 Complex Patient with Recurrent Lower Respiratory Tract Infection 109
Ann Marie Ramsey
28 Child with Racing Heart 115
Bradley Goettl
29 Teen with Headache and Seizure 119
Valarie Eichler
30 A 4-Year-Old with Bloody Diarrhea 125
Tamara Hill
31 Child with Persistent Fever 129
Misty Ellis
32 Quality Improvement Case Study 133
Peggy Dorr
33 Newborn Immediately Post Delivery 139
Heather Schniepp
34 A 5-Year-Old with Poor Weight Gain and Dehydration 143
Kimberly Young-Conner
35 Toddler with Complex Chronic Condition 147
Jennifer Wright
36 Pre-adolescent with Knee Pain 151
Katelyn Hamann and Shannon Konieczki
37 Infant with Increased Fussiness and Decreased Oral Intake 155
Stephanie Haughey
38 Infant with Prolonged Jaundice 161
Aimee Bucci
39 Child with Rash and Drooling 165
Mark Weber
Downloaded from https://onlinelibrary.wiley.com/doi/ by National Institute Of Standard, Wiley Online Library on [06/03/2024]. See the Terms and Conditions (https://onlinelibrary.wiley.com/terms-and-conditions) on Wiley Online Library for rules of use; OA articles are governed by the applicable Creative Commons License
Contents vii

40 Child with Fever and Sore Throat 169


Stephanie Robell and Danielle Van Damme
41 Teenager with Sickle Cell Disease and Fever 173
Shannon Konieczki and Katelyn Hamann
42 Ethical Case Study 177
Jamie Tumulty
43 Infant with Neurologic Changes and Respiratory Failure 183
Blythe Pollack and Stacey Sears
44 Acute Hyperglycemia 191
Jeanne Little
45 Infant with Progressive Lethargy and Poor Feeding 195
Kathleen P. Thompson
46 Neonatal Tachypnea 199
Janet Dutcher
47 Diplopia and Esotropia 203
Lindsay Chapman
48 Acetaminophen Ingestion 207
Aaron Carpenter
49 A 3-Year-Old with Sudden Onset of Breathing Difficulty 211
Judy Verger
50 A 2-Month-Old with Acute Abdomen 215
Christyne Kyper
51 Henoch–Schönlein Purpura Nephritis 219
Tamara Hill
52 Toddler with Tachypnea and Cool Extremities 223
Cathy Woodward
53 Teen with Intractable Headache 227
Tara Pezzuto
54 Seizures in School-age Child 231
Ann Marie Felauer
55 An 18-Year-Old with Upper Respiratory Symptoms, Headache and Fever 235
Misty Evans
56 Testicular Torsion 239
Amanda Mandel
57 Teen with Fever and Rash 243
Christine Renke and Elizabeth Hoppe
58 Preterm Infant Unable to Wean from Mechanical Ventilation 247
Michelle Dokas
59 Toddler with Acute Respiratory Failure 251
Ahmed Aly and Bradley Tilford
60 Adolescent with Head Injury 255
Jennette Firlein
61 Newborn with Ambiguous Genitalia 261
Barbara Wise
62 Anemia in a Toddler 265
Dyane Bunnell
Downloaded from https://onlinelibrary.wiley.com/doi/ by National Institute Of Standard, Wiley Online Library on [06/03/2024]. See the Terms and Conditions (https://onlinelibrary.wiley.com/terms-and-conditions) on Wiley Online Library for rules of use; OA articles are governed by the applicable Creative Commons License
viii Contents

63 A 2-Year-Old with Fever and Cough 269


Jessica Strohm Farber
64 Research Review 273
Lauren R. Sorce
65 Toddler with Fever and Redness Behind Ear 277
Amanda Lewis Weir
66 A 5-Year-Old with Right-Sided Weakness 281
Suzette Meinke and Jessica Spruit
67 Postoperative Heart Block 285
Cathy Woodward
68 Recognizing Human Trafficking 287
Christine DiPaolo
69 A Toddler With Stridor 291
Mari Welch
70 Delirium in a 9-Year-Old Boy 295
Heather Herrera
71 Feeding Challenge in a Child with Spinal Muscular Atrophy 299
Nicole Fragale
72 A 5-Year-Old with Osteomyelitis, Electrolyte Disturbance and Headaches 303
Kimberly L. DiMaria
73 Child with Scald Burn Injury 307
Amarilis Martin and Bradley Tilford
74 Parvovirus Requiring Blood Transfusion 313
Ashley Thibodeau
75 A Teen With High Fever, Rash and Lethargy 317
Jessica Strohm Farber
76 A 9-Year-Old With Scoliosis 321
Alicia McCarthy
77 An Unresponsive Infant 325
Charlene Pringle
78 A Toddler With Abdominal Pain, Diarrhea and Lethargy 331
Jessica Pech
79 Sore Throat and Shortness of Breath 335
Amy Manion
80 Oral Presentation and Communication 339
Jill Siegrist Thomas
81 A 7-Year-Old with Migratory Joint Pain 345
Christin Diller
82 Adolescent with Murmur and Chest Flutter 349
Natalie Hecht
83 Outpatient Management of Asthma 353
Jolene Dickerson
84 A Newborn with Abnormal Movements 357
Maura Bernardo
85 Postoperative Infant with Tachycardia 361
Cathy Woodward
Downloaded from https://onlinelibrary.wiley.com/doi/ by National Institute Of Standard, Wiley Online Library on [06/03/2024]. See the Terms and Conditions (https://onlinelibrary.wiley.com/terms-and-conditions) on Wiley Online Library for rules of use; OA articles are governed by the applicable Creative Commons License
Contents ix

86 Brain Death 363


Maria C. Woosley
87 Postoperative Spinal Fusion with Complications 367
Jaime Panton and Melissa C. Varrone
88 Orthopnea in a 10-Year-Old 371
Max Pizzo
89 Complication of Traumatic Brain Injury 375
Christine Schindler
90 Newborn with Hypothermia 379
Marshay James
91 Back Pain in an Active Teenager 383
Alicia McCarthy
92 Teen with Fever and Abdominal Pain 387
Sarah Martin
93 Complication in a 6-Year-Old with a Kidney Transplant 391
Alison Hewitt Torres
94 Acute Hypertension and Seizures in a 17-Year-Old Male 395
Alison Hewitt Torres
95 Jittery 2-Week-Old Infant 399
Christopher D. Newman
96 Child with Leg and Back Pain 403
Jill Marks
97 Fussy, Tired Infant 407
Charlene Pringle
98 Tics and Something More 411
Judy Adelizzi-DeLany
99 Lightheaded Adolescent 415
Cathy Woodward
100 Infant with Hyponatremia 419
Sarah Pihl
101 Bone Marrow Transplant in Sickle Cell Disease 423
J. Paige Rieckhoff
102 Child with Respiratory Failure and Transaminitis 427
Kim Steanson
103 A Serious Football Injury 431
Julie Resler
104 Infant with Generalized Weakness 435
Blythe Pollack
105 A 3-Day-Old with Irritability 441
Christopher D. Newman
106 Child with Fever and Rash 445
Misty Evans
107 A Teen With Vomiting and Diarrhea 449
Christyne Kyper
108 Acute Jaundice in a 7-Year-Old Child 453
Roseann Hausmann
Downloaded from https://onlinelibrary.wiley.com/doi/ by National Institute Of Standard, Wiley Online Library on [06/03/2024]. See the Terms and Conditions (https://onlinelibrary.wiley.com/terms-and-conditions) on Wiley Online Library for rules of use; OA articles are governed by the applicable Creative Commons License
x Contents

109 Idiopathic Nephrotic Syndrome 457


Tamara Hill
110 School-age Child with Hives and Lip Swelling 461
Jay M. Hunter
111 A 10-Year-Old with Asthma in the PICU 463
Catherine M. Haut
112 Skin Wound in Postoperative Neonate 467
Hayley Redmond
113 Provider Burnout and Resiliency 471
Jill Siegrist Thomas
114 Kidney Transplant Complication 475
Cathy C. McAdams
115 Infant with Influenza A and New Fever 479
Lauren Flagg
116 A 3-Year-Old with Fever and Painful Urination 483
Roseann Hausmann

Index 487
Downloaded from https://onlinelibrary.wiley.com/doi/ by National Institute Of Standard, Wiley Online Library on [06/03/2024]. See the Terms and Conditions (https://onlinelibrary.wiley.com/terms-and-conditions) on Wiley Online Library for rules of use; OA articles are governed by the applicable Creative Commons License
xi

Preface

The healthcare landscape is rapidly changing and will minutes of their encounters with these children and their
continue to evolve, providing challenges for patients and families. It is also imperative for pediatric providers to
families, but also for those providing care. Preventative engage the most recent evidence-based practice para-
care for children forms the framework for minimizing meters to substantiate care while incorporating the unique
hospitalization, injury and illness, but when children do needs of the child and utilizing critical thinking for
become ill, providing care with the goal of a quick return decision-making. In this book, pediatric cases are presented
to function and home must be collaborative and timely. with an opportunity for the reader to answer questions about
Children are basically healthy and the incidence of illness assessment, diagnosis and management, interpret diagnostic
and injury is small as compared to adults, but the chal- studies as well as to incorporate practical considerations
lenges of diagnosis and management can be dramatic. such as billing for services and participating in research at
Children bring unique and unusual genetic problems, the point of care. System-based, contemporary, common
chronic healthcare issues, trauma, and acute and critical as well as unusual clinical cases are included that will
presentations to inpatient and outpatient areas. It is espe- challenge the reader and add knowledge. We thank the
cially important for pediatric providers to recognize diag- contributors for sharing their patient stories and expertise
noses and potential clinical outcomes in the first few in creating this book.
Downloaded from https://onlinelibrary.wiley.com/doi/ by National Institute Of Standard, Wiley Online Library on [06/03/2024]. See the Terms and Conditions (https://onlinelibrary.wiley.com/terms-and-conditions) on Wiley Online Library for rules of use; OA articles are governed by the applicable Creative Commons License
xiii

List of Contributors

Judith Adelizzi-DeLany, DNP, PPCNP-BC Kristen M. Brown, DNP, CPNP-AC, CHSE-A


Nurse Practitioner Assistant Professor
Nemours/Alfred I. duPont Hospital for Children Advanced Practice Simulation Coordinator
Wilmington, DE John Hopkins University School of Nursing
USA Baltimore, MD
Chapter 98 Tics and Something More USA
Chapter 2 Infant with Diaphoresis during Feedings

Sheree H. Allen, DNP, APRN, CPNP-PC/AC Aimee Bucci, DNP, CPNP-AC


Director, Pediatric Acute Care Nurse Practitioner Specialty Lead Advanced Practice Provider
Vanderbilt University School of Nursing Phoenix Children’s Hospital
Nashville, TN Phoenix, AZ;
USA Faculty Associate
Section Editor: Infectious Disease Arizona State University
Edson College of Nursing and Health Innovation
Phoenix, AZ
Ahmed Aly, MD USA
Pediatric Critical Care Fellow Chapter 38 Infant with Prolonged Jaundice
Children’s Hospital of Michigan Dyane Bunnell, MSN, APRN-BC, AOCNS, CPON
Detroit, MI Clinical Nurse Specialist
USA Nemours/Alfred I. duPont Hospital for Children
Chapter 59 Toddler with Acute Respiratory Failure Wilmington, DE
USA
Chapter 62 Anemia in a Toddler
Cheryl Bartke, MSN, PPCNP, CPNP-AC
Nurse Practitioner Aaron Carpenter, DNP, M.Div., CPNP-PC
Pediatric Critical Care Senior Director Ambulatory and Advanced
Seattle Children’s Hospital Practice Nursing
Seattle, WA Nemours/Alfred I. duPont Hospital for Children
USA Wilmington, DE
Chapter 5 Infant with Fever and Cold Symptoms USA
Chapter 48 Acetaminophen Ingestion

Maura Bernardo, MSN, CPNP Lindsay Chapman, MSN, CPNP-PC


Nurse Practitioner Nurse Practitioner
Nemours/Alfred I. duPont Hospital for Children St. Jude Children’s Research Hospital
Wilmington, DE Memphis, TN
USA USA
Chapter 84 A Newborn with Abnormal Movements Chapter 47 Diplopia and Esotropia
Downloaded from https://onlinelibrary.wiley.com/doi/ by National Institute Of Standard, Wiley Online Library on [06/03/2024]. See the Terms and Conditions (https://onlinelibrary.wiley.com/terms-and-conditions) on Wiley Online Library for rules of use; OA articles are governed by the applicable Creative Commons License
xiv List of Contributors

Brittany Cook, MSN, FNP-BC Michelle Dokas, MSN, CPNP-AC


Nurse Practitioner Nurse Practitioner
Pediatric Emergency Medicine Pediatric Cardiovascular Surgery
Beaumont Hospital Children’s Hospital of Michigan
Troy, MI Detroit, MI
USA USA
Chapter 21 Child with Blood in Stool Chapter 58 Preterm Infant Unable to Wean from Mechanical
Ventilation
Jolene Dickerson, MSN, CPNP-PC
Nurse Practitioner
Nemours/Alfred I. duPont Hospital for Children Peggy Dorr, DNP, CPNP-PCNP
Wilmington, DE Clinical Program Manager, Women & Children’s Services
USA Nurse Practitioner, Pediatric Sedation Service
Chapter 11 Language Delay; Chapter 83 Outpatient University of Maryland Medical Center
Management of Asthma Baltimore, MD
USA
Christin Diller, DNP, CPNP-AC Chapter 32 Quality Improvement Case Study
Nurse Practitioner
Cincinnati Children’s Hospital Medical Center Janet Dutcher, DNP, NNP, APRN-BC
Cincinnati, OH; Neonatal Nurse Practitioner
Nursing Instructor Nemours/Alfred I. duPont Hospital for Children
Vanderbilt University College of Nursing Wilmington, DE
Nashville, TN USA
USA Chapter 46 Neonatal Tachypnea
Chapter 81 A 7-Year-Old with Migratory Joint Pain

Colleen Ditro, DNP, CPNP Valarie Eichler, MSN, CPNP-AC


Orthopedic APRN Supervisor, Skeletal Dysplasia Nurse Practitioner
Program Coordinator Dallas Children’s Hospital
Nemours/Alfred I. duPont Hospital for Children Dallas, TX
Wilmington, DE USA
USA Chapter 29 Teen with Headache and Seizure
Section Editor: Trauma, Musculoskeletal, Toxicology
Misty Ellis, DNP, CPNP-AC/PC
Kimberly L. DiMaria, MSN, CPNP-AC, CCRN
Lead Cardiac Nurse Practitioner
Heart Institute Clinical Practice Specialist
Norton Children’s Hospital
Children’s Hospital Colorado
Louisville, KY
Aurora, CO
USA
USA
Chapter 31 Child with Persistent Fever
Chapter 4 Toddler with Vomiting and Diarrhea;
Chapter 72 A 5-Year-Old with Osteomyelitis,
Electrolyte Disturbance and Headaches Misty Evans, DNP, CPNP-AC
Nurse Practitioner
Christine DiPaolo, NP Vanderbilt College of Nursing
Nurse Practitioner Nashville, TN
Nemours/Alfred I. duPont Hospital for Children USA
Wilmington, DE Chapter 55 An 18-Year-Old with Upper Respiratory
USA Symptoms, Headache and Fever; Chapter 106 Child with
Chapter 68 Recognizing Human Trafficking Fever and Rash
Downloaded from https://onlinelibrary.wiley.com/doi/ by National Institute Of Standard, Wiley Online Library on [06/03/2024]. See the Terms and Conditions (https://onlinelibrary.wiley.com/terms-and-conditions) on Wiley Online Library for rules of use; OA articles are governed by the applicable Creative Commons License
List of Contributors xv

Jessica Strohm Farber, DNP, CPNP-AC Cathryn Freeman, MSN, CPNP-PC


Nurse Practitioner Nurse Practitioner
Children’s Hospital of Philadelphia Pediatric Hematology/Oncology
Philadelphia, PA Children’s Hospital of Michigan
USA Detroit, MI
Chapter 63 A 2-Year-Old with Fever and Cough; USA
Chapter 75 A Teen With High Fever, Rash and Lethargy Chapter 6 Toddler with Pineoblastoma and Bone
Marrow Transplant
Ann Marie Felauer, DNP, CPNP-AC/PC
Assistant Professor and Director
Pediatric Acute and Primary Care Nurse Practitioner Richard U. Garcia, MD
Specialties Assistant Professor Department of Pediatrics
UMB CURE Anatomy and Chemistry Curriculum Tracks, Division of Cardiology and Critical Care
Program Manager Wayne State University
University of Maryland School of Nursing Detroit, MI
Baltimore, MD USA
USA Chapter 7 A 5-Year-Old with Altered Mental Status
Chapter 3 Hypotension in the Operating Room;
Chapter 54 Seizures in School-age Child
Section Editor: Endocrinology Sarah Jane Garraty, MSN, CPNP
Nurse Practitioner
Jennette Firlein, MSN, CPNP-PC Beacon Pediatrics
Nurse Practitioner Rehoboth Beach, DE
Nemours/Alfred I. duPont Hospital for Children USA
Wilmington, DE Chapter 18 An 8-Week-Old Infant with a Cough
USA
Chapter 60 Adolescent with Head Injury
Elaine Geary, MSN, CRNP
Lauren Flagg, MSN, CPNP-AC Nurse Practitioner
Nurse Practitioner Nemours/Alfred I. duPont Hospital for Children
Yale New Haven Hospital Wilmington, DE
New Haven, CT USA
USA Chapter 17 Newborn with Poor Oral Intake and Irritability
Chapter 115 Infant with Influenza A and New Fever

Louise Flynn, MSN, CCTC Bradley Goettl, DNP, AGACNP-BC, FNP-C, ENP-C
Clinical Nurse Specialist Assistant Professor/Clinical
Nemours/Alfred I. duPont Hospital for Children University of Texas Health
Wilmingon, DE San Antonio, TX
USA USA
Chapter 23 A 2-Year-Old with Liver Transplant and Fever Chapter 28 Child with Racing Heart

Nicole Fragale, MPH, RD, CSP, LDN, CNSC


Registered Dietitian Amy K. Graf, MSN, DNP, FNP-C, CPNP-AC
Nemours/Alfred I. duPont Hospital for Children Nurse Practitioner
Wilmington, DE Norton Women and Children’s Hospital
USA Louisville, KY
Chapter 71 Feeding Challenge in a Child with Spinal USA
Muscular Atrophy Chapter 9 Child with Left Eye Swelling
Downloaded from https://onlinelibrary.wiley.com/doi/ by National Institute Of Standard, Wiley Online Library on [06/03/2024]. See the Terms and Conditions (https://onlinelibrary.wiley.com/terms-and-conditions) on Wiley Online Library for rules of use; OA articles are governed by the applicable Creative Commons License
xvi List of Contributors

Katelyn Hamann, MSN, CPNP-AC Heather Herrera, MSN, CPNP-AC/PC


Nurse Practitioner Nurse Practitioner
Pediatric Hematology/Oncology Children’s Hospital of San Antonio
Children’s Hospital of Michigan San Antonio, TX
Detroit, MI USA
USA Chapter 20 Pre-school Child with Seizures;
Chapter 36 Pre-adolescent with Knee Pain; Chapter 70 Delirium in a 9-Year-Old Boy
Chapter 41 Teenager with Sickle Cell Disease and Fever
Tamara Hill, DNP, CPNP-AC
Stephanie Haughey, MSN, CPNP-AC Nurse Practitioner
Nurse Practitioner UMSON Faculty Associate/Adjunct Faculty
Spectrum Health, Helen DeVos Children’s Hospital University of Maryland Medical Center
Grand Rapids, MI Baltimore, MD
USA USA
Chapter 37 Infant with Increased Chapter 30 A 4-Year-Old with Bloody Diarrhea;
Fussiness and Decreased Oral Intake Chapter 51 Henoch–Schönlein Purpura Nephritis;
Chapter 109 Idiopathic Nephrotic Syndrome
Roseann Hausmann, MSN, CPNP-AC/PC Section Editor: Genitourinary/Renal
Nurse Practitioner
Seattle Children’s Hospital
Seattle, WA Elizabeth Hoppe, MSN, CPNP-AC
USA Nurse Practitioner
Chapter 108 Acute Jaundice in a 7-Year-Old Child; Ann & Robert H. Lurie Children’s Hospital of Chicago
Chapter 116 A 3-Year-Old with Fever and Painful Urination Chicago, IL
USA
Catherine M. Haut, DNP, CPNP-AC/PC, FAANP, CCRN Chapter 57 Teen with Fever and Rash
Coordinator of Nursing Research and Evidence Based Care
Nemours/Alfred I. duPont Hospital for Children, Lisa Hovingh, CPNP-AC
Wilmington, DE; Nurse Practitioner
Nurse Practitioner Pediatric Hematology/Oncology
Mednax-Pediatrix Medical Group Helen DeVos Children’s Hospital
Richardson, TX Grand Rapids, MI
USA USA
Chapter 12 A 6-Month-Old Infant with RSV and Respiratory Chapter 15 School-age Child with Bruising
Failure; Chapter 111 A 10-Year-Old with Asthma in the PICU
Section Editor: Respiratory Jay M. Hunter, DNP, CPNP-AC, CCRN, CPN
Book Editor: Cases in Pediatric Acute Care: Strengthening Nurse Practitioner and Assistant Clinical Professor
Clinical Decision Making UCSF Benioff Children’s Hospital and
UCSF School of Nursing
Natalie Hecht, MSN, CPNP-AC San Francisco, CA
Nurse Practitioner USA
Pediatric Cardiac Surgery Chapter 110 School-age Child with Hives
C.S. Mott Children’s Hospital and Lip Swelling
Ann Arbor, MI
USA
Marshay James, DNP, MSNEd, CPNP-AC, CPEN, CNE
Chapter 82 Adolescent with Murmur and Chest Flutter
Instructor
Jennifer Helman, MSN, CPNP-AC/PC Vanderbilt University School of Nursing Nashville, TN;
Nurse Practitioner Advanced Practice Provider Manager
Pediatric Otolaryngology Department of Critical Care Medicine
C.S. Mott Children’s Hospital St. Jude Children’s Research Hospital
Ann Arbor, MI Memphis, TN
USA USA
Chapter 24 Respiratory Decompensation in Trached Infant Chapter 90 Newborn with Hypothermia
Downloaded from https://onlinelibrary.wiley.com/doi/ by National Institute Of Standard, Wiley Online Library on [06/03/2024]. See the Terms and Conditions (https://onlinelibrary.wiley.com/terms-and-conditions) on Wiley Online Library for rules of use; OA articles are governed by the applicable Creative Commons License
List of Contributors xvii

Jennifer Joiner, MSN, CPNP-AC/PC Jeanne Little, DNP, PNP-AC/PC


Assistant Professor Baylor College of Medicine Assistant Professor
Lead Nurse Practitioner Pediatric Intensive Care Unit Rush University College of Nursing
Children’s Hospital of San Antonio Chicago, IL
San Antonio, TX USA
USA Chapter 44 Acute Hyperglycemia
Section Editor: Neurology

Ji-Yeon Kim, MD Rachel Luthen, MSN, CPNP-AC


Pediatric Critical Care Nurse Practitioner
Children’s Hospital of Michigan Children’s Hospital of San Antonio
Detroit, MI San Antonio, TX
USA USA
Chapter 1 Complication of Pneumonia Chapter 20 Pre-school Child with Seizures

Andrea M. Kline-Tilford, PhD, CPNP-AC/PC, FCCM, FAAN


Nurse Practitioner Manager Amanda Mandel, CPNP
C.S. Mott Children’s Hospital Urology Nurse Practitioner
Michigan Medicine Nemours/Alfred I. duPont Hospital for Children
Ann Arbor, MI Wilmington, DE
USA USA
Book Editor: Cases in Pediatric Acute Care: Strengthening Chapter 56 Testicular Torsion
Clinical Decision Making

Lisa M. Kohr, MSN, CPNP-AC, MPH Amy Manion, PhD, RN, CPNP-PC
Nurse Practitioner Nurse Practitioner
Acute Care Cardiology Associate Professor
Cincinnati Children’s Hospital Medical Center Rush University College of Nursing
Cincinnati, OH Chicago, IL
USA USA
Chapter 19 Desaturation in Postoperative Chapter 79 Sore Throat and Shortness of Breath
Cardiac Surgery Infant

Jill Marks, DNP, CPNP-AC


Shannon Konieczki, MSN, CPNP-AC Senior Instructor
Nurse Practitioner University of Colorado Pediatric Neurology, Boulder, CO;
Children’s Hospital of Michigan Faculty Acute Care Pediatric Nurse Practitioner Program
Detroit, MI Rush University College of Nursing
USA Chicago, IL
Chapter 36 Pre-adolescent with Knee Pain; USA
Chapter 41 Teenager with Sickle Cell Disease and Fever Chapter 96 Child with Leg and Back Pain

Christyne Kyper, MSN, CPNP AC/PC


Nurse Practitioner Amarilis Martin, MD
Alaska Native Tribal Health Consortium Pediatric Critical Care Fellow
Anchorage, AK Children’s Hospital of Michigan
USA Detroit, MI
Chapter 50 A 2-Month-Old with Acute Abdomen; USA
Chapter 107 A Teen With Vomiting and Diarrhea Chapter 73 Child with Scald Burn Injury
Downloaded from https://onlinelibrary.wiley.com/doi/ by National Institute Of Standard, Wiley Online Library on [06/03/2024]. See the Terms and Conditions (https://onlinelibrary.wiley.com/terms-and-conditions) on Wiley Online Library for rules of use; OA articles are governed by the applicable Creative Commons License
xviii List of Contributors

Sarah Martin, MS, CPNP-AC/PC, CCRN Osei Owusu, MD


Nurse Practitioner Pediatric Critical Care Medicine
Department of Pediatric Surgery Mednax-Pediatrix Medical Group
Ann & Robert H. Lurie Children’s Hospital The Herman and Walter Samuelson Children’s Hospital
Chicago, IL Baltimore, MD
USA USA
Chapter 14 Young Infant with Vomiting; Section Editor: Respiratory
Chapter 92 Teen with Fever and Abdominal Pain

Cathy C. McAdams, PPCNP-BC Jaime Panton, DNP, CPNP-AC/PC


Nurse Practitioner Assistant Professor of Nursing
Nemours/Alfred I. duPont Hospital for Children PNP Program Director
Wilmington, DE Columbia School of Nursing
USA New York, NY
Chapter 114 Kidney Transplant Complication USA
Chapter 87 Postoperative Spinal Fusion with Complications
Alicia McCarthy, MSN, CPNP-AC
Nurse Practitioner
Jessica Pech, MSN, CPNP-PC
Nemours/Alfred I. duPont Hospital for Children
Nurse Practitioner
Wilmington, DE
Ann & Robert H. Lurie Children’s Hospital of Chicago
USA
Chicago, IL
Chapter 76 A 9-Year-Old With Scoliosis;
USA
Chapter 91 Back Pain in an Active Teenager
Chapter 78 A Toddler With Abdominal Pain,
Diarrhea and Lethargy
Kristen Nelson McMillan, MD
Associate Professor
Pediatric Cardiac Critical Care
Tara Pezzuto, DNP, CPNP
Advocate Children’s Pediatric Heart Institute
Headache Program
University of Chicago Comer Children’s Hospital
Nemours/Alfred I. duPont Hospital for Children
Chicago, IL
Wilmington, DE
USA
USA
Chapter 2 Infant with Diaphoresis during Feedings
Chapter 53 Teen with Intractable Headache

Suzette Meinke, MSN, PNP-BC


Nurse Practitioner Sarah Pihl, MSN, CPNP-AC
Pediatric Hematology Nurse Practitioner and Senior Instructor
Children’s Hospital of Michigan Children’s Hospital of Colorado
Detroit, MI Aurora, CO
USA USA
Chapter 66 A 5-Year-Old with Right-Sided Weakness Chapter 100 Infant with Hyponatremia

Christopher D. Newman, PA-C, FCCM


Physician Assistant Max Pizzo, MS, CPNP-AC
Children’s Hospital of Colorado Nurse Practitioner
Aurora, CO Pediatric Critical Care Medicine
USA C.S. Mott Children’s Hospital
Chapter 95 Jittery 2-Week-Old Infant; Ann Arbor, MI
Chapter 105 A 3-Day-Old with Irritability; USA
Section Editor: Fluid and Electrolytes Chapter 88 Orthopnea in a 10-Year-Old
Downloaded from https://onlinelibrary.wiley.com/doi/ by National Institute Of Standard, Wiley Online Library on [06/03/2024]. See the Terms and Conditions (https://onlinelibrary.wiley.com/terms-and-conditions) on Wiley Online Library for rules of use; OA articles are governed by the applicable Creative Commons License
List of Contributors xix

Blythe Pollack, MSN, CPNP-AC J. Paige Rieckhoff, MSN, CPNP-AC


Nurse Practitioner Nurse Practitioner
Pediatric Critical Care Medicine C.S. Mott Children’s Hospital
C.S. Mott Children’s Hospital Ann Arbor, MI
Ann Arbor, MI USA
USA Chapter 101 Bone Marrow Transplant in Sickle Cell Disease
Chapter 43 Infant with Neurologic Changes and
Respiratory Failure; Candie Ritsema, MSN, CPNP-AC, CPON
Chapter 104 Infant with Generalized Weakness Nurse Practitioner
Pediatric Hematology/Oncology
Charlene Pringle, MSN, CPNP-AC/PC Helen DeVos Children’s Hospital
Nurse Practitioner Grand Rapids, MI
Pediatric Critical Care Medicine USA
University of Florida Chapter 15 School-age Child with Bruising
Gainesville, FL
USA Stephanie Robell, MSN, CPNP-AC
Chapter 77 An Unresponsive Infant; Nurse Practitioner
Chapter 97 Fussy, Tired Infant Pediatric Critical Care Medicine
C.S. Mott Children’s Hospital
Ann Marie Ramsey, RN, MSN, CPNP Ann Arbor, MI
Pediatric Nurse Practitioner USA
Pediatric Home Ventilator Program Chapter 40 Child with Fever and Sore Throat
C.S. Mott Children’s Hospital
Michigan Medicine Christine Schindler, PhD, RN, CPNP-PC/AC
Ann Arbor, MI Clinical Associate Professor Coordinator
USA Pediatric Acute Care Graduate Option
Chapter 27 Complex Patient with Recurrent Lower Marquette University College of Nursing
Respiratory Tract Infection Milwaukee, WI;
Advanced Practice Provider Director
Hayley Redmond, DO, FAAP Critical Care/Palliative Care Advanced Practice
Pediatric Critical Care Fellow Provider Program
Norton Children’s Hospital Medical College of WI/Children’s Hospital of WI
Louisville, KY Milwaukee, WI
USA USA
Chapter 112 Skin Wound in Postoperative Neonate Chapter 89 Complication of Traumatic Brain Injury

Christine Renke, MSN, CPNP-AC Heather Schniepp, DNP, CPNP-AC/PC


Nurse Practitioner Nurse Practitioner
Pediatric Critical Care Medicine University of Texas Health
C.S. Mott Children’s Hospital San Antonio, TX
Ann Arbor, MI USA
USA Chapter 33 Newborn Immediately Post Delivery
Chapter 57 Teen with Fever and Rash
Stacey Sears, MSN, CPNP-AC
Julie Resler, CPNP-AC Nurse Practitioner
Nurse Practitioner Pediatric Critical Care Medicine
Riley Hospital for Children C.S. Mott Children’s Hospital
Indianapolis, IN Ann Arbor, MI
USA USA
Chapter 8 An 8-Year-Old with Abdominal Trauma; Chapter 43 Infant with Neurologic Changes
Chapter 103 A Serious Football Injury and Respiratory Failure
Downloaded from https://onlinelibrary.wiley.com/doi/ by National Institute Of Standard, Wiley Online Library on [06/03/2024]. See the Terms and Conditions (https://onlinelibrary.wiley.com/terms-and-conditions) on Wiley Online Library for rules of use; OA articles are governed by the applicable Creative Commons License
xx List of Contributors

Danielle Sebbens, DNP, CPNP-AC/PC Kim Steanson, DNP, CPNP-AC/PC


Clinical Associate Professor Nurse Practitioner
Coordinator Acute Care Pediatric Nurse Practitioner Monroe Carell Jr. Children’s Hospital at Vanderbilt
Program Nashville, TN;
Arizona State University Instructor
Edson College of Nursing and Health Innovation Vanderbilt University School of Nursing
Phoenix, AZ; Nashville, TN
Alaska Native Medical Center USA
Nurse Practitioner Chapter 22 Osteomyelitis Case Study;
Anchorage, AK Chapter 102 Child with Respiratory Failure
USA and Transaminitis
Section Editor: Gastrointestinal

Ashley Thibodeau, DNP, CPNP-PC


Shari Simone, DNP, CPNP-AC, FAANP, FCCM, FAAN
Nurse Practitioner
Senior NP Clinical Program Manager
Seattle Children’s Hospital
Women & Children’s Services
Seattle, WA
Nurse Practitioner
USA
University of Maryland Medical Center
Chapter 74 Parvovirus Requiring Blood Transfusion
Baltimore, MD
USA
Section Editor: Professional Issues
Jill Siegrist Thomas, MSN, CPNP-AC
Nurse Practitioner
Dawn M. Silverman, DNP, CRNP, AGACNP-BC University of Maryland Children’s Hospital
Nurse Practitioner University of Maryland Medical Center
University of Maryland Children’s Hospital Baltimore, MD
University of Maryland Medical Center USA
Baltimore, MD Chapter 80 Oral Presentation and Communication;
USA Chapter 113 Provider Burnout and Resiliency
Chapter 10 Billing and Coding

Lauren R. Sorce, PhD, CPNP-AC/PC, CCRN, FCCM Kathleen P. Thompson, MPAS, PA-C
Founders Board Nurse Scientist Assistant Professor
Pediatric Critical Care APRN Baylor College of Medicine,
Ann & Robert H. Lurie Children’s Hospital of Chicago Houston, TX;
Chicago, IL; Physician Assistant Program & Department of Pediatrics
Assistant Professor, Division of Critical Care Medicine Pediatric Critical Care Medicine, Texas Children’s Hospital
Northwestern University Feinberg School of Medicine Houston, TX
Chicago, IL USA
USA Chapter 45 Infant with Progressive Lethargy
Chapter 64 Research Review and Poor Feeding

Jessica Spruit, DNP, CPNP-AC, CPHON, BMTCN Bradley Tilford, MD


Assistant Professor, Clinical Pediatric Intensivist
Wayne State University College of Nursing Children’s Hospital of Michigan
Detroit, MI Detroit, MI
USA USA
Chapter 66 A 5-Year-Old with Right-Sided Weakness Chapter 59 Toddler with Acute Respiratory Failure;
Section Editor: Hematology/Oncology Chapter 73 Child with Scald Burn Injury
Downloaded from https://onlinelibrary.wiley.com/doi/ by National Institute Of Standard, Wiley Online Library on [06/03/2024]. See the Terms and Conditions (https://onlinelibrary.wiley.com/terms-and-conditions) on Wiley Online Library for rules of use; OA articles are governed by the applicable Creative Commons License
List of Contributors xxi

Alison Hewitt Torres, MSN, CPNP-AC Judy Verger, ARNP, PhD, FAAN, FCCM
Nurse Practitioner University of Iowa School of Nursing
Johns Hopkins Children’s Center Iowa City, IA
Baltimore, MD USA
USA Chapter 49 A 3-Year-Old with Sudden
Chapter 93 Complication in a 6-Year-Old with a Kidney Onset of Breathing Difficulty
Transplant; Chapter 94 Acute Hypertension and Seizures
Mark Weber, CPNP-AC, FCCM
in a 17-Year-Old Male
Nurse Practitioner
Children’s Hospital of Philadelphia
Darby Tozer, MSN, CPNP-AC Philadelphia, PA
Nurse Practitioner USA
Pediatric Emergency Medicine Chapter 16 Infant with Shunted Hydrocephalus;
Beaumont Hospital Chapter 39 Child with Rash and Drooling
Troy, MI
USA Amanda Lewis Weir, CPNP-AC
Chapter 21 Child with Blood in Stool Nurse Practitioner
Norton Children’s Hospital
Louisville, KY
Yu-shan Tseng, MD
USA
Pediatric Critical Care Fellow
Chapter 65 Toddler with Fever and Redness Behind Ear
Children’s Hospital of Michigan
Detroit, MI Mari Welch, MSN, APRN, CPNP-AC
USA Nurse Practitioner
Chapter 7 A 5-Year-Old with Altered Mental Status Nemours/Alfred I. duPont Hospital for Children
Wilmington, DE
USA
Jamie Tumulty, MSN, CPNP-AC
Chapter 69 A Toddler With Stridor
Nurse Practitioner
University of Maryland Children’s Hospital Janice L. Wilson, DNP, NNP-BC
University of Maryland Medical Center Assistant Professor and NNP Specialty Director
Baltimore, MD University of Maryland School of Nursing
USA Baltimore, MD
Chapter 42 Ethical Case Study USA
Chapter 25 Neonatal Hypoglycemia
Danielle Van Damme, DNP, CPNP-AC
Nurse Practitioner Barbara Wise, PhD, CPNP-AC/PC
Norton Children’s Hospital Assistant Professor
University of Louisville University of Maryland School of Nursing
Louisville, KY Baltimore, MD
USA USA
Chapter 40 Child with Fever and Sore Throat Chapter 61 Newborn with Ambiguous Genitalia
Section Editor: Immunology/Rheumatology/HEENT/
Cathy Woodward, DNP, APRN, CPNP-AC
Dermatology
Professor
Long School of Medicine
Melissa C. Varrone, MSN, CPNP, CCRN University of Texas Health
Pediatric Orthopedic Nurse Practitioner San Antonio, TX
Morgan Stanley Children’s Hospital of New York, NY; USA
New York Presbyterian Hospital, Columbia University Chapter 13 Adolescent Abdominal Pain after Cardiac
Irving Medical Center Surgery; Chapter 52 Toddler with Tachypnea and Cool
New York, NY Extremities; Chapter 67 Postoperative Heart Block;
USA Chapter 85 Postoperative Infant with Tachycardia;
Chapter 87 Postoperative Spinal Fusion Chapter 99 Lightheaded Adolescent
with Complications Section Editor: Cardiovascular
Downloaded from https://onlinelibrary.wiley.com/doi/ by National Institute Of Standard, Wiley Online Library on [06/03/2024]. See the Terms and Conditions (https://onlinelibrary.wiley.com/terms-and-conditions) on Wiley Online Library for rules of use; OA articles are governed by the applicable Creative Commons License
xxii List of Contributors

Maria C. Woosley, DNP, CPNP-AC Kimberly Young-Conner, ARNP-C


Faculty Clinical Instructor Nurse Practitioner
Long School of Medicine Nemours/Alfred I. duPont Hospital for Children
University of Texas Health Wilmington, DE
San Antonio, TX USA
USA Chapter 26 Chronic Otitis Media;
Chapter 86 Brain Death Chapter 34 A 5-Year-Old with Poor Weight Gain
and Dehydration
Jennifer Wright, MS, RN, CPNP-PC
Nurse Practitioner
Stepping Stones Pediatric Palliative Care
C.S. Mott Children’s Hospital
Ann Arbor, MI
USA
Chapter 35 Toddler with Complex Chronic Condition
Downloaded from https://onlinelibrary.wiley.com/doi/ by National Institute Of Standard, Wiley Online Library on [06/03/2024]. See the Terms and Conditions (https://onlinelibrary.wiley.com/terms-and-conditions) on Wiley Online Library for rules of use; OA articles are governed by the applicable Creative Commons License
1

Complication of Pneumonia
Ji-Yeon Kim
Children’s Hospital of Michigan, Detroit, MI, USA

A 2-year-old previously healthy female is admitted to the Family History


pediatric intensive care unit (PICU) with tachypnea, grunt-
ing, and nasal flaring. She is hypoxic despite receiving sup- Parents are healthy. There is a 6-year-old sibling who had a
plemental oxygen with a high-flow nasal cannula (HFNC) cold and fever about a week ago. Grandparents are alive
at 12 L/min and FiO2 0.7. and well. Paternal grandmother has a history of breast can-
cer. Denies family history of asthma or allergies.

History of Present Illness

This is a 2-year-old female with no significant past medical Current Status


history who presented with 2 days of fever, cough, and dif-
ficulty breathing. Her mother reports that for the past On arrival in the ED, the patient is noted to have difficulty
2 weeks the patient has had intermittent low-grade fever breathing, with grunting and nasal flaring. She is tachy-
with cough. Difficulty breathing started 2 days ago with cardic, with heart rate of 160 bpm, and tachypneic, with
fever of 40 C (104 F). She has been eating and drinking less respiratory rate of 56 breaths per minute. On examination
and having decreased wet diapers. She has been in close the patient has diminished breath sounds over the left
contact with her older brother who has also had symptoms lung field. The patient is placed on 50% oxygen using
of cough, rhinorrhea, and congestion. Her parents decided HFNC at 8 L/min but remains hypoxic with pulse oxim-
to bring her to the emergency department (ED) when her eter reading of 90% saturation. A basic metabolic panel
fever did not defervesce with acetaminophen and she con- and complete blood count are obtained with values as
tinued to have difficulty breathing. noted in Tables 1.1 and 1.2. The chest X-ray is shown in
Figure 1.1.

Past Medical History

The patient was born at 39 weeks gestational age. Family


denies history of hospitalizations or outpatient visits for ill-
Questions
ness. Immunizations are up-to-date for age, but she has not
Answer the following questions using the details provided.
received the influenza vaccine this year.
1 What are the pathologic findings on the X-ray?
2 What are the most common pathogens causing pneu-
Past Surgical History monia in infants and children?
3 What are the recommended antimicrobial therapies for
Family denies any previous surgical procedures. bacterial pneumonia?

Cases in Pediatric Acute Care: Strengthening Clinical Decision Making, First Edition. Edited by Andrea M. Kline-Tilford and Catherine M. Haut.
© 2020 John Wiley & Sons Ltd. Published 2020 by John Wiley & Sons Ltd.
Downloaded from https://onlinelibrary.wiley.com/doi/ by National Institute Of Standard, Wiley Online Library on [06/03/2024]. See the Terms and Conditions (https://onlinelibrary.wiley.com/terms-and-conditions) on Wiley Online Library for rules of use; OA articles are governed by the applicable Creative Commons License
2 Cases in Pediatric Acute Care

Table 1.1 Complete blood count. There is complete obliteration of the lateral hemithorax
and the left lung base. The right lung is clear. Gas-distended
White blood cell count 18 900/mm3
bowel loops are noted in the left upper abdomen.
Hemoglobin 13 g/dL
Hematocrit 32%
Platelet count 303 000/mm3 What are the most common pathogens causing
pneumonia in infants and children?
Neutrophils 63%
Lymphocytes 11% See Table 1.3.
Monocytes 20%
Bands 4% What are the recommended antimicrobial
therapies for bacterial pneumonia?

Table 1.2 Basic metabolic profile. Treatment of suspected bacterial pneumonia is based on
presumed pathogen, age, and clinical appearance of the
Sodium 130 mEq/L patient. For a mildly ill child who does not require hospi-
Potassium 4.5 mEq/L talization, amoxicillin is recommended. For school-age
Chloride 97 mEq/L children or clinical features concerning for atypical pneu-
Carbon dioxide 18 mEq/L monia (Mycoplasma pneumoniae or Chlamydophila
Blood urea nitrogen 35 mg/dL pneumoniae), azithromycin is recommended. For those
who require hospitalization, the antibiotic of choice is
Creatinine 0.83 mg/dL
cefotaxime or ceftriaxone. If clinical features suggest Staph-
Glucose 135 mg/dL
ylococcus aureus (pneumatoceles, empyema or bronchopul-
C-reactive protein (CRP) 4412 mg/L monary fistula) initial antibiotic therapy should include
clindamycin or vancomycin.
If viral pneumonia is suspected, it is reasonable to with-
hold antibiotic therapy especially in patients who are clin-
ically stable and present with clinical evidence suggesting a
viral infection. Optimal total duration of antibiotic therapy
should not be less than 10–14 days, unless azithromycin is
used (duration of 5 days total).
Continuation of case: The child is admitted to the PICU
and continues to have oxygen desaturations despite an
increase to 70% oxygen using HFNC at 12 L/min. She
becomes lethargic with increased work of breathing. The
patient is intubated and a chest tube is placed in the left
pleural space. About 400 mL of frank pus is drained from
the chest tube.
4 What are the most common causes of pleural effusion?
5 What are the characteristics of an exudative effusion?
6 What is the management of pleural empyema?
Figure 1.1 Chest X-ray.

What are the most common causes of pleural


Rationale and Evidence-based Practice effusion?
Explanation See Table 1.4.

What are the pathologic findings on the X-ray?


What are the characteristics of an exudative
There is a large left pleural effusion with atelectasis of the
effusion (empyema)?
left lung. Atelectasis is suspected because of shifting of
the mediastinum toward the left suggesting volume loss. See Table 1.5.
Downloaded from https://onlinelibrary.wiley.com/doi/ by National Institute Of Standard, Wiley Online Library on [06/03/2024]. See the Terms and Conditions (https://onlinelibrary.wiley.com/terms-and-conditions) on Wiley Online Library for rules of use; OA articles are governed by the applicable Creative Commons License
Chapter 1 Complication of Pneumonia 3

Table 1.3 Pathogens causing pneumonia in infants and children.

Cause Age and epidemiology Common complications

Bacterial
Group B streptococci Neonates Shock, respiratory failure, apnea
Streptococcus pneumoniae 3 weeks to 4 years, asplenia, Consolidation, pneumatoceles, empyema, shock
immunosuppressed
Staphylococcus aureus Infants Pneumatoceles, empyema
Mycoplasma pneumoniae >5 years; summer to fall Atypical pneumonia
Group A streptococci >5 years Empyema, pneumatoceles
Chlamydophila pneumoniae Adolescents Atypical pneumonia
Chlamydia trachomatis Neonates Associated with conjunctivitis
Mixed anaerobes Underlying pulmonary disease Aspiration and necrotizing pneumonia, pneumatoceles
Gram-negative enterics Neonates Hospital-acquired pneumonia, necrotizing pneumonia,
(Pseudomonas) shock

Viral: infants and children <5 years


Respiratory syncytial virus Winter Bronchiolitis, apnea
Parainfluenza types 1–3 Fall Croup, wheezing, pneumonitis, tracheitis
Influenza A, B Winter High fever, bacterial superinfection, apnea, seizures,
Guillain–Barré syndrome, Reye syndrome
Human metapneumovirus Winter Similar to respiratory syncytial virus

Others
Foreign body, food or gastric acid aspiration, hypersensitivity to inhaled material (hydrocarbon or lipoid substances), drug- or
radiation-induced pneumonitis

Table 1.4 Causes of pleural effusion.

Category Causes

Infection (exudates) Bacterial, viral, fungal, mycobacterial


Cardiovascular Congestive heart failure, constrictive pericarditis, superior vena cava obstruction, postcardiac surgery
Pulmonary Pulmonary infarction, atelectasis, pulmonary embolism, drug-induced pleuritic reaction
Intra-abdominal Abdominal surgery, pancreatitis, hepatitis, peritonitis, subdiaphragmatic or intrahepatic abscess, cirrhosis
with ascites
Renal Uremia, urinary tract obstruction, nephrotic syndrome, peritoneal dialysis
Neoplastic Lymphoma, leukemia, carcinoma
Collagen vascular Rheumatoid arthritis, systemic lupus erythematosus, Sjögren syndrome, Wegener granulomatosis,
disease myxedema, sarcoidosis
Iatrogenic/ Extravascular central venous catheter placement (i.e. misplaced catheter), radiation therapy, hemothorax,
miscellaneous chylothorax, hypoalbuminemia, esophageal rupture

What is the management of empyema? be assessed using imaging studies such as ultrasonography
and computed tomography (CT). In stage 1 (exudative),
Treatment of empyema is based on stage of disease
fibrinous exudates, which are clear and sterile, form on
(exudative, fibrinopurulent, and organizing) which can
the surface of the pleura. In stage 2 (fibrinopurulent),
Downloaded from https://onlinelibrary.wiley.com/doi/ by National Institute Of Standard, Wiley Online Library on [06/03/2024]. See the Terms and Conditions (https://onlinelibrary.wiley.com/terms-and-conditions) on Wiley Online Library for rules of use; OA articles are governed by the applicable Creative Commons License
4 Cases in Pediatric Acute Care

Table 1.5 Characteristics of an exudative effusion (empyema).

Characteristic Transudate Empyema

Appearance Clear Cloudy


Cell count (per mm3) <1000 Usually >50 000
Cell type Lymphocytes, monocytes Neutrophils
Lactate dehydrogenase (LDH) <200 U/L >1000 U/L
Pleural fluid LDH/serum LDH <0.6 >0.6
Protein <3 g Rare Frequent
Pleural fluid protein/serum protein <0.5 >0.5
Glucose Normal (>60 mg/dL) Low (<40 mg/dL)
pH Normal (7.40–7.60) <7.30
Gram stain Negative Sometimes positive

the exudates become thick and purulent. Septations are order to debride or lyse adhesions and drain loculated
formed creating infected pleura that are loculated. In stage areas of pus, additional approaches could be utilized
3 (organizing), granulation tissues are created from including intrapleural fibrinolytic therapy (streptokinase,
proliferation of the fibroblast-creating areas of abscess urokinase, or tissue plasminogen activator) and surgical
cavities surrounding the lung. Treatment involves antibi- options (selected video-assisted thoracoscopy and
otic therapy and drainage with tube thoracostomy. In decortication).

Further Reading
Brunow de Carvalho, W., Johnston, C., and Fonseca, M.C. (ed. B.P. Fuhrman and J.J. Zimmerman), 175–178.
(2015). Bronchiolitis and pneumonia. In: Rogers’ Textbook of Philadelphia, PA: Elsevier.
Pediatric Intensive Care (ed. D.H. Shaffner and D.G. Sandora, T.J. and Sectish, T.C. (2011). Community-acquired
Nichols), 716–730. Philadephia, PA: Lippincott Williams & pneumonia. In: Nelson Textbook of Pediatrics
Wilkins. (ed. R. Kliegman), 1474–1479. Philadelphia, PA: Elsevier
Cashen, K. and Petersen, T.L. (2017). Pleural effusions and Saunders.
pneumothoraces. Pediatrics in Review 38: 170. doi: 10.1542/ Winnie, G.B. and Lossef, S.V. (2011). Pleurisy, pleural
pir.2016-0088. effusions, and empyema. In: Nelson Textbook of Pediatrics
Pasala, S., Storm, E.A., Stroud, M.H. et al. (2017). Pediatric (ed. R. Kliegman), 1505–1509. Philadelphia, PA: Elsevier
vascular access and centeses. In: Pediatric Critical Care Saunders.
Downloaded from https://onlinelibrary.wiley.com/doi/ by National Institute Of Standard, Wiley Online Library on [06/03/2024]. See the Terms and Conditions (https://onlinelibrary.wiley.com/terms-and-conditions) on Wiley Online Library for rules of use; OA articles are governed by the applicable Creative Commons License
5

Infant with Diaphoresis during Feedings


Kristen Brown and Kristen Nelson McMillan
Johns Hopkins University School of Nursing, Baltimore, MD, USA

A 10-week-old infant presents to the emergency depart- thin, pale, lethargic, and tachypneic, with mild retractions.
ment (ED) with lethargy, described as sleeping more and Her skin is clear with no rashes or other significant skin
being less playful, associated with reduced oral intake lesions. Sinus tachycardia is noted on the monitor. On auscul-
and increased work of breathing. tation, a grade II/VI holosystolic murmur at the mid lower left
sternal border with radiation to the cardiac apex is noted. An
S4 gallop is noted at the cardiac apex. There are no rubs or
History of Present Illness clicks. The abdomen is soft, non-distended, and non-tender.
Liver edge is palpable 3–4 cm below the right costal margin.
She has become diaphoretic and tachypneic with feedings There are no palpable masses or splenomegaly. Bowel
and requires multiple rest periods to finish a bottle. sounds are normal. Extremities are symmetric and cool,
with peripheral pulses 1+ in all extremities. Capillary refill
is 3–4 seconds.
Past Medical History

This infant was full term and delivered via vaginal delivery. Questions
Birthweight was 3.5 kg. Immunizations are up to date for
age. Formula fed and on multivitamins every day. Answer the following questions using the details provided.
1 Based on the case presentation, what is your differential
Past Surgical History diagnosis?
2 Which diagnostic studies are indicated?
No prior surgeries.
Continuation of case: Initial diagnostic studies show the
following results. The complete metabolic profile is unre-
Family History markable. The complete blood count demonstrates a white
blood cell count of 9000/mm3 and hemoglobin of 12.4 g/dL.
The parents have no significant medical history. The The chest radiograph shows moderate cardiomegaly with
5-year-old sibling is healthy. No one is sick at home. mild pulmonary edema. The 12-lead electrocardiogram
(EKG) shows sinus tachycardia, and normal PR and QTc
intervals with left axis deviation. Voltage evidence of biven-
Current Status tricular hypertrophy is present. No significant Q waves or
ST-segment changes are noted.
On arrival at the ED, axillary temperature is 36.8 C (98.2 F),
respiratory rate 65 breaths per minute, heart rate 165 bpm in 3 What additional studies are needed?
normal sinus rhythm, blood pressure 92/68 mmHg, and pulse 4 Which therapies and medications are appropriate in the
oximeter 99% on room air. Weight is 4.3 kg. The baby is mildly management of this child?

Cases in Pediatric Acute Care: Strengthening Clinical Decision Making, First Edition. Edited by Andrea M. Kline-Tilford and Catherine M. Haut.
© 2020 John Wiley & Sons Ltd. Published 2020 by John Wiley & Sons Ltd.
Downloaded from https://onlinelibrary.wiley.com/doi/ by National Institute Of Standard, Wiley Online Library on [06/03/2024]. See the Terms and Conditions (https://onlinelibrary.wiley.com/terms-and-conditions) on Wiley Online Library for rules of use; OA articles are governed by the applicable Creative Commons License
6 Cases in Pediatric Acute Care

Rationale and Evidence-based Practice resistance and augment cardiac output. Most often, an angio-
Explanation tensin-converting enzyme (ACE) inhibitor is used to decrease
systemic vascular resistance and decrease the renin–
Based on the case presentation, what is your angiotensin–aldosterone system which is continuously acti-
differential diagnosis? vated with CHF. Spironolactone, a weak diuretic, is often
added to the regimen for potassium-sparing effect and to
This patient has signs of cardiac failure: tachycardia, poor augment lowering of the renin–angiotensin–aldosterone sys-
capillary refill, hepatomegaly, poor weight gain, and leth- tem. Milrinone is often used in the intensive care setting in
argy. Differential diagnosis for this child includes conges- acute new-onset systemic ventricular dysfunction to improve
tive heart failure secondary to structural heart disease, cardiac output and reduce left ventricular afterload. Care must
cardiomyopathy, myocarditis, arrhythmia, and anemia. be taken to avoid hypotension as milrinone results in systemic
The initial differential diagnosis also includes an intercur- vasodilation and may also result in some pulmonary vasodila-
rent respiratory illness (e.g. respiratory syncytial virus). tion given its general vascular dilation effect. Digoxin may be
Which diagnostic studies are indicated? used to treat cardiac failure in place of or to augment ACE
inhibitor use. The action of digoxin is mediated through inhi-
The initial diagnostic studies will focus on evaluation of car- bition of Na+/K+-ATPase pumps, which leads to a slight
diac function and of infection. A complete blood count and increase in the force of contraction (inotropy).
respiratory viral panel will help determine if an infectious eti- Oxygen is not necessary for this infant; however, if respira-
ology is present; hemoglobin status is also evaluated. Blood tory support is needed, consider non-invasive respiratory sup-
and urine cultures may also be ordered to evaluate for infec- port such as continuous positive airway pressure (CPAP)
tion. A complete metabolic panel can be obtained to evaluate or HFNC at the lowest oxygen concentration to maintain nor-
hydration status and renal and hepatic function. A chest radi- mal saturations. Oxygen can result in an increase in left-to-
ograph is needed to evaluate lung fields and heart size and an right shunting as it causes lowering of pulmonary vascular
EKG is required to evaluate rhythm and voltages. resistance, which can result in worsening of pulmonary
edema; thus, oxygen is used cautiously in this presentation.
What additional studies are needed?
To promote growth, frequent high-calorie feedings deliv-
An echocardiogram is necessary to evaluate underlying ered either orally or by nasogastric tube are initiated in
structural disease due to physical examination findings patients awaiting surgical management. Support from a
and cardiomegaly on chest radiograph. The echocardio- dietitian is often needed for these children.
gram demonstrates a large perimembranous ventricular Infants with a large VSD defect and congestive heart fail-
septal defect (VSD) with non-restrictive left-to-right shunt- ure are candidates for surgical repair. Medical management
ing. All cardiac chambers are dilated. Left ventricular con- with diuretics, ACE inhibitors, and/or digoxin are often
tractility is at the lower range of normal. There is no needed to promote growth prior to VSD repair, typically per-
pericardial effusion. This acyanotic cardiac lesion is associ- formed between 3 and 6 months of age. Over time, unre-
ated with increased pulmonary blood flow and left-to-right paired, large left-to-right shunts (e.g. VSD) result in
shunting, resulting in pulmonary edema and increased pulmonary artery hypertension, increased pulmonary vascu-
work of breathing (congestive heart failure). lar resistance, and right ventricular overload, ultimately
resulting in reversal of shunt direction and Eisenmenger syn-
Which therapies and medications are appropriate drome. Traditionally, VSDs have been closed surgically, but
in the management of this child?
advances in catheter intervention procedures have expanded
Therapy is targeted at improving congestive heart failure. candidacy for transcatheter closure of perimembranous
Pharmacologic strategies may include diuretics (e.g. furose- VSDs. In infants failing to achieve adequate growth/weight
mide), afterload-reducing agents, and digoxin. Diuretics are gain on optimized medical management and tube feedings
often indicated to reduce total body water, alleviating cardiac who are not yet surgical candidates, a pulmonary artery band
workload and reducing pulmonary edema. Afterload may be placed to restrict pulmonary blood flow and augment
reduction may be considered to decrease systemic vascular systemic blood flow until the child is ready for VSD closure.

Further Reading
Park, M.K. (2014). Park’s Pediatric Cardiology for Practitioners, You, T., Yi, K., Ding, Z.-h., et al. (2017). Transcatheter closure,
6th edn. Philadelphia, PA: Elsevier Saunders. mini-invasive closure and open-heart surgical repair for
Ungerleider, R.M., Meliones, J.N., Nelson-McMillan, K., Cooper, treatment of perimembranous ventricular septal defects in
D.S., and Jacobs, J.P. (ed.) (2019). Critical Heart Disease in children: a protocol for a network meta-analysis. BMJ Open
Infants and Children, 3rd edn. Philadelphia, PA: Elsevier. 7: e015642. doi: L10.1136/bmjopen-2016-015642.
Downloaded from https://onlinelibrary.wiley.com/doi/ by National Institute Of Standard, Wiley Online Library on [06/03/2024]. See the Terms and Conditions (https://onlinelibrary.wiley.com/terms-and-conditions) on Wiley Online Library for rules of use; OA articles are governed by the applicable Creative Commons License
7

Hypotension in the Operating Room


Ann Marie Felauer
University of Maryland School of Nursing, Baltimore, MD, USA

A 12-year-old female, weighing 45 kg, is admitted to the ruptured during the surgery and an open appendectomy
pediatric intensive care unit (PICU) following a ruptured with washout was completed. She required two fluid
appendix requiring open appendectomy with washout. boluses of 20 mL/kg for a total of 1800 mL of normal
During the intraoperative period she developed hypoten- saline due to hypotension. An infusion of dopamine was
sion that was unresponsive to fluid resuscitation and initiated for continued SBP of 68–78 mmHg. Dopamine
required initiation of dopamine to maintain arterial systolic was titrated to 10 μg/kg per min to maintain SBP above
blood pressure (SBP) of 80 mmHg. The need for inotropic 80 mmHg and mean arterial pressure (MAP) above
support and concern for continuing hypotension was the 60 mmHg. All other vital signs remained within accepted
rationale for PICU admission. parameters. Despite hypotension, the remainder of the pro-
cedure went well. At the conclusion of the case, the patient
was given glycopyrrolate and neostigmine, anesthesia was
History of Present Illness reversed, and the child was extubated without difficulty.
The patient received a total of 1800 mL of crystalloid fluid,
This is a previously healthy 12-year-old female who began and there was urine output of 160 mL, with overall mini-
complaining of abdominal pain and fevers 4 days prior to mal blood loss with the surgical procedure. Because of the
admission. At that time she was seen by her primary care need for inotropic support and continued hypotension, the
provider and referred to the local emergency department patient was transferred to the PICU for continued care.
(ED). Abdominal ultrasound was negative at that time
and she was discharged home with a diagnosis of viral gas-
troenteritis. She presented to the ED last evening with Past Medical History/Past
worsening pain at McBurney’s point and positive Rovs- Surgical History
ing’s sign. Because of concerns for appendicitis she was
admitted to surgical team and taken to the operating room This patient was born at term by spontaneous vaginal deliv-
for a laparoscopic appendectomy and had an easy mask ery and was discharged home with her mother. She has had
induction with oxygen and sevoflurane. She was given all required vaccines along with annual influenza. Her
rocuronium, fentanyl, and ondansetron and was intubated weight is at the 44% percentile for gender and age and
with a 7.0 cuffed endotracheal tube. End-tidal carbon has been consistently in this trajectory, with a current body
dioxide (CO2) recording was obtained and maintained mass index (BMI) of 18.1. There are no known chronic ill-
throughout the procedure and bilateral breath sounds nesses and no prior surgical procedures or hospitalization.
were auscultated. Once asleep she had two peripheral
intravenous catheters placed in her left and right hand
as well as a left radial 20-gauge arterial line placed. Anes- Family History
thesia was maintained throughout the case with sevo-
flurane, and with intermittent doses of fentanyl and The patient’s mother has migraine headaches and her
rocuronium. Antibiotics that were started on admission father has type 2 diabetes mellitus, treated with oral
in the ED were continued during the case. The appendix anti-hyperglycemic agents.

Cases in Pediatric Acute Care: Strengthening Clinical Decision Making, First Edition. Edited by Andrea M. Kline-Tilford and Catherine M. Haut.
© 2020 John Wiley & Sons Ltd. Published 2020 by John Wiley & Sons Ltd.
Downloaded from https://onlinelibrary.wiley.com/doi/ by National Institute Of Standard, Wiley Online Library on [06/03/2024]. See the Terms and Conditions (https://onlinelibrary.wiley.com/terms-and-conditions) on Wiley Online Library for rules of use; OA articles are governed by the applicable Creative Commons License
8 Cases in Pediatric Acute Care

Current Status Table 3.2 Complete blood count.

White cell count 22 000/mm3


On arrival at the PICU, the patient is alert and talking, cur-
Hemoglobin 11 g/dL
rently on oxygen by nasal cannula at 1 L/min and with pulse
oximetry at 99–100%. Current vital signs are as follows: Hematocrit 33%

••
Platelet count 195 000/mm3
weight documented prior to surgery, 45 kg;
Neutrophils 79%
heart rate 82–89 bpm;

• arterial blood pressure 72/46 to 84/57 mmHg; MAP


55–64 mmHg;
Lymphocytes
Monocytes
9%
4%

•• temperature 38.3 C (100.9 F) via Foley catheter;


respiratory rate 14–18/min;
Eosinophils 1%

• central venous pressure (CVP) 5–8 mmHg measured via


right subclavian double-lumen central line catheter, cur-
Questions
rently infusing dopamine at 10 μg/kg per min, plus dextrose
5% in 0.9% normal saline with KCl 20 mEq/L at 85 mL/hour;

• piperacillin/tazobactam 3 g given immediately post-


operatively and continued every 6 hours due to concerns
Answer the following questions using the details provided.
1 Based on the data given, what are the differential diag-
for peritonitis;


noses for this child?
intake/output since the operating room: intake 2055 mL; 2 What additional diagnostic studies and/or consults
output 240 mL urine (1.33 mL/kg per hour). should be obtained?
3 What is the best way to maximize blood pressure and
Physical examination showed the following.


what is the goal MAP?
General: awake, alert, interactive, talking, without
Continuation of case: After discussion with the surgical
reported pain present.

• Respiratory: lungs clear to auscultation, no retractions,


full aeration.
team, the patient received another 20 mL/kg fluid bolus
on arrival at the PICU for continued hypotension and

• Cardiac: regular rate and rhythm, normal S1, S2, no


murmurs noted, capillary refill <3 seconds, pulses
was started on an epinephrine infusion at 0.03 μg/kg per
min, titrated up to a goal of maintaining MAP above
70–75 mmHg. The patient’s oxygen and respiratory effort
2+ throughout, extremities warm and well perfused.

• Abdomen: soft, tender to palpation, no organomegaly


noted, midline abdominal incision dressing intact, drain
remained stable and she is continuously monitored.
The PICU elected to obtain mixed venous oxygen saturation
level via the subclavian line with a repeat CBC, lactate level,
in place with minimal serosanguineous drainage and
and BMP, with results showing SvO2 72%, CBC and BMP sta-
absent bowel sounds.

• Neurologic: alert and oriented ×3.

A basic metabolic panel (BMP), complete blood count


ble, and lactate 1.4 mmol/L. The surgical team was reluctant to
initiate norepinephrine due to concerns for perfusion to the
surgical site, so epinephrine was titrated to 0.09 μg/kg per
(CBC) and lactate were obtained, as well as ionized calcium min with SBP maintained at 88–90 mmHg. SvO2 remained
and blood culture (see Tables 3.1 and 3.2). at 70–75% and urine output was 0.8–1.1 mL/kg per hour.
4 What evidence guides the decision to use epinephrine
Table 3.1 Basic metabolic panel. or norepinephrine?
5 Would you obtain any other diagnostic studies?
Sodium 134 mEq/L
Potassium 3.8 mEq/L
Chloride 101 mEq/L
Carbon dioxide 24 mEq/L Rationale and Evidence-based Practice
Blood urea nitrogen 12 mg/dL Explanation
Creatinine 0.6 mg/dL
Based on the data given, what are the differential
Glucose 53 mg/dL
diagnoses for this child?
Calcium 110 mg/dL
Ionized calcium 4.5 mg/dL Differential diagnoses for this child include persistent
shock due to significant fluid loss, fluid shifts or ascites,
Serum lactate 2.8 mmol/L
cardiac dysfunction, cardiac tamponade or tension
Downloaded from https://onlinelibrary.wiley.com/doi/ by National Institute Of Standard, Wiley Online Library on [06/03/2024]. See the Terms and Conditions (https://onlinelibrary.wiley.com/terms-and-conditions) on Wiley Online Library for rules of use; OA articles are governed by the applicable Creative Commons License
Chapter 3 Hypotension in the Operating Room 9

pneumothorax, or catecholamine resistance. Given this What is the best way to maximize blood pressure
child’s physical examination, cardiac dysfunction can be and what is the goal MAP?
excluded. According to the “early goal-directed therapy
In addition to capillary refill of 2 seconds or less and nor-
for treatment of shock” protocol, the patient has received
malization of blood pressure, goals for this patient include
appropriate treatment with fluid boluses and initiation
urine output above 1 mL/kg per hour, normal mental sta-
of inotropic support. However, given the other measures
tus, normalization of serum lactate (beyond the operative
of perfusion are reassuring, evaluation for adrenal
period), maintaining normal perfusion pressure (MAP –
insufficiency should be considered at this point. Adrenal
CVP for age), and SvO2 greater than 70%. Administration
insufficiency in this patient is most likely due to a proin-
of hydrocortisone will improve the hypotension and the
flammatory response related to septic shock which has
vasoactive infusions can be weaned in order of initiation,
caused decreased cortisol production and synthesis. This
with epinephrine discontinued first. Goals should continue
dysfunction of the hypothalamic–pituitary–adrenal axis
to be normalization of perfusion, cardiac output as well as
can occur during a critical illness and results in an inade-
oxygenation and ventilation.
quate response to stress.
What evidence guides the decision to use
What additional diagnostic studies and/or
epinephrine or norepinephrine?
consults should be obtained?
Epinephrine is used in the presence of cold shock, which in
Prior to treating for adrenal insufficiency, assessment of
this case was determined by hypotension as the primary
adrenocortical function should be completed. Obtaining a
symptom with the knowledge that a ruptured appendix
morning cortisol level (at 8 a.m.) would be ideal; however,
can contribute to peritonitis, or an abdominal infection,
in situations such as seen with surgery there may be an
causing sepsis. Norepinephrine is primarily used for man-
alteration in cortisol levels due to the stress of the procedure
agement of “warm shock,” during which time blood pres-
alone. An adrenocorticotropic hormone (ACTH) stimula-
sure may be normal or low in children.
tion test also evaluates adrenal gland function and is more
reasonable in this circumstance. Measurements of serum
Would you obtain any other diagnostic studies?
cortisol levels are obtained at baseline and then 60 minutes
after an infusion of synthetic ACTH. Recommended dosing Given the fact that this patient developed adrenal insuffi-
is 250 μg for children over 2 years of age, given over ciency after a ruptured appendix versus longer-term critical
1 minute. Serum cortisol levels are then evaluated in com- illness and is at risk for further hypothalamic–pituitary–
parison to baseline levels, with a normal response being a adrenal dysfunction, an endocrinology consult would be
minimum of >18 μg/dL and at least 7–10 μg/dL greater helpful for evaluation. Continued monitoring of electrolytes
than the baseline. while the patient is NPO and frequent intake and output are
Treatment of catecholamine-resistant shock requires part of PICU care. Avoidance of hypocalcemia as well as
stress dose hydrocortisone 50–100 mg/m2 (maximum 100 hypoglycemia/hyperglycemia is important for preventing
mg/dose) followed by 50 mg/m2 per 24 hours (maximum secondary complications of critical illness. The patient pro-
100 mg/dose) divided every 4–6 hours until the patient is gressed to spend another day in the PICU for weaning of
hemodynamically stable and able to be weaned from vaso- inotropic support, was stabilized, and was transferred to
active medications. the pediatric general unit on day 3 of hospitalization.

Further Reading
Bornstein, S.R., Allolio, B., Arlt, W., et al. (2016). Diagnosis and Davis, A.L., Carcillo, J.A., Aneja, R.K., et al. (2017). American
treatment of primary adrenal insufficiency: an Endocrine College of Critical Care Medicine clinical practice
Society Clinical Practice Guideline. Journal of Clinical parameters for hemodynamic support of pediatric and
Endocrinology and Metabolism 101(2): 364–389. doi: neonatal septic shock. Critical Care Medicine 45(6):
10.1210/jc.2015-1710. 1061–1093. doi: 10.1097/CCM.0000000000002425.
Downloaded from https://onlinelibrary.wiley.com/doi/ by National Institute Of Standard, Wiley Online Library on [06/03/2024]. See the Terms and Conditions (https://onlinelibrary.wiley.com/terms-and-conditions) on Wiley Online Library for rules of use; OA articles are governed by the applicable Creative Commons License
11

Toddler with Vomiting and Diarrhea


Kimberly L. DiMaria
Children’s Hospital Colorado, Aurora, CO, USA

A full-term previously healthy 18-month-old female pre- consistently at the 25th percentile on the growth curve.
sents to the emergency department (ED) with 2 days of Her vaccinations are up to date, including seasonal
diarrhea, emesis, and low-grade fever. influenza vaccine. She has no known drug allergies. No
hospitalizations.

History of Present Illness

The toddler was in her usual state of health until 3 days Past Surgical History
prior to presentation at which time she developed
decreased appetite and intermittent complaints of abdom- No previous surgeries.
inal pain. The following day, she developed a tactile tem-
perature and had frequent bowel movements. Her
mother reports that the stools were loose. The day prior
Family History
to presentation, she developed emesis and was unable to
keep any food or liquids down. The mother reported fre-
Aunt with colon cancer, mother with asthma. Three
quent trips to the bathroom for vomiting and diarrhea.
healthy older siblings.
The mother encouraged fluids but was only able to get
the child to take sips throughout the day. Overnight, she
was restless, febrile to 38.3 C (100.9 F), and complaining
of a worsening “tummy ache.” This morning, the mother Current Status
noticed that the child appeared tired and pale, so she was
brought to the ED for further evaluation. She is a well-developed toddler sleeping in her mother’s lap.
Vital signs in ED triage as follows: temperature 37.9 C
(100.2 F), heart rate 130 bpm, respiratory rate 35 breaths
Past Medical History per minute, blood pressure 92/55 mmHg, oxygen satura-
tions 96% on room air.
Born at full term via scheduled cesarean section. She had She appears lethargic, but irritable with examination.
APGAR scores of 7 and 9 after birth and was discharged Her eyes are slightly sunken and mucous membranes are
home with mother on day of life 3. She was closely followed dry. No tears are noted when crying during examination.
by primary care physician (PCP) for slow weight gain dur- Her capillary refill time is 4 seconds and she has 2+ pulses
ing the first 3 months of life that improved after initiation in all extremities. She cries out when the examiner attempts
of formula supplementation. She has had multiple upper to palpate her abdomen, but all four quadrants are palpated
respiratory tract infections and one episode of acute without evidence of tenderness. Her abdomen is round,
otitis media that required antibiotics. She is otherwise soft, and non-distended. No hepatosplenomegaly. Hyperac-
healthy, meeting developmental milestones and tracking tive bowel sounds are present.

Cases in Pediatric Acute Care: Strengthening Clinical Decision Making, First Edition. Edited by Andrea M. Kline-Tilford and Catherine M. Haut.
© 2020 John Wiley & Sons Ltd. Published 2020 by John Wiley & Sons Ltd.
Downloaded from https://onlinelibrary.wiley.com/doi/ by National Institute Of Standard, Wiley Online Library on [06/03/2024]. See the Terms and Conditions (https://onlinelibrary.wiley.com/terms-and-conditions) on Wiley Online Library for rules of use; OA articles are governed by the applicable Creative Commons License
12 Cases in Pediatric Acute Care

Questions a common illness, especially in children under 5 years of


age. The highest incidence of dehydration is in children
Answer the following questions using the details provided. under 6 months of age. Most commonly, gastroenteritis is
the result of infection with rotavirus, norovirus, adenovi-
1 Based on the data given, what is the differential diagno- rus, Salmonella, Campylobacter, and Clostridium difficile.
sis and the most likely diagnosis for this child? Etiology varies by geographic location and vaccination
2 What are the essential questions to ask while obtaining against rotavirus. Diarrheal illness characterized by fre-
the history? quent small amounts of stool, bloody stool, high fever,
3 What are the initial steps of management? and abdominal pain are most likely to be associated with
4 How would you evaluate the child’s hydration status? a bacterial etiology.
5 What is your management plan?

What are essential questions to ask while


obtaining the history?
Rationale and Evidence-based Practice Essential elements of the history include onset and dura-
Explanation tion of the emesis, including color (bloody, bilious) and
timing (immediately after eating?); onset and color
Based on the data given, what is the differential (blood, mucus) of diarrhea; fluid intake and urinary out-
diagnosis and the most likely diagnosis for put (dysuria?); immunization history (specifically rotavi-
this child? rus vaccination); and contact with others with similar
The initial differential diagnosis is broad and includes symptoms.
gastroenteritis, appendicitis, central nervous system infec-
tion, bowel obstruction, bowel perforation/acute abdomen, What are the initial steps of management?
diabetes ketoacidosis, and ingestion. When further consid- Priority is given to evaluation for presence of hypovolemic
ering the differential diagnosis, appendicitis is on the list shock. If present, this must be treated immediately, as it can
because of fever, vomiting, diarrhea, and lethargy, though be life-threatening. The child’s level of dehydration should
is less likely due to absence of rebound tenderness or tender be evaluated and treatment targeted to level of dehydration
abdomen. Central nervous system infection is considered and replacement of ongoing losses. Other steps in
on the differential diagnosis due to fever, emesis, and management include initiating an age-appropriate diet,
altered mental status (lethargic and irritable); a thorough determining the disposition (e.g. does the child require
neurologic examination including assessment for nuchal hospitalization?), and considering an antiemetic if emesis
rigidity is needed. Bowel obstruction is considered in the prohibits effective oral rehydration therapy or for patient
differential due to frequent emesis coupled with abdominal comfort.
pain, though is less likely because the emesis is not bilious In general, children with acute gastroenteritis do not
and the patient is having diarrhea. A bowel perforation/ require diagnostic evaluation with stool studies except
acute abdomen is unlikely because of reassuring abdominal when the child has underlying chronic conditions, pro-
examination. Diabetic ketoacidosis is considered in the longed symptoms, bloody diarrhea, travel history, or during
differential diagnosis due to presenting emesis, evidence outbreaks.
of dehydration, and mental status changes, but is not
accompanied by Kussmaul breathing or polyuria. Toxic
How would you evaluate the child’s hydration
ingestion should also be considered, though is less likely
status?
due to low-grade fever and prolonged illness.
The most likely diagnosis for this patient is gastroenteritis The largest driver of morbidity and hospitalization rates
because of the symptoms of diarrhea, low-grade fever, associated with acute gastroenteritis is dehydration. The
abdominal pain, and emesis in the absence of a rigid and exact proportion of a patient’s dehydration is thought to
tender abdomen and without focal neurologic symptoms. be represented by the difference in their pre-illness weight
Gastroenteritis is defined as acute onset of diarrheal illness and their weight at time of presentation. This assessment is
(e.g. increase in stool frequency and/or decrease in stool limited in its utility because most parents are unsure of the
consistency) that can occur either in isolation or in combi- patient’s exact pre-illness weight. Because of this limitation,
nation with nausea, emesis, fever, and abdominal pain. most clinicians rely on a clinical dehydration scale which
Acute gastroenteritis typically lasts less than 7 days. It is incorporates presenting symptoms to categorize the patient
Downloaded from https://onlinelibrary.wiley.com/doi/ by National Institute Of Standard, Wiley Online Library on [06/03/2024]. See the Terms and Conditions (https://onlinelibrary.wiley.com/terms-and-conditions) on Wiley Online Library for rules of use; OA articles are governed by the applicable Creative Commons License
Chapter 4 Toddler with Vomiting and Diarrhea 13

Table 4.1 Clinical dehydration scale.

Score 0 1 2

General appearance Normal Thirsty, restless or lethargic, but irritable when touched Drowsy, limp, cold or sweaty ± comatose
Eyes Normal Slightly sunken Extremely sunken
Mucous Moist Sticky Dry
membranes
Tears Tears Decreased tears Absent tears

Total score: 0, no dehydration; 1–4, some dehydration; 5–8, moderate/severe dehydration.


Source: Guarino, A., Ashkenazi, A., Gendrel, D., et al. (2014). European Society for Pediatric Gastroenterology, Hepatology, and Nutrition/
European Society for Pediatric Infectious Diseases evidence-based guidelines for the management of acute gastroenteritis in children in Europe:
update 2014. Journal of Pediatric Gastroenterology and Nutrition 59(1): 132–152. Reproduced with permission of Wolters Kluwer Health, Inc.

as having minimal (<3% loss of body weight), mild to mod- allowing her to drink more if she appears thirsty. In breast-
erate (3–9% loss of body weight), or severe (>9% of body fed infants, breastfeeding should continue through oral
weight) dehydration. Multiple clinical dehydration scales rehydration therapy. An age-appropriate diet can be
have been developed by several organizations (Table 4.1). initiated after the initial 4–6 hours of rehydration. Modified
The various scales vary in their reliability in predicting milk formula and dilution of formula is generally not
actual dehydration, but can give a healthcare provider an needed. Enteral hydration by nasogastric route is preferred
estimate of a patient’s dehydration status that will assist over intravenous hydration when oral rehydration is not
with creating a management plan. All scoring tools must feasible.
be used in conjunction with the individual clinical exami- The patient requires assessment of ongoing fluid losses
nation and history. Newer methods for estimating dehydra- and replacement as needed. An antiemetic such as ondan-
tion status have been evaluated, including ultrasound to setron can be prescribed if the child is unable to drink the
measure inferior vena cava diameter, the ratio of inferior prescribed amount of fluid without continued emesis.
vena cava to aorta diameter, and bedside hand-held bladder Probiotic therapy has been associated with reduction in
ultrasound scanning and digital videography to measure the duration of diarrhea, but there is limited evidence of
capillary refill time. its impact on duration of hospitalization.
According to the clinical dehydration scale, this child is The patient does not meet hospital admission criteria
classified as having moderate dehydration (restless, irrita- at this time. However, if she were unable to complete
ble, tachypneic, sunken eyes, dry mucous membranes). oral rehydration therapy because of ongoing emesis or
diarrhea, if oral rehydration therapy at home was unsuc-
cessful, or if she developed signs of hypovolemic shock,
What is your management plan?
she would warrant either prolonged observation in the
The management plan for this child is to initiate oral rehy- ED or hospital admission. No laboratory studies or intra-
dration, administering 80–100 mL/kg of oral rehydration venous fluids are indicated currently. Additionally, a
fluid over 2–4 hours. Oral rehydration therapy is recom- basic metabolic panel to assess electrolytes, specifically
mended for mild to moderate cases of acute gastroenteritis. sodium, as well as blood urea nitrogen and creatinine
She warrants close observation and may require intrave- to assess renal function and severity of dehydration,
nous fluid therapy. She should be offered an oral rehydra- could be considered if she was refractory to oral rehy-
tion fluid instead of water, juice, or a sports electrolyte dration therapy and developed severe dehydration. Anti-
drink. She has lost electrolytes because of her diarrhea biotics are not a recommended therapy for acute
and emesis and therefore water would not meet her electro- gastroenteritis.
lyte replacement needs. A high-glucose solution (most com- The patient requires ongoing monitoring of end-organ
mon in juice or sports drinks) can exacerbate diarrhea. perfusion and level of hydration, and for symptoms of acute
Ideally, an oral rehydration fluid would have a 1 : 1 glucose abdominal process such as appendicitis, bowel obstruction,
to sodium ratio. She should be offered 50–100 mL/kg body or perforation, which are less common. If she improves
weight of oral rehydration fluid over 2–4 hours to replace after tolerating oral rehydration therapy, she can be safely
the estimated fluid deficit. Caregivers should offer small discharged home with strict return instructions for deteri-
amounts of oral rehydration therapy via spoon or syringe, oration in status.
Downloaded from https://onlinelibrary.wiley.com/doi/ by National Institute Of Standard, Wiley Online Library on [06/03/2024]. See the Terms and Conditions (https://onlinelibrary.wiley.com/terms-and-conditions) on Wiley Online Library for rules of use; OA articles are governed by the applicable Creative Commons License
14 Cases in Pediatric Acute Care

Further Reading
Guarino, A., Ashkenazi, A., Gendrel, D., et al. (2014). Hendrickson, M.A., Zaremba, J., Wey, A.R., Gaillard, P.R., and
European Society for Pediatric Gastroenterology, Kharbanda, A.B. (2018). The use of triage-based protocol for
Hepatology, and Nutrition/European Society for Pediatric oral rehydration in a pediatric emergency department.
Infectious Diseases evidence-based guidelines for the Pediatric Emergency Care 34(4): 227–232.
management of acute gastroenteritis in children in Europe: Onyon, C. and Dawson, T. (2018). Gastroenteritis. Paediatrics
update 2014. Journal of Pediatric Gastroenterology and and Child Health 28(11): 527–532. doi: https://doi.org/
Nutrition 59(1): 132–152. doi:10.1097/ 10.1016/j.paed.2018.08.010.
MPG.000000000000000375.
Hartman, S., Brown, E., Loomis, E., and Russell, H. (2019).
Gastroenteritis in children. American Family Physician
99(3): 159–165.
Downloaded from https://onlinelibrary.wiley.com/doi/ by National Institute Of Standard, Wiley Online Library on [06/03/2024]. See the Terms and Conditions (https://onlinelibrary.wiley.com/terms-and-conditions) on Wiley Online Library for rules of use; OA articles are governed by the applicable Creative Commons License
15

Infant with Fever and Cold Symptoms


Cheryl Bartke
Seattle Children’s Hospital, Seattle, Washington, USA

A 10-week-old with fever and respiratory symptoms is He is admitted to the acute care floor from the ED with a
admitted to the pediatric floor. nasal cannula at 2 L/min and a saturation of 92%. He is
breathing at a rate of 60 breaths per minute with moderate
retractions and is still noted to have significant nasal
History of Present Illness
congestion. He continues to have intermittent wheezing.
A chest radiograph is completed (Figures 5.1 and 5.2).
This 10-week-old infant has had 3 days of rhinorrhea,
Rapid laboratory testing for respiratory viral panel is
congestion, and cough. In the past 12 hours, he has
pending.
developed post-tussive emesis, tactile fever, increased
work of breathing, and retractions.

Past Medical History Questions

Answer the following questions using the details provided.


He was born at term, is uncircumcised, and went home
with his mother. He had his 2-month vaccines last month. 1 Based on the data given, what is the differential diagno-
sis and the most likely diagnosis for this child?
2 What is the best way to increase this infant’s oxygena-
Past Surgical History
tion and how would you order this?
3 Would you consider albuterol at the same time?
Other than circumcision, he has not had any surgeries.
4 How would you determine if the therapy was effective?
5 Would you obtain any other diagnostic studies?
Family History 6 What other therapies would be recommended?
7 What anticipatory guidance should be given to the
Non-contributory. His older sibling is in daycare with parents?
current upper respiratory illness.

Current Status Rationale and Evidence-based


On arrival in the emergency department (ED), he had tach-
Practice Explanation
ypnea to 80 breaths per minute, a congested cough, subcla-
Based on the data given, what is the differential
vicular, intercostal and subcostal retractions, tracheal
diagnosis and the most likely diagnosis for
tugging, head bobbing, and nasal flaring. He was wheezing,
this child?
and hypoxic with an oxygen saturation of 86% on room air.
He was treated with nasal suctioning and supplemental The differential diagnoses for this child include viral bron-
oxygen at 100% via blow-by and a trial of albuterol, which chiolitis, viral or bacterial pneumonia, pertussis, and reac-
did not decreasing his wheezing. tive airway disease.

Cases in Pediatric Acute Care: Strengthening Clinical Decision Making, First Edition. Edited by Andrea M. Kline-Tilford and Catherine M. Haut.
© 2020 John Wiley & Sons Ltd. Published 2020 by John Wiley & Sons Ltd.
Downloaded from https://onlinelibrary.wiley.com/doi/ by National Institute Of Standard, Wiley Online Library on [06/03/2024]. See the Terms and Conditions (https://onlinelibrary.wiley.com/terms-and-conditions) on Wiley Online Library for rules of use; OA articles are governed by the applicable Creative Commons License
16 Cases in Pediatric Acute Care

Infants are obligatory nose breathers and find it difficult


to clear their nasal airways without assistance. A cardinal
manifestation of bronchiolitis in infants is copious and
tenacious airway mucus.

What is the best way to increase this infant’s


oxygenation and how would you order this?
The best way to increase oxygenation in an infant with
bronchiolitis is non-invasive nasal suctioning for airway
clearance. Infants are obligate nose breathers and have
difficulty clearing their nasal airway of the copious thick
mucus of bronchiolitis without assistance. The use of
normal saline during suctioning should be considered
to facilitate airway clearance. Deeper suctioning may
Figure 5.1 Anteroposterior (AP) chest radiograph. be required; however, it can lead to trauma and
increased airway edema which can further occlude the
upper airways.
Guidelines suggest that supplemental oxygen does not
need to be administered unless the saturation is less than
90%. If this decision is made, there are many methods to
deliver oxygen. In some cases, non-invasive positive pres-
sure ventilation or high-flow nasal cannula may decrease
the patient’s work of breathing.

Would you consider albuterol at the same time?


Guidelines include strong recommendations against the
use of bronchodilators in infants with wheezing in the set-
ting of bronchiolitis. Given this patient has no history of
wheezing, the likelihood of improving his symptoms with
albuterol are minimal. In addition, the likelihood of side
effects including tachycardia are not insignificant.

How would you determine if the therapy


was effective?
Frequent physical examinations are recommended.
Determination of improvement is based on clinical signs
and symptoms of decreasing respiratory distress. These
Figure 5.2 Lateral chest radiograph. include normalization of respiratory rate and heart rate,
and decreasing retractions, nasal flaring and head
bobbing
The most likely diagnosis for this child is viral bronchio-
litis due to the infant’s history and presenting symptoms of
Would you obtain any other diagnostic studies?
rhinorrhea, congestion, cough, fever, and tachypnea. Bron-
chiolitis is a leading cause of illness in the first year of life The diagnosis of bronchiolitis is based on physical examina-
and is often caused by respiratory syncytial virus, although tion and history. Routine radiographic and laboratory test-
other viruses such as human metapneumovirus and rhino- ing should not be performed. However, if there is suspicion
virus can be the offending pathogen. Most children tolerate of another process, they should be explored in a targeted
bronchiolitis and can be managed as outpatients with sup- manner. Infants under 12 weeks of age with a history of pre-
portive measures; however, it remains a leading cause of maturity, cardiopulmonary disease or immunodeficiency
hospitalization in infants. concerns place the child at higher risk for increased severity
Downloaded from https://onlinelibrary.wiley.com/doi/ by National Institute Of Standard, Wiley Online Library on [06/03/2024]. See the Terms and Conditions (https://onlinelibrary.wiley.com/terms-and-conditions) on Wiley Online Library for rules of use; OA articles are governed by the applicable Creative Commons License
Chapter 5 Infant with Fever and Cold Symptoms 17

of illness and require further exploration of potential causes What other therapies would be recommended?
of the symptoms.
Ensure the infant is appropriately hydrated. Given his
The young age of this patient prompted the original chest
recent post-tussive vomiting, insensible fluid losses in res-
X-ray, showing no focal abnormalities and an expected
piratory distress, and the likelihood that he has had a
heart size.
decreased oral intake prior to presentation, an assessment
Electrolyte and complete blood count testing may be con-
of his fluid status should be completed. Even slight dehy-
sidered, but usually do not provide diagnostic or manage-
dration can lead to increased tenacity of the airway secre-
ment assistance in the setting of viral bronchiolitis.
tions, potentially further occluding his airway.
Viral testing is unlikely to assist in the clinical care and
Antipyretic administration may decrease the respiratory
management of the patient. However, it may provide an
and heart rates and help to improve the overall symptoms.
explanation of the symptoms and assist with the cohorting
Antimicrobials are not recommended unless there is evi-
of patients in the inpatient setting. In the case of the pre-
dence or a strong suspicion of a concomitant bacterial infec-
senting patient, respiratory syncytial virus was detected
tion. Other therapies not recommended include chest
on the rapid respiratory panel.
physiotherapy and systemic corticosteroids.

Further Reading
American Academy of Pediatrics (2014). Clinical practice Louisdon, P. and Pringle, E. (2018). Diagnosis and
guideline: The diagnosis, management, and prevention of management of the child with acute upper and lower
bronchiolitis. Pediatrics 184(5): 1474–1502. airway disease. In: Pediatric Fundamental Critical
Dean, S. and Runde, D. (2015). Bronchiodilators for Care Support, 3rd edn (ed. M.A. Madden), 125–126.
bronchiolitis. American Family Physician 92(5): 349–350. Mount Prospect, IL: Society of Critical Care Medicine.
Fleisher, G. (2006). Infectious disease emergencies. In: Norris, C., O’Neal, P., Adams, E., and Wyatt, T. (2018).
Textbook of Pediatric Emergency Medicine, 5th edn Nasal airway clearance for bronchiolitis. American
(ed. G. Fleisher, S. Ludwig and F. Henretig), 813–814. Journal of Maternal Child Nursing 43(6): 318–323.
Philadelphia, PA: Lippincott Williams & Wilkins.
Downloaded from https://onlinelibrary.wiley.com/doi/ by National Institute Of Standard, Wiley Online Library on [06/03/2024]. See the Terms and Conditions (https://onlinelibrary.wiley.com/terms-and-conditions) on Wiley Online Library for rules of use; OA articles are governed by the applicable Creative Commons License
19

Toddler with Pineoblastoma and Bone Marrow Transplant


Cathryn Freeman
Children’s Hospital of Michigan, Detroit, MI, USA

History of Present Illness Allergies include etoposide and blood products requiring
premedication prior to platelet transfusions. Current home
A 3-year-old African American male with a history of medications include levetiracetam, magnesium carbonate,
metastatic pineoblastoma presented to clinic for scheduled potassium phosphate, sodium phosphate, polyethylene
follow-up 32 days after his most recent bone marrow trans- glycol, and docusate as needed for constipation.
plant (BMT). He had been discharged from the hospital
2 weeks prior, following his second of three-tandem autol-
ogous stem cell transplants. His father reported that he felt Past Surgical History
warm to the touch beginning that morning and appeared to
have decreased energy, although his temperature was not Gross total resection of pineal lesion, VP shunt placement,
measured at that time. He ate breakfast and tolerated power port placement, and apheresis catheter placement.
his morning medications without nausea or vomiting.
He denies any cough, nasal congestion, or known sick Family History
contacts. He has been stooling twice daily, described as soft,
non-bloody, without abdominal pain. He is not toilet His maternal grandfather has hypertension and heart dis-
trained, with no change in urine output or characteristics ease. Remainder of family history unremarkable.
reported. His family denied bruising, bleeding, headaches,
or change in mental status.
Social History
Past Medical History He lives with his parents and 1-year-old brother. He is cared
for in the home by his parents and does not attend day care.
He was born full term and was a previously healthy male.
He is up to date on immunizations through age 2 years. One
week after his third birthday, he presented to the Current Status
emergency department (ED) with a 3-week history of eye
crossing, decreased hand–eye coordination, altered gait, On initial examination, he is sitting on his father’s lap,
once-weekly emesis, and regression in toileting. Computed appearing lethargic. He is answering questions appropri-
tomography (CT) of his head revealed obstructive ately for age, at his neurologic baseline.
hydrocephalus and calcified mass concerning for pineo- Vital signs are as follows: oral temperature 38.4 C
blastoma. He underwent a gross total resection of the pineal (101.4 F), heart rate 136 bpm, blood pressure 106/57 mmHg,
lesion. Postsurgical complications included seizures, and respiratory rate 36 breaths per minute, and oxygen
he was placed on levetiracetam. He currently has a saturation 100% on room air.
ventriculoperitoneal (VP) shunt. He received three cycles There is no source of fever evident on physical examina-
of induction chemotherapy, followed by peripheral blood tion. He has 2+ peripheral pulses, capillary refill <3 sec-
stem cell harvest. He then went on to receive high-dose onds, and his extremities are well perfused. Standard
chemotherapy with stem cell rescue. post-transplant laboratory studies are collected, including

Cases in Pediatric Acute Care: Strengthening Clinical Decision Making, First Edition. Edited by Andrea M. Kline-Tilford and Catherine M. Haut.
© 2020 John Wiley & Sons Ltd. Published 2020 by John Wiley & Sons Ltd.
Downloaded from https://onlinelibrary.wiley.com/doi/ by National Institute Of Standard, Wiley Online Library on [06/03/2024]. See the Terms and Conditions (https://onlinelibrary.wiley.com/terms-and-conditions) on Wiley Online Library for rules of use; OA articles are governed by the applicable Creative Commons License
20 Cases in Pediatric Acute Care

Table 6.1 Basic metabolic profile. 3 Looking at the laboratory results, what abnormalities
require immediate attention? What additional testing
Sodium 131 mEq/L
should be performed?
Potassium 3.9 mEq/L
Chloride 98 mEq/L
Continuation of case: Within the first 24 hours the blood
cultures from both lumens of his apheresis catheter result
Carbon dioxide 25 mEq/L
positive for Enterobacter cloacae. He remains stable with
Blood urea nitrogen 6 mg/dL
persistent fever, maximum temperature 39.4 C (102.9 F).
Creatinine 0.3 mg/dL
Glucose 87 mg/dL
4 With the findings of his blood cultures and persistent
fever, should his antibiotic regimen be adjusted? When
Magnesium 2.0 mg/dL
should additional coverage be added?
Phosphorus 2.1 mg/dL
Continuation of case: The following morning his clinical
course begins to deteriorate. Vital signs reveal a blood pres-
Table 6.2 Complete blood count. sure of 62/28 mmHg, respiratory rate 38 per minute, heart
rate 142 bpm, temperature 39.6 C (103.3 F), SpO2 92% on
White blood cell count 1300/mm3 room air, and flash capillary refill. He is flushed with pete-
Hemoglobin 7 g/dL chiae to his lower extremities, and lethargic.
Hematocrit 20.3%
5 What immediate interventions are required?
Platelet count 8000/mm3
Neutrophils 30%
Eosinophils 1% Rationale and Evidence-based Practice
Monocytes 33% Explanation
Lymphocytes 31%
Bands 0% Based on the information given, what are the
Absolute neutrophil count (ANC) 400 potential differential diagnoses?
Febrile neutropenia, systemic inflammatory response syn-
a complete blood count (CBC), reticulocyte count, basic drome (SIRS), and respiratory, urinary and/or bloodstream
metabolic panel, magnesium, phosphorus, bilirubin, alka- infections are all possible differential diagnoses. A fever
line phosphatase, alanine aminotransferase, aspartate ami- may be the only sign of infection in neutropenic patients,
notransferase, lactate dehydrogenase, immunoglobulin, due to the decreased neutrophil-mediated component of
and type and screen (see Tables 6.1 and 6.2). Additionally, the inflammatory response. The Infectious Diseases Society
blood cultures from both lumens of his apheresis catheter of America defines febrile neutropenia as a single oral tem-
and Mediport are collected, and a 0.9% normal saline fluid perature of 38.3 C (101 F) or above, or a temperature of
bolus of 20 mL/kg is infused. Cefepime is initiated within 38.0 C (100.4 F) or above sustained over a 1-hour period
30 minutes of identification of fever. He is admitted to in a patient with an absolute neutrophil cell count below
the BMT unit for further monitoring. 500/mm3. SIRS is a non-specific inflammatory state that
Following admission, a sodium and phosphorus bolus is may be seen with infection, requiring the presence of two
infused to correct for his hyponatremia and hypophospha- of the following criteria: temperature >38 or <36 C
temia. He is transfused with one unit of packed red blood (>100.4 or <96.8 F), tachycardia, tachypnea, and white cell
cells and platelets after receiving premedication with aceta- count >12 000/mm3 or <4000/mm3. Bloodstream infec-
minophen and diphenhydramine. tions, specifically Gram-negative organisms, are a frequent
complication of neutropenia in oncologic patients, requir-
ing prompt evaluation and empirical antibiotic therapy.
Questions
What sources of infection must be considered
Answer the following questions using the details provided. given this patient’s medical and surgical history?
1 Based on the information given, what are the potential In immunocompromised patients, one must consider all
differential diagnoses? major systems and sources of infection, including bacterial,
2 What sources of infection must be considered given this viral, and fungal etiologies. Respiratory and urinary tract
patient’s medical and surgical history? infections may be present in asymptomatic neutropenic
Downloaded from https://onlinelibrary.wiley.com/doi/ by National Institute Of Standard, Wiley Online Library on [06/03/2024]. See the Terms and Conditions (https://onlinelibrary.wiley.com/terms-and-conditions) on Wiley Online Library for rules of use; OA articles are governed by the applicable Creative Commons License
Chapter 6 Toddler with Pineoblastoma and Bone Marrow Transplant 21

patients. All implanted devices, including this patient’s empiric treatment for pediatric febrile neutropenia.
apheresis catheter, Mediport, and VP shunt, are possible A second antibiotic to cover Gram-negative organisms or
reservoirs of infection. a glycopeptide should be added when a patient becomes
clinically unstable, in suspected resistant Gram-negative,
Looking at the laboratory results, what Gram-positive and anaerobic bacterial infection, or in hos-
abnormalities require immediate attention? What pitals with a high rate of resistant organisms.
additional testing should be performed?
This patient is anemic and thrombocytopenic requiring What immediate interventions are required?
transfusion of packed red blood cells and platelets to It is evident that this child is now developing sepsis from
replenish his volume, improve oxygen-carrying capacity, Gram-negative bacteremia. Prompt recognition and inter-
and allow for improved coagulation and reduced risk of vention is critical to prevent the progression of sepsis to
bleeding complications. Infection results in decreased pro- shock. Support the child’s airway, oxygenation, and venti-
duction of red blood cells and platelets and increased plate- lation, and continue to monitor vital signs. Rapid fluid
let consumption. Therefore, a daily CBC is required to resuscitation with isotonic crystalloid to restore the intra-
monitor the need for ongoing transfusions. Peripheral vascular volume and improve tissue perfusion is the first
blood cultures should be considered in addition to central step in correcting these findings. Reevaluate the patient’s
line cultures as they increase the identification of bactere- physiologic response after each 20 mL/kg bolus to deter-
mia; however, collection should not delay the initiation of mine the need for further fluid boluses and assess for pul-
antibiotics. Chest radiography should only be obtained in monary edema as a consequence of resuscitation.
patients who present with or develop respiratory signs Following the Pediatric Advanced Life Support algorithm,
and symptoms. A clean-catch midstream urine sample if signs of shock persist including warm extremities, flash
for urinalysis and urine culture should also be considered capillary refill, and/or low blood pressure, initiation and
when feasible given the patient’s toileting abilities and titration of norepinephrine is required. Norepinephrine is
clinical status. the vasopressor of choice for fluid-refractory septic shock
due to its potent α-adrenergic vasoconstricting effects.
With the findings of his blood cultures and
persistent fever, should his antibiotic regimen be
adjusted? When should additional coverage
Case Resolution
be added?
The clinical practice guidelines recommend continuation The patient required transfer to the pediatric intensive care
of the initial empiric antibacterial coverage in children unit for management of bacteremia and support with high-
who are persistently febrile and stable. Monotherapy with flow nasal cannula and a vasopressor. His clinical state
an antipseudomonal beta-lactam, fourth-generation cepha- improved after 48 hours and he was transferred back to
losporin, or a carbapenem remains the gold standard the BMT unit a few days later to continue his therapy.

Further Reading
Freifeld, A.G., Bow, E.J., Sepkowitz, K.A., et al. (2011). Clinical children with cancer and hematopoietic stem cell
practice guideline for the use of antimicrobial agents in transplantation recipients: 2017 update. Journal of Clinical
neutropenic patients with cancer: 2010 update by the Oncology 35(18): 2082–2094.
Infectious Diseases Society of America. Clinical Infectious Prusakowski, M.K. and Chen, A.P. (2017). Pediatric sepsis.
Diseases 52(4): e56–e93. doi: 10.1093/cid/cir073. Emergency Medicine Clinics of North America 35(1):
Lehrnbecher, T., Robinson, P., Fisher, B., et al. (2017). 123–138.
Guideline for the management of fever and neutropenia in
Downloaded from https://onlinelibrary.wiley.com/doi/ by National Institute Of Standard, Wiley Online Library on [06/03/2024]. See the Terms and Conditions (https://onlinelibrary.wiley.com/terms-and-conditions) on Wiley Online Library for rules of use; OA articles are governed by the applicable Creative Commons License
23

A 5-Year-Old with Altered Mental Status


Yu-shan Tseng and Richard Garcia
Children’s Hospital of Michigan, Wayne State University, Detroit, MI, USA

A 5-year-old is evaluated in the emergency department Social History


(ED) for decreased activity and lethargy. The child was
found lying on the floor of the grandparents’ home with Parents are divorced. The patient frequently stays at the
emesis, and appeared to have altered mental status with grandparents’ house for care and attends school and
abnormal respiration. After initial resuscitation in the kindergarten.
ED, she was admitted to the pediatric intensive care unit
(PICU) with persistent hypotension and bradycardia
despite several fluid boluses and initiation of inotropic Current Status
support.
Patient arrived in the intensive care unit intubated and
sedated. She is bradycardic and hypotensive, with cool
History of Present Illness extremities and delayed capillary refill. A basic metabolic
panel indicates an elevated creatinine of 1.75 mg/dL and
This child was previously healthy, with no history of past hyperglycemia with glucose of 312 mg/dL. Arterial blood
medical conditions. As per grandparents she arrived to gas analysis indicates metabolic acidosis with a serum lac-
their home 2 days ago and was acting fine until they found tate of 11.5 mg/dL.
her altered on the floor today in the early afternoon. Initial laboratory results are shown in Table 7.1. The
In the ED, the patient was found to have heart rate of electrocardiogram (EKG) and chest X-ray are shown in
60 bpm, blood pressure 75/45 mmHg, and glucose 272 Figures 7.1 and 7.2, respectively. Chest X-ray showed mild
mg/dL. After becoming progressively obtunded, the child vascular congestion bilaterally with normal heart silhouette.
was intubated for airway protection. She received multiple
fluid boluses and calcium, and was started on inotropic sup-
port, which quickly escalated to three vasopressors (norep- Questions
inephrine, epinephrine, and phenylephrine) at high doses
for persistent hypotension. Answer the following questions using the details provided.
1 Based on the data provided, what is the most likely diag-
nosis for this child?
Past Medical History 2 Would you obtain any other diagnostic studies?
Continuation of case: The echocardiogram reveals nor-
Non-significant past medical history. mal four-chamber intracardiac anatomy with mildly
depressed function bilaterally. There is no pericardial effu-
sion. The patient is emergently placed on veno-arterial
Past Surgical History extracorporeal membrane oxygenation (ECMO) due to per-
sistent hemodynamic instability, and to prevent further
None. end-organ damage.

Cases in Pediatric Acute Care: Strengthening Clinical Decision Making, First Edition. Edited by Andrea M. Kline-Tilford and Catherine M. Haut.
© 2020 John Wiley & Sons Ltd. Published 2020 by John Wiley & Sons Ltd.
Downloaded from https://onlinelibrary.wiley.com/doi/ by National Institute Of Standard, Wiley Online Library on [06/03/2024]. See the Terms and Conditions (https://onlinelibrary.wiley.com/terms-and-conditions) on Wiley Online Library for rules of use; OA articles are governed by the applicable Creative Commons License
24 Cases in Pediatric Acute Care

Table 7.1 Basic metabolic panel (BMP) and arterial blood Would you obtain any other diagnostic studies?
gas (ABG) analysis.
In addition to the basic laboratory tests obtained in this
BMP case, one should also obtain serum and urine drug screen-
Sodium 147 mEq/L ing tests, since multiple drug ingestions are relatively com-
Potassium 3.2 mEq/L mon. Obtaining an EKG is a must as it gives valuable
Chloride 109 mEq/L information such as conduction abnormalities leading to
several degrees of atrioventricular block and sinus node
Bicarbonate 14 mEq/L
dysfunction. An echocardiogram should be obtained to
Total calcium 8.5 mg/dL
investigate for pericardial effusion and myocardial dysfunc-
Blood urea nitrogen 15 mg/dL
tion, and evaluate for congenital pathology. Furthermore, a
Creatinine 1.75 mg/dL chest X-ray can explore for pulmonary edema, a common
Glucose 312 mg/dL occurrence in CCB toxicity or help to demonstrate other
Ionized calcium 1.12 mmol/L lung pathology.
Lactate 11.5 mg/dL At the cellular level, CCBs almost exclusively block the
L-type voltage-gated calcium channel located on the sur-
ABG
face membrane. The primary consequences are depressed
inotropic and negative chronotropic effects, and peripheral
pH 7.28
vasodilation. The depressed inotropy and peripheral vaso-
PCO2 27 mmHg
dilation are mediated by interfering with phase 2 of the
PO2 162 mmHg action potential in myocytes and vascular smooth muscle,
Bicarbonate 12.6 mEq/L which involves the slow influx of extracellular calcium via
Base excess –12.5 the L-type calcium channels. Specifically blocking the
L-type calcium channels stops the release of calcium from
the intracellular sarcoplasmic reticulum, a process known
3 What other therapies would be recommended at as calcium-induced calcium release, which is essential for
this time? muscle contraction. Moreover, the L-type calcium channels
4 What are the signs and symptoms of calcium channel are responsible for impulse generation and conduction by
blocker toxicity? allowing the slow influx of calcium during phase 4 of the
5 Why does the patient have persistent hemodynamic action potential in pacemaker cells located in the sinoatrial
instability despite multiple inotropes and fluid boluses? and atrioventricular nodes, which creates a negative chron-
6 What other therapies would be recommended in a case otropic effect.
of calcium channel blocker poisoning? Is there benefit CCBs can be divided into two major categories. Dihy-
to the use of hemodialysis? dropyridines, such as nifedipine and amlodipine, prefer-
entially block vascular smooth muscle L-type calcium
channels, producing potent vasodilatory effects. Non-
dihydropyridines, such as verapamil and diltiazem,
Rationale and Evidence-based Practice
selectively inhibit L-type calcium channels in the myo-
Explanation cardium, therefore causing delayed conduction and
depressed myocardial contractility. As dihydropyridines
Based on the data provided, what is the most likely
generally produce peripheral vasodilation and have less
diagnosis for this child?
influence on cardiac chronotropy and inotropy, reflex
During the discussion when the child was admitted, it was sinus tachycardia can be seen during vasodilatory shock.
noted that the grandmother’s purse had been found close to However, this CCB selectivity can be lost in cases of
the child on the floor and her medication bottles were large overdose. Although calcium is also critical for skel-
opened. Grandmother is taking diltiazem and amlodipine, etal muscle contraction, CCBs primarily affect cardiac
both calcium channel blockers (CCBs). Given this history and vascular smooth muscle.
of missing pills from the grandmother’s purse and the In addition, insulin secretion by pancreatic beta cells is
patient’s presentation with bradycardia, hypotension, poor gated by L-type calcium channels on the cell membrane.
perfusion, metabolic acidosis, acute kidney injury, and By blocking these L-type channels, CCBs prevent the
hyperglycemia, the most likely diagnosis would be CCB- release of insulin, inducing insulin resistance that results
induced shock. in a hyperglycemic state.
Downloaded from https://onlinelibrary.wiley.com/doi/ by National Institute Of Standard, Wiley Online Library on [06/03/2024]. See the Terms and Conditions (https://onlinelibrary.wiley.com/terms-and-conditions) on Wiley Online Library for rules of use; OA articles are governed by the applicable Creative Commons License
Chapter 7 A 5-Year-Old with Altered Mental Status 25

Figure 7.1 12-lead EKG.

What other therapies would be recommended


at this time?
While on ECMO support, the patient was started on hyper-
insulinemic euglycemia (HIE) therapy and calcium drip.
Her inotropes were weaned significantly. Within the next
couple of days, acidosis resolved and urine output improved
with down-trending creatinine. Her epinephrine, calcium,
and high insulin infusions were weaned off and her EKG
normalized to sinus rhythm. She was decannulated suc-
cessfully on day four of admission and extubated soon after.

What are the signs and symptoms of calcium


channel blocker toxicity?
The severity of CCB ingestion varies greatly; most children
are asymptomatic after accidental exposure. However, case
reports of cardiac arrests and deaths after one to two tablets
have been documented in the literature. Significant CCB
Figure 7.2 Chest radiograph. toxicity manifests in cardiovascular instabilities including
Downloaded from https://onlinelibrary.wiley.com/doi/ by National Institute Of Standard, Wiley Online Library on [06/03/2024]. See the Terms and Conditions (https://onlinelibrary.wiley.com/terms-and-conditions) on Wiley Online Library for rules of use; OA articles are governed by the applicable Creative Commons License
26 Cases in Pediatric Acute Care

hypotension, bradycardia, and arrhythmias from conduc- preparations are involved, particularly if the drug is vera-
tion abnormalities. Patients may also present with symp- pamil or diltiazem. If the patient becomes symptomatic,
toms of end-organ hypoperfusion such as dizziness, urgent admission to an intensive care unit is warranted.
syncope, altered mental status, seizures, cerebrovascular There is no unique management algorithm in children
ischemia, acute kidney injury, shock liver, nausea and vom- with CCB-induced shock refractory to conventional medi-
iting, bowel ischemia, and metabolic acidosis. It is notewor- cal therapy. Timely consultation with the institution toxi-
thy to mention that both CCBs and beta-blockers present cologist or regional poison control center is prudent.
with the detrimental combination of bradycardia and hypo- Different treatments with possible benefits have been used
tension; however, the presence of hyperglycemia in CCB in CCB toxicity, including continuous intravenous calcium
intoxication helps to distinguish it from beta-blocker inges- infusion, intravenous HIE therapy, glucagon therapy, lipid
tion, which generally presents with hypoglycemia. emulsion therapy, cardiac pacing, and veno-arterial ECMO
Finally, acute pulmonary edema can occur following support. Frequently, these therapies are provided simulta-
CCB ingestion. Pulmonary edema can be secondary to neously or in rapid succession since patients can rapidly
heart failure, but non-cardiogenic pulmonary edema has deteriorate to cardiac arrest and death.
also been reported. The mechanism behind it is yet to be Intravenous calcium is used to counteract the negative
discovered; currently, it is thought to be secondary to selec- cardiovascular effects of CCBs. Calcium chloride is pre-
tive precapillary vasodilation and aggressive fluid ferred to calcium gluconate because of the higher potency.
resuscitation. Calcium chloride should be administered through central
venous access, if possible, to avoid calcium extravasation
Why does the patient have persistent and skin necrosis. Intravenous calcium boluses can be
hemodynamic instability despite multiple repeated every 10–20 minutes or followed by an infusion.
inotropes and fluid boluses? HIE therapy has shown promising results in case reports
and animal models. CCBs appear to disrupt myocardial
Because CCBs act at a cellular level, the cardiogenic and
fatty acid metabolism and switch it to carbohydrate metab-
vasodilatory shock seen in CCB toxicity are often refractory
olism, but at the same time CCBs create a relative insulin
to initial resuscitation with isotonic fluid boluses and intra-
resistance within the myocardium by preventing the
venous vasoactive medications. It is preferred to limit fluid
release of insulin from the pancreatic beta cells. Insulin
boluses to 60 mL/kg as patients are usually euvolemic in
at high doses (1–10 units/kg per hour) has positive ino-
CCB ingestion, and aggressive fluid resuscitation carries
tropic properties and assists in myocardial uptake of
the risk of acute pulmonary edema. Also, bradycardia tends
glucose, mitigating the myocardial depression seen in
to be refractory to atropine administration because ulti-
patients with CCB toxicity. Electrolytes including potas-
mately the effects of atropine are mediated at the cellular
sium and glucose should be measured frequently, at least
level by the L-type calcium channels.
every 15–30 minutes initially until the levels are stable.
Titrate dextrose infusions and give potassium supplementa-
What other therapies would be recommended in a
tions as necessary. As CCB overdose often results in
case of calcium channel blocker poisoning? Is
hyperglycemia, patients with hyperglycemia refractory to
there benefit to the use of hemodialysis?
high-dose insulin may not require supplemental dextrose
Asymptomatic patients need to be monitored using telem- infusion.
etry; patients who ingest immediate-release formulations Glucagon activates adenylate cyclase and increases the
should be observed for at least 6–8 hours whereas those production of cyclic AMP, the intracellular second messen-
who ingest sustained-release formulations should be mon- ger downstream of β-adrenergic receptors, and hence has
itored for at least 24 hours. Timely gastrointestinal decon- inotropic, chronotropic, and dromotropic effects. However,
tamination with activated charcoal is recommended in all it offers little advantage beyond the use of common vasopres-
patients with CCB ingestion with active bowel sounds, sors. Lipid emulsion therapy has been used to treat poison-
since activated charcoal adheres well to CCBs and ings involving lipophilic medications in recent years, and
decreases their absorption. Given a secure airway, activated preliminary data have demonstrated some benefits in CCB
charcoal should be administered within 1–2 hours of inges- intoxication. Although the mechanism behind lipid emul-
tion of a short-acting CCB. In cases when an extended- sion therapy remains unclear, it is theorized that lipid emul-
release drug is the culprit, it can be done up to 4–6 hours sion acts as a “lipid sink” by sequestering the lipophilic
after a documented ingestion. Furthermore, whole bowel drugs, reducing the amount of available free drug in the
irrigation with polyethylene glycol can be considered in intravascular compartment. Another proposed mechanism
patients with clear mentation when extended-release is that by increasing fatty acid availability, lipid emulsion
Downloaded from https://onlinelibrary.wiley.com/doi/ by National Institute Of Standard, Wiley Online Library on [06/03/2024]. See the Terms and Conditions (https://onlinelibrary.wiley.com/terms-and-conditions) on Wiley Online Library for rules of use; OA articles are governed by the applicable Creative Commons License
Chapter 7 A 5-Year-Old with Altered Mental Status 27

may reverse the CCB-induced lipid to glucose metabolism in dialysis is typically ineffective in removing a significant
the myocardium. Temporary transesophageal or intrave- amount of the drug.
nous pacing can be attempted to counter bradycardia. How-
ever, it is difficult to propagate the depolarization current
through the blocked L-type calcium channels by activating Case Resolution
the myocardial action potential artificially.
Healthcare providers should be aware of the high mor- While on ECMO support, she was started on hyperinsulinemia-
bidity and mortality of CCB toxicity. Veno-arterial ECMO euglycemia (HIE) therapy and calcium drip. Her ino-
should be initiated immediately if a patient presents with tropes were weaned significantly. Within the next couple
shock refractory to pharmacotherapy. Veno-arterial ECMO days, acidosis resolved and urine output improved with
placement should be performed before end-organ dysfunc- down trending creatinine. Her epinephrine, calcium,
tion develops and generally support is only needed for a few and high insulin infusions were weaned off and her elec-
days while the drug is metabolized and excreted. trocardiogram normalized to sinus rhythm. She was
Most calcium channel blockers are highly protein decannulated successfully on day four of admission and
bound and have a large volume of distribution; therefore extubated soon after.

Further Reading
Arroyo, A.M. and Kao, L.W. (2009). Calcium channel blocker amlodipine ingestion. Critical Care Nurse 36(4): 64–69. doi:
toxicity. Pediatric Emergency Care 25(8): 532–538; quiz 10.4037/ccn2016524.
539–540. doi: 10.1097/PEC.0b013e3181b0a504. St-Onge, M., Anseeuw, K., Cantrell, F.L., et al. (2017).
Krentz, J.R. and Kaakeh, Y. (2018). An overview of Experts consensus recommendations for the management
hyperinsulinemic–euglycemic therapy in calcium of calcium channel blocker poisoning in adults. Critical Care
channel and beta blocker overdose. Pharmacotherapy Medicine 45(3): e306–e315. doi: 10.1097/
38(11): 1130–1142. doi: 10.1002/phar.2177. CCM.0000000000002087.
Reuter-Rice, K.E. and Peterson, B. (2016). Conventional and
unconventional lifesaving therapies in an adolescent with
Downloaded from https://onlinelibrary.wiley.com/doi/ by National Institute Of Standard, Wiley Online Library on [06/03/2024]. See the Terms and Conditions (https://onlinelibrary.wiley.com/terms-and-conditions) on Wiley Online Library for rules of use; OA articles are governed by the applicable Creative Commons License
29

An 8-Year-Old with Abdominal Trauma


Julie Resler
Riley Hospital for Children, Indianapolis, IN, USA

An 8-year-old girl is admitted to the pediatric intensive care Current Status


unit (PICU) on the trauma service for a grade IV liver
laceration. On arrival in the emergency department (ED), the primary
survey reveals an intact airway, breath sounds clear and equal
bilaterally, strong rapid pulse in arms and legs, Glasgow
Coma Scale (GCS) score 15, and body warm with clothing
History of Present Illness in place. She is in a cervical collar and on a spine board with
total spine precautions in place. Initial survey reveals tem-
The child was an improperly restrained (lap belt only) perature of 37.2 C (99 F), heart rate 144 bpm, respiratory rate
front seat passenger involved in a head-on motor vehicle 22/minute, oxygen saturation 98% on room air, and blood
collision at 55 m.p.h. There was no known loss of con- pressure 76/40 mmHg. Peripheral intravenous access is
sciousness and she was able to extricate herself from the placed and complete blood count (CBC), comprehensive met-
vehicle. She had a noted seat belt abrasion and abdominal abolic profile, amylase, and lipase are obtained with values as
pain with emesis at the scene. noted in Tables 8.1 and 8.2. A 20 mL/kg fluid bolus of normal
saline is given with little response in blood pressure. A second
20 mL/kg fluid bolus of normal saline is given with similar
Past Medical History response. CBC reveals a hemoglobin of 7 g/dL and hematocrit
of 21%. A transfusion of 10 mL/kg of packed red blood cells
The patient was born at term and discharged home with the is given to support blood pressure, which increases to 96/
mother. She is in the 75th percentile for height and weight. 50 mmHg. Secondary survey reveals pain to palpation in right
She is up to date on vaccinations and did not receive the upper quadrant of the abdomen and pain with inspiration.
seasonal flu vaccine. Home medications include acetamin- Chest radiograph indicates right 10th, 11th, and 12th rib
ophen as needed for pain and fever. fractures. Radiographs of the pelvis are negative. Computed
tomography (CT) of the abdomen and pelvis with intrave-
nous contrast is obtained and notes a grade IV liver laceration.
CT imaging is shown in Figure 8.1. The child is admitted to
Past Surgical History the PICU on the trauma service. She is on bedrest and is
nil by mouth with intravenous fluids running at a mainte-
No previous surgical procedures. nance rate. Serial hemoglobin and hematocrits are ordered.

Family History Questions

Answer the following questions using the details provided.


The patient’s parents, mother and father are both healthy.
Father is currently incarcerated. The child has a 5-year-old 1 Why are young children at greater risk for serious liver,
and 2-year-old siblings, both reported healthy. spleen or kidney injury from blunt abdominal trauma?

Cases in Pediatric Acute Care: Strengthening Clinical Decision Making, First Edition. Edited by Andrea M. Kline-Tilford and Catherine M. Haut.
© 2020 John Wiley & Sons Ltd. Published 2020 by John Wiley & Sons Ltd.
Downloaded from https://onlinelibrary.wiley.com/doi/ by National Institute Of Standard, Wiley Online Library on [06/03/2024]. See the Terms and Conditions (https://onlinelibrary.wiley.com/terms-and-conditions) on Wiley Online Library for rules of use; OA articles are governed by the applicable Creative Commons License
30 Cases in Pediatric Acute Care

Table 8.1 Complete blood count.

White blood cell count 10 300/mm3


Hemoglobin 7 g/dL
Hematocrit 21.1%
Platelet count 350 000/mm3
Neutrophils 41%
Lymphocytes 40%
Monocytes 0.5%
Eosinophils 4%

Table 8.2 Complete metabolic panel, including amylase


and lipase.

Sodium 138 mEq/L


Potassium 3.9 mEq/L
Chloride 108 mEq/L
Carbon dioxide 14 mEq/L
Figure 8.1 Computed tomography of abdomen/pelvis.
Blood urea nitrogen 11 mg/dL
Creatinine 0.4 mg/dL
Glucose 108 mg/dL larger-size organs, less overlying fat, and weaker abdominal
muscles. The liver has a dual blood supply via the hepatic
Calcium 9.5 mg/dL
arteries and portal vein, which adds to the possibility of
Albumin 3.0 mg/dL
rapid blood loss if these vessels are damaged along with
Total protein 6.0 g/dL damage to the parenchyma of the liver.
Aspartate aminotransferase 480 U/L
Alanine aminotransferase 385 U/L
Alkaline phosphatase 133 U/L What is the best treatment plan for this child?
Total bilirubin 0.5 mg/dL Many blunt abdominal organ injuries can be treated non-
Amylase 38 U/L operatively, including involvement of the liver. Pediatric
Lipase 35 U/L facilities have protocols for length of bedrest, frequency
of lab draws, and length of activity restriction. This allows
for stabilization of the laceration/hematoma and reabsorp-
2 What is the best treatment plan for this child? tion of the blood. However, the newest guidelines, pub-
3 What signs would prompt escalation of interventions lished by the Pediatric Trauma Society, utilize ATOMAC
for this child? (Arizona–Texas–Oklahoma–Memphis–Arkansas Consor-
4 What additional imaging would be helpful to diagnose tium), which consists of a group of Level I pediatric trauma
complications? centers across the United States who have participated in
research to improve outcomes for children who suffer
trauma injuries. This publication from 2015 offers a prac-
Rationale and Evidence-based Practice
tice guideline with four highlights. Guidelines include
Explanation managing liver and spleen trauma based on hemodynamic
stability versus grade of injury, a period of as little as
Why are young children at greater risk for serious
24 hours bedrest, and limiting transfusions to those who
liver, spleen or kidney injury from blunt abdominal
have hemoglobin levels of less than 7 g/dL. Management
trauma?
includes urgent surgery or embolization or continued
Liver, spleen, and kidney injuries resulting from blunt non-surgical intervention. In 2015, PECARN (Pediatric
trauma occur more frequently in children than adults pri- Emergency Care Applied Research Network) published
marily because of the physical features of the child, which data in regard to management of liver trauma and found
include a protuberant abdomen, more pliable ribcage and that only 5% of liver injuries required surgical intervention
Downloaded from https://onlinelibrary.wiley.com/doi/ by National Institute Of Standard, Wiley Online Library on [06/03/2024]. See the Terms and Conditions (https://onlinelibrary.wiley.com/terms-and-conditions) on Wiley Online Library for rules of use; OA articles are governed by the applicable Creative Commons License
Chapter 8 An 8-Year-Old with Abdominal Trauma 31

or embolization and 11% required blood transfusion, leav- change in abdominal examination, and decrease in appetite
ing the majority of patients on minimal therapies. There are all signs warranting further investigation.
continues to be some debate about overall hospitalization
time, length of bedrest, and recurring blood work, despite What additional imaging would be helpful
published guidelines. For this patient, serial abdominal to diagnose complications?
examinations and serial checks of hemoglobin and hemat-
Many pediatric trauma centers attempt to limit radiation in
ocrit until stabilized will help guide treatment. A diet can be
pediatric patients due to the lifetime increase in cancer risk
started and advanced as tolerated. Follow-up with the
with ongoing exposure. However, if there is suspicion for
trauma surgeon may be necessary if the child is or becomes
development of a bile leak, repeat CT imaging may be nec-
hemodynamically unstable.
essary for diagnosis. Endoscopic retrograde cholangiopan-
creatography (ERCP) is a diagnostic tool that may not be
available at all treatment centers. It allows the practitioner
What signs would prompt escalation of
to directly identify duct transection/injury and stent or per-
interventions for this child?
form a sphincterotomy. Last, fluoroscopy and interven-
Serial abdominal examinations will provide insight into tional radiology facilitates drain placement in order to
subtle changes in the patient’s condition. Location of the divert fluid and pressure away from the site of injury.
liver laceration may also raise suspicion for the possibility The patient may require follow-up imaging to assess for res-
of developing a bile leak. Fever, tachycardia, tachypnea, olution of the injury.

Further Reading
Linnaus, M.E., Langlais, C.S., Garcia, N.M., et al. (2017). GRADE. Journal of Trauma and Acute Care Surgery 79(4):
Failure of nonoperative management of pediatric blunt liver 683–693.
and spleen injuries. Journal of Trauma and Acute Care Wisner, D.H., Kuppermann, N., Cooper, A., et al. (2015).
Surgery 82(4): 672–679. Management of children with solid organ injuries after
Notrica, D.M., Eubanks, J.W., Tuggle, D.W. et al. (2015). blunt torso trauma. Journal of Trauma and Acute Care
Nonoperative management of blunt liver and spleen injury Surgery 79(2): 206–214.
in children: evaluation of the ATOMAC guideline using
Downloaded from https://onlinelibrary.wiley.com/doi/ by National Institute Of Standard, Wiley Online Library on [06/03/2024]. See the Terms and Conditions (https://onlinelibrary.wiley.com/terms-and-conditions) on Wiley Online Library for rules of use; OA articles are governed by the applicable Creative Commons License
33

Child with Left Eye Swelling


Amy Graf
Norton Children’s Hospital, Louisville, KY, USA

A 3-year-old female presents to the emergency department Current Status


(ED) for evaluation of left eye swelling.
On arrival at the ED, she is febrile to 38.9 C (102.1 F), heart
rate is 115 bpm, and oxygen saturation 94% on 1.5 L/min of
History of Present Illness oxygen per nasal cannula. She appears relatively well,
except for rhinorrhea and guarding her left eye. She has
A 3-year-old female developed symptoms of rhinosinusitis periorbital edema and is unable to open her eye
4 days ago. On day 3 of illness, she developed left upper eye- (Figure 9.1). On examination, her pupils are equal round
lid erythema, edema, eye pain, and diplopia that has and reactive to light and she has conjunctival chemosis,
become progressively worse over the past 24 hours. but it is not possible to assess extraocular movements due
to lack of cooperation.

Past Medical History


Questions
She has trisomy 21 and was born at 37 weeks’ gestation. She
spent 6 weeks in the neonatal intensive care unit (NICU). Answer the following questions using the details provided.
She has pulmonary hypertension, requiring 1.5 L/min of 1 What are the differential diagnoses and the most likely
oxygen continuously per nasal cannula. She also has a diagnosis for this child?
history of gastroesophageal reflux, hypothyroidism, and 2 What diagnostic tests are recommended to assist in the
hypotonia. Her immunizations are up to date for age, diagnosis of this eye condition?
including influenza vaccine. 3 What specialists may be consulted to manage this left
eye condition?
4 What are the most common etiologies of eye swelling?
Past Surgical History 5 What is the most appropriate treatment for this
condition?
She has had a G-tube placement, Nissen fundoplication,
closure of patent ductus arteriosus and patent foramen
ovale, multiple cardiac catheterizations, and tonsillectomy
Rationale and Evidence-based Practice
and adenoidectomy for obstructive sleep apnea.
Explanation

What are the differential diagnoses and the most


Family History likely diagnosis for this child?
Her mother has hypothyroidism and her father has a Possible diagnoses for this condition are broad and
history of hypertension. Her older brother was diagnosed include preseptal cellulitis, orbital cellulitis, allergic res-
with reactive airway disease at age 4 years. ponse, and tumors of the eye including neuroblastoma,

Cases in Pediatric Acute Care: Strengthening Clinical Decision Making, First Edition. Edited by Andrea M. Kline-Tilford and Catherine M. Haut.
© 2020 John Wiley & Sons Ltd. Published 2020 by John Wiley & Sons Ltd.
Downloaded from https://onlinelibrary.wiley.com/doi/ by National Institute Of Standard, Wiley Online Library on [06/03/2024]. See the Terms and Conditions (https://onlinelibrary.wiley.com/terms-and-conditions) on Wiley Online Library for rules of use; OA articles are governed by the applicable Creative Commons License
34 Cases in Pediatric Acute Care

Orbital cellulitis is an infection involving the orbital con-


tents including the ocular muscles and fat. It is an infection
that is more frequently seen in young children rather than
older children and adults. Rhinosinusitis is commonly the
source of orbital cellulitis as 86–98% of cases have a coexist-
ing rhinosinusitis. The most common types of sinusitis that
lead to an orbital cellulitis are pansinusitis and ethmoid
sinusitis.

What diagnostic tests are recommended to assist


in the diagnosis of this eye condition?
Computed tomography (CT) of the orbits with intravenous
contrast and magnetic resonance imaging (MRI) with intra-
venous contrast are beneficial studies in diagnosing orbital
cellulitis. However, CT scan of the orbits with intravenous
contrast is frequently the initial diagnostic study of choice
due to availability of the test and duration of the study. MRI
is superior to CT of the orbits when evaluating progression
of soft tissue disease. The Infectious Disease Society of
America recommends laboratory studies including a com-
plete blood count (CBC) with differential, blood cultures
and a C-reactive protein (CRP) in patients with soft tissue
Figure 9.1 Periorbital edema. Source: Courtesy of Robin
Scarbrough. infections who have signs and symptoms of systemic
toxicity (see Tables 9.1 and 9.2).
CT of the orbits with intravenous contrast shows left peri-
orbital and orbital cellulitis with small early abscess medi-
retinoblastoma, rhabdomyosarcoma and orbital pseudotu-
ally and myositis of medial rectus muscle. The CT also notes
mor. Additionally, conjunctivitis, cavernous sinus throm-
sinus disease in the left ethmoid and left maxillary sinuses,
bosis, granulomatosis with polyangiitis of the orbit
as well as left otomastoid disease (Figure 9.2).
(Wegner disease), Graves disease, viral infections including
herpes simplex or varicella zoster, periocular dermoid cyst,
posterior scleritis, sinus mucocele with superimposed infec-
What specialists may be consulted to manage the
tion, thrombosed orbital varix, and tuberculosis of the orbit
eye condition?
can all be included on the list of differential diagnoses.
Both a preseptal cellulitis and orbital cellulitis present A pediatric ophthalmologist is consulted to evaluate visual
with similar examination findings, including eyelid edema, acuity and extraocular eye movements. An otolaryngologist
erythema, and ocular pain. However, orbital cellulitis must is consulted if surgery is necessary to drain the abscess.
be differentiated from preseptal cellulitis as orbital cellulitis
may result in catastrophic outcomes. Orbital cellulitis may
cause abscesses of the brain, orbit or subperiosteal area, Table 9.1 Complete blood count.
vision loss or even death. Clinical manifestations of orbital
White blood cell count 9290/mm3
cellulitis include fever, leukocytosis with a neutrophil
Hemoglobin 11.2 g/dL
predominance, pain with eye movements, and proptosis.
Chemosis and opthalmoplegia with diplopia are more com- Hematocrit 32.6%
monly seen in orbital cellulitis than in preseptal cellulitis. Platelet count 202 000/mm3
The primary distinction is that preseptal cellulitis causes Neutrophils 75%
inflammation only in the eyelids, whereas orbital cellulitis Eosinophils 2%
causes extraocular muscle inflammation, fat stranding, Monocytes 4%
and anterior displacement of the globe. The leading diagno- Lymphocytes 18.3%
sis for this patient is orbital cellulitis, given the examination
Bands 0%
findings of erythema, edema, pain, and chemosis.
Downloaded from https://onlinelibrary.wiley.com/doi/ by National Institute Of Standard, Wiley Online Library on [06/03/2024]. See the Terms and Conditions (https://onlinelibrary.wiley.com/terms-and-conditions) on Wiley Online Library for rules of use; OA articles are governed by the applicable Creative Commons License
Chapter 9 Child with Left Eye Swelling 35

Table 9.2 Additional results.

Erythrocyte sedimentation rate (ESR) 49 mm/hour


C-reactive protein (CRP) 3.3 mg/dL
Blood culture Pending

What are the most common etiologies of eye


swelling?
Staphylococcus aureus and streptococci are the most com-
monly identified organisms in bacterial orbital cellulitis.
Haemophilus influenzae was previously a common patho-
gen but incidence has declined due to immunization with
the Hib vaccine. Orbital cellulitis may also be polymicrobial
and, rarely, fungal infections such as Mucorales and Asper-
gillus may cause orbital cellulitis in immunocompro-
mised hosts.

What is the most appropriate treatment for this


condition? Figure 9.2 Orbital CT scan. Source: Courtesy of Dr. Karen Moeller.

Antibiotics are necessary. Initial therapy often includes


vancomycin and a second- or third-generation cephalospo- recommended dose of antibiotics is warranted. If positive
rin or piperacillin-tazobactam. If intracranial extension of cultures are obtained, antibiotics should be tailored to
the orbital cellulitis is a concern, then use of the highest identified organism.

Further Reading
Bedwell, J. and Bauman, N. (2011). Management of pediatric Upile, N., Munir, N., Leong, S., and Swift, A. (2012). Who
orbital cellulitis and abscess. Current Opinion in should manage acute periorbital cellulitis in children?
Otolaryngology and Head and Neck Surgery 19(6): 467–473. International Journal of Pediatric Otorhinolaryngology 78(8):
Infectious Disease Society of America (2014). Practice 1073–1077.
guidelines for the diagnosis and management of skin and Wong, S. and Levi, J. (2018). Management of pediatric orbital
soft tissue infections: 2014 update by the Infectious Disease cellulitis: a systematic review. International Journal of
Society of America. Clinical Infectious Disease 59(2): Pediatric Otorhinolaryngology 110: 123–129.
e10–e52.
Downloaded from https://onlinelibrary.wiley.com/doi/ by National Institute Of Standard, Wiley Online Library on [06/03/2024]. See the Terms and Conditions (https://onlinelibrary.wiley.com/terms-and-conditions) on Wiley Online Library for rules of use; OA articles are governed by the applicable Creative Commons License
37

10

Billing and Coding


Dawn M. Silverman
University of Maryland Children’s Hospital, University of Maryland Medical Center, Baltimore, MD, USA

Billing for healthcare services is an imperative part of in the state where the delivery of care is performed
learning the provider role in any setting and it is important (Wound, Ostomy, and Continence Nurses Society, 2012).
to have proficiency in billing practices to ensure reimburse- Additionally, services must be physician services, medically
ment. Key information related to reimbursement, billing, necessary, and within the scope of practice of the PNP
and coding for pediatric outpatient and inpatient services (Munro, 2013).
are discussed in this chapter to provide an overview of The CMS also issues National Provider Identifier
professional billing practices for the acute care provider. (NPI) numbers to providers of all disciplines. This unique
In addition, critical care criteria are defined with a case 10-digit number identifies the professional as a healthcare
presentation to provide an example of critical care billing. provider. It is often used when prescribing medications
There are many providers who are able to bill for services, and always used for billing medical services. Some group
but this chapter focuses on the the acute care nurse practices also hold an NPI number. Individual contracts
practitioner. will determine whether the PNP’s services are billed under
an individual or group NPI number (Wound, Ostomy, and
Continence Nurses Society, 2012). PNPs can apply for
Questions an NPI number online at https://nppes.cms.hhs.gov/#/.
Once a PNP is licensed, board certified, and has obtained
Please answer the following questions. an NPI, the organization for whom the PNP is employed
1 As a new provider for the inpatient cardiology service, will complete credentialing and privileging. Documents
what steps are required to be a billable provider? demonstrating evidence of a diploma, state license, board
2 Once a pediatric nurse practitioner (PNP) is creden- certification, NPI, Drug Enforcement Administration
tialed, how does a PNP’s work generate a charge? license, and state-issued controlled dangerous substance
3 What documentation elements are required for billing? license must be shared with the credentialing body. Curric-
4 What codes are used to bill for services? ulum vitae and professional references are also required.
Privileges are established and granted to the PNP based
on role descriptions and state-based scope of practice
delineations.
Rationale and Evidence-based
Providers may also be required to enroll with third-party
Practice Explanation payers. The healthcare organization and associated billing
entity will facilitate this process. It is important to note
As a new provider for the inpatient cardiology
that while PNPs are typically able to claim 85–100% of a
service, what steps are required to be a
physician’s usual and customary charge, PNPs are not
billable provider?
recognized as credentialed providers by all insurers. The
The Centers for Medicare and Medicaid Services (CMS) recognition of a PNP as a credentialed provider is based
requires a PNP to be a graduate of an advanced practice on individual states’ practice authorities and agreements
nursing program, certified by a recognized national certify- with insurers. Current federal laws and regulations do not
ing body, and licensed as a registered professional nurse require private insurers to credential or reimburse PNPs.

Cases in Pediatric Acute Care: Strengthening Clinical Decision Making, First Edition. Edited by Andrea M. Kline-Tilford and Catherine M. Haut.
© 2020 John Wiley & Sons Ltd. Published 2020 by John Wiley & Sons Ltd.
Downloaded from https://onlinelibrary.wiley.com/doi/ by National Institute Of Standard, Wiley Online Library on [06/03/2024]. See the Terms and Conditions (https://onlinelibrary.wiley.com/terms-and-conditions) on Wiley Online Library for rules of use; OA articles are governed by the applicable Creative Commons License
38 Cases in Pediatric Acute Care

Once the PNP is credentialed, how does a PNP’s providers can deliver immediate and appropriate treatment
work generate a charge? and track a patient’s health over time (https://www.cms.
gov/). Of course, it is also important to reimbursement.
Once the PNP is ready to practice and is credentialed and
granted hospital privileges, it is helpful to understand the
revenue cycle by which services are billed. The cycle is What codes are used to bill for services?
initiated by services rendered. As previously mentioned,
these services must be services that a physician would typ- Evaluation and management (E/M) documentation is the
ically provide and within the scope of practice of the PNP. foundation of medical documentation. It is employed in
The PNP will engage in an encounter with a patient, pro- the ambulatory and inpatient setting and is appropriate
vide services, and then complete documentation. In most for preventive care, primary care, and critical care. The
cases, documentation is entered into an electronic medical CPT codes assigned to documentation are based on the
record (EMR) or health record. Standardized note tem- patient type (new or established) or encounter type (initial
plates assist the provider with the flow and content required or subsequent), the setting of service (outpatient, inpatient,
for documentation and billing. emergency department, nursing facility, etc.), and level of
Some healthcare organizations require providers to complexity. The complexity is based on the depth and detail
assign codes to their documentation; however, in many of history taking, the physical examination, and medical
cases, professional medical coders review documentation decision-making (https://www.cms.gov/). While no dis-
and assign appropriate current procedural terminology tinct number of minutes is associated with each level of
(CPT) codes and diagnosis codes. The US Department of E/M, there should be a correlation between the level of
Health and Human Services designates CPT codes, which E/M and the time spent with the patient.
are the most widely accepted medical nomenclature for Seven elements are used to define the level of E/M
procedures and services. The World Health Organization services:

••
compiles the International Statistical Classification of Dis-
history
eases and Related Health Problems (ICD) codes. The ICD
physical examination

••
codes are assigned based on the patient’s diagnoses. To
medical decision-making
ensure accurate and appropriate coding of services and
counseling

••
diagnoses, coders may send queries to providers with ques-
coordination of care
tions or requests for more detail or additional information
nature of presenting problem


necessary to establish a claim.
time associated with delivery.
Professional medical billers assign fees based on the
documented codes and agreements with insurers. Billers History, physical examination, and medical decision-
enter the codes into billing software and report claims to making are the most important elements when designating
insurers, who in turn generate an explanation of benefits. the level of E/M (Table 10.1) (https://www.cms.gov/).
The claim is either paid or denied. Denials can be resub- Four types of history are utilized for E/M documentation.
mitted with explanation; however, there are fees associated These histories are problem focused, expanded, detailed,
with denial challenges so it is important for providers to and comprehensive. Table 10.2 shows the required compo-
supply as much detail and evidence of medical necessity nents for each type of history.
in the documentation.

What documentation elements are required


for billing? Outpatient Case Documentation
The adage “If it is not documented, it didn’t happen” is par- Example
ticularly relevant when it comes to billing. Documentation
must not only identify the patient, the setting, and the pro- A 10-year-old male presents to the clinic with complaints of
vider, it must also contain sufficient detail in the assess- runny nose, itchy eyes, and sneezing. Example of level 3
ment, diagnosis, and plan of care to meet standards for E/M documentation (CPT 99203) for this patient follows.
billing and medical necessity. Concise and detailed docu-
mentation leads to better patient care and outcomes. Doc-
Patient
umentation in an EMR is a communication tool. It provides
the author and subsequent readers with a chronological James Doe, date of birth 1 January 2010, medical record
record of the health and disease of a patient so that number (MRN) xxxxx0123.
Downloaded from https://onlinelibrary.wiley.com/doi/ by National Institute Of Standard, Wiley Online Library on [06/03/2024]. See the Terms and Conditions (https://onlinelibrary.wiley.com/terms-and-conditions) on Wiley Online Library for rules of use; OA articles are governed by the applicable Creative Commons License
Chapter 10 Billing and Coding 39

Table 10.1 Level of coding by patient type and complexity.

Service CPT History and physical Medical decision-making Average time in minutes

New patient: outpatient 99201 Problem focused Straightforward 10


99202 Expanded Straightforward 20
99203 Detailed Low complexity 30
99204 Comprehensive Moderate complexity 45
99205 Comprehensive High complexity 60
Established patient: outpatient 99211 — — 5
99212 Problem focused Straightforward 10
99213 Expanded Low complexity 15
99214 Detailed Moderate complexity 25
99215 Comprehensive High complexity 40
Inpatient 99221 Detailed Low complexity 30
99222 Comprehensive Moderate complexity 50
99223 Comprehensive High complexity 70
Source: Data from https://www.cms.gov, 2017.

Table 10.2 History component documentation requirements for coding.

Type of history Chief complaint History of present illness Review of systems Past, family and social history

Problem focused Required Brief — —


Expanded problem focused Required Brief Problem pertinent —
Detailed Required Extended Extended Pertinent
Comprehensive Required Extended Extended Complete
Source: Data from https://www.cms.gov, 2017.

Chief complaint Past medical history


Runny nose, itchy eyes, and sneezing. Distal radius fracture in 2018 requiring casting (associated
with skateboarding injury), frequent otitis media as an
infant and toddler.
History of present illness
The patient was brought to clinic today by his mother who Past surgical history
reports that James has been suffering from a “runny nose,
Myringotomy and tube placement in 2012.
itchy eyes, and sneezing” for 3 weeks. James’s symptoms
are worse when he is outside. His symptoms are relieved
with cold compresses to his eyes and after his mother Family history
administers diphenhydramine. However, the mother is
Father has a history of seasonal allergies and takes an
concerned that James is sleepy from the medication and
antihistamine during the spring and summer.
his symptoms distract him at school. James reports, “I hate
my allergies. I can’t stop itching my face.”
Review of systems
Ears: negative for decreased hearing or pain
Medications
Nose: negative for epistaxis or pain
Multivitamin one tablet by mouth daily, Benadryl 25 mg by Mouth and throat: negative for pain, but complains of
mouth as needed for allergy symptoms. “itchiness in back of throat”
Downloaded from https://onlinelibrary.wiley.com/doi/ by National Institute Of Standard, Wiley Online Library on [06/03/2024]. See the Terms and Conditions (https://onlinelibrary.wiley.com/terms-and-conditions) on Wiley Online Library for rules of use; OA articles are governed by the applicable Creative Commons License
40 Cases in Pediatric Acute Care

Eyes: negative for visual changes, but complains of “itchi- 2 Is critical care management different from E/M docu-
ness and tears” mentation and billing?
Pulmonary: negative for cough, hemoptysis, and shortness 3 Are critical care management codes different for pedi-
of breath atric patients?
4 A critically ill 5-year-old is transferred to a larger hospi-
Physical examination tal for more advanced services. What critical care code
does the receiving PNP report and what code should the
General: well developed, no acute distress, average height
transferring PNP report?
and weight for age
5 Do time-based critical care management codes have dif-
Vital signs: blood pressure 110/65 mmHg, heart rate
ferent documentation requirements than pediatric crit-
85 bpm, respiratory rate 14/min, SpO2 99% on room air
ical care management codes?
Head: atraumatic, no sinus tenderness
6 Which services are bundled into pediatric critical
Neck: supple, trachea midline, no lymphadenopathy
care codes?
Eyes: mildly injected superficial blood vessels in medial
7 What elements need to be documented for pediatric
sclerae bilaterally, no discharge or crusting
critical care billing?
Nose: mildly injected nasal mucosa, unremarkable septum
8 What services do not merit critical care manage-
and turbinates
ment codes?
Mouth, throat: normal palate, no oral or pharyngeal ery-
thema, tonsils present and unremarkable
Ears: pearly tympanic membrane bilaterally, normal
hearing
Lungs: clear to auscultation, no cough or discolored Rationale and Evidence-based
sputum Practice Explanation
Cardiovascular: S1S2, normal rate and rhythm, no murmurs
or rubs What is the difference between ambulatory and
Extremities: well developed, palpable pulses, no edema inpatient billing encounters and which of the PNP
services are billable?
Assessment
In addition to the level of E/M, another important consid-
Allergic rhinitis in an otherwise healthy patient.
eration in documentation and billing is collaboration. PNPs
often work on multidisciplinary teams. Individual state leg-
Plan islators establish PNP practice authority. However, even in
1) Over-the-counter non-drowsy antihistamine daily full practice authority states, PNPs often collaborate with
2) Saline nasal spray b.i.d. physician partners. If both a PNP and a collaborating
3) Antihistamine eye drops for relief of allergic conjuncti- physician see a patient (face-to-face encounter) and
vitis b.i.d. contribute to the evaluation and management, then the
4) Follow-up in 4 weeks, call if no symptom relief encounter can be split or shared. Shared E/M encounters
within 1 week can be reimbursed at 100% of the physician fee schedule
5) Referral to allergy specialist if over-the-counter reme- if both the PNP and the physician see the patient on the
dies do not alleviate symptoms same calendar date. The PNP and physician may see the
patient jointly or independently, both providers must
document, and documentation should be linked in some
Total time fashion. Finally, documentation must support the level of
31 minutes service (Munro, 2013).
Another type of collaborative encounter is the incident to
service. “Incident to” encounters can also be reimbursed at
Questions 100% of the physician fee schedule. However, “incident to”
encounters are exclusive to the ambulatory setting and the
Answer the following questions using the details provided. following criteria must be met.

1 What is the difference between ambulatory and inpa-


• PNP sees the patient under the physician’s direct
supervision.


tient billing encounters and which of the PNP services
are billable? The physician must be immediately available.
Downloaded from https://onlinelibrary.wiley.com/doi/ by National Institute Of Standard, Wiley Online Library on [06/03/2024]. See the Terms and Conditions (https://onlinelibrary.wiley.com/terms-and-conditions) on Wiley Online Library for rules of use; OA articles are governed by the applicable Creative Commons License
Chapter 10 Billing and Coding 41

• The physician must perform the initial visit or


consultation.
A critically ill 5-year-old is transferred to a larger
hospital for more advanced services. What critical

• The physician is not required to see the patient at


every visit.
care code does the receiving PNP report and what
code should the transferring PNP report?

• The physician must contribute to the plan of care. Pediatric critical codes include initial inpatient encounter
codes and subsequent day codes. Only one initial inpatient
critical care encounter code can be reported per patient per
Is critical care management different from E/M
hospital stay. Every additional critical care code reported
documentation and billing?
during of the hospital stay should be a subsequent day crit-
Critical care management is the highest level of E/M. The ical care code. If a patient were to leave the intensive care
rules regarding the seven key elements of E/M do not apply unit (ICU) and later decompensate and return to the ICU, a
to critical care management documentation. Critical care subsequent day code would again be reported.
management documentation requires evidence of “a criti- The child’s age also determines the critical care code
cal illness or injury acutely impairing one or more vital reported (Table 10.3). Critical care provided to children
organ systems such that there is a high probability of immi- aged 29 days to 24 months utilizes CPT code 99471 (initial
nent or life threatening deterioration in the patient’s condi- inpatient encounter) and 99472 (subsequent days)
tion” (Britton et al., 2019, pp. 15–46). Further, as per CPT, (Lesnick, 2010). CPT code 99475 is reported for initial inpa-
critical care “involves high complexity decision making to tient critical care encounter for children aged 2 through
assess, manipulate, and support vital system function(s) 5 years and CPT code 99476 is reported for subsequent
to treat single or multiple vital organ system failure and/ day critical care (Lesnick, 2010).
or to prevent further life threatening deterioration of the When neonates or children under 6 years of age are
patient’s condition” (Britton et al., 2019, pp. 15–46). transferred from one hospital to another, the receiving pro-
The overriding principle in documenting critical care vider should report an initial per diem critical care code
E/M is to demonstrate, with data and a narrative of interval (99471 or 99475) and the transferring provider should
events, a description of organ system(s) impairments; the report time-based codes for the day of transfer.
rationale of interventions to assess, manipulate, and sup-
port vital organ function; and the patient’s response to
Do time-based critical care management codes
treatment. A review of systems and most histories, unless
have different documentation requirements than
pertinent, are not required.
pediatric critical care management codes?

Are critical care management codes different for Time-based codes are reported for pediatric and adult
pediatric patients? critical care services depending on the circumstance
(Table 10.4). If a patient is aged 29 days to 5 years, per diem
Critical care codes for neonate and pediatric patients codes are used first. Conversely, if additional critical care is
(29 days to 5 years) are per diem. Once a patient reaches rendered by a second provider from a different group or
his or her sixth birthday, time-based codes are reported. specialty, time-based codes are then reported. If the patient
Pediatric critical care codes are exclusive to the inpatient is 6 years or older, time-based codes are always reported.
setting. If critical care is initiated in a clinic or emergency CPT code 99291 represents the first 30–74 minutes of crit-
department, time-based critical care codes are used. ical care provided by a single provider. CPT code 99292 may
While only one pediatric critical care code can be be reported for each additional 30 minutes (beyond 74 min-
reported per patient per calendar day, it is not uncommon utes) of critical care provider by an initial provider or a
for multiple providers to provide critical care to a single
patient in a calendar day. For example, a pediatric patient
Table 10.3 Pediatric critical care codes.
with septic shock and seizures might be seen by both a pedi-
atric intensivist and a neurologist on the same day. The
CPT code Age of child Per admission occurrence
intensivist is managing vasoactive medications, resuscita-
tion, antibiotics, and ventilator settings while the neurolo- 99471 29 days to 2 years ×1 per admission
gist is interpreting the electroencephalogram and providing
99472 29 days to 2 years Daily as appropriate
recommendations related to anti-seizure medications. One
99475 2–5 years ×1 per admission
physician, likely the intensivist, will report the pediatric
critical care code and the neurologist will report a time- 99476 2–5 years Daily as needed
based critical care code. Source: Lesnick (2010). Reproduced with permission of Elsevier.
Downloaded from https://onlinelibrary.wiley.com/doi/ by National Institute Of Standard, Wiley Online Library on [06/03/2024]. See the Terms and Conditions (https://onlinelibrary.wiley.com/terms-and-conditions) on Wiley Online Library for rules of use; OA articles are governed by the applicable Creative Commons License
42 Cases in Pediatric Acute Care

Table 10.4 Time-based codes. time in minutes (if reporting a time-based code); and
a provider attestation (if reporting time-based codes)
Minutes CPT codes (Pohlig, 2013).
The critical care attestation is a statement necessary for
30–74 99291 ×1
time-based codes. It is a declaration attesting that the
75–104 99291 ×1, 99292 ×1 reporting provider was in exclusive attention to the criti-
105–134 99291 ×1, 99292 ×2 cally ill patient, physically present on the unit, and the time
135–164 99291 ×1, 99292 ×3 reported is exclusive of unbundled procedures and services.
165–195 99291 ×1, 99292 ×4 The EMR provides clinicians with an opportunity to
>194 99291 ×1, 99292 × appropriate number develop templates to ensure the documentation includes
necessary billing elements and the appropriate verbiage
Source: Data from Mazza (2010).
for the attestation.

What services do not merit critical care


member of the provider’s group. The time reported may be management codes?
continuous or aggregate, but the total number of minutes
must reflect a single calendar day. However, if a clinician It is important to note that admission to an ICU does not
provides continuous critical care that was initiated before necessarily merit critical care management codes. Exam-
midnight and extends into the next calendar date, the time ples of situations that will not satisfy medical necessity
is attributed to the date the services were initiated. Some criteria for critical care are listed here.
insurers may require a start and stop time (e.g. 9:15 a.m.
to 10:10 a.m.) while others accept a cumulative total of min-
utes per day.
• The patient is admitted to the ICU for 24 hours of
ventilator management before a planned extubation
the following day. Routine postoperative ventilator
Time-based codes can be reported wherever critical care
management does not meet the standard for critical care.
services are delivered, including clinics, emergency depart-
ments, non-telemetry rooms, and ICUs (Painter, 2013).
Two providers from a single group cannot report critical
• The patient is admitted to the ICU for administration
of a thrombolytic for treatment of a pulmonary embo-
lism. The patient is otherwise stable. Admissions
care for the same time period. Thus, it is important to doc-
due to institution rules requiring ICU admission for
ument start and stop times when multiple providers are
particular treatments do not meet the standard for
delivering critical care. Finally, time-based critical care
critical care.
codes must be rendered in exclusive attention to a single
patient (Painter, 2013). The provider may not assess, deliver
care, or consult on other patients during the minutes time-
• The patient is admitted to the ICU because no other hos-
pital beds are available.

based critical care codes are reported.

Which services are bundled into pediatric critical PICU Case Study
care codes?
A 7-year-old African American male was admitted to the
Many procedures and services are bundled into pediatric
pediatric intensive care unit (PICU) with a diagnosis of
and time-based codes (Table 10.5). Bundled services are
bacterial pneumonia at 7 p.m. The patient was on sup-
assumed to be included in routine critical care and cannot
plemental oxygen via high-flow nasal cannula 20 L/
be billed separately.
min and FiO2 40%. At 8:15 p.m. the patient experienced
increased work of breathing and oxygen desaturation to
What elements need to be documented for
80%. PaO2 on arterial blood gas analysis is 67 mmHg.
pediatric critical care billing?
A decision is made to intubate the patient, which is
Documentation for critical care services should include accomplished by the PNP. The patient becomes hypoten-
patient information, including name, MRN, and date of sive requiring fluid resuscitation and the insertion of
birth; a history of present critical event; summary of inter- a non-tunneled central venous catheter. A nasogastric
val events, including description of unstable or failing tube is also inserted. A bronchoscopy is performed at
organ system(s), medical necessity, and response to treat- the bedside by a pulmonologist. Thick purulent secre-
ment; corresponding diagnoses; summary of communica- tions are observed in addition to a mucous plug in the
tion with patient, family, surrogates and other providers; right middle lobe, which is successfully removed via
Downloaded from https://onlinelibrary.wiley.com/doi/ by National Institute Of Standard, Wiley Online Library on [06/03/2024]. See the Terms and Conditions (https://onlinelibrary.wiley.com/terms-and-conditions) on Wiley Online Library for rules of use; OA articles are governed by the applicable Creative Commons License
Chapter 10 Billing and Coding 43

Table 10.5 Bundled services.

Services bundled into per diem pediatric Services bundled into time-based
critical care codes critical care codes

Peripheral vessel catheterization √


Introduction of needle or intracatheter, upper or √
lower extremity artery
Arterial catheterization for sampling, monitoring, or √
transfusion
Umbilical venous catheterization √
Central vessel catheterization √
Vascular access procedures √
Vascular puncture √
Umbilical arterial catheterization √
Endotracheal intubation √
Ventilation management √ √
Bedside pulmonary function test √ √
Surfactant administration √
Continuous positive airway pressure √ √
Monitoring or interpretation of blood gases or √ √
oxygen saturation
Car seat evaluation √
Transfusion of blood components √ √
Oral or nasogastric tube placement √ √
Suprapubic bladder aspiration √ √
Bladder catheterization √ √
Lumbar puncture √
Interpretation of cardiac output measures √ √
Chest radiography √ √
Collection and interpretation of physiologic data √ √
Temporary transcutaneous pacing √ √
Vascular access procedures √
Source: Data from Grider (2019).

suction. A bronchoalveolar lavage sample is sent for 5 What is the critical care billing for a child less than
culture. The endotracheal tube and nasogastric tube 5 years of age?
are verified by portable chest and abdominal X-ray 6 Must the PNP know every rule and regulation related to
respectively. billing?

Questions Rationale and Evidence-based


Practice Explanation
Answer the following questions using the details provided.
What is the critical care time that can be billed?
1 What is the critical care time that can be billed?
2
3
What
What
services are bundled and cannot be billed?
services are unbundled and can be billed?
• Time spent providing care in exclusive attention, as well
as direct and indirect care, to this critically ill and unsta-
4 What billing codes are appropriate for this case? ble patient: 8:15 p.m. to 11:05 p.m.
Downloaded from https://onlinelibrary.wiley.com/doi/ by National Institute Of Standard, Wiley Online Library on [06/03/2024]. See the Terms and Conditions (https://onlinelibrary.wiley.com/terms-and-conditions) on Wiley Online Library for rules of use; OA articles are governed by the applicable Creative Commons License
44 Cases in Pediatric Acute Care

• The patient is greater than 5 years old so time-based critical


care codes are reported. Critical care time: 170 minutes
•• Insertion of a nasogastric tube
Review of chest X-ray and abdominal X-ray

What services are bundled and cannot be billed?


•• Endotracheal intubation
Insertion of non-tunneled central venous catheter

• Coordinating care and ordering laboratory and imaging


investigations Unbundled procedures

• Review of arterial blood gas (and other laboratory


investigations)
• Bronchoscopy (25 minutes)

•• Insertion of a nasogastric tube


Review of chest X-ray and abdominal X-ray Appropriate billing codes

• 99475 ×1 for initial critical care time for child aged


2–5 years


What services are unbundled and can be billed?


31622 ×1 for bronchoscopy
Unbundled procedures include intubation (15 minutes
spent on intubation), central venous catheter insertion
(30 minutes) and bronchoscopy (20 minutes). Total time: Must the PNP know every rule and regulation
170 – 15 – 30 – 20 = 105 minutes

••
related to billing?
The PNP bills for the procedures performed
Healthcare is a regulated business and, as such, providers
The PNP can also bill for the critical care time if the inten-
must comply with rules and regulations. The US Office of
sivist is not present and the PNP delivered the services
Inspector General enforces regulations related to medical
during the total minutes reported in exclusive attention
billing and reimbursement in the United States. The regu-
to this patient
lations are in place to prevent fraud by healthcare providers
and billers. It is therefore important that documentation is
What billing codes are appropriate for this case? accurate and reflects the services the provider rendered and

•• 99291 ×1 for first 74 minutes


99292 ×1 for additional 30 minutes
a patient’s factual state of health. There should never be an
effort to “up-code” if the facts and documentation do not

•• 31500 ×1 for endotracheal intubation


36556 ×1 for insertion of non-tunneled centrally inserted
support the higher level of billing. While details and accu-
racy help facilitate optimal billing potential, it is incumbent
upon the PNP to always accurately document.
venous catheter age 5 or older

• 31622 ×1 for bronchoscopy


The Health Insurance Portability and Accountability Act
(HIPAA) was established in 1996. The rules and regulations
put forth in the HIPAA legislation guarantees rights to
What is the critical care billing for a child less than patients with regard to their healthcare and medical record.
5 years of age? It also safeguards patient confidentiality and a secure med-

• Time spent providing exclusive attention, as well as direct


and indirect care, to this critically ill and unstable patient
ical record, whether it is paper or electronic.
Every healthcare organization should have a compliance
office. Compliance officers provide clinicians with educa-
(if he were under 5 years old): 8:15 p.m. to 11:05 p.m.

• Critical care time: 170 minutes


tion, advice and support related to rules and regulations.
In addition, billing entities often provide education and
resources to providers regarding documentation and
Bundled procedures required billing elements. PNPs should familiarize them-

• Coordinating care and ordering laboratory and imaging


investigations
selves with resources and know appropriate contacts within
an organization that can provide guidance and support

• Review of arterial blood gas (and other investigations) related to documentation and compliance.

References
Britton, F., Brown, D., Cachecho, R., et al. (2019). Coding and Grider, D. (2019). Revisiting neonatal and pediatric critical
Billing for Critical Care: A Practice Tool. Prospect, IL: Society care services. Critical Care Connections (May/June/July),
of Critical Care Medicine. 30–31.
Downloaded from https://onlinelibrary.wiley.com/doi/ by National Institute Of Standard, Wiley Online Library on [06/03/2024]. See the Terms and Conditions (https://onlinelibrary.wiley.com/terms-and-conditions) on Wiley Online Library for rules of use; OA articles are governed by the applicable Creative Commons License
Chapter 10 Billing and Coding 45

Lesnick, B. (2010). Bundled babies and bundled billing: how Painter, J. (2013). Critical care in the surgical global period.
to properly use the new pediatric critical care codes. Chest Chest 143(3): 851–855. doi: 10.1378/chest.09-0359.
137(3): 701–704. doi: 10.1378/chest.09-1059. Pohlig, C. (2013). How to avoid Medicare denials for critical-
Mazza, L.E. (2010). Understanding and coding for critical care billing. The Hospitalist (October). Available at https://
care services. JustCoding News: Inpatient, 6 January 2010. www.the-hospitalist.org/hospitalist/article/125598/health-
Available at https://www.hcpro.com/HIM-244419-3288/ policy/how-avoid-medicare-denials-critical-care-billing
Understanding-and-coding-for-critical-care- (accessed 31 March 2019).
services.html. Wound, Ostomy, and Continence Nurses Society (2012).
Munro, N. (2013). What an acute care nurse practitioner Reimbursement of advanced practice registered nurse
should know about reimbursement. AACN Advanced services: a fact sheet. Journal of Wound, Ostomy, and
Critical Care 24(2): 110–113. doi: 10.1097/ Continence Nurse 39(2 Suppl): S7–S16. doi: 10.1097/
NCI.0b013e31828c8890. WON.0b013e3182478df0.

Further Reading
Center for Medicare and Medicaid Services (2019). National Education/Medicare-Learning-Network-MLN/
Plan and Provider Enumeration System. https://nppes.cms. MLNProducts/Downloads/eval-mgmt-serv-guide-
hhs.gov/#/ (accessed 30 March 2019). ICN006764.pdf (accessed 10 April 2019).
Medicare Learning Network (2017). Evaluation and
Management Services. https://www.cms.gov/Outreach-and-
Downloaded from https://onlinelibrary.wiley.com/doi/ by National Institute Of Standard, Wiley Online Library on [06/03/2024]. See the Terms and Conditions (https://onlinelibrary.wiley.com/terms-and-conditions) on Wiley Online Library for rules of use; OA articles are governed by the applicable Creative Commons License
47

11

Language Delay
Jolene Dickerson
Nemours/Alfred I. duPont Hospital for Children, Wilmington, DE, USA

A 15-month-old boy presents with his mother to the Family History


primary care office for his well child visit. He is healthy
with no previous concerns. There is no significant family medical history. There is a
6-year-old brother and an 8-year-old sister who are both
healthy and live with the family. Parents are healthy. There
History of Present Illness
are no family members with learning disabilities or other
developmental delays.
The child’s mother reports she has no concerns for today’s
visit; however, on being asked routine developmental
screening questions, she reveals that he is not saying any
words and this does concern her.
Current Status
Past Medical History
The toddler appears well nourished and happy. His physi-
cal examination is unremarkable. A brief developmental
The toddler was born at a gestation of 38 weeks 4 days via
assessment finds that he is not speaking any words, just
spontaneous vaginal delivery. There were no complications
babbling. He does not say “mama” or “papa” specifically.
during the pregnancy or labor and delivery. His birthweight
He follows directions (such as putting trash in the trash
was 3.23 kg and he passed the newborn hearing screen, car-
can) and will point to what he wants. He is walking, climb-
diac screen, and metabolic testing. He is up to date on his
ing and beginning to run. He uses a spoon to feed himself
routine vaccinations, and he will receive 15-month vaccines
and scribbles with a crayon. Per mother’s report, he is
today. Weight is at the 40th percentile, height is at the 50th
always busy and rarely sits still.
percentile, and head circumference is at the 90th percentile.
The Pediatric Evaluation of Developmental Status (PEDS)
screening tool was administered at 9 months of age and
was negative for developmental delay. Other screening
questions at each previous visit have been within normal Questions
limits. His past medical history is significant for transient
tyrosinemia, which resolved in the newborn period, and Answer the following questions using the details provided.
two episodes of otitis media, both treated and resolved.
1 Based on the data provided, what are the differential
diagnoses and the most likely diagnosis for this child?
Past Surgical History 2 What is the best way to address this child’s developmen-
tal concerns?
Only surgical history includes circumcision during 3 What other referrals or diagnostic studies would be
newborn hospitalization. indicated at this time?

Cases in Pediatric Acute Care: Strengthening Clinical Decision Making, First Edition. Edited by Andrea M. Kline-Tilford and Catherine M. Haut.
© 2020 John Wiley & Sons Ltd. Published 2020 by John Wiley & Sons Ltd.
Downloaded from https://onlinelibrary.wiley.com/doi/ by National Institute Of Standard, Wiley Online Library on [06/03/2024]. See the Terms and Conditions (https://onlinelibrary.wiley.com/terms-and-conditions) on Wiley Online Library for rules of use; OA articles are governed by the applicable Creative Commons License
48 Cases in Pediatric Acute Care

Rationale and Evidence-based Practice are asked about his language and social development,
Explanation which appears to be normal as he is interactive and does
respond to directions and questions. Referral to a pediatric
Based on the data provided, what are the developmental specialist will help identify any specific
differential diagnoses and the most likely delays or concerns to determine what services can be uti-
diagnosis for this child? lized to help him. Early intervention is key to improving
outcomes in children with developmental delay (Bright
Differential diagnoses for this child include language delay, Futures Steering Committee, 2006, pp. 405–406).
autism, and hearing loss. Language delay is the most likely Collaboration with the developmental specialist and
diagnosis. Language development begins early: infants in others involved in his care, such as the speech therapist,
the early months of life will make cooing sounds, and begin will assist in determining if the child is meeting milestones.
to vocalize and babble. By one year of life, an infant should He will also be assessed at each well child visit, using formal
make sounds to get attention and has one to two words that developmental assessment tools, to ensure he is making
are usually most understandable to parents. By age 2, a progress and receiving all needed services. The primary
toddler should be able to put some words together in short care provider should receive reports from the pediatric
sentences. developmental specialist and support the family in utilizing
A diagnosis of autism does include lack of or limited recommendations.
language, but also will involve other social findings, includ-
ing avoidance of eye contact and hand flapping, Congenital
What other referrals or diagnostic studies would
hearing loss is less likely as the child passed the hearing
be indicated at this time?
screen at birth and the other forms of hearing loss would
be associated with illness; for example, conductive hearing This young boy is referred to the state developmental
loss would occur as a result of otitis media and/or fluid in assessment and support program, where he will undergo
the middle ear. The most likely diagnosis for this child is complete evaluation by a developmental pediatrician. He
language delay, as language development has begun; will have an assessment to determine any social, gross
however, autism and hearing loss must also be ruled out. motor, fine motor and/or language problems. Recom-
mended therapy and/or other specialty referrals will be
identified. The developmental pediatrician and hearing
What is the best way to address this child’s specialist determine that he has language delay and
developmental concerns? recommend speech therapy and an audiology referral to
Early language screening is important for identifying rule out hearing loss. The primary care provider will con-
autism and hearing loss, but also language delay as a tinue to follow this child’s developmental progress and
primary diagnosis. Recent research indicates that children his social/adaptive skills at each well child visit to deter-
with language delay present with middle ear ventilation mine if any other referrals are warranted.
disorders, have family histories of language disorders, An autism evaluation is also recommended by the state
and fall into categories representing expressive language program, and therefore the child is referred to a pediatric
delay, mixed receptive and expressive language delay, delay psychologist who specializes in developmental pediatrics
with cognitive impairment or intellectual disability, and and autism. It can be difficult to quickly refer to an autism
autism (Singleton, 2018). Another consideration which is program or evaluation specialist, and it may be hard to
more prevalent today is the issue of English as a second lan- diagnose autism at this age, but there are also office-based
guage, and the many children who will grow up in bilingual tools to utilize such as M-Chat, which is a parent-response
families. Challenges to the evaluation of these children, tool that identifies typical features and behaviors of
who may be “late talkers,” include the linguistic variations children with autism.
between the child’s language, variations of timing of
learning each language, and potential cultural differences
in screeners and the child/family (Singleton, 2018). Case Resolution
At any well child visit, the care provider should first
address the concerns of the parent. The Bright Futures At his 18-month visit, the child’s mother answers
recommendations for the 15-month visit include evaluating another PEDS screening assessment. The American
social skills and communication, sleep patterns and issues, Academy of Pediatrics recommends routine developmen-
oral hygiene, and safety. This child’s mother is concerned tal screening at the 9-, 18-, and 30-month well child visits
that he is not speaking, so a few more specific questions using a standardized screening tool such as the PEDS
Downloaded from https://onlinelibrary.wiley.com/doi/ by National Institute Of Standard, Wiley Online Library on [06/03/2024]. See the Terms and Conditions (https://onlinelibrary.wiley.com/terms-and-conditions) on Wiley Online Library for rules of use; OA articles are governed by the applicable Creative Commons License
Chapter 11 Language Delay 49

screening assessment, which is a 10-question survey of and fingers among others. This child underwent 18-month
development for parents to complete and which can be screening with use of the PEDS questionnaire. Mother has
used for young ages (Bright Futures, 2006, p. 406). This concerns that he is not speaking, but he receives speech
screen asks general questions with regard to speech, therapy twice weekly and he is also undergoing an autism
behavior, getting along with others, and use of hands assessment.

References
Bright Futures Steering Committee (2006). Identifying infants Singleton, N.C. (2018). Late talkers: why the wait and see
and young children with developmental disorders in the approach is outdated. Pediatric Clinics of North America 65:
medical home: an algorithm for developmental surveillance 13–29. doi: 10.1016/j.pcl.2017.08.018
and screening. Pediatrics 118(1): 405–420.
Downloaded from https://onlinelibrary.wiley.com/doi/ by National Institute Of Standard, Wiley Online Library on [06/03/2024]. See the Terms and Conditions (https://onlinelibrary.wiley.com/terms-and-conditions) on Wiley Online Library for rules of use; OA articles are governed by the applicable Creative Commons License
51

12

A 6-Month-Old Infant with RSV and Respiratory Failure


Catherine M. Haut1,2
1
Nemours/Alfred I. duPont Hospital for Children, Wilmington, DE, USA
2
Mednax-Pediatrix Medical Group, Richardson, TX, USA

A 6-month-old, breast-fed, term baby with a 3-day history Past Surgical History
of nasal congestion and cough presents to the primary care
provider for evaluation. No previous surgical procedures.

History of Present Illness Family History


She has had a fever to 38.8 C (101.8 F) and decreased appe- A 3-year-old sibling had one episode of wheezing with a
tite, but temperature responds well to acetaminophen and cold at age 18 months. There is no history of asthma in
she is happy and appears comfortable. Evaluation by the the family. Parents are healthy and both sets of grandpar-
primary care provider indicates scattered expiratory ents are alive and healthy. The infant attends home daycare
wheezes and mild tachypnea. The parents are instructed with her sibling.
to use saline nose drops for congestion, feed her more fre-
quently in smaller amounts, and monitor her work of
breathing. Questions
Two days later, the primary care provider is informed that
the infant is working harder to breath and the parents are Answer the following questions using the details provided.
worried, so the child is taken to the emergency department
for further care. The baby continues to be tachypneic with 1 Based on the history and physical examination, what
a respiratory rate of 70 breaths per minute and is hypoxic with are the potential differential diagnoses?
oxygen saturation of 86% on room air. She is placed on a high- 2 What diagnostic work-up is warranted and why?
flow nasal cannula (HFNC) at a flow rate of 4 L/min, but is 3 What would be the next therapy for management of res-
still working hard with diffuse wheezing throughout her lung piratory distress and why?
fields. During observation, she is noted to have a very short 4 What other therapeutic modalities are recommended at
episode of apnea, which occurs again within 15 minutes. this time?
Other than respiratory findings, the remainder of her physical Continuation of case: This baby girl could not maintain
examination is normal, except for the presence of a right otitis oxygenation and comfortable respirations on nasal cannula
media and a temperature of 38.3 C (100.9 F). at 4 L/min, so care is escalated to HFNC at 14 L/min and
40% oxygen, and then to intubation and ventilation as
she had additional episodes of apnea. She also could not
Past Medical History maintain sufficient oxygenation during nasopharyngeal
suction, and her secretions were thick and obstructive.
The patient has received all required vaccines up to Because of respiratory failure, she is admitted to the pedi-
6 months, along with one influenza vaccine and her growth atric intensive care unit (PICU) for further monitoring
and development are appropriate. She has no other medical and care. A dose of nebulized albuterol is given with little
problems. improvement, so no further medication is administered
Cases in Pediatric Acute Care: Strengthening Clinical Decision Making, First Edition. Edited by Andrea M. Kline-Tilford and Catherine M. Haut.
© 2020 John Wiley & Sons Ltd. Published 2020 by John Wiley & Sons Ltd.
Downloaded from https://onlinelibrary.wiley.com/doi/ by National Institute Of Standard, Wiley Online Library on [06/03/2024]. See the Terms and Conditions (https://onlinelibrary.wiley.com/terms-and-conditions) on Wiley Online Library for rules of use; OA articles are governed by the applicable Creative Commons License
52 Cases in Pediatric Acute Care

other than the antibiotic. The infant continues to need and include the presence of respiratory illness in children
higher oxygen levels and higher flow rates, so a decision less than 2 years of age. Clinical symptoms include rhinor-
is made to intubate her in order to improve oxygenation rhea, cough, tachypnea, wheezing, crackles, and respira-
and for more efficient suctioning. She is started on intrave- tory distress at varied levels, with the possibility of
nous dexmedetomidine and fentanyl infusions for sedation grunting, nasal flaring and retractions.
and analgesia. An intravenous catheter was in place and RSV is an enveloped, non-segmented, negative-stranded
she is now nil by mouth and receiving fluids at RNA virus with two subtypes, with subtype A causing more
maintenance. severe illness. An RSV infection begins when the virus is
replicated in the upper airway, typically causing initial
5 What is the rationale and benefit to using HFNC for
cold-like symptoms. The virus spreads to the bronchiolar
treatment of respiratory distress? When would intuba-
epithelium of the small airways, becoming a lower airway
tion and ventilation be appropriate?
disease process. Edema of the airways, along with increased
6 When is weaning from HFNC indicated?
production of mucus, results in small airway obstruction,
7 What criteria are used for transfer from the PICU to the
air trapping, and increased resistance. Wheezing is noted
pediatric inpatient unit?
when air cannot pass easily through narrowed pathways.
In addition, the damaged epithelial cells initially necrose
in the process of regeneration, producing additional debris
to further obstruct air movement.
Rationale and Evidence-based Practice
Explanation
What diagnostic work-up is warranted and why?
Based on the history and physical examination,
what are the potential differential diagnoses? The American Academy of Pediatrics (AAP) has published
guidelines for the diagnosis and management of bronchio-
Based on the history, presenting findings and timeline, the
litis outside of the intensive care unit. There are no
most likely diagnosis would be respiratory syncytial virus
documented recommended medications or therapies that
(RSV) bronchiolitis. Other possible diagnoses include aspi-
hasten resolution of symptoms of bronchiolitis, so support-
rated foreign body, reactive airways disease, and pneumo-
ive care is the treatment. Nebulized saline treatments are
nia. In order to evaluate for pneumonia, a chest radiograph,
the only therapy which can be “considered” per the AAP
both anteroposterior and lateral, would be recommended.
guidelines. Radiographs, laboratory studies and viral panels
Other symptoms of pneumonia include decreased appetite,
are not routinely recommended for diagnosis. A patient
persistent fever, and respiratory distress as tachypnea with
who requires intensive care may require additional diag-
oxygen required, which all exist for this patient. There is no
nostic studies to determine comorbidities.
history of the infant choking and she only recently started
baby foods and has not had any table foods as yet, so a
foreign body aspiration is unlikely. Wheezing is an
What would be the next therapy in management
indication of reactive airways disease, but in such a child
of respiratory distress and why?
the first episode of wheezing at less than 1 year of age
defines bronchiolitis. Hospitalization for infants with bronchiolitis is indicated
Bronchiolitis is a common diagnosis in young infants and when they are unable to manage feeding to maintain
occurs seasonally in most areas of the United States. hydration and/or have an oxygen requirement or respi-
Bronchiolitis is usually caused by a viral process and there ratory distress that requires support. If oxygenation can-
are many known viral etiologies, with the most common, not be maintained and the infant is tachypneic with
RSV, accounting for 50–80% of all hospitalizations from respiratory distress, then the next recommendation for
bronchiolitis (Meissner, 2016). Rhinovirus, parainfluenza, care is to use a non-invasive ventilation mode, in the
enterovirus, coronavirus, and human metapneumovirus form of continuous positive airway pressure (CPAP),
are others that can cause bronchiolitis. Recent data indicate bilevel positive airway pressure (BiPAP), or high-flow
that infants who are hospitalized for bronchiolitis are usu- nasal cannula (HFNC), which can be more comfortable,
ally term infants between the ages of birth and 5 months, stays in place more easily, and allows oxygen to be
with risks higher among infants who have coexisting pro- titrated. An additional level of support with intubation
blems such as prematurity, chronic lung disease of and ventilation is reserved but sometimes indicated
prematurity, and cardiac disease. Diagnostic criteria for when babies cannot maintain an adequate open airway
bronchiolitis are based on history and physical examination and/or oxygenation.
Downloaded from https://onlinelibrary.wiley.com/doi/ by National Institute Of Standard, Wiley Online Library on [06/03/2024]. See the Terms and Conditions (https://onlinelibrary.wiley.com/terms-and-conditions) on Wiley Online Library for rules of use; OA articles are governed by the applicable Creative Commons License
Chapter 12 A 6-Month-Old Infant with RSV and Respiratory Failure 53

What other therapeutic modalities are oxygenation. Also, using high humidity, HFNC prevents
recommended at this time? thickening of secretions, decreasing some of the obstructive
components of bronchiolitis.
Antibiotic treatment can be recommended for this infant,
The use of non-invasive ventilation in the PICU has
due to the presence of otitis media with fever. Ceftriaxone
many advantages: decreased risk of ventilator-associated
is not first-line therapy for otitis media, but can be given
pneumonia, decreased need for sedation, decreased length
either intravenously or intramuscularly if oral medications
of stay, and attenuation of the laryngeal and tracheal injury
are not tolerated. Ceftriaxone is broad spectrum and the
and barotrauma associated with intubation and mechani-
dosing for otitis media is 75 mg/kg once daily for 3 days.
cal ventilation. HFNC has resulted in higher nurse and
With worsening status, though, treatment can be
patient satisfaction, increased mobility for the child, and
continued, especially if there is concern for a bacterial
overall lower hospital costs of care.
pneumonia.
Infants with RSV bronchiolitis can present with apnea
and hypoxia. Because of fatigue, increased secretions and
What is the rationale and benefit to using HFNC for
respiratory failure, the next level of respiratory support is
treatment of respiratory distress? When would
intubation and mechanical ventilation.
intubation and ventilation be appropriate?
Non-invasive ventilation can be either CPAP/BiPAP or
HFNC. CPAP/BiPAP employs a set pressure to deliver When is weaning from HFNC indicated?
positive end-expiratory pressures. These systems work by
There are no standard weaning criteria for HFNC and there
using pressure that distends end-airways at the alveolar
are no evidence-based feeding protocols, although some
level, improving oxygenation. HFNC is a mode of non-
institutions have developed their own pathways. There
invasive ventilation which is not as well defined. Heated
are some that utilize a small percentage of decrease in
HFNC or high-velocity nasal insufflation supplies humidi-
HFNC flow over time, and others that wean quickly. Based
fied and blended air/oxygen via nasal cannula at different
on the nature of bronchiolitis and the variability of illness,
flow rates (≥1–2 L/min), delivering both high concentra-
weaning time frame can also vary. In this child, weaning
tions of oxygen and potentially continuous distending pres-
was slow, but after 3 days in the PICU it became easier
sure. HFNC utilizes a small, thin tapered nasal cannula,
to wean the high flow, and when she was on 8 L/min
which facilitates the velocity of flow. It is unknown exactly
she was transferred to the pediatric inpatient unit.
how much pressure is utilized based on flow rate, but this
treatment has been used successfully in neonates and
infants (Beggs et al., 2014).
What criteria are used for transfer from the PICU to
Bronchiolitis can result in the collection of intraluminal
the pediatric inpatient unit?
debris and mucus in the lower airways, causing obstruc-
tion, contributing to tachypnea, and resulting in muscle Discharge criteria include ability to feed and gain weight
fatigue and upper airway collapse. Non-invasive ventilation and no further need for oxygen. Discharge teaching should
assists in providing distending pressure to the lower air- include information that bronchiolitis can result in a cough
ways, offering a mechanism of airway conductance and which lasts a longer time and that it can also recur.
pulmonary compliance. HFNC decreases dead space in Research indicates that infants who develop bronchiolitis
the airway with washout of carbon dioxide and increased have a higher risk of wheezing or developing asthma later
fraction of oxygen in the alveoli, thus increasing in childhood.

References
Beggs, S., Wong, Z.H., Kaul, S., Ogden, K.J., and Walters, J.A. Meissner, H.C. (2016). Viral bronchiolitis in children. New
(2014). High-flow nasal cannula therapy for infants with England Journal of Medicine 374: 62–72. doi: 10.1056/
bronchiolitis. Cochrane Database of Systematic Reviews (1): NEJMra1413456.
CD009609.

Further Reading
Ralston, S.L., Lieberthal, E.S., Meissner, H.C., et al. (2014). prevention of bronchiolitis. Pediatrics 134(5): e1474–e1502.
Clinical Practice Guideline: the diagnosis, management and doi: 10.1542/peds.2014-2742.
Downloaded from https://onlinelibrary.wiley.com/doi/ by National Institute Of Standard, Wiley Online Library on [06/03/2024]. See the Terms and Conditions (https://onlinelibrary.wiley.com/terms-and-conditions) on Wiley Online Library for rules of use; OA articles are governed by the applicable Creative Commons License
55

13

Adolescent Abdominal Pain after Cardiac Surgery


Cathy Woodward
Long School of Medicine, University of Texas Health San Antonio, San Antonio, TX, USA

A 12-year-old was admitted to the step-down unit 2 days Current Status


after a repair of coarctation of the aorta.
The child develops severe abdominal pain and the regis-
tered nurse pages the healthcare provider to the bedside
to evaluate the patient. On arrival, the patient is pale, dia-
History of Present Illness phoretic and complaining of mid-epigastric abdominal
pain. The child has vomited twice in the past 2 hours,
The child was diagnosed with coarctation of the aorta by
but denies diarrhea. On further investigation, the registered
echocardiogram as part of a work-up conducted after a
nurse reports that the child has not passed stool since prior
pre-sports physical revealed hypertension. The surgery
to the surgery. The mother reports that the child, who was
and immediate postoperative course were uneventful with
on a full liquid diet, ate several slices of pizza when visited
the exception of hypertension initially treated postopera-
by family members earlier in the evening. On examination,
tively with intravenous antihypertensives, but later transi-
his abdomen is full, distended and tender to palpation over
tioned to the oral angiotensin-converting enzyme (ACE)
the epigastric area with no bowel sounds noted. His skin is
inhibitor enalapril. The child was weaned off all intrave-
slightly diaphoretic, pulses are 2+ in all extremities, and the
nous medications and continued on oral acetaminophen
thoracotomy site is clean and dry with no drainage or
and ibuprofen, as needed for pain, when admitted to the
erythema.
step-down unit.
Vital signs at the time of examination are as follows: tem-
perature 37.2 C (98.9 F), heart rate 110 bpm with regular
rhythm, respiratory rate 18 breaths per minute, blood pres-
Past Medical History sure 138/80 mmHg, and oxygen saturation 100% on
room air.
The child is up to date on immunizations; height and
weight at the 75th percentile.

Questions
Past Surgical History Answer the following questions using the details provided.

No previous surgeries or hospitalizations. 1 What diagnoses are included in your differential?


2 What diagnostic studies or imaging would you order?
3 What finding is demonstrated on the abdominal flat
plate radiograph?
Family History 4 Based on the information provided, which is the most
probable diagnosis?
The patient lives with mother and two older siblings. 5 What treatments should you initiate?

Cases in Pediatric Acute Care: Strengthening Clinical Decision Making, First Edition. Edited by Andrea M. Kline-Tilford and Catherine M. Haut.
© 2020 John Wiley & Sons Ltd. Published 2020 by John Wiley & Sons Ltd.
Downloaded from https://onlinelibrary.wiley.com/doi/ by National Institute Of Standard, Wiley Online Library on [06/03/2024]. See the Terms and Conditions (https://onlinelibrary.wiley.com/terms-and-conditions) on Wiley Online Library for rules of use; OA articles are governed by the applicable Creative Commons License
56 Cases in Pediatric Acute Care

Rationale and Evidenced-based Practice


Explanation

What diagnoses are included in your differential?


The differential diagnoses include gastric reflux, dyspepsia,
post-coarctectomy syndrome, pancreatitis, gastrointestinal
viral infection, and food poisoning.
Post-coarctectomy syndrome can occur in children after
repair of coarctation of the aorta due to return of pulsatile
blood flow distal to the coarctation including to the mesen-
teric arteries. This sudden reperfusion to arteries that have
not seen pulsatile flow can cause reflex vasoconstriction
and ischemia to the organs and tissues perfused by the mes-
enteric vessels, causing abdominal pain and distension,
vomiting, and ileus. This is the most likely diagnosis for this
patient. If it progresses, perforation with bloody stools and
possibly shock can be seen. The syndrome may be associ-
ated with early feeding and increasing oxygen requirements
of the abdominal organs. A common preventive treatment
is to avoid early feeding and progress the diet slowly
48 hours after repair of coarctation of the artery. Figure 13.1 Abdominal flat plate radiograph.

What diagnostic studies or imaging would


you order? return is indicated. A clear liquid diet can be initiated after
bowel sounds return. Avoidance of opioid analgesic medi-
A complete blood count, electrolytes, C-reactive protein
cations, monitoring of abdominal girths and ambulation
(CRP), liver enzymes, amylase and lipase are ordered as well
should be encouraged. Monitoring is needed for signs of
as an abdominal flat plate radiograph (see Figure 13.1).
worsening ileus, peritonitis, pain, fever, abdominal disten-
C-reactive protein is found to be abnormal (150 mg/L;
sion and shock. If ileus occurs, returning to nil by mouth
normal, 1–3 mg/dL).
status until bowel function returns, using non-opioid
analgesics and encouraging ambulation are the most com-
What finding is demonstrated on the abdominal
mon measures taken to reduce further complications. The
flat plate radiograph?
placement of a nasogastric tube may not be warranted in
Gas-filled loops indicative of ileus. No pneumatosis or free this case; however, if the ileus worsens it may become nec-
air noted. essary. If perforation occurs, a laparotomy and bowel re-
section may be needed.
Based on the information provided, which is the
most probable diagnosis?
Ileus due to post-coarctectomy syndrome.
Case Resolution
What treatments should you initiate?
This patient’s ileus resolved with conservative management
Nil by mouth status and initiation of intravenous fluids at and he was discharged 3 days later with adequate bowel
maintenance rate until pain resolves and bowel sounds function and persistent hypertension.

Further Reading
Roeleveld, P.P. and Zwijsen, E.G. (2017). Treatment Sapin, S.O., Rosengart, R.M., and Salem, M.M. (2002). Chest
strategies for paradoxical hypertension following surgical pain during stenting of a native aortic coarctation: a case
correction of coarctation of the aorta in children. World study for acute intercostal muscle ischemia and
Journal of Pediatric and Congenital Heart Surgery 8(2): rhabdomyolysis. Catheterization and Cardiovascular
321–331. Interventions 57: 217–220.
Downloaded from https://onlinelibrary.wiley.com/doi/ by National Institute Of Standard, Wiley Online Library on [06/03/2024]. See the Terms and Conditions (https://onlinelibrary.wiley.com/terms-and-conditions) on Wiley Online Library for rules of use; OA articles are governed by the applicable Creative Commons License
57

14

Young Infant with Vomiting


Sarah Martin
Ann and Robert H. Lurie Children’s Hospital of Chicago, Chicago, IL, USA

A 6-week-old male infant, weighing 3.6 kg, has a 3-day minute, and blood pressure 80/44 mmHg. His physical
history of projectile, non-bloody, non-bilious, postprandial examination reveals a quiet infant sucking vigorously on
emesis. a pacifier. The infant’s cardiac and respiratory examination
are unremarkable. His abdominal examination reveals a
soft, rounded, non-tender abdomen. There are no abdomi-
History of Present Illness
nal masses, hernias, or hepatosplenomegaly appreciated.
The infant is awake with normal tone and is moving all
Parents bring their 6-week-old male infant to the emergency
extremities well. His anterior fontanel is flat. The infant’s
department (ED) with a 3-day history of projectile, non-
extremities are pink, warm, and dry with a capillary refill
bloody, non-bilious, postprandial emesis. The infant has
time of 2–3 seconds. His radial and posterior tibial pulses
been afebrile, stooling daily, and has had three wet diapers
are palpable and are 3+/4+.
a day, last voided 8 hours ago. He has been breastfeeding
To further evaluate the infant a complete blood count,
eagerly every 3 hours; however, following the feeds the
basic metabolic panel (BMP) and an abdominal ultrasound
infant has been having large projectile emesis. There have
are performed. The results of the BMP are detailed in
been no sick contacts and the infant does not attend daycare.
Table 14.1.

Past Medical History


Questions
This infant was born full-term by normal vaginal delivery
and was discharged home with mother on the second day Answer the following questions using the details provided.
of life. The infant has no past medical or surgical history, 1 Based on the presentation and initial laboratory values,
except for circumcision at birth without any complications what is the most likely diagnosis and two other
including bleeding. differentials?
Continuation of case: The abdominal ultrasound reveals
Family History a thickened pyloric muscle measuring up to 0.6 cm and an
elongated pyloric channel measuring 2.2 cm. These ultra-
The infant has a healthy 2-year-old brother and several sound findings are consistent with the diagnosis of hyper-
young cousins, but no history of surgery in the first trophic pyloric stenosis (HPS) or pyloric stenosis (PS).
6 months of life. Both parents and grandparents are
healthy. There are no known family members with a his- 2 What is the physiologic basis for the electrolyte
tory of anesthetic complications or bleeding disorders. derangements seen in infants with pyloric stenosis?
3 What are the clinical criteria and diagnostic evaluation
for the diagnosis of pyloric stenosis?
Current Status 4 What is the treatment for infants with pyloric stenosis?
5 Why would anesthesia not be administered to this
On arrival in the ED the infant is normothermic, with a infant with the electrolyte derangements noted at
heart rate of 170 bpm, respiratory rate of 32 breaths per admission?
Cases in Pediatric Acute Care: Strengthening Clinical Decision Making, First Edition. Edited by Andrea M. Kline-Tilford and Catherine M. Haut.
© 2020 John Wiley & Sons Ltd. Published 2020 by John Wiley & Sons Ltd.
Downloaded from https://onlinelibrary.wiley.com/doi/ by National Institute Of Standard, Wiley Online Library on [06/03/2024]. See the Terms and Conditions (https://onlinelibrary.wiley.com/terms-and-conditions) on Wiley Online Library for rules of use; OA articles are governed by the applicable Creative Commons License
58 Cases in Pediatric Acute Care

6 What feeding regimen should be offered to an infant leading to paradoxical aciduria. An elevated unconjugated
following a pyloromyotomy? hyperbilirubinemia related to transient impairment of glu-
7 What are the signs and symptoms that would be con- curonyltransferase activity may also be present.
cerning for a postoperative complication following a
laparoscopic pyloromyotomy? What are the clinical criteria and diagnostic
evaluation for the diagnosis of pyloric stenosis?
Pyloric stenosis can be diagnosed using a combination of
Rationale and Evidence-based Practice clinical and physical examination and laboratory findings.
Explanation Projectile vomiting following feeding, with continued rav-
enous appetite, weight loss and dehydration are reported
Based on the presentation and initial laboratory symptoms of PS. A common physical examination finding,
values, what is the most likely diagnosis and two though sometimes difficult to ascertain, is the olive sign,
other differentials? which is a small round palpable mass located in the right
upper quadrant of the abdomen. Although this physical
The most likely diagnosis for this 6-month-old infant is finding may not be noted preoperatively in the awake hun-
pyloric stenosis, a gastric outlet obstruction that typically gry infant when assessed for at the time of surgery, it can be
occurs in young infants between the ages of 4 and 8 weeks palpated once the infant is asleep.
of life and is the most common surgical condition in this age Ultrasound is the gold standard for making the diagnosis
group. Previously thought to be a congenital disorder, of HPS. Ultrasound findings of a pylorus muscle wall thick-
pyloric stenosis occurs with thickening and hypertrophy ness of 0.3 cm or more, and a pyloric channel length of
of the pyloric muscles, causing an obstructive process that 1.5 cm or more is diagnostic for HPS with 100% sensitivity
results in vomiting, often projectile in nature. Pyloric steno- and specificity (Jobson et al., 2016). In addition, there are
sis occurs more frequently in males than in females and can specific measurements that apply to the diagnosis of HPS
result in weight loss, dehydration and electrolyte abnorm- in premature infants. Other radiologic studies may be
alities, especially hypochloremic alkalosis. employed and include an upper gastrointestinal study,
Two other common differential diagnoses include which may be of value if there are concurrent suspicions
milk-protein allergy and gastroesophageal reflux disease for malrotation and GERD. A string sign (elongated and
(GERD). A true milk-protein allergy can result in emesis, thickened pyloric channel), beak sign (filling of the proxi-
but the most common symptom is bloody diarrhea. GERD mal pylorus), or double shoulder sign (thickened pylorus
is also a common finding in young infants, and can be asso- compressing the antrum of the stomach) may be noted
ciated with weight loss and dehydration, but even though when HPS is present.
vomiting can occur, it is usually not projectile in nature.
Neither milk-protein allergy or GERD results in metabolic
What is the treatment for infants with pyloric
alkalosis. If a child with either of these problems has dehy-
stenosis?
dration, metabolic acidosis is the typical presentation.
An early surgical consult is recommended once there is a
What is the physiologic basis for the electrolyte suspicion of HPS. Preoperative care for the infant with
derangements seen in infants with pyloric HPS includes nil by mouth status, intravenous hydration,
stenosis? and correction of electrolyte derangements if present. The
definitive treatment for HPS is operative intervention with
With the progressive vomiting of HPS, infants become a pyloromyotomy. Rammstedt pioneered the procedure,
dehydrated and can present with a hypochloremic, hypoka- which involves the longitudinal extramucosal splitting of
lemic metabolic alkalosis with a paradoxical aciduria. the muscle. A pyloromyotomy can be performed with either
These metabolic derangements used to be seen universally an open technique or via a laparoscopic approach.
in infants with PS; however, with earlier ultrasound diag-
nosis these abnormalities are only seen in a small percent-
Why would anesthesia not be administered to this
age of infants at diagnosis. Persistent emesis results in the
infant with the electrolyte derangements noted at
loss of hydrogen and chloride ions and the pancreas is not
admission?
stimulated to secrete bicarbonate as hydrogen ions are not
reaching the duodenum, with a resultant metabolic alkalo- Even though surgical intervention is the treatment for HPS,
sis (Jacobs et al., 2019). In an effort to retain sodium and this is not an emergent procedure and many infants require
water, the kidney excretes potassium and hydrogen ions stabilization with fluid administration and electrolyte
Downloaded from https://onlinelibrary.wiley.com/doi/ by National Institute Of Standard, Wiley Online Library on [06/03/2024]. See the Terms and Conditions (https://onlinelibrary.wiley.com/terms-and-conditions) on Wiley Online Library for rules of use; OA articles are governed by the applicable Creative Commons License
Chapter 14 Young Infant with Vomiting 59

Table 14.1 Laboratory values. infants with persistent emesis in the postoperative period
or delayed ad lib feeds. Breastfeeding is considered safe
Preoperative values and appropriate and should be encouraged. If infants
Admission values (2 days later) require a regimented feeding plan, the feeds are increased
by minutes with each feeding as tolerated. Often infants
Sodium 140 mEq/L 141 mEq/L
with HPS are on specialized formulas preoperatively due
Potassium 2.6 mEq/L 3.8 mEq/L
to their emesis, but it is not necessary to continue this fol-
Chloride 89 mEq/L 106 mEq/L lowing surgery.
Carbon dioxide 36.3 mEq/L 24.7 mEq/L Postoperative vomiting is common and occurs due to gas-
Glucose 88 mg/dL 89 mg/dL tric distension and atony. Postoperative feeding intolerance
Blood urea nitrogen 20 mg/dL 8 mg/dL with emesis is more commonly seen in infants with severe
Creatinine 0.43 mg/dL 0.28 mg/dL electrolyte derangements.
The majority of infants tolerate ad lib feeds (volume of at
least 60 mL) within 24–48 hours of the operation. Prior to
correction prior to undergoing surgery. Surgery is done discharge it is desirable to have the infant tolerate two 60-
when the child is hydrated and electrolytes have been nor- mL feeds without emesis. Persistent vomiting beyond 72–96
malized. For infants with derangements as severe as in hours postoperatively may occur in cases of incomplete
this case, it can take several days for needed correction, pyloromyotomy and an upper gastrointestinal series may
as severe alkalosis can place the infant at risk for apnea be valuable to determine if this complication is present.
following extubation after general anesthesia. It is desirable
to have a carbon dioxide level less than 30 mEq/L prior to
the surgical procedure. Judicious and daily laboratory mon- What are the signs and symptoms that would be
itoring is appropriate because correction can take several concerning for a postoperative complication
days (see Table 14.1). following a laparoscopic pyloromyotomy?
Infants with PS should be placed on a cardiac and respi-
What feeding regimen should be offered to an
ratory monitor, as subtle trends in vital sign measure-
infant following a pyloromyotomy?
ment can identify significant complications. Gastric
Varying postoperative feeding regimens are used, such as mucosal tears or duodenal perforation are life-
ad lib feeds starting immediately after surgery to 6 hours threatening complications and these infants may have
postoperatively or an incremental feeding plan often tachycardia as an early sign of peritonitis that will
referred to as a “pyloric regimen.” Markel et al. (2017) develop with perforation. A temperature elevation should
reported on a prospective randomized trial comparing an be brought to the provider’s attention because this sign
incremental feeding regimen to ad lib feeds and found may be seen with a mucosal tear. Persistent vomiting
the infants that took feeds ad lib reached goal feeds sooner. beyond 72 hours may be investigated over concern for
The pyloric regimen is more commonly prescribed for an incomplete pyloromyotomy.

References
Jacobs, C., Johnson, K., Khan, F.A., et al. (2019). Life- Markel, T.A., Scott, M.R., Stokes, S.M., et al. (2017). A
threatening electrolyte abnormalities in pyloric stenosis. randomized trial to assess advancement of enteral feedings
Journal of Pediatric Surgery Case Reports 43: 16–18. following surgery for hypertrophic pyloric stenosis. Journal
doi: 10.1016/j.epsc.2019.01.002. of Pediatric Surgery 52: 534–539. doi: 10.1016/j.
Jobson, M. and Hall, N.J. (2016). Contemporary management pedsurg.2016.09.069.
of pyloric stenosis. Seminars in Pediatric Surgery 25:
219–224. doi: 10.1053/j.sempedsurg.2016.05.004.
Downloaded from https://onlinelibrary.wiley.com/doi/ by National Institute Of Standard, Wiley Online Library on [06/03/2024]. See the Terms and Conditions (https://onlinelibrary.wiley.com/terms-and-conditions) on Wiley Online Library for rules of use; OA articles are governed by the applicable Creative Commons License
61

15

School-age Child with Bruising


Lisa Hovingh and Candie Ritsema
Helen DeVos Children’s Hospital, Grand Rapids, MI, USA

A 6-year-old male presents to the pediatric emergency Family History


department (ED) with fever, extensive bruising, and
enlarged lymph nodes to his neck. No history of cancer or blood disease including bleeding or
thrombotic disorders in parents or grandparents. No immune
disorders. He has an 8-year-old sister who is currently healthy.
History of Present Illness

This previously healthy 6-year-old boy developed nasal con- Current Status
gestion and low-grade fevers about 2 weeks prior to presenta-
tion; these symptoms resolved within a few days. About 5 days Vital signs on presentation to ED: temperature 38.1 C
ago, he was noted to have bruising to his lower extremities. (100.6 F), heart rate 124 bpm, respiratory rate 28 breaths
The bruising and petechial rash have worsened over the last per minute, blood pressure 108/66 mmHg, and oxygen
few days. He was also found to have palpable lymph nodes to saturation of 98% on room air. Peripheral blood culture, com-
his neck and axilla. He reports some pain in his legs that the plete blood count (CBC), and comprehensive metabolic panel
family attributed to growing pains. Today, he was febrile to are obtained. Results are shown in Tables 15.1 and 15.2.
39.1 C (102.4 F) at home. He has had no cough, no difficulty
breathing, no bleeding symptoms, no fatigue, and no diffi-
culty sleeping. He has had no known exposure to illness. Questions

Answer the following questions using the details provided.


Past Medical History 1 Based on the data provided, what are the differential
diagnoses for this patient?
He was born at term weighing 3.23 kg and was discharged 2 What is your management plan for this patient?
home with his mother on day of life 2. All immunizations 3 How will you continue to monitor this patient to
are up to date, including annual influenza vaccines. He has determine if this treatment was effective?
a body mass index (BMI) of 14.81. He was diagnosed with a
concussion at 15 months of age following a witnessed fall
off a coffee table. At 4 years, he was seen in the ED with Rationale and Evidence-based Practice
dysphagia, throat pain, and refusal to swallow oral secre-
Explanation
tions after swallowing a quarter. Fiberoptic endoscopic
removal of foreign body was successfully performed.
Based on the data provided, what are the
differential diagnoses for this patient?
Past Surgical History Differential diagnoses for this patient include infection,
inflammation, asplenia, hemolytic disease, myeloproli-
None. ferative disorder, hereditary or chronic neutrophilia, or a

Cases in Pediatric Acute Care: Strengthening Clinical Decision Making, First Edition. Edited by Andrea M. Kline-Tilford and Catherine M. Haut.
© 2020 John Wiley & Sons Ltd. Published 2020 by John Wiley & Sons Ltd.
Downloaded from https://onlinelibrary.wiley.com/doi/ by National Institute Of Standard, Wiley Online Library on [06/03/2024]. See the Terms and Conditions (https://onlinelibrary.wiley.com/terms-and-conditions) on Wiley Online Library for rules of use; OA articles are governed by the applicable Creative Commons License
62 Cases in Pediatric Acute Care

Table 15.1 Basic metabolic profile. and inducing high urine output, decreasing the likelihood
of uric acid or calcium phosphate precipitation in the renal
Sodium 138 mEq/L
tubules. The recommended rate in both children and adults
Potassium 3.9 mEq/L is 2–3 L/m2 per day, or 200 mL/kg per day in children
Chloride 99 mEq/L weighing less than 10 kg.
Carbon dioxide 26 mEq/L With the elevated uric acid on presentation, he will need
Blood urea nitrogen 12 mg/dL both rasburicase and allopurinol. Rasburicase will rapidly
Creatinine 0.48 mg/dL break down serum uric acid that has already accumulated
Glucose 75 mg/dL
by promoting catabolism of uric acid. Recommended intra-
venous dosing of rasburicase is 0.1–0.2 mg/kg over 30 min-
Uric acid 10.4 mg/dL
utes. It is important to ask about the child’s family history
Lactate dehydrogenase >2500 U/L
prior to administration of rasburicase, as it is contraindicated
Phosphorus 5.5 mg/dL in patients with glucose 6-phosphate dehydrogenase defi-
ciency and can cause severe hemolysis with this condition.
Allopurinol will not break down serum uric acid but will
Table 15.2 Complete blood count. decrease the formation of new uric acid. Recommended dos-
ing of allopurinol is 10 mg/kg divided twice daily.
White blood cell count 180 850/mm3 With the risk for worsening electrolyte imbalances
Hemoglobin 9.6 g/dL secondary to tumor lysis syndrome, prevention of hyperka-
Hematocrit 28.9% lemia is imperative. Hydration should not contain potas-
Platelet count 18 000/mm3 sium due to its intracellular release into the circulation
Neutrophils 6% as cells break down. Hyperkalemia is a dangerous
Eosinophils 0% component of tumor lysis syndrome as it can cause sudden
death and cardiac dysrhythmia. An electrocardiogram and
Monocytes 2%
continuous cardiac monitoring, along with frequent labora-
Lymphocytes 15%
tory evaluations, are necessary.
Blasts 77%
Oftentimes, children with leukemia may present with a
mediastinal mass, necessitating a chest radiograph. This
malignant process such as leukemia or lymphoma. The should be obtained prior to any sedation for imaging studies
most likely diagnosis for this child is leukemia with or procedures as the presence of a mediastinal mass will
developing tumor lysis syndrome. Leukocytosis, anemia, increase the risk for airway compromise. Consultation with
thrombocytopenia, and the presence of blasts indicate a oncology is important in determining the diagnosis and
malignant process. Other clinical findings of leukemia treatment plan for the patient’s disease. Further information
include fever, lymphadenopathy, hepatosplenomegaly, regarding the diagnosis can be obtained from pathology
and bone pain. review and flow cytometry of his peripheral blood. A bone
The severity of his electrolyte abnormalities is due to marrow biopsy and aspirate is also needed for additional
tumor lysis syndrome, which can develop before treatment information regarding his type of leukemia and any addi-
is initiated. Tumor lysis syndrome is a group of metabolic tional cytogenetic or molecular information. A lumbar punc-
abnormalities resulting from the rapid release of intracellu- ture will also be necessary to determine if he has leukemia in
lar components. This process can cause hyperuricemia, his central nervous system. Establishing an accurate diagno-
hyperkalemia, hyperphosphatemia, renal injury, arrhyth- sis will determine the best treatment for this patient and
mias, seizures and even death. The immediate need is guide therapy per a protocol or enrollment in a clinical trial.
correction of electrolytes to prevent renal injury in the It is important to initiate therapy as soon as possible to treat
setting of hyperuricemia and hyperphosphatemia. his underlying leukemia. However, treatment will increase
cell turnover, increasing the risk for worsening tumor lysis
syndrome and subsequent kidney injury.
What is your management plan for this patient?
The priority for this patient is hydration, management of
How will you continue to monitor this patient to
his electrolyte abnormalities, and preservation of renal
determine if this treatment was effective?
function. Given the patient’s laboratory values and
extremely elevated white cell count, his tumor lysis will Frequent ongoing assessment of electrolytes including
worsen and close monitoring is imperative. Aggressive monitoring for hyperkalemia, hyperphosphatemia, hyper-
hydration is the cornerstone for preventing renal injury uricemia, hypocalcemia, elevated lactate dehydrogenase,
Downloaded from https://onlinelibrary.wiley.com/doi/ by National Institute Of Standard, Wiley Online Library on [06/03/2024]. See the Terms and Conditions (https://onlinelibrary.wiley.com/terms-and-conditions) on Wiley Online Library for rules of use; OA articles are governed by the applicable Creative Commons License
Chapter 15 School-age Child with Bruising 63

and metabolic acidosis are needed. These abnormalities are Table 15.3 Repeat basic metabolic profile.
the result of excessive cell turnover and breakdown.
Sodium 138 mEq/L
Because of his risk of impaired renal function from hyper-
uricema and hyperkalemia, his urine output, fluid balance, Potassium 4.2 mEq/L
and weight should be closely monitored. He should also Chloride 101 mEq/L
have regular cardiac and neurologic assessments due to Carbon dioxide 23 mEq/L
his associated risks. Blood urea nitrogen 15 mg/dL
Creatinine 0.4 mg/dL
Continuation of case: Diagnostic testing reveals the
following results. Glucose 135 mg/dL
Peripheral blood smear shows severe leukocytosis with Uric acid 1.6 mg/dL
marked predominance of blast-type cells, with very few Lactate dehydrogenase >2500 U/L
background neutrophils and microcytic anemia. There is Phosphorus 4 mg/dL
also severe thrombocytopenia. Peripheral flow cytometry
indicates T-lymphoblastic leukemia (90% blasts).
Chest radiograph shows interval marked enlargement Table 15.4 Repeat complete blood count.
of the cardiothymic silhouette involving both the cardiac
silhouette and hilar adenopathy, enlargement or uplifting White blood cell count 171 000/mm3
of the left pulmonary artery, perihilar interstitial promi- Hemoglobin 9 g/dL
nence, bibasilar atelectasis, small pleural effusions, and Hematocrit 27%
no pneumothorax (Figure 15.1). Platelet count 16 000/mm3
The patient is admitted to the pediatric oncology inpa-
Neutrophils 5%
tient service and continued on intravenous fluids at
Eosinophils 0%
Monocytes 1%
Lymphocytes 17%
Blasts 76%

3 L/m2 daily. He is given one dose of rasburicase and is


started on allopurinol. His diagnosis is confirmed as
T-lymphoblastic leukemia per flow cytometry of peripheral
blood. He is scheduled for bone marrow evaluation and
lumbar puncture to complete his diagnostic work-up
prior to initiating treatment. Repeat laboratory studies
are shown in Tables 15.3 and 15.4.
4 As the uric acid is normal, would you stop allopurinol?

As the uric acid is normal, would you stop


allopurinol?
The child’s uric acid responded well to rasburicase, but due
to ongoing tumor burden anticipate significant cell turno-
ver. This may worsen once chemotherapy is initiated, so
he should continue to receive allopurinol, aggressive hydra-
tion, and frequent monitoring of electrolytes (every 4–6
hours). In order to obtain accurate uric acid results follow-
ing rasburicase, blood must be collected in pre-chilled
tubes, immediately placed on ice, and assay completed in
Figure 15.1 Chest X-ray. Source: Courtesy of Catherine M. Haut, less than 4 hours. Rasburicase will cause enzymatic degra-
CPNP, Herman and Walter Samuelson Children’s Hospital at Sinai, dation of uric acid at room temperature so this is an impor-
Baltimore, MD. tant step to ensure adequate results.
Downloaded from https://onlinelibrary.wiley.com/doi/ by National Institute Of Standard, Wiley Online Library on [06/03/2024]. See the Terms and Conditions (https://onlinelibrary.wiley.com/terms-and-conditions) on Wiley Online Library for rules of use; OA articles are governed by the applicable Creative Commons License
64 Cases in Pediatric Acute Care

Further Reading
Cairo, M.S., Coiffier, B., Reiter, A., et al. (2010). Howard, S., Jones, D., and Pui, C. (2011). The tumor lysis
Recommendations for the evaluation of risk and prophylaxis syndrome. New England Journal of Medicine 364:
of tumour lysis syndrome (TLS) in adults and children with 1844–1854.
malignant diseases: an expert TLS panel consensus. British Wilson, P.F. and Berns, J. (2014). Tumor lysis syndrome: new
Journal of Haematology 149: 578–586. challenges and recent advances. Advances in Chronic Kidney
Coiffier, B., Altman, A., Pui, C., et al. (2008). Guidelines for the Disease 21(1): 18–26.
management of pediatric and adult tumor lysis syndrome: Prescribing information for rasburicase is available online at
an evidence-based review. Journal of Clinical Oncology 26: http://products.sanofi-aventis.us/elitek/elitek.html
2767–2778. (accessed on 4 June 2019).
Engorn, B. and Flerlage, J. (ed.) (2015). The Johns Hopkins
Hospital. The Harriet Lane Handbook, 20th edn.
Philadelphia, PA: Elsevier Saunders.
Downloaded from https://onlinelibrary.wiley.com/doi/ by National Institute Of Standard, Wiley Online Library on [06/03/2024]. See the Terms and Conditions (https://onlinelibrary.wiley.com/terms-and-conditions) on Wiley Online Library for rules of use; OA articles are governed by the applicable Creative Commons License
65

16

Infant with Shunted Hydrocephalus


Mark Weber
Children’s Hospital of Philadelphia, Philadelphia, PA, USA

A 3-month-old twin 35-week gestation male with a past maintenance levetiracetam with good seizure control. His
medical history of shunted hydrocephalus presents to the immunizations are up to date for age.
pediatric intensive care unit (PICU) with fever, erythema,
and swelling at his ventriculoperitoneal (VP) shunt site.
Past Surgical History

A VP shunt was placed at day 6 of life. He required a shunt


History of Present Illness revision roughly 2 months later due to a proximal shunt
obstruction.
The infant was in his usual state of health when 5 days prior
to admission his mother noticed that his head circumfer-
ence appeared to have increased. She also noticed that he Family History
had erythema, swelling, and clear drainage at the incision
site where his VP shunt was revised 15 days prior to pres- There is no significant family history. His twin has
entation. Two days before admission he developed a fever remained healthy.
of 38.6 C (101.5 F). He had no change in his mental status
or decreased oral intake. His mother called the neurosur-
gery clinic and she was advised to bring him to the local Current Status
emergency department (ED) for further evaluation. He
was then transferred to a local children’s hospital for fur- On arrival at the ED, he is hypothermic to 35.7 C (96.3 F).
ther management. He is noted to be mottled so an intravenous catheter is
placed and he is given a 20 mL/kg bolus of 0.9% sodium
chloride. He is started on vancomycin at a dose of 15 mg/kg
Past Medical History every 6 hours and cefepime 50 mg/kg every 8 hours, both
at meningitic dosing. While in ED, he is awake with stable
He was born premature as twin A at 35 weeks’ gestation. mental status. Neurosurgery performs a shunt tap after a
His hydrocephalus, diagnosed on a prenatal ultrasound, 0.05 mg/kg dose of morphine is administered for analgesia.
was due to aqueductal stenosis. He was delivered via Laboratory samples are sent for blood culture, complete
planned cesarean section in a high-risk obstetrics unit blood count (CBC), basic metabolic panel, erythrocyte sed-
and transferred to the neonatal intensive care unit where imentation rate (ESR), C-reactive protein (CRP), and a type
he was admitted for 3 weeks. He was found to have no and screen (see Tables 16.1 and 16.2). The laboratory eval-
genetic abnormalities. He was discharged with a nasogas- uation shows a leukocytosis of 18% on CBC and a CRP of
tric (NG) tube to augment his oral feeds. He has been gain- 17 mg/L (normal range 0–0.9 mg/L). Computed tomogra-
ing weight well on his feeding regimen of oral/NG feeds. He phy (CT) of the brain shows soft tissue swelling and similar
has a seizure disorder that was diagnosed during the time of levels of dilation of his lateral and third ventricles as his
his previous VP shunt malfunction. He has remained on most recent head CT (Figure 16.1).

Cases in Pediatric Acute Care: Strengthening Clinical Decision Making, First Edition. Edited by Andrea M. Kline-Tilford and Catherine M. Haut.
© 2020 John Wiley & Sons Ltd. Published 2020 by John Wiley & Sons Ltd.
Downloaded from https://onlinelibrary.wiley.com/doi/ by National Institute Of Standard, Wiley Online Library on [06/03/2024]. See the Terms and Conditions (https://onlinelibrary.wiley.com/terms-and-conditions) on Wiley Online Library for rules of use; OA articles are governed by the applicable Creative Commons License
66 Cases in Pediatric Acute Care

Table 16.1 Basic metabolic profile. Table 16.3 Cerebrospinal fluid.

Sodium 137 mEq/L Color Xanthochromic


Potassium 4.5 mEq/L
Chloride 109 mEq/L White blood 358/mm3
cell count
Carbon dioxide 17 mEq/L
Red blood cell 1690/mm3
Blood urea nitrogen 13 mg/dL count
Creatinine 0.3 mg/dL Differential Segs 71%
Glucose 86 mg/dL Monocytes 15%
Lymphocytes 13%
Glucose <20 mg/dL (normal range 40–85 mg/dL)
Protein 1165 mg/dL (normal range 14–45 mg/dL)
Table 16.2 Complete blood count.

White blood cell count 23 500/mm3


Hemoglobin 8.6 g/dL Questions
Hematocrit 26.9%
Answer the following questions using the details provided.
Platelet count 655 000/mm3
Neutrophils 66% 1 Based on the data given, what is the differential diagno-
Eosinophils 2% sis and the most likely diagnosis for this child?
Monocytes 8% 2 What therapies are indicated for this child?
3 What laboratory studies will be useful in confirming
Lymphocytes 23%
your diagnosis?
4 What other factors will be important to manage in a
child with an externalized ventricular drain awaiting
shunt re-internalization?

Rationale and Evidence-based


Practice Explanation

Based on the data given, what is the differential


diagnosis and the most likely diagnosis for
this child?
Differential diagnoses for this child would include VP shunt
infection, VP shunt failure/obstruction, peritonitis, local
abscess, seroma or other source of sepsis not yet identified.
The most likely diagnosis for this child would be a VP
shunt infection due to the history of fever, erythema, swell-
ing, and known CSF leakage at the VP shunt insertion site.
A recent (<1 month) surgical intervention on his shunt also
puts him at increased risk of infection. Other common risk
Figure 16.1 Head CT shows soft tissue swelling and similar
dilation of lateral and third ventricles. The shunt remains in good
factors for VP shunt infection include age under 5 years,
condition within the lateral ventricle. two or more previous shunt procedures, prematurity, a
neoplastic etiology for hydrocephalus, and postoperative
CSF leak.
The cerebrospinal fluid (CSF) results (Table 16.3) are
concerning for infection. A preliminary Gram stain of the
What therapies are indicated for this child?
CSF shows Gram-positive cocci in pairs and chains with
a CSF culture pending. He is admitted to PICU for close The mainstays of therapy for a VP shunt infection include
neurologic monitoring and further management. antibiotics and removal of the infected device. The choice
Downloaded from https://onlinelibrary.wiley.com/doi/ by National Institute Of Standard, Wiley Online Library on [06/03/2024]. See the Terms and Conditions (https://onlinelibrary.wiley.com/terms-and-conditions) on Wiley Online Library for rules of use; OA articles are governed by the applicable Creative Commons License
Chapter 16 Infant with Shunted Hydrocephalus 67

of antibiotics can be guided by a history of previous infec- What other factors will be important to manage in
tions and culture results. Gram-positive organisms, includ- a child with an externalized ventricular drain
ing methicillin-resistant Staphylococcus aureus (MRSA), awaiting shunt re-internalization?
can be covered with vancomycin. The recommended dose
While awaiting shunt re-internalization the pending cul-
is 15 mg/kg every 6 hours. This dosing regimen will
tures should be monitored. The growth of any bacteria will
ensure sufficient penetration into the CSF, as vancomycin
help guide the choice and duration of antibiotics. If the
tends to have poor CSF penetration. A vancomycin trough
patient requires a prolonged course of intravenous antibio-
should be checked prior to the fourth dose and target
tics, the placement of a peripherally inserted central cath-
levels between 15 and 20 μg/mL are typical. The patient
eter (PICC) or other means of secure intravenous access
should also be placed on an antibiotic to cover Gram-
should be considered.
negative organisms. If the patient has been frequently
During this period the importance of maintaining the
hospitalized, an antibiotic that would cover Pseudomonas
child’s nutrition should not be forgotten. In smaller chil-
aeruginosa should be used. Either ceftazidime (third-
dren who have a significant amount of CSF drainage from
generation cephalosporin) or cefepime (fourth-generation
their external ventricular drain, serum sodium levels
cephalosporin) would be acceptable choices.
should be followed. The loss of sodium through the CSF
Another organism that should be considered is Cutibac-
can lead to hyponatremia. Hyponatremia can lower the
terium acnes (lipophilic anaerobic Gram-positive bacte-
child’s seizure threshold, making them more prone to sei-
rium; common skin commensal). This organism is
zures. If there is concern for seizure activity, an electroen-
known to be slow-growing and may take up to 10 days to
cephalogram and antiepileptic management would be
grow in culture. Cutibacterium acnes responds well to
indicated. If the serum sodium level is falling, the CSF out-
vancomycin and third-generation cephalosporins. The
put can be replaced with intravenous sodium chloride to
treatment time should be extended to 14 days if this
maintain a normal serum sodium level. Although very rare
bacterium is isolated in culture.
in cases of VP shunt infections, other sodium dysregulation
Device removal should also be considered for infected VP
syndromes may occur, such as syndrome of inappropriate
shunts. Removal of the device and placement of a tempo-
antidiuretic hormone (SIADH), diabetes insipidus, and
rary external ventricular drain will be required until the
cerebral salt wasting.
CSF cultures become clear.
Additionally, since the child’s hospital stay will likely be
greater than 1 week, physical therapy/occupational therapy
What laboratory studies will be useful in
and child life support will be integral in promoting a rapid
confirming your diagnosis?
recovery and successful transition back to home.
Several laboratory studies are important in the diagnosis of
an infected VP shunt. First, Gram stain and culture of the
CSF, preferably from the VP shunt, will be useful in iden-
tifying the causative organism. If possible, the culture Case Resolution
should be drawn from the VP shunt prior to the administra-
tion of antibiotics, although antibiotics should not be The following morning his VP shunt was removed and an
delayed in order to successfully obtain a culture. A cell external ventriculostomy drain was placed in the operating
count, protein and glucose from the CSF can also be helpful room. The cultures from the CSF grew methicillin-sensitive
in revealing a possible infection. CSF with an elevated Staphylococcus aureus (MSSA). He was treated with oxacil-
white blood cell count, elevated protein, and low glucose lin for 2 weeks via a PICC. His CSF cultures were followed
is suggestive of a bacterial infection. A blood culture with and trended for the 2 weeks to assure that the MSSA was
a serum CBC, CRP, and procalcitonin may reveal signs of cleared and that a secondary infection was not present.
an active infection. Additionally, an infectious disease con- At the completion of the 2 weeks a new VP shunt was
sult is recommended as it can be difficult to determine placed and he was discharged to home the following day.
appropriate antibiotic selection and optimal timing of
shunt re-internalization.

Further Reading
Conen, A., Walti, L.N., Merlo, A., et al. (2008). Characteristics associated infections in adults: a retrospective analysis over
and treatment outcome of cerebrospinal fluid shunt- an 11-year period. Clinical Infectious Diseases 47: 73–82.
Downloaded from https://onlinelibrary.wiley.com/doi/ by National Institute Of Standard, Wiley Online Library on [06/03/2024]. See the Terms and Conditions (https://onlinelibrary.wiley.com/terms-and-conditions) on Wiley Online Library for rules of use; OA articles are governed by the applicable Creative Commons License
68 Cases in Pediatric Acute Care

Erps, A., Roth, J., Constantini, S., Lerner-Geva, L., and Tamber, M.S., Klimo, P., Mazzola, C.A. et al. (2014). Pediatric
Grisaru-Soen, G. (2018). Risk factors and epidemiology of hydrocephalus: systematic literature review and evidence-
pediatric ventriculoperitoneal shunt infection. Pediatrics based guidelines. Part 8: Management of cerebrospinal
International 60: 1056–1061. fluid shunt infection. Journal of Neurosurgery Pediatrics 14
Liu, C., Bayer, A., Cosgrove, S.E., et al. (2011). Clinical practice (Suppl 1): 60–71.
guidelines by the Infectious Diseases Society of America for Tunkel, A.R., Hasbun, R., Bhimraj, A. et al. (2017). Infectious
the treatment of methicillin-resistant Staphylococcus aureus Diseases Society of America’s clinical practice guidelines for
infections in adults and children: executive summary. healthcare-associated ventriculitis and meningitis. Clinical
Clinical Infectious Diseases 52: 285–292. Infectious Diseases 64: e34–e65.
Downloaded from https://onlinelibrary.wiley.com/doi/ by National Institute Of Standard, Wiley Online Library on [06/03/2024]. See the Terms and Conditions (https://onlinelibrary.wiley.com/terms-and-conditions) on Wiley Online Library for rules of use; OA articles are governed by the applicable Creative Commons License
69

17

Newborn with Poor Oral Intake and Irritability


Elaine Geary
Nemours/Alfred I. duPont Hospital for Children, Wilmington, DE, USA

A 15-day-old term infant is brought to the emergency Current Status


department (ED) with a 48-hour history of decreased oral
intake and irritability. In the ED, the infant’s electrocardiogram (EKG) demon-
strated a narrow-complex tachycardia at 280 bpm with a
regular rate. The examination demonstrated a blood
History of Present Illness pressure in the right upper extremity of 72/38 mmHg, tach-
ypnea, irritability and pale skin, but no acute distress.
The infant is accompanied by mother. She was seen by her
Pulses were palpable, capillary refill was approximately
primary care provider (PCP) for a well child visit 2 days
3 seconds, and oxygen saturation was 89% on room air.
prior to presentation. A murmur was noted on ausculta-
The cardiology team was called to evaluate the patient.
tion, otherwise the examination was normal. Because of
the heart murmur, the infant was scheduled to see a
cardiologist the following day. The night following the Questions
PCP visit, the infant was fussier and uncharacteristically
hard to settle, acting hungry but eating less. The infant Answer the following questions using the details provided.
vomited the last three feeds.
Prior to presenting, the mother denied tachypnea, poor 1 What is your differential diagnosis?
perfusion, or poor growth. The infant had been taking 2 What would be your interventions to reduce the
57–85 g (2–3 ounces) of Similac Advance every 3–4 hours, heart rate?
enjoyed eating, fed effortlessly and has been gaining 3 What additional diagnostic studies or laboratory studies
weight. The mother denies any recent fever, cough, should be ordered?
congestion, vomiting, diarrhea or ill exposures.
Rationale and Evidence-based Practice
Past Medical History Explanation

What is your differential diagnosis?


Delivered vaginally at term after an unremarkable gestation.
The infant went home with her and had no jaundice, is on no Differential diagnoses include sinus tachycardia, supraven-
medications, and has no allergies. Hepatitis B was adminis- tricular tachycardia including atrioventricular reentrant
tered prior to discharge and baby passed a hearing screen. tachycardia (AVRT) such as WPW syndrome, and atrioven-
tricular nodal reentry tachycardia (AVNRT)

Family and Social History


What would be your interventions to reduce the
heart rate?
Unremarkable. Lives at home with both parents and a
healthy 3-year-old brother. No pets and no smoking in The infant was placed on a nasal cannula delivering oxygen
the home. at 2 L/min. Intravenous (IV) access was initiated and vagal

Cases in Pediatric Acute Care: Strengthening Clinical Decision Making, First Edition. Edited by Andrea M. Kline-Tilford and Catherine M. Haut.
© 2020 John Wiley & Sons Ltd. Published 2020 by John Wiley & Sons Ltd.
Downloaded from https://onlinelibrary.wiley.com/doi/ by National Institute Of Standard, Wiley Online Library on [06/03/2024]. See the Terms and Conditions (https://onlinelibrary.wiley.com/terms-and-conditions) on Wiley Online Library for rules of use; OA articles are governed by the applicable Creative Commons License
70 Cases in Pediatric Acute Care

maneuvers (knees to chest, ice to the face) attempted. Nei- Table 17.2 Complete blood count.
ther maneuver was successful in reducing the heart rate.
White blood cell count 15 000/mm3
Hemoglobin 14.2 g/dL
What additional diagnostic studies or laboratory Hematocrit 39%
studies should be ordered?
Platelet count 481 000/mm3
An EKG is obtained. A basic metabolic panel, complete
blood count, and glucose level were obtained at time of
IV placement (see Tables 17.1 and 17.2). A second dose of adenosine was given at double the orig-
Continuation of case: The next set of vital signs included inal dose, 200 μg/kg rapid IV push followed by an immedi-
temperature 37.2 C (99 F), heart rate 280 bpm, respiratory ate normal saline flush of 10 mL. After the second dose of
rate 72 breaths per minute, oxygen saturation 94% on room adenosine, there was a brief pause in rhythm followed by
air, and blood pressure 68/35 mmHg. Rhythm on 12-lead sinus rhythm at 150 bpm.
EKG (Figure 17.1) was a narrow-complex tachycardia Echocardiogram showed normal systolic and diastolic
indicative of supraventricular tachycardia (SVT). Glucose function, patent foramen ovale (PFO), and peripheral pul-
measured at 70 mg/dL. She was attached to the monitor monic stenosis (PPS).
and defibrillation pads were placed. Her heart murmur was consistent with PPS, a common
innocent murmur of the neonate associated with flow
4 Which medication(s) should be given? acceleration across the branch pulmonary arteries. The
5 What are the features of the EKG that would indi- PFO, a small communication between the upper filling
cate SVT? chambers of the heart, is a remnant of fetal circulation
6 What would have been the course of action had the and is considered an incidental finding. Neither of these
infant quickly decompensated or if the second dose findings would result in cardiovascular symptoms and nei-
of adenosine had not been effective? ther are associated with SVT.
7 Why is it important to have a rhythm strip running
while the adenosine is administered?
8 What is the indication for the echocardiogram? What are the features of the EKG that would
9 How do we know this patient does not have Wolff– indicate SVT?

10
Parkinson–White (WPW) syndrome?
What about secondary prevention? •• Heart rate greater than 220 bpm in an infant
Lack of rate variation, constant R–R interval
11 What is the pathophysiology of SVT?
• Tachycardia unrelated to hydration status, analgesia,
or fever

Which medication(s) should be given? • Sudden change in heart rate with conversion to sinus
rhythm
While a rhythm strip was running, adenosine 100 μg/kg
rapid IV push was given with an immediate rapid normal
saline flush of 10 mL. There was a 2-second pause on the What would have been the course of action had
EKG strip, then SVT resumed at 285 bpm. By this time, the infant quickly decompensated or if the second
the infant’s blood pressure had dropped to 55/30 mmHg, dose of adenosine had not been effective?
her capillary refill was sluggish, and she was getting sleepy.
In unstable patients (e.g. depressed mental status, hypoten-
sion, signs of shock) without IV access, or without response
Table 17.1 Basic metabolic profile. to IV adenosine, the appropriate treatment is immediate
synchronized cardioversion with 0.5–1 J/kg. If this is not
Sodium 134 mEq/L
effective, increase to 2 J/kg. The elimination half-life of
Potassium 3.5 mEq/L adenosine is 10 seconds, and therefore it should be given
Chloride 101 mEql/L in an IV line as close to the heart as possible. If the adeno-
Carbon dioxide 23 mEq/L sine is administered too slowly or with inadequate saline
Blood urea nitrogen 11 mg/dL flush, less of the medication may reach the heart, decreas-
Creatinine 0.2 mg/dL ing its efficacy. For these same reasons, intraosseous
Glucose 72 mg/dL (injecting directly into the marrow of bone) may not be a
reliable route for adenosine administration.
Downloaded from https://onlinelibrary.wiley.com/doi/ by National Institute Of Standard, Wiley Online Library on [06/03/2024]. See the Terms and Conditions (https://onlinelibrary.wiley.com/terms-and-conditions) on Wiley Online Library for rules of use; OA articles are governed by the applicable Creative Commons License
Chapter 17 Newborn with Poor Oral Intake and Irritability 71

Figure 17.1 12-Lead EKG. Source: Courtesy of Dr. Richard Garcia.

Why is it important to have a rhythm strip running it will eventually lead to tachycardia-induced cardiomyop-
while the adenosine is administered? athy, a life-threatening myocardial dysfunction. The second
indication for echocardiography is her murmur. On auscul-
It is vital for diagnosis and medical management of these
tation, the murmur is consistent with PPS but, given that
patients to record the rhythm prior to, during, and after
SVT is more common in children with congenital heart dis-
adenosine administration. These strips will yield informa-
ease, an echocardiogram is warranted to rule out significant
tion on the exact mechanism of SVT when it terminates;
structural heart disease.
this information has implications for both treatment and
prognosis.
How do we know this patient does not have WPW
syndrome?
What is the indication for the echocardiogram? There are no delta waves noted in the EKG after cardiover-
sion, suggesting the accessory pathway conducts only retro-
There are two indications for the echocardiogram in this
grade. This is referred to as a “concealed pathway” because
infant. First, it is not known how long she has been in
the EKG during sinus rhythm appears normal.
SVT. An otherwise healthy infant with a structurally nor-
mal heart can tolerate SVT of rates to 220 bpm for hours
What about secondary prevention?
at a time. Sustained SVT, lasting longer than 6–12 hours,
will typically be associated with signs of congestive heart There are significant practice variations in secondary pre-
failure, specifically irritability, tachypnea, poor feeding, vention in this population. SVT may not be promptly
and pallor. If the tachycardia continues to go unrecognized, detected in infants. They cannot complain of palpitations
Downloaded from https://onlinelibrary.wiley.com/doi/ by National Institute Of Standard, Wiley Online Library on [06/03/2024]. See the Terms and Conditions (https://onlinelibrary.wiley.com/terms-and-conditions) on Wiley Online Library for rules of use; OA articles are governed by the applicable Creative Commons License
72 Cases in Pediatric Acute Care

and they can tolerate high heart rates for a significant not indicated. While she was tachypneic, her respirations
period of time. When they present to medical attention, were unlabored and effective. She was placed on the cardi-
there are generally some signs or symptoms of cardiac fail- orespiratory monitor and defibrillator patches were also
ure. While having caregivers check the heart rate may be applied. This provided the method to record her rhythm
effective, it often adds significant stress and ownership to and enabled synchronized cardioversion if necessary. Vagal
the parents of the infant. For these reasons, many centers maneuvers (glove containing ice applied to the face, above
use medications. Based on the available evidence, propran- the nose and mouth for 15–30 seconds; knees to chest) were
olol is most often used as the initial prophylactic therapy for attempted while the IV was being placed but they were
infants with SVT. unsuccessful. Treatment should not be delayed to adminis-
ter vagal maneuvers in unstable patients.
What is the pathophysiology of SVT? Serum was obtained at the time of IV placement to check
glucose and electrolytes, and to assess for evidence of
SVT is an abnormally rapid heart rhythm originating above
infection.
the ventricles. It is the most common tachyarrhythmia seen
Adenosine has a very short half-life, its effects lasting
in the pediatric age group, with an estimated incidence
approximately 10 seconds. If the first bolus of adenosine
from 1 in 25 000 to as high as 1 in 250. The two most com-
does not convert the SVT, two subsequent boluses of double
mon forms of SVT in children are AVRT, which includes
the initial dose may be administered 2 minutes apart. The
the WPW syndrome, and AVNRT.
patient must be monitored for adverse drug reactions,
A reentrant rhythm involves two distinct pathways for
which may include facial flushing, hypotension, heart
conduction with unidirectional block in one of the two
block, bradycardia, asystole, atrial fibrillation, broncho-
pathways. This creates a circuit through which an electrical
spasm and, in the older patient, chest pressure.
impulse can cycle repetitively in one direction (antegrade
If the SVT fails to respond to adenosine and the patient
or retrograde), with consequent rapid and regular ventric-
remains hemodynamically stable, an electrophysiologist
ular contractions. When the accessory pathway conducts
will prescribe drug therapy depending on the type of
only retrograde it is referred to as a “concealed pathway”
narrow-complex SVT and the patient’s heart function.
because the EKG during sinus rhythm appears normal.
Synchronized cardioversion is the treatment of choice
This is the type of dysrhythmia in this case presentation.
anytime a patient appears compromised.
The hallmark feature of SVT is a very fast rhythm with
In patients who present with SVT in early infancy,
no rate variability.
symptoms often resolve by 1 year of age and therapy
The clinical presentation of this infant is typical, with
beyond that time may not be needed. However, some
reports of atypical fussiness and poor feeding with no asso-
patients may have recurrence later in childhood. By con-
ciated fever or illness. Supplemental oxygen was instituted
trast, SVT tends to persist in children who present after
to support the myocardium and the oxygen saturations
the age of 2 years.
were low at 88% on room air. Tracheal intubation was

Further Reading
Chu, P., Hill, K.D., Clark, R.H., Smith, B., and Hornik, C.P. setting: age-related presentation, diagnosis, and
(2015). Treatment of supraventricular tachycardia in management. Journal of Pediatric Health Care 62(9):
infants: analysis of a large multicenter database. Early 289–299.
Human Development 91(6): 345–350. Wackel, P.L. (2018). Palpitations and arrhythmia. In: Common
Drago, F., Battipaglia, I,, and Di Mambro, C. (2018). Neonatal Cardiac Issues in Pediatrics (ed. J.N. Johnson and D.M.
and pediatric arrhythmias: clinical and electrocardiographic Kamat), 154–156. Itasca, IL: American Academy of
aspects. Cardiac Electrophysiology Clinics 10(2): 397–412. Pediatrics.
Park, M.K. (2014). Park’s Pediatric Cardiology for Practitioners, Zimetbaum, P. (2016). Cardiac arrhythmias with
6th edn, 414–418. Philadelphia, PA: Elsevier Saunders. supraventricular origin. In: Goldman-Cecil Medicine, 25th
Schlechte, E.A., Boramanand, N., and Funk, M. (2008). edn (ed. L. Goldman and A.I. Schafer), 356–367.
Supraventricular tachycardia in the pediatric primary care Philadelphia, PA: Elsevier Saunders.
Downloaded from https://onlinelibrary.wiley.com/doi/ by National Institute Of Standard, Wiley Online Library on [06/03/2024]. See the Terms and Conditions (https://onlinelibrary.wiley.com/terms-and-conditions) on Wiley Online Library for rules of use; OA articles are governed by the applicable Creative Commons License
73

18

An 8-Week-Old Infant with a Cough


Sarah Jane Garraty
Beacon Pediatrics, Rehoboth Beach, DE, USA

An 8-week-old female presents to her pediatrician’s office Past Surgical History


with signs and symptoms of progressively increasing work
of breathing, cough, runny nose, and wheezing. Frenectomy completed in the newborn nursery.

History of Present Illness Family History

This patient is an 8-week-old female who was brought to The infant’s mother has a history of anxiety and depression,
her pediatrician’s office for increasing work of breathing cholestasis, and asthma. She also has a history of cocaine
and wheezing over a 12-hour period. Her symptoms were and tobacco use. Her father had asthma as a child and
preceded by a 4-day history of nasal congestion and cough. her older brother has mild intermittent asthma.
The patient’s mother reports that she has been afebrile, but
has had decreased oral intake, with only one wet diaper in
the past 12 hours. Her older sibling, who is in daycare, has Current Status
had a runny nose and cough for the past week.
On presentation at her pediatric primary care office, she
weighed 3.97 kg and her heart rate was 170 bpm. Upon
inspection, she had intercostal and subcostal retractions
Past Medical History
and accessory muscle use. She had a regular rate and
rhythm on cardiac examination. On pulmonary examina-
The infant was born after a 40-week gestation, via sponta-
tion, her respiratory rate was 80 breaths per minute, and
neous vaginal delivery, with a cleft palate and mild neona-
her oxygen saturation was 92% on room air at rest. Wheez-
tal abstinence, not requiring medical therapy. Maternal
ing, rhonchi, and crackles were auscultated bilaterally on
history includes cocaine and tobacco use during preg-
lung examination.
nancy, and positivity for group B Streptococcus, but ade-
The patient was transported via emergency medical ser-
quately treated prior to delivery. The infant’s early
vices to the local community hospital. During transport, the
neonatal period was significant for tachypnea and poor
infant receives two doses of albuterol, with no significant
feeding. Tachypnea resolved rapidly, but she continued
improvement in her respiratory status.
to require feeding support because of a cleft palate and
tight lingual frenulum. She was observed in the neonatal
intensive care unit (NICU) for 24 hours. No further desa-
Questions
turations were noted during sleep or feedings during her
NICU stay. She was then transferred to the newborn nurs-
Answer the following questions using the details provided.
ery for further observation and feeding support prior to
discharge. 1 What is the differential diagnosis and the most likely
She has recently received 2-month vaccines. diagnosis?

Cases in Pediatric Acute Care: Strengthening Clinical Decision Making, First Edition. Edited by Andrea M. Kline-Tilford and Catherine M. Haut.
© 2020 John Wiley & Sons Ltd. Published 2020 by John Wiley & Sons Ltd.
Downloaded from https://onlinelibrary.wiley.com/doi/ by National Institute Of Standard, Wiley Online Library on [06/03/2024]. See the Terms and Conditions (https://onlinelibrary.wiley.com/terms-and-conditions) on Wiley Online Library for rules of use; OA articles are governed by the applicable Creative Commons License
74 Cases in Pediatric Acute Care

2 What is the pathophysiology of the underlying causing a ventilation–perfusion mismatch, and also
diagnosis? increased work of breathing in an effort to overcome the
3 What are the different management modalities and obstruction.
what is their effectiveness in treating the underlying The clinical findings of this lower airway obstructive dis-
disorder? ease include wheezing, crackles/rales, and signs of
4 Are there any preventive or prophylactic treatment increased respiratory effort (tachypnea, grunting, nasal
modalities? flaring, intercostal retractions, subcostal retractions).
The etiology of bronchiolitis can be summarized as
follows:
Rationale and Evidence-based Practice
Explanation • one-third of hospitalized cases involve two or more
viruses;

What is the differential diagnosis and the most • respiratory syncytial virus (RSV) is involved in 75%
of cases;
likely diagnosis?
•• parainfluenza virus;
influenza virus;

••
The differential diagnoses include bronchiolitis, congenital
heart disease/heart failure, pneumonia, bronchomalacia, adenovirus;
tracheal/vascular rings, and gastroesophageal reflux dis- rhinovirus is usually observed as a co-infection.
ease (GERD).
The most likely diagnosis of this infant is bronchiolitis
What are the different management modalities
due to the infant’s age (less than 12 weeks of age puts the
and what is their effectiveness in treating the
infant at risk for severe disease), reported symptoms of
underlying disorder?
upper respiratory tract infection over the last week, and
adventitious respiratory findings and signs of respiratory Albuterol: Most randomized controlled trials do not pro-
distress that included tachypnea, intercostal and subcostal vide significant evidence to indicate consistent benefit from
retractions, wheezing, and crackles. the use of β-adrenergic agents, such as albuterol, in the
The acute nature of this presentation, without any previ- management and treatment of bronchiolitis. Several
ous history of stridor or wheezing, makes the diagnoses of meta-analyses and systematic reviews have shown that
tracheal/vascular rings and bronchomalacia unlikely. Non- bronchodilators may provide short-term symptom
specific chest radiograph findings, with the presence of improvement but have little impact on overall outcome.
peribronchial cuffing, make bacterial pneumonia improba- Children with severe bronchiolitis or with respiratory
ble as the diagnosis. The diagnosis of heart failure or con- failure were generally excluded from the trials, and there-
genital heart disease is also not supported by the history of fore evidence cannot be generalized to these situations.
weight gain, the acute nature of the presentation, chest Epinephrine: This is an adrenergic agent with both α- and
radiograph findings, improvement on high-flow nasal can- β-receptor agonist activity. It has been used to treat upper
nula, and no murmurs heard on cardiac examination. and lower respiratory tract illnesses and can be adminis-
tered as a nebulized solution. Evidence suggests that epi-
nephrine should not be routinely administered to
What is the pathophysiology of the underlying
children hospitalized for bronchiolitis, except potentially
diagnosis?
as a rescue agent in severe disease.
Bronchiolitis is a disorder that most commonly occurs in Corticosteroids: Although there is sufficient evidence to
children younger than 2 years of age. The disorder is char- indicate effectiveness of corticosteroids in the treatment
acterized in the beginning by symptoms such as rhinorrhea of respiratory diseases such as asthma and croup, there is
and cough, followed by inflammation and infection of the no evidence to indicate clinical effectiveness of corticoster-
lower respiratory tract (bronchioles). Bronchiolitis occurs oid use in the treatment of bronchiolitis.
in the terminal and respiratory bronchioles. Bronchioles High-flow nasal cannula: Heated humidified air and oxy-
are small airways (<2 mm in diameter) and lack cartilage gen is delivered through a nasal cannula (tubes) at higher
and submucosal glands. Bronchiolar injury from infection, flow rates than can be achieved using conventional dry oxy-
and the subsequent inflammatory response, lead to gen delivery. This allows the comfortable delivery of high
increased mucus secretion, with mucous debris causing flow rates of an air/oxygen blend which may improve ven-
bronchial obstruction and constriction, and air trapping tilation. This may lead to a reduced need for invasive respi-
that leads to atelectasis. This results in reduced ventilation, ratory support (e.g. intubation) and may have a clinical
Downloaded from https://onlinelibrary.wiley.com/doi/ by National Institute Of Standard, Wiley Online Library on [06/03/2024]. See the Terms and Conditions (https://onlinelibrary.wiley.com/terms-and-conditions) on Wiley Online Library for rules of use; OA articles are governed by the applicable Creative Commons License
Chapter 18 An 8-Week-Old Infant with a Cough 75

advantage over other treatments by preventing drying of as preterm infants less than 32 weeks 0 days’ gestation who
the upper airway. require greater than 21% oxygen for at least the first 28 days
In one study of 1472 patients, among infants with bron- of life. Clinicians should administer a maximum of five
chiolitis who were treated with oxygen, those who received monthly doses (15 mg/kg per dose) of palivizumab during
high-flow oxygen therapy had significantly lower rates of the RSV season (typically November to March) to infants
escalation of care due to treatment failure than those in who qualify for palivizumab in the first year of life.
the group that received standard oxygen therapy. High-flow
nasal cannula use is becoming widely accepted because
of its relative safety and possible benefits (Franklin
Case Progression and Resolution
et al., 2018).
At the local community hospital, the infant tested positive
Are there any preventive or prophylactic
for RSV and had a chest radiograph completed that indi-
treatment modalities?
cated peribronchial thickening with no evidence of focal
Palivizumab is a monoclonal antibody produced by recom- pneumonia. She was started on heated humidified high-
binant DNA technology. It is used in the prevention of RSV flow nasal cannula therapy (Vapotherm) and following
infections. It is recommended for infants that are high risk intravenous access was started on administration of intra-
because of prematurity or other medical problems such as venous fluids, consisting of a bolus of normal saline
congenital heart disease. Clinicians should not administer 20 mL/kg, followed by dextrose 5% and normal saline at
palivizumab to otherwise healthy infants with a gestational maintenance. Her symptoms improved with these mea-
age of 29 weeks 0 days or greater. sures, and she was transferred to the pediatric intensive
Clinicians should administer palivizumab during the first care unit (PICU) for further care. After 5 days in the PICU,
year of life to infants with hemodynamically significant she was ready for transfer back to the general ward and was
heart disease or chronic lung disease of prematurity defined discharged home on hospital day 9.

Reference
Franklin, D., Babl, F.E., Schlapbach, L.J., et al. (2018). A bronchiolitis. New England Journal of Medicine 378:
randomized trial of high-flow oxygen therapy in infants with 1121–1131.

Further Reading
American Academy of Pediatrics Committee on Infectious Ralston, S.L., Lieberthal, A.S., Meissner, H.C., et al. (2014).
Diseases and Bronchiolitis Guidelines Committee (2014). Clinical practice guideline: The diagnosis, management, and
Updated guidance for palivizumab prophylaxis among prevention of bronchiolitis. Pediatrics 134(5): e1474–e1502.
infants and young children at increased risk of doi: 10.1542/peds.2014-2742.
hospitalization for respiratory syncytial virus infection.
Pediatrics 134(2): 415–420. doi: 10.1086/ahr.113.3.752.
Downloaded from https://onlinelibrary.wiley.com/doi/ by National Institute Of Standard, Wiley Online Library on [06/03/2024]. See the Terms and Conditions (https://onlinelibrary.wiley.com/terms-and-conditions) on Wiley Online Library for rules of use; OA articles are governed by the applicable Creative Commons License
77

19

Desaturation in Postoperative Cardiac Surgery Infant


Lisa Kohr
Cincinnati Children’s Hospital, Cincinnati, OH, USA

History of Present Illness that was weaned off 4 hours ago and a milrinone infusion
that was weaned by 50% 2 hours ago. The plan for that day
This is a 3-day-old, 2.6-kg infant with a postnatal diagnosis was to wean towards extubation. This has been a slow proc-
of total anomalous pulmonary venous drainage who is now ess due to the infant being “sleepy,” so the sedation infu-
postoperative day 3 following complete repair. sions were also weaned. Since he has been awake, he has
experienced intermittent desaturation episodes responsive
Past Medical History to low-dose morphine and temporarily increasing the
FiO2 through the ventilator circuit.
The infant (36 1/7 gestational age) was born via urgent You are called urgently to the bedside because the infant’s
cesarean section due to fetal decelerations. The birthweight oxygen saturation has fallen to 60%. The infant is currently
was 2.54 kg. The Apgar scores were 4 at 1 minute, 6 at 5 min- on an FiO2 of 0.5, ventilator rate of 15 breaths per minute,
utes, and 7 at 10 minutes. The infant was placed on nasal pressure control of 22 cmH2O, positive end-expiratory
continuous positive airway pressure (CPAP) during line pressure (PEEP) of 3 cmH2O, pressure support of 8 cmH2O,
placement; however, the cyanosis continued and the infant and inhaled nitric oxide of 10 ppm.
was intubated and mechanically ventilated. A murmur was The vital signs and hemodynamics reveal heart rate of
noted and the local pediatric cardiac intensive care unit was 189 bpm, respiratory rate 24 breaths per minute, blood pres-
called. An echocardiogram was performed that demon- sure 58/34 mmHg, right atrial pressure (RAP) 10 mmHg,
strated obstructed total anomalous pulmonary venous oxygen saturation (SpO2) 70%, cerebral near-infrared spec-
return to the hepatic veins. He was transported to the inten- troscopy (NIRS) 43%, and regional cerebral infrared spec-
sive care unit, prepared for surgery, and then urgently troscopy (rSO2 NIRS) 65%.
taken to the operating room where he underwent a com- On examination, the infant is pale and cool, with +1
plete repair with ligation of the vertical vein and a suture- peripheral pulses. He is agitated with nasal flaring and
less repair of the pulmonary veins to the left atrium. intercostal retractions. Breath sounds are heard through-
He returned intubated and sedated on dopamine and out. During the exam, the infant suddenly stops moving.
milrinone infusions as well as inhaled nitric oxide 20 ppm. Vital signs are as follows: heart rate 89 bpm, respiratory rate
24 breaths per minute, blood pressure 52/32 mmHg, RAP
12 mmHg, SpO2 55%, cerebral NIRS 32%, rSO2 NIRS 45%.
Family History

No history of heart disease, lung disease, blood diseases, Questions


reactions to anesthesia or unexpected deaths. No ill contacts.
Answer the following questions using the details provided.
Current Status 1 Based on the data, what is your differential diagnosis?
2 What additional studies would be helpful?
During handoff 8 hours after return from the OR, you are 3 What interventions should be implemented?
informed that the infant has been stable all day. The infant 4 What is the pathophysiology of pulmonary hyperten-
returned from the operating room on a dopamine infusion sion crisis?
Cases in Pediatric Acute Care: Strengthening Clinical Decision Making, First Edition. Edited by Andrea M. Kline-Tilford and Catherine M. Haut.
© 2020 John Wiley & Sons Ltd. Published 2020 by John Wiley & Sons Ltd.
Downloaded from https://onlinelibrary.wiley.com/doi/ by National Institute Of Standard, Wiley Online Library on [06/03/2024]. See the Terms and Conditions (https://onlinelibrary.wiley.com/terms-and-conditions) on Wiley Online Library for rules of use; OA articles are governed by the applicable Creative Commons License
78 Cases in Pediatric Acute Care

Rationale and Evidence-based Practice What is the pathophysiology of pulmonary


Explanation hypertension crisis?
Pulmonary hypertension is defined as a mean pulmonary
Based on the data, what is your differential artery pressure (PAP) above 25 mmHg with a normal pul-
diagnosis? monary capillary wedge pressure or a pulmonary artery
The differential diagnoses for this desaturation event pressure greater than half systemic. A pulmonary hyper-
include pulmonary hypertension crisis, obstructed pulmo- tensive crisis occurs when elevated PAP progresses to
nary veins, pulmonary edema, mechanical issues with the the point of causing significant ventricular dysfunction,
ventilator, pulmonary parenchymal issue (e.g. pneumotho- leading to bradycardia and eventually pulseless electrical
rax, effusion), cardiac tamponade, and seizures. activity. There is an increase in left atrial pressure due
Based on the findings during this scenario, the most to the interventricular septum shifting from right to left
likely diagnosis is pulmonary hypertension crisis. as a result of the elevated right ventricular pressure. This
results in acute left ventricular diastolic dysfunction. There
is decreased end-tidal CO2 (found to be 18 mmHg in this
What additional studies would be helpful?
scenario) due to poor gas exchange from limited pulmo-
An arterial blood gas, hemoglobin/hematocrit and lactate is nary blood flow. The diagnosis is dependent on the past
ordered and the results are shown in Tables 19.1 and 19.2. medical history, physical examination, and hemodynamic
A chest X-ray can be considered if there is concern for lung data.
disease, signs of intercurrent illness, or suspected malposi- Risk factors include the diagnosis and weight. Obstructed
tion of endotracheal tube. pulmonary veins cause pulmonary congestion due to the
inability of the pulmonary blood flow to return to the left
What interventions should be implemented? atrium. Initial signs of a pulmonary hypertensive event
include tachycardia, an elevation in RAP due to an increase
The primary goal should be to decrease PVR and support the in right ventricular afterload, hypotension due to decreased
hemodynamics. Bagging the patient with 100% oxygen may cardiac output, and desaturation due to constriction of pul-
increase HR and oxygen saturation as well as simultaneously monary vasculature. Triggers of a pulmonary hypertensive
considering mechanical issues triggering the event. Addi- event include acidosis, agitation, pain, hypothermia, suc-
tional interventions that should occur in rapid succession tioning, and suboptimal ventilation. In this scenario, the
include optimizing mechanical ventilation (e.g. increase infant awoke from sedation and developed agitation, which
ventilatory rate or tidal volume), initiation of an epinephrine may have triggered the event.
infusion, administration of a muscle relaxant agent, escala- While the infant is at risk for experiencing the other diag-
tion of sedation and analgesia, increasing inhaled nitric noses, generally, obstructed pulmonary veins occurs later
oxide to 40 ppm, preparing code medications, and adminis- in the postoperative trajectory. Pulmonary edema may be
tration of sodium bicarbonate, calcium, and a fluid bolus. present, but there is no evidence except for desaturation
and an elevation in RAP. Mechanical issues with the ven-
Table 19.1 Arterial blood gas analysis. tilator and pulmonary parenchymal issues may be present
but would generally be reflected in the ventilatory flow
pH 7.16 loops. Cardiac tamponade and seizures generally occur
PCO2 76 mmHg within the first 24–48 hours after surgery.
PaO2 32 mmHg The treatment plan for a pulmonary hypertensive crisis is
Bicarbonate 14 mEq/L targeted at reducing pulmonary vascular resistance and
Base excess –11 supporting cardiac output. Management for pulmonary
Lactate 8 mmol/L hypertension crisis should be immediate and aggressive
to avoid further cardiac deterioration. The management
Ionized calcium 1 mmol/L
of pulmonary hypertension includes the administration
of pulmonary vasodilators such as oxygen and nitric oxide,
which stimulates cyclic GMP. Sedation, analgesia, and neu-
Table 19.2 Hemoglobin/hematocrit.
romuscular blockade will assist in eradicating the trigger
Hemoglobin 15.3 g/dL for the event. Cardiac output can be augmented with cal-
Hematocrit 45% cium chloride boluses, epinephrine for inotropy, fluid
boluses to augment preload, and sedation/analgesia/muscle
Downloaded from https://onlinelibrary.wiley.com/doi/ by National Institute Of Standard, Wiley Online Library on [06/03/2024]. See the Terms and Conditions (https://onlinelibrary.wiley.com/terms-and-conditions) on Wiley Online Library for rules of use; OA articles are governed by the applicable Creative Commons License
Chapter 19 Desaturation in Postoperative Cardiac Surgery Infant 79

relaxation to decrease stress sympathetic stimulation. Vaso- functional residual capacity (FRC). If the patient continues
active medications are used to avoid systemic hypotension to deteriorate, extracorporeal membrane oxygenation sup-
and support coronary perfusion. Tachycardia should be port should be anticipated.
avoided to allow for adequate fill and ejection of the right
ventricle. The milrinone infusion should be increased
once adequate preload and blood pressure is achieved. Mor-
phine and nitric oxide are given for pulmonary vasodilation. Case Resolution
Sodium bicarbonate is administered to correct acidosis.
Ventilation goals should include maintaining the PEEP Following your interventions, the infant eventually stabi-
and mean airway pressures at lowest levels to provide lizes following aggressive medical management.

References
Kohr, L. and Nieves, J. (2015). Pulmonary hypertension. In: Oishi, P. and Fineman, J.R. (2016). Pulmonary hypertension.
Pediatric Cardiac Intensive Care Handbook (ed. M. Jones, Pediatric Critical Care Medicine 17(8 Suppl 1): S140–S145.
D. Klugman, R. Fitzgerald, et al.), 39–41. Washington, DC:
Pediatric Intensive Care Books.
Downloaded from https://onlinelibrary.wiley.com/doi/ by National Institute Of Standard, Wiley Online Library on [06/03/2024]. See the Terms and Conditions (https://onlinelibrary.wiley.com/terms-and-conditions) on Wiley Online Library for rules of use; OA articles are governed by the applicable Creative Commons License
81

20

Pre-school Child with Seizures


Heather Herrera and Rachel Luthen
Children’s Hospital of San Antonio, San Antonio, TX, USA

A 22-month-old male with a history of hypoxic ischemic complicated by meconium aspiration and hypoxia with
injury at birth with subsequent cerebral palsy and seizure subsequent development of hypoxic ischemic encephalop-
disorder presents with seizures and respiratory distress. athy. He required a 3-week stay in the neonatal intensive
care unit. A neurology consult was completed and he ulti-
mately required initiation of phenobarbital, clonazepam,
History of Present Illness and levetiracetam to achieve seizure control. The child con-
tinued on this regimen with intermittent adjustments for
Approximately 2 days prior to presentation, this child had growth with adequate seizure control until approximately
upper respiratory infection (URI) symptoms of congestion 1 month ago when he developed breakthrough seizures.
and cough without fever. Today, he began to experience He is up to date on his immunizations and he has been
twitching of the right upper extremity every 10 minutes, in close contact with his older sibling who attends daycare
which began mid-morning. Approximately 30 minutes later, and always has URI symptoms.
mother administered home rectal diazepam 1 mg to abort
the seizures. Despite administration of diazepam, he devel-
oped generalized convulsions 15 minutes after administration Past Surgical History
of diazepam and the mother called the emergency medical
services (EMS). EMS providers administered a second dose Circumcised at birth.
of diazepam 1 mg without effect. Upon arrival to the children’s
emergency department, an intravenous catheter was placed Family History
and lorazepam 0.1 mg/kg administered. He received a total
of four doses of lorazepam without successful resolution of Parents are healthy and there is no history of seizures.
the seizures, so he was subsequently given a loading dose of
phenobarbital. It was noted at this time that he had been
having convulsive seizures for 35 minutes. Despite the Current Status
phenobarbital load, he continued in status epilepticus. He
developed respiratory failure with hypotension and required On arrival at the PICU, the patient was having subclinical
endotracheal intubation and a fluid bolus of 20 mL/kg of seizures, noted by the neurologist as non-convulsive refrac-
isotonic fluid. He was then transferred to the pediatric inten- tory status epilepticus on EEG. A propofol infusion is
sive care unit (PICU) where he was immediately placed on initiated with the goal of burst suppression to abort the
continuous electroencephalogram (EEG) monitoring and seizures. During this time, the patient requires two addi-
care was continued. tional boluses of isotonic fluid 20 mL/kg and a norepineph-
rine drip to maintain adequate blood pressure. After several
titrations of the propofol infusion, cessation of status
Past Medical History epilepticus is achieved. A basic metabolic profile and com-
plete blood count are obtained (see Tables 20.1 and 20.2).
He was a term delivery to a 20-year-old mother with an No meningeal signs or history of fever are noted; lumbar
uncomplicated pregnancy. However, his delivery was puncture is not completed. Other results are as follows.

Cases in Pediatric Acute Care: Strengthening Clinical Decision Making, First Edition. Edited by Andrea M. Kline-Tilford and Catherine M. Haut.
© 2020 John Wiley & Sons Ltd. Published 2020 by John Wiley & Sons Ltd.
Downloaded from https://onlinelibrary.wiley.com/doi/ by National Institute Of Standard, Wiley Online Library on [06/03/2024]. See the Terms and Conditions (https://onlinelibrary.wiley.com/terms-and-conditions) on Wiley Online Library for rules of use; OA articles are governed by the applicable Creative Commons License
82 Cases in Pediatric Acute Care

Table 20.1 Basic metabolic profile. considered, including complex febrile seizures, meningitis,
encephalitis, traumatic brain injury, syncope, and meta-
Sodium 139 mEq/L
bolic disorders. With a history of seizures and the need
Potassium 3.8 mEq/L for three antiepileptic drugs (AEDs), the most likely diag-
Chloride 109 mEq/L nosis for this child is status epilepticus secondary to his
Carbon dioxide 21 mEq/L known seizure disorder.
Blood urea nitrogen 12 mg/dL
Creatinine 0.4 mg/dL What are the essential pieces of the initial
Glucose 221 mg/dL
evaluation?
An initial quick physical examination at presentation
should focus and assess respiratory and cardiac status.
Table 20.2 Complete blood count.
An adequate airway should be established if there is
White blood cell count 14 200/mm3 evidence of respiratory compromise and supportive care
Hemoglobin 11.2 g/dL must be initiated. Blood and urine studies should be sent
Hematocrit 34.2%
for serum glucose (as well as point-of-care glucose), serum
electrolytes, blood gas and pH, a complete blood count,
Platelet count 320 000/mm3
urine and blood toxicology studies, and serum AED levels.
An evidence-based review found that almost one-third of

•• Blood culture: pending.


Respiratory viral panel: rhinovirus/enterovirus positive.
children who presented in status epilepticus had drug levels
that were subtherapeutic. These findings can impact both

• Toxicology: levetiracetam level therapeutic, phenobarbi-


tal level subtherapeutic.
management and outcome. Critical laboratory studies
should be completed to identify any electrolyte abnormal-

• Urine toxicology screen: benzodiazepines positive,


remainder negative.
ities and can also reveal metabolic disorders that can pres-
ent with seizures. Key electrolyte abnormalities that can

• EEG (read by neurology): new-onset refractory status epi-


lepticus with persistent generalized periodic epileptiform
lead to seizures are hyponatremia, hypoglycemia, hypo-
magnesemia, and hypocalcemia. Metabolic studies should
discharges (GPED) as well as right periodic lateralizing be considered if there is suspicion of inborn errors of metab-
epileptiform discharges (PLED), then bilateral PLEDs olism and children with these disorders can present with
which have been difficult to suppress (see Figure 20.1). altered mental status and seizures and may have elevated
ammonia levels, acidosis, or hypoglycemia. Many children
with acute seizures will have elevated blood glucose levels
Questions that do not require intervention, but acute hypoglycemia
should be treated promptly (except in those patients who
Answer the following questions using the details provided. are on a ketogenic diet). Brain glucose levels may be low-
ered with continued seizures, and when hypoxemia is also
1 Based on the data described, what are the differential
present brain lactic acidosis and neuronal damage may be
diagnoses and the most likely diagnosis for this child?
worsened.
2 What are the essential pieces of the initial evaluation?
3 What is the best way to treat this problem?
What is the best way to treat this problem?
4 Would you obtain any other diagnostic studies?
5 What is the definition of status epilepticus? The goal of therapy for status epilepticus is rapid termina-
tion of both clinical and electrical seizure activity. Timely
cessation of status epilepticus reduces the associated mor-
bidity and mortality. Appropriate treatment may be hin-
Rationale and Evidence-based Practice
dered by therapies aimed only at reduction in seizures
Explanation rather than cessation, by the use of ineffective therapies
such as sedatives and/or paralytics, and by the administra-
Based on the data described, what are the
tion of inadequate doses of anticonvulsant medications.
differential diagnoses and the most likely
The initial phase for treatment of seizures begins when
diagnosis for this child?
seizure duration reaches 5 minutes. A benzodiazepine is
When a child presents to the emergency department with recommended as the initial therapy of choice. This includes
prolonged seizures, several differential diagnoses must be intravenous lorazepam, intramuscular midazolam, and
Downloaded from https://onlinelibrary.wiley.com/doi/ by National Institute Of Standard, Wiley Online Library on [06/03/2024]. See the Terms and Conditions (https://onlinelibrary.wiley.com/terms-and-conditions) on Wiley Online Library for rules of use; OA articles are governed by the applicable Creative Commons License
Chapter 20 Pre-school Child with Seizures 83

Figure 20.1 EEG. Source: Courtesy of Jennifer Joiner.

intravenous diazepam as these medications have demon- alternative if none of the other recommended second-line
strated efficacy, safety and tolerability. Although intrave- medications are available.
nous phenobarbital is an established medication with The third phase of seizure treatment begins when sei-
good tolerability, it is not recommended as first-line ther- zures have lasted 40 minutes or longer. There is no clear
apy as it requires a slower rate of administration when com- evidence to guide therapy in this phase. When compared
pared to the benzodiazepines. In the pre-hospital setting or with initial therapy, second-line therapy is not as effective,
in situations where the recommended benzodiazepines are just as third-line therapy is also less effective than second-
not available, rectal diazepam, intranasal midazolam, and line therapy. Third-line treatment choices include repeat-
buccal midazolam are reasonable therapy options. Initial ing second-line therapy or using anesthetic doses of
therapy should consist of single, adequate full doses rather thiopental, midazolam, pentobarbital, or propofol, with
than smaller multiple doses. Intravenous lorazepam and continuous EEG monitoring being recommended for all
diazepam can both be repeated at full doses if needed. third-line therapies. A patient is considered to have refrac-
The second phase of therapy begins when seizure dura- tory epilepsy when seizures continue despite first- and sec-
tion reaches 20 minutes and ends with cessation of seizures ond-line treatments, the seizure has lasted longer than
or when seizures last for 40 minutes despite continuation of 1 hour, or general anesthesia is required. Throughout all
treatment. In second-line therapy, fosphenytoin, valproic phases of treatment, until the patient returns to baseline,
acid, and levetiracetam are recommended. There is no evi- supportive symptomatic care should be provided.
dence to support one of these medications being superior to When treating seizures with benzodiazepines and other
the others. Intravenous phenobarbital is also a reasonable AEDs, the most common treatment-related adverse events
Downloaded from https://onlinelibrary.wiley.com/doi/ by National Institute Of Standard, Wiley Online Library on [06/03/2024]. See the Terms and Conditions (https://onlinelibrary.wiley.com/terms-and-conditions) on Wiley Online Library for rules of use; OA articles are governed by the applicable Creative Commons License
84 Cases in Pediatric Acute Care

are respiratory and cardiac depressive symptoms. Cardiac What is the definition of status epilepticus?
monitoring with frequent vital sign checks should be insti-
Brief short seizures are those seizures that last less than
tuted. In most cases, status epilepticus itself does not cause
5 minutes, while prolonged seizures are defined as those
hypotension; however, many of the medications that are
lasting between 5 and 30 minutes. Status epilepticus is
used to treat it do. If hypotension is observed prior to the
defined as more than 30 minutes of either continuous sei-
administration of AEDs, consideration should be given to
zure activity or two or more sequential seizures without full
a possible underlying infectious process, meningitis or sep-
recovery of consciousness between seizures. The 30-minute
sis. The rate of respiratory depression in patients with con-
definition of status epilepticus is based on the duration of
vulsive status epilepticus treated with benzodiazepines is
status epilepticus that may potentially lead to permanent
lower than in patients with convulsive status epilepticus
neuronal injury. Once a seizure lasts more than 5 minutes,
treated with placebo, which indicates that respiratory pro-
it is likely to become prolonged. Seizure treatment proto-
blems are an important consequence of untreated convul-
cols have used a 5-minute definition to minimize the risk
sive status epilepticus. Many children will develop
of seizures reaching 30 minutes and the potential adverse
hypotension or bradycardia secondary to AED therapy
events associated with unnecessarily intervening on brief
and vasopressor and fluid support will be initiated.
self-limiting seizures.
If there is concern for meningitis or other infectious eti-
Status epilepticus can present in several different
ology prompting seizure activity, antibiotics should be
forms: convulsive status epilepticus consisting of repeated
initiated promptly. Antibiotics should not be held until a
generalized tonic–clonic seizures with persistent postictal
lumbar puncture can be performed if there is concern for
depression of neurological function between seizures;
systemic or central nervous system infection.
non-convulsive status epilepticus in which seizures pro-
duce a continuous “epileptic twilight” state; or repeated
Would you obtain any other diagnostic studies?
partial seizures that manifest as focal motor signs, focal
There are no other diagnostic studies indicated for this sensory symptoms, or focal impairment of function that
child at this time. To evaluate for diagnoses beyond an eti- is not associated with altered awareness.
ology of status epilepticus secondary to known seizure dis-
order, a lumbar puncture could be done to determine
meningitis or encephalitis. Neuroimaging is indicated in
children for whom status epilepticus is the first presenta- Case Resolution
tion of epilepsy as well as in those children who do not fol-
low a typical recovery course after treatment for status The toddler had home medications restarted on hospital
epilepticus or if trauma is suspected or known. Computed day 2 with increase in phenobarbital dosing determined
tomography (CT) can be done quickly and should be per- by neurology. Blood cultures were negative at 48 hours
formed in patients with concern for infection to evaluate and the respiratory viral panel was positive for rhinovirus
for an intracranial mass or for increased intracranial pres- infection. Blood pressure normalized once the propofol
sure prior to a lumbar puncture. Magnetic resonance ima- was stopped and he was extubated on hospital day 3. He
ging (MRI) has better yield for determining the underlying was discharged home one week after admission with return
etiology of seizures and can be done once the patient has to baseline mental status.
been stabilized and seizure control has been achieved.

Further Reading
Alford, E.L., Wheless, J.W., and Phelps, S.J. (2015). Treatment in children and adults. Report of the guideline committee of
of generalized convulsive status epilepticus in pediatric the American Epilepsy Society. Epilepsy Currents 16(1):
patients. Journal of Pediatric Pharmacology and 48–61.
Therapeutics 20(4): 260–289. doi: 10.5863/1551-6776- Trinka, E. and Kalviainen, R. (2016). 25 years of advances in
20.4.260. the definition, classification, and treatment of status
Glauser, T., Shinnar, S., Gloss, D., et al. (2016). Evidenced- epilepticus. Seizure 44: 65–73. doi: 10.1016/j.
based guideline: treatment of convulsive status epilepticus seizure.2016.11.001.
Downloaded from https://onlinelibrary.wiley.com/doi/ by National Institute Of Standard, Wiley Online Library on [06/03/2024]. See the Terms and Conditions (https://onlinelibrary.wiley.com/terms-and-conditions) on Wiley Online Library for rules of use; OA articles are governed by the applicable Creative Commons License
85

21

Child with Blood in Stool


Brittany Cook and Darby Tozer
Beaumont Hospital, Troy, MI, USA

A 4-year-old male with a history of constipation presented Current Status


to the emergency department (ED) with concern for blood
in his stool. On arrival at the ED, he is alert and interactive and
appears well. On palpation, his abdomen is soft and
non-tender with active bowel sounds throughout. Stool
History of Present Illness
guaiac test is positive. His rectal examination is without
fissures or hemorrhoids. An abdominal radiograph is
The child had one episode of blood streaks in his stool on the
obtained which is suggestive of a focal ileus as there are
day of presentation. He had also been experiencing intermit-
dilated loops of small bowel in the right lower quadrant
tent abdominal cramping as well as two episodes of non-
with an air–fluid level. The patient tolerates oral fluids
bloody watery stools. No fever, chills, vomiting or appetite
in the ED without complaints of pain or emesis. He is
changes. No cough, nasal congestion, sore throat or rash.
therefore deemed safe for discharge with instructions to
His sibling was recently ill with vomiting. No recent travel.
follow up with his primary care provider (PCP) and a
diagnosis of viral gastroenteritis. Four days later, he
Past Medical History returns to the ED for daily non-bloody episodes of diar-
rhea since discharge. He has also had two episodes of
He was born full-term without complications. He has a his- non-bloody, non-bilious emesis over the past 2 days with
tory of bladder spasms and constipation. His medications new-onset fever on the day of presentation, with a maxi-
include oxybutynin daily for bladder spasms and controls mum temperature of 38.3 C (101 F). He received a
constipation with diet modifications and daily polyethylene weight-appropriate dose of acetaminophen just prior to
glycol. He has never been hospitalized and does not take arrival to ED. Mother states that he was refusing to drink
any other daily medications. His immunizations are up to fluids or eat food on the day of presentation. He was eval-
date for his age, excluding annual influenza vaccine. uated at PCP’s office earlier in the day and sent to the ED
for further evaluation. On arrival at the ED, he is afebrile
with a pulse of 120 bpm, blood pressure 118/96 mmHg,
Past Surgical History respiratory rate of 24 breaths per minute, and pulse oxim-
etry of 97% on room air. He appears pale with dry mucous
Bilateral myringotomy tubes placed at 2 years of age for
membranes, and delayed capillary refill of 3–4 seconds.
recurrent otitis media.
He is noted to have significant pallor. A small bruise is
noted on the right shin. His abdominal examination is
Family History unchanged from his prior visit, revealing a soft, non-
distended abdomen without tenderness or flank pain.
His father has irritable bowel syndrome. His parents deny a A complete metabolic panel, complete blood count, and
family history of Crohn’s disease, colitis, or other gastroin- urine analysis were obtained, with laboratory results as
testinal disorders. noted in Tables 21.1–21.3.

Cases in Pediatric Acute Care: Strengthening Clinical Decision Making, First Edition. Edited by Andrea M. Kline-Tilford and Catherine M. Haut.
© 2020 John Wiley & Sons Ltd. Published 2020 by John Wiley & Sons Ltd.
Downloaded from https://onlinelibrary.wiley.com/doi/ by National Institute Of Standard, Wiley Online Library on [06/03/2024]. See the Terms and Conditions (https://onlinelibrary.wiley.com/terms-and-conditions) on Wiley Online Library for rules of use; OA articles are governed by the applicable Creative Commons License
86 Cases in Pediatric Acute Care

Table 21.1 Complete metabolic profile. 2 Based on the patient’s history and laboratory work,
what is the most likely diagnosis for this child?
Sodium 134 mEq/L
3 What is the most common cause of this condition?
Potassium 6.2 mEq/L 4 What are some other causes of this condition?
Chloride 96 mEq/L 5 What treatments should be initiated in the ED for
Carbon dioxide 13 mEq/L this child?
Blood urea nitrogen 167 mg/dL 6 What additional testing and therapies should be
Creatinine 3.9 mg/dL considered?
Glucose 68 mg/dL
Aspartate aminotransferase 146 U/L
Alanine aminotransferase 52 U/L Rationale and Evidence-based
Bilirubin (total) 1.8 mg/dL Practice Explanation

What differential diagnoses should you consider


Table 21.2 Complete blood count.
for this case?
Differential diagnoses for this child include hemolytic
White blood cell count 9000/mm3
uremic syndrome, infectious colitis, inflammatory
Hemoglobin 6.2 g/dL bowel disease, disseminated intravascular coagulation,
Hematocrit 18.3% and intussusception.
Platelet count 91 000/mm3
Neutrophils 65% Based on the patient’s history and laboratory
Eosinophils 0% work, what is the most likely diagnosis for
Monocytes 13% this child?
Lymphocytes 22% The most likely diagnosis of this patient’s clinical presenta-
Bands 0% tion is shiga-like toxin-producing Escherichia coli (STEC)
hemolytic uremic syndrome (HUS). Shiga-like toxin HUS
is a disorder that results most commonly from an infection
in the gastrointestinal tract that produces toxic substances.
Table 21.3 Urinalysis.
This results in a combination of microangiopathic hemo-
Color Dark yellow lytic anemia, thrombocytopenia, and acute kidney injury.
Clarity Cloudy The child’s laboratory results displayed a triad of anemia,
Bilirubin Positive
thrombocytopenia, and acute kidney injury, with a hemo-
globin of 6.2 g/dL, platelets 91 000/mm3, an elevated blood
Ketone 30 mg/dL
urea nitrogen of 167 mg/dL, and creatinine of 3.9 mg/dL, all
Specific gravity 1.020
consistent with HUS. The patient also exhibited clinical
Blood 3+ manifestations associated with a prodromal illness that is
Protein >300 mg/dL consistent with STEC-HUS, including abdominal pain,
Nitrites Negative vomiting, and bloody diarrhea. Other signs of HUS include
Leukocyte esterase 1+ fever, bruising, low urine output, pallor, and petechiae. In
Red blood cells 3–10 cells per high-power field cases of significant metabolic derangements, encephalopa-
Casts, hyaline >20 cells per high-power field thy or seizures can also be present.
Bacteria 2+
What is the most common cause of this condition?
The most common cause of STEC-HUS is associated with
Questions
gastrointestinal infection with Escherichia coli 0157:H7.
In this case, the patient’s mother reported that 5 days prior
Answer the following questions using the details provided.
to presentation, the patient was at an apple orchard where
1 What differential diagnoses should you consider for he consumed unpasteurized cider, which can harbor E. coli
this case? bacteria. Risk of exposure to infecting agents that can lead
Downloaded from https://onlinelibrary.wiley.com/doi/ by National Institute Of Standard, Wiley Online Library on [06/03/2024]. See the Terms and Conditions (https://onlinelibrary.wiley.com/terms-and-conditions) on Wiley Online Library for rules of use; OA articles are governed by the applicable Creative Commons License
Chapter 21 Child with Blood in Stool 87

to HUS can result from ingestion of raw meat, unpasteur- cultures were obtained to assist in identifying the inciting
ized foods, swimming in contaminated bodies of water, bacterial agent.
and contact with farm animals.

What are some other causes of this condition? What additional testing and therapies
should be considered?
While STEC is the most common cause of pediatric HUS,
other infections are associated with HUS. Streptococcus Supportive care including maintenance of circulating vol-
pneumoniae is a cause of HUS and can be seen in children ume and correction of electrolyte imbalances is a priority
diagnosed with pneumonia, meningitis, sinusitis, and otitis in HUS. Close monitoring and management of blood pres-
media. HUS may also present with coexisting diseases or sure is required. Because of the degree of azotemia, dialysis
conditions, such as transplantation, malignancy, autoim- should be discussed at the receiving facility. Although his-
mune diseases, or kidney disease including malignant torically contraindicated, use of antibiotics remains a con-
hypertension or nephropathy. In the past two decades, an troversial topic, requiring consideration of risks and
additional form of HUS caused by dysregulation of the com- benefits associated with therapy. Nutritional support with
plement pathway, called atypical HUS, has been described. parenteral nutrition due to prolonged colonic inflamma-
tion may be needed, or nutritional support with nasogastric
What treatments should be initiated in the tube feedings if prolonged appetite suppression is present
ED for this child? after functional intestinal recovery is achieved. Prior to
any invasive procedures (e.g. insertion of a hemodialysis
There is no therapy to stop the cascade of HUS; therapy is catheter), the platelet count should be evaluated and the
supportive. The treatments for HUS that would be initiated patient transfused if indicated to reduce the risk of bleed-
in the ED include strict fluid and electrolyte management, ing. Depending on the procedure and associated risk of
blood and/or platelet transfusion, and transfer to the inten- bleeding, a higher baseline platelet count may be desired.
sive care unit or a higher-level care facility. There is evi-
dence to support the role of volume expansion to
enhance outcomes and minimize renal replacement ther-
apy requirements in patients with HUS. This child received Case Resolution
a normal saline bolus of 10 mL/kg while in the ED, using a
conservative volume due to concern for fluid overload from The patient required placement of a dialysis catheter and
associated renal failure. Blood products were held in the ED hemodialysis for 5 days. His urine output, fluid balance,
per the recommendation of the hematology service, who electrolytes, and hematologic laboratory measures steadily
advised that the patient would receive them on arrival to improved over his 12-day PICU hospitalization. He was dis-
the pediatric intensive care unit (PICU). Stool and urine charged home after 18 days of hospitalization.

Further Reading
Ardissino, G., Tel, F., Possenti, I., et al. (2017). Early volume Fakhouri, F., Zuber, J., Bacchi, V.F., et al. (2017). Haemolytic
expansion and outcomes of hemolytic uremic syndrome. uraemic syndrome. Lancet 390(10095): 681–696.
Pediatrics 137(1): e20152153. doi: 10/1542/peds.2015-2153. Grisaru, S., Xie, J., Samuel, S., et al. (2016). Associations
Cody, E.M. and Dixon, B.P. (2019). Hemolytic uremic between hydration status, intravenous administration, and
syndrome. Pediatric Clinics of North America 66(1): outcomes of patients infected with shiga toxin-producing
235–246. Escherichia coli. JAMA Pediatrics 171(1): 68–76.
Downloaded from https://onlinelibrary.wiley.com/doi/ by National Institute Of Standard, Wiley Online Library on [06/03/2024]. See the Terms and Conditions (https://onlinelibrary.wiley.com/terms-and-conditions) on Wiley Online Library for rules of use; OA articles are governed by the applicable Creative Commons License
89

22

Osteomyelitis Case Study


Kim Steanson
Vanderbilt University College of Nursing, Nashville, TN, USA

A 13-year-old boy presents with a 6-day history of fever and portion of his distal tibia. The rest of his physical examina-
increasing pain to his left ankle. tion is unremarkable.
Vital signs in the ED include temperature 37.4 C (99.3 F),
heart rate 86 bpm, respiratory rate 20 breaths per minute,
History of Present Illness
and blood pressure 118/62 mmHg. A complete blood count,
C-reactive protein (CRP), and blood culture are ordered.
This 13-year-old boy has had 7 days of progressively worsen-
A plain film radiograph is obtained of the left ankle. Results
ing left ankle pain with fever. He is experiencing tenderness
of complete blood count are shown in Table 22.1. Other
over the left ankle and inability to bear weight and ambulate.
results are as follows.

••
He has been treating the pain with acetaminophen and
ibuprofen, but the over-the-counter medications are no C-reactive protein: 67 mg/dL.
Blood culture: pending.


longer providing adequate pain relief. He has not had any
ill contacts or recent trauma to the ankle/extremity. He Left ankle plain film radiograph: normal bone minerali-
has had a poor appetite and malaise over the last 3 days. zation; no fracture or focal bony lesion; joint spaces
appear preserved; no appreciable joint effusion.

Past Medical History


Questions
He is a healthy child. All immunizations are up to date
Answer the following questions using the details provided.
including the influenza vaccine this season. No prior hospi-
talizations or surgeries. 1 Based on the data given, what is the differential diagno-
sis and biggest concern that would prompt further treat-
ment and investigation?
Family History 2 What additional imaging would be appropriate
and why?
Paternal grandfather with hypertension. Maternal grand-
3 Which antibiotics should be initiated?
mother with type 2 diabetes mellitus. He lives with his
4 What is the duration of therapy?
mother, father, and two younger brothers.

Rationale and Evidence-based Practice


Current Status
Explanation
On arrival to the emergency department (ED), he is slightly
Based on the data given, what is the differential
irritable but alert and cooperative. He is lying on the bed
diagnosis and biggest concern that would prompt
and is able to move all extremities, but with noticeable gri-
further treatment and investigation?
macing when asked to move his left lower extremity. He
reports significant pain with external and internal rotation Differential diagnoses for this patient includes osteomyeli-
of his left ankle and significant tenderness to the medial tis, septic arthritis, fracture, and toxic synovitis.

Cases in Pediatric Acute Care: Strengthening Clinical Decision Making, First Edition. Edited by Andrea M. Kline-Tilford and Catherine M. Haut.
© 2020 John Wiley & Sons Ltd. Published 2020 by John Wiley & Sons Ltd.
Downloaded from https://onlinelibrary.wiley.com/doi/ by National Institute Of Standard, Wiley Online Library on [06/03/2024]. See the Terms and Conditions (https://onlinelibrary.wiley.com/terms-and-conditions) on Wiley Online Library for rules of use; OA articles are governed by the applicable Creative Commons License
90 Cases in Pediatric Acute Care

Table 22.1 Complete blood count. with osteomyelitis can be difficult to detect in older chil-
dren. Because the bones of infants are less ossified, bone
White blood cell count 13 000/mm3
density changes can be seen more easily. Plain films can
Hemoglobin 11 g/dL show areas of swelling, changes in translucent fat planes,
Hematocrit 34% and in later stages may show the spread of edematous fluid.
Platelet count 232 000/mm3 However, bony destruction is usually not detectable for 2–3
weeks after the onset of symptoms in the child with
osteomyelitis.
Toxic synovitis has a milder presentation than osteomy- MRI has become the imaging modality of choice, with a
elitis or septic arthritis. Patients with toxic synovitis often 90% sensitivity for detecting osteomyelitis. If the child can
do not have concomitant fever. The white blood cell count be evaluated with MRI within hours of the suspected
(WBC) and CRP are typically not elevated and patients diagnosis of osteomyelitis, it is reasonable to proceed first
usually retain the ability to bear weight on the affected with this modality. MRI can accurately demonstrate sub-
extremity. Fracture has been excluded based on the radio- periosteal or soft tissue collections of pus that might
graph findings. The most likely diagnoses for this child are require surgical drainage without the use of ionizing
osteomyelitis and septic arthritis. Osteomyelitis and septic radiation.
arthritis may present alone or together. Both infections
present with pain to the affected joint and limitation of
mobility of the joint. Fever is also a common presenting Which antibiotics should be initiated?
symptom. The WBC and CRP are both elevated. The Gram-positive bacteria are the most common pathogens
combination of these symptoms with the absence of identified in osteomyelitis. Staphylococcus aureus and
significant trauma, which might point to fracture, suggest Streptococcus pyogenes are the most common organisms.
the diagnosis of osteomyelitis. Kingella kingae should be considered in younger chil-
Osteomyelitis is caused most commonly by bacteria and dren under the age of 3 years. In unimmunized patients,
presents with inflammation, subsequently causing destruc- Haemophilus influenzae type B should also be consid-
tion of bone. The bones of the lower limbs are affected in up ered. Empiric antibiotic selection should be targeted
to 90% of infections of children. The tibia is affected in a toward treatment of S. aureus and S. pyogenes in most
large percentage of osteomyelitis involving the lower limbs. cases. Antibiotic therapy can be tailored based on cul-
Osteomyelitis can be contiguously or hematogenously tures, sensitivities, and local area antibiograms. If the
spread. Contiguous spread from infected tissues or open S. aureus isolates are susceptible to methicillin, initial
wounds are more often polymicrobial. Hematogenously therapy may consist of a pencillinase-resistant semisyn-
spread osteomyelitis is typically caused by a single thetic penicillin such as nafcillin or oxacillin. In other
organism. geographic areas where community-acquired methicillin-
The differentiation of osteomyelitis and septic arthritis resistant S. aureus (MRSA) is common, either vancomycin
from toxic synovitis is important as treatment is signifi- or clindamycin should be included in the initial empiric
cantly different. Toxic synovitis is treated with rest and mild therapy.
analgesics. Osteomyelitis requires aggressive treatment
including antibiotics and surgical debridement. A further
complication of osteomyelitis is septic arthritis, which
What is the duration of therapy?
can cause more prominent bone destruction and synovial
joint space compromise. Because bone destruction and Most therapy plans start with intravenous antibiotics and
joint injury can occur quickly, treatment should begin continue until the patient is afebrile for more than 48 hours,
early. Septic arthritis can be a surgical emergency because is able to tolerate oral nutrition and medications, and has
of the rapid destruction that can occur. decreased local signs and symptoms of infection and pain.
The laboratory values should also be normalized before
transitioning to oral antibiotics. The WBC should be nor-
What additional imaging would be appropriate
malized and there should be a significant decrease in the
and why?
CRP. After these benchmarks have been met, antibiotic
Plain films of affected joints are important to obtain and therapy continues with oral medications for 4–6 weeks.
they can provide information about other possible causes Normalization of the CRP can be the marker for discontin-
of symptoms, but changes in bone density that may occur uation of therapy.
Downloaded from https://onlinelibrary.wiley.com/doi/ by National Institute Of Standard, Wiley Online Library on [06/03/2024]. See the Terms and Conditions (https://onlinelibrary.wiley.com/terms-and-conditions) on Wiley Online Library for rules of use; OA articles are governed by the applicable Creative Commons License
Chapter 22 Osteomyelitis Case Study 91

Case Progression and Resolution

The child was admitted to the pediatric surgical floor for


additional evaluation with magnetic resonance imaging
(MRI), antibiotics, and surgical debridement of the left
ankle. MRI of the left ankle (Figure 22.1) exhibited a heter-
ogeneous marrow signal within the distal tibial metadia-
physis with associated subperiosteal collection measuring
approximately 0.6 cm in thickness. There was extensive
edema within the musculature predominantly involving
the posterior deep compartment. There was a focal area
in the flexor digitorum longus that demonstrated disorgan-
ized muscle fibers and a loculated collection suggestive of
osteomyelitis.
In the operating room, a biopsy of the tissue surrounding
the ankle was obtained and copious irrigation and debride-
ment of the area was completed.
After hospital discharge, patients should be followed up
at 1–2 week intervals to monitor continuing clinical
improvement and to monitor for complications with antibi- Figure 22.1 MRI of the left ankle.
otic therapy.

Further Reading
Batchelder, N. and Tsz-Yin, S. (2016). Transitioning Cherry, G.J. Harrison, S.L. Kaplan, W.J. Steinbach, and P.J.
antimicrobials from intravenous to oral in pediatric acute Hotez), 516–528. Philadelphia, PA: Elsevier.
uncomplicated osteomyelitis. World Journal of Clinical Krogstad, P. (2018). Septic arthritis. In: Feigin and Cherry’s
Pediatrics 5(3): 244–250. Textbook of Pediatric Infectious Diseases, 8th edn (ed. J.D.
Keren, R., Shah, S.S., and Srivastava, R. (2015). Comparative Cherry, G.J. Harrison, S.L. Kaplan, W.J. Steinbach, and P.J.
effectiveness of intravenous vs oral antiobiotics for Hotez), 529–534. Philadelphia, PA: Elsevier.
postdischarge treatment of acute osteomyelitis in children. Stone, B., Street, M., Leigh, W., and Crawford, H. (2016).
JAMA Pediatrics 169(2): 120–128. Pediatric tibial osteomyelitis. Journal of Pediatric
Krogstad, P. (2018). Osteomyelitis. In: Feigin and Cherry’s Orthopedics 36(5): 534–540.
Textbook of Pediatric Infectious Diseases, 8th edn (ed. J.D.
Downloaded from https://onlinelibrary.wiley.com/doi/ by National Institute Of Standard, Wiley Online Library on [06/03/2024]. See the Terms and Conditions (https://onlinelibrary.wiley.com/terms-and-conditions) on Wiley Online Library for rules of use; OA articles are governed by the applicable Creative Commons License
93

23

A 2-Year-Old with Liver Transplant and Fever


Louise Flynn
Nemours/Alfred I. duPont Hospital for Children, Wilmington, DE, USA

A 2-year-old girl who received a live donor, left lateral seg- 13 months of age, and exploratory laparotomy and repair
ment, liver transplantation with a Roux anastomosis for of bile duct anastomosis 4 days after transplant.
end-stage liver disease due to biliary atresia at 13 months
of age presents with fever.
Family History
History of Present Illness
The patient lives with her parents and 5-year-old sibling
This 2-year-old girl had fever for one day, which was treated and there is no family history of liver disease. Parents
with acetaminophen at home. This morning she started and grandparents are all alive and healthy.
complaining of abdominal pain and parents had noted
decreased appetite and fatigue.
Current Status
Past Medical History
The child arrives at the emergency department (ED) with
The child was born full term by cesarean section for a 36-hour history of fever and abdominal discomfort.
failure to progress. On day 3 of life she was noted to be There is no history of vomiting or diarrhea, appetite is
jaundiced and was treated with phototherapy for 24 hours. unchanged, and activity level has decreased with increase
Bilirubin rebounded after discontinuation of photo- in temperature. She has received acetaminophen every
therapy and she was seen by the hepatologist for hepato- 4–6 hours, with the last dose 3 hours prior to ED visit.
megaly, jaundice, direct hyperbilirubinemia, and elevated Vital signs on arrival include temperature 38.7 C
gamma-glutamyltransferase (GGT). She was diagnosed (101.7 F), heart rate 138 bpm, respiratory rate 28 breaths
with biliary atresia at 5 weeks of age and a hepato- per minute, and blood pressure 98/63 mmHg.
portoenterostomy (Kasai) procedure was done. She was Blood and urine cultures and chest radiograph obtained
treated for cholangitis with intravenous antibiotics at were all normal. The results of the complete blood count
4 months and 7 months of age and remained on prophylac- and hepatic panel are shown in Tables 23.1 and 23.2.
tic-dose antibiotics until time of transplant. At 13 months of
age, she had a liver transplant from a living donor. Compli-
cations included a bile leak, managed with a peritoneal Questions
drain for 2 months post transplant, and steroid-induced
hypertension, managed with amlodipine. No other medical Answer the following questions using the details provided.
problems were identified to this age.
1 With elevated liver function studies, what is the differ-
ential diagnosis?
Past Surgical History 2 What radiologic studies should be considered?
Surgical procedures for this child include a hepato- Continuation of case: The patient was admitted to the
portoenterostomy at 5 weeks of age, liver transplant at pediatric inpatient unit and intravenous antibiotics were

Cases in Pediatric Acute Care: Strengthening Clinical Decision Making, First Edition. Edited by Andrea M. Kline-Tilford and Catherine M. Haut.
© 2020 John Wiley & Sons Ltd. Published 2020 by John Wiley & Sons Ltd.
Downloaded from https://onlinelibrary.wiley.com/doi/ by National Institute Of Standard, Wiley Online Library on [06/03/2024]. See the Terms and Conditions (https://onlinelibrary.wiley.com/terms-and-conditions) on Wiley Online Library for rules of use; OA articles are governed by the applicable Creative Commons License
94 Cases in Pediatric Acute Care

Table 23.1 Complete blood count. must also be considered with any elevation in liver
enzymes, even with therapeutic levels of immunosuppres-
White blood cell count 6000/mm3
sive medication.
Hemoglobin 10 g/dL
Hematocrit 32%
Platelet count 95 000/mm3
What radiologic studies should be considered?
An abdominal ultrasound with vascular Doppler study
should be considered. The ultrasound may show ductal dil-
Table 23.2 Hepatic panel.
atation, which would suggest a biliary stricture. The Dop-
pler study is diagnostic for patency and flow through the
Total bilirubin 0.4 mg/dL hepatic artery and portal vein. A normal ultrasound does
Direct bilirubin 0.1 mg/dL not preclude further evaluation for biliary strictures. MRCP
Aspartate aminotransferase 386 U/L is used to evaluate for biliary strictures.
Alanine aminotransferase 383 U/L
Albumin 3 mg/dL What further studies/procedures would you
recommend?
Percutaneous transhepatic cholangiogram performed in
started. At 24 hours after starting therapy, she is afebrile interventional radiology is used for both evaluation and
with normal vital signs and the pain has resolved. Repeat treatment of biliary stricture. Once the stricture is identified
laboratory investigations showed improvement in aspartate by cholangiography, treatment with cholangioplasty (with
aminotransferase (249 U/L) and alanine aminotransferase balloon dilatation) and stent placement is done. Strictures
(330 U/L) but GGT (91 U/L) remained elevated. Immuno- are reevaluated every 6–12 weeks until resolved. For
suppressive medication level was in therapeutic range. patients who received a whole liver and had a duct-to-duct
Blood and urine cultures were negative. Abdominal ultra- anastomosis, endoscopic retrograde cholangiopancreato-
sound with vascular Doppler showed no bile duct dilatation graphy (ERCP) can be performed. Surgery to revise anasto-
and patent abdominal vasculature with appropriate direc- mosis may be required if cholangioplasty is unsuccessful.
tion of flow. Magnetic resonance cholangiogram (MRCP)
showed dilatation of the proximal bile duct at the region
of the bile duct anastomosis to the Roux limb. What are the primary concerns for a child post liver
transplant with symptoms of illness?
3 What further studies/procedures would you
recommend? Outcomes in liver transplantation have improved over
4 What are the primary concerns for a child post liver time. Long-term transplant follow-up is aimed at care
transplant with symptoms of illness? guided by the primary provider and gastroenterologist.
Long-term complications can affect 30–50% of liver trans-
plant recipients (Rawal and Yazigi, 2017). Preventive care
focuses on kidney and bone protection, immunosuppres-
sion minimization, vaccination, nutritional support, and
Rationale and Evidence-based Practice
mental health assessment and support. Transplant-related
Explanation problems experienced by children post transplant include
graft rejection, vascular complications, biliary complica-
With elevated liver function studies, what is the
tions, and recurrent disease. They can also develop long-
differential diagnosis?
term gastrointestinal problems such as feeding and motility
The differential diagnoses for this child include acute disorders, cardiovascular problems such as hypertension,
cellular rejection, cholangitis, biliary strictures, and other insulin resistance and obesity. Chronic mental health con-
infectious processes. Fever, elevated liver numbers, and cerns also exist, but children are also at a similar risk as
abdominal pain would support concern for cholangitis. others for development of viral infections. Cautious and
Biliary stricture would be high on the differential list due comprehensive assessment is most important for positive
to the history of bile leak and possible cholangitis. Rejection outcomes.
Downloaded from https://onlinelibrary.wiley.com/doi/ by National Institute Of Standard, Wiley Online Library on [06/03/2024]. See the Terms and Conditions (https://onlinelibrary.wiley.com/terms-and-conditions) on Wiley Online Library for rules of use; OA articles are governed by the applicable Creative Commons License
Chapter 23 A 2-Year-Old with Liver Transplant and Fever 95

Reference Further Reading


Rawal, N. and Yazigi, N. (2017). Pediatric liver transplantation. Dunn, S.P. and Horslen, S. (ed.) (2018). Solid Organ
Pediatric Clinics of North America 64: 677–684. doi: 10.1016/ Transplantation in Infants and Children. Cham,
j.pcl.2017.02.003. Switzerland: Springer.
Downloaded from https://onlinelibrary.wiley.com/doi/ by National Institute Of Standard, Wiley Online Library on [06/03/2024]. See the Terms and Conditions (https://onlinelibrary.wiley.com/terms-and-conditions) on Wiley Online Library for rules of use; OA articles are governed by the applicable Creative Commons License
97

24

Respiratory Decompensation in Trached Infant


Jennifer Helman
C.S. Mott Children’s Hospital, Ann Arbor, MI, USA

A 3-month-old female, admitted to the pediatric intensive operation and reported to be grade 1 view. Patient’s
care unit (PICU) with complex congenital heart disease postoperative course was complicated by need for
and chronic respiratory failure, develops new-onset symp- VA-ECMO, and sternal wound infection for which she is
toms of respiratory distress. receiving 6-week intravenous antibiotic course.

History of Present Illness Past Surgical History

She was admitted to the hospital following birth due to She had an arterial switch operation and VA-ECMO cannu-
complex congenital heart disease including dextrotranspo- lation at day 3 of life. She underwent ECMO decannulation
sition of the great arteries. Patient underwent arterial 6 days later and had subsequent delayed sternal closure.
switch procedure at day 3 of life and required venous– She had tracheostomy completed 4 days ago at 3 months
arterial extracorporeal membrane oxygenation (VA-ECMO) of age.
immediately following procedure for 6 days due to low
cardiac output. Patient has had persistent ventilator
requirement with inability to wean ventilatory support. Family History
Discussion between primary team, pediatric otolaryngology
service, and family resulted in decision to proceed with tra- Her father has a history of asthma. Her mother’s family
cheostomy due to need for long-term mechanical ventila- history is negative.
tion. Tracheostomy was completed 4 days ago by pediatric
otolaryngology, without complication.
Current Status

Past Medical History She is in the PICU and has been hemodynamically stable
following tracheostomy procedure. She is receiving seda-
She was born at 37 weeks with prenatal diagnosis of dextro- tion infusions of midazolam, dexmedetomidine, and mor-
transposition of the great arteries to 30-year-old gravida 1 phine that are being administered through a peripherally
para 1 mother by cesarean section. Apgar scores were 6 inserted central catheter. She is ordered to have bedrest
and 9 following birth. She was initially placed on oxygen and limited movement due to risk for accidental decannu-
per nasal cannula at birth that was transitioned to nasal lation. She has a 3.5 neonatal cuffed tracheostomy tube in
continuous positive airway pressure support at a few hours place. Stay sutures are intact, labeled, and taped to the
of life due to increased work of breathing. She received chest (Figure 24.1). Nurse has been suctioning tracheost-
prostaglandin E infusion until arterial switch procedure omy tube every 2 hours for a small amount of thin white
at day 3 of life. Patient was intubated by anesthesia with secretions. She is on the ventilator in pressure control/
3.0 cuffed oral endotracheal tube for arterial switch assist control mode with rate 30 breaths per minute,

Cases in Pediatric Acute Care: Strengthening Clinical Decision Making, First Edition. Edited by Andrea M. Kline-Tilford and Catherine M. Haut.
© 2020 John Wiley & Sons Ltd. Published 2020 by John Wiley & Sons Ltd.
Downloaded from https://onlinelibrary.wiley.com/doi/ by National Institute Of Standard, Wiley Online Library on [06/03/2024]. See the Terms and Conditions (https://onlinelibrary.wiley.com/terms-and-conditions) on Wiley Online Library for rules of use; OA articles are governed by the applicable Creative Commons License
98 Cases in Pediatric Acute Care

to have increased work of breathing with subcostal retrac-


tions and nasal flaring. She is also noted to have mottling of
her skin. Oxygen saturations are now 85% and the ventila-
tor alarm is sounding.

Questions

Answer the following questions using the details provided.


1 What emergency supplies should be immediately avail-
able for a patient status post tracheostomy?
2 What is the purpose of stay sutures in a postoperative
tracheostomy patient?
Figure 24.1 Patient with stay sutures in place. 3 Based on this information, what are the possible differ-
ential diagnoses for sudden respiratory distress for this
patient?
4 What factors are important to assess during your phys-
ical examination for these potential differential
diagnoses?

Rationale and Evidence-based


Practice Explanation

What emergency supplies should be immediately


available for a patient status post tracheostomy?
Emergency supplies to replace the tracheostomy tube
should always be immediately available to tracheostomy
patients. This includes the same size tracheostomy tube
Figure 24.2 Chest radiography with tracheostomy tube in place. and the size smaller tracheostomy tube in case there is dif-
ficulty placing the same size tracheostomy tube. Other
needed supplies include trach ties, water-based lubricant,
scissors, hemostats, suction catheters, suction apparatus,
positive end-expiratory pressure (PEEP) 10 cmH2O, peak
and bag valve mask. An appropriately sized oral endotra-
inspiratory pressure 32 cmH2O, and FiO2 0.3. She is breath-
cheal tube should also be accessible.
ing comfortably with stable venous blood gases. Oxygen
saturations are currently 99% on the ventilator. Lung
sounds are slightly coarse bilaterally. Figure 24.2 shows
What is the purpose of stay sutures in a
the most recent chest radiograph. She is receiving a milri-
postoperative tracheostomy patient?
none infusion as recent echocardiogram demonstrated
moderately diminished left ventricular function. Emer- Stay suture placement is a surgical technique where non-
gency tracheostomy supply bag is at bedside including absorbable sutures are placed adjacent to the tracheal inci-
same size and size smaller tracheostomy tubes. Pediatric sion at the third and fourth cartilage rings, so that they
otolaryngology is planning to complete the first trach could be pulled to open the stoma and bring the trachea
change in 3 days. closer to the skin if there was accidental dislodgement of
You are called to the bedside by the nurse because of the the tracheostomy tube. These are taped to the chest and
patient’s increased work of breathing and hypoxia, which labeled “right” and “left” so that they do not inadvertently
occurred acutely after she received a bath and the bedsheets get crossed if being used. These sutures are removed at the
were changed. When you arrive at the bedside, she appears time of the first trach change.
Downloaded from https://onlinelibrary.wiley.com/doi/ by National Institute Of Standard, Wiley Online Library on [06/03/2024]. See the Terms and Conditions (https://onlinelibrary.wiley.com/terms-and-conditions) on Wiley Online Library for rules of use; OA articles are governed by the applicable Creative Commons License
Chapter 24 Respiratory Decompensation in Trached Infant 99

Based on this information, what are the possible


differential diagnoses for sudden respiratory
distress for this patient?
Possible major postoperative complications for pediatric
tracheostomy patients can include bleeding, occlusion of
tracheostomy tube most commonly by inspissated secre-
tions, and accidental decannulation. If a child with a tra-
cheostomy develops respiratory distress (typically acute in
nature), it is important to assess for these conditions.
Pneumothorax is also a risk factor with a tracheostomy
procedure. Respiratory and cardiac reasons, and mechan-
ical failure of the ventilator also need to be included as
part of the differential diagnosis of this patient.

What factors are important to assess during your


physical examination for these potential Figure 24.3 Image from tracheobronchoscopy: carina is
differential diagnoses? visualized with right and left mainstem bronchi.

It is important to immediately assess the patient’s neck to


ensure that the tracheostomy tube is in place. Crepitus from of the stoma open. The same size trach tube is inserted with
misdirected air into subcutaneous tissues can be ascer- minimal difficulty and the patient is placed back on the
tained by simple palpation. Suctioning of the tracheostomy ventilator. Trach ties are secured. Oxygen saturations are
tube should also be performed to evaluate for secretions in now 93%. Bilateral breath sounds are confirmed. Pediatric
the tube and easy passage of the suction catheter through otolaryngology completes a bedside tracheobronchoscopy
the tracheostomy tube. After these have been assessed, pro- examination after patient is placed on 100% oxygen and
ceed with the remainder of the physical examination, confirms that the tracheostomy tube is appropriately posi-
including assessment of the heart, auscultation of the chest tioned in the trachea (Figure 24.3). The patient’s color is
to ensure bilateral breath sounds, and checking vent tubing now pink and FiO2 has been quickly decreased to 50% with
from patient to machine. oxygen saturations 98%.

Continuation of case: On physical examination, assess- 5 What maneuvers could be used immediately to help
ment of the tracheostomy site shows that the tracheostomy with ventilation of this patient?
tube is decannulated from the trachea, though the tra- 6 What are the options for securing an airway in this
cheostomy ties remain secured around the neck. The stoma patient?
is slightly open and stay sutures are still intact. Patient 7 Why is it important to confirm that the tracheostomy
is continuing to have increased work of breathing, tube is in place following decannulation in an
with worsening subcostal retractions and cyanosis. Oxygen un-established airway? What methods can be used to
saturations continue to decrease to 70%. The pediatric oto- confirm placement?
laryngology team is paged stat to the bedside. The nurse
What maneuvers could be used immediately to
removes the trach ties so that the neck is visible and there
help with ventilation of this patient?
is access to the stoma. The respiratory therapist begins bag-
mask ventilation of the patient, while the nurse covers the If patients with tracheostomy tube decannulation are not
stoma with gauze to make it occlusive. There is good chest demonstrating adequate respiratory drive, they can receive
rise and oxygen saturations slowly increase to 84%. bag-mask ventilation to help with oxygenation while
Patient’s color is still slightly cyanotic, but slowly improv- attempts are made to replace the trach tube. Patients with-
ing. You begin to prepare for replacing the tracheostomy out upper airway issues that exclude them from being intu-
tube and the pediatric otolaryngology team arrives at the bated or bagged from above can receive bag-mask
bedside. They state that they will now be managing the air- ventilation with the tracheostomy stoma being covered
way for clarity to the providers in the room. They take the with gauze to make it occlusive. If the stoma is not covered,
same size trach tube and hemostats from the patient’s the air will leak out and the patient cannot receive
emergency supply bag. The right and left stay sutures are appropriate ventilation. Certain upper airway anomalies
lifted up from the chest in a V formation to hold the edges or surgical procedures (e.g. surgical history of
Downloaded from https://onlinelibrary.wiley.com/doi/ by National Institute Of Standard, Wiley Online Library on [06/03/2024]. See the Terms and Conditions (https://onlinelibrary.wiley.com/terms-and-conditions) on Wiley Online Library for rules of use; OA articles are governed by the applicable Creative Commons License
100 Cases in Pediatric Acute Care

laryngotracheal separation) do not allow patients to be Why is it important to confirm that the
bagged from above, and would require bagging through tracheostomy tube is in place following
the trach stoma. This involves placing the mask over the decannulation in an un-established airway? What
stoma with a tight seal and bagging the patient to achieve methods can be used to confirm placement?
adequate chest rise. If the patient is stable and has adequate
Appropriate placement of the tracheostomy tube is imper-
respiratory drive, bagging is not necessary while preparing
ative in order to achieve adequate ventilation and to have a
to replace the tracheostomy tube.
secure airway. False passage is a possible risk factor in pla-
cing a tracheostomy tube, especially in a stoma that is not
yet established. Physical examination findings confirming
What are the options for securing an airway in this
that the tracheostomy tube is appropriately placed include
patient?
assessing for bilateral equal breath sounds and symmetric
Foreknowledge of the patient’s airway is important because chest rise with respirations. Presence of humidification of
the team can then determine whether the patient can be the trach tube, and monitoring of end-tidal carbon dioxide
bagged from above in an emergency or whether the patient if available, can also help to demonstrate correct placement.
needs to be bagged through the trach stoma. If the tra- Tracheobronchoscopy allows direct visualization past the
cheostomy tube cannot be replaced, the patient should be distal end of the tracheostomy tube to confirm appropriate
orally intubated as long as there is no upper airway obstruc- positioning in the trachea by visualizing the wall of the tra-
tion that would exclude this as an option. chea, carina, and mainstem bronchi.

Further Reading
D’Souza, J., Levi, J., Park, D., and Shah, U. (2016). Mitchell, R., Hussey, H., Setzen, G., et al. (2013). Clinical
Complications following pediatric tracheotomy. JAMA consensus statement: tracheostomy care. Otolaryngology
Otolaryngology Head and Neck Surgery 142(5): 484–488. Head and Neck Surgery 148(1): 6–20.
Kraft, S. and Schinder, J. (2015). Tracheotomy. In: Cummings
Otolaryngology Head and Neck Surgery, 6th edn
(ed. P. Flint), 95–103. Philadelphia, PA: Saunders.
Downloaded from https://onlinelibrary.wiley.com/doi/ by National Institute Of Standard, Wiley Online Library on [06/03/2024]. See the Terms and Conditions (https://onlinelibrary.wiley.com/terms-and-conditions) on Wiley Online Library for rules of use; OA articles are governed by the applicable Creative Commons License
101

25

Neonatal Hypoglycemia
Janice L. Wilson
University of Maryland School of Nursing, Baltimore, MD, USA

A full-term male infant is delivered by emergency cesarean There is a history of morbid obesity on both sides of the
for fetal distress. family.

Current Status
History of Present Illness
The infant is noted having difficulty latching while at the
At 30 minutes of age, the infant is attempting to breastfeed.
breast and on physical examination he was jittery. Physical
He is tremulous and irritable. His heart rate is slightly ele-
examination reveals a macrosomic large-for-gestational age
vated from previously, but his color is pink and he is alert.
male. Point-of-care glucose and serum glucose are obtained
(Table 25.2).

Past Medical History


Questions
The infant’s mother is a 35-year-old primipara with gesta-
tional diabetes and non-adherence to recommended Answer the following questions using the details provided.
dietary management of glucose control. She has a history
1 What concerning risk factors does this infant exhibit?
of pre-pregnancy obesity, with a 20-kg weight gain during
2 What signs and symptoms might be anticipated?
this pregnancy. Fetal macrosomia was diagnosed in the
3 What do the laboratory results indicate?
second trimester. She was noted to have an abnormal
4 Based on the information available, what is the most
glucose tolerance test at 28 weeks and declined the
likely diagnosis and what physiologic mechanisms
recommended follow-up testing. Results of prenatal
are responsible?
laboratory investigations are shown in Table 25.1.
With advancing cervical dilation, a fetal heart rate drop to Continuation of case: Peripheral intravenous access was
60 bpm was noted, with no return to baseline. An emer- established, a 2 mL/kg bolus of dextrose 10% in water was
gency cesarean section was performed. Apgar scores were given, and the infant was subsequently transferred to the
61 and 85. Resuscitative efforts consisted of bulb suctioning neonatal intensive care unit (NICU). Point-of-care glucose
and stimulation. testing on admission to the NICU was 50 mg/dL. An intra-
Birthweight was 4.375 kg (95th percentile), length 52 cm venous infusion of dextrose 10% in water was ordered, with
(75th percentile), and head circumference 36 cm (50th a glucose infusion rate of 8 mg/kg per min. Ad lib breast-
percentile). feeding was resumed at 6 hours of age when point-of-care
glucose glucose stabilized at 50–60 mg/dL. Over the next
3 days, despite successful breastfeeding, this newborn
was unable to wean below a 5 mg/kg per min glucose
Family History infusion rate.
The maternal grandmother has type 1 insulin-dependent 5 Has the differential diagnosis changed, and if so why?
diabetes, and the paternal grandfather has type 2 diabetes. 6 What additional diagnostic testing might be warranted?
Cases in Pediatric Acute Care: Strengthening Clinical Decision Making, First Edition. Edited by Andrea M. Kline-Tilford and Catherine M. Haut.
© 2020 John Wiley & Sons Ltd. Published 2020 by John Wiley & Sons Ltd.
Downloaded from https://onlinelibrary.wiley.com/doi/ by National Institute Of Standard, Wiley Online Library on [06/03/2024]. See the Terms and Conditions (https://onlinelibrary.wiley.com/terms-and-conditions) on Wiley Online Library for rules of use; OA articles are governed by the applicable Creative Commons License
102 Cases in Pediatric Acute Care

Table 25.1 Prenatal laboratory results. greater at lower glucose concentrations. Low point-of-care
glucose results need to be confirmed or verified by stat
Blood type and Rh A positive
plasma glucose sampling. There is no consensus regarding
Group B streptococci Negative an absolute laboratory value that defines hypoglycemia.
Hepatitis B surface antigen Negative This is due to the fact that the brain response to hypoglyce-
HIV Negative mia may vary with a range of glucose concentrations and it
Herpes simplex virus Unknown is difficult to identify a single glucose concentration that
Syphilis testing Negative can cause brain injury. Additionally, there are artifacts
Gonorrhea testing Negative
and inaccuracies in determining glucose that make it diffi-
cult to interpret plasma glucose levels. Despite the lack of
Chlamydia testing Negative
evidence, however, a plasma glucose level below 47 mg/
dL is often used to define hypoglycemia.

Table 25.2 Glucose test results.

Point-of-care glucose 25 mg/dL


Based on the information available, what is the
most likely diagnosis and what physiologic
Serum glucose 30 mg/dL
mechanisms are responsible?
The most likely diagnosis for this infant is transient hyper-
7 What are the potential long-term sequelae of neonatal insulinemic hypoglycemia or early transitional adaptive
hypoglycemia? hypoglycemia. Fetal insulin production increases with
exposure to high maternal glucose levels. The act of cut-
ting the umbilical cord at delivery will cause the new-
born’s glucose level to fall, while the insulin level
Rationale and Evidence-based Practice remains elevated. As the insulin levels gradually falls over
Explanation the first few days of life, the infant will remain at risk for
hypoglycemia.
What concerning risk factors does this infant
exhibit?
Has the differential diagnosis changed, and if
Risk factors specific for this child include large for gesta-
so why?
tional age, poor maternal glycemic control, fetal distress,
and familial history of diabetes. Other common risk fac- High insulin levels, stimulated by poor maternal glucose
tors for this disorder include small for gestational age, control, should gradually drop over the first 72 hours of life.
intrauterine growth retardation, asphyxia, hypothermia, As insulin levels fall, glucose levels should begin to normal-
sepsis, polycythemia, and prematurity. Less common ize. Symptomatic hypoglycemia, the need for intravenous
disorders associated with this condition include Beck- dextrose to maintain normal plasma glucose levels, pre-
with–Wiedemann syndrome, hypothyroidism, panhypopi- feeding plasma glucose levels below 50 mg/dL at 48 hours
tuitarism, growth hormone deficiency, cortisol deficiency, of age, and pre-feeding plasma glucose levels below 60 mg/
and inborn errors of metabolism. dL after 48 hours of age require further evaluation for a
persistent hypoglycemia disorder or severe recurrent
What signs and symptoms might be anticipated? hypoglycemia.
A “critical blood sample” should be obtained when the
Symptoms can be subtle and may mimic other disease
plasma glucose falls below 40–50 mg/dL in infants who
states. Symptoms include jitteriness, tachycardia, pallor,
continue to have persistent hypoglycemia. This critical
hypothermia, lethargy, irritability, poor feeding, cyanosis,
blood sample should include measurement of plasma glu-
tachypnea, apnea, weak or high-pitched cry, poor tone,
cose, insulin, cortisol, growth hormone, β-hydroxybutyrate,
and seizure-like behavior.
and free fatty acid levels (see Table 25.3). After the critical
sampling is completed and depending on institution proto-
What do the laboratory results indicate?
col, a glucagon stimulation challenge may then be per-
It is critical to understand that point-of-care glucose analy- formed. An increase in plasma glucose greater than
zers can vary by as much as 10–20 mg/dL when compared 30 mg/dL after glucagon administration is an indication
with plasma glucose levels. This variation may be even of hyperinsulinism.
Downloaded from https://onlinelibrary.wiley.com/doi/ by National Institute Of Standard, Wiley Online Library on [06/03/2024]. See the Terms and Conditions (https://onlinelibrary.wiley.com/terms-and-conditions) on Wiley Online Library for rules of use; OA articles are governed by the applicable Creative Commons License
Chapter 25 Neonatal Hypoglycemia 103

Table 25.3 Blood sampling rationale.

Critical blood sampling/testing Rationale

Plasma glucose Identification of hypoglycemia


Insulin Inappropriately high level compared to plasma glucose (hyperinsulinism)
Cortisol Evaluates function/integrity of hypothalamic–pituitary–adrenal axis (low cortisol levels
may indicate hypopituitarism)
β-Hydroxybutyrate Decreased levels may indicate excessive insulin action (even when insulin level is not
significantly elevated)
Free fatty acids Decreased levels may indicate excessive insulin action (even when insulin level is not
significantly elevated)
Growth hormone Evaluates function/integrity of hypothalamic–pituitary–adrenal axis (low growth
hormone levels may indicate hypopituitarism)
Glucagon stimulation challenge Increase in plasma glucose ≥30 mg/dL indicates hyperinsulinism
(administer 1 mg glucagon)

What additional diagnostic testing might be What are the potential long-term sequelae of
warranted? neonatal hypoglycemia?
Additional testing may be needed to evaluate for defects in Glucose is the primary metabolic fuel for the brain and
carbohydrate metabolism, defects in amino acid metabo- drives hepatic glucose production, though the mechanisms
lism as well as other endocrine deficiencies. Genetic muta- responsible are unknown. When hypoglycemia is severe or
tions in genes responsible for insulin regulation can also prolonged, infants are at risk for significant irreversible
cause hyperinsulinemic hypoglycemia and testing can neurologic sequelae, including developmental delay, learn-
identify specific genes of concern. Beckwith–Weidemann ing and behavior problems, hyperactivity, attention deficit
syndrome is the most common syndrome associated with challenges, autism, microcephaly, and cortical blindness.
hyperinsulinism.

Further Reading
Adamkin, D.H. (2011). Clinical report: postnatal glucose Gorman, T. (2017). Neonatal effects of maternal diabetes. In:
homeostasis in late pre-term and term infants. Pediatrics 127 Cloherty and Stark’s Manual of Neonatal Care, 8th edn (ed.
(3): 575–579. E.C. Eichenwald, A.R. Hansen, C.R. Martin, and A.R. Stark),
Blake, S.M., Jnah, A.J., and Adkins, D.W. (2019). The 910–922. Philadelphia, PA: Wolters Kluwer.
metabolic and endocrine system. In: Fetal and Neonatal Long, D. and Akhtar, Y. (2019). Hyperinsulinism. Pediatrics in
Physiology for the Advanced Practice Nurse (ed. A.J. Jnah and Review 40(4): 207–210.
A.N. Trembath), 307–348. New York: Springer. Lord, K. and De Leon, D.D. (2018). Hyperinsulinism in the
Burris, H.H. (2017). Hypoglycemia and hyperglycemia: In: neonate. Clinics in Perinatology 45(1): 61–74.
Cloherty and Stark’s Manual of Neonatal Care, 8th edn (ed. Perlman, J.M. and Volpe, J.J. (2019). Glucose. In: Volpe’s
E.C. Eichenwald, A.R. Hansen, C.R. Martin, and A.R. Stark), Neurology of the Newborn, 6th edn (ed. J.J. Volpe, T.E. Inder,
312–325. Philadelphia, PA: Wolters Kluwer. B.T. Darras, et al.), 701–729. New York: Elsevier.
Devaskar, S.U. and Garg, M. (2015). Disorders of carbohydrate Thornton, P.S., Stanley, C.A., De Leon, D.D., et al. (2015).
metabolism in the neonate. In: Fanaroff and Martin’s Recommendations from the Pediatric Endocrine Society for
Neonatal–Perinatal Medicine, Diseases of the Fetus and evaluation and management of persistent hypoglycemia in
Infant, 10th edn (ed. R.J. Martin, A.A. Fanaroff, and M.C. neonates, infants, and children. Journal of Pediatrics 167(2):
Walsh), 1434–1459. Philadelphia, PA: Elsevier. 238–245.
Downloaded from https://onlinelibrary.wiley.com/doi/ by National Institute Of Standard, Wiley Online Library on [06/03/2024]. See the Terms and Conditions (https://onlinelibrary.wiley.com/terms-and-conditions) on Wiley Online Library for rules of use; OA articles are governed by the applicable Creative Commons License
105

26

Chronic Otitis Media


Kimberly Young-Conner
Nemours/Alfred I. duPont Hospital for Children, Wilmington, DE, USA

A 3-year-old male is being followed by his primary care decision, so sought the services of a general otolaryngolo-
provider for frequent episodes of otitis media. gist who did schedule tube placement at that time. At
18 months of age, one of the PE tubes was displaced and
the child had another episode of OM. Returning to the oto-
History of Present Illness laryngologist resulted in another surgical procedure for
adenoidectomy. The child has a history of chronic nasal
This 3-year-old boy is being followed by the primary care congestion and has been treated with albuterol twice for
practice for recurrent otitis media (OM) after bilateral wheezing.
tympanostomy tube (pressure-equalizing or PE tubes)
placement at 8 months of age. He has recently developed
cold symptoms and the parents are concerned that he
may have an ear infection. Past Surgical History

This child had tympanostomy tubes placed at age 7 months,


under general anesthesia. Approximately one year later, at
Past Medical History
age 18 months, he had an adenoidectomy and replacement
of one of the PE tubes.
The child was born at 38 weeks’ gestational age by vaginal
delivery, with maternal prenatal care and no complications
at birth. He lives with mother, father, and one older sister.
He attends a home-based daycare during the school year, Family History
when his mother works as a teacher. This patient is an over-
all healthy child whose growth and development have been No chronic illness has been identified in any family
on target, although there was initial concern at 15 months members. Parents are both healthy. Older sibling is healthy,
of age about his speech and language. He is currently in the but also had a history of OM with PE tubes placed at age
40% for weight and 50% for height with steady growth noted 1 year. The family resides in a single-family home without
on charts. He was diagnosed with a first episode of bilateral pets or smokers.
OM at 6 months of age and was treated with amoxicillin. He
did not receive the full course of amoxicillin and 2 weeks
later presented with continued findings of OM, so was trea-
ted with cefdinir with resolution. Parents consulted an oto- Current Status
laryngologist at that time and were told to watch and see
what happens. One month later the child developed bilat- In the primary care office today, this 3-year-old child is
eral OM and was treated with ceftriaxone intramuscularly noted to be happy, with clear speech and gross and fine
for 3 days. Parents again reached out to the otolaryngologist motor skills at age level. His physical examination is
who wanted to wait for full resolution of illness before normal, with one PE tube still in place to the left ear, but
another office visit. Parents were not happy with this the right one is missing.

Cases in Pediatric Acute Care: Strengthening Clinical Decision Making, First Edition. Edited by Andrea M. Kline-Tilford and Catherine M. Haut.
© 2020 John Wiley & Sons Ltd. Published 2020 by John Wiley & Sons Ltd.
Downloaded from https://onlinelibrary.wiley.com/doi/ by National Institute Of Standard, Wiley Online Library on [06/03/2024]. See the Terms and Conditions (https://onlinelibrary.wiley.com/terms-and-conditions) on Wiley Online Library for rules of use; OA articles are governed by the applicable Creative Commons License
106 Cases in Pediatric Acute Care

Questions The recommendation is for providers to offer tympanost-


omy tubes for children who have had three episodes of
Answer the following questions using the details provided. OM in 6 months or four episodes in one year, with one
occurring in the previous 6 months (Lieberthal et al.,
1 What concerns exist regarding recurrent OM diagnosed 2013). The benefits of PE tube placement include
in an infant or young child? decreased opportunity to develop OM, with venting of
2 When is tympanostomy tube placement recommended? the tympanic membrane allowing any fluid accumulation
What is the benefit? to drain. Clearing fluid from the middle ear assists in
3 What are the risks of the procedure? allowing a child to hear more consistently, and also allows
4 What outcomes should be expected? the process of language development.
5 What further care or consults should this child receive?

What are the risks of the procedure?


Placement of PE or tympanostomy tubes is accomplished
Rationale and Evidence-based Practice
by creating a small incision in the tympanic membrane
Explanation and placing a ventilation tube in that area. The procedure
is done under general anesthesia, which poses risks to any
What concerns exist regarding recurrent OM
child but especially to young infants. Complications of PE
diagnosed in an infant or young child?
tube placement include otorrhea, tympanosclerosis, and
Acute OM affects approximately 25% of children in the first permanent perforation. PE tubes will stay in place for var-
year of life and more than 50% of children by the age of ied amounts of time, and shorter dwelling times may result
5 years. Factors associated with the development of OM in replacement of the tube. Development of a cholestea-
include attendance at daycare, bottle feeding versus breast- toma and medial migration of the PE tube to the middle
feeding, atopy, and vaccine refusal, especially pneumococ- ear space are two complications which are rare, but pose
cal and influenza vaccines. There are also other factors, more serious problems. Both of these conditions most often
including anatomic problems such as cleft palate and require surgical intervention.
genetic conditions such as Down syndrome. The main con-
cern regarding recurrent or persistent OM is primarily What outcomes should be expected?
speech and language problems due to the conductive hear-
ing loss that occurs with constant or frequent episodes of Children who meet criteria for PE tube placement have
fluid in the eustachian tube, especially in young infants overall good outcomes following the procedure. Despite
and children. Other documented problems in children with some of these children requiring speech and language refer-
OM include pain, decreased appetite, sleep disturbances, rals, it is noted that most children continue to do well with
and behavioral problems. Perforation of the tympanic improved hearing after tube placement. Quality of life was
membrane is another complication of OM, and can result evaluated in a study of 1208 children between the ages of 6
in conductive hearing loss if there is significant damage and 24 months of age, with higher quality of life noted for
and/or scarring of the tympanic membrane. Other pro- those who had PE tubes placed as compared to those who
blems include learning difficulties when hearing is were referred and waiting for the procedure (Grindler
impaired, even temporarily. Another issue with recurrent et al., 2014).
OM is frequent antibiotic use, which can result in allergic
reactions, development of resistant organisms, as well as What further care or consults should this child
the rare potential for fungal infection. Development of receive?
OM early in life also increases the risk of recurrent OM later
This child is doing well currently so should not need follow-
on in childhood.
up by the otolaryngologist at this point, although if the PE
tube remains in place it will need to be removed. The time
When is tympanostomy tube placement
frame for surgical removal is typically 3 years from place-
recommended? What is the benefit?
ment. This child was seen by the otolaryngologist too soon
The American Academy of Pediatrics published clinical prac- during infancy, and had tubes placed prior to reaching the
tice guidelines in 2013 that outline evidence-based recom- recommended number of OM episodes, which did place the
mended care and consults for children with recurrent OM. child at higher risk of at least anesthesia complications.
Downloaded from https://onlinelibrary.wiley.com/doi/ by National Institute Of Standard, Wiley Online Library on [06/03/2024]. See the Terms and Conditions (https://onlinelibrary.wiley.com/terms-and-conditions) on Wiley Online Library for rules of use; OA articles are governed by the applicable Creative Commons License
Chapter 26 Chronic Otitis Media 107

References
Grindler, D.J., Blank, S.J., Schulz, K.A., Witsell, D.L., and Lieu, Lieberthal, A.S., Carroll, A.E., Chonmaitree, T., et al. (2013).
J.E. (2014). Impact of otitis media severity on childrens’ The diagnosis and management of acute otitis media.
quality of life. Pediatric Otolaryngology 151(2): 333–340. doi: Pediatrics 131(3): e964–e999. doi: 10.1542/
10.1177/0194599814525576. peds.2012-3488.

Further Reading
Pichichero, M.E. (2018). Helping children with hearing loss
from otitis media with effusion. Lancet 392(10147): 533–534.
Downloaded from https://onlinelibrary.wiley.com/doi/ by National Institute Of Standard, Wiley Online Library on [06/03/2024]. See the Terms and Conditions (https://onlinelibrary.wiley.com/terms-and-conditions) on Wiley Online Library for rules of use; OA articles are governed by the applicable Creative Commons License
109

27

Complex Patient with Recurrent Lower Respiratory Tract Infection


Ann Marie Ramsey
C.S. Mott Children’s Hospital, Ann Arbor, MI, USA

A 5-year-old female born with myelomeningocele at the


L3–6 level and Chiari malformation with shunted hydro-
• Flovent (fluticasone propionate 44 μg) two puffs
twice daily;
cephalus and acquired central hypoventilation requires tra-
cheotomy and ventilator support during all times of sleep.
• tobramycin 300 mg nebulizer mist treatment twice daily,
28 days on and 28 days off;
She is admitted to the pediatric intensive care unit (PICU)
with acute-on-chronic respiratory failure in the setting of
•• loratadine 5 mg daily;
ranitidine 84.5 mg twice daily.
an acute febrile illness.
Immunizations are up to date for age. She has had
the 2019 seasonal influenza vaccine. Received the poly-
History of Present Illness saccharide pneumococcal vaccine in 2016 (next dose
due 2021).
This patient was in her baseline state of health until 3 days The patient also has allergies to latex and levofloxacin
prior to admission when she developed oxygen desatura- (adverse reaction includes severe nausea and vomiting).
tions at night requiring 1–2 L of supplemental oxygen along
with usual ventilation. She also developed thick yellow tra-
cheostomy secretions and fever to 38.3 C (101 F) axillary. Family History
Mother contacted the ventilator team nurse practitioner
who instructed mother to begin inhaled tobramycin 300 Both parents healthy, no siblings. Father with seasonal
mg nebulized twice daily and increase albuterol 2.5 mg neb- allergies.
ulized treatments to every 4 hours followed by oscillatory
vest and cough-assist therapies. Mother was also instructed
to provide additional ventilator support during the day. The Social History
patient received her first dose of tobramycin that afternoon.
Despite this treatment, she continued to have fever to Lives with both parents in a single-family home. Attends
38.9 C (102 F) and developed spasmodic coughing with all-day school program in local elementary school 5 days
post-tussive emesis. Mother again contacted the ventilator weekly. Has a nurse in school.
team staff who recommended evaluation in the emergency
department (ED).
Current Status

Past Medical History On arrival at the ED the child has a temperature of 39 C


(102.2 F), spontaneous respiratory rate of 40 breaths per
The patient is taking the following medications at home: minute, heart rate of 140 bpm, and oxygen saturation of
86% on room air. She has intercostal and substernal retrac-

• albuterol 2.5 mg nebulizer mist treatment twice daily


when well, every 4 hours when sick;
tions and is placed on the Trilogy ventilator with home set-
tings: pressure control/assured volume average pressure

Cases in Pediatric Acute Care: Strengthening Clinical Decision Making, First Edition. Edited by Andrea M. Kline-Tilford and Catherine M. Haut.
© 2020 John Wiley & Sons Ltd. Published 2020 by John Wiley & Sons Ltd.
Downloaded from https://onlinelibrary.wiley.com/doi/ by National Institute Of Standard, Wiley Online Library on [06/03/2024]. See the Terms and Conditions (https://onlinelibrary.wiley.com/terms-and-conditions) on Wiley Online Library for rules of use; OA articles are governed by the applicable Creative Commons License
110 Cases in Pediatric Acute Care

Table 27.1 Complete blood count.

White blood cell count 1980/mm3


Hemoglobin 11.7 g/dL
Hematocrit 35.6%
Platelet count 248 000/mm3
Red blood cell count 4.29 × 106/mm3
Mean corpuscular volume 83.0 fL
Mean corpuscular hemoglobin 27.3 pg
Mean corpuscular hemoglobin 32.9 g/dL
concentration
Red cell distribution width 13.0%
Mean platelet volume 9.0 fL
Neutrophils 82.7%
Lymphocytes 7.5%
Monocytes 6.9%
Eosinophils 2.4%
Absolute neutrophil count 9.0%

Figure 27.1 Anteroposterior chest radiograph.


Table 27.2 Basic metabolic panel.

Sodium 148 mEq/L


Potassium 4.5 mEq/L
Chloride 101 mEq/L track with 2 L/min of supplemental oxygen. With this inter-
Carbon dioxide 29 mEq/L vention the retractions resolved, respiratory rate decreased
Urea nitrogen 22 mg/dL to 24 per minute, and oxygen saturation improved to 96%.
Creatinine 0.29 mg/dL She received acetaminophen 15 mg/kg and albuterol 2.5 mg
Glucose 70 mg/dL in line with the ventilator. An intravenous catheter was
placed and she received a 20 mL/kg normal saline fluid
Calcium 9.3 mg/dL
bolus. The results of the complete blood count, basic met-
Anion gap 9 mEq/L
abolic panel, and respiratory pathogen panel are shown in
Tables 27.1–27.3. The chest radiograph is shown in
Figure 27.1. Tracheotomy sputum was obtained and sent
Table 27.3 Respiratory pathogen panel by PCR. for culture.

Test name Result

Human metapneumovirus Detected Questions


Human rhinovirus/enterovirus Detected
Answer the following questions using the details provided.
1 What is the differential diagnosis?
ventilation of 12 breaths per minute, tidal volume 145 mL 2 What is the pathophysiology associated with the diag-
(8 mL/kg), maximum inspiratory pressure (IPAPmax) nosis and how does it affect the patient’s respiratory
30 cmH2O, minimum inspiratory pressure (IPAPmin) status?
12 cmH2O, end-expiratory pressure (EPAP) 6 cmH2O, 3 What is the evidence-based treatment plan for this
inspiratory time 1.2, rise 3, flow trigger sensitivity auto patient?
Downloaded from https://onlinelibrary.wiley.com/doi/ by National Institute Of Standard, Wiley Online Library on [06/03/2024]. See the Terms and Conditions (https://onlinelibrary.wiley.com/terms-and-conditions) on Wiley Online Library for rules of use; OA articles are governed by the applicable Creative Commons License
Chapter 27 Complex Patient with Recurrent Lower Respiratory Tract Infection 111

Rational and Evidence-based Practice What is the pathophysiology associated with the
Explanation diagnosis and how does it affect the patient’s
respiratory status?
What is the differential diagnosis? The airway edema and swelling result in decreased airway
The primary cause for this patient’s symptoms is an acute radius, and an associated increase in airway resistance,
respiratory viral co-infection with human rhinovirus/ therefore necessitating an increased work of breathing to
enterovirus (the PCR test cannot distinguish between overcome the resistance. Also, there is an increase in atelec-
enterovirus and rhinovirus) and human metapneumo- tasis due to airway inflammation and mucus plugging, and
virus. Because of this infection, she demonstrated this results in decreased lung compliance, forcing the
acute airway inflammation as noted by symptoms of patient to generate higher negative pressures to move the
cough and wheeze and peribronchial infiltrates on the same volume of air.
chest radiograph. She is also demonstrating hypersecre-
tion of mucus from the airway. Viral infection likely
contributes to overgrowth of existing colonizing What is the evidence-based treatment plan for this
patient?
bacteria, triggering even further inflammation. The left
shift in the differential white count supports a bacterial The patient will require supportive care to allow her
component. immune system to respond to the viral pathogens, and this
The patient has additional risk factors to consider, includes respiratory support to reduce the work of breath-
including the potential for aspiration due to her history ing and promote optimal ventilation and oxygenation. The
of vocal cord paralysis. There is also evidence of mild goal should be to acquire normal blood gases. A key factor
dehydration as noted by the elevated sodium and blood in children in acute respiratory failure secondary to viral
urea nitrogen on laboratory assessment. She has been una- infection is airway inflammation leading to air trapping.
ble to tolerate tube feedings secondary to post-tussive The patient will benefit from a 5-day course of methylpred-
emesis. nisolone 2 mg/kg daily. She should continue receiving albu-
The patient has multiple risk factors for severe compli- terol 2.5 mg every 2–4 hours around the clock with chest
cations from respiratory viral infections. The presence of a physiotherapy and suctioning.
foreign body in the airway (tracheostomy tube) increases The ventilator may require adjustments based on
the risk of bacterial colonization and leads to the forma- blood gas analysis and patient status. The clinician
tion of biofilm (a film of bacteria that adheres to a surface) may be tempted to treat elevation of PCO2 by temporarily
which is difficult to eradicate. Children with tracheostomy increasing respiratory rate. This may be appropriate if
requiring ventilator support have been identified as hav- the child’s respiratory drive is depressed, but if airway
ing higher risk for hospital readmission. Pseudomonas aer- obstruction is severe, increasing the rate will decrease
uginosa has been specifically linked to an increased risk of the time available for exhalation and only make air trap-
readmission (McCaleb et al., 2016; Russell et al., 2017). ping worse. Increasing IPAP or tidal volume may also
This patient has had multiple trach cultures positive for worsen air trapping if there is not enough time for exha-
this organism. The relationship between an acute viral lation, leading to a condition called “auto-peep.”
infection and bacterial overgrowth is not well understood. Decreasing inspiratory time to allow more time for exha-
There is speculation that increased vascular permeability lation may help. As the child responds to anti-
and secretions provide a nutritive media which promotes inflammatory therapy and blood gases improve, she
bacterial organism proliferation. Viral infection may may be slowly returned to home settings.
depress the capacity of airway epithelial and immune cells Nutrition and hydration are other goals which support
to clear bacteria. In practice, many experts note that chil- healing and patient improvement. As anti-inflammatory
dren with an acute viral infection as documented on viral agents take effect, the cough and post-tussive emesis should
panel also demonstrate organism growth on tracheal cul- improve. As symptoms improve, gastrostomy tube feedings
ture. There continues to be debate on the importance of with an electrolyte solution running continuously may be
treating these organisms as some assert these are simply an initial method to begin feeding and, if tolerated, slowly
colonizing organisms. Other experts note patients often advance to a full maintenance volume, ultimately to her
demonstrate more timely improvement when treated with home nutrition regimen. Intravenous fluids would then
antibiotics. be discontinued.
Downloaded from https://onlinelibrary.wiley.com/doi/ by National Institute Of Standard, Wiley Online Library on [06/03/2024]. See the Terms and Conditions (https://onlinelibrary.wiley.com/terms-and-conditions) on Wiley Online Library for rules of use; OA articles are governed by the applicable Creative Commons License
112 Cases in Pediatric Acute Care

Empiric antibiotic therapy with ceftazidime and gentami- Based on the culture results, what antibiotics, if
cin, along with inhaled tobramycin 300 mg twice daily, any, would you prescribe for discharge and should
should continue until final culture and sensitivities are the inhaled tobramycin be continued?
known. At this point in her admission, the patient is now
The presence of numerous polymorphonuclear leukocytes
improving. Blood gases have normalized and she has
and two organisms in the culture is suggestive of a bacterial
returned to home settings on room air. She is tolerating
infection in the lower airways. In order to treat both organ-
1 hour off ventilator sprints and home tube feeding
isms, this patient will benefit from a course of trimetho-
regimen. Tracheotomy secretions are white but remain
prim/sulfamethoxazole (Bactrim) which will treat the
thick and she continues to need tracheotomy suctioning
Stenotrophomonas maltophilia infection. At a minimum,
once or twice hourly.
she should continue the inhaled tobramycin to complete
Continuation of case: On day 3 of admission the culture the 28-day course. Some experts would endorse systemic
results are available (Table 27.4). treatment of Pseudomonas by adding ciprofloxacin to the
antimicrobial regimen (Abu-Salah and Dhand, 2011).
4 Based on the culture results, what antibiotics, if any,
The use of inhaled antibiotics for children with trache-
would you prescribe for discharge and should the
otomy and ventilator dependence with acute lower respi-
inhaled tobramycin be continued?
ratory bacterial infections is not well studied. There is an
emerging body of evidence studying their use in adults
requiring invasive ventilation in the critical care setting
(Abu-Salah and Dhand, 2011). The current opinion favors
inhaled antibiotics as an adjunct to systemic therapy.
Table 27.4 Sputum culture results.
The patient demonstrated pseudomonal resistance to
Gram stain, respiratory
tobramycin in culture; however, it is important to recog-
nize that “resistance” is determined based on antibiotic
Abnormal: numerous polymorphonuclear leukocytes,
numerous Gram-negative rods levels typically obtained during systemic administration.
During inhaled antibiotic therapy, concentrations of anti-
Respiratory culture/smear biotics in the lung are significantly higher than those
Abnormal: Stenotrophomonas maltophilia complex, achieved during systemic administration, and therefore
Pseudomonas aeruginosa Pseudomonas infection may be susceptible to inhaled
tobramycin by virtue of the greater concentration from
Susceptibility topical delivery. However, If the patient continues to dem-
Stenotrophomonas maltophilia onstrate recurrent lower respiratory bacterial infections
Doxycycline: MIC ≤2 μg/mL with tobramycin-resistant Pseudomonas, she may benefit
from transition to inhaled colistin (Vardakas et al., 2018)
Levofloxacin: MIC 4 μg/mL (intermediate)
or aztreonam.
Minocycline: MIC ≤2 μg/mL (sensitive)
Trimethoprim/sulfamethoxazole: MIC ≤0.5 μg/mL (sensitive) Continuation of case: After receiving doses of ceftazidime
Pseudomonas aeruginosa and gentamicin in the emergency room, the child was
admitted to the PICU on the ventilator (via tracheostomy)
Amikacin: MIC 4 μg/mL (sensitive)
with 2 L/min of supplemental oxygen, and normal saline
Aztreonam: MIC 8 μg/mL (sensitive)
running at maintenance. She was placed nil by mouth
Cefepime: MIC 4 μg/mL (sensitive)
and the gastrostomy tube vented to gravity. There was
Ciprofloxacin: MIC 0.5 μg/mL (sensitive) approximately 30 mL of light green mucoid drainage
Colistin: MIC 2 μg/mL (sensitive) in the gastrostomy tube drainage bag. Vital signs on admis-
Gentamicin: MIC 2 μg/mL (sensitive) sion as follows: temperature 37.5 C (99.5 F), heart rate
Imipenem: MIC 4 μg/mL (intermediate) 124 bpm, respiratory rate 24 per minute, blood pressure
Levofloxacin: MIC 2 μg/mL (sensitive) 96/66 mmHg, and oxygen saturation 95%. She continued
Meropenem: MIC ≤1 μg/mL (sensitive)
to have intermittent spasmodic coughing with retching
and emesis. On auscultation the breath sounds were coarse
Piperacillin/tazobactam: MIC 8 μg/mL (sensitive)
with diffuse wheezing throughout. She required almost
Tobramycin: MIC 8 μg/mL (resistant)
constant tracheotomy suctioning for large amounts of thick
MIC, minimum inhibitory concentration. green/yellow secretions.
Downloaded from https://onlinelibrary.wiley.com/doi/ by National Institute Of Standard, Wiley Online Library on [06/03/2024]. See the Terms and Conditions (https://onlinelibrary.wiley.com/terms-and-conditions) on Wiley Online Library for rules of use; OA articles are governed by the applicable Creative Commons License
Chapter 27 Complex Patient with Recurrent Lower Respiratory Tract Infection 113

Case Resolution • patient requires pulmonary toileting no more frequently


than every 4 hours
The indications for discharge are as follows:
• patient back or close to home ventilator settings without

••
any extra oxygen requirement
patient remains afebrile
patient tolerates oral feeds • patient tolerating oral and inhaled antibiotics.

References
Abu-Salah, T. and Dhand, R. (2011). Inhaled antibiotic therapy tracheotomy bacterial infection readmissions.
for ventilator associated tracheobronchitis and ventilator Pediatric Pulmonology 52(9): 1212–1218. doi: 10.1002/
associated pneumonia: an update. Advances in Therapy 28 ppul.23716.
(9): 728–747. doi: 10.1007/s12325-011-0051-z. Vardakas, K.Z., Voulgaris, G.L., Samonis, G., and Falagas, M.E.
McCaleb, R., Warren, R.H., Willis, D., et al. (2016). Description (2018). Inhaled colistin monotherapy for respiratory tract
of respiratory microbiology in children with long term infections in adults without cystic fibrosis: a systemic
tracheostomies. Respiratory Care 61(4): 447–452. review and meta-analysis. International Journal of
Russell, C.J., Simon T.T., Mamey, M.R., Newth, C.J.L., and Antimicrobial Agents 51(1): 1–9. doi: 10.1016/j.
Neely, M.N. (2017). Pseudomonas aeruginosa and post ijantimicag.2017.05.016.

Further Reading
Nackers, A., Ehlenbach, M., Kelly, M.M., et al. ((2019). medical complexity. Hospital Pediatrics 9(1): 6–15. doi:
Encounters from device complications among children with 10.1542/hpeds.2018-0103.
Downloaded from https://onlinelibrary.wiley.com/doi/ by National Institute Of Standard, Wiley Online Library on [06/03/2024]. See the Terms and Conditions (https://onlinelibrary.wiley.com/terms-and-conditions) on Wiley Online Library for rules of use; OA articles are governed by the applicable Creative Commons License
115

28

Child with Racing Heart


Bradley Goettl
University of Texas Health, San Antonio, TX, USA

A previously healthy 5-year-old female was brought to the Current Status


emergency department (ED) for evaluation of irritability
and racing heart. The triage nurse noted a rapid heart rate On admission, she is awake, alert and anxious. Her respira-
and the child was admitted to a monitored bed. tions are shallow with a respiratory rate of 28 breaths per
minute with mild subclavicular retractions. The lung
sounds are clear bilaterally. Her skin is warm but pale.
History of Present Illness
Her capillary refill time is 2 seconds with peripheral pulses
present and no peripheral edema. Her abdomen is soft,
Caregivers reported that the child complained of a racing
non-distended, and without hepatosplenomegaly.
heart after playing outside yesterday. Since then, the
Vital signs are as follows: oral temperature 36.4 C
patient has been irritable, anxious, and not eating well.
(97.6 F), heart rate 250 bpm, respiratory rate 28 breaths
Fever, pain, vomiting, constipation, and diarrhea were all
per minute, blood pressure 104/62 mmHg, and oxygen sat-
denied. There were no reported sick exposures.
uration (SpO2) 98% on room air.

Past Medical History Questions


She was born at 37 weeks’ gestation. She has had no prior
Answer the following questions using the details provided.
hospitalizations and has an established primary care pro-
vider. All vaccinations are up to date for age, including 1 What should be your initial approach to managing this
influenza. patient?
2 What initial diagnostic testing is needed for this
patient?
Past Surgical History 3 How would you interpret the electrocardiogram (EKG)
shown in Figure 28.1?
No previous surgeries. 4 Which additional diagnostic studies would be appropri-
ate to order in the emergency department?
Family History
Rationale and Evidence-based
Parents deny family history of heart disease, arrhythmias, Practice Explanation
sudden cardiac death, or pulmonary diseases.
What should be your initial approach to managing
this patient?
Social History
After assuring adequate airway and breathing, oxygen
She lives with her parents. Has no siblings. Parents deny would be ordered if there was a concern for hypoxemia.
tobacco or drug exposure. There are no signs of shock, but he is at risk of

Cases in Pediatric Acute Care: Strengthening Clinical Decision Making, First Edition. Edited by Andrea M. Kline-Tilford and Catherine M. Haut.
© 2020 John Wiley & Sons Ltd. Published 2020 by John Wiley & Sons Ltd.
Downloaded from https://onlinelibrary.wiley.com/doi/ by National Institute Of Standard, Wiley Online Library on [06/03/2024]. See the Terms and Conditions (https://onlinelibrary.wiley.com/terms-and-conditions) on Wiley Online Library for rules of use; OA articles are governed by the applicable Creative Commons License
116 Cases in Pediatric Acute Care

Figure 28.1 EKG 1.

decompensation due to significant tachycardia and requires dehydration/hypovolemia can contribute to the develop-
intravenous access. If it is not possible to achieve intrave- ment of an arrhythmia. Severe anemia can cause tachycar-
nous access rapidly, intraosseous catheter placement would dia, so a complete blood count should be obtained. A urine
be indicated. She is placed on a cardiac monitor to follow drug screen will evaluate for toxicologic causes for tachycar-
her heart rate and rhythm. dia. A thyroid-stimulating hormone (TSH) level can be
obtained. It is unlikely that the child will have thyrotoxico-
What initial diagnostic testing is needed for sis, but L-thyroxine ingestion can cause tachyarrhythmias.
this patient? A chest radiograph will evaluate for radiologic evidence of
acute cardiopulmonary disease and cardiomegaly.
An EKG is needed to determine if this is a narrow- or wide- A point-of-care ultrasound (POCUS) limited echocardio-
complex tachycardia. gram can quickly evaluate overall cardiac function and
quickly evaluate for pericardial effusion or right heart strain.
How would you interpret the EKG shown A formal echocardiogram will likely be obtained after
in Figure 28.1? admission to evaluate cardiac function and wall motion
This EKG demonstrates narrow-complex tachycardia with abnormalities and to rule out congenital heart defects.
heart rate of 252 bpm. Continuation of case: Diagnostic studies were initiated,
including chest radiograph, basic metabolic profile, com-
Which additional diagnostic studies would be plete blood count, urine drug screen, cardiac ultrasound,
appropriate to order in the emergency
and 12-lead EKG. Results for basic metabolic profile and
department?
complete blood count are shown in Tables 28.1 and 28.2.
An EKG should be obtained with each rhythm change. The chest radiograph (Figure 28.2) shows the lung fields
A metabolic panel should be obtained, including calcium, free of infiltrate, edema and atelectasis. No significant pleu-
magnesium, and potassium. Electrolyte imbalances and ral effusions were noted and there is no cardiomegaly.
Downloaded from https://onlinelibrary.wiley.com/doi/ by National Institute Of Standard, Wiley Online Library on [06/03/2024]. See the Terms and Conditions (https://onlinelibrary.wiley.com/terms-and-conditions) on Wiley Online Library for rules of use; OA articles are governed by the applicable Creative Commons License
Chapter 28 Child with Racing Heart 117

Table 28.1 Basic metabolic profile. 6 What are the treatment options for this type of
tachyarrhythmia?
Sodium 141 mEq/L
7 What is unique about the EKG shown in Figure 28.3?
Potassium 4.2 mEq/L What is the relationship between this EKG and the
Chloride 109 mEq/L EKG in Figure 28.1?
Carbon dioxide 21 mEq/L
Blood urea nitrogen 8 mg/dL Based on the information provided, what is the
Creatinine 0.2 mg/dL
differential diagnosis and the most likely
diagnosis for this child?
Glucose 104 mg/dL
Magnesium 2.5 mg/dL The differential diagnosis includes, but is not limited to,
Thyroid stimulating hormone 0.625 mU/L palpitations, dysrhythmia including supraventricular tach-
ycardia (SVT) and Wolff–Parkinson–White syndrome,
Long–Ganong–Levine syndrome, electrolyte abnormality,
infection/sepsis, overdose/toxidrome, congenital malfor-
Table 28.2 Complete blood count.
mation, and cardiac/respiratory abnormalities.
White blood cell count 12 000/mm3
Wolf–Parkinson–White (WPW) syndrome is the most
common accessory pathway syndrome and the most likely
Hemoglobin 11 g/dL
diagnosis for this child. The diagnosis can be made from the
Hematocrit 32.7%
EKG. Often, WPW is found on routine EKGs and/or a
Platelet count 368 000/mm3 sports physical. For some patients, the initial presentation
could be for a symptomatic SVT or other tachyarrhythmia.

What are the treatment options for this type of


tachyarrhythmia?
This patient was in stable tachyarrhythmia due to his phys-
ical examination findings and vital signs. The following
should be considered as first-line therapies.
1 Vagal maneuvers: ice pack to eyes and forehead or ask-
ing the patient to blow through an occluded straw are
options for a child of this age.
2 Intravenous adenosine 0.1 mg/kg rapidly, followed
immediately by 10 mL normal saline flush. Prepare a
second dose of adenosine 0.2 mg/kg, double the initial
dose, if the first dose of adenosine is not successful in
converting rhythm.
3 If unable to obtain intravenous/intraosseous access,
or the rhythm is refractory to a second dose of adeno-
sine, perform synchronized cardioversion. Initial syn-
chronized cardioversion would be administered at
Figure 28.2 Chest radiograph. 0.5–1 J/kg.
4 Pediatric cardiology and/or electrophysiology consult.
This patient’s rhythm did not convert with vagal maneu-
Other results were as follows: (i) urine drug screen was
vers or adenosine and he was cardioverted with 1 J/kg.
negative; (ii) point-of-care cardiac ultrasound showed nor-
After cardioversion, the EKG shown in Figure 28.3 was
mal cardiac function with no effusion or signs of right heart
obtained. Her heart rate dropped to 131 bpm. A pediatric
strain; and (iii) 12-lead EKG indicated normal right ventric-
cardiology and electrophysiology consult should be
ular size and systolic function and normal left ventricular
obtained. Initially, antiarrhythmics are prescribed to pre-
size, wall thickness, and systolic function.
vent SVT. Common antiarrhythmics include, but are not
5 Based on the information provided, what is the differ- limited to, digoxin, beta-blockers, and amiodarone. Infants
ential diagnosis and the most likely diagnosis for with this diagnosis often outgrow the symptoms of the syn-
this child? drome by 1 year of age. Older children should have limited
Downloaded from https://onlinelibrary.wiley.com/doi/ by National Institute Of Standard, Wiley Online Library on [06/03/2024]. See the Terms and Conditions (https://onlinelibrary.wiley.com/terms-and-conditions) on Wiley Online Library for rules of use; OA articles are governed by the applicable Creative Commons License
118 Cases in Pediatric Acute Care

Figure 28.3 EKG 2.

vigorous activity until cleared by cardiology. An electrophy- (i) shortened PR interval of less than 120 ms; (ii) delta wave,
siologic study and cardiac radiofrequency ablation is con- i.e. initial slurring of the QRS; (iii) QRS greater than 110 ms;
sidered the gold standard for patients with recurrent and (iv) ST-segment and T-wave changes indicating repo-
arrhythmia or those at high risk for sudden cardiac death. larization abnormalities.
Delta waves are diagnostic for WPW and represent early
What is unique about the EKG shown in ventricular depolarization via an accessory pathway. Atrio-
Figure 28.3? What is the relationship between this ventricular reentrant tachycardia is a common form of par-
EKG and the EKG in Figure 28.1? oxysmal SVT for patients with WPW syndrome.
This EKG demonstrated sinus tachycardia with a heart rate
of 131 bpm. A WPW pattern is recognized, including

Further Reading
American Heart Association. Web-based integrated guidelines Human Development 91: 345–350. doi: 10.1016/j.
for cardiopulmonary and emergency cardiovascular care. earlhumdev.2015.04.001.
Part 12. Pediatric advanced life support. https:// Cohen, M.I., Triedman, J.K., Cannon, B.C., et al. (2012).
eccguidelines.heart.org/index.php/circulation/cpr-ecc- PACES/HRS expert consensus statement on the
guidelines-2/part-12-pediatric-advanced-life-support/ management of the asymptomatic young patient with a
Chu, P.Y., Hill, K.D., Clark, R.H., Smith, P.B., and Hornik, C.P. Wolff–Parkinson–White (WPW, ventricular preexcitation)
(2015). Treatment of supraventricular tachycardia in electrocardiographic pattern. Heart Rhythm 9(6): 1006–1024.
infants: analysis of a large multicenter database. Early doi: 10.1016/j.hrthm.2012.03.050.
Downloaded from https://onlinelibrary.wiley.com/doi/ by National Institute Of Standard, Wiley Online Library on [06/03/2024]. See the Terms and Conditions (https://onlinelibrary.wiley.com/terms-and-conditions) on Wiley Online Library for rules of use; OA articles are governed by the applicable Creative Commons License
119

29

Teen with Headache and Seizure


Valarie Eichler
Dallas Children’s Hospital, Dallas, TX, USA

A 15-year-old girl with recent past medical history of Past Surgical History
headaches, otherwise healthy, was admitted to the inten-
sive care unit with altered mental status and new-onset Tonsillectomy and adenoidectomy at age 3 years for recur-
seizures. rent streptococcal infections.

Family History
History of Present Illness
Positive history of maternal and paternal migraines.
The adolescent reported intermittent headaches that have
been intensifying over the past 3 weeks prior to admission.
She was seen by her primary care provider and noted to Social History
have a strong family history of migraine headaches. She
was prescribed ibuprofen and sumatriptan. She has used Lives in a house in an urban city with parents and teenage
sumatriptan twice in the past 2 weeks, most recently on sister, no pets, no smokers, attends 10th grade and partici-
the day prior to admission. On the night prior to admis- pates in competitive gymnastics and swim team. Family
sion, she experienced nausea and emesis twice and pain denies drug, tobacco or alcohol use. She is not sexually
that was not relieved by ibuprofen or sumatriptan. She active and has not taken any contraceptives in the past.
was found by her sister at approximately 4:30 a.m. when
she screamed out and then began “shaking” and the emer-
gency medical service was called. On their arrival approx-
Current Status
imately 10 minutes later, the adolescent was unresponsive
Five minutes after her arrival in the ED she begins having
but breathing. En route to the emergency department
tonic–clonic seizures and was given intravenous lorazepam
(ED), she began to have seizure activity again which was
2 mg and an intravenous load of levetiracetam 60 mg/kg.
initially focal on the left hand and arm, then generalized
Vital signs are as follows: heart rate 77 bpm, respiratory
to a tonic–clonic seizure. She was given one dose of rectal
rate 16 breaths per minute (hand ventilating), blood pres-
diazepam for her seizure, became apneic and was intu-
sure 148/92 mmHg, oxygen saturation 98% with 100% bag
bated for airway control.
ventilation. Blood is obtained for arterial blood gas analysis,
complete blood count with differential, electrolytes, glucose,
coagulation panel (PT/PTT/INR), lactate, pregnancy test
Past Medical History (human chorionic gonadotropin), and blood culture. Urine
was collected for urinalysis and urine drug screen.
Born at term. She was discharged home with her mother on Results of the basic metabolic profile, complete blood
day 2 of life. She is up to date on her immunizations for age, count and coagulation panel, and blood gas analysis are
including the influenza vaccine for the season. She has oth- shown in Tables 29.1–29.3. Test for β-human chorionic
erwise been healthy. Last menses 3 weeks ago. gonadotropin was negative. Urine drug screen pending.

Cases in Pediatric Acute Care: Strengthening Clinical Decision Making, First Edition. Edited by Andrea M. Kline-Tilford and Catherine M. Haut.
© 2020 John Wiley & Sons Ltd. Published 2020 by John Wiley & Sons Ltd.
Downloaded from https://onlinelibrary.wiley.com/doi/ by National Institute Of Standard, Wiley Online Library on [06/03/2024]. See the Terms and Conditions (https://onlinelibrary.wiley.com/terms-and-conditions) on Wiley Online Library for rules of use; OA articles are governed by the applicable Creative Commons License
120 Cases in Pediatric Acute Care

Table 29.1 Basic metabolic profile. and is hemodynamically stable. It should be assumed that
the child has increased intracranial pressure (ICP), based
Sodium 141 mEq/L
on the rapid neurologic decline manifested by the seizure,
Potassium 4.2 mEq/L and should be treated with intravenous hypertonic (3%)
Chloride 102 mEq/L saline 5 mL/kg prior to obtaining additional neuroimaging.
Carbon dioxide 21 mEq/L Emergent treatment is the same as any other patient with
Blood urea nitrogen 11 mg/dL an acute neurologic decline and signs and symptoms of
Creatinine 0.5 mg/dL increased ICP: positioning, hyperosmolar therapy, mild
Glucose 147 mg/dL
hyperventilation, temperature control, seizure control,
sedation, analgesia, and pharmacologic paralysis if needed.
Lidocaine is also an important medication to use prior to
Table 29.2 Complete blood count. any noxious stimulus as it blunts the sympathetic response
so as not to increase the heart rate and blood pressure
White blood cell count 7200/mm3 acutely, as this increases cerebral blood flow/volume
Hemoglobin 9.3 g/dL thereby exacerbating ICP. She is given a fluid bolus of nor-
Hematocrit 28.5% mal saline 20 mL/kg and ceftriaxone at meningitic dosing.
Platelet count 99 000/mm3
Prothrombin time (PT) 15 s
Which imaging studies would be appropriate for
International normalized ratio (INR) 1.7 this patient?
Partial thromboplastin time (PTT) 37 s
A chest radiograph and head computed tomography (CT)
are obtained. A chest radiograph is indicated to evaluate
Table 29.3 Blood gas analysis. the position of the endotracheal tube and the presence of
underlying lung disease. The chest radiograph reveals clear
pH 7.37 lung fields and a correctly positioned endotracheal tube.
PaCO2 37 mmHg After initial stabilization has occurred, a non-contrasted
PaO2 109 mmHg CT scan of the brain should be the next study. CT scan is
Bicarbonate 20 mEq/L the initial diagnostic study for any acute change in neuro-
Base excess 1.9 logic status to evaluate for underlying hemorrhage, mass,
Lactate 1.9 mmol/L ventricle size, and presence of edema. The head CT
revealed a large right temporal–parietal intraparenchymal
hemorrhage that extended into the right lateral ventricle
Questions and fourth ventricle with a 4-mm shift to the left and efface-
ment of the basal cisterns (Figures 29.1 and 29.2). Neuro-
Answer the following questions using the details provided. surgery was consulted to evaluate for surgical intervention.

1 What are the immediate next steps in intervention for


this patient? Based on the data given, what are the differential
2 Which imaging studies would be appropriate for this diagnoses and the most likely diagnosis for this
patient? patient?
3 Based on the data given, what are the differential diag- Differential diagnoses for this child include new-onset
noses and the most likely diagnosis for this patient? seizure disorder, migraine headache, Moyamoya disease,
medial or posterior cerebral artery stroke, vein of Galen
malformation, cerebral aneurysm, intracranial hemor-
rhage related to blunt trauma, cavernous hemangioma,
Rationale and Evidence-based Practice
subarachnoid hemorrhage, meningitis, hypertensive intra-
Explanation cranial hemorrhage, and hemorrhagic neoplasm.
The head CT scan revealed an intraparenchymal hemor-
What are the immediate next steps in intervention
rhage which allowed narrowing of the differential diagno-
for this patient?
sis. Based on the presenting symptoms, history of prior
Initially, ensure the patient has a secure airway, adequate unsuccessful treatment of migraine headache, and acute
oxygenation and ventilation via mechanical ventilation, neurologic decompensation, the most likely diagnosis for
Downloaded from https://onlinelibrary.wiley.com/doi/ by National Institute Of Standard, Wiley Online Library on [06/03/2024]. See the Terms and Conditions (https://onlinelibrary.wiley.com/terms-and-conditions) on Wiley Online Library for rules of use; OA articles are governed by the applicable Creative Commons License
Chapter 29 Teen with Headache and Seizure 121

monitor. Placement of an EVD would allow for not only


monitoring of ICP but also removal of cerebrospinal fluid
(CSF), which leads to a decrease in ICP as more room is cre-
ated with the fluid drainage. This may be performed at the
bedside. However, if the hematoma is causing mass effect
and a midline shift that may be exerting downward pres-
sure and which could lead to herniation of the brainstem
and brain death, the patient should be taken to the operat-
ing room for evacuation of the hematoma and possibly a
decompressive craniectomy to allow room for the brain
to swell during this acute phase.
4 How would you determine if the therapy was effective?
5 What other therapies would be recommended at
this time?
6 What additional imaging would be warranted?
7 What would be considered the next step in surgical
Figure 29.1 CT head. intervention?

How would you determine if the therapy


was effective?
Continued frequent neurologic monitoring of ICP, pupils,
and response to pain or other stimuli is paramount. The
goal is to prevent secondary injury and decrease the mor-
bidity associated with a ruptured AVM. If after all the inter-
ventions described the ICP fails to fall to an acceptable level
(<20 cmH2O), then the patient may be placed into a barbi-
turate coma to slow cerebral metabolism and allow the
brain time to cool down. Monitoring with a continuous
electroencephalogram (EEG) will allow the provider to
observe for burst suppression. The usual goal is one burst
per 10 seconds or five to six bursts per minute. An epilep-
tologist would be helpful in reading the EEG and reporting
results periodically to the provider. Of note, when using
barbiturates the patient may become hemodynamically
unstable and require fluid boluses and inotropic support.
This is usually managed with the use of an epinephrine
or norepinephrine infusion. Arterial and central lines are
needed to adequately monitor and provide medications to
Figure 29.2 CT head: intraventricular extension of hemorrhage.
the patient.
The overall goal is an ICP of less than 20 cmH2O. To
this teenager is an intracranial hemorrhage from a ruptured
ensure this, frequent neurologic monitoring, physical
arteriovenous malformation (AVM).
examinations, maintaining serum sodium goals of
Continuation of case: After returning from the radiology 150–155 mEq/L, and repeated head CT scans are needed
department, the patient is admitted to the intensive care to assess the effectiveness of therapy. Monitoring of the
unit for further management. A ruptured AVM is consid- child’s ICP, arterial blood pressure, cerebral perfusion pres-
ered a medical emergency secondary to the cerebral edema sure, PaCO2 and PaO2 are all components of the examination
and increased ICP associated with the hemorrhage. Neuro- needed to verify effectiveness of treatment. Maintaining
surgery reviewed the patient and neuroimaging to evaluate adequate sedation, analgesia, temperature control, and sei-
for placement of an intracranial monitoring device. Options zure control will help ensure that ICP goals are met. The
included an external ventricular drain (EVD) or a fiberoptic goal is to maintain a cerebral perfusion pressure (CPP)
Downloaded from https://onlinelibrary.wiley.com/doi/ by National Institute Of Standard, Wiley Online Library on [06/03/2024]. See the Terms and Conditions (https://onlinelibrary.wiley.com/terms-and-conditions) on Wiley Online Library for rules of use; OA articles are governed by the applicable Creative Commons License
122 Cases in Pediatric Acute Care

adequate to perfuse the brain tissue. General CPP goals are is performed, intraoperative angiography or magnetic
as follows: infants 40–50 mmHg, children 50–60 mmHg, resonance angiography would be obtained to evaluate
adolescents 60–65 mmHg. the removal of the AVM. If embolization is undertaken,
it is usually with a staged approach. Specialized glue,
What other therapies would be recommended at beads or coils are used and deployed into the involved
this time? vessel(s) to decrease the size of the nidus and ultimately
cease blood flow to the AVM. Embolization may be cur-
If the ICP is refractory to medical management, the neuro- ative or may be performed prior to surgery to decrease
surgeon may be asked to perform a decompressive craniect- the size of the AVM to enhance successful removal. If
omy on the child. This is the surgical removal of the skull the AVM is located deep in the parenchyma, then a
over the affected region to allow more room for the brain to Gamma knife (stereotactic radiosurgery) may be used
swell during the acute phase, which is usually 72–96 hours. to focus high-energy beams of radiation to a specific tar-
After the acute phase has resolved and the child is stable geted area, ultimately reducing injury to the tissue near
and ICP is no longer an issue, the surgeon will replace the target.
the bone flap in a procedure called cranioplasty. An AVM is defined as a congenital intracranial mal-
formation distinguished by a persistently abnormal con-
What additional imaging would be warranted? nection between arteries and veins within the brain
without an interposed or developed capillary bed.
After the patient is stable from a clinical standpoint, and
AVMs develop during the third week of gestation when
ideally after a period of “cooling off” that allows for reab-
the capillary bed fails to develop between the arteries
sorption of the hematoma, the patient should be evaluated
and veins. They are seen in approximately 1% of the
with cerebral angiography (Figure 29.3). Considered the
pediatric population, with 1 in 100 000 children having
gold standard, this study identifies the arteries involved
AVMs that are asymptomatic. The majority of AVMs
in the AVM, the nidus, and the large draining vessels. If
are supratentorial and within the parenchyma of the
the angiogram is performed too early in the course, the
brain. They are often associated with neurologic deficits
hematoma and cerebral edema surrounding the hematoma
such as hemiparesis and seizures and, depending on the
may obscure or hide important vasculature.
location, vision and speech may be affected. A small
percentage of the time they are found in the brainstem,
What would be considered the next step in
thalamus, basal ganglia, and cerebellum. These smaller
surgical intervention?
group of AVMs are more detrimental because of their
Approach and successful excision of the AVM is depend- location and often lead to catastrophic outcomes includ-
ent on the location. If surgical clipping and obliteration ing death.

Figure 29.3 Cerebral angiogram reveals right middle cerebral artery AVM.
Downloaded from https://onlinelibrary.wiley.com/doi/ by National Institute Of Standard, Wiley Online Library on [06/03/2024]. See the Terms and Conditions (https://onlinelibrary.wiley.com/terms-and-conditions) on Wiley Online Library for rules of use; OA articles are governed by the applicable Creative Commons License
Chapter 29 Teen with Headache and Seizure 123

Case Resolution important to continue to support the patient throughout


the recovery and convalescent phase. This is accom-
The child successfully recovered after surgical removal of plished through a multidisplinary approach with physi-
the AVM involving the right middle cerebral artery. cal therapy, occupational therapy, child life specialists,
There was residual left-sided weakness but this was and physical medicine and rehabilitation services. When
improving with intense inpatient rehabilitation. After medically stable the patient is transferred to an inpatient
the acute management has been completed, it is rehabilitation facility.

Further Reading
Eichler, V. (2016). Neurologic disorders: arteriovenous O’Brien, N.F. (2012). Traumatic brain injury. In: Pediatric
malformation. In: Lippincott Certification Review: Pediatric Acute Care: A Guide for Interprofessional Practice (ed. K.
Acute Care Nurse Practitioner (ed. A.M. Kline-Tilford and C. Rueter-Rice and B. Bolick), 1197–1203. Burlington, MA:
Haut), 181–184. Philadelphia, PA: Wolters Kluwer. Jones & Bartlett Learning.
Marcoux, K.K. (2012). Arteriovenous malformation. In: Ravindra, V.M., Bollo, R.J., Eli, I.M., et al. (2019). A study of
Pediatric Acute Care: A Guide for Interprofessional Practice pediatric cerebral arteriovenous malformations: clinical
(ed. K. Rueter-Rice and B. Bolick), 916–920. Burlington, MA: presentation, radiological features, and long-term functional
Jones & Bartlett Learning. and educational outcomes with predictors of sustained
Meling, T.R. and Patel, G. (2019). What is the best therapeutic neurological deficits. Journal of Neurosurgery Pediatrics 24
approach to a pediatric patient with a deep-seated brain (1): 1–8. doi: 10.3171/2019.2.PEDS18731.
AVM? Neurosurgical Review 42(2): 409–416. doi: 10.1007/
s10143-019-01101-8.
Downloaded from https://onlinelibrary.wiley.com/doi/ by National Institute Of Standard, Wiley Online Library on [06/03/2024]. See the Terms and Conditions (https://onlinelibrary.wiley.com/terms-and-conditions) on Wiley Online Library for rules of use; OA articles are governed by the applicable Creative Commons License
125

30

A 4-Year-Old with Bloody Diarrhea


Tamara Hill
University of Maryland Medical Center, Baltimore, MD, USA

A 4-year-old female is brought to the emergency depart- Family History


ment (ED) by her mother with a chief complaint of bloody
diarrhea and is later admitted to the pediatric intensive care Mother has a history of seizures and takes medication for
unit (PICU). same. No other family history.

Current Status
History of Present Illness The child is alert but fussy, appears pale, and is non-toxic.
Her conjunctiva are pale, pharynx and tonsils appear nor-
This 4-year-old child became ill after attending a birthday mal, and tympanic membranes intact. Her neck is supple
party 7 days prior where she had consumed hot dogs and without adenopathy. Cardiac examination notes a regular
hamburgers and went swimming in a nearby lake. She rhythm with tachycardia and a grade II/VI vibratory systolic
has had no fever, ill contacts, or recent exposure to children ejection murmur at the left sternal border without radiation.
with diarrhea. No heaves, lifts, thrills, rubs, or gallops are present. Her lungs
She is normally a very energetic and happy child but for a are clear with good aeration and abdomen is flat, soft, and
week now she has been fussy and tired with abdomi- non-tender, with the liver edge palpable 3 cm below the right
nal pain. costal margin. The spleen is non-palpable. Her pulses and
perfusion are good. There is no edema, rash, or petechiae.
Vital signs are as follows: temperature 37.7 C (99.9 F),
heart rate 150 bpm, respiratory rate 28 breaths per minute,
Past Medical History blood pressure 100/45 mmHg, and oxygen saturation 100%
on room air. Weight is 17 kg (75th percentile).
The child was born via uncomplicated spontaneous vaginal A basic metabolic panel, complete blood count, and
delivery at term and went home with her mother. She is urinalysis were obtained, with results as shown in
current on all immunizations, including influenza vaccine. Tables 30.1–30.3. A blood peripheral smear shows schisto-
She has no other medical history. cytes, helmet cells, and polychromasia.

Questions

Answer the following questions using the details provided.


Past Surgical History
1 Based on data given, what are the differential diagnoses
The child has not had any surgical procedures. and the most likely diagnosis for this child?

Cases in Pediatric Acute Care: Strengthening Clinical Decision Making, First Edition. Edited by Andrea M. Kline-Tilford and Catherine M. Haut.
© 2020 John Wiley & Sons Ltd. Published 2020 by John Wiley & Sons Ltd.
Downloaded from https://onlinelibrary.wiley.com/doi/ by National Institute Of Standard, Wiley Online Library on [06/03/2024]. See the Terms and Conditions (https://onlinelibrary.wiley.com/terms-and-conditions) on Wiley Online Library for rules of use; OA articles are governed by the applicable Creative Commons License
126 Cases in Pediatric Acute Care

Table 30.1 Complete blood count. Table 30.3 Urinalysis.

White blood cell count 16 000/mm3 Color Clear yellow


Hemoglobin 6.9 g/dL pH 6.0
Hematocrit 22.4% Specific gravity 1.012
Platelet count 37 000/mm3 Protein 3+
Neutrophils 56% Nitrites Negative
Lymphocytes 27% Urobilirubin 1+
Monocytes 2% Blood 3+
Eosinophils 3% Leukocytes Negative
Bands 12% Ketones Negative
Glucose Negative
White blood cells 51%
Table 30.2 Complete metabolic profile and coagulation Red blood cells 21%
parameters.
Hyaline casts 3–5
Sodium 133 mEq/L Bacteria Occasional
Potassium 5.8 mEq/L Crystals None
Chloride 106 mEq/L
Carbon dioxide 13 mEq/L
Blood urea nitrogen 48 mg/dL
of cases. Bloody diarrhea and abdominal pain are usually
Creatinine 3.82 mg/dL manifestations of hemorrhagic enterocolitis caused by
Glucose 145 mg/dL Shiga toxin-producing bacteria, most commonly Escheri-
Calcium 7.8 mg/dL chia coli. Patients with Shiga toxin-associated HUS have
Albumin 3.0 mg/dL been described who do not have obvious diarrheal pro-
Total protein 6.6 g/dL dromes. On the other hand, patients with atypical HUS
Aspartate aminotransferase 28 U/L
commonly present with abdominal pain and diarrhea,
which may begin days to weeks before the patient seeks
Alanine aminotransferase 32 U/L
medical care, thus giving rise to the impression of diar-
Alkaline phosphatase 125 U/L
rhea-associated HUS. Diarrhea, with or without blood, is
Total bilirubin 0.2 mg/dL found in approximately 30% of patients with atypical
Phosphorus 7.1 mg/dL HUS at disease onset because of gut involvement. For
Lactate dehydrogenase 300 U/L this reason, categorization of HUS according to clinical
Prothrombin time 12.5 s presentation as diarrhea-positive or diarrhea-negative is
Partial thromboplastin time 41 s inappropriate.
International normalized ratio 0.9 Causes of HUS are typically related to exposure to con-
taminated food, especially undercooked beef, but can also
result from certain medications, including quinine, ciclos-
porin, tacrolimus, some drugs of abuse including cocaine,
Rationale and Evidence-based Practice
some chemotherapeutics, and antiplatelet agents.
Explanation If a patient presents with a diarrheal prodrome, Shiga
toxin-associated HUS can easily be differentiated by the
Based on data given, what are the differential
presence of Shiga toxin in the stools and/or serum antibo-
diagnoses and the most likely diagnosis for
dies against Shiga toxin. Another infectious form of HUS
this child?
occurs with T-antigen activation in association with pneu-
If a child presents with microangiopathic hemolytic ane- mococcal sepsis and this can also easily be differentiated
mia, thrombocytopenia, and renal failure preceded by from atypical HUS.
abdominal pain and diarrhea, the diagnosis can be made Other potential differential diagnoses for a child who pre-
for Shiga toxin-associated hemolytic uremic syndrome sents with bloody diarrhea include necrotizing enterocoli-
(HUS). This accounts for 90% of HUS cases in children, tis, colitis, and inflammatory bowel disease among
with Streptococcus pneumoniae responsible for at least 5% others. In this case, the child presents with very typical
Downloaded from https://onlinelibrary.wiley.com/doi/ by National Institute Of Standard, Wiley Online Library on [06/03/2024]. See the Terms and Conditions (https://onlinelibrary.wiley.com/terms-and-conditions) on Wiley Online Library for rules of use; OA articles are governed by the applicable Creative Commons License
Chapter 30 A 4-Year-Old with Bloody Diarrhea 127

symptoms and laboratory findings of hemolytic anemia, How would you determine if the therapy was
thrombocytopenia, and renal failure. effective?
Electrolytes will normalize and the child will show
improved renal function along with normal blood pressure.
Additional Questions and Evidence- Early hydration during the diarrheal phase may lessen the
based Rationale severity of renal insufficiency. Most children with Shiga
toxin-associated HUS recover and do not have chronic
2 What is the best management for HUS? symptoms or problems, although some may show evidence
3 How would you determine if the therapy was effective? of long-term renal complications and other morbidity.
4 What other therapies would be recommended for HUS?
5 Would you obtain any other diagnostic studies?
What other therapies would be recommended
for HUS?
What is the best management for HUS?
Patients who experience volume overload, metabolic acido-
Therapy for HUS is supportive and includes volume reple- sis, electrolyte abnormalities, uremia or oliguria may
tion, hypertension control, and managing complications of require dialysis. Non-oliguric patients usually recover well,
renal insufficiency, including dialysis when warranted. Red but there still remains an increased risk for future renal
blood cell transfusions may be indicated. Platelet transfu- complications, including chronic kidney disease.
sions should be avoided since they may contribute to the
thrombotic microangiopathy, and are indicated only by
Would you obtain any other diagnostic studies?
active hemorrhage or prior to a procedure. Early hydration
during the diarrheal phase may lessen the severity of renal Additional tests to include are stool culture for E. coli, stool
insufficiency. Most children with Shiga toxin-associated test for Shiga toxin, and liver enzymes. A renal ultrasound
HUS survive the acute phase and recover normal renal may be helpful in determining category of acute kidney
function. injury.

Further Reading
Cody, E. and Dixon, B.P. (2019). Hemolytic uremic syndrome. Dixon, B. and Gruppo, R. (2018). Atypical hemolytic uremic
Pediatric Clinics of North America 66(1): 235–246. syndrome. Pediatric Clinics of North America 65(3): 509–525.
Downloaded from https://onlinelibrary.wiley.com/doi/ by National Institute Of Standard, Wiley Online Library on [06/03/2024]. See the Terms and Conditions (https://onlinelibrary.wiley.com/terms-and-conditions) on Wiley Online Library for rules of use; OA articles are governed by the applicable Creative Commons License
129

31

Child with Persistent Fever


Misty Ellis
Norton Children’s Hospital, Louisville, KY, USA

A 5-month-old male is admitted with persistent fever for Past Surgical History
18 days.
No prior surgeries.

History of Present Illness


Family History
A 5-month-old male presents with persistent daily fever.
The fevers began 18 days ago, with temperature to 38.3 C His father has type 1 diabetes and his mother is healthy.
(101 F). He also developed an erythematous macular rash His sister had a recent upper respiratory infection.
on his torso, back, and neck with redness of feet and hands,
and had two episodes of emesis. On his second day of fever,
his temperature was 39.3 C (102.8 F) and he was taken to Current Status
the local emergency department (ED) where he was diag-
nosed with a viral infection and sent home. His mother In the ED, he was fussy with examination, but was consoled
reports that the rash improved for a few days then returned by his mother. His examination was significant for temper-
in conjunction with intermittent fevers, and eventually ature of 38.5 C (101.3 F), cracked and peeling lips, breath
completely resolved. On day 4 of illness, patient returned sounds clear to auscultation bilaterally with transmitted
for a follow-up visit with his primary care provider (PCP) upper airway sounds, but otherwise no other abnormal
and again was diagnosed with a viral infection. Addition- examination findings.
ally, he was prescribed eye drops for a non-exudative con-
junctivitis, which resolved after 5 days. His rash resolved,
but he continued to have daily fevers greater than 38 C Questions
(100.4 F) (responsive to acetaminophen), and developed
cough, sneezing, and rhinorrhea. By day 9 of fevers, he Answer the following questions using the details provided.
was diagnosed with respiratory syncytial virus (RSV). On
1 Based on the data given, what are the differential diag-
day 12 of fever, his upper respiratory infection symptoms
noses for this child?
were improving, but due to persistent fevers he returned
2 What diagnostic tests would you obtain based on the
to the PCP. The PCP transferred him to the ED for further
differential diagnoses?
evaluation.
Continuation of case: Diagnostic study results are shown
in Tables 31.1 and 31.2. Urinalysis showed sterile pyuria.
Past Medical History 3 What is the most likely diagnosis?
4 What other diagnostic studies should be obtained?
He was born at 36 weeks via cesarean section and did not 5 Based on the diagnosis what is the treatment plan for
require extended hospital stay following birth. this patient?

Cases in Pediatric Acute Care: Strengthening Clinical Decision Making, First Edition. Edited by Andrea M. Kline-Tilford and Catherine M. Haut.
© 2020 John Wiley & Sons Ltd. Published 2020 by John Wiley & Sons Ltd.
Downloaded from https://onlinelibrary.wiley.com/doi/ by National Institute Of Standard, Wiley Online Library on [06/03/2024]. See the Terms and Conditions (https://onlinelibrary.wiley.com/terms-and-conditions) on Wiley Online Library for rules of use; OA articles are governed by the applicable Creative Commons License
130 Cases in Pediatric Acute Care

Table 31.1 Complete metabolic profile. What is the most likely diagnosis?

Sodium 132 mEq/L The most likely diagnosis for this patient is Kawasaki
Potassium 4 mEq/L disease due to the patient’s clinical symptoms of fever,
Chloride 105 mEq/L
erythematous macular rash, redness of hands and feet,
non-exudative conjunctivitis, cracked/peeling lips, hypoal-
Carbon dioxide 22 mEq/L
buminemia, thrombocytopenia, elevated CRP, anemia, and
Blood urea nitrogen 14 mg/dL
sterile pyuria.
Creatinine 0.2 mg/dL Kawasaki disease is an acute self-limited vasculitis of
Glucose 98 mg/dL unknown etiology that occurs predominantly in infants
Albumin 2.4 g/dL and children. If not treated early with high-dose intrave-
Aspartate aminotransferase 153 U/L nous immunoglobulin, one in five children develop coro-
Alanine aminotransferase 200 U/L nary artery aneurysms. The clinical presentation of
Total bilirubin 0.5 mg/dL Kawasaki disease varies over time, with the clinical course
conventionally divided into three stages: acute, subacute,
Erythrocyte sedimentation rate 8.5 mm/hour
and convalescent phase. The acute stage begins with an
C-reactive protein 6.4 mg/dL
abrupt onset of fever and lasts approximately 7–14 days.
During this phase, the fever is typically high, usually ran-
ging from 39 to 40.5 C (101.2–104.9 F), and is unresponsive
Table 31.2 Complete blood count. to antibiotics or antipyretics. Additional classic clinical cri-
teria of Kawasaki disease include at least four of the five
White blood cell count 23 000/mm3 following features: extremity changes, polymorphous exan-
Hemoglobin 9.8 g/dL thema, conjunctival injection, oral/lip changes, and cervi-
Hematocrit 29.5% cal lymphadenopathy. Patients with Kawasaki disease
Platelet count 958 000/mm3 will usually have an elevated CRP and/or ESR. Kawasaki
Neutrophils 49%
disease is treated with immunoglobulin and aspirin with
the goal of preventing the development of coronary artery
Eosinophils 1%
aneurysms.
Monocytes 6%
Lymphocytes 44%
Bands 0% What other diagnostic studies should be obtained?
Other important diagnostic studies to include for this
patient would be an echocardiogram and an electrocardio-
Rationale and Evidence-based Practice gram (EKG). An echocardiogram is the primary imaging
Explanation modality for cardiac assessment during the acute phase
(Figures 31.1 and 31.2). An EKG may demonstrate sinus
Based on the data given, what are the differential node dysfunction with prolonged PR interval, and non-
diagnoses for this child? specific ST- and T-wave changes. Invasive angiography
(cardiac catheterization) can be of value for long-term fol-
Differential diagnoses for this child include RSV, adenovi- low-up of coronary artery aneurysm evaluation but is rarely
rus, enterovirus, parainfluenza virus, bacterial sepsis, utilized during the acute phase (Figure 31.3).
staphylococcal or streptococcal toxic shock syndrome,
and Kawasaki disease.
Based on the diagnosis what is the treatment plan
for this patient?
What diagnostic tests would you obtain based on
The goal of treatment is to promptly eradicate systemic and
the differential diagnoses?
tissue-level inflammation and to prevent development of
Diagnostic studies to include are complete blood count, aneurysms. Primary treatment during the acute phase is
complete metabolic panel, erythrocyte sedimentation rate high-dose intravenous immune globulin (IVIG) for
(ESR), C-reactive protein (CRP), urinalysis, urine culture, immune mediation. Aspirin is often used for antiplatelet
blood culture, and respiratory virus panel. effect to prevent the development of coronary aneurysm
Downloaded from https://onlinelibrary.wiley.com/doi/ by National Institute Of Standard, Wiley Online Library on [06/03/2024]. See the Terms and Conditions (https://onlinelibrary.wiley.com/terms-and-conditions) on Wiley Online Library for rules of use; OA articles are governed by the applicable Creative Commons License
Chapter 31 Child with Persistent Fever 131

RCA

Aorta

Figure 31.1 Echocardiogram showing aorta and dilated right


coronary artery (RCA).

Figure 31.3 Cardiac catheterization showing left coronary artery


dilation.

Aorta
LCA
The following important points should be remembered.

• Infants less than 6 month of age are at significant risk of


aneurysm formation and require echocardiography every
few days until stable as well as frequently upon discharge.

• Patients with significant coronary artery aneurysm devel-


opment are at high risk for thrombosis, and therefore sys-
temic anticoagulation in addition to antiplatelet therapy
should be administered.

• Measles and varicella immunizations should be deferred


for 11 months after receiving IVIG administration.

Figure 31.2 Echocardiogram showing aorta and left coronary


artery (LCA).
• Reye syndrome has been reported in patients taking
high-dose aspirin for a prolonged period following Kawa-
saki disease. All children over 6 months of age and their
families should receive a seasonal influenza vaccine prior
from thrombus. Management is tailored based on repeat to discharge from the hospital. Inactivated vaccine
echocardiograms and measurement of coronary arteries. should be administered to children on aspirin therapy.

Further Reading
McCrindle, B., Rowley, A., Newburger, J., et al. (2017). Newburger, J., Takahashi, M., and Burns, J. (2016). Kawasaki
Diagnosis, treatment and long-term management of disease. Journal of the American College of Cardiology 67:
Kawasaki disease. Circulation 135: e927–e999. doi: 10.1161/ 1738–1749. doi: 10.1016/j.jacc.2015.12.073.
cir.0000000000000484.
Downloaded from https://onlinelibrary.wiley.com/doi/ by National Institute Of Standard, Wiley Online Library on [06/03/2024]. See the Terms and Conditions (https://onlinelibrary.wiley.com/terms-and-conditions) on Wiley Online Library for rules of use; OA articles are governed by the applicable Creative Commons License
133

32

Quality Improvement Case Study


Peggy Dorr
University of Maryland Medical Center, Baltimore, MD, USA

An acute care nurse practitioner (ACNP) working in a pedi- 4 What would be an appropriate goal(s) for this problem?
atric emergency department (ED) has noted a lack of under- What are some guidelines to use when developing or
standing on how to correctly use a metered-dose inhaler defining a goal?
(MDI). Discussions with families indicate that most children, 5 What would be a way to filter literature articles to
regardless of age, and many parents are either not comfortable ensure the retrieved data is accurate and applicable?
using the inhaler or are demonstrating incorrect technique 6 Besides developing the standardized education pro-
with the device. There is also noted variance in discharge gram and a plan for implementation with the nursing
teaching, possibly due to a recent influx of new graduate staff, what other factors must be discussed and
nurses working on the unit. The nurse practitioner decides addressed?
it would be beneficial to evaluate this problem and potentially 7 Prior to implementation, is it necessary to submit your
develop a solution to benefit both staff and patients. project to your Institutional Review Board for approval?
Understanding there are numerous stakeholders involved 8 Are there any final steps that could or need to be taken?
in this issue, the nurse practitioner begins to develop a team
that can discuss this perceived issue and decide if interven-
tion is necessary. After a team is in place, an improvement
model can help provide guidance and structure as the prob- Rationale and Evidence-based Practice
lem is assessed and a plan is developed. There are numerous Evaluation
models that can be utilized to guide this quality improve-
ment (QI) project. Who should be included in this QI team and why?
The QI team should include the following key members.

Questions • Nursing representative who would be instrumental in the


education process and best understands the needs of the
nursing group.


Answer the following questions using the details provided.
Respiratory therapists routinely work with these children
1 Who should be included in this QI team and why? and parents and should contribute to and support the
2 Of the following, which process is best for this situation unit’s choice of standardized education.
and why?
a) Plan, Do, Study, Act (PDSA) cycle.
• Pharmacy team member represents the prescriptive
impacts, as orders for medication and medical equip-
b) Root cause analysis (RCA). ment, such as a spacer, would potentially be obtained
c) Six Sigma (define, measure, analyze, improve and from the hospital pharmacy. Consistency in education
control or DMAIC). and any process impacts that occur from orders written
d) Failure mode and effects analysis (FMEA). within the inpatient or outpatient settings need to be
addressed.
3 What would be an appropriate question to guide
analysis of the problem and devise a potential solution
• Medical representative, in addition to the lead nurse
practitioner, should be invited to join this group. Physi-
given this case scenario? cians and physician trainees should be aware of changes
Cases in Pediatric Acute Care: Strengthening Clinical Decision Making, First Edition. Edited by Andrea M. Kline-Tilford and Catherine M. Haut.
© 2020 John Wiley & Sons Ltd. Published 2020 by John Wiley & Sons Ltd.
Downloaded from https://onlinelibrary.wiley.com/doi/ by National Institute Of Standard, Wiley Online Library on [06/03/2024]. See the Terms and Conditions (https://onlinelibrary.wiley.com/terms-and-conditions) on Wiley Online Library for rules of use; OA articles are governed by the applicable Creative Commons License
134 Cases in Pediatric Acute Care

to discharge education, may contribute with recent leansixsigmaonline/blog/healthcare-advancement-with-


research or evidence-based medical practice, and may lean-six-sigma/). The problem-solving methodology within
contribute current American Academy of Pediatrics this principle is the DMAIC method, which focuses on
(AAP) guidelines or recommendations. improving existing processes and if any deficiencies in these

• Nurse manager for the unit has financial and staffing


responsibilities. Nursing staff will be required to attend
processes are resolved, they will have the potential to
improve outcomes and are measurable (https://goleansix-
educational sessions that may impact staffing numbers sigma.com/dmaic-five-basic-phases-of-lean-six-sigma).
or incur financial expenditures, which will impact the The DMAIC mnemonic refers to specific steps defined
unit budget. as follows.

A team working on a QI problem must comprise all key Define: the problem is defined and focused. Problems
stakeholders. A stakeholder is “anyone who has an interest should be very specifically written to allow for better
in a project or can influence its success or failure” (Silver analysis and resolution.
et al., 2016, p. 893). To ensure success, all stakeholders must Measure: in the measure phase, the team needs to know
be identified, even those persons who can have a negative how the process operates and perform data collection
influence on the outcome of the suggested project. Stake- regarding best practices. This step includes the
holder mapping can be helpful for identifying all potential performance of a gap analysis via a thorough literature
internal and external individuals and defining the relation- review.
ship they have to the problem being addressed (Swanson Analyze: in this phase the team decides what are the actual
and Pearlman, 2017, p. 113). causes of the problem and these causes are validated
After all stakeholders have been identified, decisions by data.
must be made regarding which of these to engage in the Improve: the goal is to determine a solution to the problem
project. Consideration must be given to motivations (e.g. at hand, select the best solution given the team’s input,
patient care, finances, process efficiency, and staff satisfac- implement the solution, and collect data to confirm there
tion) or perspective of potential members. The team of sta- is measurable improvement.
keholders can be dynamic, based on the needs of the project Control: this phase is about sustainability. The implemen-
at a specific time. tation plan is reviewed and modified or refined as
In this setting, QI project groups are typically composed necessary.
of interdisciplinary team members. The team should be
small enough to promote efficiency and large enough to PDSA is a rapid and continuous process, more commonly
provide expertise in all areas impacted by the project. used for testing a potential change in a real work setting.
Members should be committed to review of the problem, This method implements the change on a small scale, then
development of a solution, and the implementation. Care- analyzes the results, refines the change, and re-implements
ful analysis of stakeholders is important to ensure accept- (https://innovations.ahrq.gov/qualitytools/plan-do-study-
ance of the suggested change or improvement, but act-pdsa-cycle), allowing for a rapid evaluation and change
influence is not the only criterion to consider. The team process. While this method of implementing and evaluat-
will require participants to take on many roles including ing change is routinely referenced for QI projects and will
team lead, clinical lead, technical experts, and an execu- be utilized during this project, the details provided in the
tive sponsor (Silver et al., 2016, p. 894). Ensuring roles DMAIC methodology will provide concrete guidance for
such as these are filled will help balance the team and this team through the entire process.
encourage success. An RCA is a retrospective analysis of an untoward event
and an assessment of factors that led to the event and ave-
nues to avoid it in the future.
Of the following, which process is best for this
FMEA is a systematic prospective risk assessment to
situation and why?
anticipate vulnerabilities and potential adverse outcomes
Based on the identified issue, the Six Sigma process of a process in order to identify the parts of that process that
(DMAIC) best fits this situation. Six Sigma methodologies most need change (http://www.ihi.org/resources/Pages/
are used in a range of industries to improve process effi- Tools/FailureModesandEffectsAnalysisTool.aspx).
ciency and reduce defects. In healthcare, this means a The first step in guiding an analysis of the problem and
reduction in medical errors, an increase in customer satis- devising a potential solution is to develop an answerable
faction, and a decrease in costs (https://www.purdue.edu/ evidence-based practice question.
Downloaded from https://onlinelibrary.wiley.com/doi/ by National Institute Of Standard, Wiley Online Library on [06/03/2024]. See the Terms and Conditions (https://onlinelibrary.wiley.com/terms-and-conditions) on Wiley Online Library for rules of use; OA articles are governed by the applicable Creative Commons License
Chapter 32 Quality Improvement Case Study 135

What would be an appropriate question given this c) Parents/guardians will immediately feel more confi-
case scenario? dent in helping their child use the MDI.
d) Patients and parents will demonstrate correct use of
A PICO format can help with question development.
an MDI inhaler.
P Patient, population or problem: consider attributes that
help narrow the focus. 2 All nursing and respiratory staff of the pediatric unit
I Intervention: this can be a treatment or clinical or will complete the standardized MDI education within
education intervention.
C Comparison with other interventions: will the intervention 1 month of roll-out.
be compared to another intervention? Not all questions
These goals will help guide project design and the data
have comparisons.
O Outcomes: identify the outcomes of interest. collection process. As the group decides what would be
an appropriate goal(s), they must also decide how to meas-
An appropriate PICO question for this project would be: ure attainment of that goal. Does a tool already exist or does
one need to be developed? This will add time to project
For pediatric patients, ages 5–18 years old and their development and implementation.
parents/guardians, does the use of standardized edu- After the group agrees on the problem and the goal, a gap
cation on the correct use of MDI device increases the analysis must be performed. This process informs the group
accuracy of its use and the comfort level of users with on the differences between the evidence-based standard of
the device? care and their current practice. Key words are discussed
and all members perform a review of the literature. Numer-
The developed question will drive the strategies used to ous articles are brought back to the next meeting for review.
search for evidence. Making the evidence-based practice
question as specific as possible helps to identify and narrow What would be a way to filter literature articles to
search terms, focus the project, and communicate the target ensure the retrieved data is accurate and
population so the translation is appropriate (Dang and applicable?
Dearholt, 2017).
The literature review demonstrates several research and QI
The problem of inconsistent MDI education is agreed
studies which have confirmed that standardized education
upon by the group. Now a decision must be made to deter-
has a positive impact on accuracy of MDI use and improved
mine as to what is a measurable goal that indicates educa-
outcomes in asthma management. Determining a list of
tion has had an impact on the problem.
guidelines for literature review can be accomplished by
using filters. For example, the following could be included:
What would be an appropriate goal(s) for this only data from pediatric reviews or studies, English lan-
problem? What are some guidelines to use when guage only, within 5 years of current date, only scientific
developing or defining a goal? research from known journal/site (do not use Wikipedia,
The acronym SMART can help when defining a goal. but peer-reviewed journals), using well-known medical
SMART stands for specific, measurable, actionable, relevant, subject headings (MeSH) as defined in PubMed or the
and time bound. A SMART objective format provides a Cumulative Index to Nursing and Allied Health
structured approach, sets the stage for measuring perfor- Literature (CINAHL).
mance, succinctly communicates intended impact and
progress to stakeholders, and concretely describes how Besides developing the standardized education
goals will be met. program and a plan for implementation with the
There may be several goals for this project. nursing staff, what other factors must be discussed
and addressed?
1 After education is complete:
Barriers to implementation
a) Nursing staff will feel confident providing the newly
standardized MDI education to patients of varying
• Time for staff education: time away from clinical care,
cost of staff attendance.
ages and their families.
b) Documentation of MDI education in the electronic
• Staff members other than nursing that need to be aware
of new education format (e.g. respiratory therapy, medi-
medical record (EMR) will be greater than 90% cine, rehabilitative services, child life).
within 30 days of implementation.
• Integration into EMR for documentation.
Downloaded from https://onlinelibrary.wiley.com/doi/ by National Institute Of Standard, Wiley Online Library on [06/03/2024]. See the Terms and Conditions (https://onlinelibrary.wiley.com/terms-and-conditions) on Wiley Online Library for rules of use; OA articles are governed by the applicable Creative Commons License
136 Cases in Pediatric Acute Care

• Recruitment of “champions” or key adopters to ensure


sustainability over time and act as a clinical resource
To better understand if a project needs IRB approval,
research must be clearly defined in comparison to QI, as
for staff with regard to process and content. both types of activities can be data driven and involve
human participants (Table 32.1). A group from the Has-
tings Center defined QI as “systematic, data-guided activ-
Measurement of outcomes ities designed to bring about immediate improvements in

• Process measures: for example, is the education tool easy


to use, is documentation easy, is there a checklist to
health delivery in particular settings” (Baily, 2016),
whereas research is a systematic investigation that includes
ensure completeness? research development, testing and evaluation designed to

• Outcome measures: for example, is there before and after


measurement of knowledge on use of MDI, is there
develop or contribute to generalizable knowledge.
IRB approval for QI activities is typically required if:


improvement after education, are patients tracked at
future hospitalizations or ED visits for sustained the project seeks to develop new knowledge or validate
new treatments;


performance?

• Patient and family satisfaction with the teaching method, a standard research methodology such as randomization
is used;

••
the perceived necessity of the education and the out-
comes as seen by the family are also important. a protocol is rigid and fixed;
there is outside funding; and/or
The team has developed a standardized education pro-
gram for patients and families as well as an educational
• risks from participation are greater than minimal.

There may be institutional variability about submitting


process for staff. They have also developed evaluation tools
QI process requests through the IRB but if there is a rapid
to assess efficacy of the education for patients, families
review process that assigns the project a title of “exempt
and staff.
research” or “not human subjects” research, this is encour-
aged. When in doubt, contact your IRB staff or process a
Prior to implementation, is it necessary to submit
submission.
your project to your Institutional Review Board for
Implementation is complete and the education proc-
approval?
ess has been ongoing for 3 months. There are several
The purpose of an Institutional Review Board (IRB) is to unit-based champions who are available on various
protect the rights and welfare of human research subjects shifts and the ACNP, nurse manager and nursing repre-
who are involved in biomedical or behavioral research. sentative have met monthly to review outcomes to date

Table 32.1 Defining research versus quality improvement.

Human subject research Quality improvement

Purpose Designed to develop or contribute to generalizable knowledge Apply known solutions to a focused problem or
process
Starting point Knowledge seeking is independent of routine care and Knowledge seeking is integral to ongoing
intended to answer a question or test a hypothesis management system for delivering healthcare
Design Systematic, methodological approach Adaptive, iterative design; less rigorous, often
involves a PDSA
Benefits Might or might not benefit current subjects Directly benefits a process, system or program
Risks May put subjects at risk Does not increase risk to patients with exception of
possible patient privacy or data
IRB required Always required Typically not required but need to work through
process
Dissemination Expected to publish and present as an IRB-approved research Internal communication, may publish or present as
study QI work
Adoption of Little urgency to disseminate results quickly Results rapidly adopted into local care delivery
results

IRB, Institutional Review Board; PDSA, Plan, Do, Study, Act cycle.
Source: Adapted from Gregory (2015) and Merrill (2015) and (https://irb.research.chop.edu/quality-improvement-vs-research).
Downloaded from https://onlinelibrary.wiley.com/doi/ by National Institute Of Standard, Wiley Online Library on [06/03/2024]. See the Terms and Conditions (https://onlinelibrary.wiley.com/terms-and-conditions) on Wiley Online Library for rules of use; OA articles are governed by the applicable Creative Commons License
Chapter 32 Quality Improvement Case Study 137

and discuss any issues that have come up. The process
seems to be well integrated into the unit’s clinical
• A review of recent ED admissions or readmissions for
asthma exacerbations compared to a time period prior
practice. to implementation would be beneficial to evaluate
impact on patient care quality as well as financial savings
Are there any final steps that could or need to for family and institution.
be taken?
• The team should look to disseminate the results of the

• Modifications to the newly implemented plan based on


feedback or data results. The team needs to review all
project within their institution as well as to a broader
audience through a manuscript or presentation at an
appropriate conference.
results and determine the need for changes to process,
documentation or implementation based on those
results. Satisfaction data must be reviewed as well.

References
Baily, M.A. (2016). Quality improvement methods in health Merrill, K.C. (2015). Is this quality improvement or research?
care. Hastings Center Bioethics Briefings for Journalists, American Nurse Today 10(4). https://www.
Policymakers and Educators. https://www. americannursetoday.com/quality-improvement-research/
thehastingscenter.org/briefingbook/quality-improvement- (accessed 11 June 2019).
methods-in-health-care/ (accessed 11 June 2019). Silver, S.A., Harel, Z., McQuillan, R., et al. (2016). How
Dang, D. and Dearholt, S.L. (2017). Johns Hopkins Nursing to begin a quality improvement project. Clinical
Evidence-Based Practice: Model and Guidelines, 3rd edn. Journal of the American Society of Nephrology 11:
Indianapolis, IN: Sigma Theta Tau International. 893–900.
Gregory, K.E. (2015). Differentiating between research and Swanson, J.R. and Pearlman, S.A. (2017). Roadmap to a
quality improvement. Journal of Perinatal and Neonatal successful quality improvement project. Journal of
Nursing 29(2): 100–102. Perinatology 37: 112–115.

Further Reading
Agency for Healthcare Research and Quality (2013). Plan-Do- Institute for Healthcare Improvement. Science of
Study-Act (PDSA) Cycle. https://innovations.ahrq.gov/ improvement: how to improve. http://www.ihi.org/
qualitytools/plan-do-study-act-pdsa-cycle resources/Pages/HowtoImprove/
Children’s Hospital of Philadelphia. Quality improvement vs ScienceofImprovementHowtoImprove.aspx
research. https://irb.research.chop.edu/quality- McQuillan, R.F., Silver, S.A., Harel, Z., et al. (2016). How to
improvement-vs-research (accessed 11 June 2019). measure and interpret quality improvement data. Clinical
DMAIC: the 5 Phases of Lean Six Sigma. https:// Journal of the American Society of Nephrology 11: 908–914.
goleansixsigma.com/dmaic-five-basic-phases-of-lean-six- Purdue University Lean Six Sigma Online. Advances in
sigma (accessed 11 June 2019). healthcare with Lean Six Sigma. https://www.purdue.edu/
Institute for Healthcare Improvement (http://www.ihi.org). leansixsigmaonline/blog/healthcare-advancement-with-
Institute for Healthcare Improvement. Failure modes and lean-six-sigma/ (accessed 11 June 2019).
effects analysis (FMEA) tool. http://www.ihi.org/resources/ White, K.M., Dudley-Brown, S., and Terhaar, M.F. (2016).
Pages/Tools/FailureModesandEffectsAnalysisTool.aspx Translation of Evidence into Nursing and Health Care, 2nd
(accessed 11 June 2019). edn. New York: Springer Publishing Company.
Downloaded from https://onlinelibrary.wiley.com/doi/ by National Institute Of Standard, Wiley Online Library on [06/03/2024]. See the Terms and Conditions (https://onlinelibrary.wiley.com/terms-and-conditions) on Wiley Online Library for rules of use; OA articles are governed by the applicable Creative Commons License
139

33

Newborn Immediately Post Delivery


Heather Schniepp
University of Texas Health, San Antonio, TX, USA

A newborn infant is brought to the newborn nursery post Rationale and Evidence-based
precipitate vaginal delivery 2 hours after mother’s arrival Practice Explanation
to the hospital. Apgar scores were 6 at 1 minute and 8 at
5 minutes. There was no nuchal cord and no reports of res- After drying and warming this infant and ensuring
piratory distress or meconium. stability, what diagnostic testing, in addition to
normal newborn laboratory evaluations and
physical examination, is anticipated for this
History of Present Illness
neonate on arrival in the newborn nursery?
The 17-year-old mother has received no medications during A 12-lead EKG and echocardiogram are indicated as soon
delivery and has not been able to give an adequate health as the neonate is stabilized. The EKG is required to evaluate
history or pregnancy history. She had little prenatal care for heart block in infant of mother with possible lupus. The
and has not been seen in the obstetrics clinic since the infant will also need an echocardiogram to determine if
fourth month of her pregnancy where a note suggests a pos- there are any structural heart defects. Neonates can also
sible maternal diagnosis of lupus erythematosus. Mother be tested for antibodies associated with autoimmune syn-
denied alcohol, drug, and tobacco use during pregnancy. dromes. Maternal laboratory testing can also be completed
for systemic lupus erythematosus (SLE), as some mothers
may be unaware that they have an autoimmune disease.
Current Status Of note, even though a mother may carry these autoantibo-
dies, only a small number of fetuses/neonates develop
The newborn’s initial vital signs are as follows: birthweight
heart block.
3120 g, length 50 cm, head circumference 34.5 cm, heart
Because of lack of prenatal care and the mother being a
rate 138 bpm, blood pressure 72/34 mmHg, respiratory rate
poor historian, the infant should have urine and meconium
52 breaths per minute, and temperature 37.1 C (98.8 F)
toxicology screening to evaluate for drug use during preg-
axillary.
nancy. There should be a low threshold to begin treatment
if you suspect neonatal infections. Close monitoring of the
Questions infant’s vital signs including heart rate and temperature are
important as well.
Answer the following questions using the details provided.
Continuation of case: The results of the 12-lead electro-
1 After drying and warming this infant and ensuring sta- cardiogram (EKG) are shown in Figure 33.1. Diagnostic
bility, what diagnostic testing, in addition to normal study results were as follows: urine and meconium toxicol-
newborn laboratory evaluations and physical examina- ogy screens negative; echocardiogram showed normal
tion, is anticipated for this neonate on arrival in the heart structure; and physical examination was within nor-
newborn nursery? mal limits except capillary refill of 4 seconds to feet.

Cases in Pediatric Acute Care: Strengthening Clinical Decision Making, First Edition. Edited by Andrea M. Kline-Tilford and Catherine M. Haut.
© 2020 John Wiley & Sons Ltd. Published 2020 by John Wiley & Sons Ltd.
Downloaded from https://onlinelibrary.wiley.com/doi/ by National Institute Of Standard, Wiley Online Library on [06/03/2024]. See the Terms and Conditions (https://onlinelibrary.wiley.com/terms-and-conditions) on Wiley Online Library for rules of use; OA articles are governed by the applicable Creative Commons License
140 Cases in Pediatric Acute Care

Figure 33.1 12-Lead EKG.

2 What rhythm is noted on the infant’s EKG?


3 What treatment may be needed?
4 What anticipatory guidance should be provided to the
family?

What rhythm is noted on the infant’s EKG?


This EKG shows atrioventricular dissociation, probable
complete heart block. This is a 1-day-old infant with a post-
natal diagnosis of heart block.
Heart block is identified as first-, second-, or third-degree
heart block. First-degree heart block is defined by a PR
interval of greater than 0.2 seconds where a normal PR
interval is 0.12–0.2 seconds. Mobitz type I or Wenckebach
second-degree heart block occurs when the P wave
becomes blocked because of the prolonged PR interval.
Mobitz type II second-degree heart block occurs when there
is a sudden block of impulses from the atria to the ventricles
with no prolonged PR interval. Third-degree or complete
heart block occurs when there is no electrical coordination
between the atria and the ventricles.
Congenital atrioventricular block is very rare, occurring Figure 33.2 Chest radiograph with internal pacemaker in 6-day-
once in about 20 000 live births annually worldwide. The old neonate.
Downloaded from https://onlinelibrary.wiley.com/doi/ by National Institute Of Standard, Wiley Online Library on [06/03/2024]. See the Terms and Conditions (https://onlinelibrary.wiley.com/terms-and-conditions) on Wiley Online Library for rules of use; OA articles are governed by the applicable Creative Commons License
Chapter 33 Newborn Immediately Post Delivery 141

Figure 33.3 12-Lead EKG post pacemaker placement.

majority of cases are due to maternal transfer of autoantibo- determine if and when a permanent pacemaker system
dies to the fetus causing an immune-mediated cardiac dis- can be placed. Temporary pacing can be completed
ease or neonatal lupus. Mothers may have antibodies due through transesophageal pacing or with temporary trans-
to the following disorders: SLE, Sjögren syndrome, antipho- venous pacing wires placed surgically and connected to an
spholipid syndrome, or other autoimmune syndrome. Other external pacemaker box. A permanent pacemaker is typi-
causes can include structural heart defects, which cause dis- cally placed surgically in neonates and infants by a pedi-
turbance between the atria and ventricles, and, less com- atric cardiothoracic surgeon (Figure 33.2). Placement of
monly, maternal drug use or fetal viral infections. a pacemaker is in consultation with the electrophysiolo-
gist. Intravenous epinephrine, atropine, or dopamine
What treatment may be needed? infusions may be used to improve chronotropy until a
pacemaker can be placed.
Fetal echocardiography is currently the best diagnostic tool
to detect fetal heart block. If left untreated, the mortality
What anticipatory guidance should be provided to
of fetal and neonatal heart block is between 14 and 34%.
the family?
Mortality increases in premature neonates, those with
fetal hydrops, and those with ventricular rates less than This infant will require close follow-up with an electrophys-
55 bpm. Prenatal or transplacental options, including iologist to monitor heart rhythm and pacemaker settings
maternal administration of dexamethasone or terbutaline, and a pediatric rheumatologist for treatment and monitor-
are not used in all cases due to the side effects for mother ing of neonatal lupus.
and fetus. The follow-up EKG (Figure 33.3) of the infant after pace-
Postnatal cardiac pacing and pharmacologic therapy is maker placement demonstrated atrial-sensed ventricular-
the mainstay of treatment in severe bradycardia and paced rhythm. Pacemaker spikes are noted prior to each
hemodynamic insufficiency. The size of the neonate will QRS complex.
Downloaded from https://onlinelibrary.wiley.com/doi/ by National Institute Of Standard, Wiley Online Library on [06/03/2024]. See the Terms and Conditions (https://onlinelibrary.wiley.com/terms-and-conditions) on Wiley Online Library for rules of use; OA articles are governed by the applicable Creative Commons License
142 Cases in Pediatric Acute Care

Further Reading
Baruteau, A.-E., Pass, R.H., Thambo, J.-B., et al. (2016). Johnson, B. (2014). Overview of neonatal lupus. Journal of
Congenital and childhood atrioventricular blocks: Pediatric Health Care 28: 331–341. doi: 10.1016/j.
pathophysiology and contemporary management. European pedhc.2013.07.015.
Journal of Pediatrics 175: 1235–1248. doi: 10.1007/s00431- Levesque, K., Morel, N., Maltret, A., et al. (2015). Description of
016-2748-0. 214 cases of autoimmune congenital heart block: results of
Brito-Zeron, P., Izmirly, P.M., Ramos-Casals, M., Buyon, J.P., the French neonatal lupus syndrome. Autoimmunity
and Khamashta, M.A. (2016). Autoimmune congenital heart Reviews 14: 1154–1160. doi: 10.1016/j.autrev.2015.08.005
block: complex and unusual situations. Lupus 25: 116–128.
doi: 10.1177/0961203315624024.
Downloaded from https://onlinelibrary.wiley.com/doi/ by National Institute Of Standard, Wiley Online Library on [06/03/2024]. See the Terms and Conditions (https://onlinelibrary.wiley.com/terms-and-conditions) on Wiley Online Library for rules of use; OA articles are governed by the applicable Creative Commons License
143

34

A 5-Year-Old with Poor Weight Gain and Dehydration


Kimberly Young-Conner
Nemours/Alfred I. duPont Hospital for Children, Wilmington, DE, USA

A 5-year-old female presents to the primary care office for a included an auditory brainstem response hearing test,
routine well child appointment at 4 p.m. She is accompa- and was found to have bilateral sensorineural hearing loss.
nied by her maternal grandmother and grandfather who She has had consistent follow-up with speech and language
are also her legal guardians. This is the first visit for the services and now has a left cochlear implant.
child to see this pediatric provider who is not familiar with The child has received all her required immunizations as
the patient and family. well as the influenza vaccine each year. Her height has been
consistently between the 25th and 40th percentiles, with
her weight between the 3rd and 6th percentiles. There is
History of Present Illness no other medical history.

The patient is in kindergarten and was last seen in the


primary care office one year ago. She is scheduled today Past Surgical History
for a routine health visit, but grandparents voice concern
over her slow weight gain. They feel that she just can’t gain The child had a left cochlear implant at age 4, and is
weight even though she “eats all the time.” Grandfather followed by the local children’s hospital.
states for a snack she might eat a frozen dinner and then
still eats a full cooked meal. Grandparents report a recent
diagnosis of strep throat, along with an asthma exacerba-
tion, which was treated in an urgent care center about Family History
3 weeks ago. The child received a 10-day course of
amoxicillin, a 5-day course of prednisolone 2 mg/kg daily, Mother of the child, who is not currently involved in her
and has been using albuterol metered-dose inhaler (MDI) care, has uncontrolled insulin-dependent diabetes. The
two to three times per day up until this current week. child has an older sibling aged 8 who also lives with mater-
nal grandparents and is reported to be healthy. Maternal
grandparents cannot provide information about the child’s
biological father or his family history. There is no known
Past Medical History
family history of hearing concerns nor of juvenile diabetes.
The child was born by vaginal delivery to a mother who has
type 1 diabetes mellitus and received appropriate prenatal
care. The infant was large for gestational age, weighing 4.2 Current Status
kg and did not initially pass her newborn hearing screen,
but did pass the second attempt. At age 8 months, the child The child’s height and weight remain in the same percen-
was admitted to the hospital for respiratory syncytial virus tiles, with a weight gain of 1.4 kg in one year. Her heart rate
bronchiolitis and has had several episodes of wheezing is 122 bpm and her blood pressure is 96/44 mmHg. The
since then, but none requiring hospitalization. child appears thin and very anxious, with dry lips and
At age 2, this child had not met appropriate language dry mucous membranes. She has several enlarged cervical
milestones and had another hearing evaluation, which lymph nodes on palpation of her neck and is noted to have a
Cases in Pediatric Acute Care: Strengthening Clinical Decision Making, First Edition. Edited by Andrea M. Kline-Tilford and Catherine M. Haut.
© 2020 John Wiley & Sons Ltd. Published 2020 by John Wiley & Sons Ltd.
Downloaded from https://onlinelibrary.wiley.com/doi/ by National Institute Of Standard, Wiley Online Library on [06/03/2024]. See the Terms and Conditions (https://onlinelibrary.wiley.com/terms-and-conditions) on Wiley Online Library for rules of use; OA articles are governed by the applicable Creative Commons License
144 Cases in Pediatric Acute Care

“sweet” odor to her breath. Other physical examination fea-


tures are within normal limits.
• pancreatic islet autoantibodies and C-peptide levels,
which are useful in differentiating between type 1 and
type 2 diabetes mellitus;

Questions • thyroid stimulating hormone, antithyroid peroxidase and


antithyroid antibodies;

Answer the following questions using the details provided.


• celiac antibody panel.

Continuation of case: Normal physical examination


1 Despite the setting, the child is evaluated, what would results were reviewed with guardians. Based on the
be the three top differential diagnoses? growth chart and previous measurements, there was a
2 What diagnostic testing should be considered? concern for poor weight gain along with increased appetite.
A glucometer was not available in the office, so a urinalysis
dipstick test was accomplished and this revealed a glucose
of over 500 mg/dL, no leukocytes or bacteria, and with a
Rationale and Evidence-based Practice trace of blood and ketones. The presence of glucosuria
Explanation and ketonuria along with physical findings and poor weight
gain or weight loss in a child do indicate the possibility of
Despite the setting, the child is evaluated. What diabetes, typically at this age type 1 or insulin-dependent
would be the three top differential diagnoses? diabetes mellitus.
The three top differential diagnoses for this child include These results were shared with the child’s grandparents
failure to thrive, steroid-induced hyperglycemia, and type along with the potential diagnosis which would be further
1 or type 2 diabetes mellitus, the former being most likely confirmed by blood testing and examination at the chil-
given the child’s age, history, and clinical presentation. dren’s hospital. After consulting with pediatric endocrinol-
Since the child has not been weighed or measured in this ogy, the child and her grandparents were referred to the
office in one year, it is difficult to know if she has sustained emergency department at the local children’s hospital, with
weight loss or has historical slow weight gain, so failure to an explanation that she would most likely be admitted for
thrive could be considered. Steroid-induced hyperglycemia further evaluation, rehydration and education about the
would not be likely in a healthy child 2 weeks after comple- disease.
tion of dosing. 3 As the primary care provider for this child, what are
Diabetes mellitus typically presents with symptoms of important management skills to support the new diag-
polyuria, polydipsia, and polyphagia with unexplained nosis and continued management?
weight loss. About 25% of patients will present with diabetic
ketoacidosis on initial diagnosis. Diabetes can be immune-
mediated and noted following a viral or bacterial illness. It As the primary care provider for this child, what
can also be genetic with risk of type 1 diabetes conferred by are important management skills to support the
two susceptible haplotypes, with a 5% lifetime risk for chil- new diagnosis and continued management?
dren who have a first-degree relative with type 1 diabetes. According to the American Diabetes Association, a diagno-
Children with cystic fibrosis have a higher incidence of dia- sis of diabetes must meet criteria that include symptoms of
betes mellitus, occurring in 20% of the patients in this polyuria, polydipsia or weight loss and a random blood glu-
population. cose of 200 mg/dL or greater. This patient’s final diagnosis
was type 1 diabetes mellitus. The child was admitted to the
What diagnostic testing should be considered? hospital and started on a subcutaneous insulin regimen.
Depending on what is available in the primary care office, it Over the course of the hospitalization, the family received
is helpful to obtain at least a urinalysis and/or a fingerstick survival skill education, including carbohydrate counting
blood glucose. The full laboratory work-up for a diagnosis and treatment for hypoglycemia. They also received
of diabetes mellitus would include: instructions on insulin administration and monitoring of
ketones.

•• fasting blood glucose;


basic metabolic panel to assess for dehydration, acidosis
Follow-up with the primary care provider included edu-
cation on the honeymoon period, monitoring for ketones,
and renal function; and sick day rules. The primary care provider should be

• hemoglobin A1c, which indicates diabetes if higher


than 6.5%;
in contact with the endocrinology team for awareness of
laboratory results, insulin dosing and planned follow-up,
Downloaded from https://onlinelibrary.wiley.com/doi/ by National Institute Of Standard, Wiley Online Library on [06/03/2024]. See the Terms and Conditions (https://onlinelibrary.wiley.com/terms-and-conditions) on Wiley Online Library for rules of use; OA articles are governed by the applicable Creative Commons License
Chapter 34 A 5-Year-Old with Poor Weight Gain and Dehydration 145

which would be every 3 months in the endocrinology clinic. learn carbohydrate counting. The school nurse was also
Discussion should focus on the need for healthy nutrition, contacted in order to be included in the child’s care and
proper hydration and exercise along with using labels to monitoring of blood sugars while in school.

Further Reading
Glassford, M.A. (2017). Recognizing type 1 diabetes mellitus complications. Best Practice and Research in Clinical
in children and adolescents. The Nurse Practitioner 42(6): Obstetrics and Gynecology 29(2): 256–269. doi: 10.1016/j.
16–22. doi: 10.1097/01.NPR.0000516121.77126.61. bpobgyn.2014.08.004.
Mitanchez, D., Yzydorczyk, C., Siddeek, B., et al. (2015). The
offspring of the diabetic mother: short- and long-term
Downloaded from https://onlinelibrary.wiley.com/doi/ by National Institute Of Standard, Wiley Online Library on [06/03/2024]. See the Terms and Conditions (https://onlinelibrary.wiley.com/terms-and-conditions) on Wiley Online Library for rules of use; OA articles are governed by the applicable Creative Commons License
147

35

Toddler with Complex Chronic Condition


Jennifer Wright
C.S. Mott Children’s Hospital, Ann Arbor, MI, USA

A 13-month-old male with a medical history significant for Past Medical History
preterm birth, hypoxic ischemic encephalopathy, cerebral
palsy, bronchopulmonary dysplasia, tracheostomy without Preterm birth at 30 weeks’ gestation, hypoxic ischemic
ventilator dependence, gastroesophageal reflux disease encephalopathy, cerebral palsy, bronchopulmonary
(GERD) status post Nissen fundoplication, focal epilepsy dysplasia, tracheostomy without ventilator dependence,
with tonic and myoclonic seizures, and infantile spasms. GERD, focal epilepsy with tonic and myoclonic seizures,
He has presented to the hospital with increased seizures and infantile spasms.
and is admitted to the pediatric intensive care unit
(PICU) with respiratory failure in the setting of status
epilepticus. This is his third PICU admission this month.
Past Surgical History

Circumcision and Nissen fundoplication with g-tube for


History of Present Illness GERD/feeding disorder at 2 months of age. A tracheostomy
for upper airway obstruction was placed at 5 months of
He was discharged from general care the evening prior to age. Bilateral submandibular gland excision for excess
admission. He had been hospitalized the previous 2 weeks secretions at 11 months of age.
with status epilepticus and septic shock in the setting of
pneumonia. During that admission, clobazam was added
to his antiepileptic regimen, and his levetiracetam and phe-
nobarbital doses were increased. Vigabatrin was kept at the Family History
same dose. Ketogenic diet ratio was increased to 4.5 from 4,
with a β-hydroxybutyrate goal of 4–6.5. Non-contributory.
When the toddler arrived home after being discharged,
his mother noticed that he was breathing deeper and harder
without retractions or tachypnea. He was also having more
frequent nasal and tracheal secretions. The morning of Social History
admission, he began having a seizure with shaking of all
extremities, consistent with prior semiology. After 10 min- Lives at home with mother, father, and 3-year-old brother.
utes, he was given clonazepam rescue, but the seizure con- Mother has a 15-year-old son who is living with his father.
tinued. After 20 minutes, he was given rectal diazepam, but She also has three daughters who live with their maternal
the seizure persisted so emergency medical services was grandmother, who has custody. She states she has good
contacted. He was brought to the emergency department support from her mother. Has nursing care five nights
and continued to seize. His glucose was within normal lim- and three days per week due to complex care with
its at the time of transport. tracheostomy.

Cases in Pediatric Acute Care: Strengthening Clinical Decision Making, First Edition. Edited by Andrea M. Kline-Tilford and Catherine M. Haut.
© 2020 John Wiley & Sons Ltd. Published 2020 by John Wiley & Sons Ltd.
Downloaded from https://onlinelibrary.wiley.com/doi/ by National Institute Of Standard, Wiley Online Library on [06/03/2024]. See the Terms and Conditions (https://onlinelibrary.wiley.com/terms-and-conditions) on Wiley Online Library for rules of use; OA articles are governed by the applicable Creative Commons License
148 Cases in Pediatric Acute Care

Current Status been adjusted, his awareness and interactivity


decline and he has increased dependence on ventilation
Vital signs on arrival notable for a temperature of 36.8 C support. Attempts to reduce sedation have resulted in
(98.2 F), heart rate 128 bpm, respiratory rate 29 breaths increased seizures, neuro-agitation, and respiratory
per minute, blood pressure 82/58 mmHg, and oxygen decompensation. His mother has begun confiding in
saturations 98% on humidified trach collar. He continued his nurses that she is worried that he is suffering and
to seize despite receiving lorazepam 1 mg, phenobarbital that it feels that we are no longer doing things “for”
20 mg/kg, and levetiracetam 20 mg/kg. His seizure him but rather “to” him.
stopped after receiving fosphenytoin 20 mg/kg. Mechani- 3 It becomes clear that it is unlikely he will ever success-
cal ventilation was initiated through his tracheostomy. fully wean from ventilator support and continuing the
He was given a fluid bolus and started on maintenance ventilator long term is viewed as extraordinary mea-
intravenous fluids. A venous blood gas analysis and com- sures and outside of his mother’s goals of care. In
plete blood count were largely unremarkable, with a white addition, continued attempts to adjust medications
cell count of 10 200/mm3. Comprehensive metabolic panel are leading to perceived and visible suffering. As
was notable for alkaline phosphatase of 362 U/L (normal you work to transition the child to a comfort-focused
range 44–147 U/L), procalcitonin 0.08 ng/L (normal range plan of care, how will you discuss this with his
0.1–0.49 ng/L), and electrolytes within normal limits. mother?
A respiratory culture was sent from tracheal sections. 4 What are the potential symptoms at end of life and how
His chest radiograph was notable for interval increase in will you prepare both the care team and his family?
perihilar and retrocardiac airspace opacities, most consist-
ent with atelectasis or edema. Pneumonia could not be
excluded so intravenous vancomycin and piperacillin/
tazobactam were initiated. He was admitted to the PICU
for further management. Rationale and Evidence-based Practice
Currently, the PICU and neurology teams are working on Explanation
antiepileptic drug adjustments and readdressing home res-
piratory support needs as he is requiring increased FiO2 and In order to more clearly address his treatment plan
flow during this stay. It is possible that he is chronically going forward, what additional information would
under-ventilated, leading to progressive hypoxia that trig- be important to elicit from his mother?
gers seizures.
As his medical journey continues to unfold, it will be
increasingly important for those caring for him to begin
to elicit from his mother, who is his surrogate decision-
Questions maker, her understanding of his current status along
with the trajectory of his illness. In addition, it will be
Answer the following questions using the details provided. helpful to know what she is hoping for and what her big-
gest worries are for him. Asking open-ended questions
1 As you learn more about this patient, you become con-
about understanding, hopes, and worries can assist the
cerned about the overall trajectory of his care and
provider in gaining insight into a parent’s perspective
increasing medical complexity. In order to more clearly
that will create opportunities to clarify medical status,
address his treatment plan going forward, what addi-
share mutual concerns, and address goals of care going
tional information would be important to elicit from
forward. When asking about hopes and worries, it is
his mother?
important to elicit parental perspectives on both the
2 You learn that while his mother has interest in continu-
immediate situation and expectations for the future.
ing to pursue all treatments or interventions that may
Offering validation, where appropriate, builds trust
help him, she has growing worries about his decline
and alignment. For example, his mother may state “I
in function and overall quality of life. How might you
hope that he will be able to come off the ventilator
further assess alignment of parental and treatment
and not have any more seizures.” A provider response
team goals of care?
that offers validation and moves to seeking alignment
Continuation of case: He continues to have periods in goals may be something like “We hope for that too,
of increased seizure activity despite optimization of but if that may not be possible, what else would be
anti-seizure regimen. As the anti-seizure regimen has important for him?”
Downloaded from https://onlinelibrary.wiley.com/doi/ by National Institute Of Standard, Wiley Online Library on [06/03/2024]. See the Terms and Conditions (https://onlinelibrary.wiley.com/terms-and-conditions) on Wiley Online Library for rules of use; OA articles are governed by the applicable Creative Commons License
Chapter 35 Toddler with Complex Chronic Condition 149

How might you further assess alignment of care” can be received negatively as though the child is no
parental and treatment team goals of care? longer going to receive medical attention. Clarifying with
parents that we will continue to aggressively care for their
Gaining insight into how this child’s daily life is interpreted
child, but with the primary focus now on attending to their
by his mother with respect to enjoyment and suffering,
comfort can ease parental distress and maintain a trusting
along with her focus as his parent, can continue to elucidate
relationship.
what goals are most consistent with her values as his sur-
rogate. Questions that may help further understanding
could include “Tell me what a good and bad day looks like What are the potential symptoms at end of life and
for him” or “How have things changed for him in the past how will you prepare both the care team and his
6 months” and “What is most important to you at this time family?
as his mother?” Being a “good parent” is often critical to
The most common symptoms at end of life are pain, secre-
parental interpretation of current circumstances and can
tions, dyspnea, agitation, sometimes seizures, and nausea/
change as the child’s complex chronic condition changes.
vomiting. It is important to anticipate the most likely
symptoms given a particular clinical condition and to be
As you work to transition the child to a comfort-
prepared to respond effectively and efficiently. In circum-
focused plan of care, how will you discuss this with
stances similar to this child’s, when there is time for
his mother?
thoughtful planning and transition in care, careful consid-
When you have reached a point in delivering medical care eration for the medical plan as well as logistical plan is key.
where the perceived suffering outweighs the real or poten- Time should be spent with mother offering anticipatory
tial benefits of continuing aggressive intervention and the guidance, opportunities for memory making, and bereave-
likelihood of regaining acceptable neurologic and func- ment preparation. Taking care of required post-mortem
tional baseline is low, it is important to compassionately items such as discussion of autopsy and identification of
and directly address the status with surrogate decision- a funeral home in advance of the death can be helpful to
makers. Often, as in this case, parents and medical teams some families. Time should be spent with the medical team
are separately wondering if “we are doing too much” long in review of the plan for de-escalation and preparation for
before we directly address this together. For most parents, possible symptoms. When planning for ventilator weaning
using a model of shared decision-making allows them to with compassionate extubation or removal of ventilator, it
maintain their agency as “good parents” while offering is important to have a stepwise approach and acknowledge
assurance and lessening of the burden of difficult decisions. that while death is expected relatively soon after cessation
In this model, the medical provider identifies alignment of of ventilation, this is not under our control and we will con-
medical and parental goals, offering recommendations for tinue to follow the child’s lead, responding to symptoms of
next steps rather than asking a parent what they would like distress or discomfort. Consideration for timing of cessation
to do. Statements such as “Given his current status and of fluids or feeding should be taken to lessen the burden of
what you have told me is most important for him, this is secretions and gastrointestinal distress as end of life
what we should do next” can be utilized to move forward approaches. Allowing the family time after death to say
to comfort focus without laying the full burden of this deci- their goodbyes and prepare for separation is important.
sion on the parent. When offering this form of communica- For some families, having a familiar team member such
tion of recommendation, it is crucial to allow the parent as the bedside nurse stay with, and even hold the child,
time to consider what you have proposed and ask questions when the family leaves can lessen their sense of abandon-
or clarify to be sure you have accurately interpreted their ment. Asking parental preferences and continuing compas-
perspective. When moving forward with plans to de- sionate care at each step along the way can mean the
escalate medical interventions, the phrase “withdrawal of difference in supporting a “good” death.

Further Reading
de Vos, M.A., Bos, A.P., Plotz, F.B., et al. (2015). Talking with requesting advice about value-laden decisions. Pediatrics
parents about end-of-life decisions for their children. 136(4): 573–583.
Pediatrics 135(2): e465–e476. Field, M. and Behrman, R. (ed.) (2003). When Children Die:
Edmonds, B.T., Torke, A.M., Helft, P., and Wocial, L.D. (2015). Improving Palliative and End-of-Life Care for Children and
Doctor, what would you do? An ANSWER for patients their Families. Washington, DC: National Academies Press.
Downloaded from https://onlinelibrary.wiley.com/doi/ by National Institute Of Standard, Wiley Online Library on [06/03/2024]. See the Terms and Conditions (https://onlinelibrary.wiley.com/terms-and-conditions) on Wiley Online Library for rules of use; OA articles are governed by the applicable Creative Commons License
150 Cases in Pediatric Acute Care

Michelson, K.N., Patel, R., Haber-Baker, N., Emanuel, L., and critical decisions in the PICU. Pediatric Critical Care
Frader, J. (2013). End-of-life decisions in the pediatric Medicine 15(4): 291–298.
intensive care unit: roles professionals play. Pediatric University of Michigan Pediatric Palliative Care Team (2013).
Critical Care Medicine 14(1): e34–e44. Algorithms for end-of-life care in anticipated pediatric
October, T.W., Fisher, K.R., and Feudtner, C. (2014). The deaths. https://open.umich.edu/sites/default/files/
parent perspective: “Being a good parent” when making downloads/code_cards_december_2013.pdf
Downloaded from https://onlinelibrary.wiley.com/doi/ by National Institute Of Standard, Wiley Online Library on [06/03/2024]. See the Terms and Conditions (https://onlinelibrary.wiley.com/terms-and-conditions) on Wiley Online Library for rules of use; OA articles are governed by the applicable Creative Commons License
151

36

Pre-adolescent with Knee Pain


Katelyn Hamann and Shannon Konieczki
Children’s Hospital of Michigan, Detroit, MI, USA

A 12-year-old African American female presents with new- Current Status


onset left knee pain.
Her initial physical examination was within normal limits.
Her initial diagnostic evaluation included a radiograph of
the affected limb (Figures 36.1 and 36.2). A complete
History of Present Illness blood count and complete metabolic panel (Table 36.1)
were also completed and the results are shown in
She began complaining of left knee pain approximately Tables 36.1 and 36.2.
1 month ago without any prior injury, trauma or specific The radiographs show a sclerotic lesion with periosteal
event preceding the onset. Her pain is localized to the left reaction in the left proximal tibia.
distal knee, described as constant aching, and causes a limp
with ambulation. Her pain was minimally relieved with
ibuprofen. She denies fever, numbness, paresthesias, pallor,
weight loss, or malaise.
Questions

Answer the following questions using the details provided.


1 What are the differential diagnoses for this patient?
Past Medical History 2 What diagnostic evaluation would you order for this
patient?
She was born full term with no complications and dis- 3 What is your next step in treating this patient?
charged home with her mother. Her immunizations are
up to date for age including annual influenza vaccines.
She has never been hospitalized. She has no surgical history
and no known medication allergies. Rationale and Evidence-based Practice
Explanation

What are the differential diagnoses for this


Family History patient?

Her maternal grandfather died of lung cancer; no other sig- Differential diagnoses for this patient based on the initial
nificant family history. presentation would include septic arthritis, osteomyelitis,
osteosarcoma, Ewing sarcoma, fracture, sprain/strain,
Osgood–Schlatter disease, and osteochondritis dissecans.
Based on the imaging provided, the differential should also
Social History include myositis ossificans, bone fibrosarcoma, and giant
cell bone tumor. Clues in the patient history can be used
She lives at home with her mother, aunt, and grandmother. to determine the most likely differential diagnosis. Absence
Attends 7th grade. She is not involved in any sports. of a history of trauma makes fracture, sprain and strain less
Cases in Pediatric Acute Care: Strengthening Clinical Decision Making, First Edition. Edited by Andrea M. Kline-Tilford and Catherine M. Haut.
© 2020 John Wiley & Sons Ltd. Published 2020 by John Wiley & Sons Ltd.
Downloaded from https://onlinelibrary.wiley.com/doi/ by National Institute Of Standard, Wiley Online Library on [06/03/2024]. See the Terms and Conditions (https://onlinelibrary.wiley.com/terms-and-conditions) on Wiley Online Library for rules of use; OA articles are governed by the applicable Creative Commons License
152 Cases in Pediatric Acute Care

Table 36.1 Basic metabolic profile.

Sodium 139 mEq/L


Potassium 4.3 mEq/L
Chloride 102 mEql/L
Carbon dioxide 27 mEq/L
Blood urea nitrogen 12 mg/dL
Creatinine 0.5 mg/dL
Glucose 101 mg/dL
Phosphorus 4.1 mg/dL
Anion gap 10 mEq/L
Alanine aminotransferase 23 U/L
Aspartate aminotransferase 19 U/L
Alkaline phosphatase 333 U/L
Lactate dehydrogenase 377 U/L
Uric acid 2.4 mg/dL

Table 36.2 Complete blood count.

White blood cell count 11 700/mm3


Hemoglobin 11.1 g/dL
Hematocrit 33.8%
Figure 36.1 Lateral radiograph of knee. Platelet count 365 000/mm3
Mean corpuscular volume (MCV) 69.2 fL
Neutrophils 73%
Eosinophils 1%
Monocytes 4%
Lymphocytes 3%
Bands 0%

sometimes fever which are not present in this child.


Absence of a prominent/edematous tibial tubercle makes
Osgood–Schlatter disease less likely. With these considera-
tions and the radiographic findings, malignancy as the eti-
ology of the pain moves to the top of the differential
diagnosis list.

What diagnostic evaluation would you order for


this patient?
A plain radiograph is the first step in diagnosis and was
Figure 36.2 Anteroposterior radiograph of knee. completed on the child’s presentation. Radiographic dem-
onstration of sclerotic lesions indicates a thickening of
likely. Absence of fever moves osteomyelitis further down the bone that is typically associated with a slow-growing
the differential diagnosis list. Septic arthritis is typically process rather than acute changes. Sclerotic bone lesions
associated with extreme discomfort and is often accompa- can be benign when asymptomatic, and are often found
nied by an erythematous and edematous joint and as an incidental finding. Malignant bone disease is often
Downloaded from https://onlinelibrary.wiley.com/doi/ by National Institute Of Standard, Wiley Online Library on [06/03/2024]. See the Terms and Conditions (https://onlinelibrary.wiley.com/terms-and-conditions) on Wiley Online Library for rules of use; OA articles are governed by the applicable Creative Commons License
Chapter 36 Pre-adolescent with Knee Pain 153

associated with pain that increases over time. Osteosar- of long bones, with the most common locations being
coma lesions can appear as lytic and/or sclerotic with an the distal femur, proximal tibia, and humerus, consecu-
internal or external osteoid matrix, which is associated with tively. Of children diagnosed with osteosarcoma, 20%
soft tissue involvement. Periosteal reaction often occurs have metastatic disease, most commonly to the lung.
with the classic “sunburst” pattern or with Codman’s trian- Age of occurrence is typically associated with puberty,
gle. Codman’s triangle presents due to a break in the cortex with highest incidence for males 15–19 years of age
of the bone, causing a triangle of reactive bone to appear. and females 10–14 years of age. Certain genetic syn-
Skip lesions can also occur in patients with osteosarcoma; dromes are associated with osteosarcoma, including
these are osseous metastases that are non-contiguous. Fur- bilateral retinoblastoma, Li–Fraumeni syndrome, Bloom
ther imaging is required, based on the radiograph results. It syndrome, Paget disease and Thomson syndrome. Chil-
would be best to obtain magnetic resonance imaging (MRI) dren with hereditary bilateral retinoblastoma are 500
of the lower extremity. MRI and computed tomography times more likely to develop osteosarcoma than a child
(CT) are tools utilized to assess the extent of the tumor into without retinoblastoma. Children typically present with
vessels, nerves and soft tissues, as well as identify the extent pain as their chief complaint, which may or may not be
of metastases. associated with a trauma in the recent months. Although
MRI results include subperiosteal and soft tissue mass osteosarcoma is not caused by trauma, it is often discov-
located in the lateral tibial metaphysis with extension into ered following a trauma. Swelling does not immediately
the lateral tibial epiphysis, posterior leg musculature and follow the trauma, but often occurs over time. Pathologic
neurovascular bundle (Figure 36.3). fractures are often found in conjunction with the solid
According to the National Institutes of Health’s Sur- tumor. Associated laboratory findings include elevated
veillance, Epidemiology and End Results Program for alkaline phosphatase, elevated lactate dehydrogenase,
the National Cancer Institute (2019), there will be an mild anemia, or elevated erythrocyte sedimentation rate
estimated 3500 new cases of osteosarcoma diagnosed (ESR). This child’s laboratory findings include elevations
in the United States in 2019. Although rare, osteosar- in alkaline phosphatase and lactate dehydrogenase, and
coma is the most common bone tumor in childhood mild anemia which support the diagnosis of osteosar-
and adolescence. It typically arises in the metaphysis coma. An ESR has not yet been obtained.

What is your next step in treating this patient?


CT of chest, abdomen and pelvis is needed to evaluate for
metastasis to other organs. Although CT exposes the
patient to higher doses of radiation, it is imperative to
obtain these scans so that the disease can be staged appro-
priately based on the presence of metastasis. CT showed left
lower lobe heterogeneous mass with ground glass opacities
scattered to bilateral lung fields, concerning for osteosar-
coma metastases. Tracheobronchial tree was patent. There
was no pleural effusion or pneumothorax (Figure 36.4).
Biopsy of the lesion by an orthopedic surgeon should be
performed to confirm the definitive diagnosis. Osteosar-
coma is histologically classified by the World Health Organ-
ization and includes parosteal (juxtacortical), low-grade
central (intramedullary), periosteal (juxtacortical chondro-
blastic), telangiectatic, small cell, high-grade surface, and
conventional (chondroblastic, fibroblastic or osteoblastic).
Parosteal and low-grade central are considered low-grade
subtypes. Periosteal is considered intermediate subtype.
Telangiectatic, small cell, high-grade surface, and conven-
tional are considered high grade. Conventional osteosar-
coma is the most common subtype.
Osteosarcoma treatment includes surgical resection of
Figure 36.3 Initial MRI. the primary tumor. This may include an amputation or
Downloaded from https://onlinelibrary.wiley.com/doi/ by National Institute Of Standard, Wiley Online Library on [06/03/2024]. See the Terms and Conditions (https://onlinelibrary.wiley.com/terms-and-conditions) on Wiley Online Library for rules of use; OA articles are governed by the applicable Creative Commons License
154 Cases in Pediatric Acute Care

Figure 36.4 CT of chest and abdomen.

a limb salvage, which would be determined by the Case Resolution


orthopedic surgeon. Neoadjuvant chemotherapy is the
standard of care, typically consisting of methotrexate, After biopsy confirming the diagnosis of osteosarcoma, she
doxorubicin, and cisplatin. Radiation is not utilized underwent ampulation of the left lower extremity, followed
as treatment because osteosarcoma is resistant to by chemotherapy with methotrexate, doxorubicin, and
radiation. cisplatin.

Reference
National Institutes of Health (2019). Cancer Stat Facts: Bone
and Joint Cancer. https://seer.cancer.gov/statfacts/html/
bones.html

Further reading
Dyson, K.A., Stover, B.D., Grippin, A., et al. (2019). Emerging children and young adults. Current Problems in Cancer 37:
trends in immunotherapy for pediatric sarcomas. Journal of 181–191. doi: 10.1016/j.currproblcancer.2013.07.005.
Hematology Oncology 12: 1–10. doi: 10.1186/s13045-019- Whelan, J.S. and Davis, L.E. (2018). Osteosarcoma,
0756-z. chondrosarcoma, and chordoma. Journal of Clinical
Rana, K.A., Meyer, J., Ibrahim, S., Ralls, M., and Kent, P.M. Oncology 36(2): 188–193. doi: 10.1200/JCO.2017.75.1743.
(2013). The role of imaging of malignant bone tumors in
Downloaded from https://onlinelibrary.wiley.com/doi/ by National Institute Of Standard, Wiley Online Library on [06/03/2024]. See the Terms and Conditions (https://onlinelibrary.wiley.com/terms-and-conditions) on Wiley Online Library for rules of use; OA articles are governed by the applicable Creative Commons License
155

37

Infant with Increased Fussiness and Decreased Oral Intake


Stephanie Haughey
Helen DeVos Children’s Hospital, Grand Rapids, MI, USA

An 11-month-old boy with no significant prior medical his- centrally but cold peripherally with capillary refill of 5–6
tory presents to the emergency department (ED) with fever, seconds, and has dry mucous membranes. He
irritability, fatigue and vomiting. is tachypneic with abdominal muscle use but lungs
are clear and no retractions are present. Tachycardia
is present but normal S1 and S2 and no murmur. Abdomen
History of Present Illness is soft and non-distended, with no hepatomegaly. Moans
with IV poke but otherwise non-interactive. Petechiae
This baby is a previously healthy 11-month-old boy present-
noted across chest and abdomen. The infant weighs 10 kg.
ing to the ED with a 4-hour history of irritability, fatigue,
Initial vital signs are as follows: temperature 40.1 C
poor oral intake, vomiting, decreased urine output, and
(104.2 F) rectally, heart rate 189 bpm, blood pressure
high fever. He was deteriorating quickly in the car to
72/34 mmHg, respiratory rate 45 breaths per minute, and
the ED.
SpO2 88% on room air. Results for complete blood count
(CBC) and complete metabolic panel are shown in
Past Medical History Tables 37.1 and 37.2. Results for inflammatory markers
as follows: C-reactive protein 121.6 mmol/L, procalcitonin
He has had two episodes of otitis media, the last one a >100.0 ng/mL.
month ago. He had a recent upper respiratory infection.
He is up to date on vaccinations. There are no developmen-
Questions
tal concerns. He eats formula and baby foods.
Answer the following questions using the details provided.
Past Surgical History 1 What are the differential diagnoses for this child?
2 What is the most likely diagnosis for this child?
Circumcision as newborn without complication. 3 What are the immediate interventions in the emer-
gency department?
Family History 4 Are there any other studies you would obtain?
5 What are the next steps to help stabilize this patient?
Healthy 7-year-old sister, father with type 2 diabetes,
mother with kidney stones.
Rationale and Evidence-based Practice
Current Status Explanation

What are the differential diagnoses for this child?


Infant is lethargic, difficult to arouse, and with pallor and
mottling. His pupils are equal and reactive, and the anterior Differential diagnoses for this child should include causes of
fontanelle is full. He is positive for opisthotonos, feels hot septic shock, meningismus, and petechiae. This would

Cases in Pediatric Acute Care: Strengthening Clinical Decision Making, First Edition. Edited by Andrea M. Kline-Tilford and Catherine M. Haut.
© 2020 John Wiley & Sons Ltd. Published 2020 by John Wiley & Sons Ltd.
Downloaded from https://onlinelibrary.wiley.com/doi/ by National Institute Of Standard, Wiley Online Library on [06/03/2024]. See the Terms and Conditions (https://onlinelibrary.wiley.com/terms-and-conditions) on Wiley Online Library for rules of use; OA articles are governed by the applicable Creative Commons License
156 Cases in Pediatric Acute Care

Table 37.1 Complete blood count. meningococcemia with meningitis and septic shock.
Meningococcemia is defined as bloodstream infection with
White blood cell count 23 080/mm3
the highly virulent Gram-negative bacterium Neisseria
Hemoglobin 7.6 g/dL meningitidis. It usually causes many of the signs and symp-
Hematocrit 22.9% toms seen with other types of bacteremia and bacterial
Platelet count 19 000/mm3 meningitis such as fever, nausea, vomiting, headache, neck
Neutrophils 51% stiffness, cold hands and feet, pallor and/or mottling,
Lymphocytes 2% decreased concentration, and myalgias. Its differentiating
Monocytes 1%
features include the severity of myalgias (particularly leg
pain), the sudden onset and rapid progression, a non-
Eosinophils 0%
specific rash rapidly progressing to a hemorrhagic rash,
Bands 37%
and toxin-induced profound capillary leak.
Meningococcemia caused by Neisseria meningitidis can
Table 37.2 Complete metabolic panel. cause fulminant disease or just remain harbored in the
nasal cavity. The difference in illness is related to the fact
Sodium 139 mEq/L that meningococcemia is widespread vascular injury char-
Potassium 4.1 mEq/L acterized by endothelial necrosis, intraluminal thrombosis,
Chloride 100 mEq/L and perivascular hemorrhage. Endotoxin, cytokines, and
Carbon dioxide 12 mEq/L
free radicals damage the vascular endothelium, producing
platelet deposition and vasculitis. Cytokine response will
Blood urea nitrogen 30 mg/dL
result in severe hypotension, reduced cardiac output, and
Creatinine 1.8 mg/dL
increased endothelial response. Intravascular infection
Glucose 56 mg/dL and intracranial bacterial growth and inflammation reflect
Calcium 8.5 mg/dL the pathology of the disease. Pathogens bind to the endo-
Albumin 3.5 mg/dL thelial cells, and microcolonies are formed on the apical
Total protein 6.0 g/dL portion of the endothelial base. These bacteria invade the
Aspartate aminotransferase 85 U/L subarachnoid space with resultant meningitis with multi-
Alanine aminotransferase 56 U/L ple organ failure, shock, and the possibility of death.
Patients with meningococcemia develop thrombosis and
Alkaline phosphatase 145 U/L
hemorrhage in the skin, which is often apparent on admis-
Total bilirubin 1.1 mg/dL
sion to the ED. Thrombosis of the glomerular capillaries
Magnesium 1.6 mEq/L may cause renal cortical necrosis. Meningococcemia is a med-
Phosphorus 4.0 mg/dL ical emergency requiring prompt recognition, therapy, and
hospitalization. Illness can range from transient fever and
bacteremia to fulminant disease with multisystem organ fail-
include Gram-negative bacteria such as Neisseria meningiti-
ure and death occurring within hours of the onset of illness.
dis, Haemophilus influenzae, Rickettsia species (which cause
Rocky Mountain spotted fever or typhus), and Ehrlichia
(which causes ehrlichiosis), and Gram-positive bacteria What are the immediate interventions in the
such as Staphylococcus aureus, Streptococcus pneumoniae, emergency department?
and group A Streptococcus. Other causes for petechiae
Address airway and breathing by placing the child on high-
and/or purpura, such as Henoch–Schönlein purpura,
flow nasal cannula with low threshold for initiating non-
drug-induced vasculitis, scurvy, immune thrombocytope-
invasive positive pressure support or intubation and
nia, and acute leukemia, should be considered, but are less
mechanical ventilation. Circulation is supported by giving
likely given the presentation of shock with fever. Hemolytic
fluid boluses 20 mL/kg.
uremic syndrome does not explain all the symptoms, but
Additional interventions include correction of hypogly-
would explain the anemia, thrombocytopenia, and kidney
cemia with dextrose bolus and management of other abnor-
injury and can rarely present with purpura.
mal laboratory results. A blood culture followed by
immediate administration of antibiotics is necessary to
What is the most likely diagnosis for this child?
treat potential pathogens. The antibiotic of choice for sus-
Given the sudden onset, rapid progression, and severity of pected meningococcemia is a third-generation cephalospo-
illness, the most likely diagnosis for this child is rin such as ceftriaxone or cefotaxime. Vancomycin should
Downloaded from https://onlinelibrary.wiley.com/doi/ by National Institute Of Standard, Wiley Online Library on [06/03/2024]. See the Terms and Conditions (https://onlinelibrary.wiley.com/terms-and-conditions) on Wiley Online Library for rules of use; OA articles are governed by the applicable Creative Commons License
Chapter 37 Infant with Increased Fussiness and Decreased Oral Intake 157

also be administered to target cephalosporin-resistant 16 What other potential complications should you
Gram-positive organisms. Antibiotics should be dosed for anticipate?
meningitis and adjusted for renal dysfunction as indicated. 17 Does anything need to be done for close contacts of
Antibiotic administration should not be delayed and this patient?
intraosseous or intramusclar routes should be used if intra-
venous access cannot be obtained swiftly. Rectal acetamin-
ophen can be administered to help reduce the fever and How would you interpret the chest radiograph and
how might this change your ventilation
therefore metabolic demand.
strategies?
Are there any other studies you would obtain? The chest radiograph findings of diffuse bilateral infiltrates
together with his acute hypoxia are consistent with acute
Given the hemorrhagic rash, full fontanelle, and altered
respiratory distress syndrome (ARDS), a common manifes-
level of consciousness, a head computed tomography
tation of sepsis that is caused by alveolar capillary leak.
(CT) scan should be done to assess for intracranial bleeding
Because the risk of further lung injury is high, a strategy
and cerebral edema.
involving a higher ventilation rate and lower tidal volume
with tolerance of relative hypoxia and hypercapnia will
What are the next steps to help stabilize this
likely be warranted.
patient?
This patient is rapidly deteriorating and needs a central
venous line to administer vasopressors. The safest and most Would a lumbar puncture be indicated and
appropriate approach is to intubate the patient and place
appropriate?
him on a mechanical ventilator. Not only will this allow Although a lumbar puncture would be indicated given a
you to sedate him for the central line placement, but it will high suspicion for meningitis, this child is too unstable to
reduce the metabolic demand due to breathing. tolerate both the positioning and sedation for a lumbar
Continuation of case: With admission to the pediatric puncture. His petechiae are also a clue that he could be
intensive care unit (PICU) the infant now has subcostal at risk for spinal epidural hematoma and a positive blood
retractions, nasal flaring, and crackles on auscultation. culture can also help to guide treatment. If available,
Capillary refill remains prolonged. Vital signs are currently PCR assays can detect meningococcal DNA in cerebrospi-
temperature 39.1 C (102.4 F) rectally, heart rate 175 bpm, nal fluid, plasma, and serum and results are less affected
blood pressure 68/31 mmHg, respiratory rate 36 breaths per by antibiotics. The main priorities for this child are to start
minute, and SpO2 88% on 15 L/min high-flow nasal can- antibiotics, administer fluids, and transfer him to the PICU.
nula with 100% oxygen. The patient has received ceftriax-
one and three boluses of normal saline 20 mL/kg; What vasopressor would you choose and why?
dextrose 5% in 0.9% normal saline (40 mL/hour) and van- This child is in cold shock (cold extremities, delayed capil-
comycin are currently infusing. lary refill) so the most appropriate medication to start with
6 How would you interpret the chest radiograph and would likely be epinephrine, which provides inotropy,
how might this change your ventilation strategies? chronotropy and, at larger doses, vasoconstriction. Norep-
7 Would a lumbar puncture be indicated and inephrine would be more appropriate for a child with warm
appropriate? shock (warm extremities, brisk capillary refill).
8 What vasopressor would you choose and why?
9 How would you define his shock at this point? How would you define his shock at this point?
10 What other studies could be obtained that might help
He has fluid-refractory and catecholamine-resistant shock.
manage the patient’s shock?
11 How do you interpret this blood gas analysis and what
What other studies could be obtained that might
ventilator changes would you make?
help manage the patient’s shock?
12 What else can you do to improve his blood pressure?
13 What medication can you add to improve perfusion A repeat blood gas analysis with lactate and electrolytes
once you have control of his blood pressure? including ionized calcium would help determine ventila-
14 What explains his coagulopathy and how will you tor changes, further fluid resuscitation, and whether
treat it? electrolyte replacements such as calcium, potassium or
15 What is the prognosis for this child? magnesium are needed to support cardiac function.
Downloaded from https://onlinelibrary.wiley.com/doi/ by National Institute Of Standard, Wiley Online Library on [06/03/2024]. See the Terms and Conditions (https://onlinelibrary.wiley.com/terms-and-conditions) on Wiley Online Library for rules of use; OA articles are governed by the applicable Creative Commons License
158 Cases in Pediatric Acute Care

Myocardial suppression should be suspected with Table 37.3 Arterial blood gas analysis 1 hour after intubation.
meningococcemia, and could be evaluated with an
pH 7.19
echocardiogram. A random cortisol level would screen
for adrenal insufficiency as is common with meningo- PaCO2 59 mmHg
coccemia due to adrenal infarction. A repeat CBC (for PaO2 68 mmHg
platelet and hemoglobin levels) and set of coagulation Bicarbonate 20 mEq/L
parameters including fibrinogen, prothrombin time Base excess –6.3
(PT), and partial thromboplastin time (PTT) would help Lactate 4.2 mmol/L
guide blood product replacement in the setting of his
purpura fulminans with active bleeding.
Continuation of case: Petechial rash on his abdomen and Table 37.4 Additional laboratory studies.
chest has evolved into purpuric lesions extending down his
legs. Blood-tinged gastric drainage is present in nasogastric Cortisol 1.2 μg/dL
tube. Vital signs include temperature 38.6 C (101.5 F) rec- Fibrinogen 175 mg/dL
tally, heart rate 170 bpm, blood pressure 71/35 mmHg, res- Prothrombin time (PT) 21.4 s
piratory rate 24 breaths per minute, and SpO2 90% on 80% INR 2.3
FiO2 via the ventilator. The child has been intubated for Partial thromboplastin time (PTT) 39 s
approximately 1 hour with central venous and arterial lines
placed.
His chest radiograph is shown in Figure 37.1. His
Results of the blood gas analysis, cortisol level, and coag-
ventilator settings on pressure-regulated volume control
ulation panel are shown in Tables 37.3 and 37.4.
with spontaneous intermittent mandatory ventilation
(PRVC-SIMV) include a tidal volume of 80 mL, rate of
24 breaths per minute, positive end-expiratory pressure
How do you interpret this blood gas analysis and
what ventilator changes would you make?
(PEEP) of 6 cmH2O, and pressure support of 10 cmH2O.
His peak inspiratory pressure is 33 cmH2O. He is receiving He has a mixed respiratory and metabolic acidosis. Now
an epinephrine infusion 0.14 μg/kg per min to support his that he is sedated and muscle relaxed, he is unable to com-
blood pressure. Norepinephrine is being sent from the pensate for his metabolic acidosis by increasing his own res-
pharmacy for additional augmentation of blood pressure. piratory rate. Administration of sodium bicarbonate is not
An arterial line has been placed in his right femoral artery indicated except in cases of severe acidosis. The metabolic
after failed attempts at both radial arteries, and a nurse acidosis will continue to improve with fluid resuscitation
technician is still maintaining pressure on his left wrist and lactate clearance. Based on his chest radiograph, he
because it has not stopped bleeding. Bruising is noted is developing ARDS, and his peak inspiratory pressures
where his blood pressure cuff has been. are high. A lower tidal volume balanced with a faster rate
for ventilation would be targeted. A pressure control (PC)
mode of ventilation will allow tighter control of his peak
pressures to limit barotrauma. He is also hypoxic which
may respond to increases in PEEP. In ARDS, PEEP can
improve oxygenation by maintaining lung volume during
exhalation. For example, you could change his ventilator
settings to PC-SIMV rate of 30 breaths per minute, peak
inspiratory pressure 30 cmH2O, PEEP 9 cmH2O, and pres-
sure support 10 cmH2O and repeat a blood gas in 30–60
minutes.

What else can you do to improve his blood


pressure?
Based on the cortisol level, hydrocortisone at stress dosing
is initiated. This therapy may be initiated prior to receiving
the laboratory confirmation due to the severity of illness.
Figure 37.1 Chest radiograph. Transfusion of PRBCs could improve his blood pressure
Downloaded from https://onlinelibrary.wiley.com/doi/ by National Institute Of Standard, Wiley Online Library on [06/03/2024]. See the Terms and Conditions (https://onlinelibrary.wiley.com/terms-and-conditions) on Wiley Online Library for rules of use; OA articles are governed by the applicable Creative Commons License
Chapter 37 Infant with Increased Fussiness and Decreased Oral Intake 159

as well as improve oxygen delivery to the tissues. Replete survive, 20% have permanent disabilities (Wick et al.,
calcium, magnesium, and potassium as indicated to sup- 2013). Therefore, despite its relative rarity compared to
port cardiac function. the past, it remains important to recognize meningococce-
mia and act promptly. The most important intervention is
What medication can you add to improve to reduce the risk of death by early administration of appro-
perfusion once you have control of his blood priate antibiotics. Antibiotic administration should not be
pressure? delayed and intraosseous or intramusclar routes should
be used if intravenous access cannot be obtained swiftly.
Once blood pressure is better controlled, milrinone can be
added to help improve cardiac function and improve
perfusion. What other potential complications should you
anticipate?
What explains his coagulopathy and how will you Potential short-term complications include purpura fulmi-
treat it? nans leading to deep necrosis of digits or limbs, renal fail-
Meningococcemia is known for causing disseminated intra- ure, ARDS, nosocomial pneumonia due to superinfection,
vascular coagulation (DIC) due to factor loss from capillary neurologic complications including coma, diabetes insipi-
leak, endothelial activation, and clotting factor consump- dus, stroke and seizures, and immunologic complications
tion. The purpuric rash this causes in meningococcemia such as arthritis, cutaneous vasculitis, iritis, and pericardi-
is known as purpura fulminans, which is a coagulopathy tis. Long-term complications include hearing loss, scarring,
similar to DIC. The primary treatment for DIC is treating and orthopedic sequelae. Amputations are not uncommon,
the underlying infection. Therapy aimed at treating the and orthopedics should be consulted early to monitor extre-
DIC specifically should be directed toward clinical stability mities (Wick et al., 2013).
rather than correcting laboratory values. In the context of
active bleeding, vitamin K and blood products such as pla- Does anything need to be done for close contacts
telets, fresh frozen plasma, cryoprecipitate, and PRBCs can of this patient?
be used.
Close contacts should be given post-exposure antibiotic
prophylaxis with a course of rifampin, ciprofloxacin, or cef-
What is the prognosis for this child?
triaxone, ideally within 24 hours of patient diagnosis. Refer
The current mortality rates for meningococcal meningitis to the Centers for Disease Control for the most recent
are approximately 10–15%. Of the 90% of patients who do recommendations.

Reference
Wick, J.M., Krajbich, I., Kelly, S., and Dewees, T. (2013).
Meningococcemia: the pediatric orthopedic sequelae. AORN
Journal 97(5): 559–578. doi: 10.1016/j.aorn.2013.03.005.

Further Reading
Killien, E.Y. and Watson, R.S. (2017). Early death from Reports 2018: bcr-2018-226914. doi: 10.1136/bcr-2018-
pediatric severe sepsis: still a problem and a mandate for 226914.
clinical research. Pediatric Critical Care Medicine 18(9): Olbrich, K.J., Muller, D., Schumacher, S., et al. (2018).
891–892. doi: 10.1097/PCC.0000000000001239. Systematic review of invasive meningococcal disease:
Medeiros, I., Melo, A.R., Baptista, V., and Ribeiro, A. (2018). sequelae and quality of life impact on patients and their
Meningococcemia: rare but life-threatening. BMJ Case caregivers. Infectious Disease Therapeutics 7: 421–438. doi:
10.1007/s40121-018-0213-2.
Downloaded from https://onlinelibrary.wiley.com/doi/ by National Institute Of Standard, Wiley Online Library on [06/03/2024]. See the Terms and Conditions (https://onlinelibrary.wiley.com/terms-and-conditions) on Wiley Online Library for rules of use; OA articles are governed by the applicable Creative Commons License
161

38

Infant with Prolonged Jaundice


Aimee Bucci1,2
1
Phoenix Children’s Hospital, Phoenix, AZ, USA
2
Arizona State University, Edson College of Nursing and Health Innovation, Phoenix, AZ, USA

A 22-day-old female is transferred from the acute care Current Status


pediatric unit to the pediatric intensive care unit (PICU)
with chief complaint of fatigue, dehydration, and jaundice. The infant appears dehydrated and lethargic. She does not
appear to have dysmorphic features. She has jaundice with
scleral icterus present. Her abdomen is mildly distended
History of Present Illness and she has evidence of hepatomegaly with the liver border
noted to be 2 cm below the right costal margin. Urine is
This infant has been overall healthy, but parents describe dark amber in color. Her heart rate is 165 bpm, blood pres-
her as yellow appearing and state that her eyes have sure is 72/35 mmHg, respiratory rate is 45 breaths per
become more yellow over the last week. Parents also report minute, and oxygen saturations 95% on room air.
that her stools have become very pale and her urine is a A complete metabolic panel was obtained with results
dark color. She has had decreased appetite over the last sev- shown in Table 38.1.
eral days and has appeared more tired. They deny any
fevers.
Questions

Past Medical History Answer the following questions using the details provided.
1 Based on the history and examination findings, what
This baby was born full term to a gravida 1, para 1 mother
differential diagnoses should be considered for this
who is breastfeeding. Mother’s pregnancy was uncompli-
patient?
cated except for a maternal fever at delivery so the infant
2 What additional laboratory evaluation is indicated for
was treated with antibiotics for 48 hours and discharged at
this infant?
3 days of life with negative cultures. She received vitamin
3 What diagnostic studies should be considered?
K and the first dose of hepatitis B vaccine while in the hos-
4 What other consultants should be considered in this
pital. Newborn metabolic screens revealed no abnormalities.
infant’s care?

Past Surgical History


Rationale and Evidence-based Practice
None. Explanation
Based on the history and examination findings,
what differential diagnoses should be considered
Family History for this patient?
No reports of liver disease, hemolytic conditions, vascular Differential diagnoses for this infant should include
conditions, or cardiac disease in the family. diseases that cause physiologic infant cholestasis

Cases in Pediatric Acute Care: Strengthening Clinical Decision Making, First Edition. Edited by Andrea M. Kline-Tilford and Catherine M. Haut.
© 2020 John Wiley & Sons Ltd. Published 2020 by John Wiley & Sons Ltd.
Downloaded from https://onlinelibrary.wiley.com/doi/ by National Institute Of Standard, Wiley Online Library on [06/03/2024]. See the Terms and Conditions (https://onlinelibrary.wiley.com/terms-and-conditions) on Wiley Online Library for rules of use; OA articles are governed by the applicable Creative Commons License
162 Cases in Pediatric Acute Care

Table 38.1 Complete metabolic profile. past 2–3 weeks of age have conjugated/direct bilirubin
levels measured. Basic additional studies to assess
Sodium 135 mEq/L
the synthetic function of the liver should include
Potassium 3.9 mEq/L alanine aminotransferase (ALT), aspartate aminotrans-
Chloride 101 mEq/L ferase (AST), alkaline phosphate (AP), gamma-
Carbon dioxide 18 mEq/L glutamyltranspeptidase (GGT), prothrombin time (PT),
Blood urea nitrogen 25 mg/dL international normalized ratio (INR), albumin, glucose,
Creatinine 0.7 mg/dL ammonia, complete blood count, urinalysis, and bacte-
Glucose 78 mg/dL
rial cultures of blood and urine if clinically indicated
(Fawaz et al., 2017, pp. 160–161). More advanced testing
Calcium 9.1 mg/dL
may be initiated by a hepatologist to evaluate for other
Albumin 3.0 mg/dL
causes of cholestatic jaundice.
Total protein 5.4 g/dL
Aspartate aminotransferase 147 U/L
Alanine aminotransferase 186 U/L What diagnostic studies should be considered?
Alkaline phosphatase 84 U/L
An abdominal radiograph can be obtained to assess the
Total bilirubin 18.6 mg/dL
abdominal distension. Fasting abdominal ultrasound is a
Direct bilirubin 4.6 mg/dL
non-invasive and easy study to obtain and can help to
identify visible obstructions of the biliary tree, the pres-
including biliary causes such as obstruction of large extra- ence of a choledochal cyst, and evidence of advanced liver
hepatic or small intrahepatic bile ducts or hepatocellular disease (Fawaz et al., 2017, p. 161). Hepatobiliary scintig-
etiology including defects in membrane transport, embryo- raphy can be used to confirm biliary tract patency but is
genesis, or metabolic dysfunction (Fawaz et al., 2017, not commonly used because of limitations in differentiat-
p. 155). Common conditions can include biliary atresia, ing idiopathic hepatitis from biliary atresia (Fawaz et al.,
common duct gallstones, choledochal cysts, inborn errors 2017, p. 161). Endoscopic retrograde cholangiopancreato-
of metabolism, Alagille syndrome, and various infections graphy (ERCP) can be used to detect the patency of the
(Fawaz et al., 2017, pp. 155, 163–165). extrahepatic biliary tree but can be technically challenging
The most likely diagnosis for this infant given the age, in small infants and requires specialized equipment that
persistent jaundice, and hyperbilirubinemia is biliary atre- some centers may not have available (Fawaz et al., 2017,
sia. Biliary atresia is a rare condition that affects the func- pp. 161–162; Verkade et al., 2016, p. 632). Liver biopsy
tion or the anatomy along the canalicular-bile tract remains the most widely used diagnostic tool, yielding
(Verkade et al., 2016, p. 631). Biliary atresia is the most com- a 90–95% accuracy for biliary atresia (Fawaz et al.,
mon cause of persistent cholestatic jaundice, with an inci- 2017, p. 162).
dence of 1 in 10 000–19 000 (Verkade et al., 2016, p. 631).
The common laboratory finding in infants with biliary atre-
sia is conjugated or direct hyperbilirubinemia. Infants with What other consultants should be considered in
benign or physiologic jaundice will present with an uncon- this infant’s care?
jugated or indirect hyperbilirubinemia (Verkade et al., Pediatric gastroenterologists or hepatologists should be
2016, p. 631). Examination findings of poor oral intake, consulted for any infant with direct hyperbilirubinemia
fatigue, abdominal distension, persistent jaundice, hepato- and presence of liver dysfunction. Pediatric surgeons
megaly, acholic stools, and dark urine should alert practi- should be consulted for the potential for a Kasai portoenter-
tioners to evaluation for liver disease. ostomy procedure to provide a temporary fix for biliary
atresia. A pediatric neurologist can be considered if the
What additional laboratory evaluation is indicated alterations in mental status are clinically significant.
for this infant?
A pediatric hematologist could be considered if the hyper-
The American Academy of Pediatrics (2004, p. 300) bilirubinemia is caused from unconjugated or indirect
recommends that all infants with jaundice persisting causes.
Downloaded from https://onlinelibrary.wiley.com/doi/ by National Institute Of Standard, Wiley Online Library on [06/03/2024]. See the Terms and Conditions (https://onlinelibrary.wiley.com/terms-and-conditions) on Wiley Online Library for rules of use; OA articles are governed by the applicable Creative Commons License
Chapter 38 Infant with Prolonged Jaundice 163

References
American Academy of Pediatrics Subcommittee on the European Society for Pediatric Gastroenterology,
Hyperbilirubinemia (2004). Management of Hepatology and Nutrition. Journal of Pediatric
hyperbilirubinemia in the newborn infant 35 or more weeks Gastroenterology and Nutrition 64(1): 154–168. doi: 10.1097/
of gestation. Pediatrics 14: 297–316. MPG.0000000000001334.
Fawaz, R., Baumann, U., Ekong, U., et al. (2017). Guideline for Verkade, H.J., Bezerra, J.A., Davenport, M., et al. (2016).
the evaluation of cholestatic jaundice in infants: joint Biliary atresia and other cholestatic childhood diseases:
recommendations of the North American Society for advances and future challenges. Journal of Hepatology 65:
Pediatric Gastroenterology, Hepatology, and Nutrition and 631–642.
Downloaded from https://onlinelibrary.wiley.com/doi/ by National Institute Of Standard, Wiley Online Library on [06/03/2024]. See the Terms and Conditions (https://onlinelibrary.wiley.com/terms-and-conditions) on Wiley Online Library for rules of use; OA articles are governed by the applicable Creative Commons License
165

39

Child with Rash and Drooling


Mark Weber
Children’s Hospital of Philadelphia, Philadelphia, PA, USA

A 32-month-old male with no significant past medical Past Surgical History


history presents to the emergency department (ED) with
a 2-day history of a rash and respiratory concerns. No prior surgeries.

History of Present Illness


Family History
Two days prior to presentation, the child developed lip
swelling after eating a watermelon. He was given acetamin- His mother has a family history of childhood asthma. There
ophen, Benadryl and ibuprofen. His lips became more blis- is no other significant family history.
tered and he developed a fever of 38.5 C (101.3 F). He
developed a red macular rash on lips, hands including
palms, feet and trunk. He was seen at a local ED and
was diagnosed with hand, foot and mouth disease. He Current Status
was discharged to home following this visit.
Two days later his rash and blistering progressed and his His physical examination reveals a preference to keep his
fever persisted. He had poor oral intake secondary to pain eyes closed. His conjunctiva are injected bilaterally. His
and he complained of pain with urination. His mother lips and oral mucosa show significant erythema and des-
brought him back to the ED for further evaluation. While quamation. He is drooling with a hoarse voice and inter-
in the ED, he did not have frank hematuria, melena/ mittent coughing. The rest of his body has several red/
hematochezia, hemoptysis or hematemesis. There was no purple macules and plaques as well as vesicles across
respiratory distress, although his mother did note hoarse- the face, trunk and extremities, including palms and soles.
ness and increasing drooling on the day of admission. He He has several bullae on his trunk and extremities
did not have recent upper respiratory infection (URI) (Figure 39.1) covering about 35% of his body surface
symptoms, though he has been around other children with area (BSA).
URI symptoms. He has not taken any medications other On arrival in the pediatric intensive care unit (PICU), he
than acetaminophen, Benadryl and ibuprofen recently. continues with a hoarse voice and drooling. His initial vital
He does not take any medications on a daily basis. signs are temperature 40.5 C (104.9 F), heart rate 144 bpm,
blood pressure 115/56 mmHg, and oxygen saturation 98%
on room air. His respiratory examination shows a respira-
Past Medical History tory rate of 44 breaths per minute with clear breath sounds
bilaterally and no increased work of breathing. His weight
His history was only significant for a wheeze in the past but is 14.5 kg. He has received a normal saline fluid bolus and
has not required the use of his albuterol inhaler for several one dose of oxacillin and clindamycin and initial laboratory
months. evaluation is obtained (Table 39.1).

Cases in Pediatric Acute Care: Strengthening Clinical Decision Making, First Edition. Edited by Andrea M. Kline-Tilford and Catherine M. Haut.
© 2020 John Wiley & Sons Ltd. Published 2020 by John Wiley & Sons Ltd.
Downloaded from https://onlinelibrary.wiley.com/doi/ by National Institute Of Standard, Wiley Online Library on [06/03/2024]. See the Terms and Conditions (https://onlinelibrary.wiley.com/terms-and-conditions) on Wiley Online Library for rules of use; OA articles are governed by the applicable Creative Commons License
166 Cases in Pediatric Acute Care

Rationale and Evidence-based Practice


Explanation

What considerations are needed for his disposition


considering the dermatologic findings involving
approximately 35% of his BSA?
With the large BSA involvement, he would be best served in
a facility that has experience in managing burn patients. He
will require intensive wound management requiring an
expert multidisciplinary approach.

What are the differential diagnoses and the


leading diagnosis?
Figure 39.1 Plaques and bullae on back during the day of There are several diagnoses on the differential, most nota-
admission. Source: Private collection.
bly Stevens–Johnson syndrome and toxic epidermal necro-
lysis (SJS/TEN). Early in his disease course, his hands and
the soles of his feet were suspicious for hand, foot and
mouth disease. The areas of early erythema on the hands
Table 39.1 Basic metabolic profile.
and feet and his fever could also be concerning for Kawa-
Sodium 144 mEq/L saki disease. With the blisters and erythematous skin the
diagnosis of staphylococcal scalded skin syndrome should
Potassium 4.1 mEq/L
also be considered. The progressive nature of the skin
Chloride 114 mEq/L
lesions and the transition to blisters makes SJS/TEN more
Carbon dioxide 25 mEq/L likely. Both SJS and TEN will have mucosal involvement in
Blood urea nitrogen 18 mg/dL 90% of cases. The distinguishing feature between the two is
Creatinine 0.4 mg/dL the percentage of BSA involved. SJS has skin detachment
Glucose 178 mg/dL that is usually less than 10% BSA. If there is 10–30% BSA
Albumin 2.6 g/dL of skin involvement, the disease is referred to as SJS/TEN
overlap. The diagnosis of TEN is made when there is over
30% BSA skin involvement.
SJS and TEN are severe cutaneous adverse reactions.
Questions Most commonly, SJS/TEN is triggered by medications
and is associated with relatively high morbidity and mortal-
Answer the following questions using the details ity. Medications that have been associated with SJS/TEN
provided. include phenobarbital, carbamazepine (Tegretol), non-
1 What considerations are needed for his disposition con- steroidal anti-inflammatory medications, allopurinol, and
sidering the dermatologic findings involving approxi- fluconazole. Typically, medications with longer half-lives
mately 35% of his BSA? result in more serious disease. Therapy for SJS/TEN
2 What are the differential diagnoses and the leading includes removing the inciting agent (e.g. medication),
diagnosis? proper fluid and electrolyte balance, maintenance of aseptic
3 What is the most concerning feature of this examina- environment, dressing changes/wound debridement, and
tion that needs to be addressed in the PICU? exquisite eye care with referral to ophthalmologist as indi-
4 What subspecialties should be consulted in his care and cated by individual patient. Systemic steroids are indicated
what treatments do you anticipate they would in the treatment of SJS/TEN, often in conjunction with
recommend? other therapies including ciclosporin, tacrolimus, IVIG,
5 What other concerns would you need to anticipate and cyclophosphamide. Other strategies include plasma-
when managing his fluids and nutrition? pheresis and biologic agents (e.g. anti-tumor necrosis factor
Downloaded from https://onlinelibrary.wiley.com/doi/ by National Institute Of Standard, Wiley Online Library on [06/03/2024]. See the Terms and Conditions (https://onlinelibrary.wiley.com/terms-and-conditions) on Wiley Online Library for rules of use; OA articles are governed by the applicable Creative Commons License
Chapter 39 Child with Rash and Drooling 167

recommendation to not give acetaminophen or ibuprofen


in the future as these medications may have been implicated
in the diagnosis. Dermatology also recommended treating
him with intravenous gamma globulin (IVIG) 1 g/kg
daily for 3 days as well as methylprednisolone 2 mg/kg daily
for 3 days. The use of IVIG can block the keratinocyte death
receptor Fas, therefore limiting the destruction of the
epidermis. It was also recommended to keep blisters intact
if possible (Figure 39.2).
Ophthalmology was consulted to assist in the manage-
ment of his conjunctivitis. The initial recommendation
was for erythromycin eye ointment every 2 hours alternat-
ing with Lacri-Lube every 2 hours. Ten days after admission
he began to develop epithelial injury to his cornea and he
was taken to the operating room for an amniotic membrane
Figure 39.2 Progression of epidermal destruction on back transplantation (AMT) over his corneas bilaterally. An
13 days after admission. Source: Private collection. AMT can help repair a damaged cornea by acting as a scaf-
fold that promotes cell migration and epithelial regenera-
alpha medications), but there remain controversies with tion on the cornea.
regard to all recommended therapeutics.

What is the most concerning feature of this What other concerns would you need to anticipate
examination that needs to be addressed in when managing his fluids and nutrition?
the PICU? On presentation, he required fluid resuscitation related to
While in the ED and PICU, he was noted to have drooling 2-day history of poor oral intake and alterations in his skin
and a hoarse voice, which are features that indicate concern barrier. Once his initial deficit was replete, he was placed
about his ability to protect his airway and the possibility of on a maintenance intravenous fluid rate. Patients with
edema and sloughing that could obstruct his upper airway. SJS/TEN may not require as much fluid as those with
His aiway was secured via nasotracheal intubation per- burns; however, they have increased insensible losses
formed by anesthesia with fiberoptic guidance. The naso- and higher fluid requirements than individuals with intact
tracheal tube was chosen as it provides more stability for skin. In addition to fluid loss due to disruption in epithelial
securement and provides an increased level of comfort with skin cells, he is also at significant risk for infection due to
potential prolonged intubation. loss of this barrier. He was transitioned to early enteral
feeds via a nasogastric tube to help optimize his nutrition
What subspecialties should be consulted on his for wound healing; a high-protein diet is needed to promote
care and what treatments do you anticipate they wound healing.
would recommend?
The etiology of the disease was never determined for the
child, but a consult with dermatology indicated, first, a

Further Reading
Barron, S.J., Del Vecchio, M.T., and Aronoff, S.C. (2015). Gupta, L.K., Martin, A.M., Agarwal, N., et al. (2016).
Intravenous immunoglobulin in the treatment of Stevens– Guidelines for the management of Stevens–Johnson
Johnson syndrome and toxic epidermal necrolysis: a meta- syndrome/toxic epidermal necrolysis: an Indian perspective.
analysis with meta-regression of observational studies. Indian Journal of Dermatology, Venereology and Leprology 82
International Journal of Dermatology 54(1): 108–115. (6): 603–625.
Downloaded from https://onlinelibrary.wiley.com/doi/ by National Institute Of Standard, Wiley Online Library on [06/03/2024]. See the Terms and Conditions (https://onlinelibrary.wiley.com/terms-and-conditions) on Wiley Online Library for rules of use; OA articles are governed by the applicable Creative Commons License
168 Cases in Pediatric Acute Care

Kumar, R., Das, A., and Das, S. (2018). Management of diagnosis, differential diagnosis, prevention and
Stevens–Johnson syndrome/toxic epidermal necrolysis: treatment. Journal of the American Academy of
looking beyond the guidelines. Indian Journal of Dermatology 69(2): 187.e1–187.e16. doi: 10.1016/j.
Dermatology 63(2): 117–124. jaad.2013.05.002.
Liu, W,. Nie, X., and Zhang, L. (2016). A retrospective analysis Sharma, N., Thenarasun, S.A., Kaur, M., et al. (2016). Adjuvant
of Stevens–Johnson syndrome/toxic epidermal necrolysis role of amniotic membrane transplantation in acute ocular
treated with corticosteroids. International Journal of Stevens–Johnson syndrome: a randomized control trial.
Dermatology 55: 1408–1413. Ophthalmology 123(3): 484–491.
Schwartz, M.A., Mcdonough, P.H., and Lee, B.W. (2013).
Toxic epidermal necrolysis: Part II. Prognosis, sequelae,
Downloaded from https://onlinelibrary.wiley.com/doi/ by National Institute Of Standard, Wiley Online Library on [06/03/2024]. See the Terms and Conditions (https://onlinelibrary.wiley.com/terms-and-conditions) on Wiley Online Library for rules of use; OA articles are governed by the applicable Creative Commons License
169

40

Child with Fever and Sore Throat


Stephanie Robell1 and Danielle Van Damme2
1
C.S. Mott Children’s Hospital, Ann Arbor, MI, USA
2
Norton Children’s Hospital, University of Louisville, Louisville, KY, USA

A 4-year-old male presents to the emergency department Social History


(ED) with fever, right-sided neck swelling, and sore throat.
Lives at home with both parents and one sibling. He is not
exposed to smoke and attends school and daycare.
History of Present Illness

He has a 1-week history of headache and sore throat. He


was evaluated by his primary care provider (PCP) at the Current Status
time and diagnosed with viral upper respiratory infection
(URI) and pharyngitis. This morning, he woke up with a On arrival to the ED, he appears moderately ill but non-
fever, right-sided neck swelling, difficulty swallowing, toxic. On examination, he has an erythematous right tym-
and pain with neck movement. His mother took him back panic membrane with effusion and his oropharynx is ery-
to the PCP, where an influenza swab and streptococcus thematous with midline uvula. His right neck is swollen
throat culture were obtained. The PCP then referred the with an 8 × 5 cm mass that is tender and without fluctu-
child to the ED for further evaluation. ance. He has decreased range of motion of his neck and
mild trismus. Vital signs include temperature of 39.1 C
(102.3 F), heart rate 118 bpm, blood pressure 96/65 mmHg,
Past Medical History respiratory rate 18 breaths per minute, and SpO2 98% in
room air.
The patient was born full term via spontaneous vaginal
delivery with no complications. Immunizations are up to
date for age, with exception of influenza vaccine this year.
There have been no recent sick contacts. Questions

Answer the following questions using the details provided.


Past Surgical History 1 Based on the clinical data, what are the differential
diagnoses and what is the leading diagnosis consistent
No prior surgeries. with the presenting symptoms and physical examina-
tion findings?
2 Based on your differential diagnoses, what diagnostic
Family History studies are warranted?
3 What treatment would be recommended?
No pertinent family history. 4 What are the management priorities?

Cases in Pediatric Acute Care: Strengthening Clinical Decision Making, First Edition. Edited by Andrea M. Kline-Tilford and Catherine M. Haut.
© 2020 John Wiley & Sons Ltd. Published 2020 by John Wiley & Sons Ltd.
Downloaded from https://onlinelibrary.wiley.com/doi/ by National Institute Of Standard, Wiley Online Library on [06/03/2024]. See the Terms and Conditions (https://onlinelibrary.wiley.com/terms-and-conditions) on Wiley Online Library for rules of use; OA articles are governed by the applicable Creative Commons License
170 Cases in Pediatric Acute Care

Rationale and Evidence-based Practice


Explanation

Based on the clinical data, what are the


differential diagnoses and what is the leading
diagnosis consistent with the presenting
symptoms and physical examination findings?
Differential diagnoses for this child include retropharyngeal
abscess (RPA), peritonsillar abscess, bacterial tracheitis, epi-
glottitis, viral and bacterial pharyngitis, Lemierre syndrome,
meningitis, and tonsillitis. The leading diagnosis is RPA due
to the patient’s age and presenting symptoms of fever, right-
sided neck swelling, difficulty swallowing, pain with neck
movement, and sore throat. He also has a history of viral
URI and pharyngitis. Peritonsillar abscess does have a sim- Figure 40.1 CT image: retropharyngeal opacity on lateral neck/
ilar presentation to RPA, including sore throat, fever, dys- cervical spine view.
phagia, odynophagia, voice change, drooling, and trismus.
However, in peritonsillar abscess there is uvular deviation
toward the contralateral side, ipsilateral tonsillar bulging
and/or submandibular lymphadenopathy, and neck mass.
RPA is a suppurative deep neck infection that occurs in
the area extending from the base of the skull to the posterior
mediastinum between the posterior pharyngeal wall and
prevertebral fascia. It typically occurs following a URI of
one or more of these areas disseminating through the cap-
illary lymphatic drainage pathways, leading to suppuration
of the retropharyngeal lymph nodes and abscess formation.
RPA is frequently seen in children under 5 years of age and
more often in the winter to spring seasons.

Based on your differential diagnoses, what


diagnostic studies are warranted?
Based on the differential diagnoses, it is reasonable to ini-
Figure 40.2 CT image: worsening fluid collection/abscess on
tially obtain a lateral soft tissue radiograph of the neck; lateral neck/cervical spine view.
however, there is a high false-positive rate due to variability
in the technique of obtaining X-ray images. Therefore, com-
puted tomography (CT) with contrast is the imaging modal-
ity of choice as it can differentiate phlegmon and cellulitis
from abscess (Figures 40.1–40.3). CT scan is also helpful for
surgical planning. Findings of gas or air–fluid levels in the
soft tissue and widening of the prevertebral space beyond
5–7 mm at the level of the second cervical vertebra or
greater than 14 mm at the level of the sixth cervical vertebra
are consistent with a diagnosis of RPA.
Laboratory studies to consider include a throat culture to
evaluate for specific bacterial organisms to guide antimicro-
bial therapy. Complete blood count with differential will
typically demonstrate leukocytosis and possibly thrombo-
cytosis as an inflammatory response. C-reactive protein
may be elevated due to inflammation from a serious bacte-
rial infection. The results from these laboratory studies are Figure 40.3 CT image: demonstrating retropharyngeal fluid
shown in Tables 40.1 and 40.2. collection.
Downloaded from https://onlinelibrary.wiley.com/doi/ by National Institute Of Standard, Wiley Online Library on [06/03/2024]. See the Terms and Conditions (https://onlinelibrary.wiley.com/terms-and-conditions) on Wiley Online Library for rules of use; OA articles are governed by the applicable Creative Commons License
Chapter 40 Child with Fever and Sore Throat 171

Table 40.1 Basic metabolic profile. most common pathogens. Occasionally, Haemophilus spe-
cies, Neisseria species, and Gram-negative enteric microbes
Sodium 137 mEq/L
are isolated. Antibiotic selections to eradicate these types of
Potassium 4.0 mEq/L organisms include beta-lactamase penicillins, cephalospor-
Chloride 107 mEql/L ins, clindamycin, and metronidazole. Vancomycin can be
Carbon dioxide 23 mEq/L considered if there is high suspicion for methicillin-resistant
Blood urea nitrogen 6 mg/dL Staphylococcus aureus (MRSA). Consideration should also
Creatinine 0.3 mg/dL be given to the patient’s ability to swallow, severity of illness,
Glucose 114 mg/dL
and respiratory distress when deciding between intravenous
and oral medication therapy. If there is no response to anti-
Total protein 5.8 g/dL
biotic therapy after 24–48 hours or there is airway compro-
Albumin 2.9 g/dL
mise, then surgical incision and drainage is usually
Calcium 9.0 mg/dL indicated. Steroids may be considered in patients exhibiting
Total bilirubin 0.3 mg/dL severe torticollis, stridor or odynophagia, with symptoms
Alanine aminotransferase 18 U/L typically demonstrated to improve 24 hours after use. Ibu-
Aspartate aminotransferase 23 U/L profen and acetaminophen can also be beneficial in treating
Alkaline phosphatase 130 U/L symptoms of fever, pain, and swelling.
C-reactive protein 7.9 mg/dL
What are the management priorities?
Management of a child with RPA should focus on mainte-
Table 40.2 Complete blood count.
nance and stabilization of the airway, including assessment
White blood cell count 28 200/mm3 of respiratory rate and effort, oxygenation, presence of stri-
Hemoglobin 13 g/dL dor, aeration in lung fields, ability to speak, position of com-
fort, and level of consciousness. It is important to allow the
Hematocrit 38%
patient to assume a position of comfort and to avoid agita-
Platelet count 399 000/mm3
tion in an effort to decrease the risk of airway compromise.
Neutrophils 85.9% Care should be taken during oral examination of these chil-
Eosinophils 2% dren to avoid rupturing the abscess. Supplemental humidi-
Monocytes 7% fied oxygen, nasal trumpet, or positive pressure ventilation
Lymphocytes 5.2% may be beneficial for moderate airway obstruction and
Bands 0% endotracheal intubation or tracheostomy may be indicated
for severe cases. Endotracheal intubation should be per-
formed by skilled providers with extensive training in air-
way management, and an otolaryngologist should be
What treatment would be recommended?
consulted urgently for a potential surgical airway and/or
First-line treatment for RPA, in a clinically stable patient, is operative drainage. A patient will require initial evaluation
antibiotic therapy as approximately 60% of infections will in the ED and close monitoring in a pediatric intensive care
resolve with medical management. Empiric antibiotic unit or acute care unit depending on the severity of illness,
agents should be selected to predominantly cover Gram- due to the risk of airway obstruction requiring an emergent
positive and anaerobic organisms. Streptococcal species, intervention until antibiotic therapy has treated the abscess
staphylococcal species, and respiratory anaerobes are the or surgical drainage is deemed necessary.

Further Reading
Bochner, R.E., Gangar, M., and Belamarich, P.F. (2017). A Klein, M.R. (2019). Infections of the oropharynx. Emergency
clinical approach to tonsillitis, tonsillar hypertrophy, and Medicine Clinics of North America 37: 69–80.
peritonsillar and retropharyngeal abscesses. Pediatrics in Woods, C.R., Cash, E.D., Smith, A.M., et al. (2016).
Review 38(2): 81–92. Retropharyngeal and parapharyngeal abscesses among
Jain, A., Singh, I., Meher, R., et al. (2018). Deep neck space children and adolescents in the United States: epidemiology
abscesses in children below 5 years of age and their and management trends, 2003–2012. Journal of the Pediatric
complications. International Journal of Pediatric Infectious Disease Society 5(3): 259–268.
Otorhinolaryngology 109: 40–43.
Downloaded from https://onlinelibrary.wiley.com/doi/ by National Institute Of Standard, Wiley Online Library on [06/03/2024]. See the Terms and Conditions (https://onlinelibrary.wiley.com/terms-and-conditions) on Wiley Online Library for rules of use; OA articles are governed by the applicable Creative Commons License
173

41

Teenager with Sickle Cell Disease and Fever


Shannon Konieczki and Katelyn Hamann
Children’s Hospital of Michigan, Detroit, MI, USA

A 17-year-old male with sickle cell anemia presented to the he was diagnosed with mononucleosis. He has mild
emergency department (ED) after developing a fever of intermittent asthma for which he takes albuterol as needed.
40 C (104 F) at home.

Past Surgical History


History of Present Illness No prior surgeries.

On presentation, he reports that he felt warm when he


woke up today, and also developed a cough. He took ibu- Family History
profen after taking his temperature, but his fever persisted
and he began to feel dizzy. Given his history of sickle cell Both parents have sickle cell trait; no one else in the family
anemia, he immediately presented to the ED for evaluation. has sickle cell anemia. His maternal grandmother has
He reports that no one else living in his home is sick. He hypertension and diabetes mellitus. No one in the family
attends high school, but had no knowledge of any sick con- has a history of cancer, autoimmune disease, bleeding, or
tacts. He has no recent travel history. He has been able to clotting disease.
attend school/participate in activities without difficulty. He
has not been more tired than normal, and denies night
sweats, rashes, bleeding or bruising. He has been eating
Current Status
well, with no recent dietary change, and bowel movements
are normal. He had no persistent pain or joint swelling. Of In the ED, a fever of 39.3 C (102.7 F) is noted. He has a mild
note, he was diagnosed with mononucleosis one month cough and chest pain. He also has a headache and feels dizzy.
prior and since that time has experienced a 2.3-kg weight His blood pressure is 109/55 mmHg, heart rate 120 bpm,
loss (3% weight loss from baseline). respiratory rate 20 breaths per minute, and oxygen satura-
tion 96% on room air. Bilateral lung sounds are clear and
equal and he has a comfortable respiratory effort.

Past Medical History

He was born full-term without complications. Sickle cell


Questions
anemia was diagnosed on newborn screen, and he has
Answer the following questions using the details provided.
had appropriate follow-up appointments since birth. He
has a history of acute chest syndrome that required a blood 1 Based on the data given, what are the differential diag-
exchange transfusion when he was 4 years old. His baseline noses for this patient?
hemoglobin ranges from 7 to 9 g/dL. His last admission to 2 What initial diagnostic testing is indicated?
the hospital was 1 month prior with symptoms of fever, and 3 What therapy would you initiate?

Cases in Pediatric Acute Care: Strengthening Clinical Decision Making, First Edition. Edited by Andrea M. Kline-Tilford and Catherine M. Haut.
© 2020 John Wiley & Sons Ltd. Published 2020 by John Wiley & Sons Ltd.
Downloaded from https://onlinelibrary.wiley.com/doi/ by National Institute Of Standard, Wiley Online Library on [06/03/2024]. See the Terms and Conditions (https://onlinelibrary.wiley.com/terms-and-conditions) on Wiley Online Library for rules of use; OA articles are governed by the applicable Creative Commons License
174 Cases in Pediatric Acute Care

Table 41.1 Basic metabolic profile.

Sodium 141 mEq/L


Potassium 4.1 mEq/L
Chloride 102 mEq/L
Carbon dioxide 24 mEq/L
Blood urea nitrogen 7 mg/dL
Creatinine 0.7 mg/dL
Glucose 100 mg/dL

Table 41.2 Complete blood count.

White blood cell count 25 000/mm3


Hemoglobin 7 g/dL
Hematocrit 20.2%
Platelet count 234 000/mm3
Neutrophils 88%
Eosinophils 0%
Monocytes 2%
Figure 41.1 Chest radiograph showing bilateral lower lobe
Lymphocytes 7% pneumonia.
Bands 3%

Continuation of case: The results of the initial laboratory Table 41.3 CMV viral load.
investigations become available (Tables 41.1 and 41.2).
Day 2 of admission 353 853
Chest X-ray is obtained and noted to be unremarkable.
Day 4 of admission 485 082
On day 3 of admission, his fevers persist and he reports
chest tightness. His pulse-oximeter reading drops to 88% Day 7 of admission 15 862
and he requires 2 L/min of oxygen via nasal cannula. On
auscultation, his chest sounds are diminished bilaterally.
No crackles are heard. A chest radiograph is obtained
Table 41.4 Complete blood count day 7 of admission.
and is significant for bilateral lower lobe pneumonia
(Figure 41.1). Polymerase chain reaction (PCR) for cyto- White blood cell count 27 000/mm3
megalovirus (CMV) and Epstein–Barr virus (EBV) returns
Hemoglobin 6.9 g/dL
positive but blood culture remains negative.
Hematocrit 19.8%
Infectious disease service is consulted, and ganciclovir is
Platelet count 158 000/mm3
started for suspected CMV pneumonia. Azithromycin is
added for possible concurrent mycoplasma pneumonia, Neutrophils 68%
and cefepime is continued. The CMV viral load decreases Eosinophils 3%
appropriately (Table 41.3), but fevers are persistent. On Monocytes 5%
day 7 after admission, another complete blood count is Lymphocytes 8%
obtained (Table 41.4). Bands 4%

Rationale and Evidence-based Practice


Explanation infectious etiologies. Infectious etiologies include viral
and bacterial causes of upper respiratory tract infection
Based on the data given, what are the differential
and pneumonia, as well as reactivation of EBV, blood-
diagnoses for this patient?
stream infection, and sexually transmitted infections
Differential diagnoses for acute fever, dizziness, chest pain (STIs). Non-infectious causes include autoinflammatory
and weight loss can be separated into infectious and non- disorders, immune-mediated disease, neoplasia, and
Downloaded from https://onlinelibrary.wiley.com/doi/ by National Institute Of Standard, Wiley Online Library on [06/03/2024]. See the Terms and Conditions (https://onlinelibrary.wiley.com/terms-and-conditions) on Wiley Online Library for rules of use; OA articles are governed by the applicable Creative Commons License
Chapter 41 Teenager with Sickle Cell Disease and Fever 175

hypersensitivity disorders. Specifically, acute chest syn- Table 41.5 HIV testing.
drome must be considered in the setting of sickle cell dis-
HIV antigen/antibody combined Reactive
ease with chest pain.
HIV-1 antibody Geenius Positive
What initial diagnostic testing is indicated?
Initial diagnostic testing should include a basic metabolic
written consent is required. The general consent for medi-
profile to evaluate for electrolyte derangements, dehydra-
cal care required on admission is sufficient consent for test-
tion and renal function, and a complete blood count is
ing. Patients may decline testing, referred to as “opt-out
needed to evaluate for infection. C-reactive protein can also
screening,” and should be documented in the medical
be obtained as a marker of inflammation/infection. A blood
record.
culture is needed to evaluate for bloodstream infection.
Because of the new cough, a chest radiograph is needed Continuation of case: On further questioning, the patient
to evaluate for a pulmonary process. Because of the recent discloses to family and the medical team that he has been
history of mononucleosis, PCR for CMV and EBV is also sexually active and that he engaged in receptive sex with
obtained. one male partner 2 months previously. He was unaware
of his partner’s HIV status. Laboratory work to evaluate
What therapy would you initiate? HIV status is obtained (Table 41.5).
Given his history of sickle cell anemia, he is started on 6 What are the symptoms of acute HIV and what testing
intravenous 0.9% saline solution for rehydration and a should be done to diagnose it?
dose of ibuprofen is administered for his fever. He is trans- 7 What is the treatment of acute HIV?
ferred to the hematology department from the ED for con-
tinued evaluation of his fever and monitoring. He
What are the symptoms of acute HIV and what
continues to have fevers up to 40.4 C (104.7 F) despite
testing should be done to diagnose it?
receiving ibuprofen and acetaminophen (alternating)
every 6 hours. He is started on intravenous cefepime for Acute infection of HIV is the period between viral transmis-
broad-spectrum bacterial coverage while cultures are sion and the emergence of HIV-specific antibodies, gener-
pending, and he is monitored for signs of acute chest syn- ally accompanied by a burst of viremia. Symptoms of acute
drome. He is continued on intravenous 0.9% saline at HIV infection are non-specific and include fever, rash,
maintenance fluid rate. malaise, myalgia, and headache. Acute HIV infection is
often described as mononucleosis or influenza-like. Usually
4 Because of the persistent fevers, what further questions
symptoms develop 1–4 weeks after transmission, and per-
would you ask the patient?
sist for 2–4 weeks. Physical examination can be notable
5 What other tests would be appropriate?
for hepatosplenomegaly. Presentation may consist of new
opportunistic infections, indicating a compromised
Because of the persistent fevers, what further immune system.
questions would you ask the patient? According to CDC, initial testing for HIV should be done
via the HIV-1/HIV-2 antigen/antibody immunoassay (a
According to the history of the present illness, the patient
fourth-generation HIV test). This test detects HIV antibo-
had not recently traveled, had no family history risk fac-
dies as well as the HIV p24 antigen. Prior testing using
tors for immune-mediated disease, and did not show
the third-generation HIV tests only detected HIV-specific
any symptoms of a neoplasm such as bone pain or night
antibodies, missing the earliest pre-antibody phase, and
sweats. He is also older than the typical patient with an
may result in false negatives. If the first HIV antigen/anti-
autoinflammatory disease. It would be appropriate to
body combined test is “reactive,” it is considered positive
ask questions about drug use and about sexual history
for HIV. Next, testing should be performed to differentiate
to evaluate for risk of STI.
HIV-1 and HIV-2 antibodies. The confirmatory test, as used
in this case, reports the HIV-1 antibody as “positive” con-
What other tests would be appropriate?
firming HIV-1.
Testing should be done for HIV and other STIs such as gon- New rapid HIV tests are HIV antibody-based tests (RNA
orrhea, syphilis, chlamydia, and hepatitis. According to the viral load) that measure the level of HIV nucleic acid using
Centers for Disease Control and Prevention (CDC), patients quantitative PCR testing, branched DNA, or nucleic acid
should be notified that HIV screening will be done, but no amplification assays, and can show positive 11 days from
Downloaded from https://onlinelibrary.wiley.com/doi/ by National Institute Of Standard, Wiley Online Library on [06/03/2024]. See the Terms and Conditions (https://onlinelibrary.wiley.com/terms-and-conditions) on Wiley Online Library for rules of use; OA articles are governed by the applicable Creative Commons License
176 Cases in Pediatric Acute Care

time of infection. To confirm diagnosis, viral load testing and mortality. Infected individuals should be identified
should be done with ELISA or Western blot test again early so they can benefit from early ART, as well as redu-
within 4–6 weeks. cing transmission to others.
Additional laboratory tests should be done, including
CD4 and CD8 lymphocyte counts to monitor immune sup-
pression, hepatitis B and C, and tuberculosis.
Case Resolution
What is the treatment of acute HIV?
The patient improved and was discharged from the hospital
Initial management of acute HIV includes psychosocial on day 14. He has since followed up at an HIV clinic. There,
evaluation and stabilization including a domestic violence he is under the care of a multidisciplinary team including a
screen, substance abuse screen, and referral to counseling pediatrician, a nurse practitioner, several nurses, a social
and support services. Providers should educate patients worker, and a psychologist. He was started on bictegravir,
about their infectiousness and how to prevent risk in emtricitabine and tenofovir alafenamide (Biktarvy), and
others. Advances in antiretroviral therapy (ART) have continued on valganciclovir to complete his CMV treat-
transformed HIV into a chronic and treatable disease, but ment. He was also started on levofloxacin for prophylactic
many patients are still being diagnosed at a late stage. Pro- coverage of bacterial infections. He was given education on
viders must be trained to recognize indicators of infection HIV, strategies of medication adherence, counseling on
and subsequent testing for HIV must be promoted. Late safe sex and risk reduction, and was screened for depres-
presentation of HIV is associated with increased morbidity sion/anxiety.

Further Reading
Centers for Disease Control and Prevention (2014). Laboratory Tominski, D., Katchanov, J., Driesch, D., et al. (2016). The late-
Testing for the Diagnosis of HIV Infection: Updated presenting HIV-infected patient 30 years after the
Recommendations. Atlanta, GA: CDC. doi: 10.15620/ introduction of HIV testing: spectrum of opportunistic
cdc.23447. diseases and missed opportunities for early diagnosis. HIV
Rutstein, S., Ananworanich, J., Fidler, S., et al. (2017). Clinical Medicine 18: 125–132.
and public health implications of acute and early HIV Torreggiani, S., Filocamo, G., and Esposito, S. (2016).
detection and treatment: a scoping view. Journal of the Recurrent fever in children. International Journal of
International AIDS Society 20: 21579. doi: 10.7448/ Molecular Science 17(4): 448. doi: 10.3390/ijms17040448.
IAS.20.1.21579.
Downloaded from https://onlinelibrary.wiley.com/doi/ by National Institute Of Standard, Wiley Online Library on [06/03/2024]. See the Terms and Conditions (https://onlinelibrary.wiley.com/terms-and-conditions) on Wiley Online Library for rules of use; OA articles are governed by the applicable Creative Commons License
177

42

Ethical Case Study


Jamie Tumulty
University of Maryland Children’s Hospital, University of Maryland Medical Center, Baltimore, MD, USA

A 15-year-old male with cerebral palsy, hypotonia, develop- Family History


mental delay, asthma, chronic lung disease, gastroesopha-
geal reflux disease, and epilepsy was admitted to the The child is adopted. His biological mother had a rare
pediatric intensive care unit (PICU) with hypoxic X-linked genetic disorder causing severe intellectual
respiratory failure secondary to pneumonia with pleural disability and hypotonia. She is now deceased.
effusion, and septic shock.
This ethical case study will address issues faced by the
acute care pediatric provider to include effective communi- Social History
cation for obtaining informed consent, addressing decision-
making conflict, and when to initiate palliative care consul- The patient has been in medical foster care since age 2 years
tation or an ethical consult. and was recently adopted by the foster parents. His adop-
tive parents have a biological son who is a pediatric ED phy-
sician in another state, and two other children aged 7 and
8 years for whom they provide medical foster care. Parents
History of Present Illness provide all care for their son without home nursing.

The patient is a medically complex child with cerebral


palsy, hypotonia, developmental delay, asthma, chronic Current Status
lung disease, gastroesophageal reflux disease, and epi-
lepsy who presented with fever and acute onset of respi- In the ED, the teen requires aggressive respiratory treatment
ratory distress. Two days prior to presentation he but is ultimately intubated and mechanically ventilated. He
developed increased secretions and was febrile to is transferred to the pediatric intensive care unit (PICU)
38.9 C (102 F). On the day of presentation, his family where a chest tube is placed to evacuate a pleural effusion,
noted labored breathing and he became less interactive, noted on the admission chest radiograph. This child was
which prompted the visit to the emergency depart- intubated with attempts to extubate twice through his PICU
ment (ED). stay. As a result of the two failed extubation attempts, his
underlying muscle weakness and restrictive lung disease,
the PICU team felt a tracheostomy may be of benefit.

Past Medical and Surgical History


Questions
The teen is wheelchair dependent, with problems already
Answer the following questions using the details provided.
described. At baseline, he is non-verbal but alert, interac-
tive, smiles and laughs. He was intubated once for status 1 Has the PICU team given this teen significant opportu-
asthmaticus. nity to recover from his illness and adequate chance for

Cases in Pediatric Acute Care: Strengthening Clinical Decision Making, First Edition. Edited by Andrea M. Kline-Tilford and Catherine M. Haut.
© 2020 John Wiley & Sons Ltd. Published 2020 by John Wiley & Sons Ltd.
Downloaded from https://onlinelibrary.wiley.com/doi/ by National Institute Of Standard, Wiley Online Library on [06/03/2024]. See the Terms and Conditions (https://onlinelibrary.wiley.com/terms-and-conditions) on Wiley Online Library for rules of use; OA articles are governed by the applicable Creative Commons License
178 Cases in Pediatric Acute Care

successful extubation before suggesting tracheostomy? Rationale and Evidence-based Practice


If so, when is the appropriate time to perform Explanation
tracheostomy?
2 How does the team gain consensus with the family on Has the PICU team given this teen significant
plan of care? opportunity to recover from his illness and
3 As a family–provider communication opportunity, adequate chance for successful extubation before
what are the essential components of the family suggesting tracheostomy? If so, when is the
meeting? appropriate time to perform tracheostomy?

Continuation of case: A family meeting was scheduled There is no consensus about the appropriate time to con-
and included the attending physician, acute care nurse sider performing a tracheostomy for pediatric patients. Sev-
practitioner (ACNP), nurse, social worker, child life spe- eral retrospective studies have shown that tracheostomy
cialist, and the chaplain. The family members included performed before 14 days is associated with shorter hospital
the adoptive mother, father, and physician brother who length of stay and fewer complications than tracheostomy
attended via video conferencing. Although a family meet- performed after 14 days (Holloway et al., 2015). In this case,
ing may have been conducted earlier in this child’s admis- there were two failed extubations, a 22-day PICU length of
sion, this family meeting was timely as it occurred within stay, and an expected prolonged hospitalization even if a
24 hours of a crucial decision point. Table 42.1 describes tracheostomy is performed. The patient’s parents stated
this patient’s family meeting. they were not interested in pursuing a tracheostomy.

4 What successful processes are utilized in the family


How does the team gain consensus with the family
meeting?
on plan of care?
5 When should the palliative care team be consulted?
6 Based on this case, when would an ethics committee Families consistently rate communication as one of the
consult be warranted? most important skills of clinicians, often as important as

Table 42.1 Script of family meeting.

Participant Script

Attending physician
• Introduces health team members

• Identifies the problem: patient has had failed attempts at intubation, would benefit from tracheostomy, but
family members are not in agreement

• Inquires about family goals for patient, asking about child’s life at home

• Child was adopted at age 2 and has always been cared for at home by family members


Parents
He has been happy and interactive and it is difficult to see him so ill, especially with breathing issues

Attending physician • Offers options:


– Wean ventilator and add non-invasive technology

••
– Tracheostomy
Educates about the purpose of tracheostomy
Asks what questions the family has

Father • Expresses interest in trying gradual weaning approach, and offers that the tracheostomy and ventilator at
home are overwhelming and is concerned about the effect on family life and other children

Attending physician • Adds information about the possibility of home nursing and child life therapist to support the other children
if tracheostomy is decided

Parents • Share that child has been deteriorating slowly and they would like to inquire about hospice and
palliative care

Attending physician • Offers to organize a meeting with the palliative care team and closes the meeting by asking if there are other
questions
Downloaded from https://onlinelibrary.wiley.com/doi/ by National Institute Of Standard, Wiley Online Library on [06/03/2024]. See the Terms and Conditions (https://onlinelibrary.wiley.com/terms-and-conditions) on Wiley Online Library for rules of use; OA articles are governed by the applicable Creative Commons License
Chapter 42 Ethical Case Study 179

Table 42.2 VALUE acronym. should elicit the patient and family’s preferred method of
decision-making and tailor their own role to match the
V Value family statements
family’s preference.
A Acknowledge family emotions
L Listen to the family
U Understand the patient as a person
As a family–provider communication opportunity,
E Elicit family questions what are the essential components of the family
Source: Data from Curtis (2008). meeting?
There are several platforms for family communication
including participation in daily ICU rounds, daily updates,
clinical skills, especially during critical illness. Parents
electronic patient portals, and interdisciplinary family
want honest, clear, and complete information in order to
meetings (Seaman et al., 2017). Teleconferencing and video
participate in the decision-making for the care of their chil-
conferencing offer the added benefit of allowing ICU provi-
dren even if it is distressing. Seaman et al. (2017) identified
ders to communicate with physically distanced family
five goals of clinician–family communication, including
members, other family support individuals such as faith
establishing a trusting relationship, providing emotional
leaders, family friends, and other members of the health-
support, helping families understand the prognosis and
care team (De Havenon et al., 2015). Ultimately, the ICU
treatment options, allowing clinicians to understand the
family meeting is an essential means of communication
patient as a person, and creating conditions for careful
about patient status and subsequent decision-making about
deliberation about difficult decisions. Table 42.2 shows
goals of care.
an acronym for communication that can be helpful for
Although the evidence to support the benefits of family
interaction with families.
meetings is substantial, they often fail to occur in a timely
Shared decision-making is a central component of
manner. Some studies have shown these benefits from fam-
patient-centered care in the intensive care unit (ICU)
ily meeting that occur 3–6 days after admission, but the
(Kon et al., 2016). The Informed Medical Decisions Foun-
ACCM recommends that multidisciplinary family meetings
dation, American College of Critical Care Medicine
occur within 24–48 hours of ICU admission and are sched-
(ACCM), and American Thoracic Society have defined
uled at regular intervals. Family meetings should be timely,
shared decision-making as a collaborative process that
occur in a setting that is quiet with adequate seating, and
allows patients, or their surrogates, and providers to make
include appropriate interprofessional team members.
healthcare decisions together, based on the best scientific
There are many barriers to family meetings, including
evidence available and the patient’s values, goals, and
burdens of schedule coordination, lack of provider availa-
preferences.
bility, and physical distance of pertinent family partici-
The standard approach to shared decision-making
pants. Other barriers to family meetings include
occurs when:
inadequate communication skill training of providers, cul-

• providers share information with families about the


patient’s medical condition, prognosis, and therapeutic
tural barriers, language, and emotional stress for providers
(and families). Table 42.3 shows strategies to overcome
options along with the associated risks of each option, some of these barriers. The ACCM Guideline for Family
including the option of palliative care; Conferences advocates for multiple strategies to improve

• the family shares with the provider the patient’s values,


goals and preferences;
the effectiveness of family conferences (Kon et al., 2016).
The family meeting structure allows for other team mem-

• the provider and family deliberate together to determine


the best treatment option for the patient and the patient
bers to listen, clarify issues as appropriate, and affirm deci-
sions. A scribe documents the discussion and may use a
is included if developmentally appropriate and clini- template to transfer information to the chart.
cally able;

• once global goals and preferences are determined, the


provider is responsible for making routine decisions
What successful processes are utilized in the
family meeting?
(e.g. frequency of vital signs and laboratory data);

• the provider explains the care provided and rationale in


language that is understandable to the patient and family
• The family meeting is planned to include all identified
family and healthcare team members.
and elicits their preferences in the decision-making proc-
ess (Kon et al., 2016; Davidson et al., 2017). The clinician
• A venue for inclusion of distant family members is pro-
vided by video conferencing.
Downloaded from https://onlinelibrary.wiley.com/doi/ by National Institute Of Standard, Wiley Online Library on [06/03/2024]. See the Terms and Conditions (https://onlinelibrary.wiley.com/terms-and-conditions) on Wiley Online Library for rules of use; OA articles are governed by the applicable Creative Commons License
180 Cases in Pediatric Acute Care

Table 42.3 Goals of family communication.

Goal Skills Strategies

Establish a trusting relationship Patient and sensitive listening VALUE


Providing consistent message Ascertain family’s communication preferences
about patient’s condition and Manage specialists’ input (may require a preliminary
plan of care multidisciplinary meeting)
Provide emotional support Addressing emotions (e.g. anger, VALUE
grief, guilt) Consider consults to pastoral care, child life specialist
If necessary, preparing patient
and family for death
Help families understand Recognizing cultural and Include interpreter if language barrier exists or
diagnosis, prognosis, and language barriers requested by family
treatment options Tailoring explanation of complex Avoid medical jargon, use simple lay terms
physiology and technology to
family’s level of knowledge Tailor language to the comprehension level of the
family
Use written or electronic supplementary materials
(e.g. pamphlets, diagrams)
Develop awareness of own bias to remain open to
patient and family preferences that differ from
your own
Allow clinicians to understand Eliciting information about VALUE
the patient as a person values, goals, and treatment
preferences
Create conditions for careful Attending to family dynamics Use shared decision-making
deliberation about difficult Conflict resolution Dedicated multidisciplinary family meetings
decisions Assisting in decisions about life
sustaining therapies Ethics consultation
Palliative care consultation
Facilitate restful sleep for families (e.g. private room,
noise reduction strategies)
Consider involving the child in decisions if
developmentally appropriate and clinically able
Source: Seaman et al. (2017).

• The physician uses a strategy to seek information from


the family about their child, acknowledge their emotions,
(2016), children with complex medical problems are surviv-
ing longer due to new and improved treatments, especially
listen to their concerns, and offer options. among preterm infants and neonates with congenital disor-

• The meeting is held in a quiet place, gaining family per-


ception by asking for their input, inviting them to share
ders. It is currently estimated that there are approximately
400 000 children living with life-threatening or chronic
their knowledge, and providing knowledge about care complex conditions, with about 17 million adults serving
options. as caregivers to these children. Despite some concern about

• Shared decision-making was facilitated by providing


information about care choices, eliciting family prefer-
consulting too soon, it is recommended to introduce the
concept of palliative care early in the life of a child who
ences, and deliberating to reach a decision. has a life-limiting condition (Center to Advance Palliative
Care, 2016). The American Academy of Pediatrics recom-
When should the palliative care team be mends initiating a palliative care consult when a diagnosis
consulted? is made and integrating interventions, both curative and
Palliative care strives to manage pain, incorporate psycho- comfort measures, throughout the course of illness to sup-
social and spiritual care, identify values of patients and port quality of life. Unfortunately, only about half of all
their loved ones, and resolve conflicts in cases in which children’s hospitals have formal palliative care programs,
the patient is terminally ill (Celie and Prager, 2016). so much more work needs to be done to improve access
According to the Center to Advance Palliative Care for children and families.
Downloaded from https://onlinelibrary.wiley.com/doi/ by National Institute Of Standard, Wiley Online Library on [06/03/2024]. See the Terms and Conditions (https://onlinelibrary.wiley.com/terms-and-conditions) on Wiley Online Library for rules of use; OA articles are governed by the applicable Creative Commons License
Chapter 42 Ethical Case Study 181

Based on this case, when would an ethics


committee consult be warranted? •• Beneficence: the therapy must help the patient.
Non-maleficence: the therapy must not harm the patient.
Ethics consultation is a commonly accepted procedure
for facilitating communication, resolving conflict, and
•• Justice: therapies must be applied fairly.
Autonomy: the patient must be free to decide.
easing moral distress in healthcare. It should be pur- In this case, ethical questions may include the following.
sued when there is disagreement among team members
or between the medical team and family. Some situa-
tions that may warrant an ethics consult include family
• Would a tracheostomy be of benefit or could it
cause harm?
preference to withdraw life-sustaining therapy when
the medical team believes continuing therapy is in
•• Is it fair that a child receive a tracheostomy?
Given that tracheostomy is a standard therapy, can par-
the best interests of the child; family request for inter- ents opt not to pursue tracheostomy?
ventions the medical team believes are futile; and fam-
ily desire to perform rituals that violate state health In this case, the ethical considerations were not chal-
and safety codes (Kon et al., 2016). The ethics commit- lenged. The values and judgments of parents should gener-
tee is advisory only and consults can come from indi- ally be honored in difficult ethical matters, although
vidual healthcare providers, a healthcare team or providers can seek to override parental wishes when the par-
family members. The consult will provide recommen- ent’s request is not in the best interest of the child. However,
dations based on the posed question. Studies have some of these decisions are state regulated (Weise et al.,
shown that ethics consults decrease ICU length of stay 2017). Some ethicists believe the decision should be based
and life-sustaining therapies for patients who poten- on the best interests of the child only, while others consider
tially will not survive to discharge. These consults the best interests of the family. In this patient case, either
may reduce prolonged suffering for patients and decision was supported by the medical team. When making
families. this type of decision, though, families want reassurance that
Ethicists use four main ethical principles to guide recom- they will not be abandoned by the healthcare team if they
mendations for complex cases. choose a palliative care strategy or do not resuscitate.

References
Celie, K. and Prager, K. (2016). Health care ethics consultation Holloway, A.J., Spaeder, M.C., and Basu, S. (2015). Association
in the United States. AMA Journal of Ethics 18(5): 475–478. of timing of tracheostomy on clinical outcomes in PICU
doi: 10.1001/journalofethics.2017.18.5.fred1-1605. patients. Pediatric Critical Care Medicine 16(3): e52–e58. doi:
Center to Advance Palliative Care (2016). Pediatric palliative 10.1097/PCC.0000000000000336.
care field guide. Available at https://www.capc.org/toolkits/ Kon, A., Davidson, J., Morrison, W., et al. (2016). Shared
designing-a-pediatric-palliative-care-program/ decision making in ICUs: an American College of Critical
Curtis, J.R. (2008). 34th Donald F. Egan Scientific Memorial Care Medicine and American Thoracic Society Policy
Lecture: Caring for patients with critical illness and their Statement. Critical Care Medicine 44(1): 188–201. doi:
families: the value of the integrated clinical team. 10.1097/CCM.0000000000001396.
Respiratory Care 53(4): 480–487. Seaman, J., Arnold, R., Scheunemann, L.P., and White, D.B.
Davidson, J., Aslakson, R., Long, A., et al. (2017). Guidelines (2017). An integrated framework for effective and efficient
for family centered care in the neonatal, pediatric, and adult communication with families in the adult intensive care
ICU. Critical Care Medicine 45(1): 103–128. doi: 10.1097/ unit. Annals of the American Thoracic Society 14(6): 1015–
CCM.0000000000002169 1020. doi: 10.1513/AnnalsATS.201612-965OI.
De Havenon, A., Petersen, C., Tanana, M., et al. (2015). A pilot Weise, K.L., Okun, A.L., Carter, B.S., Christian, C.W. and the
study of audiovisual family meetings in the intensive care Committees on Bioethics, Section on Hospice and Palliative
unit: a randomized controlled trial. Journal of Critical Care Medicine and Committee on Child Abuse and Neglect
30(5): 881–883. doi: 10.1016/j.jcrc.2015.05.027. (2017). Guidance on forgoing life-sustaining treatment.
Pediatrics 140(3): e20171905; doi: 10.1542/peds.2017-1905

Further Reading
Moon, M. and American Academy of Pediatrics Committee on Statement. Pediatrics 143(5): e20190659. doi: 10/1542/
Bioethics (2019). Institutional Ethics Committees Policy peds.2019-0659.
Downloaded from https://onlinelibrary.wiley.com/doi/ by National Institute Of Standard, Wiley Online Library on [06/03/2024]. See the Terms and Conditions (https://onlinelibrary.wiley.com/terms-and-conditions) on Wiley Online Library for rules of use; OA articles are governed by the applicable Creative Commons License
183

43

Infant with Neurologic Changes and Respiratory Failure


Blythe Pollack and Stacey Sears
C.S. Mott Children’s Hospital, Ann Arbor, MI, USA

The patient is a 5-month-old male, with no significant med- Family History


ical history, admitted to the pediatric intensive care unit
(PICU) with neurologic changes and acute respiratory No pertinent family medical history. There are two older
failure. siblings in the home, who are currently well, but occasion-
ally have intermittent viral illnesses.

History of Present Illness Current Status

The infant was in his usual state of health until 1 day prior On examination, he appears ill with dry cracked lips,
to his emergency department (ED) visit and subsequent sunken eyes, and flat anterior fontanelle. Extremities are
admission. His parents brought him in with concerns for cool to the touch, with warm head and trunk. He has
one day of fever over 38.9 C (102 F), bilious non-bloody decreased bowel sounds with abdominal distension. Nurs-
emesis, generalized abdominal pain, and decreased oral ing staff have witnessed an episode of emesis while breast-
intake. Ibuprofen was administered for fever relief, but feeding, but he has easy work of breathing, without
the fever returned despite multiple doses. Primarily adventitious sounds. Vital signs on arrival to the ED are
breast-fed, but showed no interest in feeding during the temperature 38.6 C (101.5 F), heart rate 138 bpm, blood
hours preceding visit. He was reported to have three wet pressure 124/59 mmHg, respiratory rate 34 breaths per
diapers during the day. Parents deny constipation or minute, and oxygen saturation 100% on room air.
diarrhea. With concern for an abdominal obstruction and fever in
an infant, an abdominal radiograph, urinalysis, and a res-
piratory viral nasal swab are completed. Because of signs
of dehydration, a peripheral intravenous catheter is placed,
Past Medical History blood work obtained, and rehydration therapy started fol-
lowing an initial 40 mL/kg of normal saline. While in the
He was born term at 39 2/7 weeks with a non-complicated ED, he shows positional preference with his head, and
vaginal delivery. He did not require airway interventions at no longer turns it to the right (with negative meningeal
birth, tolerated breastfeeding, and was discharged home on signs). He has undergone computed tomography (CT) of
day of life 2. His immunizations are up to date. Parents deny the neck without pertinent findings. He continues to have
recent travel. fever and blood cultures are obtained and a single dose of
ceftriaxone administered. An abdominal ultrasound and
upper gastrointestinal investigation are completed for fur-
ther evaluation of his emesis. He is admitted to the general
Past Surgical History care ward. Results of basic metabolic profile, complete
blood count and urinalysis are shown in Tables 43.1–43.3.
No past surgical history. Other initial results are as follows.

Cases in Pediatric Acute Care: Strengthening Clinical Decision Making, First Edition. Edited by Andrea M. Kline-Tilford and Catherine M. Haut.
© 2020 John Wiley & Sons Ltd. Published 2020 by John Wiley & Sons Ltd.
Downloaded from https://onlinelibrary.wiley.com/doi/ by National Institute Of Standard, Wiley Online Library on [06/03/2024]. See the Terms and Conditions (https://onlinelibrary.wiley.com/terms-and-conditions) on Wiley Online Library for rules of use; OA articles are governed by the applicable Creative Commons License
184 Cases in Pediatric Acute Care

Table 43.1 Basic metabolic profile. colon. There is minimal small bowel gas. No pneumoper-
itoneum is seen.
Sodium
Potassium
136 mEq/L
5.1 mEq/L • Abdominal ultrasound: non-visualization of the appen-
dix; however, no free fluid seen in the right lower quad-
Chloride 103 mEq/L rant and no visible inflammatory changes.
Carbon dioxide
Blood urea nitrogen
23 mEq/L
7 mg/dL
• Upper gastrointestinal examination: negative, no evi-
dence of malrotation.
Creatinine 0.2 mg/dL
Glucose 118 mg/dL Questions
Calcium 10.2 mg/dL
Albumin 4.4 g/dL
Answer the following questions using the details provided.
1 Based on the information given, what is the differential
diagnosis?
2 What are meningeal signs?
Table 43.2 Complete blood count.

White blood cell count 15 000/mm3


Hemoglobin 9.9 g/dL Rationale and Evidence-based Practice
Hematocrit 30.8% Explanation
Platelet count 528 000/mm3
Neutrophils 66% Based on the information given, what is the
Eosinophils 1% differential diagnosis?
Monocytes 11% This case was well differentiated in the ED. The differential
Lymphocytes 21% diagnosis includes viral illness, bronchiolitis, dehydration,
Bands 0% constipation, bowel obstruction, malrotation, testicular tor-
sion, pyloric stenosis, meningitis, and peritonsillar and ret-
ropharyngeal abscess.
While in the ED, respiratory and urine samples were sent
Table 43.3 Urinalysis. for viral and bacterial cultures for evaluation of infection.
A chest radiograph was completed to evaluate for infil-
Color Normal
trates. Meningitis was also considered; however, the clini-
Appearance Cloudy
cal examination was not consistent, as he had no nuchal
Specific gravity 1.025 rigidity, was consolable by parents, the fontanelle was flat,
pH 5.5 and there was no evidence of lethargy. However, there was
Leukocyte esterase Negative positional head preference and possible neck stiffness so a
Nitrate Negative CT of the neck was completed to evaluate for retropharyn-
Protein <30 mg/dL geal and peritonsillar abscess. The abdomen was consid-
Glucose Normal ered a concerning source, and therefore appendicitis,
malrotation, pyloric stenosis, and testicular torsion were
Ketones <5 mg/dL
evaluated with abdominal ultrasound, radiography, and
clinical examination.

••
When the differential is wide, supportive care and fre-
Respiratory panel: rhinovirus/enterovirus positive. quent reassessment is optimal. It was noted at the time of
Neck CT: no findings consistent with tonsillar, peritonsil- presentation that he was dehydrated. He had continued
lar or retropharyngeal abscess. Mild prominence of Wal- emesis and poor oral intake, likely in the setting of a viral
deyer’s ring lymphoid tissue without significant illness. Dehydration is a commonly treated symptom in the
associated nasopharyngeal or oropharyngeal airway nar- pediatric population. Pediatric patients have significant
rowing. Presumed reactive lymph nodes identified evaporative losses due to the high ratio of surface area to
throughout the upper neck.


weight. He was given a 40 mL/kg bolus in the addition to
Abdominal radiograph: normal, non-obstructive bowel maintenance intravenous fluids to expand plasma volume
gas pattern. Moderate gas is present within normal-caliber while preventing circulatory collapse.
Downloaded from https://onlinelibrary.wiley.com/doi/ by National Institute Of Standard, Wiley Online Library on [06/03/2024]. See the Terms and Conditions (https://onlinelibrary.wiley.com/terms-and-conditions) on Wiley Online Library for rules of use; OA articles are governed by the applicable Creative Commons License
Chapter 43 Infant with Neurologic Changes and Respiratory Failure 185

What are meningeal signs? setting of acute neurologic decline, his trachea is orally
intubated for airway protection. He is transported to the
Meningeal signs are clinical findings which help the prac-
PICU for further management of his acute neurologic
titioner assess for meningeal inflammation as a sign of
decline with hypoxic respiratory failure.
meningitis. Meningeal clinical signs include neck pain/
Focused neurologic examination produces the following
stiffness, presence of Kernig and/or Brudzinski sign, and
results. His eyes are closed and he does not respond to voice
tripod phenomenon. The Kernig sign evaluates flexion con-
or tactile stimuli other than with triple flection to noxious
tracture in the knees, with inability to flex further than
stimuli in the right lower extremity. His pupils are 1 mm,
135 . To perform the Kernig maneuver, the patient is placed
round and minimally responsive to light. Oculocephalic
in the supine position with hips flexed. Examiner will
reflex not clearly present. He has symmetric facial move-
slowly extend one knee at a time. If the patient experiences
ment with a midline tongue. His four limbs are flaccid. He
pain or spasms with the popliteal angle less than 135 , it is
has no response to noxious stimuli in bilateral upper extre-
considered positive for meningeal irritation. When per-
mities or left lower extremity. He has a 1+ deep tendon reflex
forming the Brudzinski test, the examiner places one hand
(DTR) at the bilateral brachioradialis. Trace DTR at the
on the patient’s chest and the other behind the head while
patella and achilles bilaterally. No clonus. His toes are mute
the patient is in the supine position. When passive flexion
on the left, downgoing on the right with plantar stimulation.
of the neck is performed, there is an involuntary flexion of
the hips and knees. The tripod sign is a different from the 3 What diagnostic studies could be performed to help nar-
tripod position clinicians may associate with asthma. When row the differential diagnosis and determine plan
the clinician observes the patient attempting to sit up, they of care?
will use the support of their hands behind them. This posi- 4 What is the indication for tracheal intubation?
tion allows the weight to be taken off the spine, and prevent 5 What is the triple flexion reflex?
flexion. This maneuver can only be completed in patients
Continuation of case: After 24 hours in the PICU, he
who are old enough to support themselves upright.
develops decreased lung compliance and worsening respi-
There are situations when these maneuvers will not be
ratory examination, likely due to aspiration pneumonia.
present, or are not clinically capable of being performed.
The infant is placed on minimal conventional ventilator
In these situations, clinical findings for meningitis in
settings with a dexmedetomidine infusion for sedation
infants would include lethargy, a full fontanelle, marked
and analgesia. Antibiotic coverage is broadened to cover
irritability with a variable fever, poor feeding, jaundice
for sepsis, aspiration pneumonia, and viral encephalitis.
and decrease in muscle tone. In older children, reported
A CT scan of the head (Figures 43.1–43.3) and lumbar
findings can include photophobia, frontal headaches,
puncture (Table 43.4) are completed. CT evaluation of
myalgia, anorexia, ataxia, and confusion. If the older child
the head is unremarkable, with no convincing evidence
exhibits an increase in intracranial pressure, bradycardia,
of parenchymal or extraparenchymal hemorrhage noted.
hypertension and altered mental status may also be
Focal area of increased attenuation seen over the left pari-
present.
etal convexity appears to be artifactual.
Continuation of case: During the first day of his inpatient
stay, he continues to be febrile and tachycardic, responsive
to antipyretics. He continues to favor his left side and does
not rotate his head to the right. He is irritable with poor oral
intake, but is soothed when held by parents. During his
afternoon feed, he starts to cough which is followed by a
large emesis. An hour following the emesis, the nurse is
called to the room to find patient with retractions, tachyp-
nea, and grunting. A provider is called to bedside, pulse
oximetry placed, and suction attempted. Oxygen saturation
readings are in the 70s, respiratory rate in the 70s, and he is
pale and mottled with weak thready pulses. The rapid
response team is called to evaluate the patient.
The rapid response team arrive and initiate bag mask
ventilation. The infant is not fighting against the positive
pressure. A peripheral intravenous line is placed and he
does not withdraw to the pain when it is inserted. In the Figure 43.1 CT scan of brain on admission to PICU.
Downloaded from https://onlinelibrary.wiley.com/doi/ by National Institute Of Standard, Wiley Online Library on [06/03/2024]. See the Terms and Conditions (https://onlinelibrary.wiley.com/terms-and-conditions) on Wiley Online Library for rules of use; OA articles are governed by the applicable Creative Commons License
186 Cases in Pediatric Acute Care

Table 43.4 Lumbar puncture.

Appearance Clear
Color Colorless
Xanthochromia Absent
Red blood cell count 9/mm3
Leukocytes 108/mm3
Protein 21 mg/dL
Glucose 101 mg/dL
Herpes simplex virus Negative

consistent with mild encephalopathy. No patterns with spe-


cific correlation with seizure were present, and no focal
abnormalities were found.
Magnetic resonance imaging (MRI) of the brain is
completed due to progression of neurologic symptoms.
Findings on MRI suggest rhombencephalitis (likely
viral) due to bilateral symmetric signal abnormalities
involving the brainstem and extending to the cervical
cord. Additionally, there is high suspicion for acute flac-
Figure 43.2 CT scan of brain on admission to PICU. cid myelitis due to his MRI and clinical findings (see
Figures 43.4 and 43.5).

What other diagnostic studies could be performed


to help differentiate and determine plan of care?
At this point in the case, there are only a few diagnostic
studies he has yet to complete. He will require a chest radi-
ograph to evaluate for an aspiration pneumonia in the set-
ting of his acute decline during a choking episode. A CT of

Figure 43.3 CT Scan of brain on admission to PICU.

An electroencephalogram (EEG) is placed for continued


evaluation of brain activity. This 2-day EEG recording
showed an abnormal awake and asleep pattern. The diffuse
background slowing and disorganization was initially con-
sistent with moderate encephalopathy, but improved to Figure 43.4 MRI and magnetic resonance venogram (MRV) of
demonstrate state cycling and sleep architecture, more brain and spine: completed day 2 of admission while in PICU.
Downloaded from https://onlinelibrary.wiley.com/doi/ by National Institute Of Standard, Wiley Online Library on [06/03/2024]. See the Terms and Conditions (https://onlinelibrary.wiley.com/terms-and-conditions) on Wiley Online Library for rules of use; OA articles are governed by the applicable Creative Commons License
Chapter 43 Infant with Neurologic Changes and Respiratory Failure 187

response without brain death. When triple flexion is noted


on examination, further imaging with flow studies, func-
tional imaging or electrophysiologic examinations are
indicated.
6 What is IVIG? Why would you use it?
7 What is rhombencephalitis? What is the most com-
mon cause?
8 What is acute flaccid myelitis?
9 What is the difference between rhombencephalitis and
encephalitis?

What is IVIG? Why would you use it?


IVIG is a blood product, collected from 1000–1500 donors.
The mechanism of action is not well understood, and is
dose dependent. Dosing is based on treatment recommen-
dations and can be adjusted based on clinical outcome. In
this case, the dose was 2 g/kg over 2 days (high dose). Mon-
itoring immunoglobulins (IgG, IgM, IgE) can help deter-
mine appropriate timing of additional dosing of IVIG.
Figure 43.5 MRI/MRV of brain and spine: completed day 2 of IVIG can be used in a wide range of neurologic, immuno-
admission while in PICU.
logic, and hematologic conditions. It was initially studied in
the head and a lumbar puncture are indicated in the setting the 1980s for treatment of immune thrombocytopenia. Later
of acute neurologic decline. However, prior to the lumbar research was expanded to autoimmune and inflammatory
puncture, a CT scan assists in determining acute risk for conditions. IVIG can reduce the number of circulating neu-
herniation. A lumbar puncture can be performed to evalu- trophils and leukocytes, and generally protect against inflam-
ate the cerebrospinal fluid (CSF) for infections, including, matory tissue damage. Literature is available describing
but not limited to, herpes simplex virus, arbovirus, rabies, dosing, outcomes, and overall benefit with use of IVIG and
West Nile virus, varicella zoster, and cytomegalovirus. Pleo- encephalitis, but large-scale studies have not been completed.
cytosis is supportive but not necessarily a diagnostic crite-
rion for encephalitis, as it is only 50% sensitive. CSF glucose What is rhombencephalitis? What is the most
less than 40 mg/dL and protein above 100–500 mg/dL is common cause?
indicative of bacterial meningitis. Predominant lympho-
Rhombencephalitis is inflammation of the hindbrain
cytes in the CSF are indicative of tuberculous and viral
(brainstem and cerebellum). The terms “brainstem enceph-
meningitis, while predominant neutrophils are more indic-
alitis” and “rhombencephalitis” are synonymous. Causes
ative of bacterial meningitis.
include infectious, autoimmune etiology, or paraneoplastic
syndromes. The most common infectious agents are Liste-
What is the indication for tracheal intubation? ria, enterovirus 71, and herpes simplex virus. Behçet’s dis-
The deteriorating neurologic examination is the indication ease, systemic lupus erythematosus, and relapsing
for endotracheal intubation. Pediatric patients can quickly polychondritis have also been documented to cause rhom-
become unable to protect their airway due to loss of pharyn- bencephalitis. MRI is the gold standard for evaluation. The
geal reflexes, significantly increasing the risk of aspiration. most notable abnormal hyperintense areas on MRI that
A poor respiratory effort can lead to hypoventilation, hyper- indicate viral encephalitis are seen in T2-weighted
carbia, hypoxia, and further progress to a respiratory arrest. imaging and fluid-attenuated inversion recovery pulse
image (FLAIR) sequences. Some studies suggest using
diffusion-weighted imaging during MRI as it could be used
What is the triple flexion reflex?
as a predictor of outcome. A CT is often done early in the
Triple flexion is a spasm in which the thigh flexes towards course of illness to evaluate the etiology of mental status
the pelvis, the calf to the thigh, and the foot to the calf. This changes, but it often cannot capture changes associated
movement is a spinal reflex, which may be present as a with rhomboencephalitis.
Downloaded from https://onlinelibrary.wiley.com/doi/ by National Institute Of Standard, Wiley Online Library on [06/03/2024]. See the Terms and Conditions (https://onlinelibrary.wiley.com/terms-and-conditions) on Wiley Online Library for rules of use; OA articles are governed by the applicable Creative Commons License
188 Cases in Pediatric Acute Care

What is acute flaccid myelitis?


Acute flaccid myelitis is a condition of the nervous system
that primarily affects the gray matter, leading to weakness
of the muscle and reflexes. Symptoms are consistent with
sudden weakness of the limbs, difficulty moving eyes or eye-
lids, difficulty swallowing or speaking, and a facial droop. It
has historically been associated with poliovirus, but now is
seen in association with non-polioviruses such as enterovi-
rus 68 and 71, coxsackievirus, adenovirus, and West Nile
virus. Supportive care is the best treatment, with aggressive
physical and occupational therapy to maximize function.
Most children with acute flaccid myelitis display residual
motor deficits 12 months following initial diagnosis.

What is the difference between rhombencephalitis


and encephalitis?
Figure 43.7 MRI of brain and spine 10 days following prior study.
Common terms used for encephalitis are not interchange-
able. Encephalitis is defined as altered level of conscious-
ness lasting for 24 hours or more, plus two or more of
the following: fever (temperature >38 C), seizures (gener-
alized or partial) or abnormality on EEG, pleocytosis (white
blood cells in CSF ≥5/mm3), neuroimaging consistent with
encephalitis, or detection of autoantibodies in CSF
(Figure 43.6). When the criteria for encephalitis are not
met due to lack of inflammatory criteria (e.g. absence of
fever, CSF pleocytosis, and/or imaging changes) and with
altered mental status lasting more than 24 hours, it is
defined as encephalopathy. Both encephalitis and

Brain Parenchymal
Neurologic Dysfunction + Inflammation

Encephalopathy:
altered mental
status >24 h
Major
• EEG c/w + • Fever
Encephalitis
Minor Figure 43.8 MRI of brain and spine 10 days following prior study.
2: possible • CSF pleocytosis
• Seizures
≥3: probable
• Neuroimaging c/w
• Focal neurologic brain inflammation
findings encephalopathy include all structures of the brain paren-
chyma, while rhombencephalitis is specific inflammation
of the hindbrain structures only.

Encephalitis
Case Resolution
Figure 43.6 Overview of diagnostic criteria for encephalitis.
Source: Messacar, K., Fischer, M., Dominguez, S., Tyler, K.L., and
Abzug, M.J. (2018). Encephalitis in US children. Infectious Disease
High-dose steroids and intravenous immunoglobulin
Clinics of North America 32(1): 145–162. Reproduced with (IVIG) are administered without improvement. He receives
permission of Elsevier. multiple rounds of antibiotics. Twenty days after initial
Downloaded from https://onlinelibrary.wiley.com/doi/ by National Institute Of Standard, Wiley Online Library on [06/03/2024]. See the Terms and Conditions (https://onlinelibrary.wiley.com/terms-and-conditions) on Wiley Online Library for rules of use; OA articles are governed by the applicable Creative Commons License
Chapter 43 Infant with Neurologic Changes and Respiratory Failure 189

admission, subtype of enterovirus resulted as D71. Addi- The patient did not regain neurologic function following
tionally, the pulmonary and stable ventilator teams are con- his course in the PICU. After multiple attempts to wean
sulted due to prolonged intubation course and poor ventilatory support, he underwent a tracheostomy place-
respiratory drive. Additional neuroimaging with repeat ment on day 27 of admission. He required a ventilator for
MRI of the brain is completed due to lack of clinical respiratory support, and a nasogastric feeding tube for
improvement despite aggressive care and interventions nutrition. He regained only small movements of his fingers
(see Figures 43.7 and 43.8). In comparison with the previ- and toes on the right side of his body, and no spontaneous
ous brain MRI, there is significant improvement in the dif- large motor movements. He could not track past midline,
fusion signal involving the brainstem, median thalami, and and still favored head position to the left. He continued
upper cervical spinal cord. with physical therapy to facilitate maximal recovery.

Further Reading
Campos, L., Trindade, R., Faistauer, Â., et al. (2016). Iro, M., Martin, N., Absoud, M., and Pollard, A.J. (2017).
Rhombencephalitis: pictorial essay. Radiologia Brasileira Intravenous immunoglobulin for the treatment of childhood
49(5): 329–336. doi: 10.1590/0100-3984.2015.0189. encephalitis. Cochrane Database of Systematic Reviews (10):
Centers for Disease Control and Prevention (2019). Acute CD011367. doi: 10.1002/14651858.
flaccid myelitis. http://www.cdc.gov/features/acute-flaccid- Messacar, K., Fischer, M., Dominguez, S., Tyler, K.L., and
myelitis/ (accessed 31 March 2019). Abzug, M.J. (2018). Encephalitis in US children. Infectious
Gogou, M., Papadopoulou-Alataki, E., Spilioti, M., Alataki, S., Disease Clinics of North America 32(1): 145–162. doi:
and Evangeliou, A. (2017). Clinical applications of 10.1016/j.idc.2017.10.007.
intravenous immunoglobulins in child neurology. Current Reuter-Rice, K. and Bolick, B. (2012). Pediatric Acute Care:
Pharmaceutical Biotechnology 18(8): 628–637. doi: 10.2174/ A Guide to Interprofessional Practice. Burlington, MA:
1389201018666170915123700 Jones & Bartlett Learning LLC.
Graeter, S., Simon, H., and von Gunten, S. (2019). Granulocyte Shaffner, D. and Nichols, D. (2016). Rogers’ Textbook of
death mediated by specific antibodies in intravenous Pediatric Intensive Care, 5th edn. Philadelphia, PA: Wolters
immunoglobulin (IVIG). Pharmacological Research 2019: Kluwer.
104168. doi: 10.1016/j.phrs.2019.02.007. Venkatesan, A. and Geocadin, R. (2014). Diagnosis and
Hopkins, S. (2017). Acute flaccid myelitis: etiologic challenges, management of acute encephalitis: a practical approach.
diagnostic and management considerations. Current Neurology: Clinical Practice 4(3): 206–215. doi: 10.1212/
Treatment Options in Neurology 19(12): 48. doi: 10.1007/ CPJ.0000000000000036.
s11940-017-0480-3.
Downloaded from https://onlinelibrary.wiley.com/doi/ by National Institute Of Standard, Wiley Online Library on [06/03/2024]. See the Terms and Conditions (https://onlinelibrary.wiley.com/terms-and-conditions) on Wiley Online Library for rules of use; OA articles are governed by the applicable Creative Commons License
191

44

Acute Hyperglycemia
Jeanne Little
Rush University College of Nursing, Chicago, IL, USA

An 8-year-old female with no significant past medical A 5-year-old brother is healthy. Paternal grandmother
history presents to the emergency department (ED) with has adult-onset diabetes.
lethargy, new abdominal pain with nausea and vomiting.

Current Status
History of Present Illness
On arrival to the ED, the child appears dehydrated on
This 8-year-old female presented to her primary care pro- examination with dry buccal mucosa and lips, capillary
vider (PCP) 1 day ago with nausea and vomiting, without refill of 3 seconds, tenting skin turgor, and slightly dimin-
diarrhea for 3 days. She has had a 1.8 kg weight loss since ished pedal and radial pulses, with cool extremities. She is
her last well child visit, which was 2 months ago. General- drowsy but opens her eyes when spoken to, and has a Glas-
ized abdominal discomfort started today so she was brought gow Coma Scale score of 14. Vital signs are as follows: tem-
to the ED. The child’s appetite has been worse over the past perature 37.1 C (98.7 F), heart rate 125 bpm, blood
month, and she admits to drinking more fluids. Her abdom- pressure 112/65 mmHg, respiratory rate 26 breaths per
inal pain is a constant dull ache and nothing makes it worse minute, oxygen saturation 96% on room air, and general-
or better. The child reports using the bathroom more at night ized abdominal pain 4/10. She has no retractions or signs
and she wet the bed last week so sleep has been disrupted. of increased work of breathing. No localized abdominal
There are no known sick contacts and the patient is attend- tenderness during palpation. A peripheral intravenous
ing some summer day camps where she felt fatigued. catheter is inserted. A point-of-care test (POCT) glucose
is 482 mg/dL. Serum basic metabolic profile (BMP), venous
blood gas analysis, complete blood count, and urine
Past Medical History ketones are also obtained (see Tables 44.1–44.4).

The child was born full-term and went home with mother.
She has had all recommended immunizations including the Questions
annual influenza vaccine. She has no previous hospitaliza-
tions or surgeries, does not take any medications, and has Answer the following questions using the details provided.
no known allergies. Her stature and weight are both in the
50% band for age and gender. Her current weight is 22 kg, 1 Based on the data, what are the differential diagnoses
height 119 cm, and body mass index (BMI) 15.5. and the most likely diagnosis for this child?
2 What is the best way to treat dehydration, acidosis, and
hyperglycemia and how would these things be ordered?
3 Which monitoring parameters and laboratory studies
Family History would you follow, how often, and why?
Mother has a history of hypothyroidism, but it is well Continuation of case: The patient is admitted to the
controlled. Her father is on medication for hypertension. pediatric intensive care unit (PICU) and receives an initial

Cases in Pediatric Acute Care: Strengthening Clinical Decision Making, First Edition. Edited by Andrea M. Kline-Tilford and Catherine M. Haut.
© 2020 John Wiley & Sons Ltd. Published 2020 by John Wiley & Sons Ltd.
Downloaded from https://onlinelibrary.wiley.com/doi/ by National Institute Of Standard, Wiley Online Library on [06/03/2024]. See the Terms and Conditions (https://onlinelibrary.wiley.com/terms-and-conditions) on Wiley Online Library for rules of use; OA articles are governed by the applicable Creative Commons License
192 Cases in Pediatric Acute Care

Table 44.1 Basic metabolic panel. 4 If the patient develops a headache during the treatment,
what is the most important treatment?
Sodium 134 mEq/L
5 What indicators are used to transfer from intravenous
Potassium 3.7 mEq/L insulin to subcutaneous administration?
Chloride 101 mEq/L
Carbon dioxide 12 mEq/L
Blood urea nitrogen 24 mg/dL
Creatinine 0.9 mg/dL
Rationale and Evidence-based Practice
Glucose >511 mg/dL
Explanation
Calcium 9.0 mg/dL
– –
Based on the data, what are the differential
Anion gap (Na + K ) – (Cl + HCO3 )
+ +
25 mEq/L diagnoses and the most likely diagnosis for
this child?
Differential diagnoses for this child include insulin-
Table 44.2 Venous blood gas analysis.
dependent diabetes mellitus (IDDM or type 1 diabetes) with
pH 7.18 diabetic ketoacidosis (DKA), type 2 diabetes under extreme
PCO2 35 mmHg
stress (infection, trauma), hyperglycemic hyperosmolar
state, and other causes of high anion gap acidosis such as
PO2 38 mmHg
ingestion of salicylate, methanol, and ethylene glycol. Gas-
Bicarbonate 13 mEq/L
troenteritis, which can precipitate DKA, may also be
Base excess –8 considered.
The most likely diagnosis for this child is new-onset
IDDM with DKA. This child presents with classic symp-
Table 44.3 Complete blood count. toms of polyuria, nocturia, enuresis, polydipsia, and weight
loss. Abdominal discomfort and vomiting are also common.
White blood cell count 14 000/mm3 IDDM results from immune-mediated pancreatic beta-cell
Hemoglobin 14 g/dL destruction leading to an absolute deficiency in insulin
Hematocrit 45% secretion. Insulin is required in the metabolism of glucose
Platelet count 232 000/mm3 for energy. Without insulin, glucose levels rise and the body
shifts to a catabolic state by breaking down glycogen stores,
using triglycerides from adipose tissues, and using amino
Table 44.4 Urinalysis. acids from muscle for energy. The cause of IDDM is
thought to be multifactorial, but the exact roles of genetic
Nitrites Negative susceptibility, environmental factors, and the immune sys-
Blood Negative tem are unclear. Without prompt diagnosis, IDDM results
Ketones Moderate in weight loss, persistent hyperglycemia, dehydration from
Leukocytes Negative
osmotic diuresis/glycosuria, ketone body production, and
acidosis. The biochemical criteria for the diagnosis of
Glucose 3+
DKA are (i) hyperglycemia (blood glucose >200 mg/dL);
Pregnancy test Negative
(ii) venous pH <7.3 or serum bicarbonate <15 mmol/L;
and (iii) ketonemia or ketonuria.

fluid bolus of 0.9% normal saline 20 mL/kg over 1 hour. What is the best way to treat dehydration, acidosis,
and hyperglycemia and how would these things be
Replacement fluids of 0.9% normal saline with 20 mmol/L
ordered?
potassium acetate and 20 mmol/L potassium phosphate are
initiated to account for dehydration (10%) and daily main- Goals of therapy for IDDM with DKA are to improve cir-
tenance fluids. One hour after fluid replacement therapy culatory volume and tissue perfusion, correct electrolyte
has started a continuous intravenous insulin infusion is derangements, reverse acidosis and ketosis, gradually
started at 0.1 unit/kg per hour. A second peripheral intra- reduce serum glucose and osmolality to near normal,
venous line is placed. After 1 hour on continuous insulin, a and to prevent and monitor for DKA complications.
repeat POCT glucose is 390 mg/dL. Calculations for fluid deficit can be completed with
Downloaded from https://onlinelibrary.wiley.com/doi/ by National Institute Of Standard, Wiley Online Library on [06/03/2024]. See the Terms and Conditions (https://onlinelibrary.wiley.com/terms-and-conditions) on Wiley Online Library for rules of use; OA articles are governed by the applicable Creative Commons License
Chapter 44 Acute Hyperglycemia 193

knowledge of previous weight or using the percentage of Initial serum sodium may appear low because hypergly-
dehydration along with the child’s weight. A fluid deficit cemia causes osmosis of water into the extracellular space
of 10–15% of body weight is usually present in patients leading to dilutional hyponatremia. Corrected sodium
with DKA and should be replaced along with mainte- values can be calculated to estimate the expected
nance intravenous fluids over 24–48 hours (subtract sodium if hyperglycemia was absent. After insulin and
fluid boluses). Volume expansion begins with isotonic fluids are administered, the serum sodium should gradu-
fluids 20 mL/kg. Normal saline or 0.45% fluids with ally increase. Total body potassium deficits are often pres-
potassium (with chloride, acetate, or phosphate) are ent from acidosis, increased plasma osmolality, and
preferred for maintenance fluids. There are varied vomiting. The first potassium level may appear normal,
recommendations in the literature for replacement intra- but insulin administration and acidosis correction moves
venous fluids, but the overall goal is to provide glucose potassium intracellularly, resulting in decreased serum
and electrolytes, especially to replace potassium which potassium. Blood urea nitrogen may appear high from
moves intracellularly as insulin treatment is initiated. dehydration. Hypophosphatemia, hypomagnesemia, and
Similarly, phosphorus also moves intracellularly with hypocalcemia are also complications of DKA and should
insulin treatment, but there is not as much concern as be monitored closely. Hemoglobin A1c levels are usually
replacement of potassium. Using fluids with higher obtained to provide information regarding the duration
concentrations of sodium chloride, as normal saline, of hyperglycemia.
can assist in preventing cerebral edema. A 3% saline
can also be used in the treatment of DKA when a patient
has neurologic complications such as seizures, related to If the patient develops a headache during the
cerebral edema. A continuous intravenous infusion of treatment, what is the most important treatment?
insulin 0.1 unit/kg per hour should be started as soon
A new headache during treatment or a worsening headache
as possible along with intravenous fluids. Fluids and
is concerning for cerebral edema. Other findings include
insulin reverse acidosis by allowing metabolism of
inappropriate slowing of heart rate or rising blood pressure,
ketoacids, although replacement of insulin is most
vomiting recurrence, and change in mental status (irritabil-
important. Bicarbonate administration is not routinely
ity, increased drowsiness). If cerebral edema is suspected,
recommended except in cases of extreme hyperkalemia
immediately administer 3% saline, or intravenous mannitol
or pH less than 6.85. A large systematic review indicated
0.5–1 g/kg over 10–15 minutes. Prevent hypotension to
that sodium bicarbonate use in patients with DKA
maintain cerebral perfusion pressure, but avoid excessive
increases the risk of cerebral edema and prolongs acido-
fluid administration that might lead to worsening cerebral
sis and overall treatment (Chua et al., 2011).
edema.
An aggressive decrease in plasma glucose, greater than
90 mg/dL per hour, can lead to hypoglycemia and rapid
osmolality changes. When serum glucose is below
What indicators are used to transfer from
300 mg/dL or blood glucose falls more than 90 mg/dL
intravenous insulin to subcutaneous
per hour, dextrose (5–12.5%) is added to fluids. Insulin rates
administration?
may be decreased to 0.05 unit/kg per hour to prevent rapid
glucose decline, but treatment relies on insulin, so this Transitioning to subcutaneous insulin occurs when the
should not be a first choice. metabolic acidosis has improved with evidence of a normal-
ized anion gap. Venous pH (>7.3), normal anion gap, bicar-
bonate (>15 mmol/L), and BOHB (<1 mmol/L) are used to
Which monitoring parameters and laboratory
determine DKA resolution. Ketonuria often continues for
studies would you follow, how often, and why?
several hours after blood ketone levels have normalized
Vital signs and neurologic checks should be monitored so is not a good indicator of DKA recovery. The ideal time
hourly as well as POCT glucose. It is recommended that to convert to subcutaneous insulin is just before a meal.
serum electrolytes, or BMP and venous blood gases are A dose of basal subcutaneous insulin (long- or intermedi-
checked every 2 hours. A POCT glucose and if available a ate-acting) can be administered hours before intravenous
blood β-hydroxybutyrate (BOHB) concentration should insulin is stopped or at the time the patient typically
be measured immediately; a BOHB level of 3 mmol/L or receives it. Rapid-acting insulin, given for meal coverage,
more indicates serum ketosis. Frequent neurologic checks should be administered 15–30 minutes before stopping
are necessary to evaluate for signs of cerebral edema. Tach- the insulin infusion and just before eating. Endocrinology
ycardia usually resolves with rehydration. specialists can help guide this conversion.
Downloaded from https://onlinelibrary.wiley.com/doi/ by National Institute Of Standard, Wiley Online Library on [06/03/2024]. See the Terms and Conditions (https://onlinelibrary.wiley.com/terms-and-conditions) on Wiley Online Library for rules of use; OA articles are governed by the applicable Creative Commons License
194 Cases in Pediatric Acute Care

Reference
Chua, H.R., Schneider, A., and Bellomo, R. (2011). Annals of Intensive Care 1(1): 23. doi: 10.1186/2110-
Bicarbonate in diabetic ketoacidosis: a systematic review. 5820-1-23.

Further Reading
Mayer-Davis, E., Kahkoska, A., Jefferies, C., et al. (2018). diabetes: evaluation of various definitions. Diabetes Research
ISPAD Clinical Practice Consensus Guidelines 2018: and Clinical Practice 135: 76–84. doi: 10.1016/j.
definition, epidemiology, and classification of diabetes in diabres.2017.09.011.
children and adolescents. Pediatric Diabetes 19(Suppl 27): Wolfsdorf, J., Glaser, N., Agus, M., et al. (2018). ISPAD Clinical
7–19. doi: 10.1111/pedi.12773. Practice Consensus Guidelines 2018: diabetic ketoacidosis
Von Oettingen, J., Rhodes, E.T., and Wolfsdorf, J. (2018). and the hyperglycemic hyperosmolar state. Pediatric
Resolution of ketoacidosis in children with new onset Diabetes 19(Suppl 27): 155–176. doi: 10.1111/pedi.12701.
Downloaded from https://onlinelibrary.wiley.com/doi/ by National Institute Of Standard, Wiley Online Library on [06/03/2024]. See the Terms and Conditions (https://onlinelibrary.wiley.com/terms-and-conditions) on Wiley Online Library for rules of use; OA articles are governed by the applicable Creative Commons License
195

45

Infant with Progressive Lethargy and Poor Feeding


Kathleen P. Thompson1,2
1
Baylor College of Medicine, Houston, TX, USA
2
Pediatric Critical Care Medicine, Texas Children’s Hospital, Houston, TX, USA

A 4-month-old male presents to the emergency department Family History


(ED) with progressive lethargy and poor feeding over the
past several days. Father has hypertension that is well controlled with medi-
cation. One male paternal cousin died unexpectedly at
6 months of age of an unknown cause approximately two
decades ago. No other significant family medical history.
History of Present Illness

The parents report that over the last 2 days, they have had
to wake him from sleep so that he will eat. This morning, he Current Status
would not wake up and they noted that his breathing was
very shallow. He has also had rhinorrhea and cough at He is sleeping during the examination, although he easily
night over the past week. His parents report approximately awakens and cries in response to tactile stimulation. His
one to two episodes of emesis each day for the past month. vital signs are temperature 36.4 C (97.5 F), heart rate 180
There are no known sick contacts. bpm, blood pressure 90/50 mmHg, and respiratory rate
50 breaths per minute. His weight is 4.5 kg. Other examina-
tion findings are significant for a full fontanelle with dry
mucous membranes, and clear rhinorrhea. He is tachyp-
Past Medical History neic, but has comfortable work of breathing. He is tachycar-
dic with 2+ pulses and capillary refill time of 2–3 seconds.
The infant was born full-term and was initially healthy. He No organomegaly. He moves all extremities spontaneously
has been noted to have poor weight gain over the last and falls asleep easily at the end of the examination. Initial
month. At approximately 3 months of age, he transitioned laboratory studies are obtained in the ED. Results of basic
from expressed breast milk (EBM) to formula. Prior to the metabolic profile and arterial blood gas analysis are shown
start of formula, he would feed every 2–3 hours with in Tables 45.1 and 45.2. Complete blood count and chest
approximately 113–170 g of EBM. Since transitioning to for- radiograph are unremarkable.
mula, he has been feeding approximately every 4–6 hours, While in the ED, the patient develops apnea and requires
taking about 113 g per feed. He does not have any known intubation and mechanical ventilation. Transport to a ter-
allergies and is not on any medications. He immunizations tiary children’s hospital is arranged and he is accepted by
are up to date. the pediatric intensive care unit (PICU) for presumed men-
ingitis. While in the ED, he receives fluid resuscitation with
60 mL/kg of 0.9% normal saline for tachycardia and
dehydration. Blood, urine, and cerebrospinal fluid (CSF)
Past Surgical History cultures are obtained and empiric broad-spectrum antimi-
crobials comprising ceftriaxone 50 mg/kg every 12 hours
Circumcision on second day of life without complications. and vancomycin 15 mg/kg every 6 hours are initiated.

Cases in Pediatric Acute Care: Strengthening Clinical Decision Making, First Edition. Edited by Andrea M. Kline-Tilford and Catherine M. Haut.
© 2020 John Wiley & Sons Ltd. Published 2020 by John Wiley & Sons Ltd.
Downloaded from https://onlinelibrary.wiley.com/doi/ by National Institute Of Standard, Wiley Online Library on [06/03/2024]. See the Terms and Conditions (https://onlinelibrary.wiley.com/terms-and-conditions) on Wiley Online Library for rules of use; OA articles are governed by the applicable Creative Commons License
196 Cases in Pediatric Acute Care

Table 45.1 Basic metabolic profile. Table 45.4 Complete blood count and coagulation studies
in PICU.
Sodium 138 mEq/L
Potassium 4.0 mEq/L White blood cell count 12 100/mm3

Chloride 108 mEq/L Hemoglobin 9.1 g/dL

Carbon dioxide 20 mEq/L Hematocrit 28%

Glucose 115 mg/dL Platelet count 175 000/mm3


Differential Pending
Prothrombin time 13.0 s
Table 45.2 Arterial blood gas analysis. Partial thromboplastin time 34.5 s
International normalized ratio 1.3
pH 7.56
PaCO2 22 mmHg
PaO2 90 mmHg Table 45.5 Arterial blood gas analysis in PICU.
Bicarbonate 20 mmol/L
Lactate 2.7 mmol/L pH 7.40
PaCO2 35 mmHg
Pao2 93 mmHg
Bicarbonate 21 mmol/L
Table 45.3 Complete metabolic profile in PICU.
Lactate 1.5 mmol/L
Sodium 140 mEq/L
Potassium 4.3 mEq/L
Chloride 108 mEq/L
Table 45.6 Urinalysis in PICU.
Carbon dioxide 22 mEq/L
Blood urea nitrogen 15 mg/dL Color Yellow
Creatinine 0.5 mg/dL Clarity Clear
Glucose 115 mg/dL pH 5.0
Calcium 8.5 mg/dL Specific gravity 1.015
Magnesium 2.0 mg/dL Glucose Negative
Phosphorus 4.5 mg/dL Ketones Negative
Alanine aminotransferase 55 U/L Nitrites Negative
Aspartate aminotransferase 60 U/L Leukocyte esterase Negative
Alkaline phosphatase 160 U/L Bilirubin Negative
Gamma-glutamyltransferase 50 U/L
Albumin 3.0 g/dL
Conjugated bilirubin <0.35 mg/dL Repeat laboratory results in the PICU are shown in
Unconjugated bilirubin <1.0 mg/dL Tables 45.3–45.6. CSF analysis demonstrates no cells, and
normal protein and glucose. Remaining CSF Gram stain
Ammonia 500 μg/dL
and culture are pending.

In transport, he is placed on intravenous fluids comprising


20 mEq/L KCl in 10% dextrose and 0.9% normal saline at
Questions
1600 mL/m2 daily.
Answer the following questions using the details provided.
On arrival at the PICU, he has a full bulging fontanelle
with minimal movement of his extremities with tactile 1 Based on the data given, what is the differential
stimulation. Sedation is not required for his invasive proce- diagnosis?
dures or diagnostic studies. Non-contrast computed tomog- 2 Describe the pathophysiology that causes acute crises
raphy (CT) of the brain is obtained after intubation at the secondary to inborn error of metabolism (IEM) to pres-
referring hospital and was unremarkable. ent later than the newborn period.
Downloaded from https://onlinelibrary.wiley.com/doi/ by National Institute Of Standard, Wiley Online Library on [06/03/2024]. See the Terms and Conditions (https://onlinelibrary.wiley.com/terms-and-conditions) on Wiley Online Library for rules of use; OA articles are governed by the applicable Creative Commons License
Chapter 45 Infant with Progressive Lethargy and Poor Feeding 197

3 What are some common presenting signs and symp- What are some common presenting signs and
toms of IEM listed by system? symptoms of IEM listed by system?


4 Describe your diagnostic evaluation of a patient with
Constitutional: dysmorphic features; failure to thrive;
suspicion of IEM. What is your initial approach? What
unusual odor of urine, breath, cerumen, or other
additional studies may be warranted as you continue to
body fluid.


suspect this diagnosis?
Respiratory: tachypnea without pulmonary pathol-
5 What are some causes of anion gap metabolic acidosis?
ogy, apnea.


How is it calculated?
Cardiac: congestive heart failure, cardiomegaly, ischemic
6 Based on the initial laboratory screening, what three
heart disease.


major diagnostic categories may be identified in chil-
Neurologic: hypotonia, lethargy, encephalopathy, sei-
dren presenting with acute encephalopathy or acute
zures, inconsolability, change in mental status, unex-
metabolic emergencies due to an IEM?
plained developmental delay or regression of
7 What is your initial management of the patient with
developmental milestones.


suspected acute metabolic encephalopathy due to
Gastrointestinal: jaundice, diarrhea, vomiting, abdomi-
an IEM?
nal pain.

Acute presentation with multisystem involvement is typical


and strongly suggestive of a possible IEM. The onset and
Rationale and Evidence-based Practice progression of symptomology and severity of presentation
Explanation are all important clues that point to an acute metabolic
and encephalopathic IEM.
Based on the data given, what is the differential
diagnosis?
Describe your diagnostic evaluation of a patient
A number of diagnoses may be contributing to this patient’s with suspicion of IEM. What is your initial
presentation and acute deterioration. Some differential approach? What additional studies may be
diagnoses to consider include sepsis, central nervous sys- warranted as you continue to suspect this
tem infection, acute liver failure, toxic ingestion, IEM with diagnosis?
hyperammonemia, or child maltreatment.
The initial evaluation of a child with suspected IEM should
Although it is difficult to differentiate between these
include laboratory evaluation of glucose, serum electro-
diagnoses, it is important to include IEM on the differential
lytes, complete blood count with differential, lactate, pH,
diagnosis any critically ill neonate or child with abnormal
ammonia, liver function, clotting studies, venous blood
unexplained laboratory results and unknown etiology of ill-
gas, and urinalysis. As these patients often present with
ness. The clinical suspicion for a possible IEM should be
vague symptoms that may be consistent with overwhelm-
heightened if there is a family history of death during
ing illness, an infectious work-up should also be completed
infancy, as noted in this case.
in a timely manner. The infectious evaluation is also impor-
tant given increased risk for infection in an IEM and the
potential of infection to cause increased catabolism and
Describe the pathophysiology that causes acute
emergent presentation.
crises secondary to IEM to present later than the
If the initial screening reveals an elevated ammonia,
newborn period
anion gap metabolic acidosis, or hypoglycemia, consulta-
Children with inherited metabolic disorders almost always tion with genetics is warranted and further confirmatory
appear normal at birth. This is because the small metabolic studies should be sent with their collaboration. In some
intermediate that is responsible for the disorder is easily fil- cases, initial screening results are unremarkable due to
tered by the placenta and the maternal metabolism acts as the interventions that are performed during resuscitation.
an effective dialyzer of fetal metabolic byproducts. In these cases, additional laboratory studies may be war-
A metabolic crisis occurs when there is a build-up of toxic ranted to further evaluate the possible diagnosis of IEM.
metabolites. An acute metabolic decompensation is usually Finally, IEM should continue to be evaluated in the setting
triggered by factors that cause increased catabolism or of a patient who fails to respond to standard therapy. Addi-
increased consumption of protein. The duration of the tional diagnostic studies may include plasma and urine
symptom-free period and timing of presentation may be amino acids, urine organic acids, lactate, pyruvate, plasma
anywhere from hours to months, or possibly even years. acylcarnitine profile, and carnitine analysis.
Downloaded from https://onlinelibrary.wiley.com/doi/ by National Institute Of Standard, Wiley Online Library on [06/03/2024]. See the Terms and Conditions (https://onlinelibrary.wiley.com/terms-and-conditions) on Wiley Online Library for rules of use; OA articles are governed by the applicable Creative Commons License
198 Cases in Pediatric Acute Care

What are some causes of anion gap metabolic sequalae. Life-saving treatment is often initiated prior to con-
acidosis? How is it calculated? firmation of the diagnosis, and largely consists of supportive
care standard in the PICU. Ventilatory support should be
In the setting of metabolic acidosis, the anion gap should be
initiated and managed according to the etiology of the
calculated as follows:
respiratory failure; current recommendations for acute
Anion gap = Na + + K + – Cl – + HCO3 – respiratory distress syndrome (ARDS), cerebral edema,
Normal anion gap is 8–16 mEq/L. There are several etiol- and other causes should be considered. Fluid resuscitation
ogies that may cause an anion gap metabolic acidosis. An with 0.9% sodium chloride should be administered to main-
acronym to aid in the evaluation of anion gap acidosis is tain adequate perfusion and circulation; because of the risk
MUDPILES: methanol, uremia, diabetic ketoacidosis, par- of increased lactic acidosis, lactated Ringer’s solution should
aldehyde, intoxication, lactic acidosis, ethylene glycol, and be avoided, and hypotonic fluids are discouraged due to the
salicylates. risk of cerebral edema. Management of cerebral edema and
neuroprotective measures should follow evidence-based
Based on the initial laboratory screening, what guidelines.
three major diagnostic categories may be Hypoglycemia should be corrected immediately. If possi-
identified in children presenting with acute ble, it is preferable to obtain blood samples prior to initia-
encephalopathy or acute metabolic emergencies tion of therapy. Acidosis may be corrected with careful
due to an IEM? administration of bicarbonate, but this must be done cau-
tiously to avoid the risk of fluid shifts and cerebral edema.
Three major diagnostic categories include hyperammone-
Hyperammonemia may be addressed with emergent initia-
mia, anion gap metabolic acidosis, and hypoglycemia.
tion of hemodialysis. Additional interventions to reduce
One of the most important laboratory findings when an
ammonia in the absence of metabolic acidosis may include
IEM is suspected is hyperammonemia; levels of ammonia
intravenous dextrose and lipid emulsion to provide protein-
greater than 200 μmol/L should heighten concern for an
free energy, arginine hydrochloride with sodium phenyla-
IEM. The second most important laboratory feature is met-
cetate, and sodium benzoate. Antimicrobial therapy should
abolic acidosis with an increased anion gap; this is a prom-
be initiated due to the potential for concurrent serious
inent finding in glycogen storage diseases, pyruvate
infection.
metabolism defects, and organic acid disorders, among
Protein intake should be discontinued immediately if a
others. The presence of hypoglycemia can assist in differen-
urea cycle defect or protein intolerance is suspected in
tiation of metabolic disorders, especially when considered
the setting of organic academia. Catabolism can be pre-
with other findings such as hepatomegaly, acidosis, and
vented through liberal administration of intravenous dex-
the presence or absence of ketosis.
trose, sometimes requiring concurrent administration of
insulin. Intravenous intralipid may also be administered
What is your initial management of the patient to meet caloric requirements during physiologic stress.
with suspected acute metabolic encephalopathy
With the support of genetics consultation, cofactors may
due to an IEM?
be initiated while awaiting confirmation of a diagnosis.
Patients with an IEM require immediate management in an This presumptive cocktail may include vitamin B12, thia-
effort to prevent further deterioration and reduce neurologic mine, biotin, riboflavin, folic acid, and carnitine.

Further Reading
MacNeill, E.C. and Walker, C.P. (2018). Inborn errors of 20th edn (ed. R.M. Kliegman, B.F. Stanton, J.W. St Geme,
metabolism in the emergency department (undiagnosed and N.F. Schor and R.E. Behrman), 634–636. Philadelphia, PA:
management of the known). Emergency Medicine Clinics 36 Elsevier.
(2): 369–385. Rice, G.M. and Steiner, R.D. (2016). Inborn errors of
Rezvani, I. and Rezvani, G.A. (2016). An approach to inborn metabolism (metabolic disorders). Pediatrics in Review 37(1):
errors of metabolism. In: Nelson’s Textbook of Pediatrics, 3–15.
Downloaded from https://onlinelibrary.wiley.com/doi/ by National Institute Of Standard, Wiley Online Library on [06/03/2024]. See the Terms and Conditions (https://onlinelibrary.wiley.com/terms-and-conditions) on Wiley Online Library for rules of use; OA articles are governed by the applicable Creative Commons License
199

46

Neonatal Tachypnea
Janet Dutcher
Nemours/Alfred I. duPont Hospital for Children, Wilmington, DE, USA

A newborn male is admitted to the neonatal intensive care pregnancy. Fetal ultrasonography at 20 weeks revealed
unit (NICU) at 2 hours of life for respiratory distress. normal anatomy.

History of Present Illness


Family History
Baby was born at 39 weeks’ gestation to a 30-year-old grav-
Four-year-old sister has asthma. There is no family history
ida 2 woman via planned, repeat cesarean section. The
of genetic or cardiac disorders. Both parents are healthy.
delivery was uneventful, and the infant cried immediately
after birth. He received no advanced resuscitation and was
assigned Apgar scores of 8 and 9 at 1 and 5 minutes of life,
respectively. The newborn was noted to have intermittent Current Status
grunting in the delivery room. Pulse oximetry was
90–92%, so he went to recovery with his mother where On admission to the NICU the newborn is placed on an
he continued to receive routine care and monitoring. How- open radiant warmer and attached to a cardiopulmonary
ever, at approximately 2 hours of life the pediatric team was monitor and pulse oximeter. Pertinent findings on physical
called to evaluate for signs of increasing respiratory examination include mildly diminished breath sounds
distress, including tachypnea (respiratory rate 84 breaths bilaterally, mild intermittent subcostal retractions, tachyp-
per minute), retractions, and supplemental oxygen require- nea, no murmur, and mildly decreased tone. Temperature,
ment to maintain oxygen saturations (SpO2) greater than heart rate, and four-extremity blood pressures are all nor-
90%. At that time the newborn was moved to the NICU mal for gestational age. He is treated with humidified
for evaluation and admission. high-flow nasal cannula at 2 L/min FiO2 30%, targeting oxy-
gen saturation of 94–96%. At 4 hours of life, pre-ductal
(right hand) and post-ductal (left foot) were 96% and
Past Medical History 92%, respectively.
An arterial blood gas analysis, complete blood count
Prenatal laboratory results are as follows: blood type A+, (CBC), blood culture, and blood glucose are completed
antibody negative, rubella immune, syphilis negative, hep- via radial arterial stick. Empirical antibiotics are ordered
atitis B negative, HIV negative, gonorrhea negative, chla- including ampicillin 100 mg/kg per dose every 12 hours
mydia negative, group B β-hemolytic Streptococcus (GBS) and gentamicin 4 mg/kg per dose every 24 hours.
untested. Glucose tolerance test was abnormal at 1 hour, A supine anteroposterior chest radiograph is obtained with
but normal at 3 hours. Mother has a history of previous available reading reporting increased lung volumes with
cesarean section and asthma with occasional albuterol neb- flat diaphragm, increased vascular markings (perihilar
ulizer use; however, the antenatal period during this preg- streaking), and fluid in the horizontal fissure.
nancy was otherwise uncomplicated. There was no Results for CBC and arterial blood gas analysis are shown
apparent exposure to tobacco, alcohol, or drugs during in Tables 46.1 and 46.2 and the chest radiograph in

Cases in Pediatric Acute Care: Strengthening Clinical Decision Making, First Edition. Edited by Andrea M. Kline-Tilford and Catherine M. Haut.
© 2020 John Wiley & Sons Ltd. Published 2020 by John Wiley & Sons Ltd.
Downloaded from https://onlinelibrary.wiley.com/doi/ by National Institute Of Standard, Wiley Online Library on [06/03/2024]. See the Terms and Conditions (https://onlinelibrary.wiley.com/terms-and-conditions) on Wiley Online Library for rules of use; OA articles are governed by the applicable Creative Commons License
200 Cases in Pediatric Acute Care

Table 46.1 Complete blood count. 2 What results would you expect from a hyper-oxygen test
with this disorder? Would you consider doing any other
White blood cell count 9800/mm3
screening tests or diagnostics on the newborn? If yes,
Hemoglobin 17.9 g/dL what would you include?
Hematocrit 49.5% 3 What would be your management goals for the care of
Platelet count 241 000/mm3 this infant?
Neutrophils 56% 4 Are antibiotics indicated for this patient?
Lymphocytes 33% 5 Which of the following strategies, if any, have been
Monocytes 9%
shown to help in the management of the patient’s diag-
nosis: diuretic therapy, fluid restriction, and/or inhaled
salbutamol therapy?
6 While obtaining consent for NICU admission and
Table 46.2 Arterial blood gas analysis. updating the family on the newborn’s status and plan
of care, the mother wants to know what “causes” the
pH 7.30
condition and how long you expect it to last. How
PCO2 38 mmHg
would you respond?
PaO2 75 mmHg
Bicarbonate 22 mEq/L
Base excess –5 Rationale and Evidence-based
Practice Explanation
Based on the data provided, what is the most likely
diagnosis and differential diagnosis for this
newborn? What are the perinatal risk factors for
this diagnosis?
After ruling out all other pathologic causes of respiratory
distress, transient tachypnea of the newborn (TTN) would
be the most likely diagnosis for this infant. Since confirm-
atory diagnosis of TTN comes after symptoms are no longer
present, other conditions presenting with respiratory
distress in a term newborn shortly after birth would
include:

•• respiratory distress syndrome (RDS)


early-onset sepsis

•• congenital pneumonia
meconium aspiration syndrome (MAS)

Figure 46.1 Chest radiograph. Source: Courtesy of


•• congenital heart disease (CHD)
persistent pulmonary hypertension of the new-
born (PPHN)

••
D. Dirnberger, MD.
congenital metabolic acidosis
Figure 46.1. Other results show blood glucose 59 mg/dL, pneumothorax.
while blood culture is pending.
TTN is the result of residual fluid in a newborn’s lungs and
is the most common cause of respiratory distress in the
immediate newborn period. The maternal and birth history,
Questions clinical presentation, physical examination, and chest radi-
ograph findings for the newborn support the diagnosis of
Answer the following questions using the details provided.
TTN. Delivery by cesarean section, male sex, and family his-
1 Based on the data provided, what is the most likely diag- tory of asthma are perinatal risk factors for TTN. The deliv-
nosis and differential diagnosis for this newborn? What ery was uncomplicated with no noted intrauterine distress
are the perinatal risk factors for this diagnosis? or perinatal asphyxia, clear amniotic fluid, and blood gas
Downloaded from https://onlinelibrary.wiley.com/doi/ by National Institute Of Standard, Wiley Online Library on [06/03/2024]. See the Terms and Conditions (https://onlinelibrary.wiley.com/terms-and-conditions) on Wiley Online Library for rules of use; OA articles are governed by the applicable Creative Commons License
Chapter 46 Neonatal Tachypnea 201

analysis with no evidence of significant acidosis (which is 100% oxygen for 10 minutes. If the cause of the cyanosis is
more common in conditions such as MAS, RDS, and PPHN). primarily pulmonary in nature, the PaO2 will increase to
Since the patient was delivered close to term, he was also at greater than 150 mmHg after 10 minutes on 100% oxygen.
less risk for RDS and congenital metabolic disease. This will not occur in cardiac disease of a shunt physiology
Infants with TTN may have cyanosis with a supplemental (right-to-left shunt).
oxygen requirement, but they usually do not require FiO2 Hyperoxia test could be useful for differentiating between
greater than 0.4. They generally have blood gases that do pulmonary and cardiac causes of cyanosis, and in the case
not reflect carbon dioxide retention, and rarely require of TTN would show improvement in oxygen saturation
advanced respiratory support (i.e. intubation and mechan- with the administration of 100% oxygen.
ical ventilation); such intervention is more common with A C-reactive protein may be helpful if there is a concern
other causes of neonatal respiratory distress such as RDS, for bacterial illness. An echocardiogram would confirm or
MAS, and PPHN. exclude CHD or PPHN. Repeat chest X-ray may be useful if
While neonates with CHD can present with cyanosis and clinical history suggests pneumonia, respiratory status wor-
tachypnea, there is no family history of CHD and there were sens, or for questionable diagnoses.
no other clinical symptoms (i.e. murmur, decreased or absent
pulses, poor perfusion, active precordium, acidosis) to suggest What would be your management goals
a diagnosis of CHD. In addition, the baby’s oxygen saturation for the care of this infant?
response to the administration of oxygen and the pre-ductal
Since TTN is a self-limited disorder, the goal of therapy is
and post-ductal SpO2 values less than 10% decrease the likeli-
supportive and includes the following:
hood of CHD or PPHN as a contributing diagnosis.
The infant’s chest radiograph showed the classic patho-
physiologic findings of TTN, including normal to slightly
• Respiratory support, including supplemental oxygen via
hood, nasal cannula, or continuous positive airway pres-
hyperinflated lung volumes, mildly hazy diffuse alveolar sure (CPAP), to provide adequate (>90%) arterial oxygen
appearance, and central radiating linear opacities demon- saturation.
strating excess fluid being carried out via the pulmonary
vasculature. The early radiographs in TTN can appear sim-
• Maintaining a neutral thermal environment (keeping
patient normothermic).
ilar to other neonatal respiratory disorders, particularly
congenital pneumonia, which typically takes longer to
• Fluid and nutrition support with intravenous fluids and/
or gavage feedings until the respiratory rate is low
clear. Radiographically, RDS will typically show hypoinfla- enough to allow oral feedings.
tion and a ground-glass appearance, as opposed to the
hyperinflation generally noted with TTN. The patient’s
• Family support.

It is important to monitor for signs of respiratory fatigue


chest X-ray also excluded pneumothorax as a diagnosis.
or clinical deterioration, which may suggest an alternative
Despite an unknown maternal GBS status at birth, rup-
diagnosis. Infants with TTN independent of other comor-
ture of membranes was at the time of delivery and there
bidities rarely have oxygen requirements greater than
were no other perinatal risk factors for infection, including
40% and invasive mechanical ventilation is usually not
prolonged rupture of membranes, complicated labor, and
required.
chorioamnionitis (e.g. maternal fever, elevated maternal
white cell count, purulent amniotic fluid, uterine tender-
Are antibiotics indicated for this patient?
ness, and fetal tachycardia). Likewise, the infant’s CBC
showed normal immature to total neutrophil, white cell TTN alone is not an indication for antibiotic therapy. How-
and platelet counts. The baby had no symptoms such as ever, owing to the difficulty of distinguishing TTN from
lethargy, hypothermia, apnea, and respiratory failure that congenital pneumonia at presentation, many newborns
may accompany neonatal infection. with TTN receive empiric antibiotics (i.e. ampicillin, which
covers GBS and Listeria, in combination with gentamicin
for Gram-negative coverage) for a minimum of 48 hours
What results would you expect from a pending blood culture results.
hyper-oxygen test with this disorder? Would you
There is evidence to suggest that empiric antibiotics may
consider doing any other screening tests or
be avoidable with the use of strict criteria for TTN and dil-
diagnostics? If yes, what would you include?
igent patient monitoring (Salama et al., 2013). However,
Hyperoxia test is performed by measuring the arterial blood these authors recommend further studies employing rando-
gases of the patient while he or she breathes room air, then mized controlled trials to confirm the feasibility and safety
remeasuring the blood gases after the patient has breathed of this approach.
Downloaded from https://onlinelibrary.wiley.com/doi/ by National Institute Of Standard, Wiley Online Library on [06/03/2024]. See the Terms and Conditions (https://onlinelibrary.wiley.com/terms-and-conditions) on Wiley Online Library for rules of use; OA articles are governed by the applicable Creative Commons License
202 Cases in Pediatric Acute Care

Which of the following strategies, if any, have Case Resolution


been shown to help in the management of the
patient’s diagnosis: diuretic therapy, fluid Over the course of several hours, the neonate was weaned
restriction, and/or inhaled salbutamol therapy? off oxygen and the nasal cannula discontinue by 12 hours of
A Cochrane review finds no current evidence to support life. He became stable in room air, with oxygen saturations
diuretic therapy in the management of TTN. There are greater than 95%, and tachypnea resolved by 32 hours of
limited data showing that infants with severe TTN might life. The antibiotics (ampicillin and gentamicin) that had
benefit from fluid restriction, and there is some support been started due to clinical symptoms and oxygen require-
for the use of salbutamol therapy with TTN. However, ment were discontinued with the discovery of negative
current data remain inconclusive with a need for larger- blood cultures at 48 hours.
scale studies. Therefore, none of these therapies is currently Breastfeeding was initiated at 36 hours of life, when
recommended as standard therapy. respiratory rate was less than 65 breaths per minute
and intravenous fluids were weaned as he tolerated
While obtaining consent for NICU admission and breastfeeding and was feeding adequately. His clinical
updating the family on the newborn’s status and state continued to remain stable, and he was transferred
plan of care, the mother wants to know what to couplet care with his mother at approximately
“causes” the condition and how long you expect it 48 hours of life.
to last. How would you respond?
TTN typically carries a good prognosis, and symptoms
generally resolve within 24–48 hours and rarely go beyond
72 hours.

Reference
Salama, H., Abughalwa, M., Taha, S., Sharaf, N., and Mansour, antimicrobial therapy needed? Journal of Neonatal Perinatal
A. (2013). Transient tachypnea of the newborn: is empiric Medicine 6(3): 237–241. doi: 10.3233/NPM-1367012.

Further Reading
Hagen, E., Chu, A., and Lew, C. (2017). Transient tachypnea of Database of Systematic Reviews (11): CD003064. doi:
the newborn. NeoReviews 18(3): e141–e149. 10.1002/14651858.CD003064.pub3.
Kassab, M., Khriesat, W.M., and Anabrees, J. (2015). Diuretics
for transient tachypnoea of the newborn. Cochrane
Downloaded from https://onlinelibrary.wiley.com/doi/ by National Institute Of Standard, Wiley Online Library on [06/03/2024]. See the Terms and Conditions (https://onlinelibrary.wiley.com/terms-and-conditions) on Wiley Online Library for rules of use; OA articles are governed by the applicable Creative Commons License
203

47

Diplopia and Esotropia


Lindsay Chapman
St. Jude Children’s Research Hospital, Memphis, TN, USA

An 8-year-old male with no significant past medical history He has achieved his developmental milestones appropri-
is evaluated for new complaints of diplopia and esotropia ately. He ambulated at 12 months. He was a toe walker so
following a 2-day history of low-grade fever and upper res- had physical therapy starting at age 4 years and had casts
piratory infection. applied to both of his feet. No other developmental
concerns.

History of Present Illness


Past Surgical History
An 8-year-old right-handed male was in his normal state of
health until 2.5 weeks prior to presentation. Upon return- No past surgeries.
ing from a vacation 2.5 weeks ago, he reported diplopia and
stated he did not feel well. He developed a fever of 38.8 C
(101.8 F) that persisted for approximately 2 days that was
associated with upper respiratory symptoms and mild Family History
headache. He received acetaminophen and ibuprofen dur-
ing this time. His maternal great grandmother was a previous smoker
Parents noted left esotropia after resolution of illness, and and was diagnosed with lung cancer in her seventies.
the child told his teacher about his diplopia. He presented Another maternal great grandmother was diagnosed with
to his primary care provider and his evaluation was positive stomach cancer in her late eighties. No family history of
for sinus drainage; he was prescribed cefdinir for sinusitis. childhood cancer, neurologic disorders, or ophthalmologic
An ophthalmologic evaluation confirmed the presence of diseases.
left eye esotropia with normal visual acuity. As part of
the evaluation for esotropia, magnetic resonance imaging
(MRI) of the brain was performed, which demonstrated Current Status
the presence of an expansile cystic and solid lesion in the
right side of the pons measuring 2.4 × 3.4 × 2.7 cm with He is admitted to the pediatric intensive care unit (PICU).
nodular enhancement. He was then referred for neurosur- On physical examination, his vital signs are unremarka-
gical evaluation. ble: temperature 37.2 C (99.0 F), heart rate 83 bpm, respi-
ratory rate 20 breaths per minute, blood pressure 115/
56 mmHg, and oxygen saturation 100% on room air. He
Past Medical History is a well-developed, well-nourished male who is active
and cooperative. His neurologic examination is remarka-
No significant past medical history. He is up to date on his ble for an adduction deficit in the right eye and in the left
immunizations, including annual influenza vaccine. He is eye. He has horizontal nystagmus on lateral gazes bilater-
not on any medication and does not have any food or ally, and esotropia. His strength and reflexes were intact.
environmental allergies. The remainder of the examination is unremarkable.

Cases in Pediatric Acute Care: Strengthening Clinical Decision Making, First Edition. Edited by Andrea M. Kline-Tilford and Catherine M. Haut.
© 2020 John Wiley & Sons Ltd. Published 2020 by John Wiley & Sons Ltd.
Downloaded from https://onlinelibrary.wiley.com/doi/ by National Institute Of Standard, Wiley Online Library on [06/03/2024]. See the Terms and Conditions (https://onlinelibrary.wiley.com/terms-and-conditions) on Wiley Online Library for rules of use; OA articles are governed by the applicable Creative Commons License
204 Cases in Pediatric Acute Care

Table 47.1 Basic metabolic profile. 4 What is the best imaging modality to identify lesions in
the posterior fossa?
Sodium 141 mEq/L
Potassium 4.1 mEq/L
Chloride 103 mEq/L
Rationale and Evidence-based Practice
Carbon dioxide 27 mEq/L
Explanation
Blood urea nitrogen 19 mg/dL
Creatinine 0.3 mg/dL
Based on these data, what are the differential
Glucose 74 mg/dL diagnoses for this child?
Central nervous system (CNS) tumors are the most com-
mon solid tumor of childhood, and most of these tumors
Table 47.2 Complete blood count. occur in the infratentorium. The differential diagnosis of
infratentorial tumors may include medulloblastoma, epen-
White blood cell count 7200/mm3 dymoma, atypical teratoid rhabdoid tumor, pilocytic astro-
Hemoglobin 12.6 g/dL cytoma, and brainstem glioma that includes the diffuse
Hematocrit 37.1% midline glioma H3 K27M-mutant. Brainstem gliomas
Platelet count 316 000/mm3 account for 15–20% of all childhood CNS tumors, and dif-
Neutrophils 43.3% fuse midline gliomas may account for up to 85% of these
Eosinophils 1.4% tumors. Diffuse midline glioma with histone H3 K27M
mutation is a new diagnostic entity, previously known as
Monocytes 8.4%
a diffuse intrinsic pontine glioma. This new classification
Lymphocytes 46.1%
is not limited to the pons but may be found in supratentor-
Bands 0%
ial midline structures of the brain.
Diffuse midline gliomas are high-grade tumors classified
as WHO grade IV, and typically children have quick onset
of symptoms. The most common cranial nerve deficit asso-
A basic metabolic profile and complete blood count are
ciated with this type of tumor is an abducens nerve palsy,
obtained on arrival at the PICU (results shown in
the second most common being facial nerve palsy. Abdu-
Tables 47.1 and 47.2). He receives one dose of intravenous
cens nerve palsy is dysfunction of cranial nerve VI which
dexamethasone 4 mg followed by dexamethasone 2 mg
is responsible for causing contraction of the lateral rectus
every 8 hours and famotidine 10 mg by mouth twice daily
muscle that results in abduction of the eye. Therefore, a
for gastrointestinal protection. He undergoes repeated
patient with an abducens nerve palsy will have a difficult
sedated imaging the following day at which time the previ-
time turning the eye laterally, and this may result in an
ous findings are verified (see Figures 47.1 and 47.2).
esotropia. A facial nerve palsy is dysfunction of cranial
The brain MRI demonstrates a heterogeneous mass cen-
nerve VII that innervates the facial muscles. On examina-
tered in the pons measuring 2.4 × 3.4 × 2.7 cm with two
tion, a patient with a facial nerve palsy will exhibit facial
dominant components. Associated mass effect effaces the
weakness (e.g. drooping of the face that may involve a
prepontine cistern and partially effaces the fourth ventricle.
decrease in the prominence of the nasolabial fold, an ina-
There is no hydrocephalus. The spine MRI demonstrates no
bility to close the eyelid completely and an inability to
evidence of disease.
raise the eyebrows).

Why was he started on dexamethasone?


Questions
Patients with a diagnosis of H3 K27M diffuse midline glio-
mas may exhibit a variety of neurologic deficits that include
Answer the following questions using the details provided.
cranial nerve deficits, hemiparesis, and ataxia in addition to
1 Based on these data, what are the differential diagnoses signs and symptoms of increased intracranial pressure
for this child? (ICP), hallmark features that include headache, nausea,
2 Why was he started on dexamethasone? emesis, and lethargy. Dexamethasone is administered on
3 What are the signs and symptoms of increased intracra- diagnosis to reduce peritumoral edema and associated neu-
nial pressure? rologic deficits. Symptoms may improve with use of
Downloaded from https://onlinelibrary.wiley.com/doi/ by National Institute Of Standard, Wiley Online Library on [06/03/2024]. See the Terms and Conditions (https://onlinelibrary.wiley.com/terms-and-conditions) on Wiley Online Library for rules of use; OA articles are governed by the applicable Creative Commons License
Chapter 47 Diplopia and Esotropia 205

Figure 47.1 Brain MRI.

Figure 47.2 Brain MRI.

dexamethasone until the tumor responds to irradiation or determine presence of hemorrhage, and evaluate gray/
other indicated therapy. white matter.

What is the best imaging modality to identify


What are signs and symptoms of increased
lesions in the posterior fossa?
intracranial pressure?
MRI provides slice images of the brain and better resolution
Increased ICP is an increase in pressure in the CNS that
of different structures of the brain than head CT.
may occur due to cranial trauma, tumor, hypoxic and ische-
A radiologist can see structures in the posterior fossa and
mic states, encephalitis, meningitis, or cerebral hemor-
cervicomedullary junction much better than with CT, as
rhage and infarction. The clinical manifestations of
MRI provides views in three planes without interfering
increased ICP in children and adults may include head-
bony artifact.
ache, nausea, projectile emesis, lethargy, vision changes,
papilledema, and ocular palsies. Clinical manifestations Continuation of case: The patient undergoes a right sub-
in infants with increased ICP may include a bulging fonta- occipital transcerebellar burr hole and stereotactic brain-
nelle and increased head circumference, as their cranial stem biopsy that he tolerated well; he was discharged on
sutures may not have fused. It is imperative to recognize postoperative day 2. After surgery, his examination was sig-
and treat increased ICP in a timely manner. If not treated nificant for a right facial palsy, dysconjugate gaze, left lower
appropriately, cerebral damage or death may occur. Cush- extremity weakness, and unsteady gait that was improving.
ing’s triad, an ominous sign indicative of impending brain- His fatigue was improving daily. Pathology revealed a dif-
stem herniation, is characterized by irregular respirations, fuse midline glioma, H3 K27M-mutant, World Health
bradycardia, and hypertension. Non-contrast computed Organization (WHO) grade IV. He and his parents met with
tomography (CT) can be used to diagnose cerebral edema, the oncologist and discussed diagnosis, prognosis, and
evaluate ventricle size (e.g. presence of hydrocephalus as a treatment options including irradiation and chemotherapy
component of increased ICP), evaluate mass lesions, on clinical trials.
Downloaded from https://onlinelibrary.wiley.com/doi/ by National Institute Of Standard, Wiley Online Library on [06/03/2024]. See the Terms and Conditions (https://onlinelibrary.wiley.com/terms-and-conditions) on Wiley Online Library for rules of use; OA articles are governed by the applicable Creative Commons License
206 Cases in Pediatric Acute Care

5 Based on this additional information, what is your man- Molecular analysis of the biopsy sample also helps detect
agement plan? the presence of genetic alterations in the tumor and direct
the use of molecularly targeted therapies. Several such
agents are currently undergoing clinical trials. However,
Based on this additional information, what is your
biopsy does not come without risk. Irradiation continues
management plan?
to be the gold standard of therapy, and there are research
Diffuse midline gliomas are difficult to treat and are the efforts to develop and explore targeted therapies to over-
most challenging of all brainstem gliomas due to location come the identified mutations gathered through tissue
and their diffuse infiltrative nature. Children with this diag- biopsies in the hope of generating effective treatments for
nosis have a poor prognosis. Median survival is approxi- this type of tumor. Based on the life-threatening nature
mately 10 months, and the 2-year survival rate is less of this tumor, consultation to palliative care services or inte-
than 10%. Radiation therapy is the gold standard of treat- gration of palliative care concepts on diagnosis is also
ment, but provides only temporary relief of symptoms recommended.
and is primarily palliative in nature. The patient typically
receives irradiation daily for about 6 weeks. This is not a
curative treatment, but the hope is that with irradiation
the child’s neurologic symptoms may improve for a short Case Resolution
time. Studies have shown that children who receive con-
ventional chemotherapy do not have better outcomes than The patient and his parents made the decision to enroll him
those who receive irradiation. More recently, some children on a clinical trial that incorporated radiation therapy fol-
with this diagnosis are undergoing biopsies of their tumor lowed by use of a PI3 kinase/mTOR inhibitor. He demon-
in order to obtain tissue confirmation of the diagnosis and strated transient symptom relief and is engaged in palliative
understand the biological characteristics of these tumors. care services.

Further Reading
American Academy of Pediatrics Section on Hospice and Gwak, H.S. and Park, H.J. (2017). Developing chemotherapy
Palliative Medicine and Committee on Hospital Care for diffuse pontine intrinsic gliomas (DIPG). Critical Reviews
(2013). Pediatric palliative care and hospice care in Oncology/Hematology 120: 111–119.
commitments, guidelines and recommendations. Pediatrics Hargrave, D., Bartels, U., and Bouffet, E. (2006). Diffuse
132(5): 966–972. brainstem glioma in children: critical review of clinical
Beltran, C., Sharm, S., and Merchant, T. (2011). Role of trials. Lancet Oncology 7(3): 241–248.
adaptive radiation therapy for pediatric patients with diffuse Ropper, A.H., Samuels, M.A., Klein, J.P., and Prasad, S. (2019).
pontine glioma. Journal of Applied Clinical Medical Physics Adams and Victor’s Principles of Neurology, 11th edn. New
12(2): 1–11. York: McGraw Hill.
D’Arco, F., Khan, F., Mankad, K., et al. (2018). Differential Weaver, M.S., Heinze, K.E., Kelly, K.P., et al. (2015). Palliative
diagnosis of posterior fossa tumours in children: new care as a standard of care in pediatric oncology. Pediatric
insights. Pediatric Radiology 48: 1955–1963. Blood and Cancer 62(Suppl 5): S829–S833.
Downloaded from https://onlinelibrary.wiley.com/doi/ by National Institute Of Standard, Wiley Online Library on [06/03/2024]. See the Terms and Conditions (https://onlinelibrary.wiley.com/terms-and-conditions) on Wiley Online Library for rules of use; OA articles are governed by the applicable Creative Commons License
207

48

Acetaminophen Ingestion
Aaron Carpenter
Nemours/Alfred I. duPont Hospital for Children, Wilmington, DE, USA

A 19-month-old female presents to the emergency tracking at the 50th to 60th percentile and height gain at
department (ED) for parental concern of acetaminophen the 35th to 40th percentile. She attends daycare and has
overdose. had two episodes of wheezing associated with upper respi-
ratory virus infection for which she was prescribed albu-
terol as needed. She does not take any regularly
History of Present Illness prescribed medications. Her height and weight on the
day of presentation to the ED are 82 cm and 11 kg (body
The patient’s mother reports that this 19-month-old child mass index 16.4).
developed cough, runny nose, and fever 2 days prior to pres-
entation with a maximum temperature at home of 39.1 C
(102.3 F). The 5-year-old sibling developed similar symp-
toms 4 days prior. A new bottle of acetaminophen liquid Current Status
(160 mg per 5 mL) was purchased by the mother and two
doses had been given from the new bottle the previous The child appears well, sitting on her mother’s lap, smiling,
day, one dose for this patient and one dose for her older playful and interactive; she is appropriately fearful of
brother. Two hours prior to arrival in the ED, the patient’s the examiner. Temperature is 37.7 C (99.9 F), heart rate
mother took the acetaminophen bottle out of the cabinet to 108 bpm, respiratory rate 28 breaths per minute, non-
administer a dose and, while distracted by the sibling, the labored blood pressure 88/52 mmHg, and oxygen satura-
19-month-old child took the bottle and carried it to another tion 99% on room air.
room. The mother found the young child with the bottle
opened and more than half empty with no evidence of spill-
age. The mother then called the child’s pediatric nurse
practitioner at the primary care provider who recom- Questions
mended calling the poison control center, and referred
her to the ED. Answer the following questions using the details provided.
1 Based on the history, would you be concerned with a
potentially toxic dose of acetaminophen?
Past Medical History 2 What data would you need to determine the appropri-
ate intervention, if any?
The patient was born at 39 weeks’ gestation via spontane- 3 What pharmacologic interventions are available for
ous vaginal delivery with no complications. Mother experi- acute acetaminophen toxicity?
enced positive prenatal care and pregnancy was 4 What are the contraindications for the use of NAC?
unremarkable. The child has met her developmental mile- 5 How should the patient be monitored during the course
stones and has been growing well, with weight gain of treatment?

Cases in Pediatric Acute Care: Strengthening Clinical Decision Making, First Edition. Edited by Andrea M. Kline-Tilford and Catherine M. Haut.
© 2020 John Wiley & Sons Ltd. Published 2020 by John Wiley & Sons Ltd.
Downloaded from https://onlinelibrary.wiley.com/doi/ by National Institute Of Standard, Wiley Online Library on [06/03/2024]. See the Terms and Conditions (https://onlinelibrary.wiley.com/terms-and-conditions) on Wiley Online Library for rules of use; OA articles are governed by the applicable Creative Commons License
208 Cases in Pediatric Acute Care

Rationale and Evidence-based Practice failure early in the ingestion phase and which therapies
Explanation are most effective.
Chiew et al. (2018) published a systematic review of
Based on the history, would you be concerned with 11 randomized controlled trials assessing interventions
a potentially toxic dose of acetaminophen? for acetaminophen overdose in approximately 700 people.
Although the evidence was classified as low or very low,
Determining concern for acetaminophen toxicity requires the authors concluded that activated charcoal seemed to
identifying, as accurately as possible, the total dose ingested be the best agent for preventing the absorption of
and the timing of the dose. Fortunately, in this case the par- acetaminophen. NAC has been shown, in observational
ent brought the bottle of acetaminophen to the ED. After studies, to reduce morbidity and mortality from liver
subtracting the volume of doses given the previous day toxicity.
and measuring the amount remaining in the bottle, it
was estimated that the patient had ingested a total of
60 mL of acetaminophen suspension (160 mg per 5 mL) What pharmacologic interventions are available
2 hours prior to evaluation in the ED. The total acetamin- for acute acetaminophen toxicity?
ophen dose ingestion was 1920 mg or 174.5 mg/kg. A dose
of 150 mg/kg or more is concerning for potential toxicity When administered in therapeutic doses, acetaminophen is
(Ogilvie et al., 2012). metabolized in the liver, primarily through glucuronidation
and sulfation. A toxic metabolite, N-acetyl-p-benzoquinone
imine (NAPQI), is conjugated by glutathione into mercap-
What data would you need to determine turic acid, which can be safely excreted through the kid-
the appropriate intervention, if any? neys. In the setting of an overdose, glutathione stores can
be overwhelmed leading to an increase in the toxic metab-
The total dose, time since ingestion, and the Rumack–
olite, which can lead to liver cell death.
Matthew nomogram (Figure 48.1) are used in the setting
Activated charcoal, which may reduce the absorption of
of an acetaminophen overdose to direct treatment of the
acetaminophen, can be given if ingestion occurs within
patient. The Rumack–Matthew nomogram was devised in
4 hours of presentation and the dose is within the potential
1975 and has been used to determine acetaminophen tox-
for toxicity (150–200 mg/kg). NAC can replenish glutathi-
icity based on timing of ingestion and serum acetamino-
one stores, thereby allowing for increased conjugation of
phen level. Consultation with the poison control center
the toxic metabolite NAPQI into mercapturic acid, which
also assists in management and monitoring. Serum aceta-
is then safely excreted through the kidneys (Ogilvie et al.,
minophen levels are obtained no sooner than 4 hours after
2012). Given the relative accuracy of total dose ingested
ingestion to assess for peak serum level. Determining that
by this patient, activated charcoal and NAC were started
the accidental ingestion occurred 2 hours prior to arrival in
3 hours after ingestion. NAC can be administered orally
the ED is important in patient management as it informs
on a 72-hour regimen or intravenously on a 21-hour regi-
the decision to administer pharmacologic agents to
men. The oral form is not well tolerated in children due
reduce the risk of liver toxicity. Additionally, liver enzyme
to its odor and may cause vomiting.
testing aids in the determination of damage to liver from
toxic metabolites.
The toddler’s serum acetaminophen level drawn 4 hours What are the contraindications for the use of NAC?
after ingestion was 187 mg/L, a level that fits within the rec-
ommendation for treatment with N-acetylcysteine (NAC). As with most pharmacologic agents, hypersensitivity to
Liver enzymes were all within normal range and the patient NAC is a contraindication. Caution is recommended in
remained alert, active and playful during the course of her patients with a history of bronchospasm or asthma as these
ED stay. patients may be at increased risk for hypersensitivity reac-
Managing acetaminophen poisoning has been a tion (Lexicomp, 2019).
long-term concern for healthcare providers due to rapid Treatment duration for acetaminophen toxicity is
liver involvement and the potential for liver failure. dependent on the patient’s course and timing of initiation
Figure 48.2 shows the normal metabolism of acetamino- of NAC from ingestion. In this case, the serum acetamino-
phen and the safe excretion of a toxic metabolite, while phen level was zero and alanine aminotransferase
Figure 48.3 shows the pathways of acetaminophen toxicity remained within normal limits at 20 hours of treatment,
that result in the development of liver disease or failure. allowing for discontinuation of NAC at 21 hours of treat-
The challenge remains in identifying potential for liver ment and discharge of the patient.
Downloaded from https://onlinelibrary.wiley.com/doi/ by National Institute Of Standard, Wiley Online Library on [06/03/2024]. See the Terms and Conditions (https://onlinelibrary.wiley.com/terms-and-conditions) on Wiley Online Library for rules of use; OA articles are governed by the applicable Creative Commons License
Chapter 48 Acetaminophen Ingestion 209

Single Acute Acetaminophen Overdose Nomogram

μg/mL
500
400

300

200

150
Rumack-Matthew Line

100
90
ACETAMINOPHEN PLASMA CONCENTRATION

80
70
60
50
40

30

20

Treatment Line

10
9
8
7
6
5
4

Treatment should be administered


2
if level is above solid line

4 8 12 16 20 24 28 32 36

HOURS POSTINGESTION

Nomogram: acetaminophen plasma concentration vs time after CAUTIONS FOR USE OF THIS CHART:
acetaminophen ingestion (adapted with permission from Rumack 1. Time coordinates refer to time postingestion.
and Mathew, Pediatrics 1975:55-871-876). The nomogram has 2. Graph relates only to plasma concentrations following a single,
been developed to estimate the probability of whether a plasma acute overdose ingestion.
acetaminophen concentration in relation to the interval postinges- 3. The Treatment Line is plotted 25% below the Rumack-Matthew
tion will result in hepatotoxicity and, therefore, whether acetylcys- Line to allow for potential errors in plasma acetaminophen
teine therapy should be administered. assays and estimated time from ingestion of an overdose
(Rumack et al, Arch latern Med. 1981:141(suppl):380-385).

Figure 48.1 Rumack–Matthew nomogram. Source: Merlin Cyrstal, https://upload.wikimedia.org/wikipedia/commons/9/9b/


Rumack_Matthew_nomogram_with_treatment_%28study%29_line.pdf. Licensed under CC BY SA 3.0.
Downloaded from https://onlinelibrary.wiley.com/doi/ by National Institute Of Standard, Wiley Online Library on [06/03/2024]. See the Terms and Conditions (https://onlinelibrary.wiley.com/terms-and-conditions) on Wiley Online Library for rules of use; OA articles are governed by the applicable Creative Commons License
210 Cases in Pediatric Acute Care

Acetaminophen
Acetaminophen Liver Liver
Metabolism with
Metabolism
Toxicity

CYP450 2E1 CYP450 2E1


Glucuronidation Sulfation Glucuronidation Sulfation
oxidation oxidation
8
%

Forms a reactive metabolite Major pathways Increased production of


N-acetyl-p-benzoquine (NAPQI) become saturated N-acetyl-p-benzoquine (NAPQI)
90% with drug
Conjugated and
excreted through
kidney
Toxic metabolite reacts with When glutathione is
sulfhydryl groups in depleted, NAPQI
glutathione to harmless accumulates in hepatocytes
metabolites, then excreted resulting in hepatic damage

Other names: N-acetyl-p-aminophenol, APAP, paracetamol, PARA


High oral bioavailability: reaches peak concentration in 90 minutes Figure 48.3 Metabolism of acetaminophen toxicity.
Toxic dose can cause liver damage: 120–150 mg/kg in a single dose can cause
hepatotoxicity
Liver, kidneys and intestine are organs implicated in metabolism
Maximum pediatric dosing per day: 50–75 mg/kg
Half-life increases from 1.5–2.5 hours to 4–8 hours with toxic doses
admission with frequent monitoring of liver function. Edu-
Ingestion or overdose: glucuronidation and sulfation hepatic pathways are saturated cation on the importance of safe storage of medications is
with drug, causing more tylenol to be metabolized by CYP450 system
N-Acetylcysteine is the antidote, by replacing glutathione to metabolize drug another essential piece of patient and family management,
with referral to social work sometimes necessary.
Figure 48.2 Metabolism of acetaminophen.

How should the patient be monitored during the


course of treatment?
Patients who present with liver toxicity or fulminant liver
failure require expert consultation and intensive care unit

References
Chiew, A.L., Gluud, C., Brok, J., and Buckley, N.A. (2018). Lexicomp: Alfred I. duPont Hospital for Children formulary
Interventions for paracetamol (acetaminophen) (2019). Wolters Kluwer Clinical Drug information, Inc.
overdose. Cochrane Database of Systematic Ogilvie, J.D., Rieder, M.J., and Lim, R. (2012). Acetaminophen
Reviews (2): CD003328. doi: 10.1002/14651858. overdose in children. Canadian Medical Association Journal
CD003328.pub3. 184(13): 1492–1496. doi: 10.1503/cmaj.111338.
Downloaded from https://onlinelibrary.wiley.com/doi/ by National Institute Of Standard, Wiley Online Library on [06/03/2024]. See the Terms and Conditions (https://onlinelibrary.wiley.com/terms-and-conditions) on Wiley Online Library for rules of use; OA articles are governed by the applicable Creative Commons License
211

49

A 3-Year-Old with Sudden Onset of Breathing Difficulty


Judy Verger
University of Iowa School of Nursing, Iowa City, IA, USA

A 3-year-old female with Tetralogy of Fallot (TOF) is being Family History/Social History
admitted to the pediatric intensive care unit (PICU) for
abdominal pain. The nurse practitioner has been tasked She has no significant family medical history. She was
to administer moderate sedation for abdominal computed transferred from Qatar to the United States for further
tomography (CT) prior to transfer to PICU. care. She lives with both parents and her younger
brother in Qatar. Her father is the only one who speaks
English.

History of Present Illness

Patient is a 3-year-old female with history of CHARGE syn-


drome who is to undergo an abdominal CT with and with- Current Status
out contrast under moderate sedation. Topical lidocaine
cream was used when gaining peripheral intravenous Vital signs are as follows: temperature 36.8 C (98.2 F),
access, and midazolam 0.05 mg/kg and fentanyl 0.5 mcg/kg heart rate 167 bpm, blood pressure 52/28 mmHg, respira-
were administered at the beginning of the procedure. The tory rate 56 breaths per minute, and oxygen saturation
patient begins to have difficulty breathing in the middle 82% on room air. Weight is 12.5 kg.
of the procedure, during contrast infusion. The child appears flushed and anxious and is thrashing
and attempting to sit up.

Past Medical History • Head, eyes, ears, nose and throat: malformed protruding
pinna, wide-set ears, perioral edema, erythematous
mouth, swollen lips, very hoarse/raspy voice. Coloboma
of right eye noted. Right-sided facial palsy.


The child has CHARGE syndrome with the following asso-
ciated anomalies: TOF, tracheomalacia with gastroesopha- Respiratory: retractions, poor air entry bilaterally with
inspiratory and expiratory wheeze.


geal reflux disease, ocular coloboma, deafness, and
developmental delay. She also has failure to thrive and Cardiovascular: tachycardic, first and second heart
sounds heard, harsh systolic murmur.

••
asthma.
She has no known drug allergies. Her immunizations are Extremities: cool, normal musculoskeletal examination.
Abdomen: soft and round, positive bowel sounds.


up to date as reported by her parents. Home medications
include aldactone, digoxin, ranitidine, and albuterol. Skin: large hives over face, neck, chest, abdomen
and back.


She was nil by mouth at the time of the event, but is oth-
erwise fed using a nasogastric tube with homemade Neurologic: pupils 5 mm equal and react to light
blended food. briskly.

Cases in Pediatric Acute Care: Strengthening Clinical Decision Making, First Edition. Edited by Andrea M. Kline-Tilford and Catherine M. Haut.
© 2020 John Wiley & Sons Ltd. Published 2020 by John Wiley & Sons Ltd.
Downloaded from https://onlinelibrary.wiley.com/doi/ by National Institute Of Standard, Wiley Online Library on [06/03/2024]. See the Terms and Conditions (https://onlinelibrary.wiley.com/terms-and-conditions) on Wiley Online Library for rules of use; OA articles are governed by the applicable Creative Commons License
212 Cases in Pediatric Acute Care

Questions What is the differential diagnosis of the patient’s


acute deterioration during the CT scan, and what is
Answer the following questions using the details provided. the most likely diagnosis?

1 What are the features and genetic mutations associated Because of this patient’s complex history, there are many
with CHARGE syndrome? potential differential diagnoses:
2 What is the differential diagnosis of the patient’s acute
deterioration during the CT scan, and what is the most •• anaphylactic shock
anxiety/panic attack

3
likely diagnosis?
What is the pathophysiology of the most likely •• TET spell
pulmonary embolism

4
diagnosis?
What are the initial steps in treating this patient for •• airway obstruction due to tracheomalacia
acute asthmatic attack.
anaphylaxis?
Most likely diagnosis is anaphylactic shock, due to the
sudden onset of breathing difficulty, rash (hives), hypoten-
sion, and hypoxia with associated biphasic wheeze after
Rationale and Evidence-based Practice
administration of intravenous contrast. Intravenous CT
Explanation contrast is known to be associated with hypersensitivity/
anaphylactic reactions, with an incidence of 0.001–0.004%
What are the features and genetic mutations
(Kim, M.H. et al., 2012), and can be fatal (Kim, S.M.
associated with CHARGE syndrome?
et al., 2016).
CHARGE syndrome is an autosomal dominant genetic dis- Anxiety of panic attacks may be associated with wheeze
order typically caused by pathogenic variants of the CHD7 and occasionally erythematous rash, but not hypotension.
gene. CHARGE syndrome comprises the following (Kahl Asthma may also be associated with wheezing, but rarely
and Hughes, 2018, p. 496): hypotension. Obstruction from tracheomalacia is typically
C Coloboma associated with stridor, but no hypotension or sudden onset
H Heart anomalies of rash. Pulmonary embolism, and TET spells are typically
A Atresia choanae associated with hypotension, but not rash and wheezing.
R Retardation of growth and development
G Genitourinary anomalies What is the pathophysiology of the most likely
E Ear anomalies (including deafness and vestibular diagnosis?
disorders). Anaphylaxis is a serious allergic or hypersensitivity reac-
These features are present in various combinations and to tion mediated by IgE. It is rapid in onset and usually life-
varying degrees. No single feature is universally present. threatening (Castells, 2017). A previously sensitized
Clinical diagnosis of CHARGE syndrome requires the B lymphocyte produces IgE against a specific antigen.
presence of at least four major features or three major fea- The IgE resides on the mast cells and basophils. Mast cell
tures plus three minor features. Major features include: and basophil degranulation occurs when a specific antigen,

••
or one similar to it, binds to the IgE receptor on the mast
choanal atresia or stenosis cell and basophil, leading to release of histamine.
cranial nerve anomalies

••
The clinical presentations seen in anaphylaxis are due to
eye coloboma activation of multiple histamine receptors. Histamine H1
auditory/auricular anomalies. and H2 receptor activity result in the following.
Minor features include:
• Respiratory: bronchial smooth muscle constriction and

••
increased mucus viscosity result from H1 receptor activ-
cleft lip or palate
ity. H2 activity causes increased mucus production.

••
tracheoesophageal fistula
congenital heart defects
genitourinary anomalies
• Cardiovascular: H1 and H2 receptor stimulation results in
increased vascular permeability resulting in decreased

••
preload and hypotension, and vasodilation leading to
developmental delay
flushing, reflex tachycardia, and headache.
distinctive facial dysmorphology (asymmetrical square
face, malar flattening, unilateral facial nerve paralysis, H1 and H3 receptor activity is associated with cutaneous
micrognathia, low-set cupped ears). itch and nasal congestion.
Downloaded from https://onlinelibrary.wiley.com/doi/ by National Institute Of Standard, Wiley Online Library on [06/03/2024]. See the Terms and Conditions (https://onlinelibrary.wiley.com/terms-and-conditions) on Wiley Online Library for rules of use; OA articles are governed by the applicable Creative Commons License
Chapter 49 A 3-Year-Old with Sudden Onset of Breathing Difficulty 213

1
Anaphylaxis can therefore be defined three ways.
Sudden onset of an illness that involves the skin and/
• Place patient supine and administer 20 mL/kg of isotonic
fluid bolus, and elevate legs at 30 above the head to help
maximize preload (unless prominent upper airway swell-
or mucous membranes with associated sudden respi-
ing or compromise, in which case elevating legs may be
ratory symptoms, or hypotension/signs of end-organ
counterproductive) (Lieberman et al., 2015; Restauri
dysfunction.
et al., 2016; Castells, 2017; Hughes and Kahl,
2 Sudden occurrence of two or more of the following after
2018, pp. 4–5).


exposure to a likely allergen:

•• alterations of the skin/mucous membrane


respiratory signs/symptoms
Consider administering histamine H1 (and H2) receptor
antagonist (Lieberman et al., 2015; Simons et al., 2015,

••gastrointestinal symptoms
hypotension/signs of end-organ dysfunction. •
p. 10; Castells, 2017; Hughes and Kahl, 2018, pp. 4–5).
Consider albuterol or racemic epinephrine for wheezing
and bronchospasm.
3 Hypotension after exposure to known allergen (Simons
et al., 2015, p. 10). • Start intravenous steroids (methylprednisolone 2 mg/kg):
has no immediate effect but can prevent late or biphasic
reactions.
What are the initial steps in treating this patient
for anaphylaxis? If symptoms persist or there is impending respiratory

••
arrest, carry out the following.
Immediately stop administration of the contrast.

••
Remove child from scanner.
Assess airway, breathing and circulation (ABC).
• Administer 10–20 mL/kg of intravenous fluids (normal
saline, Ringer’s lactate) followed by continuous infusion
Call for assistance from radiology personnel and/or of intravenous fluids.


resuscitation team.
Provide supplemental oxygen (preferably high flow) via
•• Start continuous intravenous epinephrine.
Position the airway to facilitate aeration and provide bag-
facemask. mask ventilation. Use nasopharyngeal airway to aid in

• Administer epinephrine with EpiPen autoinjector intra-


muscularly (or epinephrine 1 in 1000 0.01 mg/kg, up to

keeping airway patent.
Prepare for intubation. Severe anaphylaxis can cause
0.5 mg) in the mid-outer thigh. If no response, repeat angioedema, obstructing the airway (Lieberman et al.,
injection every 5 minutes. Epinephrine administration 2015; Castells, 2017; Hughes and Kahl, 2018, p. 87).
addresses both airway obstruction and hypotension by The most experienced practitioners should perform intu-
decreasing bronchospasm and increasing vascular bation, as airway narrowing may make intubation
resistance. difficult.

References
Castells, M.J. (2017). When to order contrast-enhanced CT. Lieberman, P., Nicklas, R.A., Randolph C., et al. (2015).
Journal of Allergy and Clinical Immunology 140: 321–333. Anaphylaxis: a practice parameter update 2015. Annals of
Kahl, L. and Hughes, H. (2018). The John’s Hopkins Hospital Allery, Asthma and Immunology 115: 341–384.
Harriet Lane Handbook, 21st edn. Philadelphia, PA: Mosby Restauri, N., Lio, E., Glueck, D., et al. (2016). Best practice for
Elsevier. safe and effective administration of epinephrine for the
Kim, M.H., Park, C.H., Kim, D.I., et al. (2012). Surveillance of treatment of anaphylaxis in the radiology department.
contrast-media-induced hypersensitivity reactions using Journal of the American College of Radiology 13(3): 303–306.
signals from an electronic medical recording system. Annals doi: 10.1016/j.jacr.2015.08.018.
of Allergy, Asthma and Immunology 108: 167–171. Simons, F.E., Ebisawa, M., Sanchez-Borges, M., et al. (2015).
Kim, S.M., Ko, B.S., Kim, J.Y., et al. (2016). Clinical factors for 2015 update of the evidence base: World Allergy Organization
developing shock in radiocontrast media induced anaphylaxis guidelines. World Allergy Organization Journal
anaphylaxis. Shock 45: 315–319. 8(1): 32. doi: 10.1186/s40413-015-0080-1.
Downloaded from https://onlinelibrary.wiley.com/doi/ by National Institute Of Standard, Wiley Online Library on [06/03/2024]. See the Terms and Conditions (https://onlinelibrary.wiley.com/terms-and-conditions) on Wiley Online Library for rules of use; OA articles are governed by the applicable Creative Commons License
215

50

A 2-Month-Old with Acute Abdomen


Christyne Kyper
Alaska, Native Tribal Health Consortium, Anchorage, AK, USA

A 2-month-old male infant, born at term, presents with Past Surgical History
vomiting and decreased oral intake for 1 week. He is being
evaluated for an acute abdomen, with laboratory analysis Circumcision during birth hospitalization.
and radiologic procedure. He will go to the operating room
emergently once the work-up is completed.

Family History

History of Present Illness Mother has gestational diabetes and history of asthma as a
child. Father is not involved with infant care, but also had
This 2-month-old infant was healthy until 1 week prior to asthma as a child. The maternal grandmother is deceased
admission when he developed persistent non-bilious vom- from lung cancer, maternal grandfather has hypertension,
iting. He was seen in the emergency department where an and paternal grandparents are alive and healthy.
abdominal radiograph and abdominal ultrasound were
negative. An upper gastrointestinal examination was con-
cerning for intestinal obstruction. After consultation with
the pediatric surgery team, he was taken to the operat- Questions
ing room.
Answer the following questions using the details provided.
1 What differential diagnoses would you consider and
what would be the top diagnosis on the list?
Past Medical History 2 What diagnostic study is most accurate when evaluat-
ing for anatomic abnormalities?
The male infant was born at 38 weeks’ gestation by 3 What additional information should be obtained before
spontaneous vaginal delivery to a mother with gesta- taking the infant to the operating room?
tional diabetes. Other than being large for gestational 4 What is involved in the process of an upper gastrointes-
age, there were no other complications at birth. The tinal examination and what findings would indicate
infant has reported colic and is a “spitter” according malrotation?
to mother, but has been gaining weight and has regular 5 What is the pathophysiology of intestinal malrotation?
bowel movements. The infant recently had his 2-month
Continuaton of case: This infant is diagnosed with malro-
well child visit, and received second hepatitis B, rotavi-
tation and plans are being made to take him to the operat-
rus, diphtheria, tetanus and pertussis (DTaP), Haemo-
ing room for surgical intervention.
philus influenzae, and first pneumococcal vaccines at
that time. 6 What is a Ladd procedure?

Cases in Pediatric Acute Care: Strengthening Clinical Decision Making, First Edition. Edited by Andrea M. Kline-Tilford and Catherine M. Haut.
© 2020 John Wiley & Sons Ltd. Published 2020 by John Wiley & Sons Ltd.
Downloaded from https://onlinelibrary.wiley.com/doi/ by National Institute Of Standard, Wiley Online Library on [06/03/2024]. See the Terms and Conditions (https://onlinelibrary.wiley.com/terms-and-conditions) on Wiley Online Library for rules of use; OA articles are governed by the applicable Creative Commons License
216 Cases in Pediatric Acute Care

Rationale and Evidence-based Practice obstruction and/or constipation are two other problems
Explanation that can be identified using plain abdominal films. Findings
that are diagnostic of a bowel perforation are a nasogastric
What differential diagnoses would you consider or orogastric tube that extends into an abnormally posi-
and what would be the top diagnosis on the list? tioned duodenum and the “double-bubble” sign, which is
present when partial obstruction of the duodenum causes
Differential diagnoses for this infant include gastroesopha- distension of the stomach and first part of the duodenum.
geal reflux disease (GERD), pyloric stenosis, intussuscep-
tion, malrotation (with or without volvulus), duodenal
What is involved in the process of an upper
obstruction, and gastroenteritis.
gastrointestinal examination and what findings
Gastroenteritis in an otherwise well appearing infant
would indicate malrotation?
without diarrhea is unlikely and there is no history of expo-
sure to illness. GERD is a possibility, but given the acute An upper gastrointestinal contrast study is the best exami-
nature of the vomiting, it is important to evaluate for other nation for visualizing the duodenum in infants and chil-
diagnoses first. dren. Sensitivity for this study is 96% for malrotation. An
Pyloric stenosis, intussusception, duodenal obstruction, upper gastrointestinal examination is performed under
and malrotation are all anatomic abnormalities determined fluoroscopy by a radiologist. The infant is given a bottle
by radiographic studies. An infant with a bowel obstruction with contrast to drink and then a series of images are taken
would typically present with bilious emesis. Patients pre- as the contrast passes through the small bowel. Radio-
senting with an acute abdomen should have a plain abdom- graphic signs of malrotation include a clearly misplaced
inal radiograph to evaluate for the possibility of an duodenum with the ligament of Treitz on the right side
intestinal perforation, which is a life-threatening surgical of the abdomen, a duodenum with a “corkscrew” appear-
emergency. Laboratory studies can assist in the identifica- ance, or duodenal obstruction.
tion of a bowel perforation. A simple metabolic panel can
indicate the presence of acidosis, which is typically present What is the pathophysiology of intestinal
with a perforation. malrotation?
Rotational abnormalities occur as a result of an arrest of
What diagnostic study is most accurate when normal rotation of the embryonic gut. Malrotation is often
evaluating for anatomic abnormalities?
associated with other abnormalities (duodenal atresia,
Ultrasound is a reliable method for diagnosis of pyloric ste- imperforate anus, Meckel diverticulum, biliary atresia,
nosis, but not for anatomic abnormalities. Upper gastroin- and cardiac anomalies are among some of the associated
testinal investigation is considered the gold standard for findings), but can also occur in otherwise healthy infants
diagnosing these conditions. and children. Over half of children with intestinal malrota-
tion will present by 1 year of age. Primary clinical findings
What additional information should be obtained in infants are vomiting (93%), typically bilious but can be
before taking the infant to the operating room? non-bilious, abdominal distention and tenderness (which
is difficult to assess in infants), and hemodynamic instabil-
An infant who has been vomiting is at risk for dehydration ity from hypovolemia and/or septic shock if there is a per-
and electrolyte abnormalities, which should both be cor- foration. Less common presentations are failure to thrive,
rected prior to going to the operating room. Most parents malabsorption, solid food intolerance, chronic diarrhea,
and nurses have difficulty quantifying the volume of eme- pancreatitis, biliary obstruction, gastrointestinal motility
sis, which makes the accuracy of this assessment question- disorder, and chylous ascites. Malrotation can lead to intes-
able. Hydration status can be assessed clinically by tinal volvulus, a life threatening condition that can result in
checking the infant’s weight, condition of fontanelle, skin bowel ischemia, necrosis, sepsis and death, so it is
turgor, and mucous membranes, and by asking how many extremely important to begin work-up quickly, diagnose
wet diapers the infant has had in the last 24 hours. Evalu- the problem and take the infant to the operating room.
ation of electrolytes will also provide an indication of
hydration status.
What is a Ladd procedure?
Plain radiographs are most often obtained first with a gas-
trointestinal complaint or problem. An abdominal X-ray The Ladd procedure, first described by William Ladd, is
taken in the left-lateral decubitus position can assist in used to correct malrotation and midgut volvulus. At the
the diagnosis of intestinal perforation. Suggestion of an time of surgery, if a volvulus is visualized, it is untwisted
Downloaded from https://onlinelibrary.wiley.com/doi/ by National Institute Of Standard, Wiley Online Library on [06/03/2024]. See the Terms and Conditions (https://onlinelibrary.wiley.com/terms-and-conditions) on Wiley Online Library for rules of use; OA articles are governed by the applicable Creative Commons License
Chapter 50 A 2-Month-Old with Acute Abdomen 217

or detorsed in a clockwise fashion. If present, division of performed. In the absence of volvulus or bowel ischemia,
the Ladd bands, which are the peritoneal bands crossing this procedure can be done laparoscopically. Complica-
from the right paracolic gutter over the abdomen, is per- tions are rare from a Ladd procedure if it is an isolated
formed along with widening of the base of the mesentery malrotation. If it is associated with a volvulus, bowel re-
by dividing any adhesions between the duodenum and section could be necessary and this can result in short
cecum. In a neonate, a tube is passed through the duode- bowel syndrome.
num to evaluate for associated duodenal obstruction. Via- Preoperative management includes cardiopulmonary
ble bowel is placed in a position of non-rotation, small and circulatory resuscitation if necessary, placement of a
bowel on the right, colon on the left, and adhesions are gastric decompression tube, and administration of broad-
created to hold the intestines in place, making subsequent spectrum antibiotics to treat bowel flora which may have
volvulus less likely; an incidental appendectomy is also spilled into the peritoneal cavity.

Further Reading
Graziano, K., Islam, S., Dasgupta, R., et al. (2015). Hasosah, M.Y., Meer, W.I., and Alsaleem, K. (2016). Intestinal
Asymptomatic malrotation: diagnosis and surgical volvulus: a life-threatening disease. Journal of Clinical and
management. Journal of Pediatric Surgery 50(10): Diagnostic Research 10(3): SJ01.
1783–1790. doi: 10.1016/j.jpedsurg.2015.06.019
Downloaded from https://onlinelibrary.wiley.com/doi/ by National Institute Of Standard, Wiley Online Library on [06/03/2024]. See the Terms and Conditions (https://onlinelibrary.wiley.com/terms-and-conditions) on Wiley Online Library for rules of use; OA articles are governed by the applicable Creative Commons License
219

51

Henoch–Schönlein Purpura Nephritis


Tamara Hill
University of Maryland Medical Center, Baltimore, MD, USA

A 16-year-old female presents with fever, gross hematuria Family History


and sore throat for 5 days.
The girl’s mother has had hypertension since age 40 and
maternal grandfather has also been treated for the same.
Father of child is healthy.
History of Present Illness

This 16-year-old female was treated for strep throat with


oral penicillin by her primary care provider. After a day,
Current Status
she developed an erythematous non-pruritic rash which
On physical examination, there is pharyngeal erythema,
progressed proximally from both feet to thighs and upper
petechiae on the soft palate, cervical lymphadenopathy,
extremities including palms and soles. Later, her feet
and a nodular, non-tender, non-blanching purpuric rash
became swollen with moderately intense (7/10) burning
involving both upper and lower extremities with non-
pain, aggravated by ambulation. Two days later, the patient
pitting pedal edema. There is no truncal involvement.
developed vomiting and abdominal pain involving the right
A comprehensive metabolic panel, complete blood count
and left upper quadrant that was constant, colicky in
and urinalysis are obtained and the results are shown in
nature, 8/10 in intensity, and aggravated with meals and
Tables 51.1–51.3. Results of additional investigations are
bloody stools.
as follows.

•• C-reactive protein (CRP): 57.5 mg/L


Prothrombin time: 18.0 seconds
Past Medical History
•• International normalized ratio: 4
Complement: C3 125 mg/dL; C4 11 mg/dL
The patient was born by spontaneous vaginal delivery at
term and went home with her mother. She is current on •• Antistreptolysin O titer: 823 IU
Anti-nuclear antibody: negative
all immunizations and has received annual influenza
immunization. She has a history of mild intermittent • Stool for occult blood: positive.

asthma.

Questions
Past Surgical History
Answer the following questions using the details provided.
This teen had a gastrojejunostomy following a remote 1 Based on the data given, what is the differential diagno-
motor vehicle accident 2 years ago. sis and the most likely diagnosis for this child?

Cases in Pediatric Acute Care: Strengthening Clinical Decision Making, First Edition. Edited by Andrea M. Kline-Tilford and Catherine M. Haut.
© 2020 John Wiley & Sons Ltd. Published 2020 by John Wiley & Sons Ltd.
Downloaded from https://onlinelibrary.wiley.com/doi/ by National Institute Of Standard, Wiley Online Library on [06/03/2024]. See the Terms and Conditions (https://onlinelibrary.wiley.com/terms-and-conditions) on Wiley Online Library for rules of use; OA articles are governed by the applicable Creative Commons License
220 Cases in Pediatric Acute Care

Table 51.1 Complete blood count. 2 What is the best initial management for Henoch–
Schönlein purpura?
White blood cell count 23 000/mm3
3 How would you determine if the therapy was effective?
Hemoglobin 10.8 g/dL 4 What other therapies would be recommended for
Hematocrit 32% Henoch–Schönlein purpura?
Platelet count 693 000/mm3 5 Would you obtain any other diagnostic studies?

Table 51.2 Complete metabolic profile.


Rationale and Evidence-based Practice
Explanation
Sodium 138 mEq/L
Potassium 4.6 mEq/L Based on the data given, what is the differential
Chloride 101 mEq/L diagnosis and the most likely diagnosis for
this child?
Carbon dioxide 22 mEq/L
Blood urea nitrogen 24 mg/dL IgA vasculitis (IgAV), also called Henoch–Schönlein
Creatinine 2.4 mg/dL purpura (HSP), is a systemic vasculitis with a prominent
Glucose 88 mg/dL cutaneous component. HSP is an acute, systemic, immune
complex-mediated leukocytoclastic vasculitis. It is charac-
Calcium 7.8 mg/dL
terized by a clinical triad of palpable purpura (without
Albumin 2.9 mg/dL
thrombocytopenia), abdominal pain, and arthritis. Glomer-
Total protein 4.9 g/dL
ulonephritis and gastrointestinal bleeding are common
Aspartate aminotransferase 24 U/L complications. In patients with HSP, IgA immune
Alanine aminotransferase 24 U/L complexes are deposited in small vessels, which causes
Alkaline phosphatase 60 U/L petechiae and palpable purpura. When immune complexes
Total bilirubin 0.5 mg/dL occur in small vessels of the intestinal wall, gastrointestinal
Magnesium 1.6 mEq/L hemorrhage may develop. If the immune complexes affect
Phosphorus 4.2 mg/dL
the renal mesangium, it may produce mild proliferative to
severe crescentic glomerulonephritis.
IgAV (HSP) is characterized by the tissue deposition of
IgA-containing immune complexes. The histologic findings
in the kidney are identical to those in IgA nephropathy,
Table 51.3 Urinalysis. suggesting that the two disorders may have a similar path-
ogenesis. Likewise, the finding of high circulating levels of
Color Dark yellow galactose-deficient IgA1 in patients with both IgA nephro-
pH 6.3 pathy and IgAV (HSP) suggest a common underlying path-
Specific gravity 1.008 ogenetic mechanism. Exposure to an antigen from an
Protein 3+ infection, medication, or other environmental factor may
Nitrites Negative trigger antibody and immune complex formation.
Urobilirubin 1+
Blood 3+ What is the best initial management
Leukocytes Negative for Henoch–Schönlein purpura?
Ketones Negative
No specific therapy for HSP exists. The main goal of treat-
Glucose Negative
ment is to relieve symptoms such as joint pain, abdominal
White blood cell count 26 per high power field pain, and swelling. People with kidney involvement may
Red blood cell count 12 per high power field receive treatment aimed at preventing long-term kidney
Hyaline casts None disease.
Bacteria Occasional Treatment is rarely required for the rash. Joint pain is
Crystals none often treated with non-steroidal anti-inflammatory medica-
tions such as aspirin or ibuprofen. Corticosteroids may be
Downloaded from https://onlinelibrary.wiley.com/doi/ by National Institute Of Standard, Wiley Online Library on [06/03/2024]. See the Terms and Conditions (https://onlinelibrary.wiley.com/terms-and-conditions) on Wiley Online Library for rules of use; OA articles are governed by the applicable Creative Commons License
Chapter 51 Henoch–Schönlein Purpura Nephritis 221

more effective in treating joint pain. Corticosteroids are also Patients may require antihypertensives to treat hyperten-
used to treat abdominal pain. sion, including angiotensin-converting enzyme (ACE) inhi-
Though rare, surgery may be needed to treat intussuscep- bitors and angiotensin receptor blockers (ARBs). These
tion or to determine the cause of swollen testicles, if present medications have proven effective in slowing the progres-
in a male. sion of kidney disease. Many people require two or more
medications to control their blood pressure. In addition
How would you determine if the therapy was to an ACE inhibitor or an ARB, diuretics may also be
effective? warranted. Beta-blockers, calcium channel blockers, and
other blood pressure medications may also be needed.
There will be resolution of symptoms and a decrease in
Blood and urine tests are used to check the kidney func-
erythrocyte sedimentation rate (ESR), CRP and IgA levels.
tion of patients with HSP for at least 6 months after the
main symptoms disappear.
What other therapies would be recommended
for Henoch–Schönlein purpura?
Would you obtain any other diagnostic studies?
HSP that affects the kidneys may be treated with cortico-
steroid and immunosuppressive medications. Immunosup- An abdominal or renal ultrasound may be required to rule
pressive medications prevent the body from making out other causes of abdominal pain and to check for possi-
antibodies. Patients with severe acute kidney failure are ble complications such as a bowel obstruction. A renal
treated with high-dose corticosteroids and the immunosup- biopsy may be useful in distinguishing between HSP and
pressive cyclophosphamide (Cytoxan). IgA nephropathy.

Further Reading
Dillon, M.J. and Ozen, S. (2016). A new international Pohl, M. (2015). Henoch–Schönlein purpura nephritis.
classification of childhood vasculitis. Pediatric Nephrology 21 Pediatric Nephrology 30(2): 245–252. doi: 10.1007/s00467-
(9): 1219–1222. 014-2815-6
Downloaded from https://onlinelibrary.wiley.com/doi/ by National Institute Of Standard, Wiley Online Library on [06/03/2024]. See the Terms and Conditions (https://onlinelibrary.wiley.com/terms-and-conditions) on Wiley Online Library for rules of use; OA articles are governed by the applicable Creative Commons License
223

52

Toddler with Tachypnea and Cool Extremities


Cathy Woodward
Long School of Medicine, University of Texas Health San Antonio, San Antonio, TX, USA

A 3-year-old previously healthy toddler presented to the extremities. His abdomen is soft and his liver is palpated
emergency department (ED). Caregivers report cool 2 cm below the right costophrenic angle. Vital signs include
extremities, rapid breathing for past 3 days, and he is axillary temperature 37 C (98.6 F), heart rate 150 bpm, reg-
refusing to eat today. ular rhythm, blood pressure 72/48 mmHg, and oxygen
saturations 92% on room air.
A complete metabolic panel, complete blood count, and
History of Present Illness capillary blood gas analysis are obtained and the results are
shown in Tables 52.1–52.3. Additional investigations to
This 3-year-old child developed symptoms 3 days ago when help differentiate an infectious or inflammatory cause for
caregivers noticed the toddler breathing fast and not as illness include blood and urine cultures, respiratory viral
active as normal. Over the past 3 days he has been sleeping polymerase chain reaction (PCR) nasal swab, cardiac tropo-
more, is irritable and only taking small amounts from his nins, C-reactive protein, and erythrocyte sedimentation
sippy cup. It is reported that the patient had a “cold” about rate (ESR), with results as follows.
2 weeks ago with fever, cough and congestion which lasted
approximately 7 days. The family feels the toddler has not • Serum cardiac troponin I: 8.0 ng/mL (normal <2.0
ng/mL).
returned to baseline activity since that illness.
•• C-reactive protein: 25 mg/L (normal 0–10 mg/L).
ESR: 35 mm/hour (normal 0–10 mm/hour).

Past Medical History • Respiratory viral PCR nasal swab: positive for
adenovirus.

He was born at term and went home with his mother. All
required vaccines along with annual influenza vaccines are Questions
up to date. Weight and height are at the 80th and 85th per-
centiles for his age and gender. His does not take any reg- Answer the following questions using the details provided.
ular medications. No previous surgeries. 1 Which other diagnostic studies would be appropriate
to order?
2 Based on the data given, what is the differential diagno-
Current Status sis and the most likely diagnosis for this child?
3 What is the best way to improve this child’s work of
On arrival at the ED, the toddler responds by crying and breathing?
weakly pulling away. His initial respiratory rate is 4 What medications might improve his condition in
48 breaths per minute with mild subclavicular retractions, the ED?
and coarse crackles bilaterally. No wheezing is noted. His 5 What other therapies or medications would be appro-
extremities are cool to the touch, with 1+ pulses in all priate on admission to the pediatric intensive care unit?

Cases in Pediatric Acute Care: Strengthening Clinical Decision Making, First Edition. Edited by Andrea M. Kline-Tilford and Catherine M. Haut.
© 2020 John Wiley & Sons Ltd. Published 2020 by John Wiley & Sons Ltd.
Downloaded from https://onlinelibrary.wiley.com/doi/ by National Institute Of Standard, Wiley Online Library on [06/03/2024]. See the Terms and Conditions (https://onlinelibrary.wiley.com/terms-and-conditions) on Wiley Online Library for rules of use; OA articles are governed by the applicable Creative Commons License
224 Cases in Pediatric Acute Care

Table 52.1 Basic metabolic profile. Based on the data given, what is the differential
diagnosis and the most likely diagnosis for
Sodium 141 mEq/L this child?
Potassium 3.8 mEq/L
The differential includes myocarditis, community-acquired
Chloride 101 mEq/L
pneumonia, acute respiratory distress syndrome, cardiomy-
Carbon dioxide 15 mEq/L
opathy, and sepsis.
Blood urea nitrogen 24 mg/dL The most likely diagnosis for this child is myocarditis sec-
Creatinine 0.6 mg/dL ondary to recent viral respiratory illness. Myocarditis is an
Glucose 86 mg/dL inflammatory disease of the myocardial muscle, and in the
United States the most common cause of myocarditis is
viral infection. Some viruses associated with myocarditis
are adenovirus, enterovirus, parvovirus, human herpesvi-
Table 52.2 Complete blood count.
rus, hepatitis C, Epstein–Barr virus (EBV), and cytomegalo-
White blood cell count 10 000/mm3 virus (CMV). In other countries, and less commonly in the
Hemoglobin 13 g/dL United States, the causes include autoimmune disease,
Chagas disease, diphtheria, hypersensitivity reactions,
Hematocrit 40%
and toxins. The clinical presentation of myocarditis in chil-
Platelet count 346 000/mm3
dren is varied, from children presenting with relatively mild
Neutrophils 79% symptoms to those in cardiogenic shock with dysrhyth-
Eosinophils 6% mias. A mild early case of myocarditis may be missed as
Monocytes 6% the symptoms mimic respiratory illnesses.
Lymphocytes 9%
Bands 0% What is the best way to improve this child’s work
of breathing?
Assistive support is indicated. A non-invasive strategy pro-
Table 52.3 Capillary blood gas analysis. vides an ideal stepwise approach. He is placed on 6 L/min of
50% oxygen via high-flow nasal cannula. Oxygen satura-
pH 7.30 tions improve to 96% and work of breathing is improved,
PCO2 44 mmHg with a respiratory rate of 30 breaths per minute, and min-
PaO2 60 mmHg imal subclavicular retractions and bilateral crackles.
Bicarbonate 16 mEq/L
Base excess –5 What medications might improve his condition
in the ED?
Lactate 2.5 mmol/L
Cardiac failure is suspected so an intravenous dose of furo-
semide 1 mg/kg is given to reduce intravascular volume
load and workload on the heart. A single intravenous dose
Rationale and Evidence-based Practice of ceftriaxone is administered, while awaiting culture
Explanation results, before the child is transferred to the pediatric inten-
sive care unit.
Which other diagnostic studies would
be appropriate to order? What other therapies or medications would be
appropriate on admission to the pediatric
Cardiac troponins and echocardiogram are ordered
intensive care unit?
because his non-febrile respiratory findings and liver
enlargement suggests a cardiac cause for his failure. On admission to the pediatric intensive care unit, his heart
A chest radiograph report demonstrates mild pulmonary rate is 150 bpm, blood pressure 65/35 mmHg, and oxygen
edema. An echocardiogram demonstrates reduced left ven- saturation 94% on 6 L/min of 50% oxygen via high-flow
tricular ejection, mild left ventricular dilation, and reduced nasal cannula. Decreasing the workload on the heart begins
septal wall function. An electrocardiogram (EKG) reveals with rest, oxygen therapy and inotropes to improve diastolic
low-voltage QRS complexes and mild non-specific ST–T filling, systolic function and cardiac output. A 5 mL/kg
wave abnormalities. bolus of normal saline is administered over several
Downloaded from https://onlinelibrary.wiley.com/doi/ by National Institute Of Standard, Wiley Online Library on [06/03/2024]. See the Terms and Conditions (https://onlinelibrary.wiley.com/terms-and-conditions) on Wiley Online Library for rules of use; OA articles are governed by the applicable Creative Commons License
Chapter 52 Toddler with Tachypnea and Cool Extremities 225

minutes. A milrinone infusion is started at 0.3 μg/kg per function. Milrinone, a phosphodiesterase 3 inhibitor, is
min and an epinephrine infusion at 0.03 μg/kg per min. an effective medication for improving cardiac filling during
The child is placed nil by mouth and intravenous fluids diastole, improving systolic function and reducing after-
are started at 100 mL/kg daily. A central line is placed to pro- load. Epinephrine can be used as an adjunct to milrinone
vide venous samples for laboratory blood draws and to to improve inotropy and systolic function. If this medical
administer medications. A cardiac magnetic resonance study management is started early it may be possible to avoid
is ordered. intubation. If he was experiencing worsening cardiac fail-
The gold standard diagnostic test for myocarditis is endo- ure, cardiac dysrhythmias or worsening respiratory failure,
myocardial tissue biopsy; however, obtaining the sample is his myocardial workload could be reduced by administra-
not without risk and can be delayed if the child improves on tion of sedation, muscle relaxation and intubation. On
medical therapies. A recent study has demonstrated the occasion, children with severe heart failure myocarditis
possible value of cardiac magnetic resonance to help in who show no sign of improvement may require heart
the determination of myocarditis in children with signs transplantation.
and symptoms of cardiac infarction (i.e. chest pain, EKG
changes and increased cardiac biomarkers). This diagnostic
test could be considered in order to avoid endomyocardial
biopsy. Case Resolution
Oxygen will allow the child to maintain normal oxygen
saturations. This child was hypotensive perhaps due Over the next few hours, the child’s condition stabilizes. His
partially to poor oral intake for the past few days leading metabolic acidosis improves and the epinephrine and
to dehydration, but also due to poor cardiac output associ- oxygen are weaned. The toddler remains extubated and
ated with myocardial dysfunction. A volume bolus of 5 mL/ on milrinone infusion for several days as myocardial
kg can be administered while closely monitoring effect as function improves, as demonstrated by echocardiogram,
the chest radiograph and respiratory assessment were improved blood gases, and improvement in clinical signs
concerning for pulmonary congestion due to poor cardiac and symptoms.

Further Reading
Butts, R., Boyle, G., Deshpande, S., et al. (2017). Characteristics Martinez-Villar, M., Gran, F., Sabate-Rotes, A., et al. (2018).
of clinically diagnosed pediatric myocarditis in a Acute myocarditis with infarct-like presentation in a
contemporary multi-center cohort. Pediatric Cardiology 38: pediatric population: Role of cardiovascular magnetic
1175–1182. resonance. Pediatric Cardiology 39(1): 51–56.
Canter, C. and Simpson, K. (2014). Diagnosis and treatment of
myocarditis in children in the current era. Circulation 129:
115–128.
Downloaded from https://onlinelibrary.wiley.com/doi/ by National Institute Of Standard, Wiley Online Library on [06/03/2024]. See the Terms and Conditions (https://onlinelibrary.wiley.com/terms-and-conditions) on Wiley Online Library for rules of use; OA articles are governed by the applicable Creative Commons License
227

53

Teen with Intractable Headache


Tara Pezzuto
Nemours/Alfred I. duPont Hospital for Children, Wilmington, DE, USA

A 13-year-old girl diagnosed at age 11 years with intermit- feeding, vision and hearing screening were normal, with
tent migraine headaches at a frequency of four to five times no congenital anomalies, no cardiac concerns, and no infec-
per month presents to the emergency department (ED) tion or fever. She achieved developmental milestones on
with an intractable constant headache for 1 week, diplopia time and is doing very well on the honor roll at school.
intermittently for 3 days, and nausea. She is overweight with a body mass index (BMI) of 29.47;
she is in the 95th percentile for weight and the 27th percen-
tile for height.
History of Present Illness

She has a history of anxiety, migraine with aura, acne, and Past Surgical History
exercise-induced asthma. She is not taking any prevention
medications for migraine, is receiving counseling for anxi- No overnight hospitalization or any surgeries.
ety but is not on any anxiolytics. She uses her albuterol
inhaler as needed for her asthma. Three weeks ago she
was started on minocycline for her acne. She has been Family History
doing well with her headache hygiene: drinking ade-
quately, sleeping adequately, eating well and three meals No history of epilepsy, cerebral palsy, muscular dystrophy,
a day, limiting screen time, and exercising routinely. She neurofibromatosis, tuberous sclerosis, autism, schizophre-
has tried ibuprofen 400 mg every 8 hours for 1 week with nia, Tourette syndrome, multiple sclerosis, or developmen-
no relief. While she reports her typical migraine features tal delay. Positive family history for migraine in mother and
of frontal head pain, photophobia, phonophobia, nausea maternal grandmother, kidney stones in paternal uncle,
and vertigo, she is now also complaining of seeing double hypertension on paternal side of family, high cholesterol
and is bumping into walls when she attempts to walk. on maternal side of family, and stroke in maternal grand-
She has no fever or history of trauma. She has no history mother at age 65 years and in maternal aunt at 40 years.
of recent acute illness. She has been going through a signif- There is celiac disease noted in another maternal aunt
icant growth spurt and parents report an increase in appe- and fibromyalgia in paternal aunt. Anxiety and depression
tite. She has gained 4.5–6.8 kg in the last 4–6 months. are common on paternal side of the family and a maternal
cousin was diagnosed with bipolar disorder.

Past Medical History


Current Status
She was full-term to a 36-year-old gravida 3, para 2, miscar-
riage 1 mother via cesarean section due to fetal stress and The patient is currently complaining of head pain (7/10 on
prolonged labor. Her birthweight was 3.09 kg and her numeric scale) with photophobia and phonophobia. She is

Cases in Pediatric Acute Care: Strengthening Clinical Decision Making, First Edition. Edited by Andrea M. Kline-Tilford and Catherine M. Haut.
© 2020 John Wiley & Sons Ltd. Published 2020 by John Wiley & Sons Ltd.
Downloaded from https://onlinelibrary.wiley.com/doi/ by National Institute Of Standard, Wiley Online Library on [06/03/2024]. See the Terms and Conditions (https://onlinelibrary.wiley.com/terms-and-conditions) on Wiley Online Library for rules of use; OA articles are governed by the applicable Creative Commons License
228 Cases in Pediatric Acute Care

nauseated and is having trouble walking because of diplo-


pia. Her examination includes appropriate vital signs for
her age. She is awake, alert and oriented, answering ques-
tions appropriately. Her pupils were equal, round and reac-
tive to light. Her fundoscopic examination suggests cloudy
disk margins and her finger-to-nose exercise is inaccurate.
She moves all extremities with full range of motion; she has
normal muscle bulk, strength and tone. Deep tendon
reflexes are +3 in all extremities. She has difficulty with
tandem walking and Romberg test. She follows a five-step
command but struggles with visual acuity. She is resting in
an emergency room with the lights off. An intravenous
catheter has been placed and normal saline is infusing at
maintenance rates, she was given ondansetron 4 mg intra-
venously, and the neurology team has been consulted.
Neurology suggests an ophthalmologic consult with vis-
ual acuity testing, dilated fundoscopic examination, and
optic nerve imaging. Bilateral papilledema is noted on
examination, while visual acuity testing indicates visual
field cuts in bilateral peripheral vision. Magnetic resonance
imaging (MRI) of the brain with and without contrast as
Figure 53.1 MRI of brain.
well as magnetic resonance venography (MRV) is ordered.
The imaging suggests a normally structured brain
except for enlargement of the optic nerves suggesting Questions
papilledema. Neurology reviews the images and suggests
a lumbar puncture for opening pressure, and cerebrospinal Answer the following questions using the details provided.
fluid (CSF) for protein, glucose, white blood cell count, red
blood cell count, viral panel, Lyme titer and an extra vial on 1 Based on the data, what are the differential diagnoses
hold for possible further testing. and the most likely diagnosis for this child?
Results of CSF analysis are shown in Table 53.1 and MRI 2 What is the best way to treat this problem and how
brain scan in Figure 53.1. Analysis of MRI scan found intra- would you order this?
cranial hypertension, narrowed anterior superior sagittal 3 What other therapies would be recommended at
sinus and straight sinus, and patent deep and superficial this time?
cerebral venous drainage without evidence of current
thrombosis, with normal intracranial magnetic resonance
angiography.
Rationale and Evidence-based Practice
Other results showed opening pressure of 46 cmH2O and
viral panel negative.
Explanation

Based on the data, what are the differential


diagnoses and most likely diagnosis for this child?

Table 53.1 Cerebrospinal fluid analysis. The differential diagnoses include migraine with aura,
pseudotumor cerebri syndrome (PTCS), Lyme meningitis,
Reference range Result viral encephalitis, venous sinus thrombosis, and tumor.
Lyme meningitis, viral encephalitis, tumor, and venous
Color Colorless sinus thrombosis were excluded based on the results of lab-
Clarity Unknown Clear oratory testing, diagnostic testing, and focused clinical
RBC manual count <100/mm 3
5/mm3 examination. Lyme meningitis would have been suspected
WBC manual count 0–5/mm 3
1/mm3 if there was presence of an erythema migrans rash, head-
Glucose 40–75 mg/dL 53 mg/dL
ache lasting more than 7 days, CSF pleocytosis, or positive
Lyme titer. The presentation for viral encephalitis would
Protein 15–45 mg/dL 32 mg/dL
typically include other symptoms, including fever, ataxia,
Downloaded from https://onlinelibrary.wiley.com/doi/ by National Institute Of Standard, Wiley Online Library on [06/03/2024]. See the Terms and Conditions (https://onlinelibrary.wiley.com/terms-and-conditions) on Wiley Online Library for rules of use; OA articles are governed by the applicable Creative Commons License
Chapter 53 Teen with Intractable Headache 229

change in mental status, and possibly seizures. Also, she What other therapies would be recommended at
does not have MRI findings indicative of venous sinus this time?
thrombosis or tumor. She may have secondary migraine
Reoccurrence of PTCS is commonly associated with weight
with aura; however, the primary cause for increased head-
gain so many patients are referred to a nutritionist, a psy-
ache frequency is PTCS.
chologist and possibly a bariatric specialist for assistance.
She is diagnosed with PTCS, which is caused by the rapid
Studies have indicated that even a 5–10% weight loss signif-
overproduction of CSF. Visual loss is a potential complica-
icantly improved PTCS symptoms. Often children have
tion of this syndrome. The goal of treatment is preservation
other coexisting health problems that will need to be
of vision and alleviation of symptoms. Visual field testing,
assessed and treated, including obstructive sleep apnea,
dilated fundoscopic examination, and imaging of optic
anemia, hypertriglyceridemia, and fatty liver disease. In
disks are necessary for proper diagnosis and should be
advanced cases, neurosurgical consult will be needed for
included initially and at follow-up examinations every
possible ventricular shunt, stent or decompression.
2–6 weeks. Pseudotumor cerebri most often occurs in
Low-pressure headache is often a side effect of a lumbar
women of childbearing years who have had significant
puncture and can be avoided by lying flat for a period of
weight gain. When the BMI is over 40, the severity of papil-
time and transitioning to increased activity slowly. Low-
ledema and visual loss rises significantly. The diagnosis of
pressure headaches are positional headaches that are
PTCS is made with the assistance of the modified Dandy
decreased by lying flat but which are increased by sitting
criteria. These criteria include papilledema (and other signs
up or standing and can happen after a lumbar puncture.
of increased intraranial pressure), a normal neurologic
examination (except possible cranial nerve VI palsy), nor-
mal neurologic imaging and CSF composition, and an
opening lumbar puncture pressure greater than 28 cmH2O
Case Resolution
for children.
The patient was admitted to the hospital overnight while
she recovered from the lumbar puncture, making sure
What is the best way to treat this problem and how
she was lying flat for at least 4 hours and transitioning to
would you order this?
sitting and standing positions slowly after this. Her head-
Lumbar puncture is not only a diagnostic measure for PTCS ache resolved after the lumbar puncture. She was dis-
but also a treatment for it. Normal opening pressure for an charged to home the next day and started on
adult patient would be below 25 cmH2O; however, some acetazolamide 250 mg twice daily for 1 week and then
sedative agents can increase opening pressure and over- increased to 500 mg twice daily for 4 months. Her minocy-
weight patients can have a slightly elevated opening pres- cline was stopped and she was encouraged to increase activ-
sure by virtue of being overweight. In this case, the ity and healthy food choices. The family was requested to
minocycline was stopped because PTCS can be a side effect monitor for side effects and monitor papilledema with
of this medication. Initial treatment strategies should ophthalmologic examination every 2 weeks to 2 months
include weight reduction and a salt-restricted diet. In many until resolved. If any symptoms occurred again (e.g. blurry
instances, acetazolamide is used as the first-line pharmaco- vision, intractable headache or any other neurologic focal
logic therapy to reduce the risk of recurrence of PTCS. symptoms) they were advised to contact ophthalmology
A slow titration of acetazolamide is recommended to and neurology. Neurology will follow up monthly to mon-
reduce side effects. Alternates to acetazolamide include a itor the acetazolamide treatment and for any signs of
trial of topiramate or a loop diuretic. increased pressure.

Further Reading
Bell, S. (2016). Idiopathic intracranial hypertension Sheldon, C.A., Paley, G.L., Beres, S.J., and McCormack, S.E.
(pseudotumor cerebri). Journal of Neuroscience Nursing 48 (2017). Pediatric pseudotumor cerebri syndrome: diagnosis,
(6): 303–310. classification and underlying pathophysiology. Seminars in
Friedman, D., Grant, T.L., and Digre, K.B. (2013). Revised Pediatric Neurology 24(2): 110–115.
diagnostic criteria for pseudotumor cerebri syndrome in
adults and children. Neurology 81: 1159–1165.
Downloaded from https://onlinelibrary.wiley.com/doi/ by National Institute Of Standard, Wiley Online Library on [06/03/2024]. See the Terms and Conditions (https://onlinelibrary.wiley.com/terms-and-conditions) on Wiley Online Library for rules of use; OA articles are governed by the applicable Creative Commons License
231

54

Seizures in School-age Child


Ann Marie Felauer
University of Maryland School of Nursing, Baltimore, MD, USA

An 8-year-old female, weighing 35 kg, with a past history of 52% for gender and age since age 3, and her current body
febrile seizures and new diagnosis of epilepsy is admitted to mass index (BMI) is 19.3. She had her first simple febrile
the intermediate care unit for status epilepticus. seizure at 18 months. She continued to have febrile seizures
intermittently with illness, a total of nine by age 6 years.
When she was 7 years old she presented to her local com-
History of Present Illness munity ED after a brief seizure lasting less than 2 minutes
at home. This seizure was generalized tonic–clonic and was
Two days prior to arrival this child developed nausea and not associated with fever or illness at the time. She returned
had two episodes of non-bloody non-bilious emesis. She to her baseline neurologic status when she was seen in the
denies any diarrhea. Her brother and father had similar ED and her family was told to follow up with her primary
symptoms last week so her parents felt that she had gastro- care provider (PCP) and neurologist. Subsequent visits with
enteritis. She continued on her levetiracetam maintenance her PCP and neurology resulted in a diagnosis of seizure
seizure medication and over the last 24 hours was able to disorder and she was started on levetiracetam. No other
keep some clear liquids down. There were no reports of medical history. Recent visit to PCP for complaints of
fevers and she denies chills or rigors. She reported feeling growing pains.
tired and that she did not want to play on her computer.
On the day of presentation the patient was watching televi-
sion when it was noted that her head arched back and she Past Surgical History
stiffened and began full body shaking. Rectal diazepam
(Diastat) 10.5 mg was administered by her parents when No previous history of surgeries.
the activity continued for several minutes, and they called
911. When emergency medical services (EMS) arrived the
activity was continuing and lorazepam 2 mg was given
intravenously with eventual cessation of seizure activity. Family History
EMS transported her to the local children’s hospital emer-
gency department (ED) for further care and evaluation. In The girl’s mother has had hypothyroidism since adoles-
the ED, the child was post-ictal, but was able to maintain cence, her father has had type 1 diabetes mellitus since
her own airway. diagnosis at age 10, and she has a brother who was diag-
nosed as type 1 diabetes mellitus at age 5.

Past Medical History


Current Status
This patient was born at 38 weeks of gestation via normal
spontaneous vaginal delivery and was discharged home In the ED, the child is postictal, slow to arouse, but main-
with her mother. She has had all required vaccines along taining patent airway. Vital signs as follows: temperature
with annual influenza vaccines. Her weight has been at 38.1 C (100.6 F) axillary, heart rate 112 bpm, blood

Cases in Pediatric Acute Care: Strengthening Clinical Decision Making, First Edition. Edited by Andrea M. Kline-Tilford and Catherine M. Haut.
© 2020 John Wiley & Sons Ltd. Published 2020 by John Wiley & Sons Ltd.
Downloaded from https://onlinelibrary.wiley.com/doi/ by National Institute Of Standard, Wiley Online Library on [06/03/2024]. See the Terms and Conditions (https://onlinelibrary.wiley.com/terms-and-conditions) on Wiley Online Library for rules of use; OA articles are governed by the applicable Creative Commons License
232 Cases in Pediatric Acute Care

Table 54.1 Complete blood count. 3 Is hypoparathyroidism a common finding in children?


4 What therapies are used to treat hypocalcemia and
White blood cell count 9000/mm3
hypoparathyroidism?
Hemoglobin 12 g/dL
Hematocrit 36%
Platelet count 190 000/mm3 Rationale and Evidence-based Practice
Neutrophils 66% Explanation
Lymphocytes 22%
Monocytes 1% Are any other diagnostic studies recommended?
Eosinophils 10% Further studies to help determine the diagnosis include
measurement of serum vitamin D, 25-hydroxyvitamin
D and 1,25-dihydroxyvitamin D, as well as serum parathy-
Table 54.2 Basic metabolic profile. roid hormone (PTH), magnesium, and phosphorus levels. It
is also important to check levels of levetiracetam; if low,
Sodium 141 mEq/L
this could have been the reason for the seizure, but a low
Potassium 3.8 mEq/L calcium can also contribute to seizure activity.
Chloride 101 mEq/L There is a relationship between vitamin D and calcium.
Carbon dioxide 24 mEq/L Vitamin D is essential for utilization and absorption of dietary
Blood urea nitrogen 12 mg/dL calcium. In a vitamin D-deficient state, the amount of calcium
Creatinine 0.6 mg/dL absorbed is decreased, resulting in the increased production
Glucose 133 mg/dL and secretion of PTH. Tubular reabsorption of calcium in
the kidneys is decreased when PTH is low and increased with
Calcium 7.3 mg/dL
higher circulation of PTH. In a vitamin D-deficient state,
Ionized calcium 3.8 mg/dL
inadequate amounts of 1,25-dihydroxyvitamin D are pro-
duced, resulting in calcium production and utilization from
bone, which has other consequences. Another effect of higher
pressure 95/66 mmHg, and oxygen saturation 97% on 1 L/ levels of PTH is phosphaturia, resulting in hypophosphate-
min oxygen by nasal cannula. mia, where low levels of PTH will result in hyperphosphate-
Physical examination reveals the following. mia. Low calcium levels, associated with low PTH, also result

•• General: sleepy, slow to arouse, no acute distress.


Respiratory: clear breath sounds throughout, symmetric
in low magnesium levels.

chest rise, no wheezing. Based on the data given, what are the differential

• Cardiovascular: heart with regular rate/rhythm, no mur-


murs noted.
diagnoses and the most likely diagnosis for
this child?

•• Abdomen: soft, non-tender, bowel sounds hyperactive.


Musculoskeletal: full movement, normal strength
Differential diagnoses for this child include hypocalcemia
due to hypoparathyroidism, hyperparathyroidism, or vita-
and tone.


min D deficiency. The most likely diagnosis for this child
Neurologic: waking slowly, Glasgow Coma Scale (GCS) is hypocalcemia due to hypoparathyroidism. Both parents
score 12 at this time, no focal deficits. have endocrine disorders, so this could be a familial risk
factor. Hypocalcemia can be seen as a consequence of
A complete blood count, basic metabolic profile and severe or chronic vitamin D deficiency, but low serum cal-
ionized calcium were obtained and the results are shown cium is not usually associated with mild vitamin
in Tables 54.1 and 54.2. D deficiency. A low calcium level resulting from hypopar-
athyroidism can also lead to magnesium deficiency, so fur-
ther studies are needed to document the diagnosis.
Questions
Is hypoparathyroidism a common finding in
Answer the following questions using the details provided. children?
1 Are any other diagnostic studies recommended? Hypoparathyroidism is a rare disease in any age group, but
2 Based on the data given, what are the differential diag- especially so in children. It is characterized by low levels of
noses and the most likely diagnosis for this child? PTH and low levels of calcium with high circulating levels
Downloaded from https://onlinelibrary.wiley.com/doi/ by National Institute Of Standard, Wiley Online Library on [06/03/2024]. See the Terms and Conditions (https://onlinelibrary.wiley.com/terms-and-conditions) on Wiley Online Library for rules of use; OA articles are governed by the applicable Creative Commons License
Chapter 54 Seizures in School-age Child 233

of phosphorus. In children, hypoparathyroidism can result What therapies are used to treat hypocalcemia and
from an autoimmune syndrome, can be genetically hypoparathyroidism?
acquired, or can be transient, most commonly seen in the
A patient who presents with seizures as a result of hypocal-
newborn period. Hypoparathyroidism has been associated
cemia requires treatment with intravenous calcium, recom-
with DiGeorge syndrome or 22q11 chromosome microdele-
mended as calcium gluconate, with continuous cardiac
tion, can be X-linked recessive or familial or be associated
monitoring for arrhythmias in the intensive care unit. Once
with other genetic syndromes. Hypoparathyroidism can
calcium levels are near normal (at least 7.5 mg/dL), oral cal-
result from neck surgery, with damage to the parathyroid
cium can be administered along with calcitriol, the active
or thyroid gland or surgical removal of the parathyroid
form of 1,25-dihydroxyvitamin D, but the patient should
gland. Symptoms of hypoparathyroidism are the same as
continue to be monitored for 24 hours to ensure that the
those associated with hypocalcemia, including muscle
calcium is absorbed. Endocrinology referral and continued
spasm, muscle aches, seizures, and syncope.
care is also recommended.
The pathophysiology of hypoparathyroidism is complex.
Simply, PTH increases serum calcium and decreases serum Continuation of case: This 8-year-old girl is given cal-
phosphorus. PTH acts on both bone and kidneys to increase cium gluconate intravenously (1.86 mg elemental cal-
serum concentrations of 1,25-dihydroxyvitamin D, which cium per kilogram) over 10 minutes with a goal to
indirectly stimulates calcium and phosphorus reabsorp- correct serum calcium to above 7.5 mg/dL. She is then
tion. Hypothyroidism results in loss of both direct and indi- transferred to the pediatric intensive care unit (PICU)
rect effects of PTH, with effects on bone and kidney and and started on supplemental calcium with close monitor-
reabsorption of calcium and phosphorus, resulting in low ing of her electrocardiogram and frequent monitoring of
calcium and high phosphorus levels. her calcium levels.

Further Reading
Hassan-Smith, Z. and Gittoes, N. (2017). Hypocalcaemia. hypoparathyroidism in children, does not require
Medicine 55(9): 555–558. antiepileptic drugs. Childs Nervous System 33(2): 297–305.
Liu, M.J., Li, J.W., Shi, X.Y., Hu, L.Y., and Zou, L.P. (2017). Winer, K.K. (2019). Advances in the treatment of
Epileptic seizure, as the first symptom of hypoparathyroidism with PTH 1–34. Bone 120: 535–541.
Downloaded from https://onlinelibrary.wiley.com/doi/ by National Institute Of Standard, Wiley Online Library on [06/03/2024]. See the Terms and Conditions (https://onlinelibrary.wiley.com/terms-and-conditions) on Wiley Online Library for rules of use; OA articles are governed by the applicable Creative Commons License
235

55

An 18-Year-Old with Upper Respiratory Symptoms, Headache and Fever


Misty Evans
Vanderbilt College of Nursing, Nashville, TN, USA

An 18-year-old male presents with upper respiratory tract Family History


infection (URI) symptoms, nausea, headache, and neck
stiffness. Parents are alive and healthy with no contributory medical
history. Father of the teen does have hypertension, treated
with medication, and mother recently had a cold.
History of Present Illness

He reports having a sore throat and mild nasal congestion


over the past week. Today, he developed a severe headache,
Current Status
“the worst he had ever had.” Approximately 2 hours prior
Vital signs are as follows: temperature 37 C (98.6 F), heart
to admission, he developed severe headache, neck pain,
rate 120 bpm, respiratory rate 22 breaths per minute, and
and photophobia. He presents to the emergency depart-
blood pressure 89/36 mmHg. He appears clinically ill but
ment for further evaluation. He is a freshman college stu-
fully oriented. Nuchal rigidity was present on neurologic
dent who lives in the dormitory on campus. He has no
examination (positive Brudzinski and Kernig signs).
known sick contacts.
The results of basic metabolic profile, complete blood
count and coagulation profile are shown in Tables 55.1
and 55.2.
Past Medical History

The patient has no recent history of tick bites, viral illness,


or recent travel. No significant medical history. He reports Questions
that he believes all immunizations are up to date; how-
ever, he does recall that he recently received his first Answer the following questions using the details provided.
meningococcal vaccine and still needs to receive the sec-
1 What are the significant abnormalities noted on physi-
ond dose.
cal examination and laboratory results?
2 Based on the data given, what are the differential diag-
noses and the most likely diagnosis for this teenager?
Past Surgical History 3 Are other diagnostic studies needed?
4 What is the best initial therapy for this patient?
The teen had a circumcision at birth, but no other surgical 5 What other therapies would be recommended at
procedures. this time?

Cases in Pediatric Acute Care: Strengthening Clinical Decision Making, First Edition. Edited by Andrea M. Kline-Tilford and Catherine M. Haut.
© 2020 John Wiley & Sons Ltd. Published 2020 by John Wiley & Sons Ltd.
Downloaded from https://onlinelibrary.wiley.com/doi/ by National Institute Of Standard, Wiley Online Library on [06/03/2024]. See the Terms and Conditions (https://onlinelibrary.wiley.com/terms-and-conditions) on Wiley Online Library for rules of use; OA articles are governed by the applicable Creative Commons License
236 Cases in Pediatric Acute Care

Table 55.1 Basic metabolic profile. rigidity is the inability to place the chin to the chest and is
due to inflammation of the meninges. Abnormal laboratory
Sodium 138 mEq/L
results include normocytic anemia, thrombocytopenia,
Potassium 4 mEq/L prolonged prothrombin time (PT) and partial thromboplas-
Chloride 102 mEq/L tin time (PTT), elevated D-dimer, decreased fibrinogen,
Carbon dioxide 22 mEq/L elevated creatinine, hypocalcemia, hypomagnesemia, hypo-
Blood urea nitrogen 28 mg/dL phosphatemia, and elevation in transaminases. Thrombocy-
Creatinine 1.7 mg/dL topenia, prolonged PT and PTT, elevated D-dimer, and
Glucose 81 mg/dL
decreased fibrinogen are indicative of disseminated intra-
vascular coagulation (DIC) secondary to sepsis.
Calcium 7.8 mg/dL
Magnesium 1.2 mEq/L
Phosphorus 1.4 mg/dL Based on the data given, what are the differential
diagnoses and the most likely diagnosis for this
Aspartate transaminase 51 U/L
teenager?
Alanine transaminase 48 U/L
Alkaline phophatase 124 U/L The differential diagnoses include bacterial meningitis,
Gamma-glutamyltransferase 45 U/L viral meningitis, encephalitis, meningococcemia, focal
infections such as brain abscess, brain tumor, and infec-
Creatine kinase 181 U/L
tious thrombophlebitis. Because of immunization against
Lactate dehydrogenase 850 U/L
meningitis, the incidence of bacterial meningitis has
decreased, but it does still affect adults between the ages
Table 55.2 Complete blood count and coagulation profile. of 18 and 49. There are four main processes as the infection
invades the body: colonization, invasion in the blood-
White blood cell count 5500/mm3 stream, survival in the bloodstream, and then passage into
Hemoglobin 11.5 g/dL the subarachnoid space. The gold standard for diagnosing
Hematocrit 33% meningitis is evaluation of cerebrospinal fluid (CSF).
Platelet count 90 000/mm3 Table 55.3 includes normal results as compared to results
Prothrombin time (PT) 13.4 s
with bacterial and viral meningitis.
Fever, headache, and nuchal rigidity are typical
Partial thromboplastin time (PTT) 40 s
symptoms which occur with meningitis along with the
International normalized ratio 1.8
possibility of mental status changes. Infection can start
D-dimer 23 ng/mL with a fever and headache in a previously healthy individ-
Fibrinogen 100 mg/dL ual. Streptococcus pneumoniae, Neisseria meningitidis, and
Haemophilus influenzae type B are the most common
etiologies beyond the newborn period. Teenagers are
Rationale and Evidence-based Practice
specifically susceptible to Neisseria meningitidis and
Explanation Streptococcus pneumoniae. College campuses have
reported outbreaks of meningococcal disease caused by
What are the significant abnormalities noted on
N. meningitidis, which is also the organism found in bac-
physical examination and laboratory results?
terial meningitis. Profound alterations in consciousness
The physical examination reveals that this patient has are less common in viral meningitis than in bacterial men-
tachycardia, hypotension, and nuchal rigidity. Nuchal ingitis. Meningitis differs from other brain infections such

Table 55.3 Cerebrospinal fluid: typical results.

Opening WBC Predominant Protein


Appearance pressure (cmH2O) (/mm3) cell type (g/L) Glucose (mg/dL)

Normal Clear 10–20 <5 NA <0.4 2.6–4.5


Bacterial Cloudy, turbid, Raised >100 Lymphocytes Raised, Low
purulent >1.0
Viral Clear Normal or mildly Raised, Lymphocytes Mildly Normal or slightly
increased <1000 raised low
Downloaded from https://onlinelibrary.wiley.com/doi/ by National Institute Of Standard, Wiley Online Library on [06/03/2024]. See the Terms and Conditions (https://onlinelibrary.wiley.com/terms-and-conditions) on Wiley Online Library for rules of use; OA articles are governed by the applicable Creative Commons License
Chapter 55 An 18-Year-Old with Upper Respiratory Symptoms, Headache and Fever 237

as encephalitis, focal infections, and infectious thrombo- reaction (PCR) assays can detect meningococcal DNA in
phlebitis, which generally result in focal neurologic defi- CSF, plasma, and serum. Sensitivity and specificity are over
cits. The patient in this case had symptoms consistent 90%. PCR testing is more sensitive than blood cultures and a
with meningitis (nuchal rigidity and headache) in addition confirmatory diagnosis can be made in 4–8 hours. In this
to evidence of infection resulting in mild DIC. The most case, PCR testing for meningococcal disease should be sent
likely diagnosis for this patient is N. meningitidis bactere- on plasma or serum.
mia and meningitis. Neisseria meningitidis (meningococcal Serial laboratory monitoring will be needed in the acute
disease) is the leading cause of bacterial meningitis in chil- phase of illness. Blood glucose should be monitored closely
dren and young adults in the United States, with an over- due to alterations during stress.
all mortality rate of 10–25% related to the urgency of
treatment. Meningitis is often associated with striking
What is the best initial therapy for this patient?
morbidity including hearing loss, cognitive dysfunction,
visual impairment, educational difficulties, developmental Continuous respiratory and hemodynamic monitoring
delays, motor nerve deficits, seizure disorders, and behav- should be initiated. Antibiotics including cefotaxime, cef-
ioral problems. triaxone and penicillin are the preferred medications to
Teenagers living in crowded conditions such as dormi- treat meningitis, with penicillin as first-line therapy.
tories and barracks are at high risk of infection. The The New England Journal of Medicine has published a
highest incidence of disease occurs during the winter study which recommends the use of high-dose dexametha-
months when respiratory viral illnesses are at their peak. sone in cases of suspected bacterial meningitis before (ide-
Meningococci can colonize the upper respiratory tract of ally within 4 hours), and no longer than 12 hours following,
healthy individuals, and infection occurs by either direct the first dose of parenteral antibiotics and which should
contact or by respiratory droplets. Immunization with continue for 2–4 days (de Gans and van de Beek, 2002).
two vaccines at least 6 months apart offers immunity An elevated serum creatinine indicates acute kidney
to meningitis type A and a newer vaccine has been devel- injury in the presence of infection and septic shock. Phys-
oped to protect against meningitis caused by serotype B. ical examination also reveals hypotension and tachycardia.
The pathophysiology of meningitis includes entry of the Aggressive fluid replacement and vasopressor support is
bacteria typically through the respiratory system; the indicated for sepsis/shock. Laboratory findings suggest
bacteria often colonize the nasopharyngeal cavity and DIC in the setting of sepsis. PT, PTT and fibrinogen should
then progress to the bloodstream. The infection crosses be monitored serially. The coagulopathies require correc-
the blood–brain barrier and proliferates in the CSF, tion with fresh frozen plasma and cryoprecipitate.
resulting in meningeal inflammation and increased
intracranial pressure. Much of the damage caused by
What other therapies would be recommended
meningitis is believed to be caused by cytokines released
at this time?
within the CSF as the host mounts an inflammatory
response. Vascular collapse and shock are frequent early manifesta-
tions, predominantly caused by the bacterial toxin. Cardiol-
ogy involvement is essential due to evidence of myocardial
Are other diagnostic studies needed? dysfunction, which may be seen in sepsis. Correction of
electrolyte imbalance, volume resuscitation, and inotropic
Additional diagnostic studies include a blood culture to iso-
support may improve cardiac function. The clinical mani-
late the infectious pathogen, a chest radiograph to evaluate
festations of meningitis may vary in severity, ranging from
for underlying lung pathology, and echocardiogram to eval-
fever and bacteremia to fulminant disease with death
uate underlying function. Continuous respiratory and
occurring within hours of the onset of symptoms. Early
hemodynamic monitoring should be initiated.
treatment and aggressive supportive care is critical in the
Computed tomography (CT) of the brain is often used in
treatment of patients with meningococcal disease.
patients with neurologic changes or decreased level of con-
sciousness who are diagnosed with meningitis. It is fre- Continuation of case: The patient is admitted to the pedi-
quently recommended when there are broad or uncertain atric intensive care unit. On admission he was treated
differential diagnoses. Although a lumbar puncture is empirically for suspected meningitis with ceftriaxone ini-
standard of care for the evaluation and diagnosis of menin- tially, then with penicillin. An echocardiogram revealed
gitis and to assess intracranial pressure, this patient pre- mild left ventricular enlargement with decreased function,
sents with significant coagulopathy and may be at risk of and ejection fraction of 40% (normal 50–70%). A chest radi-
bleeding if a this procedure is pursued. Polymerase chain ograph demonstrates developing pulmonary edema.
Downloaded from https://onlinelibrary.wiley.com/doi/ by National Institute Of Standard, Wiley Online Library on [06/03/2024]. See the Terms and Conditions (https://onlinelibrary.wiley.com/terms-and-conditions) on Wiley Online Library for rules of use; OA articles are governed by the applicable Creative Commons License
238 Cases in Pediatric Acute Care

Reference
de Gans, J. and van de Beek, D. (2002). Dexamethasone Journal of Medicine 347: 1549–1556. doi: 10.1056/
in adults with bacterial meningitis. New England NEJMoa021334.

Further Reading
Mcgill, F., Heyderman, R.S., Panagiotou, S., et al. (2016). Nadel, S. (2016). Treatment of meningococcal disease. Journal
Acute bacterial meningitis in adults. Lancet 366: of Adolescent Health 59(2 Suppl): S21–S28. doi: 10.1016/j.
3036–3047. jadohealth.2016.04.013.
Downloaded from https://onlinelibrary.wiley.com/doi/ by National Institute Of Standard, Wiley Online Library on [06/03/2024]. See the Terms and Conditions (https://onlinelibrary.wiley.com/terms-and-conditions) on Wiley Online Library for rules of use; OA articles are governed by the applicable Creative Commons License
239

56

Testicular Torsion
Amanda Mandel
Nemours/Alfred I. duPont Hospital for Children, Wilmington, DE, USA

A 13-year-old male presents to the emergency department Current Status


(ED) with acute left testicular pain at 4 a.m.
The patient presents to the triage nurse in acute distress. He
is having difficulty walking and is hunched over and unable
History of Present Illness to stand up straight. His vital signs include temperature of
36.7 C (98 F), heart rate 110 bpm, blood pressure 100/
On arrival to the ED, the patient and his parents advise that 70 mmHg, and respiratory rate 20 breaths per minute.
he awoke at 2 a.m. with acute, sudden left testicular pain. His weight is 48 kg. Urine sample is taken and results of uri-
The pain was severe enough to wake him from sleep and nalysis are shown in Table 56.1.
contributed to vomiting. He also reports that the left side The triage nurse immediately notifies the ED attend-
of his scrotum is red, swollen and painful to touch. He finds ing physician of the teen’s symptoms and he is immedi-
it difficult to walk. His parents called his primary care pro- ately brought back to a room and examined. Most noted
vider’s after-hours number and they directed him immedi- on physical examination is his pain level and difficulty
ately to the ED. He denies dysuria and any trauma to the ambulating. Heart and lungs are normal. Abdominal
scrotum, and is not sexually active. He has never had this examination indicates normal bowel sounds, soft to pal-
pain before. pation, non-tender, without rebound or pain at McBur-
ney point. The genitourinary examination is significant
for left scrotal swelling, with significant erythema,
Past Medical History exquisite tenderness to palpation, and testis difficult to
palpate due to pain but appears descended and has more
The teen is an otherwise healthy male, born full term, and of a horizontal lie with absent cremasteric reflex. The
has received vaccines as required. He has no other signifi- right scrotum appears normal, with testis descended
cant medical health problems. and non-tender to palpation (see Figure 56.1). Tanner
scoring is level II.
On completion of the physical examination, the ED pro-
Past Surgical History vider promptly calls radiology for an immediate ultrasound
of the scrotum with Doppler and calls urology for an urgent
He was circumcised as a newborn without complications. consult. Results of the scrotal ultrasound with Doppler are
as follows.

Family History • Left testis: enlarged; measures 4.9 × 2.8 × 3.2 cm with a
calculated volume of 31.2 mL.

No significant family history. Parents are both alive, • Right testis: normal in size and echotexture with no focal
abnormality; measures 4.4 × 2.2 × 2.4 cm with a calcu-
healthy and deny any family history of bleeding or anesthe- lated volume of 16.5 mL.
sia problems.

Cases in Pediatric Acute Care: Strengthening Clinical Decision Making, First Edition. Edited by Andrea M. Kline-Tilford and Catherine M. Haut.
© 2020 John Wiley & Sons Ltd. Published 2020 by John Wiley & Sons Ltd.
Downloaded from https://onlinelibrary.wiley.com/doi/ by National Institute Of Standard, Wiley Online Library on [06/03/2024]. See the Terms and Conditions (https://onlinelibrary.wiley.com/terms-and-conditions) on Wiley Online Library for rules of use; OA articles are governed by the applicable Creative Commons License
240 Cases in Pediatric Acute Care

Table 56.1 Urinalysis.

Color Clear, yellow


Specific gravity 1.030
pH 7.0
Nitrite Negative
Ketones Negative
Urobilirubin Negative
Leukocytes Negative
Protein Negative
Glucose Negative
Blood Negative

Figure 56.2 Intraoperative testicular torsion. Source: Courtesy of


T. Ernesto Figueroa, MD, Nemours Alfred I. duPont Hospital for
Children, Wilmington, Delaware.

the testis appeared quite dusky but improved after the tor-
sion was reduced. The torsion was reduced and the testis
was fixated. The right testis was also fixated. At the end
of the case, the left testis looked to be perfusing well.

Questions

Answer the following questions using the details provided.


1 Based on the data given, what are the differential diag-
noses and most likely diagnosis for this child?
2 Why was the child instructed to go directly and urgently
to the ED for assessment?
3 What additional diagnostic testing would be
considered?
4 When would urology be consulted?
5 What are the key educational points we need to discuss
with families regarding testicular torsion?
6 What is the long-term prognosis of boys with testicular
torsion?
Figure 56.1 External scrotal examination. Source: Courtesy of
T. Ernesto Figueroa, MD, Nemours Alfred I. duPont Hospital for
Children, Wilmington, Delaware.
Rationale and Evidence-based Practice


Explanation
Normal blood flow and normal testicle on the right. Left
testis enlarged with no Doppler blood flow (venous or Based on the data given, what are the differential
arterial) to the testis. diagnoses and most likely diagnosis for this child?
The urology team was present during the ultrasound and A patient who presents with unilateral testicular pain, usu-
examined the patient and he was taken emergently to the ally with nausea and vomiting, should be managed imme-
operating room for scrotal exploration. Surgery was started diately, with clinical suspicion for acute testicular torsion
at 5 a.m., 3 hours after the onset of symptoms. The left tes- requiring immediate surgical exploration. If imaging is
ticle was found to have a 360 torsion (Figure 56.2). Initially not available, it is unnecessary to wait for an ultrasound
Downloaded from https://onlinelibrary.wiley.com/doi/ by National Institute Of Standard, Wiley Online Library on [06/03/2024]. See the Terms and Conditions (https://onlinelibrary.wiley.com/terms-and-conditions) on Wiley Online Library for rules of use; OA articles are governed by the applicable Creative Commons License
Chapter 56 Testicular Torsion 241

before calling surgery. The use of bedside ultrasound in the should also be done to rule out an infectious cause for
ED is helpful with diagnosis but requires a skilled provider the pain.
to obtain images and assess results. Testicular torsion can
occur at any age, but is more common in adolescent males. When would urology be consulted?
Differential diagnoses for this child include testicular tor-
sion, torsion of the appendix testis, and epididymitis. The In patients with high suspicion of testicular torsion, urology
most likely diagnosis is testicular torsion where the testis should be called immediately after assessment in the ED.
twists on itself. Testicular torsion presents as acute sudden This will help expedite emergent surgery if it is warranted.
onset of significant testis pain. It will often wake a child up
from sleep and can cause vomiting. Torsion of the appendix What are the key educational points we need to
testis can also cause significant scrotal swelling, redness discuss with families regarding testicular torsion?
and pain. Epididymitis would be unusual in a child who It is very important that boys and their families be educated
is not sexually active, but can affect any age boy and on testicular torsion during well child examinations. Four
involves inflammation of the epididymis and causes pain out of ten young men who have testicular torsion will lose a
and inflammation of the testicles. It is always important testicle, often due to a delay in evaluation. It is important to
to rule out testicular torsion prior to accepting the diagnosis stress to families that prompt evaluation is needed when-
of epididymitis, which is usually bacterial in origin and ever a boy has complaints of scrotal pain. Boys should be
treated with antibiotics. encouraged to tell their parents immediately if they ever
have testis pain. It is also important to perform a scrotal
examination on any boy who is complaining of abdominal
Why was the child instructed to go directly and pain. Boys may not always endorse scrotal pain and an
urgently to the ED for assessment? examination is vital to make sure testicular pathology is
Time is of the essence in boys with testicular torsion as not missed.
there is no blood supply to the testis. The longer the testis
is not perfused, the less chance there is of being able to sur- What is the long-term prognosis of boys with
gically restore circulation. Any male patient who has testis testicular torsion?
pain needs emergent evaluation.
Patients who have had testicular torsion will need contin-
ued urologic follow-up after their surgery. The viability of
What additional diagnostic testing would be
the affected testis long term will depend on the duration
considered?
of ischemia. There is a risk of testicular atrophy in patients
Scrotal ultrasound with Doppler is the primary diagnostic with testicular torsion that has been surgically corrected.
test to evaluate a patient with testis pain. It is needed to The affected testis is not always salvageable in the operating
assess the blood flow to the testis and helps differentiate room and an orchiectomy sometimes has to be performed.
torsion from a torsion of the appendix testis (which is Patients with solitary testis must use a protective scrotal cup
self-limiting and will resolve on its own). A urinalysis during sports.

Further Reading
Bowlin, P.R., Gatti, J.M., and Murphy, P. (2017). Pediatric across United States pediatric emergency
testicular torsion. Surgical Clinics of North America 97: departments. American Journal of Emergency Medicine
161–172. 36(2): 208–212.
Lee, L.K., Monuteaux, M.C., Hudgins, J.D., et al. (2018).
Variation in the evaluation of testicular conditions
Downloaded from https://onlinelibrary.wiley.com/doi/ by National Institute Of Standard, Wiley Online Library on [06/03/2024]. See the Terms and Conditions (https://onlinelibrary.wiley.com/terms-and-conditions) on Wiley Online Library for rules of use; OA articles are governed by the applicable Creative Commons License
243

57

Teen with Fever and Rash


Christine Renke1 and Elizabeth Hoppe2
1
C.S. Mott Children’s Hospital, Ann Arbor, MI, USA
2
Ann and Robert H. Lurie Children’s Hospital of Chicago, Chicago, IL, USA

A previously healthy 16-year-old female presents to the Family History


emergency department (ED) with a 2-week history of tactile
fever and rash with progression to abdominal pain and She was adopted at 3 weeks of age and therefore the family
emesis yesterday. history is limited. Father with hypertension and high cho-
lesterol. Paternal grandfather with heart disease. No known
history of cancer. Mother’s medical history unknown.
History of Present Illness

She was in her previous state of health until approximately


Social History
2 weeks ago, when she developed a fever followed by a rash.
She was taking intermittent doses of acetaminophen with
Lives at home with adoptive parents and adopted sister. She
some relief. She describes her rash as limited to her nose
attends school and is in the 10th grade. Denies smoking,
and cheeks. It was dry and red in appearance. She has been
alcohol, or illicit drug use. Reports sexual activity with
applying lotion without change in the rash. She also reports
one partner.
puffy eyes and believes this may be related to her rash. She
denies any new lotions, shampoos or soaps, or laundry
detergents.
The day prior to presentation she reports non-bloody,
Current Status
non-bilious emesis, and was only able to keep some fluids
On arrival to the ED, she is non-toxic in appearance. Her
down. She denies diarrhea and reports a normal stooling
initial vital signs are as follows: temperature 38.3 C
pattern. She reports that she has been voiding less than nor-
(100.9 F) orally, heart rate 118 bpm, blood pressure
mal and her urine is darker than normal. She denies dysu-
140/88 mmHg, oxygen saturation 99% on room air, and res-
ria. No sick contacts and reports overall low energy
piratory rate 14 breaths per minute. She has generalized,
throughout the course of her symptoms.
non-focal tenderness and does not have hepatosplenome-
galy. You note a red rash localized to her face and extending
Past Medical History from cheek to cheek across the bridge of her nose with
nasolabial sparing. She has mild periorbital edema.
She has exercise-induced asthma and eczema, and takes Otherwise, her examination is unremarkable.
Her initial laboratory values are shown in
albuterol as needed for cough and wheezing. No history
Tables 57.1–57.3.
of hospitalizations.

Past Surgical History Questions

No surgical history. Answer the following questions using the details provided.

Cases in Pediatric Acute Care: Strengthening Clinical Decision Making, First Edition. Edited by Andrea M. Kline-Tilford and Catherine M. Haut.
© 2020 John Wiley & Sons Ltd. Published 2020 by John Wiley & Sons Ltd.
Downloaded from https://onlinelibrary.wiley.com/doi/ by National Institute Of Standard, Wiley Online Library on [06/03/2024]. See the Terms and Conditions (https://onlinelibrary.wiley.com/terms-and-conditions) on Wiley Online Library for rules of use; OA articles are governed by the applicable Creative Commons License
244 Cases in Pediatric Acute Care

Table 57.1 Basic metabolic profile. 3 Based on the initial laboratory evaluation, clinical
examination, and vital signs, what findings are
Sodium 135 mEq/L
significant?
Potassium 4.2 mEq/L 4 What specialty service(s) should be consulted to be
Chloride 104 mEq/L involved in her care?
Carbon dioxide 20 mEq/L
Continuation of case: She is admitted to the general pedi-
Blood urea nitrogen 20 mg/dL
atric floor due to her ongoing emesis and inability to remain
Creatinine 1.7 mg/dL hydrated with oral intake. It is noted that her blood pres-
Glucose 109 mg/dL sure remains elevated, with systolic blood pressure sus-
Alkaline phosphatase 92 U/L tained at 130 mmHg. The rest of her examination
Aspartate aminotransferase 26 U/L remains stable. The rheumatology evaluation reveals ele-
Alanine aminotransferase 34 U/L vated erythrocyte sedimentation rate (ESR), positive anti-
Albumin 2.5 g/dL nuclear antibody (ANA), positive anti-double-stranded
DNA, positive anti-Smith, positive anti-phospholipid anti-
Total protein 6.7 g/dL
body, direct Coombs positive, and low C3 and C4. Labora-
C-reactive protein 0.4 mg/dL
tory results ultimately diagnose her with systemic lupus
erythematosus (SLE) with suspected lupus nephritis.

Table 57.2 Complete blood count. 5 Are any other diagnostic studies warranted?
6 What interventions would you implement?
White blood cell count 3800/mm3
Hemoglobin 10.2 g/dL
Hematocrit 30.6%
Platelet count 164 000/mm3
Rationale and Evidence-based Practice
Neutrophils 69%
Explanation
Eosinophils 4%
What initial differential diagnoses should be
Monocytes 7%
considered?
Lymphocytes 9%
Given the non-specific presentation, differential diagnoses
Bands 0%
are broad and include viral gastroenteritis with dehydra-
Monocytes 6%
tion, nephrotic syndrome, pyelonephritis, urinary tract
infection, contact dermatitis, and autoimmune disorders.

Table 57.3 Urinalysis. What initial diagnostics test should be obtained?

Color Straw The initial differential diagnoses are broad, and therefore
pH 6.2 your initial diagnostic testing is also broad. You should
obtain a urinalysis, given hypertension and concern for uri-
Specific gravity 1.025
nary tract infection (UTI) or nephrotic syndrome, as well as
Clarity Cloudy
a complete metabolic panel to assess blood urea nitrogen
Glucose Negative
and creatinine as markers of renal function and hydration
Ketones Trace status. Finally, a complete blood count with differential can
Leukocytes Trace quickly help to assess for signs of infection, hematologic
Nitrites Negative disease, or abnormalities indicative of chronic disease.
Occult blood Moderate
Protein Large Based on the initial laboratory evaluation, clinical
examination, and vital signs, what findings are
significant?
1 What initial differential diagnoses should be The abnormal laboratory values of concern include leuko-
considered? penia with lymphocytopenia, mild anemia, elevated creat-
2 What initial diagnostics test should be obtained? inine, hypoalbuminemia, proteinuria, and hematuria.
Downloaded from https://onlinelibrary.wiley.com/doi/ by National Institute Of Standard, Wiley Online Library on [06/03/2024]. See the Terms and Conditions (https://onlinelibrary.wiley.com/terms-and-conditions) on Wiley Online Library for rules of use; OA articles are governed by the applicable Creative Commons License
Chapter 57 Teen with Fever and Rash 245

Table 57.4 Revised criteria for classification of SLE.

Criterion Definition

Hematologic Anemia defined as >2SD below the mean for age and sex
Leukopenia: white blood cell count <4000/mm3
Thrombocytopenia: platelet count <100 000/mm3
Lymphopenia <1500/mm3
Hemolysis
Mucocutaneous Oral or nasopharyngeal ulceration, usually painless, observed by provider
Non-scarring alopecia
Subacute cutaneous or discoid lupus
Malar rash or maculopapular rash
Musculoskeletal Non-erosive arthritis or synovitis involving two or more peripheral joints, characterized by tenderness,
swelling, or effusion
Pleural or pericardial Imaging evidence of pleural effusion, or
effusion Pericarditis: documented by EKG or imaging evidence of pericardial effusion
Renal disorder Persistent proteinuria >0.5 g/day or
Cellular casts: may be red cell, hemoglobin, granular, tubular, or mixed
Class II, III, IV or V lupus nephritis on renal biopsy
Neurologic disorder Seizures: in the absence of offending drugs or known metabolic derangements, or
Psychosis: in the absence of offending drugs or known metabolic derangements
Delirium
Fever Temperature >38.3 C
Immunologic disorders Anti-DNA: antibody to native DNA in abnormal titer, or
Anti-Smith (Sm): presence of antibody to Sm nuclear antigen, or
Positive findings of anti-phospholipid antibodies on:

••
an abnormal serum level of IgG or IgM anti-cardiolipin antibodies


a positive test result for lupus anticoagulant using a standard method
or a false-positive test for at least 6 months confirmed by Treponema pallidum immobilization or
fluorescent treponemal antibody absorption test
Anti-nuclear antibodies ANA titer ≥1 : 80 or an equivalent positive test at least once
Complement Low C3 or C4 or low C3 and C4

SD, standard deviation.

The sustained hypertension and fever are abnormal vital Rheumatology 1997 Revised Criteria for Classifications of
signs. On examination, her malar rash and periorbital SLE (Table 57.4).
edema are also significant. The laboratory findings suggest This patient had the following criteria for diagnosis of
that the most likely diagnosis is an autoimmune disorder SLE: malar rash, persistent proteinuria, leukopenia, lym-
with renal disease. phopenia, thrombocytopenia, positive anti-double-
stranded DNA, and positive ANA test. She also presented
with evidence of lupus nephritis, including hematuria, pro-
What specialty service(s) should be consulted to teinuria, hypertension, and acute kidney injury.
be involved in her care?
When autoimmune disorders are suspected, the rheumatol- Are any other diagnostic studies warranted?
ogy service should be consulted. Begin an autoimmune
Hypocomplementemia adds to the suspicion for lupus
evaluation and obtain ESR and ANA. Additional rheuma-
nephritis, so renal biopsy is the gold standard for diagnosis,
tologic tests may include anti-double-stranded DNA, anti-
classifying degree of nephritis (I–VI), and guiding therapy.
Smith, anti-phospholipid antibody, direct Coombs, and
complement with C3 and C4. With the concern for renal
What interventions would you implement?
disease coupled with hypertension, the nephrology team
should be consulted. Renal involvement with SLE occurs in approximately 75%
The diagnosis of SLE is based on clinical manifestations of children, usually within first two years of disease. Spe-
and laboratory testing. Diagnosis of SLE is defined by meet- cific treatment is formulated for each patient based on
ing four of eleven criteria based on the American College of organ involvement. Sunscreen is advised for all SLE
Downloaded from https://onlinelibrary.wiley.com/doi/ by National Institute Of Standard, Wiley Online Library on [06/03/2024]. See the Terms and Conditions (https://onlinelibrary.wiley.com/terms-and-conditions) on Wiley Online Library for rules of use; OA articles are governed by the applicable Creative Commons License
246 Cases in Pediatric Acute Care

patients to prevent exacerbation and non-steroidal anti- (MMF) is second-line therapy (class III–V) and allows
inflammatory drugs (NSAIDs) help with fevers and arthri- hopeful reduction in steroid resistance. Class VI SLE
tis. With renal involvement, glucocorticoids are the first- nephritis may require dialysis and renal transplant. Current
line therapy. Multiple side effects, including hypertension, research is being conducted regarding benefits of autolo-
hyperglycemia, and immune suppression, have led to gous stem cell transplant for refractory SLE. Treatment of
improved steroid-dosing techniques such as pulse doses hypertension secondary to SLE nephritis and steroid use
or every other day doses. Immunosuppressive therapy includes low salt diet and antihypertensives.
including cyclophosphamide and mycophenolate mofetil

Further Reading
Aringer, M., Costenbader, K., and Daikh, D. (2019). 2019 Collaborating Clinics classification criteria for systemic
European League Against Rheumatism/American College lupus erythematosus. Arthritis and Rheumatism 64(8): 2677–
of Rheumatology classification criteria for systemic lupus 2686. doi: 10.1002/art.34473.
erythematosus. Arthritis and Rheumatology 71(9): 1400– Wenderfer, S.E. and Eldin, K.W. (2019). Lupus nephritis.
1412. doi: 10.1002/art.40930. Pediatric Clinics of North America 66(1): 87–99. doi: 10.1016/
Petri, M., Orbai, A.-M., Alarcón, G.S., et al. (2012). Derivation j.pcl.2018.08.007.
and validation of the Systemic Lupus International
Downloaded from https://onlinelibrary.wiley.com/doi/ by National Institute Of Standard, Wiley Online Library on [06/03/2024]. See the Terms and Conditions (https://onlinelibrary.wiley.com/terms-and-conditions) on Wiley Online Library for rules of use; OA articles are governed by the applicable Creative Commons License
247

58

Preterm Infant Unable to Wean from Mechanical Ventilation


Michelle Dokas
Children’s Hospital of Michigan, Wayne State University, Detroit, MI, USA

This is a 10-day-old infant born at 26 weeks’ gestation to a Oxygen saturations were 92%. Most recent vital signs are
gravida 1 para 0 mother after spontaneous rupture of mem- temperature 37.5 C (99.5 F), heart rate 182 bpm, respiratory
branes. Birthweight was 620 g, current weight is 635 g. rate 74 breaths per minute, and blood pressure 72/28 mmHg.
Efforts were made to wean the ventilator towards extubation
with recurrent development of respiratory acidosis, tachyp-
History of Present Illness nea, and increased work of breathing. Most recent laboratory
results are shown in Tables 58.1–58.3 and most recent chest
The infant is intubated in an Isolette, is hemodynamically radiograph in Figure 58.1.
stable, and has a right arm peripherally inserted central
catheter in place through which she is receiving total par-
enteral nutrition. Questions

Answer the following questions using the details provided.


Past Medical History
1 Based on the data given, what are the differential diag-
Apgar scores were 3 and 7 at 1 and 5 minutes, respectively. noses and the most likely diagnosis for this child?
Amniotic fluid was noted to be clear on delivery. The infant 2 What additional testing would you order?
was intubated at birth. The mother’s membranes had 3 What are the treatment options for this diagnosis?
ruptured 14 hours before birth. Blood and respiratory 4 How would you determine if the therapy was effective?
cultures sent at birth were negative. Mother had received
betamethasone prior to delivery.
Rationale and Evidence-based Practice
Explanation
Family History
Based on the data given, what are the differential
Mother is 28 years old, married and received prenatal care. diagnoses and the most likely diagnosis for
She denies drug or alcohol use during pregnancy. She this child?
smokes one and a half packs of cigarettes per day, and con-
tinued through pregnancy. No sexually transmitted dis- Differential diagnoses include viral pneumonia, broncho-
eases noted, and no significant past medical history. pulmonary dysplasia (BPD), PDA, congenital heart disease,
and meconium aspiration.
The most likely diagnosis for this neonate is PDA. This
Current Status infant is very premature. By day of life 7, 65% of neonates
born at 25–28 weeks’ gestation have failure of the PDA to
The infant was intubated on the following ventilator settings: close. In addition, her PDA is hemodynamically significant
FiO2 0.45, rate 50 breaths per minute, pressure control based on her widened pulse pressure, elevated creatinine,
10 cmH2O, pressure support 8 cmH2O, PEEP 4 cmH2O. and inability to wean off the ventilator.

Cases in Pediatric Acute Care: Strengthening Clinical Decision Making, First Edition. Edited by Andrea M. Kline-Tilford and Catherine M. Haut.
© 2020 John Wiley & Sons Ltd. Published 2020 by John Wiley & Sons Ltd.
Downloaded from https://onlinelibrary.wiley.com/doi/ by National Institute Of Standard, Wiley Online Library on [06/03/2024]. See the Terms and Conditions (https://onlinelibrary.wiley.com/terms-and-conditions) on Wiley Online Library for rules of use; OA articles are governed by the applicable Creative Commons License
248 Cases in Pediatric Acute Care

Table 58.1 Basic metabolic profile. PDA occurs when the ductus arteriosus present in the
fetal circulation fails to close 48–72 hours after birth. The
Sodium 138 mEq/L
persistent blood flow from the aorta to the pulmonary
Potassium 4.1 mEq/L artery consists of a left-to-right shunt through the duct. This
Chloride 112 mEq/L leads to pulmonary over-circulation as the pulmonary vas-
Carbon dioxide 28 mEq/L cular pressures drop after birth, and an essential “flooding”
Blood urea nitrogen 22 mg/dL of the lungs. This increased flow to the lungs leads to a
Creatinine 1.1 mg/dL “steal” from the systemic circulation causing decreased
Glucose 78 mg/dL
blood flow to the mesentery and renal systems. This is man-
ifested by either low systolic and diastolic blood pressures
or adequate systolic pressure with low diastolic pressure
(widened pulse pressure). Complications from this left-
to-right shunting include necrotizing enterocolitis (NEC),
Table 58.2 Complete blood count. renal failure, pulmonary hemorrhage, IVH, and BPD.
White blood cell count 18 000/mm3
What additional testing would you order?
Hemoglobin 12 g/dL
Hematocrit 32.2% An echocardiogram to confirm the presence of the PDA, the
Platelet count 221 000/mm 3 size of the shunt (e.g. communication), and the direction of
blood flow between the vessels is required. The PDA can
only be ligated if the flow in the shunt is left to right, indi-
cating excessive blood flow from the systemic system to the
lungs. Right-to-left shunting of blood flow indicates high
Table 58.3 Arterial blood gas analysis. pulmonary pressures (greater than systemic) and is a con-
traindication to ligation. A respiratory viral panel to rule
pH 7.28 out infection can also be considered, although this diagno-
PaCO2 59 mmHg sis is unlikely given the absence of temperature instability.
PaO2 86 mmHg
Bicarbonate 29 mEq/L What are the treatment options for this diagnosis?
Lactate 1.6 mmol/L Pharmacologic therapy comprising cyclooxygenase inhibi-
Oxygen saturation 92% tors is the first-line intervention for PDA closure. Indo-
methacin and ibuprofen are the most studied agents.
However, indomethacin has been associated with NEC,
intestinal perforation, renal impairment, and platelet dys-
function. Other studies have investigated the use of ibupro-
fen and, more recently, acetaminophen. Ibuprofen also has
nephrotoxic effects and has been associated with pulmo-
nary hypertension, hyperbilirubinemia, and retinopathy
of prematurity. Surgical intervention with ductus ligation
can be used as primary intervention or as a secondary inter-
vention after failure of pharmacologic therapy. It provides
rapid and complete ductal closure, and is done at the bed-
side at most institutions in the United States. It usually
requires marked escalation of intensive care support in
the immediate postoperative period. Long-term complica-
tions of surgical intervention include left vocal cord paresis,
left hemidiaphragm paresis, chylothorax, and increased
incidence of BPD. An additional option for some infants
is transcatheter closure. This therapy is performed in the
cardiac catheterization laboratory and negates the need
for surgical closure. Venous access is established and an
Figure 58.1 Chest radiograph. occlusion device is placed directly in the PDA. Typically,
Downloaded from https://onlinelibrary.wiley.com/doi/ by National Institute Of Standard, Wiley Online Library on [06/03/2024]. See the Terms and Conditions (https://onlinelibrary.wiley.com/terms-and-conditions) on Wiley Online Library for rules of use; OA articles are governed by the applicable Creative Commons License
Chapter 58 Preterm Infant Unable to Wean from Mechanical Ventilation 249

this is done in adults, children, and infants over 6 months of


age; however, research continues on using this method in
very low birthweight infants. Another management is
non-intervention in very low birthweight infants with
hemodynamically significant PDA since these babies may
have less incidence of BPD compared to infants undergoing
medical or surgical closure. Spontaneous closure rates are
low, and some infants require mechanical closure at later
ages. The incidence of NEC, IVH, and mortality is not sig-
nificantly different from that in earlier intervention. Very
limited evidence exists that the consequences of ductal
dependency over time can be ignored. When watchful wait-
ing is selected, careful fluid management and judicious use
of diuretics is needed.

How would you determine if the therapy was


effective?
Clinical signs of PDA closure include an increase in dias- Figure 58.2 Post-surgical chest radiograph.
tolic pressure, resulting in a narrower pulse pressure and
improvement in blood pressure. These changes can be
noted immediately after the surgical clip is applied. These
left-to-right shunting. Her follow-up creatinine level was
changes are also noted with successful medical treatment.
1.3 mg/dL and she continued with widened pulse pressure.
A gradual improvement in renal function is also noted after
Pediatric cardiovascular surgery was consulted and surgical
PDA closure because of improved systemic blood flow and
ligation was performed at the bedside. Her blood pressure
less pulmonary blood flow. Ventilator settings may be
immediately after the surgical clip was applied to the PDA
weaned as the left-to-right shunting is eliminated and pul-
was 80/38 mmHg with oxygen saturations of 95%. The
monary over-circulation is reduced. The chest radiograph
infant required increased ventilator settings the first night
after removal of the left-to-right shunt also demonstrated
after surgery but was subsequently weaned and extubated
improvement (Figure 58.2).
to non-invasive positive pressure ventilation 1 week post-
Continuation of case: The infant was given one dose of operatively. Her respiratory support was gradually weaned,
indomethacin. A repeat head ultrasound revealed a grade feedings were initiated and successfully advanced, she
II intraventricular hemorrhage (IVH), and repeat echocar- gained weight nicely, and was discharged home in stable
diogram revealed large patent ductus arteriosus (PDA) with condition by her due date.

Further Reading
Benitz, W.E. (2016). Patent ductus arteriosus in preterm premature infants weighing less than 1200g. Archives of
infants. Pediatrics 137(1): e20153730. doi: 10.1542/ Disease in Childhood, Fetal and Neonatal Edition 103: F198–
peds.2015-3730. F201. doi: 10.1136/archdischild-2016-312582.
Mitra, S., Florez, I., Tamayo, M., et al. (2016). Effectiveness and Prescott, S. and Keim-Malpass, J. (2017). Patent ductus
safety of treatments used for the management of patent arteriosus in the preterm infant. Advances in Neonatal Care
ductus arteriosus (PDA) in preterm infants: a protocol for a 17(1): 10–18.
systematic review and network meta-analysis. BMJ Open 6 Sung, S., Chang, Y., Chun, J., et al. (2016). Mandatory closure
(7): e011271. doi: 10.1136/bmjopen-2016-011271. versus nonintervention for patent ductus arteriosus in very
Morville, P., Douchin, S., Bouvaist, H., and Dauphin, C. (2018). preterm infants. Journal of Pediatrics 177: 66–71.
Transcatheter occlusion of the patent ductus arteriosus in
Downloaded from https://onlinelibrary.wiley.com/doi/ by National Institute Of Standard, Wiley Online Library on [06/03/2024]. See the Terms and Conditions (https://onlinelibrary.wiley.com/terms-and-conditions) on Wiley Online Library for rules of use; OA articles are governed by the applicable Creative Commons License
251

59

Toddler with Acute Respiratory Failure


Ahmed Aly and Bradley Tilford
Children’s Hospital of Michigan, Detroit, MI, USA

Patient is a 22-month-old boy with a history of reactive air- was administered continuous albuterol and steroids during
way disease. He presents with acute respiratory failure with that admission.
hypoxia and hypercarbia requiring intubation and mechan- Immunizations are up to date for age including annual
ical ventilation. influenza vaccine. He is taking albuterol as needed and
montelukast daily. He has no known drug allergies.

History of Present Illness


Past Surgical History
Patient is a 22-month-old boy who developed low-grade
fever, occasional sneezing, and cough 2 days prior to admis- None.
sion. His mother has been giving him albuterol treatments
twice daily. Mother reports that early this evening after din-
ner, the family sat down on the couch for movie night with Social History
popcorn and pretzels. After a short time, the patient lost
interest in the movie and began playing with the family Lives with mother, maternal grandmother, and older sister.
dog. He abruptly began gasping for air while climbing over No smoke exposure at home. Two dogs at home.
the dog onto the coffee table. Mother reports she responded
quickly by hitting the patient on the back. He began to
cough, but continued to have labored breathing, became Current Status
very agitated, and his lips turned blue. The patient went
limp while mother attempted to give him another albuterol On arrival in the emergency department (ED), the patient is
treatment. He was unresponsive to voice and his eyes rolled hypoxic with an oxygen saturation of 65% on 100% oxygen
into the back of his head. Mother immediately called the via non-rebreather mask. He has poor respiratory effort and
emergency medical service (EMS) who arrived in approxi- a capillary blood gas analysis demonstrated pH of 7.1 and
mately 10 minutes. EMS found him to be cyanotic and PCO2 of 67 mmHg. The child is immediately intubated. Fol-
hypoxic, with gasping respirations and a bounding pulse lowing intubation, a basic metabolic panel, complete blood
on arrival to the scene. They provided 100% oxygen via count, and portable chest X-ray are obtained. The results of
non-rebreather mask and transported him to the nearest the laboratory investigations are shown in Tables 59.1 and
hospital. 59.2 and the chest radiograph in Figure 59.1.

Past Medical History Questions

Answer the following questions using the details provided.


Admitted to the pediatric intensive care unit (PICU) at
13 months of age for viral pneumonia and wheezing, 1 Based on the data given, what are the differential diag-
requiring non-invasive positive-pressure ventilation. He noses and the most likely diagnosis for this child?

Cases in Pediatric Acute Care: Strengthening Clinical Decision Making, First Edition. Edited by Andrea M. Kline-Tilford and Catherine M. Haut.
© 2020 John Wiley & Sons Ltd. Published 2020 by John Wiley & Sons Ltd.
Downloaded from https://onlinelibrary.wiley.com/doi/ by National Institute Of Standard, Wiley Online Library on [06/03/2024]. See the Terms and Conditions (https://onlinelibrary.wiley.com/terms-and-conditions) on Wiley Online Library for rules of use; OA articles are governed by the applicable Creative Commons License
252 Cases in Pediatric Acute Care

Table 59.1 Basic metabolic profile. Initial ventilator settings of synchronized intermittent
mandatory ventilation (SIMV) pressure control mode, rate
Sodium 138 mEq/L
34 breaths per minute, pressure control 19 cmH2O, pres-
Potassium 3.5 mEq/L sure support 10 cmH2O, PEEP 5 cmH2O, inspiratory time
Chloride 105 mEq/L 0.7 seconds, FiO2 1.0. Initial vital signs as follows: heart rate
Carbon dioxide 20 mEq/L 135 bpm, respiratory rate 34 breaths per minute, blood pres-
Blood urea nitrogen 4 mg/dL sure 108/60 mmHg, and SpO2 100% on mechanical
Creatinine 0.32 mg/dL ventilation.
Glucose 140 mg/dL
His physical examination is significant for normal right-
sided breath sounds, but diminished breath sound on the
left, with mild expiratory wheezing. He is started on nebu-
lized continuous albuterol at 10 mg/hour and methylpred-
Table 59.2 Complete blood count.
nisolone 2 mg/kg daily. Ceftriaxone is started for suspected
White blood cell count 20 600/mm3 acute bacterial pneumonia. Because of the unusual findings
Hemoglobin 10.9 g/dL on lung examination and chest radiograph, otolaryngology
Hematocrit 33.9% (ENT) was consulted. A bedside flexible bronchoscopy
demonstrated complete occlusion of the left mainstem
Platelet count 393 000/mm3
bronchus with secretions and a visible, circular foreign
Neutrophils 27%
body. Child was subsequently taken for rigid bronchoscopy
Monocytes 7% and removal of foreign body. The foreign body was found to
Lymphocytes 63% be a popcorn kernel. It was successfully removed from left
mainstem bronchus on the first attempt. The child was
brought back to the PICU intubated. Albuterol, methypred-
nisolone, and ceftriaxone were stopped. He received dexa-
methasone for airway edema. He was successfully weaned
from mechanical ventilation and extubated 24 hours
following foreign body removal. He was discharged home
the following day in stable condition.
5 Would you use flexible versus rigid bronchoscopy in
evaluating a child with suspected foreign-body
aspiration?
6 Would you obtain other diagnostic studies?
7 Would you continue steroids and antibiotic treatment
following foreign body removal?

Rationale and Evidence-based Practice


Explanation

Figure 59.1 Portable chest radiograph of foreign body. Based on the data given, what are the differential
diagnoses and the most likely diagnosis for
this child?
2 What is the pathophysiology associated with this
Differential diagnoses for the cause of acute respiratory fail-
diagnosis?
ure in this child include status asthmaticus, pneumonia,
3 What are the most common presenting signs and symp-
and foreign body aspiration.
toms of this diagnosis?
The most likely diagnosis for this child is foreign body
4 What are the radiologic features of this diagnosis?
aspiration due to the sudden presentation of cough,
Continuation of case: Child is admitted to the pediatric hypoxia, and gasping for air following eating popcorn.
intensive care unit and placed on mechanical ventilation. The abnormalities localized to the left lung field on physical
Downloaded from https://onlinelibrary.wiley.com/doi/ by National Institute Of Standard, Wiley Online Library on [06/03/2024]. See the Terms and Conditions (https://onlinelibrary.wiley.com/terms-and-conditions) on Wiley Online Library for rules of use; OA articles are governed by the applicable Creative Commons License
Chapter 59 Toddler with Acute Respiratory Failure 253

examination and chest X-ray are consistent with this diag- What are the radiologic features of this diagnosis?
nosis. In asthma, there is often a history of recurrent symp-
The most common feature is lung atelectasis. Other signs
toms with good response to bronchodilators and a positive
include air trapping, lung consolidation, and shifting of
family history. Wheezing is typically audible in all lung
the mediastinum. A radiopaque foreign body is only visible
fields. Patients with pneumonia may have a history of fever
in 20% of cases.
and respiratory distress, but no history of choking or acute-
onset symptoms. Physical examination and chest radio-
graph findings may be similar to foreign body aspiration. Would you use flexible versus rigid bronchoscopy
Pneumonia may occur as a complication of foreign body in evaluating a child with suspected foreign-body
inhalation. Foreign body aspiration into the airway is a aspiration?
common and dramatic pediatric emergency. It is most com-
The tracheobronchial tree should be examined in all cases
mon in children aged 6 months to 5 years of age, predom-
with a moderate or high suspicion of foreign body aspira-
inantly in children under 2 years of age. Immaturity of the
tion, typically using rigid bronchoscopy so that the object
swallowing mechanism, small caliber of the upper airways,
can be safely removed. Flexible rather than rigid bronchos-
curiosity of young children that is expressed by putting a
copy may be used for diagnostic purposes in cases in which
new object into the mouth, and an immature mastication
the diagnosis is unclear, or if the foreign body is known but
system are all factors in the occurrence of foreign body
the location of the object is unclear. In this case, most cen-
aspiration.
ters also arrange to have rigid bronchoscopy available on
standby because this is the preferred approach for foreign
What is the pathophysiology associated with this
body removal.
diagnosis?
The pathophysiology depends on the type of foreign body
Would you obtain other diagnostic studies?
and location in the airway. The object itself may cause air-
way obstruction. A foreign body causing total obstruction Foreign body aspiration is a medical emergency. Securing
of the trachea/larynx may cause respiratory failure, car- the airway is the top priority. Proximally located foreign
diac arrest, and death within minutes. Acute obstruction bodies may be removed during direct laryngoscopy. Neck
at the level of the carina or mainstem bronchus may cause and chest radiographs (both inspiratory and expiratory
lung collapse and hypoxemia. Pooled secretions caused by films) and urgent ENT consult should be obtained once
partial obstruction may progress to acute bacterial pneu- the airway is secured. Fluoroscopy and computed tompgra-
monia. Vegetable matter may swell over hours or days phy (CT) do not improve diagnostic accuracy compared to
leading to worsened airway obstruction. Organic foreign history, physical examination, and chest radiograph. CT
bodies may produce inflammation and edema. A sharp scan can be useful for diagnosing radiolucent foreign
foreign body may lead to erosion, as is the case with pins. bodies.
If foreign body aspiration is excluded from the differen-
What are the most common presenting signs tial diagnosis, then continued evaluation for viral or bacte-
and symptoms of this diagnosis? rial illness is warranted. With the 2-day history of viral
prodrome in our patient scenario, that evaluation may
A clear history of aspiration (choking event) is the most fre-
include viral nasal swabs/washes. Blood culture and
quent presenting symptom. Complete obstruction can pres-
C-reative protein may be obtained to evaluate for bacterial
ent with:
illness. Blood gas analysis may be required in patients

• severe respiratory distress, inability to speak or cough,


cyanosis, altered mental status;
with respiratory distress.

• universal choking sign.


Partial obstruction can present with:
Would you continue steroids and antibiotic
treatment following foreign body removal?

• cough, tachypnea, stridor, focal/generalized mono-


phonic wheezing or decreased air entry, regional varia-
No medications are routinely needed. If significant swelling
is observed in the airway or if granulation tissue is present,
tion in aeration; a corticosteroid (e.g. prednisolone, dexamethasone) may be

• “classic triad” of wheeze, cough, diminished breath


sounds.
administered. Unless airway secretions are infected, anti-
biotics are not necessary.
Downloaded from https://onlinelibrary.wiley.com/doi/ by National Institute Of Standard, Wiley Online Library on [06/03/2024]. See the Terms and Conditions (https://onlinelibrary.wiley.com/terms-and-conditions) on Wiley Online Library for rules of use; OA articles are governed by the applicable Creative Commons License
254 Cases in Pediatric Acute Care

Further Reading
Boufersaoui, A., Smati, L., Benhalla, K.N., et al. (2013). Foreign Lowe, D.A., Vasquez, R., and Maniaci, V. (2015). Foreign body
body aspiration in children: experience from 2624 patients. aspiration in children. Clinical Pediatric Emergency Medicine
International Journal of Pediatric Otorhinolaryngology 77 16(3): 140–148.
(10): 1683–1688. doi: 10.1016/j.ijporl.2013.07.026. Shlizerman, L., Mazzawi, S., Rakover, Y., and Ashkenazi, D.
Committee on Injury, Violence, and Poison Prevention (2010). (2010). Foreign body aspiration in children: the effects of
Prevention of choking among children. Pediatrics 125(3): delayed diagnosis. American Journal of Otolaryngology 31
601–607. doi: 10.1542/peds.2009-2862. (5): 320–324. doi: 10.1016/j.amjoto.2009.03.007.
Donato, L., Hong Tran, T.M., Ghori, U.K., and Musani, A.L. Yang, C., Hua, R., Xu, K., et al. (2015). The role of 3D computed
(2017). Pediatric interventional pulmonology. Clinics in tomography (CT) imaging in the diagnosis of foreign body
Chest Medicine 39(1): 229–238. doi: 10.1016/j. aspiration in children. European Review for Medical and
ccm.2017.11.017. Pharmacological Sciences 19(2): 265–273.
Downloaded from https://onlinelibrary.wiley.com/doi/ by National Institute Of Standard, Wiley Online Library on [06/03/2024]. See the Terms and Conditions (https://onlinelibrary.wiley.com/terms-and-conditions) on Wiley Online Library for rules of use; OA articles are governed by the applicable Creative Commons License
255

60

Adolescent with Head Injury


Jennette Firlein
Nemours/Alfred I. duPont Hospital for Children, Wilmington, DE, USA

The patient is a 16-year-old male hockey player who pre- He has allergies to amoxicillin, cefdinir (Omnicef ) and
sents to the emergency department (ED) with a head injury. lactose.

History of Present Illness Past Surgical History


He was playing ice hockey 4 hours ago when he was Colonoscopy and biopsy 2 years ago. Circumcision at birth
elbowed in the right jaw and hit the left side of his head with revision (ureteral meatus opening) at 1 year of age.
on the boards at the hockey arena. He was wearing a helmet
at the time of injury, and there was no loss of consciousness
(LOC). Initial symptoms included dizziness and loss of bal-
Family History
ance. He was removed from the ice a few minutes later as
dizziness persisted. After the game, he experienced epi-
Mother with scoliosis and constipation, maternal grand-
staxis briefly. He experienced some numbness in his hands
mother with IBS, sister with gastroesophageal reflux, father
and feet that lasted about 20 minutes while transported to
with migraines, and brother with bipolar disorder.
the ED, but this resolved prior to arrival.

Social History
Past Medical History
Attends 11th grade, is an A/B student, and participates in
A concussion 2 years prior required 3 months to recover.
ice hockey year-round. No developmental delays, learning
Also has a history of irritable bowel syndrome (IBS), growth
disabilities, anxiety, depression, sleeping difficulties,
delay issues, pediatric migraine (none since 6 years of age),
speech therapy, or attention deficit disorder. Lives with
and asthma.
his mother, father, brother, and sister.
Current medications include:

•• amitriptyline (Elavil) 10 mg daily in the evening;


cyproheptadine (Periactin) 4 mg daily in the evening; Current Status
• tretinoin 0.05% cream applied to affected area daily at
bedtime; Review of systems is positive for epistaxis, dizziness, photo-

•• anastrozole 1-mg tablet daily;


albuterol (ProAir) 90 μg per two puffs as needed every
phobia, visual disturbance (peripheral blurred vision), nau-
sea, neck pain, headache (mainly in the frontal region), and
4 hours for wheezing; decreased concentration. He also complains of right clavi-

•• Lactaid 1-mg tablet as needed;


hyoscyamine 0.125-mg tablet as needed for cramping or
cle pain but has full range of motion in his right arm. He
received acetaminophen for his headache and used ice
diarrhea. for his sore neck during transport to the hospital. Glascow

Cases in Pediatric Acute Care: Strengthening Clinical Decision Making, First Edition. Edited by Andrea M. Kline-Tilford and Catherine M. Haut.
© 2020 John Wiley & Sons Ltd. Published 2020 by John Wiley & Sons Ltd.
Downloaded from https://onlinelibrary.wiley.com/doi/ by National Institute Of Standard, Wiley Online Library on [06/03/2024]. See the Terms and Conditions (https://onlinelibrary.wiley.com/terms-and-conditions) on Wiley Online Library for rules of use; OA articles are governed by the applicable Creative Commons License
256 Cases in Pediatric Acute Care

Figure 60.1 C-spine three-view radiograph.

Coma Scale (GCS) score is 15 on arrival. He is placed in a likely secondary to rotation of the C-spine during the radi-
cervical collar with complaints of neck pain and cervical ographic evaluation. They recommend follow-up with neu-
spine (C-spine) tenderness. Ibuprofen 600 mg oral tablet rosurgery as an outpatient. An hour later, the patient begins
and a C-spine three-view radiograph are ordered on his to complain of increased right-sided facial pain despite ace-
arrival. taminophen and ibuprofen. He is also reporting increasing
Vital signs are temperature 36.9 C (98.4 F), heart rate head pressure focal to the left temporal region. One hour
89 bpm, respiratory rate 16 breaths per minute, and blood later, he is in tears with increased left facial pain. There
pressure 113/60 mmHg. His weight is 56.7 kg. are no new neurologic deficits noted.
The C-spine radiograph (Figure 60.1) showed that the The patient is experiencing persistent and worsening
lateral distance between the C1 arch and the dens is larger headache and jaw pain. Neuroimaging is ordered due to
on the left than on the right, appreciated on the odontoid increasing intensity of neurologic symptoms despite appro-
(open mouth) view. The radiologist recommends further priate management. Additionally, further imaging of the
evaluation with computed tomography (CT) examination. cervical spine was recommended from X-ray results. Fur-
ther imaging included CT of head, facial bones, and cervical
spine, with results as follows.
Questions

Answer the following questions using the details provided.


1 Based on the data given, what are the differential diag-
• CT head/brain without contrast and three-dimensional
reconstruction: no acute intracranial abnormality, no
visualized calvarial fracture.

2
noses and the most likely diagnosis for this child?
Are any additional diagnostic studies indicated at
this time?
• CT cervical spine without contrast but with three-
dimensional rendering: redemonstration of minimal
asymmetry between the lateral masses of C1 and the dens
Continuation of case: The patient continued with nausea as described of unclear clinical significance. This finding
and pain. He was given acetaminophen 975 mg oral tablet may be within normal limits and may be positional. Cor-
and ondansetron (Zofran) 4 mg oral tablet. The trauma relate with physical examination and point tenderness. If
team is consulted and evaluates the patient due to neck film there is continued clinical concern for ligamentous
results. Trauma team recommends neurosurgery consult. injury, MRI may be obtained. No evidence of fracture,
Neurosurgery reviews the case, stating the C-spine is most jumped facets, or rotatory subluxation.
Downloaded from https://onlinelibrary.wiley.com/doi/ by National Institute Of Standard, Wiley Online Library on [06/03/2024]. See the Terms and Conditions (https://onlinelibrary.wiley.com/terms-and-conditions) on Wiley Online Library for rules of use; OA articles are governed by the applicable Creative Commons License
Chapter 60 Adolescent with Head Injury 257

• CT facial bones/mandible without contract but with


three-dimensional rendering: normal non-contrast CT
headache, cervicalgia, cervical spine injury, and facial frac-
ture. The most likely diagnosis is mild TBI/concussion with
of the facial bones. concomitant cervicalgia and/or cervical spine injury. The
symptom report is consistent with concussion or mild
Neurosurgery reviews the CT, stating results were sec- TBI, so this case should not be classified as head injury only.
ondary to a neck spasm and rotation of C1 on the dens, with There was a report of trauma that resulted in rapid acceler-
no further evaluation warranted. The case is discussed with ation of the brain through a hit to the face and a direct hit of
the trauma team; given intractable pain and nausea, they his head to the boards of the hockey arena. Symptoms sug-
recommend admission for an overnight stay. At this time, gesting concussion are typically non-specific and can
the patient receives 1000 mL of 0.9% saline and intermittent include headache, fatigue, dizziness, nausea, self-limiting
intravenous dosing of ketorolac. vomiting, unsteadiness, feeling mentally slowed/foggy, dif-
The patient is admitted and monitored overnight with ficulties remembering, trouble with concentration, distur-
neurologic assessments and pain management every bance of sleep, and emotional changes. This patient was
4 hours. On arrival to the unit, he reports 5/10 pain and reporting headache, dizziness, nausea, blurred vision,
nausea which decreases to 0/10 shortly thereafter. The and difficulties concentrating. Because of clinical examina-
patient sleeps comfortably most of the morning, alert and tion and low suspicion for intracranial injury (ICI), based
oriented when awake, and reports dizziness with getting on Pediatric Emergency Care Applied Research Network
out of bed. The nurse also notes difficulties holding steady (PECARN) assessment, his diagnosis is less likely a TBI.
gaze and some photophobia. Neuro checks are within nor- A tool that can be helpful in assessing athletes for concus-
mal limits. He is evaluated by rehabilitation medicine and sion on the sideline or on the field is the SCAT5 (Sport Con-
neurosurgery and diagnosed with concussion/mild TBI. cussion Assessment Tool). This resource provides guidance
The patient has an abnormal vestibular ocular motor screen and immediate assessment, clinical follow-up, and recom-
on examination, and poor balance on single foot stance. mendations for returning to school and play after a
The patient tolerates lunch and vital signs remain stable. mild TBI.
Concussion education, discharge instructions, and a school Primary headache could be considered early in the differ-
note are provided and reviewed with the family. The patient ential diagnosis as headache can be a non-specific symptom
is discharged home with the family. caused by several other etiologies, including cervicogenic,
3 What pain management strategies are used for migraine, tension, and/or temporomandibular joint injury.
concussion? At this time in assessment, there was known cervicalgia
4 What is the initial acute brain rest management for and jaw pain related to the injury, which may be contribut-
concussion? ing to the patient’s report of headache. However, primary
5 What recommendations for follow-up after discharge headache can be excluded as there was a reported blow
would be made based on the diagnosis of concussion? to the face, as well as a direct hit to the head, suggesting that
6 When would the patient be able to return to normal the symptom is likely secondary to concussion.
activities after receiving a diagnosis of concussion? With complaints of neck pain, C-spine tenderness, and
7 What recommendations would be made for return to positive X-ray imaging findings, the differential should
school and sport after discharge? include cervicalgia and C-spine injury. Cervicalgia is a term
8 What typical initial home medication management is for neck pain and stiffness often referred to as “whiplash.”
recommended after concussion?
9 What are the red flags for families to recognize and Are any additional diagnostic studies indicated at
would warrant return for evaluation? this time?
According to the Centers for Disease Control (CDC), provi-
ders should not routinely obtain a head CT to diagnose mild
TBI in children. Recommendations suggest that use of vali-
Rationale and Evidence-based Practice
dated clinical decision rules that assess multiple risk factors
Explanation are more effective in identifying children at low risk for ICI,
negating the use of CT, or those with higher risks, warrant-
Based on the data given, what are the differential
ing a head CT. PECARN guidelines are an example of these
diagnoses and the most likely diagnosis for
recommendations which assess a multitude of factors that
this child?
can influence the associated risk. Following the PECARN
The differential diagnoses for this child are head injury, guidelines, this patient is 5 hours from initial injury with
concussion/mild traumatic brain injury (TBI), TBI, primary no focal neurologic signs, GCS score of 15, no vomiting,
Downloaded from https://onlinelibrary.wiley.com/doi/ by National Institute Of Standard, Wiley Online Library on [06/03/2024]. See the Terms and Conditions (https://onlinelibrary.wiley.com/terms-and-conditions) on Wiley Online Library for rules of use; OA articles are governed by the applicable Creative Commons License
258 Cases in Pediatric Acute Care

no LOC, mild mechanism of injury, no amnesia, and no Sumatriptan can produce pain/swelling at the injection
non-frontal scalp hematoma. While magnetic resonance site, and other side effects can include redness in the face
imaging (MRI) has increased the sensitivity in identifying and neck, burning feeling, feelings of tightness, and
structural abnormalities compared to CT, it should not be drowsiness.
routinely used in the acute evaluation of suspected or
diagnosed mild TBI. MRI can often require sedation in What is the initial acute brain rest management for
the pediatric population, and is more expensive than CT. concussion?
Skull X-ray should not be used in diagnosing pediatric mild
TBI. It is not the ideal test to diagnose a skull fracture with Recommendation for initial management in mild TBI
ICI, specifically because it has a 63% sensitivity for diagnos- includes brain rest for a brief period (24–48 hours) followed
ing a single skull fracture in children, cannot detect ICI, by slow gradual return to normal school and daily activities.
and utilizes radiation. Healthcare professionals should counsel the patient to limit
There is currently no laboratory test that clearly identifies physical and cognitive activity during the first several days
concussion/mild TBI. Studies using advanced neuroimag- after mild TBI and resume gradual return of activity with-
ing tools, such as functional MRI and diffusion tensor ima- out exacerbating symptoms. Close monitoring of symptoms
ging, are being performed in an attempt to identify during this time is recommended. During brain rest,
potential brain biomarkers found in serum and genetic screens should be significantly limited, especially if they
tests. However, there is insufficient evidence to obtain trigger symptoms for the patient.
serum markers outside of a research setting for the sole pur-
pose of diagnosing a child with mild TBI. What recommendations for follow-up after
discharge would be made based on the diagnosis
of concussion?
What pain management strategies are used for
concussion? The patient should follow up with their primary care pro-
vider 1–2 days after discharge from the ED. Patients should
Acutely, acetaminophen in doses of 15 mg/kg and ibupro-
not return to sport or gym/recess until cleared by a pro-
fen in doses of 7.5–10 mg/kg are recommended. The patient
vider. CDC guidelines recommend referral to a specialist
received 975 mg of oral acetaminophen, 4 mg of oral ondan-
if symptoms do not resolve after 3–5 days.
setron for nausea, and 600 mg of oral ibuprofen. The patient
did not experience relief, reports an increase in headache,
and has a history of migraine. Patients with a history of When would the patient be able to return to
migraine may develop a migraine that is triggered by a head normal activities after receiving a diagnosis of
injury. The physician has already identified red flags and concussion?
ordered imaging to exclude intracranial pathology. Treat- Healthcare providers should counsel families that 70–80%
ment should begin with a silent dark environment and of children with mild TBI do not show lasting symptoms
encouragement of sleep or rest. Next steps include intrave- 1–3 months after injury. Providers should also advise that
nous rehydration and medication. After a failed trial of oral post injury, there is a lower threshold or vulnerability for
non-steroidal anti-inflammatory drugs (NSAIDs), intrave- re-injury. Patients should return to activities as tolerated,
nous NSAIDs are a reasonable choice for migraine treat- as long as they do not trigger symptoms. Patients will be
ment in an adolescent. Rehydration can specifically allowed to return to full activity after they have returned
prevent possible acute renal failure caused by NSAIDs such to their premorbid performance and have remained symp-
as intravenous ketorolac. One study recommends using tom-free at rest and with increasing levels of physical
ketorolac starting with 0.5 mg/kg to a maximum of activity.
30 mg per dose. Ketorolac proved effective in half of the
patients within 1 hour of administration. Side effects are
What recommendations would be made for return
not common with these medications and are therefore con-
to school and sport after discharge?
sidered a first-line option. Other options include neurolep-
tics/antiemetics, which can have side effects such as Return to school after mild TBI needs to be individualized
restlessness in the legs or body, or uncontrollable muscle and monitored closely by a collaborating team, while meet-
movements such as tics and tremors. Sumatriptan spray ing the academic needs of the child without significant
is effective and can also be considered for the acute treat- symptom exacerbation. Return to school should be gradual,
ment of migraine. There are inadequate data to make a with accommodations to ease burden of return. Students
judgment on use of subcutaneous sumatriptan. should be functioning at full days of school, symptom-free,
Downloaded from https://onlinelibrary.wiley.com/doi/ by National Institute Of Standard, Wiley Online Library on [06/03/2024]. See the Terms and Conditions (https://onlinelibrary.wiley.com/terms-and-conditions) on Wiley Online Library for rules of use; OA articles are governed by the applicable Creative Commons License
Chapter 60 Adolescent with Head Injury 259

before they begin the step-by-step process for returning to What are the red flags for families to recognize
sports and physical activities. and would warrant return for evaluation?
Families should be counseled about red flags that warrant
What typical initial home medication
return for evaluation. These include drowsiness or inabil-
management is recommended after concussion?
ity to rouse, unequal pupil sizes, headache that gets worse
Medication management comprising non-opioid analgesics and does not go away, slurred speech, weakness, numb-
such as acetaminophen and ibuprofen is most effective in ness, decreased coordination, repeated vomiting, convul-
treating headache in the home environment. Opioids are sions, seizures, unusual behavior, confusion, agitation,
not recommended as therapy for headache. Patients need and LOC. For infants and toddlers, red flags would include
to be counseled about analgesic overuse (more than food refusal, inconsolability or any of the symptoms
15 times in a month) and possibility of rebound headaches. listed above.

Further Reading
Echemendia, R.J., Meeuwisse, W., McCrory, P., et al. (2017). among children. JAMA Pediatrics 172(11): e182853. doi:
The Sport Concussion Assessment Tool 5th Edition 10.1001/jamapediatrics.2018.2847.
(SCAT5): Background and rationale. British Journal of McCory, P., Messuwisse, W.H., Dvorak, J., et al. (2018).
Sports Medicine 51: 848–850. Consensus statement on concussion in sport: the 5th
Lumba-Brown, A., Yeates, K.O., Sarmiento, K., et al. (2018). international conference on concussion in sport held in
Centers for Disease Control and Prevention guideline on the Berlin, October 2016. British Journal of Sports Medicine 51
diagnosis and management of mild traumatic brain injury (11): 838–847. doi: 10.1136/bjsports-2017-097699.
Downloaded from https://onlinelibrary.wiley.com/doi/ by National Institute Of Standard, Wiley Online Library on [06/03/2024]. See the Terms and Conditions (https://onlinelibrary.wiley.com/terms-and-conditions) on Wiley Online Library for rules of use; OA articles are governed by the applicable Creative Commons License
261

61

Newborn with Ambiguous Genitalia


Barbara Wise
University of Maryland School of Nursing, Baltimore, MD, USA

An infant girl, born at 38 weeks’ gestation via normal Family History


spontaneous vaginal delivery to a gravida 1, para 1, 27-
year-old mother, is now 24 hours old. She was admitted Mother is 27 years old and has a history of asthma, with no
to the neonatal intensive care unit (NICU) for evaluation recent admissions or urgent care visits. Father is 30 years
and management of ambiguous genitalia and old and in good health. Parents are married and both are
hypoglycemia. employed. This is a first child for both parents. There is
no previous history of endocrine disorders in the family,
infertility or consanquinity. However, a second cousin
has a son diagnosed with hypospadias and cryptorchidism.
History of Present Illness

This 24-hour-old infant weighed 2560 g at birth. At delivery


Apgar scores were 91 and 95, the infant only requiring suc- Current Status
tioning after delivery. Rupture of membranes occurred
1 hour prior to delivery. On admission to the NICU the physical examination
revealed atypical genitalia with clitoromegaly and posterior
labial fusion. Vital signs are temperature 36.5 C (97.7 F),
heart rate 172 bpm, respiratory rate 36 breaths per minute,
Past Medical History and blood pressure 60/30 mmHg. Length is 48 cm and head
circumference is 35 cm. The infant has mild jaundice to the
Maternal history includes prenatal care beginning at face, increasing irritability, and vomited twice after the
10 weeks’ gestation, maternal blood type O positive, nega- initial breastfeeding. The remainder of the physical
tive for group B Streptococcus, and negative for HIV and examination is unremarkable.
sexually transmitted infections. No prenatal medications Results of complete blood count, basic metabolic profile
except prenatal vitamins. and steroid profile are shown in Tables 61.1–61.3.
Neonatal history includes vitamin K 1 mg intramuscu-
larly once after birth to prevent vitamin K deficiency bleed-
ing, hepatitis B vaccine, and erythromycin ophthalmic
ointment 1 cm applied to both eyes after birth. Infant is
Questions
breastfeeding every 2–3 hours, has voided multiple times,
Answer the following questions using the details provided.
and passed a meconium stool. The infant was jittery at
2 hours, so a point-of-care blood glucose test was obtained 1 Based on the data given, what are the differential diag-
and found to be 50 mg/dL, with the need for continued noses and the most likely diagnosis for this child?
monitoring. Plan for first metabolic screening to be 2 What diagnostic studies should be obtained to confirm
obtained. the diagnosis?

Cases in Pediatric Acute Care: Strengthening Clinical Decision Making, First Edition. Edited by Andrea M. Kline-Tilford and Catherine M. Haut.
© 2020 John Wiley & Sons Ltd. Published 2020 by John Wiley & Sons Ltd.
Downloaded from https://onlinelibrary.wiley.com/doi/ by National Institute Of Standard, Wiley Online Library on [06/03/2024]. See the Terms and Conditions (https://onlinelibrary.wiley.com/terms-and-conditions) on Wiley Online Library for rules of use; OA articles are governed by the applicable Creative Commons License
262 Cases in Pediatric Acute Care

Table 61.1 Complete blood count. Rationale and Evidence-based Practice


White blood cell count 11 500/mm3
Explanation
Hemoglobin 16.2 g/dL
Based on the data given, what are the differential
Hematocrit 45.9%
diagnoses and the most likely diagnosis for
Platelet count 205 000/mm3 this child?
Differential diagnoses for this newborn include congenital
adrenal hyperplasia, primary adrenal insufficiency, and
Table 61.2 Basic metabolic profile. ovotesticular disorder of sexual development.
The most likely diagnosis for this child is congenital adre-
Sodium 131 mEq/L nal hyperplasia (CAH, of which the most common cause is
Potassium 4.7 mEq/L 21-hydroxylase deficiency) based on the infant’s presenting
Chloride 99 mEq/L symptoms of ambiguous genitalia and laboratory studies
Carbon dioxide 25 mEq/L that indicate low sodium and chloride. It is important to dif-
Blood urea nitrogen 10 mg/dL ferentiate 21-hydroxylase deficiency from other less com-
Creatinine 0.2 mg/dL mon forms of CAH, including 11β-hydroxylase
deficiency, which occurs in 5–10% of children with CAH
Glucose 42 mg/dL
(presenting symptoms include virilization and hyperten-
Calcium 8.5 mg/dL
sion), and 3β-hydroxysteroid dehydrogenase deficiency, a
rare form that occurs in less than 1% of children diagnosed
with CAH (presents with salt wasting in males and females,
Table 61.3 Steroid profile.
but minimal virilization of the external genitalia).
The parents of this infant did not undergo genetic testing,
Test Normal values Results on day of life 6
as there were no prenatal concerns. For future pregnancies,
chorionic villous testing may be performed during the first
17-OHP <40 ng/dL 2400 ng/dL trimester to identify a mutation in the allele CYP21A2 that
ACTHa 0.7 pg/mL >9 pg/mL results in deficiency of 21-hydroxylase, the enzyme that is
Testosterone 20–62.8 ng/dL 1740 ng/dL necessary to convert 17-hydroxyprogesterone (17-OHP) to
11-deoxycortisol.
Androstenedione 20–300 ng/dL 1020 ng/dL
Cortisol 5–25 g/dL 11.3 g/dL
a
Pre-cortisol level of 0.9 g/dL and post-cortisol level of 5.9 g/dL. What diagnostic studies should be obtained
to confirm the diagnosis?
Newborn metabolic screening is available in all 50 states in
3 What therapies should be initiated for the neonate and the United States and is an appropriate tier one diagnostic
how would you determine if the therapy was effective? study for the diagnosis and management of CAH and
4 Would other studies need to be obtained? should be obtained at 48–72 hours of life. Newborn screen-
ing tests include an immunoassay to measure 17-OHP.
Continuation of case: The child was discharged home at
Exact cutoff values vary by laboratory and assay available
1 week of life after completing genetic testing, and the fam-
but are usually set low, resulting in a 1% positive rate of
ily provided with discharge instructions about administra-
detection. Approximately 1 in 100 neonates with a positive
tion of mineralocorticoid and glucocorticoid replacements
screening test will be diagnosed with CAH. Although 17-
and sodium chloride supplementation. At 3 weeks of age
OHP levels are initially elevated in most newborns and fall
the child was seen in the emergency department with a
rapidly in the first days of life, 17-OHP levels continue to
24-hour history of vomiting. The infant was tolerating
rise in children with CAH. Premature, sick, or stressed
breast milk feeds every 3 hours and had been tolerating
infants typically have higher levels of 17-OHP than term
the oral hydrocortisone.
infants and generate many false positives unless higher cut-
5 What other therapies would be recommended at offs are used. Antenatal corticosteroids used to stimulate
this time and are there other laboratory studies lung maturation in premature infants may reduce
indicated? 17-OHP levels. Premature infants should be retested after
Downloaded from https://onlinelibrary.wiley.com/doi/ by National Institute Of Standard, Wiley Online Library on [06/03/2024]. See the Terms and Conditions (https://onlinelibrary.wiley.com/terms-and-conditions) on Wiley Online Library for rules of use; OA articles are governed by the applicable Creative Commons License
Chapter 61 Newborn with Ambiguous Genitalia 263

several days of life related to inconsistent effects of antena- 6 weeks and then every 3 months until transitioning to
tal steroids. Obtaining a blood sample prior to 48 hours may adult care to prevent intercurrent illnesses and support
cause a false-positive result. growth.
Psychological support at the time of diagnosis is essential
What therapies should be initiated for the neonate to aid parents in decision-making, especially in children
and how would you determine if the therapy was with significant virilization of the external genitalia. The
effective? multidisciplinary team, including endocrinology, urology,
gynecology, genetic counselor, psychologist, social worker,
The goal of therapy in the treatment of CAH is to decrease
and primary care providers, coordinate the clinical care
excessive androgen secretion and prevent adrenal crises
with patient/family preferences.
and virilization. All patients diagnosed with classic CAH
should be treated with a combination of glucocorticoids
(hydrocortisone 20 mg/m2 daily) and mineralocorticoids Would other studies need to be obtained?
(fludrocortisone 0.2 mg/day) and 1–2 g daily of sodium
chloride supplements divided and administered with feeds This infant should have a metabolic panel obtained every
in the newborn period and early infancy. During the neo- other day to monitor for hyponatremia and hyperkalemia
natal and early infancy periods, a higher dose of mineralo- while starting sodium supplementation, mineralocorticoid
corticoids may be required because of immature kidney and glucocorticoids, with consideration of close monitoring
function that contributes to poor water and sodium regula- after discharge. A rapid fluorescence in situ hybridization
tion. Both prednisolone and dexamethasone should be (FISH) test for the sex-determining region on the
avoided to prevent growth suppression in children. Hydro- Y chromosome can be obtained. The karyotype results in
cortisone solution is not adequate for the control of CAH in this case revealed 46XX female. A pelvic ultrasound was
children because of the uneven distribution of drug in a liq- scheduled to evaluate internal reproductive organs, and
uid formulation. The glucocorticoid is usually administered findings indicated a normal uterus and enlarged adrenal
three times daily, with the highest dose (two-thirds total glands, which exhibited a cerebriform pattern characteris-
dosing) administered in the morning to mimic the circa- tic of CAH in neonates.
dian rhythm of cortisol secretion. During periods of illness
(gastroenteritis or fever associated with an ear infection) or
What other therapies would be recommended at
surgical procedures, the family should understand that
this time and are there other laboratory studies
trauma/stress dosing (doubling or tripling) of the oral
indicated?
steroid is necessary. Families should have an emergency
hydrocortisone intramuscular injection available for use The emergency department physician consulted with the
if their child is unable to take oral medications. Careful endocrinologist who recommended increasing the oral
monitoring for poor feeding, vomiting, lethargy, and sep- hydrocortisone to 30 mg/m2 divided into four doses every
sis-like symptoms is necessary to prevent life-threatening 6 hours. A basic metabolic panel was obtained and vital
hypovolemic shock. Adequate management of CAH is indi- signs monitored with attention to the child’s blood pres-
cated by normal linear growth, achievement of develop- sure. At discharge, parental education was reviewed regard-
mental milestones, and prevention of salt wasting crisis. ing adrenal insufficiency and the need for hydrocortisone
Infants less than 6 months of age should be evaluated every stress dosing in the future for intercurrent illnesses.

Further Reading
El-Maouche, D., Arlt, W., and Merke, D. (2017). Congenital deficiency: an Endocrine Society clinical practice guideline.
adrenal hyperplasia. Lancet 390(10108): 2194–2210. doi: Journal of Clinical Endocrinology and Metabolism 95(9):
10.1016/S0140-6736(17)31431-9. 4133–4160.
Levy-Shraga, Y. and Pinhas-Hamiel, O. (2016). High 17- Webb, E.A. and Krone, N. (2015). Current and novel
hydroxyprogesterone level in newborn screening test for approaches to children and young people with congenital
congenital adrenal hyperplasia. BMJ Case Reports 2016: adrenal hyperplasia and adrenal insufficiency. Best Practice
bcr2015213939. doi: 10.1136/bcr-2015-213939. and Research in Clinical Endocrinology and Metabolism 29:
Speiser, P.W., Aziz, R., Baskin, L.S., et al. (2010). Congenital 449–468.
adrenal hyperplasia due to steroid 21-hydroxylase
Downloaded from https://onlinelibrary.wiley.com/doi/ by National Institute Of Standard, Wiley Online Library on [06/03/2024]. See the Terms and Conditions (https://onlinelibrary.wiley.com/terms-and-conditions) on Wiley Online Library for rules of use; OA articles are governed by the applicable Creative Commons License
265

62

Anemia in a Toddler
Dyane Bunnell
Nemours/Alfred I. duPont Hospital for Children, Wilmington, DE, USA

A 20-month-old infant was referred for management of Family History


abnormal laboratory values.
No siblings. No significant family history reported.

History of Present Illness


Social History
This 20-month-old infant was investigated at the Women
Infants and Children (WIC) office where she was found She lives with her mother, maternal grandmother, and 23-
to have a hemoglobin level of 4.8 g/dL. She was referred year-old maternal uncle. Mother is in a relationship with
to an outside hospital emergency department (ED) where her father, but they are not married and he does not live
her hemoglobin level was confirmed to be 5.2 g/dL. Her in the home.
white blood cell (WBC) count was 117 000/mm3 and plate-
let count was 24 000/mm3. A basic metabolic profile (BMP)
was within normal limits at that time. She was started on Current Status
maintenance intravenous fluids during transport from
the outside hospital. Her mother also reported that she She is alert and playful with pale conjunctiva. She has hepa-
has been pale and has bruised easily in the past month. tosplenomegaly and bruising along her distal extremities.
No recent history of fever. Her weight has remained the The rest of her examination is unremarkable. A complete
same over the last 3 months. She does not take any medica- blood count and BMP are obtained, with results as noted
tions at baseline. in Tables 62.1 and 62.2. The chest radiograph is shown in
Figure 62.1.
On the chest radiograph the lungs are hypoventilated and
Past Medical History clear. There is no effusion of pneumothorax. The cardiothy-
mic silhouette is normal and the aortic arch is on the left.
She was born full term with a birthweight of 3.005 kg after There is no mediastinal adenopathy. The spleen appears
an uncomplicated pregnancy and delivery. Her weight is at enlarged and the bowel pattern is unremarkable.
the 26th percentile for gender and age and her body mass
index (BMI) is at the 16.3th percentile for gender and
age. Immunizations are up to date, including annual influ- Questions
enza vaccination.
Answer the following questions using the details provided.
1 Based on the data given, what are the differential diag-
Past Surgical History noses and the most likely diagnosis for this child?
2 What emergencies may be associated with this
No prior surgical history. diagnosis?

Cases in Pediatric Acute Care: Strengthening Clinical Decision Making, First Edition. Edited by Andrea M. Kline-Tilford and Catherine M. Haut.
© 2020 John Wiley & Sons Ltd. Published 2020 by John Wiley & Sons Ltd.
Downloaded from https://onlinelibrary.wiley.com/doi/ by National Institute Of Standard, Wiley Online Library on [06/03/2024]. See the Terms and Conditions (https://onlinelibrary.wiley.com/terms-and-conditions) on Wiley Online Library for rules of use; OA articles are governed by the applicable Creative Commons License
266 Cases in Pediatric Acute Care

Table 62.1 Basic metabolic profile. Rationale and Evidence-based Practice


Sodium 142 mEq/L
Explanation
Potassium 4.1 mEq/L
Based on the data given, what are the differential
Chloride 109 mEq/L
diagnoses and the most likely diagnosis for
Carbon dioxide 20 mEq/L this child?
Blood urea nitrogen 11 mg/dL
Differential diagnoses for this patient include infection,
Creatinine 0.3 mg/dL
bone marrow failure (aplastic anemia), and leukemia.
Glucose 94 mg/dL
New-onset leukemia is the most likely diagnosis due to
the presentation of bruising, pallor, hepatosplenomegaly,
and abnormal laboratory findings demonstrating anemia,
Table 62.2 Complete blood count.
thrombocytopenia and leukocytosis with evidence of blasts.
White blood cell count 142 200/mm3 Viral infection/bone marrow suppression is unlikely with-
Hemoglobin 4.8 g/dL
out focal signs of infection. Bone marrow failure such as
aplastic anemia is less likely given the presence of leukocy-
Hematocrit 13%
tosis rather than leukopenia.
Platelet count 26 000/mm3
Neutrophils 3%
What emergencies may be associated
Monocytes 2% with this diagnosis?
Lymphocytes 24%
Hyperleukocytosis is an oncologic emergency requiring
Blasts 68%
prompt recognition and intervention. Hyperleukocytosis
presents in up to 20% of children diagnosed with leukemia.
It is defined as a WBC count greater than 100 000/mm3 and
has been associated with early morbidity and mortality. The
elevated WBC count and presence of leukemic blasts that
do not deform easily may lead to leukostasis and occlusion
of small vessels. Leukostasis may present as intracranial
hemorrhage, stroke, acute kidney injury, or acute respira-
tory distress syndrome. Patients at highest risk for compli-
cations from hyperleukocytosis include those with acute
lymphoblastic leukemia (ALL) and acute myeloid leukemia
(AML), especially the monocytic (M4) and promyelocytic
(M5) subtypes. Patients with AML are more likely to be
symptomatic and suffer more complications from hyperleu-
kocytosis than those with ALL, even at WBC counts that
are considerably lower than those with ALL.
A secondary concern in the setting of hyperleukocytosis
is tumor lysis syndrome (TLS). TLS is an oncologic emer-
gency that is caused by tumor cell lysis with the release
of intracellular components including potassium, phos-
phate, and nucleic acids into the circulation. One or more
of these molecular abnormalities may lead to acute kidney
injury and cardiac arrhythmias.

Figure 62.1 Chest radiograph.


What are the goals of management and initial
interventions?
3 What are the goals of management and initial
interventions? Oxygenation and circulation need to be optimized. Intra-
4 How would you determine if the interventions are venous hydration will be administered to promote circu-
effective? lation. Interventions to reduce the WBC count including
5 Which additional evaluations may be necessary? chemotherapy and hydroxyurea may be administered.
Downloaded from https://onlinelibrary.wiley.com/doi/ by National Institute Of Standard, Wiley Online Library on [06/03/2024]. See the Terms and Conditions (https://onlinelibrary.wiley.com/terms-and-conditions) on Wiley Online Library for rules of use; OA articles are governed by the applicable Creative Commons License
Chapter 62 Anemia in a Toddler 267

Once the immediate threat of leukostasis secondary to curve can assist in determining the need for antibiotics. If
hyperleukocytosis is decreased, it is important to con- febrile, obtain blood cultures and initiate broad-spectrum
servatively transfuse packed red blood cells (5 mL/kg antibiotics with pseudomonal coverage.
over 3 hours) to promote oxygenation; if the patient is Continue close assessment of pulmonary signs and symp-
hemodynamically unstable, slow transfusion may be toms, such as dyspnea and hypoxia. Evaluation may
needed prior to administration of cytoreductive therapy. include chest radiograph and computed tomography (CT)
Ongoing assessment of pulse oximetry and physical as needed to evaluate for interstitial or alveolar infiltrates
examination including neurologic status should be due to leukostasis. Changes in neurologic function (e.g.
monitored. visual changes, headache, dizziness, tinnitus, unsteady gait,
Anticipation of TLS is also important in this case. Fre- confusion, and somnolence) may indicate intracranial
quent laboratory evaluation, aggressive hydration, close hemorrhage. Continuous monitoring of the electrocardio-
monitoring of renal function, and consideration of allopu- gram is important given the risk of arrhythmias associated
rinol to prevent hyperuricemia or rasburicase to treat with electrolyte derangements.
hyperuricemia is necessary. Close laboratory monitoring for disseminated intravascu-
lar coagulation, including prothrombin time (PT), partial
How would you determine if the interventions thromboplastin time (PTT), fibrinogen, fibrin split
are effective? products, and D-dimer, which may worsen following initi-
ation of chemotherapy. Consult pediatric hematology/
The effectiveness of initial interventions can be determined
oncology to consider leukapheresis or exchange transfu-
by normalization of laboratory values, adequate fluid/elec-
sion, as there is conflicting evidence regarding the utility
trolyte balance, normal renal function, and the absence of
and possible complications of these interventions. To eval-
secondary complications. Frequent physical examinations
uate the leukemia diagnosis, a bone marrow aspirate,
with attention to cardiac, respiratory, and central nervous
biopsy, and lumbar puncture should be obtained. Empiric
system function are important.
intrathecal chemotherapy will be administered during that
procedure. The marrow and cerebrospinal fluid will be
Which additional evaluations may be necessary?
evaluated to further describe the disease, identify prognos-
Evaluation for an infectious etiology is also warranted. tic factors, and guide therapy. After the results of diagnostic
Bone marrow failure and leukemia may be associated with studies are final, the initiation of chemotherapy should
bacterial or viral illnesses. A close evaluation of the com- commence, with vigilance in the prevention and manage-
plete blood count with differential and fever history/fever ment of TLS.

Further Reading
Creutzig, U., Rossig, C., Dworzak, M., et al. (2016). Exchange Korkman, S. (2018). The management of hyperleukocytosis in
transfusion and leukapheresis in pediatric patients with 2017: do we still need leukapheresis? Transfusion and
AML with high risk of early death by bleeding and Apheresis Science 57(1): 4–7.
leukostasis. Pediatric Blood and Cancer 63(4): 640–645. Nguyen, R., Jeha, S., Zhou, Y., et al. (2016). The role of
Kar, A. and Hijiya, N. (2018). Diagnosis and initial leukapheresis in the current management of
management of pediatric acute leukemia in the emergency hyperleukocytosis in newly diagnosed childhood acute
department setting. Clinical Pediatric Emergency Medicine lymphoblastic leukemia. Pediatric Blood and Cancer 63(9):
19(2): 135–144. 1546–1551.
Downloaded from https://onlinelibrary.wiley.com/doi/ by National Institute Of Standard, Wiley Online Library on [06/03/2024]. See the Terms and Conditions (https://onlinelibrary.wiley.com/terms-and-conditions) on Wiley Online Library for rules of use; OA articles are governed by the applicable Creative Commons License
269

63

A 2-Year-Old with Fever and Cough


Jessica Strohm Farber
Children’s Hospital of Philadelphia, Philadelphia, PA, USA

A 2-year-old female with a history of prematurity presents has a 4-year-old sibling who has also recently had a cold but
to the emergency department (ED) with cough, fever, and is otherwise healthy.
increased work of breathing.

History of Present Illness Current Status

This 2-year-old female developed an upper respiratory On arrival to the ED, the child is in moderate respiratory
infection including low-grade fever, congestion, rhinor- distress with nasal flaring, intermittent grunting, and mod-
rhea, and cough that improved by day 4 of illness. On erate retractions. She is hypoxic with an oxygen saturation
day 6, she developed fever to 40 C (104 F), worsened pro- of 89% in room air. Respiratory rate is 48 breaths per
ductive cough, malaise, and decreased appetite. On day 7 of minute. Evaluation of breath sounds indicates dense
illness, she was evaluated by the pediatric provider for con- crackles in the left lower lobe with no wheezing and no
tinued fever and productive cough who noted focal crackles other adventitious sounds.
on examination. The child had post-tussive emesis, The child has a temperature of 38.9 C (102 F) by oral
decreased urine output, malaise, and poor appetite. She route, with a heart rate of 155 bpm. She is well perfused
was referred to the ED for further evaluation. but slightly cool peripherally. Capillary refill is 2–3 seconds.
Abdomen is soft, non-tender, and non-distended without
focal findings. She is neurologically appropriate for age
Past Medical and Surgical History and situation.
Current treatment includes oxygen via nasal cannula.
The patient was born at 29 weeks’ gestation, was intubated A peripheral intravenous line is inserted, and blood work
at birth and remained on mechanical ventilation for and a chest radiograph are obtained. The patient remains
approximately 1 week, then transitioned to CPAP (contin- tachypneic with increased work of breathing and fever.
uous positive airway pressure) for approximately 2 weeks. Results of complete blood count and basic metabolic pro-
She spent a total of 8 weeks in the neonatal intensive care file are shown in Tables 63.1 and 63.2. Anteroposterior and
unit and has not had any major illnesses, surgical proce- lateral chest radiographs are shown in Figures 63.1
dures or hospitalizations since discharge. Immunizations and 63.2.
are current and the child does not have any known allergies
to any food or medications.

Questions
Family History Answer the following questions using the details provided.
Both parents and grandparents are alive and healthy. There 1 Based on the data given, what are the differential diag-
is no family history of asthma or food allergies. The patient noses and the most likely diagnosis for this child?

Cases in Pediatric Acute Care: Strengthening Clinical Decision Making, First Edition. Edited by Andrea M. Kline-Tilford and Catherine M. Haut.
© 2020 John Wiley & Sons Ltd. Published 2020 by John Wiley & Sons Ltd.
Downloaded from https://onlinelibrary.wiley.com/doi/ by National Institute Of Standard, Wiley Online Library on [06/03/2024]. See the Terms and Conditions (https://onlinelibrary.wiley.com/terms-and-conditions) on Wiley Online Library for rules of use; OA articles are governed by the applicable Creative Commons License
270 Cases in Pediatric Acute Care

Table 63.1 Complete blood count.

White blood cell count 22 000/mm3


Hemoglobin 13 g/dL
Hematocrit 39%
Platelet count 435 000/mm3
Neutrophils 77%
Lymphocytes 8%
Monocytes 10%
Eosinophils 5%

Table 63.2 Basic metabolic profile.

Sodium 145 mEq/L


Potassium 4 mEq/L
Chloride 103 mEq/L
Carbon dioxide 19 mEq/L
Blood urea nitrogen 18 mg/dL
Creatinine 0.7 mg/dL
Glucose 85 mg/dL Figure 63.2 Lateral chest radiograph. Source: Courtesy of Cheryl
Bartke.
Calcium 9 mg/dL

Rationale and Evidence-based Practice


Explanation

Based on the data given, what are the differential


diagnoses and the most likely diagnosis for
this child?
Differential diagnoses for this child include a respiratory
viral illness such as bronchiolitis or viral pneumonia, ate-
lectasis, bacterial pneumonia, atypical pneumonia, pleural
effusion, aspiration, and status asthmaticus. The most
likely diagnosis for this child is a community-acquired bac-
terial pneumonia. The history of viral symptoms with a
period of improvement then worsening with fever and pro-
ductive cough provides the explanation for the develop-
ment of pneumonia. The respiratory rate and retractions
are physical examination findings that are considered reli-
able in the diagnosis of community-acquired pneumonia
(CAP) (Florin et al., 2017). Focal crackles on examination,
Figure 63.1 Anteroposterior chest radiograph. Source: Courtesy new fever after a period of defervescence, and hypoxia add
of Cheryl Bartke. to the likelihood of pneumonia as a diagnosis. Without
audible wheezing and the presence of high fever, bronchi-
olitis, status asthmaticus and atelectasis can be ruled out as
2 What therapy may improve this child’s work of possible differentials. The chest radiograph is suggestive of
breathing? left lower lobe infiltrate, with volume loss on the left side of
3 What additional studies may be indicated? the chest. These findings, together with the clinical picture,
4 What antibiotic therapy is appropriate? make the diagnosis of a bacterial pneumonia most likely.
Downloaded from https://onlinelibrary.wiley.com/doi/ by National Institute Of Standard, Wiley Online Library on [06/03/2024]. See the Terms and Conditions (https://onlinelibrary.wiley.com/terms-and-conditions) on Wiley Online Library for rules of use; OA articles are governed by the applicable Creative Commons License
Chapter 63 A 2-Year-Old with Fever and Cough 271

This child also has a history of prematurity, and there may for the treatment of both Gram-positive and Gram-negative
be a component of chronic lung disease. However, there is organisms. Vancomycin or clindamycin may be
no evidence of chronic lung disease on the radiograph. considered as an additional antimicrobial for patients with
There is always a possibility for the presence of a pleural evidence or concern for Staphylococcus aureus infection. If
effusion in a child with pneumonia, but no indication of the clinical history and findings are more suggestive of
this is noted on the chest radiograph. atypical pneumonia, then a macrolide such as
azithromycin can be administered (Bradley et al., 2011).
What therapy may improve this child’s work In this clinical case, ceftriaxone should be initiated for treat-
of breathing? ment of left lower lobe pneumonia, with clindamycin or
vancomycin strongly considered given the presence of
In the presence of hypoxia, tachypnea, and increased work moderate to severe pneumonia. A positive blood culture
of breathing, non-invasive positive pressure ventilation with antibiotic sensitivities can also assist in guiding
(NIPPV) would be indicated to include supplemental oxy- treatment.
gen. NIPPV therapy such as CPAP or bilevel positive airway
pressure (BiPAP) suggests the need for intensive care unit Continuation of case: The 2-year-old child is admitted to
(ICU) admission or admission to a unit that is able to per- the PICU and placed initially on non-invasive ventilation
form continuous cardiorespiratory monitoring (Bradley with CPAP, (positive end-expiratory pressure) of 8 cmH2O
et al., 2011). The need for NIPPV also indicates respiratory for hypoxia and labored breathing, then escalated to BiPAP
failure or impending failure, so admission to the pediatric with settings of inspiratory positive airway pressure
intensive care unit (PICU) is definitely warranted for +16 cmH2O and expiratory positive airway pressure
this child. +6 cmH2O with 30% set oxygen. Ceftriaxone and vancomy-
cin are chosen for broad-coverage antibacterial therapy,
What additional studies may be indicated? with the awareness that monitoring for therapeutic and
A blood culture should be obtained in febrile children who toxic vancomycin levels is necessary.
require hospitalization for CAP that is complicated or asso- With BiPAP, the patient’s respiratory rate slows to
ciated with moderate to severe disease. A blood culture 30 breaths per minute, but continues with mild subcostal
should be strongly considered in children with comorbid- retractions. She is weaned to 21% oxygen, but remains on
ities, those who require ICU hospitalization, and those with BiPAP settings for work of breathing.
complicated pneumonia such as the presence of a pleural
effusion (Neuman et al., 2017). Testing for influenza and
other viruses can also be accomplished, but antimicrobial Additional Questions and
therapy should not be withheld in the presence of markers Evidence-based Rationale
of bacterial infection such as radiologic evidence, clinical
evidence, or history concerning for a bacterial co-infection. 5 What other therapies would be recommended at
If radiologic findings or history are more suggestive of atyp- this time?
ical pneumonia, then testing for Mycoplasma pneumoniae 6 What are indicators of clinical improvement?
should be considered as this may impact antibiotic selec-
tion (Bradley et al., 2011).
What other therapies would be recommended
What antibiotic therapy is appropriate? at this time?
Knowledge of immunization status is important in the diag- Given the presence of fever, tachycardia, and history of
nosis and treatment of CAP, as missing vaccinations may emesis, volume expansion with crystalloid boluses should
assist the provider in determining likely organisms to con- be considered, as a 20 mL/kg volume. Maintenance intra-
sider for antimicrobial therapy. The most likely causes of venous fluids should be initiated. The child should remain
pneumonia in children between birth and age 2 include nothing by mouth (NPO) if there is a concern for progres-
primarily viral organisms, along with Streptococcus sion to respiratory failure. Airway clearance therapies could
pneumoniae, Mycoplasma pneumoniae and Chlamydia also be considered for secretion mobilization. Additional
pneumoniae. This child failed outpatient antibiotic therapy, imaging, such as chest ultrasound, to evaluate for the pres-
meets criteria for moderate to severe disease and is not yet ence of parapneumonic effusion or empyema should be
fully immunized, so a third-generation cephalosporin considered as their presence my require surgical interven-
administered parenterally (such as ceftriaxone) is indicated tion (Bradley et al., 2011).
Downloaded from https://onlinelibrary.wiley.com/doi/ by National Institute Of Standard, Wiley Online Library on [06/03/2024]. See the Terms and Conditions (https://onlinelibrary.wiley.com/terms-and-conditions) on Wiley Online Library for rules of use; OA articles are governed by the applicable Creative Commons License
272 Cases in Pediatric Acute Care

What are indicators of clinical improvement? of breathing and need for respiratory support should
improve. Heart rate should also normalize, and the fever
Clinical improvement would be evaluated by serial physical
curve should decrease. Repeat chest radiograph could be
examinations considering respiratory rate, work of breath-
considered if there is failure of clinical improvement.
ing, and oxygen requirement. As the child improves, work

References
Bradley, J.S., Byington, C.L., Shah, S.S., et al. (2011). The acquired pneumonia. Pediatrics 140(3): e20170310. doi:
management of community-acquired pneumonia in infants 10.1542/peds.2017-0310.
and children older than 3 months of age: Clinical practice Neuman, M.I., Hall, M., Lipsett, S.C., et al. (2017). Utility of
guidelines by the Pediatric Infectious Diseases Society and blood culture among children hospitalized with community-
the Infectious Diseases Society of America. Clinical acquired pneumonia. Pediatrics 140(3): e20171013. doi:
Infectious Diseases 53(7): e25–e76. doi: 10.1093/cid/cir531. 10.1542/peds.2017-1013.
Florin, T.A., Ambroggio, L., Brokamp, C., et al. (2017).
Reliability of examination findings in suspected community-

Further Reading
Dean, P. and Florin, T.A. (2018). Factors associated with Shan, W., Shi, T., Chen, K., et al. (2019). Risk factors for severe
pneumonia severity in children: a systematic review. community-acquired pneumonia among children
Journal of the Pediatric Infectious Diseases Society 7(4): hospitalized with CAP younger than 5 years of age. Pediatric
323–334. Infectious Disease Journal 38(3): 224–229.
Messinger, A.I., Kupfer, O., Hurst, A., and Parker, S. (2017).
Management of pediatric community acquired bacterial
pneumonia. Pediatrics in Review 38(9): 394–409.
Downloaded from https://onlinelibrary.wiley.com/doi/ by National Institute Of Standard, Wiley Online Library on [06/03/2024]. See the Terms and Conditions (https://onlinelibrary.wiley.com/terms-and-conditions) on Wiley Online Library for rules of use; OA articles are governed by the applicable Creative Commons License
273

64

Research Review
Lauren R. Sorce1,2
1
Ann & Robert H. Lurie Children’s Hospital of Chicago, Chicago, IL, USA
2
Northwestern University Feinberg School of Medicine, Chicago, IL, USA

The acute care provider in a busy pediatric intensive care


unit (PICU) has been asked to participate in a multiprofes-
• Research nurses are formally or informally educated on
the conduct of research and can provide useful informa-
sional clinical research study on biobanking. After meeting tion on study implementation.
with the research team, she has been identified as an inte-
gral part in the study through contributions to the develop-
• Nurse scientists are researchers who design and imple-
ment independent research and can facilitate learning
ment and implementation of the clinical study. However, for those who wish to be part of data collection for a
she has not participated in clinical research at this level research study.
and is uncertain of what is required for her participation
and does not really know where to start.
• Clinical research coordinators are team members who
facilitate research in many hospitals by managing the
research studies for principal investigators.
Questions Participating in the research process can be an over-
whelming task regardless of one’s role. At the graduate level,
Answer the following questions using the details provided: formal classroom education on research is variable depend-
1 Who could help the provider learn about the research ing on the program of study. While many nurse practitioner
process? programs have transitioned to the Doctor of Nursing Practice
2 Where can the healthcare provider seek information (DNP) degree, the focus of these programs is evidence-based
regarding education on the protection of human practice, not conducting research. Master’s degree programs
subjects? do include varying elements of the research process, but
3 What education is required for the protection of human unless the provider has a Doctor of Philosophy (PhD) degree,
subjects? formal education in research is usually limited. Yet, many
4 When are consents and assents required in the research healthcare providers, including nurse practitioners, learn
process? about research on the job and have performed independent
5 How is child assent different from informed consent? research and participated in the research of others.
6 What responsibilities do those who participate in data
collection for a research study have? Where can the healthcare provider seek
information regarding education on the protection
of human subjects?
Rationale and Evidence-based Practice
Explanation Critical to the performance of research is the protection of
human subjects. As such, education prior to the participa-
Who could help the provider learn about the tion in a research study is required.


research process?
The Department of Health and Human Services website

• Members of the biobanking research team can provide


insight into why this work is important and how and
contains information on federal policy for the protection
of human subjects called the Common Rule. More
why her contributions will strengthen it. specifically, the Office of Human Research Protections

Cases in Pediatric Acute Care: Strengthening Clinical Decision Making, First Edition. Edited by Andrea M. Kline-Tilford and Catherine M. Haut.
© 2020 John Wiley & Sons Ltd. Published 2020 by John Wiley & Sons Ltd.
Downloaded from https://onlinelibrary.wiley.com/doi/ by National Institute Of Standard, Wiley Online Library on [06/03/2024]. See the Terms and Conditions (https://onlinelibrary.wiley.com/terms-and-conditions) on Wiley Online Library for rules of use; OA articles are governed by the applicable Creative Commons License
274 Cases in Pediatric Acute Care

(https://www.hhs.gov/ohrp/) is responsible for protect- provide consent. To obtain assent, one must provide the
ing the rights of human subjects and provides regulatory information in an understandable way to the minor.
oversight regarding the performance of behavioral and
biomedical research that is federally funded.

• The local institutional review board (IRB) is responsible


for approval of research specifically ensuring the protec-
How is child assent different from informed
consent?
tion of human subjects. Although some research Recently, Tait and Geisser (2017) recommended the follow-
approved through the IRB may not be federally funded, ing operational definition of assent given the overall lack of
investigators must comply with federal regulations. consensus:
What education is required for the protection
of human subjects? Children who lack the legal authority to provide
informed consent per state laws should provide their
The primary program which outlines information on the assent to participate in a research study unless they
protection of human subjects for those conducting or par- either lack the cognitive ability, their clinical condi-
ticipating in research activities is called the Collaborative tion precludes their ability to communicate a choice,
Institutional Training Initiative (CITI) program. Individual or the research holds out the prospect of direct ben-
organizations such as hospitals and universities can use the efit that is only available in the context of the
CITI training programs and/or outline their own specific research. Assent is an interactive process between
content. a researcher and child participant involving disclo-

• The CITI program (https://about.citiprogram.org/en/


homepage/) provides education on research ethics and
sure of cognitively and emotionally appropriate
information regarding, at minimum, why the child
is being asked to participate, a description of the pro-
compliance. CITI training is required by nearly all IRBs
cedures and how the child might experience them,
to ensure the researcher(s) are educated in the protection
and an understanding that participation in the study
of human subjects. Therefore, each healthcare or educa-
is voluntary. Children should understand that they
tional institution has access to this education. Renewal of
can decline participation or withdraw from the study
CITI training is required every 2 years.

• Good clinical practice (GCP) training is also provided by


the CITI program as a separate educational requirement
at any time. Assent requires that the child explicitly
affirms his or her agreement to participate in a man-
ner that reflects their age-appropriate understanding
prior to engagement in research. This training is geared
and that is free of undue influence or coercion. In the
towards the ethical conduct of clinical research (as
absence of an explicit agreement, mere failure of the
opposed to research in the laboratory) and scientific stan-
child to object cannot be construed as assent.
dards for study design, implementation and dissemina-
tion. The courses are devised based on the type of
research (e.g. investigational drugs, medical devices, clin- Obtaining consent and/or assent in research is required
ical research). Renewal of GCP training may vary by in any study where there is a posed risk to participants, such
institution, but many institutions allow 3 years until as an interventional drug trial or when personal health
expiration. information will be used in research. Most institutions
require assent for children beginning at the age of 7 in addi-
In addition to these educational requirements for clinical
tion to informed consent obtained from their parents or
research, institutions may have individualized require-
guardian. Most institutions also offer templates or recom-
ments. Pediatric clinical research can be particularly com-
mendations for creating assent and consent forms, which
plicated as children are the research subjects yet the legal
include required elements. The IRB often requires that con-
authorized guardian, in many cases the parent(s), provides
sent and assent forms be submitted with a research package
consent. Although the rule for child assent may be different
to be approved with the methodology of the study.
state to state or institution to institution, child assent is nec-
Another issue associated with pediatric research is tran-
essary for participation in research.
sition of the subject to adulthood at 18 years of age. Any
subject participating in research who turns 18 years old
When are consents and assents required in the
during the research study must provide consent for ongoing
research process?
participation. This becomes particularly problematic if the
Assent differs from consent in that assent is obtained from subject cannot be located. If consent cannot be obtained,
someone (usually a minor) who is not legally able to the subject’s participation ceases.
Downloaded from https://onlinelibrary.wiley.com/doi/ by National Institute Of Standard, Wiley Online Library on [06/03/2024]. See the Terms and Conditions (https://onlinelibrary.wiley.com/terms-and-conditions) on Wiley Online Library for rules of use; OA articles are governed by the applicable Creative Commons License
Chapter 64 Research Review 275

Relevant to this study on biobanking, the subject must be What responsibilities do those who participate in
consented upon turning 18 years of age, otherwise the sam- data collection for a research study have?
ples must be destroyed if they are linked to identifiable data.
Those healthcare providers who want to participate in
If the samples are not linked to identifiable data (described
research also need awareness of how to safeguard and pro-
by the Health Insurance Portability and Accountability Act;
tect the information related to a human subject. In most
https://www.hhs.gov/hipaa/for-professionals/privacy/
cases, this involves keeping the data locked or double
laws-regulations/index.html), the samples may continue to
locked, within a password-protected computer, or in a
be used. In a recent study on biobanking in the pediatric
locked desk drawer. It is the responsibility of the research
critical care setting, adolescents wanted to have a say in
team to maintain confidentiality of study participants at
the use of their samples (Paquette et al., 2018). If partici-
all times.
pants in any research study do not agree to all of the stipu-
lations outlined in the consent/assent process, and agree
with a witnessed signature, the participant cannot be a part
of the study.

References
Paquette, E.D., Derrington, S.F., Shukla, A., et al. (2018). Tait, A.R. and Geisser, M.E. (2017). Development of a
Biobanking in the pediatric critical care setting: adolescent/ consensus operational definition of child assent for research.
young adult perspectives. Journal of Empirical Research on BMC Medical Ethics 18: 41. doi: 10.1186/s12910-017-0199-4.
Human Research Ethics 13: 391–401. doi: 10.1177/
1556264618782231.

Further Reading
Collaborative Institutional Training Initiative (CITI) (2019). Privacy Rule (2013). https://www.hhs.gov/hipaa/for-
https://about.citiprogram.org/en/homepage/ (accessed professionals/privacy/laws-regulations/index.html
19 June 2019). (accessed 19 June 2019).
United States Department of Health and Human Services. United States Department of Health and Human Services,
Health information privacy. https://www.hhs.gov/hipaa/ Office for Human Research Protection (2019). https://www.
index.html (accessed 19 June 2019). hhs.gov/ohrp/ (accessed 19 June 2019).
United States Department of Health and Human Services.
Health information privacy: summary of the HIPPAA
Downloaded from https://onlinelibrary.wiley.com/doi/ by National Institute Of Standard, Wiley Online Library on [06/03/2024]. See the Terms and Conditions (https://onlinelibrary.wiley.com/terms-and-conditions) on Wiley Online Library for rules of use; OA articles are governed by the applicable Creative Commons License
277

65

Toddler with Fever and Redness Behind Ear


Amanda Lewis Weir
Norton Children’s Hospital, Louisville, KY, USA

A 2-year-old boy presents to pediatric emergency depart- Past Surgical History


ment (ED) with a chief complaint of “fussiness and redness
behind ear.” He had myringotomy and placement of bilateral tympa-
nostomy tubes at 10 months of age, which fell out at approx-
imately 18 months of age.

History of Present Illness


Family History
He was seen last night in the same ED with a 1-week history
of runny nose, cough, and congestion and 2 days of fever up The mother denies any pertinent maternal medical history.
to 40.5 C (105 F). On examination, he was found to have a The patient’s sibling also has a history of asthma. The
bilateral acute otitis media (AOM) and was discharged mother is unsure of medical history on the father’s side
home with amoxicillin 90 mg/kg daily for 10 days. On of the family.
the morning of presentation, the patient woke up again
with fever of 40.5 C and tugging on the right ear. On further Social History
investigation, the mother noted that he had redness behind
his right ear and it appeared to be “sticking out” further He lives at home with single mother and one older sibling.
than his left ear. He has had decreased oral intake, but His mother is an emergency medical technician. He attends
has been urinating normally. daycare. The mother denies any sick contacts at home but is
unsure of sick contacts at daycare. There is no tobacco expo-
sure in the home.

Past Medical History


Current Status
He was born full term at 39 weeks and was discharged
home with his mother. He has a history of mild persistent On examination, he appears an ill toddler, lying in his
asthma, seasonal allergies, and recurrent otitis media. He mother’s arms. He becomes appropriately agitated by the
has been evaluated by pediatric infectious disease with con- examination. He has clear nasal drainage and occasional
cerns for frequent infections. An evaluation for primary congested cough. On otoscopic examination, both right
immune deficiency was negative. He has no previous hos- and left tympanic membranes are red and bulging with
pitalizations. All immunizations are up to date, including purulence. Both external ear canals are normal without
influenza. redness and swelling. The right pinna is displaced upward
His current chronic medications are albuterol 2.5 mg and outward and there is development of postauricular
nebulized every 4 hours as needed for wheezing, montelu- swelling, redness, and tenderness. There is no obvious
kast 4 mg every evening, and cetirizine 5 mg every evening. abscess or fluid accumulation noted behind the right ear.

Cases in Pediatric Acute Care: Strengthening Clinical Decision Making, First Edition. Edited by Andrea M. Kline-Tilford and Catherine M. Haut.
© 2020 John Wiley & Sons Ltd. Published 2020 by John Wiley & Sons Ltd.
Downloaded from https://onlinelibrary.wiley.com/doi/ by National Institute Of Standard, Wiley Online Library on [06/03/2024]. See the Terms and Conditions (https://onlinelibrary.wiley.com/terms-and-conditions) on Wiley Online Library for rules of use; OA articles are governed by the applicable Creative Commons License
278 Cases in Pediatric Acute Care

He does have mild end-expiratory wheezing on examina- concern for sepsis, a blood culture should be obtained prior
tion but has no retractions or tachypnea. His neck is with- to the initiation of antibiotics.
out signs of meningismus. His vital signs are within normal Acute mastoiditis can be diagnosed and treated without
range for his age, except for a temperature of radiographic evidence. However, if clinical examination
39.1 C (102.3 F). alone is unable to determine the severity of the infection
or if there is evidence of subperiosteal abscess or intracra-
nial involvement, imaging can be obtained. The two pri-
Questions mary imaging modalities used with diagnosis of acute
mastoiditis are computed tomography (CT) with contrast
Answer the following questions using the details provided. and magnetic resonance imaging (MRI) with gadolinium.
A CT with contrast is helpful in determining bony changes,
1 Based on the history and examination what are the dif-
while MRI is used when there are clinical concerns for
ferential diagnoses?
intracranial involvement.
2 What is the likely diagnosis?
3 What diagnostics are helpful in making the diagnosis?
Continuation of case: A basic metabolic profile, CBC, and
CRP are obtained and the results are shown in Tables 65.1
and 65.2. CT with intravenous contrast is also obtained and
Rationale and Evidence-based Practice is significant for opacification of the mastoid air cells on the
Explanation right side (left side of Figure 65.1).

Based on the history and examination what are Additional Questions and Evidence-
the differential diagnoses? based Rationale
Differential diagnoses for this patient include mastoiditis,
parotitis, postauricular lymphadenitis, periauricular cellu- 4 What are the common pathogens of this diagnosis?
litis with or without otitis externa, periauricular inflamma- 5 What is the treatment for a patient with this diagnosis?
tion secondary to insect bite or trauma, mumps, or tumor.
Table 65.1 Basic metabolic profile.
What is the likely diagnosis?
Sodium 135 mEq/L
The likely diagnosis for this patient given concurrent AOM Potassium 4.6 mEq/L
with postauricular redness, edema, and tenderness and Chloride 102 mEq/L
upward and outward displacement of the left pinna is acute Carbon dioxide 30 mEq/L
mastoiditis. Acute mastoiditis is the most common compli-
Blood urea nitrogen 12 mg/dL
cation of AOM in children and affects 1.2–6 per 100 000 chil-
Creatinine 0.3 mg/dL
dren per year. It develops concurrently with AOM when
purulent material is unable to drain and blocks the commu- Glucose 94 mg/dL
nication between the mastoid cavity and the tympanic cav-
ity. As this purulence builds within the mastoid cells it
causes ongoing inflammation and breakdown of the mas- Table 65.2 Complete blood count.
toid bone, leading to acute mastoiditis. Often, patients will
develop a periosteal abscess secondary to the infection White blood cell count 35 000/mm3
and fluctuance behind the ear can be noted on examination. Hemoglobin 13 g/dL
Hematocrit 42%
What diagnostics are helpful in making the Platelet count 256 000/mm3
diagnosis? Neutrophils 72%
Both a complete blood count (CBC) and C-reactive protein Eosinophils 1%
(CRP) are non-specific for acute mastoiditis; however, an Monocytes 4%
increased CRP and elevated white blood cell count with left Lymphocytes 1%
shift on differential is often noted at the time of diagnosis. Basophils 1%
CBC and CRP can be helpful in distinguishing the severity Bands 11%
of acute mastoiditis. In those patients with a clinical
Downloaded from https://onlinelibrary.wiley.com/doi/ by National Institute Of Standard, Wiley Online Library on [06/03/2024]. See the Terms and Conditions (https://onlinelibrary.wiley.com/terms-and-conditions) on Wiley Online Library for rules of use; OA articles are governed by the applicable Creative Commons License
Chapter 65 Toddler with Fever and Redness Behind Ear 279

it is reasonable for patients who are well appearing


and without evidence of abscess to be discharged
home with outpatient management, most patients present-
ing with acute mastoiditis have already failed outpatient
measures for AOM and will need admission to the hospital
for parenteral treatment. With increasing prevalence of
Staphylococcus aureus, initial treatment should include
an anti-staphylococcal agent until culture and sensitivities
have been obtained. Initial antibiotic choices vary between
centers and regional antibiograms, but common initial
therapy often includes empiric amoxicillin/clavulanic acid
or a second- or third-generation cephalosporin such as
cefuroxime or ceftriaxone with or without the addition of
clindamycin. For patients with complicated acute mastoid-
itis, clinical judgment can be used in adding Gram-negative
coverage for those with high risk for Pseudomonas aerugi-
nosa and in the addition of antibiotic coverage for methicil-
lin-resistant Staphylococcus aureus. If it is possible to obtain
cultures from placement of tympanostomy tube, abscess
drainage, or mastoidectomy, antibiotics should be tailored
Figure 65.1 CT scan. Source: Courtesy of Dr. Karen Moeller.
to susceptibilities.
Some patients with acute mastoiditis will require surgical
What are the common pathogens of this intervention. Often treatment involves myringotomy for
diagnosis? drainage of the middle ear with or without tympanostomy
tube placement for ongoing drainage. Patients with ima-
The most common pathogens to consider when treating
ging demonstrating a subperiosteal abscess will need inci-
acute mastoiditis include Streptococcus pneumoniae, Pseu-
sion and drainage by a trained provider. Some patients
domonas aeruginosa, Streptococcus pyogenes, Haemophilus
may require mastoidectomy if treatment with myringot-
influenzae, and Staphylcoccus aureus. Less common patho-
omy and intravenous antibiotics fails. Recent case reviews
gens, but still isolated in some cases, are Escherichia coli,
show that older children have a longer duration of treat-
Acinetobacter, Enterococcus, Serratia, Klebsiella oxytoca,
ment and are much more likely to need mastoidectomy
and Moraxella catarrhalis.
than younger children. This is likely because older children
take longer to present with clinical signs of acute
What is the treatment for a patient with this
mastoiditis.
diagnosis?
An evaluation by an otolaryngologist or ear, nose and
throat (ENT) specialist is needed. While some studies show

Further Reading
Alkhateeb, A., Morin, F., Aziz, H., et al. (2017). Outpatient International Journal of Pediatric Otorhinolaryngology
management of pediatric acute mastoiditis. International 92: 32–37.
Journal of Pediatric Otorhinolaryngology 102: 98–102. Luntz, M., Brodsky, A., Nusem, S., et al. (2001). Acute
Bilavsky, E., Yarden- Bilvasky, H., Samra, Z., Amir, J., and mastoiditis: the antibiotic era. A multicenter study.
Nussinovitch, M. (2009). Clinical, laboratory and International Journal of Pediatric Otorhinolaryngology
microbiological differences between children with simple or 57: 1–9.
complicated mastoiditis. International Journal of Pediatric Marom, T., Roth, Y., Boaz, M., et al. (2016). Acute mastoiditis
Otorhinolaryngology 73: 1270–1273. in children: necessity and timing of imaging. Pediatric
Chesney, J., Black, A., and Chou, D. (2014). What is the best Infectious Disease Journal 35: 30–34.
practice for acute mastoiditis in children? Laryngscope 124: Mierzwinkski, J., Tyra, J., Haber, K., et al. (2018). Pediatric
1057–1059. recurrent acute mastoiditis: risk factors and insights into
Garcia, C., Salguerio, A.B., Luis, C., Correia, P., and Brito, M.J. pathogenesis. International Journal of Pediatric
(2017). Acute mastoiditis in children: middle ear cultures Otorhinolaryngology 111: 142–148.
may help in reducing use of broad spectrum antibiotics.
Downloaded from https://onlinelibrary.wiley.com/doi/ by National Institute Of Standard, Wiley Online Library on [06/03/2024]. See the Terms and Conditions (https://onlinelibrary.wiley.com/terms-and-conditions) on Wiley Online Library for rules of use; OA articles are governed by the applicable Creative Commons License
281

66

A 5-Year-Old with Right-Sided Weakness


Suzette Meinke1 and Jessica Spruit2
1
Children’s Hospital of Michigan, Detroit, MI, USA
2
Wayne State University College of Nursing, Detroit, MI, USA

A 5-year-old female with sickle cell anemia (homozygous has no asthma-related hospitalizations and uses albuterol
SS type) presents to the emergency department (ED) with metered-dose inhaler as needed for asthma symptoms.
right-sided weakness. Mostly recently, she presented to the ED with complaints
of headache and right hand pain. She reported waking
up with a mild headache and attending school when she
History of Present Illness injured her hand on the bathroom stall. She had a radio-
graph of the right hand that did not demonstrate a fracture.
Mother reports that last evening she noticed the child was She was given intravenous ketorolac and experienced reso-
“walking funny” but thought that she was just “off lution of pain. She remained stable overnight and was dis-
balance,” until this morning when she noticed that the charged home the following day.
child could not sit up by herself. There were no complaints
of pain or fevers. Mother denied any associated upper res-
piratory symptoms or any traumatic events. Mother reports Past Surgical History
that the child was recently seen in the hospital for a history
of headache and right hand pain during which time a frac- She has no prior surgical history.
ture was excluded and they were sent home.

Family History
Past Medical History
Her parents both have the sickle cell trait. Her brother is
She was born in Florida as a full-term infant by cesarean 4 years old and does not have sickle cell disease or trait.
section. She spent no additional time at the hospital after
birth. She is up to date on her vaccines including Pneumo-
vax and her annual influenza vaccine. She has always been Social History
considered a smaller than average child (weight percentiles
range from 1 to 10%) yet her body mass index (BMI) on Born in Florida and moved to Michigan at age 2 years with
presentation was 15%. She has been admitted to the hospi- her mother and brother. She is currently in kindergarten
tal for fever and pain related to her diagnosis of sickle cell and developing appropriately for her age.
disease. Her first transcranial Doppler (TCD) assessment at
age 2 was normal. A repeat study was abnormal 1 year later,
although the subsequent study was normal. She was due for Current Status
an additional study at age 4 years but that was not com-
pleted. She was diagnosed with cough variant asthma at On arrival to the ED, she has a right gaze deviation and
age 3 years after a 1-month history of cough that was worse right arm and leg weakness. Her vital signs are temperature
at night and which did not respond to antihistamines. She 36.7 C (98.1 F), heart rate 117 bpm, respiratory rate

Cases in Pediatric Acute Care: Strengthening Clinical Decision Making, First Edition. Edited by Andrea M. Kline-Tilford and Catherine M. Haut.
© 2020 John Wiley & Sons Ltd. Published 2020 by John Wiley & Sons Ltd.
Downloaded from https://onlinelibrary.wiley.com/doi/ by National Institute Of Standard, Wiley Online Library on [06/03/2024]. See the Terms and Conditions (https://onlinelibrary.wiley.com/terms-and-conditions) on Wiley Online Library for rules of use; OA articles are governed by the applicable Creative Commons License
282 Cases in Pediatric Acute Care

24 breaths per minute, blood pressure 99/67 mmHg, and


oxygen saturation 98% on room air. She is alert, orientated,
and following commands. Deep tendon reflexes are brisk
on the right and normal on the left. Sensation is intact
through all extremities. Cough, gag, and corneal reflexes
are present. She has the ability to wiggle her fingers and
some of her toes but there is minimal large muscle move-
ment on the right side.

Questions

Answer the following questions using the details provided.


1 Based on the data given, what are the differential diag-
noses for this child and the most likely diagnosis?

Rationale and Evidence-based Practice


Explanation

Based on the data given, what are the differential Figure 66.1 Brain MRI. Source: Courtesy of Valarie Eichler.
diagnoses for this child and the most likely
diagnosis?
Trans-ischemic attack, complicated migraine, cerebrovas-
Table 66.1 Basic metabolic profile.
cular accident, and conversion disorder should all be con-
sidered when evaluating this patient. Based on the history Sodium 136 mEq/L
and presentation, the most likely diagnosis for this child is a Potassium 4.8 mEq/L
cerebrovascular accident (CVA).
Chloride 98 mEq/L
Continuation of case: The child was sent immediately for Carbon dioxide 24 mEq/L
magnetic resonance imaging (MRI)/magnetic resonance Blood urea nitrogen 14 mg/dL
venography (MRV)/magnetic resonance angiogram (MRA) Creatinine 0.13 mg/dL
of the brain that demonstrated an ischemic stroke in the left Glucose 74 mg/dL
temporal and parietal regions. She had asymmetrical nar-
rowing of the distal right internal carotid artery and a beaded
appearance of the middle cerebral arteries bilaterally with
possible collaterals (Figure 66.1). The pediatric intensive care Table 66.2 Complete blood count.
unit (PICU) and neurology teams were consulted and she
was transferred to PICU for close monitoring. Complete White blood cell count 20 100/mm3
blood count, comprehensive metabolic panel, and coagula- Hemoglobin 7.5 g/dL
tion studies were obtained (Tables 66.1 and 66.2). Hematocrit 21.6%
Her initial orders on arrival to the PICU included the
Platelet count 415 000/mm3
following:
Reticulocytes 26.5%

•• maintaining pulse oximetry above 92%;


permissive hypertension;
Prothrombin time (PT) 12.2 s

••
International normalized ratio 1.13
continuous cardiorespiratory monitoring; Partial thromboplastin time (PTT) <21 s
echocardiogram;
Downloaded from https://onlinelibrary.wiley.com/doi/ by National Institute Of Standard, Wiley Online Library on [06/03/2024]. See the Terms and Conditions (https://onlinelibrary.wiley.com/terms-and-conditions) on Wiley Online Library for rules of use; OA articles are governed by the applicable Creative Commons License
Chapter 66 A 5-Year-Old with Right-Sided Weakness 283

• normal saline intravenous fluids at 1.5 times


maintenance;
the absence of severe vasculopathy, hydrocycarbamide may
be utilized rather than chronic transfusions to maintain

•• monitoring sodium and glucose every 4 hours;


preventing hypoglycemia/hyperglycemia and hyponatre-
TCD velocities and prevent primary stroke.

mia/hypernatremia;

• monitoring for syndrome of inappropriate antidiuretic


hormone secretion (SIADH);
When a CVA is suspected in a child with sickle cell
anemia, what evaluation and immediate

•• nothing to eat or drink by mouth;


occupational therapy consult for swallow study
interventions should be initiated?
Children who have symptoms of a stroke should be evalu-
evaluation; ated immediately by pediatric neurology and undergo diag-

•• partial exchange transfusion;


neuro checks every 1 hour, lay flat for 48 hours.
nostic imaging, including a CT followed by MRI/MRA. The
stroke team should be activated, if in a center with this
availability. Once neuroimaging confirms an ischemic
stroke, initial therapy should be administered in consulta-
tion with a sickle cell expert. Intravenous fluids should be
Additional Questions and initiated and an exchange transfusion should be performed.
Evidence-based Rationale The goal of the exchange transfusion is to decrease the
hemoglobin S level to below 30%. If it is not possible to per-
2 What diagnostic screening is used to identify a child form an exchange transfusion, a standard packed red blood
who may be at increased risk for a stroke? cell transfusion may be administered, with caution to keep
3 When a CVA is suspected in a child with sickle cell ane- the hemoglobin less than 13 g/dL to prevent further risk of
mia, what evaluation and immediate interventions increased viscosity/sludging.
should be initiated?
4 What are the long-term treatment options for children
with overt strokes to help prevent subsequent strokes? What are the long-term treatment options for
children with overt strokes to help prevent
subsequent strokes?
What diagnostic screening is used to identify a
child who may be at increased risk for a stroke? Children with sickle cell anemia who have a history of overt
strokes are treated with chronic transfusion therapy, which
Children with sickle cell anemia should be screened with is thought to decrease the risk of stroke by decreasing cere-
TCD imaging annually between 2 and 16 years of age. bral blood flow and oxygen extraction fraction. Chronic
TCD imaging detects increased velocities in the large intra- transfusion therapy usually includes monthly simple or
cranial blood vessels, which is secondary to stenosis and exchange transfusions to maintain the hemoglobin
predictive of increased stroke risk. The results of TCD S level below 30%. Because of the risks of chronic transfu-
can guide management; patients with abnormally elevated sions, ongoing monitoring including evaluation of liver
velocity (called conditional or elevated depending on the function, serum ferritin, infectious disease screening for
flow per second) may require chronic transfusions to pre- hepatitis B, hepatitis C and HIV, and evaluation of iron
vent primary stroke. If prevention is not provided, approx- overload should be performed according to periodic evalu-
imately 10% of children with sickle cell anemia will ation recommendations.
experience an overt stroke and an additional 20–35% will
have silent cerebral infarcts. Asymptomatic children do
not require screening with MRI or computed tomogra-
phy (CT). Case Resolution
The goal of chronic transfusion therapy is to reduce the
risk of primary stroke by maintaining the hemoglobin She spent several weeks in the PICU recovering from her
S level less than 30%. Because of the risks associated with stroke. She had a repeat MRI 1.5 weeks after the CVA
chronic transfusions, a multicenter trial was conducted to and a head CT 3 weeks from the CVA. Figures 66.2
evaluate hydroxycarbamide in children with abnormal and 66.3 demonstrate the progression of the CVA. She
TCD velocities and increased risk for stroke. The study con- was transferred to inpatient rehabilitation for fur-
cluded that following 1 year of transfusion therapy and in ther care.
Downloaded from https://onlinelibrary.wiley.com/doi/ by National Institute Of Standard, Wiley Online Library on [06/03/2024]. See the Terms and Conditions (https://onlinelibrary.wiley.com/terms-and-conditions) on Wiley Online Library for rules of use; OA articles are governed by the applicable Creative Commons License
284 Cases in Pediatric Acute Care

Figure 66.3 Head CT 3 weeks after CVA. Source: Courtesy of


Valarie Eichler.

Figure 66.2 Brain MRI 1.5 weeks after CVA. Source: Courtesy of
Valarie Eichler.

Further Reading
Guilliams K.P., Fields, M.E., Ragan, D.K., et al. (2018). Red cell an evidence-based update. Journal of Emergency Medicine 51
exchange transfusions lower cerebral blood flow and oxygen (4): 370–381.
extraction fraction in pediatric sickle cell anemia. Blood 131 Ware, R.E., Davis, B.R., Schultz, W.H., et al. (2016).
(9): 1012–1021. Hydroxycarbamide versus chronic transfusion for
National Heart, Lung, and Blood Institute (2014). Evidence- maintenance of transcranial Doppler flow velocities in
based Management of Sickle Cell Disease: Expert Panel children with sickle cell anaemia, TCD With
Report. Available at https://www.nhlbi.nih.gov/sites/ Transfusion Changing to Hydroxyurea (TWiTCH): a
default/files/media/docs/sickle-cell-disease-report% multi-centre, open-label, phase 3 non-inferiority trial.
20020816_0.pdf Lancet 387: 661–670.
Simon, E., Long, B., and Koyfman, A. (2016). Emergency
medicine management of sickle cell disease complications:
Downloaded from https://onlinelibrary.wiley.com/doi/ by National Institute Of Standard, Wiley Online Library on [06/03/2024]. See the Terms and Conditions (https://onlinelibrary.wiley.com/terms-and-conditions) on Wiley Online Library for rules of use; OA articles are governed by the applicable Creative Commons License
285

67

Postoperative Heart Block


Cathy Woodward
Long School of Medicine, University of Texas Health, San Antonio, TX, USA

A 10-month-old female is admitted to the cardiac intensive saturations 100% on 50% oxygen through mechanical ven-
care unit after complete repair of atrioventricular septal tilation. Approximately 6 hours postoperatively, her RN
defect (AVSD). calls you into the room because she has noticed that the
rhythm has changed. Vital signs have changed and are
now heart rate 60 bpm, blood pressure 55/30 mmHg, and
History of Present Illness oxygen saturations 92% on the same mechanical ventilation
settings as previously postoperatively.
She is a 38-week gestation infant born to a 33-year-old The electrocardiogram (EKG) rhythm strip is shown in
mother. At birth the diagnosis of trisomy 21 was confirmed Figure 67.1.
by chromosome microarray and a diagnosis of AVSD was
made. The infant went home with mother and struggled
with feeding, weight gain, and worsening heart failure Questions
despite medical management.
Answer the following questions using the details provided.

Past Medical History 1 What is your interpretation of the rhythm strip?


2 What are your first interventions?
She has received all age-appropriate immunizations, Continuation of case: The cardiologist and surgeon are
including influenza and palivizumab. Her birthweight notified of the change in the patient’s status. After these
was 3.1 kg and current weight is 7.5 kg. She was bottle- changes, the rhythm shown in Figure 67.2 was noted.
fed with 24 calories/ounce Similac Sensitive, and has just A pulse is present with each contraction and vital signs
started to take cereal. Current medications include multivi- are improving.
tamin daily, furosemide 3 mg orally twice daily, and digoxin
35 μg orally twice daily.

Rationale and Evidence-based Practice


Current Status Explanation

Intraoperative report of brief third-degree heart block when What is your interpretation of the rhythm strip?
coming off cardiopulmonary bypass and atrial/ventricular This strip demonstrates atrioventricular discordance or
temporary pacing wires were placed and are securely third-degree atrioventricular block. The ventricular
attached to a pulse generator that was not turned on imme- response is only 60 bpm.
diately postoperatively. She remained intubated on dexme-
detomidine drip 0.5 μg/kg per hour and milrinone 0.5 μg/kg
What are your first interventions?
per min postoperatively.
Current vital signs are heart rate 130 bpm in sinus Immediate treatment with basic life support is necessary,
rhythm, blood pressure 80/50 mmHg, and oxygen including compressions and manual ventilations while
Cases in Pediatric Acute Care: Strengthening Clinical Decision Making, First Edition. Edited by Andrea M. Kline-Tilford and Catherine M. Haut.
© 2020 John Wiley & Sons Ltd. Published 2020 by John Wiley & Sons Ltd.
Downloaded from https://onlinelibrary.wiley.com/doi/ by National Institute Of Standard, Wiley Online Library on [06/03/2024]. See the Terms and Conditions (https://onlinelibrary.wiley.com/terms-and-conditions) on Wiley Online Library for rules of use; OA articles are governed by the applicable Creative Commons License
286 Cases in Pediatric Acute Care

Figure 67.1 Rhythm strip.

Figure 67.2 Rhythm strip with ventricular pacing.

the pulse generator is prepared. Since there is atrial electri- surgical area. Since this infant had sinus rhythm on emer-
cal activity which is being blocked and a slow ventricular ging from surgery followed by development of atrioventric-
response, an emergent setting on the pulse generator (e.g. ular block, the prognosis was encouraging for return of
pacemaker) would be ventricular pacing at a rate of spontaneous conduction as the edema and inflammation
100–130 bpm based on vital signs and perfusion. resolved. Temporary pacing is the best option and most cen-
ters place pacing wires in the operating room for high-risk
procedures. If pacing wires are not immediately available,
Additional Information transcutaneous pacing could be attempted while making
surgical plans for placement of pacing wires. The decision
Atrioventricular block is a risk factor for children under- to place a permanent pacemaker is usually made 7–14 days
going cardiac surgery. The surgeries most commonly asso- after surgery if the blocked rhythm continues.
ciated with heart block requiring permanent pacemaker
placement are the arterial or atrial switch and repair of
tricuspid or mitral valve. In one multicenter study, approx-
imately 4.1% of pediatric cardiac surgery patients developed Case Resolution
second- or third-degree heart block after surgery. In this
case, the patient underwent a repair of AVSD which can There was a return of conduction 3 days after surgery and
involve the atrioventricular node conducting system. Often, the pacing wires were removed on postoperative day 5 and
the heart block occurs immediately postoperatively due to the infant continued to recover uneventfully from the sur-
direct injury or to edema and inflammation around the gical procedure.

Further Reading
Liberman, L., Silver, E.S., Chai, P.J., and Anderson, B.R. Journal of Thoracic and Cardiovascular Surgery 152(1):
(2015). Incidence and characteristics of heart block after 197–202.
heart surgery for pediatric patients: a multicenter study.
Downloaded from https://onlinelibrary.wiley.com/doi/ by National Institute Of Standard, Wiley Online Library on [06/03/2024]. See the Terms and Conditions (https://onlinelibrary.wiley.com/terms-and-conditions) on Wiley Online Library for rules of use; OA articles are governed by the applicable Creative Commons License
287

68

Recognizing Human Trafficking


Christine DiPaolo
Nemours/Alfred I. duPont Hospital for Children, Wilmington, DE, USA

A 15-year-old adolescent girl was recently placed in foster Past Medical History
care when her mother trafficked her for sex in order to
pay the rent. She was last seen in this practice when she The patient was born at term and has had no previous sig-
was 13 years old for irregular periods. nificant medical history. Immunizations current except
human papillomavirus (HPV) has not initiated. Has never
been hospitalized.
She experienced initial menses at 12 years old. Periods
History of Present Illness are monthly, lasting 5 days. She uses four tampons a day
and they are not soaked. She has cramps but does not need
This teen presents to the adolescent practice with medication. Last menstrual period was 2 days ago.
her foster mother for sexually transmitted infection
(STI) screening and birth control counseling. She
endorses vaginal discharge and odor. The patient was Past Surgical History
recently placed in foster care after she disclosed to a
friend that for the past year she was having sexual inter- No previous surgical procedures.
course with her mother’s boyfriend and a couple of his
male friends in exchange for rent money. The teen’s
friend shared this with her mother who contacted the
Family History
police, resulting in a referral to family social services
Mother has addiction and was diagnosed with bipolar dis-
and the teen being removed from her home. She was
order. Father’s history is unknown, as is history of paternal
not evaluated in the emergency room since the abuse
grandmother and paternal grandfather. Sister aged 4 years
had occurred over 72 hours previous to family services
is healthy. Maternal grandmother with depression, mater-
involvement.
nal grandfather has history of addiction
The teen has not had contact with her mother up to this
point. She admits to smoking marijuana and taking “pills”
sometimes. She was getting marijuana and other medicines Social History
from her mother’s boyfriend, and is not sure what the pills
were but thinks Xanax. The patient states, “It made it easier Currently lives with foster mother, foster father, 14-year-
to be with them.” She reports the last time she had sex was old foster sister, and her 4-year-old biological sister. Prior
2 weeks ago and “they never used condoms. They didn’t to foster placement she was living in a two-bedroom apart-
care if I got pregnant. Sometimes they would laugh at the ment with mother, mother’s boyfriend, and her 4-year-old
possibility of my getting pregnant.” She reports being a biological sister. She denies being physically abused, except
virgin prior to having sex with her mother’s friends. She for forced intercourse and other sexual activities. The
reports it was mostly vaginal sex and sometimes oral. patient admits to smoking marijuana almost daily which
Denies anal sex. she receives from mother’s boyfriend. Has not smoked

Cases in Pediatric Acute Care: Strengthening Clinical Decision Making, First Edition. Edited by Andrea M. Kline-Tilford and Catherine M. Haut.
© 2020 John Wiley & Sons Ltd. Published 2020 by John Wiley & Sons Ltd.
Downloaded from https://onlinelibrary.wiley.com/doi/ by National Institute Of Standard, Wiley Online Library on [06/03/2024]. See the Terms and Conditions (https://onlinelibrary.wiley.com/terms-and-conditions) on Wiley Online Library for rules of use; OA articles are governed by the applicable Creative Commons License
288 Cases in Pediatric Acute Care

for the past 2 weeks since being in foster care. Feels she does institution, including acute care settings. Human traffick-
not miss it. She also takes “pills” that she thinks are Xanax ing is defined by the United States Victims of Trafficking
“once in a while.” Last used 2 months ago. She denies and Violence Protection Act as the “recruitment, harbor-
alcohol or any other drug use, also denies juuling or vaping. ing, transportation, provision or obtaining of a person for
The teen does report having a boyfriend, who attends her labor or services through the use of force, fraud or coercion”
school, for the past 3 months. Feels they have a good rela- (US Trafficking in Persons Report, 2019). Commercial sex-
tionship and is having consensual sex with him. Is not using ual exploitation of children, otherwise known as child sex
condoms consistently in this relationship. trafficking, is a major public health issue in the United
The teen is in 10th grade. She used to be a B student but States and globally. Children can be sexually trafficked over
now gets Cs and Ds. Her favorite subject is science. She likes international borders, within the United States, a state or a
to dance and listen to music. single neighborhood, or in this case a specific home.
Domestic minor sex trafficking (DMST) refers to those chil-
dren or teens who are US citizens under the age of 18 who
Current Status are trafficked within the borders of the United States.
It is difficult to document the prevalence of DMST as
The only positive in the review of systems is genitourinary/ there are many variables, especially related to reporting,
gynecology, with lower persistent abdominal pain with vag- but one source conservatively estimates that 200 000 US
inal discharge. The physical examination is basically nor- minors are involved (Goldberg and Moore, 2018). Risk
mal except for the pelvic examination, which indicates factors for involvement include history of child abuse
pale thick discharge noted at the cervical os, positive for or neglect, family dysfunction or conflict, homelessness,
cervical motion tenderness. foster care, LGBTQ, legal system involvement, peer pres-
Diagnostic testing for STI is positive for Chlamydia, and sure, social isolation, gang involvement, and under-
the patient is treated for pelvic inflammatory disease with resourced schools, neighborhoods and communities
Doxycycline 100 mg orally twice daily for 14 days and intra- and overall lack of resources. Knowledge deficit or lack
muscular ceftriaxone 250 mg. Blood test for HIV and rapid of awareness of commercial sexual exploitation is
plasma reagin (RPR) test for syphilis are completed and another factor. In this case, the patient had a relationship
prove negative. with the traffickers and the parent and children were at
The patient is counseled about birth control and Nexpla- risk with a lack of financial resources to survive. Traffick-
non placed per patient decision. ers can have a relationship with the victim or can be
strangers. In addition, victims of human trafficking are
often forced to use illicit drugs, such as the marijuana
Questions and Xanax given to this teen, which may in turn result
in an addiction problem for the teen. The use of drugs
Answer the following questions using the details provided. can be coercive on the side of the trafficker and/or a cop-
ing mechanism for the child.
1 Where does human trafficking occur and what different Healthcare providers need to be aware and sensitive to
types exist? the possibility of human trafficking, especially when chil-
2 What signs or complaints are common when children dren present to a non-familiar clinic, practice, urgent care
who are being trafficked come for healthcare? or emergency department with concerns of drug or alcohol
3 What screening can be done in any environment to abuse, STIs, suicidal ideation or physical trauma. Youth
identify children who are being trafficked? may be fearful of their exploiters, distrust the clinician,
or have an interest in protecting their traffickers, so will
not describe their experiences or the real reasons for their
complaints. Providers are often not trained nor equipped to
Rationale and Evidence-based Practice
acknowledge these victims.
Explanation

Where does human trafficking occur and what What signs or complaints are common when
different types exist? children who are being trafficked come for
healthcare?
It is important for providers in any environment to under-
stand human trafficking and its implications for children Signs of involvement in DMST include the presence of
and teens. These patients can present to any healthcare tattoos, evidence of physical abuse or neglect such as
Downloaded from https://onlinelibrary.wiley.com/doi/ by National Institute Of Standard, Wiley Online Library on [06/03/2024]. See the Terms and Conditions (https://onlinelibrary.wiley.com/terms-and-conditions) on Wiley Online Library for rules of use; OA articles are governed by the applicable Creative Commons License
Chapter 68 Recognizing Human Trafficking 289

malnutrition or wound infections, multiple or recurrent and confirming this identity is also important, although this
STIs, multiple sexual partners, inappropriate clothing or may be difficult in a “walk-in” setting, where this type of
expensive items such as shoes or purses, and cutting or other questioning may cause the patient to leave without treat-
self-harm behaviors. Psychological or mental health signs ment. The child should also be asked in confidence about
can include depression, withdrawal, fear, distrust and indi- where they are living, if they have food, shelter and cloth-
cations of illicit drug or alcohol use. In addition, there are ing, attending school and other activities, forced or coerced
social signs such as controlling behavior on the part of the sexual experiences, having sex with someone other than the
person accompanying the patient, history of running away trafficker, or having sex in exchange for food, clothing, shel-
from home, or involvement with child protection services. ter or other physical items. These are just some of the
recommended interview questions that can assist in expos-
ing the trafficking.
What screening can be done in any environment to
A most important part of addressing human trafficking is
identify children who are being trafficked?
awareness and prevention. Whether in well care visits or in
Interviewing the child away from the adult who accompa- any healthcare setting it is most appropriate to discuss this
nies the patient is important; asking the adult for identity problem, especially with high-risk youth.

References
Goldberg, A. and Moore, J. (2018). Domestic minor sex US Trafficking in Persons Report, June 2019. https://www.
trafficking. Child and Adolescent Psychiatric Clinics of North state.gov/wp-content/uploads/2019/06/2019-Trafficking-in-
America 27(1): 77–92. doi: 10.1016/j.chc.2017.08.008 Persons-Report.pdf

Further Reading
Moore, J.L., Kaplan, D.M., and Barron, C.E. (2017). Sex Reap, V.J. (2019). Sex trafficking: a concept analysis for health
trafficking of minors. Pediatric Clinics of North America 64 care providers. Advanced Emergency Nursing Journal 41(2):
(2): 413–421. doi: 10.1016/j.pcl.2016.11.013 183–188. doi: 10.1097/TME.0000000000000236
Downloaded from https://onlinelibrary.wiley.com/doi/ by National Institute Of Standard, Wiley Online Library on [06/03/2024]. See the Terms and Conditions (https://onlinelibrary.wiley.com/terms-and-conditions) on Wiley Online Library for rules of use; OA articles are governed by the applicable Creative Commons License
291

69

A Toddler With Stridor


Mari Welch
Nemours/Alfred I. duPont Hospital for Children, Wilmington, DE, USA

A 21-month-old male is admitted to the pediatric intensive Past Surgical History


care unit (PICU) with a 1-day history of increased work of
breathing and harsh cough. None.

History of Present Illness


Family History
This patient is a 21-month-old male with no significant past
Non-contributory. Parents are both healthy, and there is a
medical history who was diagnosed with acute otitis media
younger sibling.
(AOM) and started on a 10-day course of amoxicillin 4 days
prior to admission in the PICU. He initially had fever,
which resolved after antibiotics were initiated. He was
noted to have developed rhinorrhea, significant barky Current Status
cough, and hoarseness 2 days prior to the day of admission.
He also had decreased oral intake. On arrival at an outlying hospital ED, the patient is noted to
The toddler was with grandparents on the day of presen- have inspiratory and expiratory stridor. His initial respira-
tation when they noticed that his cough was worsening, tory rate is 50 breaths per minute with oxygen saturation of
with increasing work of breathing and wheezing. He had 88% and moderate supraclavicular and subcostal retrac-
significantly decreased oral intake and number of wet dia- tions. He is given dexamethasone (Decadron) and three
pers. These concerns prompted the grandparents to bring 0.5-mL doses of racemic epinephrine 2.25%, and is started
him to the emergency department (ED). on 100% oxygen via high-flow nasal cannula (HFNC) at 8 L/
The patient’s mother reports that the 11-month-old min, with minimal improvement in work of breathing. As
brother was diagnosed with croup 6 days ago. he continues to exhibit increased work of breathing, he is
placed on continuous positive airway pressure (CPAP) sup-
port. Despite ongoing administration of racemic epineph-
Past Medical History rine and CPAP support, his work of breathing continues
to worsen, with persistent oxygen desaturation and worsen-
This child was born at 38 week’s gestation after a normal ing retractions. A venous blood gas (VBG) analysis demon-
pregnancy and normal perinatal and early neonatal periods strates respiratory acidosis, pH of 6.94, PCO2 100 mmHg,
and was discharged home from the newborn nursery on and bicarbonate 25 mg/dL. He is emergently intubated
day 1 of life. All required immunizations are up to date for hypercarbic respiratory failure and placed on the venti-
including annual influenza vaccine. lator with settings of FiO2 0.4, positive end-expiratory pres-
His present weight and height are at the 11th percentile sure (PEEP) 5 cmH2O, tidal volume 60 mL, inspiratory time
for age, and he has remained on the same growth curve 0.6 seconds, and rate 30/min. A chest X-ray demonstrates
since 6 months of age. diffuse hazy bilateral airspace opacity without evidence

Cases in Pediatric Acute Care: Strengthening Clinical Decision Making, First Edition. Edited by Andrea M. Kline-Tilford and Catherine M. Haut.
© 2020 John Wiley & Sons Ltd. Published 2020 by John Wiley & Sons Ltd.
Downloaded from https://onlinelibrary.wiley.com/doi/ by National Institute Of Standard, Wiley Online Library on [06/03/2024]. See the Terms and Conditions (https://onlinelibrary.wiley.com/terms-and-conditions) on Wiley Online Library for rules of use; OA articles are governed by the applicable Creative Commons License
292 Cases in Pediatric Acute Care

Table 69.1 Venous blood gas analysis before and after Rationale and Evidence-based Practice
intubation.
Explanation
Before intubation After intubation
Based on the data given, what are the differential
pH 6.94 7.15 diagnoses and the most likely diagnosis for
PCO2 (mmHg) 100 82
this child?
PO2 (mmHg) 80 73 The differential diagnoses for this child include the
– following.
HCO3 (mmol/L) 25.6 28.6

• Croup (laryngotracheobronchitis): associated with viral


illness.
Table 69.2 Complete blood count.

White blood cell count 11 460/mm3


• Acute epiglottitis (rare in setting of Haemophilus influen-
zae vaccination): patients have toxic appearance, sitting
upright and drooling.
Hemoglobin
Hematocrit
11.4 g/dL
33.5% • Peritonsillar abscess: patients sit upright, drooling, with
extended neck, neck swelling and muffled voice.
Platelet count 292 000/mm3
• Retropharyngeal abscess: similar in presentation to peri-
tonsillar abscess, with neck stiffness, barky cough is not
present.

Table 69.3 Basic metabolic panel.


• Foreign body aspiration: often a history of acute choking,
coughing, and focal wheeze.

Sodium 136 mEq/L • Anaphylaxis: acute presentation without cold symptom


prodrome.
Potassium 3.7 mEq/L
Chloride 102 mEq/L Croup is the most likely diagnosis due to the presenting
Carbon dioxide 29 mEq/L
symptoms of hoarseness, barky cough, and stridor, after a
few days of coryza and viral prodrome.
Blood urea nitrogen 9 mg/dL
Creatinine 0.3 mg/dL
What is the pathophysiology of the diagnosis?
Glucose 270 mg/dL
Calcium 8.8 mg/dL
A viral infection causes inflammation of the larynx, tra-
chea, bronchi, and occasionally bronchioles, and lung
parenchyma. This inflammation leads to narrowing of
the laryngeal area and tracheal and bronchial lumens, caus-
of pneumothorax or pleural effusion. Respiratory viral ing obstruction that is pronounced in the subglottic region.
panel and repeat VBG analysis, complete blood count, basic Obstruction increases the work of breathing, occasionally
metabolic panel and blood culture are all obtained and the resulting in hypercapnia and respiratory failure. Atelectasis
results are shown in Tables 69.1–69.3. may occur concurrently if the bronchioles become
obstructed. Croup is most common in children aged
6 months to 3 years. Symptoms of croup are almost always
preceded by a viral prodrome.
Questions
What are the indications for admission?
Answer the following questions using the details provided.
Only 8–15% of patients with croup require hospitalization,
1 Based on the data given, what are the differential diag- of which 1% are admitted to the PICU (Rosychuk
noses and the most likely diagnosis for this child? et al., 2010).
2 What is the pathophysiology of the diagnosis? Severe croup leads to poor air entry, significant stridor at
3 What are the indications for admission? rest, altered consciousness, impending respiratory failure,
4 What are the recommended therapies and their persistent or deteriorating respiratory distress after treatment
mechanisms of action? with nebulized epinephrine and corticosteroids, “toxic”
Downloaded from https://onlinelibrary.wiley.com/doi/ by National Institute Of Standard, Wiley Online Library on [06/03/2024]. See the Terms and Conditions (https://onlinelibrary.wiley.com/terms-and-conditions) on Wiley Online Library for rules of use; OA articles are governed by the applicable Creative Commons License
Chapter 69 A Toddler With Stridor 293

appearance or clinical picture suggesting serious secondary children may require fluid boluses and maintenance
bacterial infection, need for supplemental oxygen, and severe intravenous fluid administration.
dehydration or intolerance of adequate oral intake.
The indications for PICU admission include:
• Intubation: rarely indicated, with less than 3% of croup
admissions to the PICU requiring intubation. Intubation


should be performed by a highly skilled practitioner
respiratory failure with imminent respiratory arrest, (ENT, anesthesiology), because airway edema may make
requiring endotracheal intubation;


intubation difficult. The median duration for intubation
persistent severe symptoms requiring frequent nebulized is 60 hours (Tyler et al., 2017).


epinephrine dosing;
underlying conditions placing the child at high risk for • Diagnostic testing: a neck radiograph is often considered
for children with croup. However, croup is a clinical diag-
progressive respiratory failure (e.g. neuromuscular dis- nosis and an X-ray is not required nor is there always an
ease or bronchopulmonary dysplasia) (Petrocheilou evident classic “steeple” sign.
et al., 2014).
Continuation of case: The patient’s respiratory viral panel
is positive for coronavirus OC43. He patient is admitted to
What are the recommended therapies and their the PICU and placed on conventional ventilator, with set-
mechanisms of action? tings of PEEP 5 cmH2O, tidal volume 6 mL/kg (60 mL), rate

•• Supportive care: humidified air, antipyretics.


Glucocorticoids: the mainstay of treatment for croup.
35/min, and FiO2 0.3. He is started on dexamethasone
(Decadron) 0.6 mg/kg every 6 hours. A bronchoalveolar
lavage is obtained and sent for evaluation. Subsequent
Glucocorticoids decrease the severity of croup symptoms
VBG analyses are normal.
in 2–24 hours, reducing hospital length of stay and rate of
On day 2 of admission, the toddler is weaned to CPAP and
return visits (Gates et al., 2018). The mechanism of action
extubated to helium/oxygen mixture (Heliox 70/30) combi-
involves reduction of inflammation by suppression of
nation via HFNC on day 3 of admission. He completes four
neutrophil migration, decreased production of inflam-
doses of Decadron and receives another four doses for per-
matory mediators, and reversal of capillary permeability.
sistent stridor post extubation. A nasoduodenal tube is
Dosing in infants and children is 0.6 mg/kg, routinely
placed on PICU day 3 prior for nutrition.
used once, but in certain situations where the patient
The child is weaned to room air on day 5 of admission,
exhibits recurrent stridor it has been repeated.

• Racemic epinephrine: nebulized racemic epinephrine


2.25% should be considered for hypoxia or severe croup.
and at this time he is breathing at a rate of 36/min with
minimal intermittent inspiratory stridor. He is transferred
to the regular inpatient care area, but restarted on Heliox
The adrenergic effects induce vasoconstriction, leading to
for worsening stridor and hypoxia, receiving two further
decreased subglottic edema. Onset is less than 10 minutes
doses of racemic epinephrine and is therefore transferred
with duration of 2–3 hours; patient therefore requires
back to the PICU.
observation for 3–4 hours for recurrence of stridor before
discharge.

• Heliox: There is evidence to suggest that Heliox therapy,


which comprises 70% helium and 30% oxygen, may be
Additional Question and Evidence-
based Rationale
used for moderate to severe distress or illness not respon-
sive or refractory to racemic epinephrine, and before the How would you determine if the patient is ready
full effects of glucocorticoids are established (Moraa for discharge?
et al., 2013). It works by establishing laminar gas flow
Typically, croup is self-limited and once the acute period
in edematous airways, thereby decreasing the effect of
has resolved it is unlikely for severe symptoms to recur.
increased airway resistance and decreases the respiratory
Indications for discharge include:
muscle work of breathing. If oxygen needs are more than


30%, then Heliox may not be effective.
Hydration: an assessment of the child’s hydration status
•• no stridor at rest;
normal pulse oximetry in room air;
is also necessary. Children with respiratory distress have
insensible fluid loss requirements. Also, children who are
•• good air exchange;
normal color;
ill may have had difficulty drinking adequate volumes of
fluids preceding the hospitalization. These infants and
•• normal level of consciousness;
demonstrated ability to tolerate fluids by mouth.
Downloaded from https://onlinelibrary.wiley.com/doi/ by National Institute Of Standard, Wiley Online Library on [06/03/2024]. See the Terms and Conditions (https://onlinelibrary.wiley.com/terms-and-conditions) on Wiley Online Library for rules of use; OA articles are governed by the applicable Creative Commons License
294 Cases in Pediatric Acute Care

References
Gates, A., Gates, M., Vandermeer, B., et al. (2018). Rosychuk, R.J., Klassen, T.P., Metes, D., et al. (2010). Croup
Glucocorticoids for croup in children. Cochrane Database of presentations to emergency departments in Alberta,
Systematic Reviews (8): CD001955. Canada: a large population-based study. Pediatric
Moraa, I., Sturman N., McGuire T., and van Driel, M.L. (2013). Pulmonology 45(1): 83–91.
Heliox for croup in children. Cochrane Database of Tyler, A., McLeod, L., Beaty, B., et al. (2017). Variation in
Systematic Reviews (12): CD006822. inpatient croup management and outcomes. Pediatrics 139
Petrocheilou, A., Tanou, K., Kalampouka, E., et al. (2014). Viral (4): pii: e20163582. doi: 10.1542/peds.2016-3582.
croup: diagnosis and a treatment algorithm. Pediatric
Pulmonology 49: 421–429.

Further Reading
Cherry, J.D. (2008). Clinical practice. Croup. New England
Journal of Medicine 358(4): 384–391.
Downloaded from https://onlinelibrary.wiley.com/doi/ by National Institute Of Standard, Wiley Online Library on [06/03/2024]. See the Terms and Conditions (https://onlinelibrary.wiley.com/terms-and-conditions) on Wiley Online Library for rules of use; OA articles are governed by the applicable Creative Commons License
295

70

Delirium in a 9-Year-Old Boy


Heather Herrera
Children’s Hospital of San Antonio, San Antonio, TX, USA

The patient is a 9-year-old male with sepsis, ventilated with Family History
sedation and analgesia, who has developed delirium.
The patient lives with the paternal grandmother, as neither
parent is involved in his care. Grandmother has hyperten-
History of Present Illness sion and chronic lung disease. The father has drug depend-
ency and is in and out of rehabilitation and jail. The
This 9-year-old boy presented to the pediatric intensive care grandmother does not know where the mother of the child
unit (PICU) 5 days ago with septic shock secondary to is and does not know her health history.
pneumonia. He became hypoxic with subsequent respira-
tory failure unresponsive to non-invasive ventilation, so
was intubated and placed on a ventilator. The patient then
Current Status
developed fluid-refractory shock and required vasopressor
support. After 48 hours, his respiratory status began to
Over the last 2 days, the patient has been noted to be awake
improve along with his blood pressure, and vasopressor
more at night than during the day. Today, his behavior has
support was weaned off. He remained on a ventilator for
ranged from being intermittently quiet and non-interactive
a total of 6 days, with sedation and pain management
to agitated, crying, and fearful of a “man” he is seeing in the
achieved with morphine and midazolam continuous infu-
corner of his room. He will occasionally correctly state his
sions. As his status has improved, sedation has been
name and where he is, but then reverts back to being con-
weaned to allow him to be more awake yet comfortable.
fused and unable to identify family members or his current
location. After assessing this boy, the bedside provider
orders delirium scoring using the Cornell assessment of
Past Medical History pediatric delirium.

This patient was born at term via spontaneous vaginal


delivery and discharged home with his mother. He has
received required immunizations along with a flu vaccine
Questions
this season. His weight has been steady at 75% for gender
Answer the following questions using the details provided.
and age and his current body mass index (BMI) is 22.5.
He has had no previous hospitalizations. 1 What is the pathophysiology of delirium and what fac-
tors can predispose a patient to develop this problem?
2 What factors contribute to the development of
Past Surgical History delirium?
3 What are the different types of delirium and what are
This child had tympanostomy tubes placed at 15 months of the available screening tools?
age for frequent episodes of otitis media; no complications 4 What are the best ways to optimize therapy: environ-
during the procedure or postoperatively. mental or pharmacologic?

Cases in Pediatric Acute Care: Strengthening Clinical Decision Making, First Edition. Edited by Andrea M. Kline-Tilford and Catherine M. Haut.
© 2020 John Wiley & Sons Ltd. Published 2020 by John Wiley & Sons Ltd.
Downloaded from https://onlinelibrary.wiley.com/doi/ by National Institute Of Standard, Wiley Online Library on [06/03/2024]. See the Terms and Conditions (https://onlinelibrary.wiley.com/terms-and-conditions) on Wiley Online Library for rules of use; OA articles are governed by the applicable Creative Commons License
296 Cases in Pediatric Acute Care

Rationale and Evidence-based Practice mobility, disruption of sleep–wake cycle, and lack of
Explanation familiar environment and supports (Malas et al., 2017).
Sedatives may also contribute to the development of
What is the pathophysiology of delirium and what delirium in critically ill pediatric patients. A possible
factors can predispose a patient to develop this replacement therapy is the use of dexmetomidine which
problem? has been noted to prevent and/or treat delirium; however,
it is not currently FDA approved for the treatment of pedi-
Delirium is best described as an acute brain dysfunction or atric delirium. Identification and treatment of iatrogenic
a disruption in cerebral homeostasis with significant effects withdrawal syndrome (IWS) is also very important. Once
on patient outcomes, medical costs, morbidity, and mortal- IWS has been managed, if the patient continues to experi-
ity. Although delirium is multifactorial, some medications, ence symptoms of delirium, environmental modifications
such as benzodiazepines, glucocorticoids, anesthetics, may be needed as increasing sedative medications may
diuretics, opiates, and antihistamines, are thought to poten- actually end up prolonging delirium (Patel et al., 2017).
tiate its development (Patel et al., 2017). Children under the
age of 2 years and those with developmental delay, high
severity of illness, low albumin, mechanical ventilation, What are the different types of delirium and what
and preexisting medical condition(s) are predisposed to are the available screening tools?
the development of delirium (Patel et al., 2017). There The Diagnostic and Statistical Manual of Mental Disorders
are three hypotheses for the development of pediatric delir- recognizes three types of delirium: hypoactive, hyperactive,
ium as discussed by Patel et al. (2017); neuro-inflammatory, and mixed type (Patel et al., 2017). Typical symptoms of
neurotransmitter, and oxidative stress. According to the hypoactive delirium include lethargy, inattentiveness,
neuro-inflammatory hypothesis, systemic inflammation, and decreased responsiveness. Hyperactive delirium is
such as that seen in critical illness, “leads to either compro- characterized by agitation, restlessness, hypervigilance,
mise in the integrity of the blood–brain barrier or de novo and combative behavior. Mixed-type delirium is character-
production of inflammatory products within the brain.” ized by a combination of both hypoactive and hyperactive
The neurotransmitter hypothesis relates to use of medica- symptoms. Hypoactive delirium has been associated with
tions that change neurotransmitter function and the oxida- poor prognosis and occurs at the greatest frequency. Emo-
tive stress hypothesis “suggests that reduced oxygen tional lability as well as visual and auditory hallucinations
delivery in critical illness, coupled with increased cerebral can also be experienced by pediatric patients with delirium
metabolism, leads to the production of reactive oxygen spe- along with disordered sleep. Delirium most often occurs
cies that cause global CNS dysfunction.” It is important to early in the ICU course of children, with median time of
note that hypoxia has been clearly associated with delirium onset 1–3 days and typically lasting about 2 days. About
development. one-third of pediatric patients who develop delirium early
in their hospital course will experience recurrent episodes
during their ICU stay. Children with mild to severe delir-
What factors contribute to the development
ium can experience longer time on mechanical ventilation,
of delirium?
longer hospital length of stay, longer time to emerge from
Factors contributing to the development of pediatric delir- sedation, longer time to extubation, and higher resource
ium in critically ill children can be separated into three cate- utilization (Patel et al., 2017). Delirium is often a temporary
gories: predisposing, precipitating, and perpetuating (Malas condition, although it has been strongly linked with poor
et al., 2017). Predisposing risk factors cannot be modified outcomes, including increase in mortality and enduring
and include age less than 2, developmental delay, high cognitive impairment in survivors (Patel et al., 2017). In
severity of illness, low albumin, genetic susceptibility, 2013, the Society of Critical Care Medicine (SCCM)
mechanical ventilation, and preexisting medical conditions. released guidelines recommending that screening for delir-
Precipitating factors are treatment related and can poten- ium become a standard of care in adults, but there are not
tially be modified for the patient, such as anticholinergic guidelines specific to pediatrics at this time.
medications, benzodiazepines, cardiac bypass surgery, Current assessment of delirium includes two versions of
immobilization, prolonged length of stay in the intensive validated pediatric delirium screens: the Pediatric Confu-
care unit (ICU), and restraints. Perpetuating factors are sion Assessment Method for the ICU with two versions,
found in the patient’s care environment, including lack of the pCAM-ICU for children older than age 5 years and
Downloaded from https://onlinelibrary.wiley.com/doi/ by National Institute Of Standard, Wiley Online Library on [06/03/2024]. See the Terms and Conditions (https://onlinelibrary.wiley.com/terms-and-conditions) on Wiley Online Library for rules of use; OA articles are governed by the applicable Creative Commons License
Chapter 70 Delirium in a 9-Year-Old Boy 297

the psCAM-ICU for those aged 6 months to 5 years. The can reduce delirium risk and improve time to improvement
other option is the CAPD, which is an observation screen (Malas et al., 2017).
useful for patients aged 0–21; it can provide a picture of a If physical and environmental adjustments have not been
pediatric patient over a period of time and has been vali- successful, a trial of pharmacologic therapy can begin.
dated in both developmentally normal and developmen- Atypical antipsychotics are recommended because of their
tally delayed children (Patel et al., 2017). Prior to precognitive effects and favorable side-effect profile (Patel
assessing a child for delirium, their level of consciousness et al., 2017), but this use is off-label at this time. Haloperidol
should be assessed using a tool such as the RASS (Smith has been used clinically for many years to manage delirium
et al., 2013). Routine monitoring of critically ill children symptoms, but also comes with side effects (Turkel, 2017).
should occur at least once per shift. The challenge is provid- When managing pediatric delirium with atypical antipsy-
ing the most minimal sedation required to keep patients chotics, quetiapine, risperidone, and olanzapine are cur-
comfortable, especially those requiring mechanical ventila- rently the usual first choices. As mentioned previously,
tion or other invasive treatments, along with assessment dexmetomidine can also be used. The ability to discontinue
and treatment for delirium. pharmacologic therapy should be discussed no later than
5–7 days after initiation (Children’s Hospital of Philadel-
What are the best ways to optimize therapy: phia, 2019).
environmental or pharmacologic? Prevention of delirium should be the first goal of ICU
care, followed by astute evaluation. The risk of developing
When delirium is diagnosed in a critically ill child, the key
delirium in the ICU environment is high, so early evalua-
to effective treatment is to identify the underlying cause
tion and intervention can assist in management. With
(Patel et al., 2017). Delirium is most often caused by a com-
improved awareness of pediatric delirium and its adverse
bination of factors, including the ICU environment, iatro-
effects, implementation of improved sedation techniques,
genic effects of treatment, and an effect of the underlying
early mobilization, and involvement of family members
illness. Assessment for infection, treating hypoxemia, opti-
in daily care, delirium may be able to be prevented in
mizing pain management, and correcting any metabolic
critically ill children.
abnormalities are some recommendations. Iatrogenic
factors should be determined, followed by environmental
modifications to help improve symptoms; alternatively,
pharmacologic treatment may be needed. Case Resolution
Modifications of the child’s environment can help in
decreasing the risk of delirium as well as aiding in alleviat- The patient scored 17 on the delirium scale, which indicates
ing symptoms. This can include adjusting the room temper- the presence of delirium. His nurses scheduled a daily rou-
ature and lighting to reflect day and night time, reducing tine and family brought some familiar objects from home.
noise, and decreasing the use of physical restraints (when Nursing was also encouraged to cluster care at night for
safe). Objects and individuals familiar to the child should fewer disturbances. The child initially had improvement
be close by. Consistent caregivers along with stable daily in his symptoms, but he continued to have moments of
routines can also reduce the risk of confusion and help reg- confusion and hallucinations. He was thus started on an
ulate the sleep–wake cycle. Reorientation strategies have antipsychotic medication to help allay delirium symptoms.
been shown to produce a statistically significant decrease Within a week of starting the medication, his symptoms
in symptoms. It is important to address both sensory and completely resolved and the patient returned to his neuro-
communication limitations when able. Early mobilization logic baseline.

References
Children’s Hospital of Philadelphia (2019). CICU/PCU/PICU considerations. Current Psychiatry Reports 19(9): 65.
Clinical pathway for delirium. http://www.chop.edu/ doi: 10.1007/s11920-017-0817-3.
clinical-pathway/picu-pcu-delirium-clinical-pathway Patel, A.K., Bell, M.J., and Traube, C. (2017). Delirium in
Malas, N., Brahmbhatt, K., McDermott, C., et al. (2017). pediatric critical care. Pediatric Clinics of North America
Pediatric delirium: evaluation, management, and special 64(5): 1117–1132. doi: 10.1016/j.pcl.2017.06.009.
Downloaded from https://onlinelibrary.wiley.com/doi/ by National Institute Of Standard, Wiley Online Library on [06/03/2024]. See the Terms and Conditions (https://onlinelibrary.wiley.com/terms-and-conditions) on Wiley Online Library for rules of use; OA articles are governed by the applicable Creative Commons License
298 Cases in Pediatric Acute Care

Smith, H.A.B., Brink, E., Fuchs, D.C., Wesley, E., and Turkel, S.B. (2017). Pediatric delirium: recognition,
Pandharipande, P.P. (2013). Pediatric delirium monitoring management, and outcome. Current Psychiatry Reports 19
and management in the pediatric intensive care unit. (12):101. doi: 10.1007/s11920-017-0851-1.
Pediatric Clinics of North America 60(3): 741–760. doi:
10.1016/j.pcl.2013.02.010.

Further Reading
Children’s Hospital of Philadelphia (2019). Clinical team to
bedside to assess patient: BRAIN MAPS. http://www.chop.
edu/clinical-pathway/picu-pcu-delirium-brain-maps
Downloaded from https://onlinelibrary.wiley.com/doi/ by National Institute Of Standard, Wiley Online Library on [06/03/2024]. See the Terms and Conditions (https://onlinelibrary.wiley.com/terms-and-conditions) on Wiley Online Library for rules of use; OA articles are governed by the applicable Creative Commons License
299

71

Feeding Challenge in a Child with Spinal Muscular Atrophy


Nicole Fragale
Nemours/Alfred I. duPont Hospital for Children, Wilmington, DE, USA

A 16-month-old child with spinal muscular atrophy (SMA) Current Status


type 1, diagnosed as an infant, is admitted to the pediatric
intensive care unit (PICU) with severe malnutrition and She is hypoglycemic on admission and requires
aspiration pneumonia leading to acute hypercarbic respira- Vapotherm® (high-flow nasal cannula therapy), progres-
tory failure. sing to BiPAP due to respiratory distress. She is treated with
Unasyn (ampicillin sulbactam) due to a left-sided pneumo-
nia, seen as a white-out on X-ray. Her admission weight
(6.4 kg, z-score –3.78) is noted as a 17% reduction in body
History of Present Illness weight since last seen in the hospital 6 months prior.
Her admission length (71 cm) is below the 1st percentile
The child was in her usual state of health until 2 days ago (z-score –5). Her weight/length 6 months prior was following
when she began to have increased work of breathing and the 75th percentile (z-score 0.65) and on admission to the hos-
post-tussive emesis. Parents report that she pulled out pital is now less than the 1st percentile (z-score –3). She meets
her nasogastric feeding tube 2 days prior to admission. criteria for severe malnutrition (see Table 71.1 for indicators
The tube was not replaced by the family, and instead the of the diagnosis of malnutrition).
child was fed orally until she was brought into the hospital. The results of the basic metabolic panel are shown in
Table 71.2.

Past Medical History


Questions
This child was diagnosed with SMA type 1 during infancy.
At 10 months of age, a bedside feeding evaluation by a
Answer the following questions using the details provided.
speech language pathologist indicated that the infant had
discoordination with suck and swallow. A modified barium 1 Based on the background information and the labora-
swallow study indicated that the patient had evidence of tory results on admission, what is your assessment of
aspiration, and nil by mouth status was recommended. this child’s fluid and nutritional status?
As a result, a nasogastric tube was placed and the patient 2 How should enteral nutrition be resumed in this patient
was sent home with tube feedings to meet 100% of her and what parameters will be monitored?
nutritional requirements. This child also has a night-time 3 What are the calorie and protein goals for this patient?
bilevel positive airway pressure (BiPAP) requirement. 4 How will you determine if the nutrition being provided
Between the two admissions, she was lost to follow-up is adequate and what feeding protocol should be recom-
and there is concern for medical neglect. mended for discharge and home care?

Cases in Pediatric Acute Care: Strengthening Clinical Decision Making, First Edition. Edited by Andrea M. Kline-Tilford and Catherine M. Haut.
© 2020 John Wiley & Sons Ltd. Published 2020 by John Wiley & Sons Ltd.
Downloaded from https://onlinelibrary.wiley.com/doi/ by National Institute Of Standard, Wiley Online Library on [06/03/2024]. See the Terms and Conditions (https://onlinelibrary.wiley.com/terms-and-conditions) on Wiley Online Library for rules of use; OA articles are governed by the applicable Creative Commons License
300 Cases in Pediatric Acute Care

Table 71.1 Primary malnutrition indicators when two or more data points available.

Mild malnutrition Moderate malnutrition Severe malnutrition

Weight gain velocity Less than 75% of the norm for Less than 50% of the norm for Less than 25% of the norm for
(<2 years of age) expected weight gain expected weight gain expected weight gain
Weight loss (2–20 years of 5% usual body weight 7.5% usual body weight 10% usual body weight
age)
Deceleration in weight for Decline of 1 z-score Decline of 2 z-score Decline of 3 z-score
length/height z-score
Inadequate nutrient intake 51–75% estimated energy/ 26–50% estimated energy/ ≤25% estimated energy/protein
protein need protein need need
Source: Becker et al. (2014). Reproduced with permission of Elsevier.

Table 71.2 Basic metabolic panel. neuromotor control can be affected, including feeding and
breathing. Children with SMA can have feeding problems,
Sodium 145 mEq/L dysphagia or gastrointestinal dysfunction, and because of
Potassium 3.9 mEq/L muscle laxity and overall neuromuscular weakness, it is
Chloride 115 mEq/L not uncommon for these children to gain unnecessary
Carbon dioxide 14 mEq/L weight. Consequently, they need to be monitored very care-
Blood urea nitrogen 18 mg/dL
fully and nutrition needs to include vitamin D, calcium and
phosphorus for bone health.
Creatinine <0.1 mg/dL
This child has a history of poor feeding abilities and aspi-
Glucose 53 mg/dL
ration on modified barium swallow. She presents to the
Calcium 8.5 mg/dL hospital after having been without her nasogastric feeding
Magnesium 1.7 mEq/L tube for 2 days (by parent report). Her elevated sodium,
Phosphorus 3.0 mg/dL chloride, and blood urea nitrogen indicate she is dehy-
drated, which is likely due to inability to consume adequate
fluids by mouth. The significant decrease in weight on
admission and the stunted length indicate that this child
Rationale and Evidence-based Practice is chronically malnourished and she has likely not received
Explanation adequate nutrition in at least 3 months.
Because this 16-month-old child with type 1 SMA is sus-
Based on the background information and the pected to have been without adequate nutrition for several
laboratory results on admission, what is your months, she is at risk for refeeding syndrome. Refeeding
assessment of this child’s fluid and nutritional
syndrome presents as metabolic derangements that occur
status?
as a result of the rapid shift from fat and protein catabolism
This child has a history of SMA type 1 which is considered a (which occur with starvation) to carbohydrate metabolism,
severe progressive neuromuscular illness. SMA is due to a resulting in an increase in circulating insulin levels. When
defect in the survival motor neuron 1 (SMN1) gene and is insulin levels increase in the body, there is rapid uptake of
present in types 1 to 4. SMA is characterized by degenera- glucose, potassium, phosphorus and magnesium into the
tion of the motor neurons in the spinal cord, resulting in cells, resulting in decreased serum concentrations. Fluid
progressive muscle atrophy, weakness, and paralysis. Typ- retention is common with these shifts and can result in
ically, infants with SMA type 1 present with hypotonia acute heart failure as a result of increased cardiac demand
early in life along with weakness of the legs as compared with accompanying acute respiratory distress.
to the arms, and weakness of the intercostal muscles result-
ing in ineffective or paradoxical breathing patterns. SMA
How should enteral nutrition be resumed in this
type 1 is typically diagnosed within the first 6 months of life
patient and what parameters will be monitored?
and is the most severe form, often resulting in death at a
very early age. Diagnosis is accomplished by clinical find- In order to avoid refeeding syndrome, electrolytes should
ings, genetic testing, electromyography, and creatine be normalized prior to starting enteral feeds, which should
kinase levels, which may be normal, mildly elevated, or sig- then be initiated at 50–75% of intended goal and advanced
nificantly elevated. Any physiologic function that requires by only 10–20% daily. Electrolytes should continue to be
Downloaded from https://onlinelibrary.wiley.com/doi/ by National Institute Of Standard, Wiley Online Library on [06/03/2024]. See the Terms and Conditions (https://onlinelibrary.wiley.com/terms-and-conditions) on Wiley Online Library for rules of use; OA articles are governed by the applicable Creative Commons License
Chapter 71 Feeding Challenge in a Child with Spinal Muscular Atrophy 301

monitored two to four times daily, and repleted as needed. How will you determine if the nutrition being
Phosphorus (250 mg daily) and thiamine supplementation provided is adequate and what feeding protocol
(100 mg three times daily) is indicated when initiating feeds should be recommended for discharge and
in order to aid carbohydrate metabolism. Strict monitoring home care?
of fluid status is required and fluid restriction may be nec-
It is most important to include the interprofessional
essary if edema is noted. Refeeding syndrome can occur up
team to assist in making sure that this child comes back
to 2 weeks after initiation of enteral feeds.
for follow-up. Engaging the social worker, case man-
ager, and neurology service can assist in beginning this
What are the calorie and protein goals for this
process. The registered dietitian is also extremely
patient?
important in evaluating nutritional status through mon-
On admission, the patient has a post-pyloric nasoduodenal itoring weekly weight trends and mid-upper arm cir-
tube placed. The team would like feeding recommenda- cumferences. Normal weight gain for a 16-month-old
tions. Prior to determining calorie goals for the patient, child ranges from 5 to 9 g/day. When catch-up growth
an assessment of the diet followed prior to admission is is indicated, daily weight gain goals should be two to
indicated. In this situation, the parent reports a tube feed- three times this amount. Increases in the percentile
ing regimen that mimics what the patient received in the and z-score of patient length should also be seen
hospital 6 months prior, in addition to taking bottles by over time.
mouth. If the patient had been receiving the reported reg- Ideally, this patient will have a surgically placed gastric
imen, she would have been receiving 135 cal/kg per day. feeding tube. If this is not possible, the nasoduodenal tube
A typical 16-month-old child would require 100 cal/kg should be moved to become a nasogastric tube so that the
daily. A malnourished child requires more for catch-up patient can be given bolus feeds. This will mimic normal
growth (100–120 cal/kg daily). In order to avoid refeeding physiologic eating. Gastroenterology service can assist in
syndrome, the following recommendation is given for start- supporting the patient and insuring that the gastric tube
ing continuous feedings. is placed in a good position, along with education for the


family on how to manage, maintain and use for feeding.
On initiating enteral feeds, use Nutren Junior started at
Frequent follow-up with an outpatient dietitian is recom-
15 mL/hour (50% goal). Advance by 5 mL/hour every
mended so that adjustments can be made as the patient
day until goal of 30 mL/hour is reached (will take 3 days).


continues to grow.
Using weight of 6.4 kg, regimen at goal will provide 720 mL
(112 mL/kg), 720 cal (112 cal/kg), 22 g protein (3.3 g/kg).

Reference
Becker, P.N., Carney, L.N., Corkins, M.R., et al. (2014). and documentation of pediatric malnutrition
Concensus Statement of the Academy of Nutrition and (undernutrition). Journal of the Academy of Nutrition and
Dietitics/American Society for Parenteral and Enteral Dietetics 114(12): 1988–2000.
Nutrition: indicators recommended for the identification

Further Reading
Beer, S., Bunting, D., Canada, N., et al. (2016). Pediatric
Nutrition Reference Guide, 11th edn. Houston, TX: Texas
Children’s Hospital.
Downloaded from https://onlinelibrary.wiley.com/doi/ by National Institute Of Standard, Wiley Online Library on [06/03/2024]. See the Terms and Conditions (https://onlinelibrary.wiley.com/terms-and-conditions) on Wiley Online Library for rules of use; OA articles are governed by the applicable Creative Commons License
303

72

A 5-Year-Old with Osteomyelitis, Electrolyte Disturbance and Headaches


Kimberly L. DiMaria
Children’s Hospital Colorado, Aurora, CO, USA

A 5-year-old previously healthy male is admitted with oste- Current Status


omyelitis. He has been hospitalized for 3 days and his
course is now complicated by electrolyte abnormalities He is a well-developed, well-nourished, pale and tired-
and associated mental status changes. appearing school-age child lying in bed watching a movie.
His vital signs are heart rate 136 bpm, respiratory rate
30 breaths per minute, blood pressure 86/42 mmHg, and
History of Present Illness oxygen saturation 94% on room air. He is 18 kg on admis-
sion; there is no subsequent weight documented.
A 5-year-old boy with osteomyelitis of the right ankle is His examination is notable for the following. He has dry
admitted for parenteral antibiotics and peripherally mucous membranes, clear breath sounds with easy work of
inserted central catheter line placement. He developed breathing, sinus tachycardia, 2+ pulses throughout, and
severe diarrhea 2 days after initiation of antimicrobial ther- capillary refill time of 4 seconds with warm/dry extremities.
apy. Today, he is lethargic and complaining of a headache. He has a non-tender abdomen with hyperactive bowel
He has had decreased urine output since yesterday. The sounds and no organomegaly. He is awake, but lethargic
rapid response team is activated. and irritable. It takes several attempts for him to look at
the healthcare provider, respond to questions, and follow
simple commands. He tells the healthcare provider that
Past Medical History his “head hurts.” His cranial nerves II–XII are grossly
intact.
He was born full term after an uncomplicated pregnancy At the time of his rapid response team call, his mother
and delivery. He had one episode of croup at 6 months of shares with the team that the child had diarrhea almost
age that required hospitalization and steroids. He has sea- every hour last night. She is not sure if he had urine output
sonal allergies for which he takes a daily cetirizine. No with any of his frequent trips to the bathroom.
known drug allergies. He is transferred to the pediatric intensive care unit
(PICU) for further evaluation and management. He is
placed on the cardiorespiratory monitor and a basic meta-
Past Surgical History bolic profile is obtained. The result of his laboratory work is
shown in Table 72.1.
No previous surgeries.

Family History Questions

Answer the following questions using the details provided.


Non-contributory. He lives with both parents and three
older siblings, two dogs and a parakeet. Is in kindergarten. 1 Based on the information provided, what is the most
No recent travel. likely diagnosis for this patient?

Cases in Pediatric Acute Care: Strengthening Clinical Decision Making, First Edition. Edited by Andrea M. Kline-Tilford and Catherine M. Haut.
© 2020 John Wiley & Sons Ltd. Published 2020 by John Wiley & Sons Ltd.
Downloaded from https://onlinelibrary.wiley.com/doi/ by National Institute Of Standard, Wiley Online Library on [06/03/2024]. See the Terms and Conditions (https://onlinelibrary.wiley.com/terms-and-conditions) on Wiley Online Library for rules of use; OA articles are governed by the applicable Creative Commons License
304 Cases in Pediatric Acute Care

Table 72.1 Basic metabolic profile. values during rehydration and sodium correction. The
morbidity and mortality associated with hypernatremia is
Sodium 160 mEq/L
typically not from the disorder itself, but from inappropri-
Potassium 4.5 mEq/L ate management (e.g. excessively rapid correction). He will
Chloride 125 mEq/L require slow rehydration with frequent monitoring of
Carbon dioxide 16 mEq/L serum sodium levels because a rapid decrease in serum
Blood urea nitrogen 26 mg/dL sodium can result in cerebral edema, seizures, or osmotic
Creatinine 1.6 mg/dL demyelination syndrome.
Glucose 102 mg/dL
What are the initial steps of management?
Throughout the management of this patient, it is essential
2 Is transfer to the PICU indicated? to ensure slow and careful rehydration. Rapid decrease in
3 What are the initial steps of management? serum sodium can result in severe neurologic complica-
4 How will you correct the electrolyte derangement? tions. The initial step in management is to begin careful
5 What complications will you monitor for? fluid resuscitation to restore the intravascular volume. Iso-
tonic fluid must be used for fluid resuscitation, using small
aliquots (10–20 mL/kg) unless the patient has evidence of
Rationale and Evidence-based Practice hypovolemic shock. Normal saline has a sodium concentra-
Explanation tion of 154 mEq/L and is thus technically a hypotonic fluid
in the presence of a serum sodium of 160 mEq/L in this
Based on the information provided, what is the hypernatremic child. Therefore, if the child remains hemo-
most likely diagnosis for this patient? dynamically stable, only a single, small, fluid bolus of nor-
mal saline should be administered prior to rechecking the
This patient has hypernatremia. Supported by his diarrheal serum sodium.
losses, physical examination and vital signs, he is also dehy-
drated. He has hypernatremic dehydration. Hypernatremia
How will you correct the electrolyte
is defined as serum sodium greater than 150 mEq/L and
derangement?
hypernatremic dehydration signifies a greater water loss
in relation to total body sodium. There are various ways The goal is to decrease his serum sodium by 10–15 mEq/L
that a patient can develop hypernatremic dehydration. per day, which is roughly 0.5 mEq/L every hour. To achieve
The patient in this case study developed hypernatremic this slow decrease in serum sodium, begin by calculating
dehydration from fluid losses. All gastrointestinal losses the free water deficit using the following formula:
(emesis or diarrhea) are hypotonic because they have a
Free water FW deficit = 0 6 ×
sodium concentration less than the plasma sodium concen-
tration. An excessive loss of hypotonic fluid (emesis or diar- weight kg × current serum sodium 140 – 1
rhea) resulting in an increase in output can lead to This child’s free water deficit, if targeting a serum sodium
hypernatremia. This type of electrolyte disturbance typi- of 145 mEq/L (rather than 140 mEq/L in the equation
cally occurs gradually, which explains why the patient is above) is 1.1 L.
less symptomatic than if the hypernatremic state had
occurred abruptly. FW deficit = 0 6 × 18 kg × 160 145 – 1
Severe hypernatremic dehydration can cause a spectrum = 1 117 L rounded down to 1 1 L
of neurologic symptoms, including altered mental status,
This amount of fluid (1.1 L or 1100 mL) will be adminis-
cerebral hemorrhage, seizures, and encephalopathy. The
tered over 48 hours at a rate of 23 mL/hour. It is important
mechanism of intracranial hemorrhage is explained by
to note that the rehydration fluid (the FW deficit calculated
tearing of the cerebral blood vessels (bridging veins) as
rate) will be administered in combination with mainte-
the dehydrated brain tissue shrinks.
nance fluids. For the patient, this would be a total fluid rate
of 79 mL/hour (23 mL/hour of rehydration fluid plus
Is transfer to the PICU indicated?
56 mL/hour of maintenance fluids). Once the rate of intra-
Yes, this patient is experiencing hypernatremic dehydra- venous fluid therapy has been determined, the next step is
tion with associated mental status changes. He will require to select the type of fluid to be administered. Dextrose 5%
close monitoring of his neurologic status and laboratory with 0.2% normal saline is the typical intravenous fluid
Downloaded from https://onlinelibrary.wiley.com/doi/ by National Institute Of Standard, Wiley Online Library on [06/03/2024]. See the Terms and Conditions (https://onlinelibrary.wiley.com/terms-and-conditions) on Wiley Online Library for rules of use; OA articles are governed by the applicable Creative Commons License
Chapter 72 A 5-Year-Old with Osteomyelitis, Electrolyte Disturbance and Headaches 305

of choice for mild to moderate dehydration, but the sodium What complications will you monitor for?
concentration may need to be increased depending on the
Seizures, rapid decrease in sodium, signs of cerebral edema
serum sodium value and rate of decrease in serum sodium.
(pupillary changes, mental status changes and late findings
The serum sodium should be evaluated 2 hours after the
of Cushing’s triad), and cerebral sinus venous thrombosis
initiation of intravenous therapy, and then every 2–4 hours
should be be monitored for. The speed of sodium reduction
thereafter. If the serum sodium is decreasing by more than
is the key determinant as to whether a patient develops
0.5 mEq/L per hour, increase the sodium concentration in
complications from electrolyte shifts.
the intravenous fluids.

Further Reading
Adtogue, H.J. and Madias, N.E. (2000). Hypernatremia. New Powers, K.S. (2015). Dehydration: isonatremic, hyponatremic,
England Journal of Medicine 342(20): and hypernatremic recognition and management. Pediatrics
1493–1499. in Review 36(7): 274–285. doi: 10.1542/pir.36-7-274.
Liamis, G., Filippatos, T.D., and Elisaf, M.S. (2016). Santilanes, G. and Rose, E. (2018). Evaluation and
Evaluation and treatment of hypernatremia: a management of dehydration in children. Emergency
practical guide for physicians. Postgraduate Medicine Medicine Clinics of North America 36(2): 259–273.
128(3): 299–306. doi: 10.1080/00325481.2016. Schwaderer, A.S. and Schwartz, G.J. (2005). Treating
1147322. hypernatremic dehydration. Pediatrics in Review 26(4): 148–150.
Downloaded from https://onlinelibrary.wiley.com/doi/ by National Institute Of Standard, Wiley Online Library on [06/03/2024]. See the Terms and Conditions (https://onlinelibrary.wiley.com/terms-and-conditions) on Wiley Online Library for rules of use; OA articles are governed by the applicable Creative Commons License
307

73

Child with Scald Burn Injury


Amarilis Martin and Bradley Tilford
Children’s Hospital of Michigan, Detroit, MI, USA

Patient is a 4-year-old non-verbal male with Down syn- of his cardiac defects, he has had no other hospitalizations.
drome, developmental delay, and a history of repaired com- He takes oral ranitidine for acid reflux and nebulized albu-
plete atrioventricular canal (CAVC) defect and patent terol as needed for intermittent asthma. He is followed
ductus arteriosus (PDA) at 5 months of age. He is admitted outpatient for his developmental and speech delays. Immu-
to the burn unit with an 8-day history of scald burns to nizations are up to date for age, including the annual influ-
the feet. enza vaccine. He has a weight at the 75th percentile for
gender and age based on the Down syndrome growth curve
and a current body mass index (BMI) slightly greater than
History of Present Illness the 95th percentile for gender and age.

This 4-year-old child suffered 4.5% total body surface area


scald burns to the feet 8 days ago, leaving a circumferential Past Surgical History
mark at the ankles. According to the mother, she placed the
child in the bath tub, turned on the water, and momentarily Repair of CAVC defect and PDA ligation at 5 months of age.
left the room. She returned when she heard him scream. He
quickly developed blisters to both feet. Mother did not seek
medical attention and managed the burns at home with Family History
topical lidocaine cream and dry gauze dressings. Over the
week following the burns, the wounds worsened in appear- Mother has type 2 diabetes mellitus. Father’s medical his-
ance. On the night of admission, he developed poor oral tory is unknown.
intake, nausea, and bilious emesis for which she brought
him to an emergency department. He was then transferred
to a tertiary care center as a direct admission to the burn Social History
service. There is no history of fever, voiding or stooling
concerns, or change in behavior. Mother is unsure if he Patient lives with his mother and his mother’s boyfriend.
is in pain because he is developmentally delayed and His father is not involved in his care. He has no siblings.
non-verbal.

Current Status
Past Medical History
In the burn unit, he is found to have scald burns to both feet
He was born at term and went home with his mother after a with a circumferential mark at the ankles, a right buttock
2-day stay in the newborn nursery for jaundice and hypo- wound with healthy granulation tissue and sloughing of
glycemia. His cardiac defects were discovered after birth skin along the gluteal fold, and a right knee wound with
due to feeding difficulty and failure to thrive. After repair associated scale along the edges. His skin is dry. His vital

Cases in Pediatric Acute Care: Strengthening Clinical Decision Making, First Edition. Edited by Andrea M. Kline-Tilford and Catherine M. Haut.
© 2020 John Wiley & Sons Ltd. Published 2020 by John Wiley & Sons Ltd.
Downloaded from https://onlinelibrary.wiley.com/doi/ by National Institute Of Standard, Wiley Online Library on [06/03/2024]. See the Terms and Conditions (https://onlinelibrary.wiley.com/terms-and-conditions) on Wiley Online Library for rules of use; OA articles are governed by the applicable Creative Commons License
308 Cases in Pediatric Acute Care

Table 73.1 Complete metabolic profile. Table 73.3 Venous blood gas analysis.

Sodium 147 mEq/L pH 7.01


Potassium 5.0 mEq/L PCO2 60 mmHg
Chloride 107 mEq/L PaO2 15 mmHg
Carbon dioxide 20 mEq/L Bicarbonate 14 mEq/L
Blood urea nitrogen 56 mg/dL Base excess –17
Creatinine 2.41 mg/dL SvO2 8%
Alanine aminotransferase 794 U/L Lactate 12.7 mmol/L
Aspartate aminotransferase 554 U/L Sodium 145 mmol/L
Lipase 137 U/L Chloride 115 mmol/L
Potassium 7.9 mmol/L
Ionized calcium 1.39 mmol/L

Table 73.2 Complete blood count and coagulation studies.

White blood cell count 14 900/mm3


1 Based on the history and presented data, what are the
Hemoglobin 12.8 g/dL differential diagnoses and the most likely diagnosis
Hematocrit 41.5% for this child?
Platelet count 136 000/mm3 2 What, if anything, is concerning about his complete
Neutrophils 75% metabolic panel?
Eosinophils 0% 3 Would you continue the current maintenance IVF at
Monocytes 8% this time?
Lymphocytes 12% Continuation of case: Patient has bilious emesis following
Bands 4% completion of his burn dressings and IV ondansetron is
Prothrombin time (PT) 17.0 s given. An hour later, during morning rounds by the burn
Partial thromboplastin time (PTT) 25.6 s service, the patient is found unresponsive and pulseless
and a code blue is called. His electrocardiogram (EKG)
International normalized ratio 1.67
leads and pulse oximeter had become disconnected when
he vomited and were not replaced. Cardiopulmonary resus-
citation (CPR) is started. He is intubated and bilious secre-
signs on admission are temperature 36.2 C (97.2 F), heart tions are noted in the endotracheal tube. Intraosseous
rate 153 bpm, respiratory rate 32 breaths per minute, blood access is obtained. A venous blood gas analysis obtained
pressure 80/43 mmHg, and oxygen saturation 93% on room during the resuscitation demonstrates the results shown
air. A basic metabolic panel, complete blood count with dif- in Table 73.3.
ferential, liver function tests, lipase, coagulation panel, and
4 In addition to following the Pediatric Advanced Life
type and screen are obtained. He is kept nil by mouth and is
Support (PALS) guidelines for cardiac arrest, what is
given maintenance intravenous fluids (IVF) consisting of
the best way to manage the patient’s condition?
5% dextrose in 0.9% normal saline containing 20 mEq/L
5 Would you obtain any other diagnostic studies?
of potassium chloride (KCl). Over the next 2 hours,
6 How would you determine if the therapy was effective?
his wounds are cleaned and dressed with silver sulfadiazine
(Silvadene) and sterile gauze. He is given intravenous (IV) Continuation of case: Code doses of IV epinephrine, IV
diphenhydramine and a small dose of IV morphine for pain calcium chloride 20 mg/kg per dose, and IV sodium bicar-
associated with cleaning and dressing the burns. His bonate 1 mEq/kg per dose are administered during the
laboratory results are presented in Tables 73.1 and 73.2. resuscitation. A blood culture is obtained and he is started
on broad-spectrum antibiotics. The echocardiogram does
not show a pericardial effusion and he continues to have
bilateral breath sounds without hypoxemia. He is started
Questions on an epinephrine infusion. While CPR is ongoing, the
patient is emergently transferred to the pediatric intensive
Answer the following questions using the details provided. care unit (PICU), where he undergoes extracorporeal
Downloaded from https://onlinelibrary.wiley.com/doi/ by National Institute Of Standard, Wiley Online Library on [06/03/2024]. See the Terms and Conditions (https://onlinelibrary.wiley.com/terms-and-conditions) on Wiley Online Library for rules of use; OA articles are governed by the applicable Creative Commons License
Chapter 73 Child with Scald Burn Injury 309

Figure 73.1 Sine wave pattern seen on EKG tracing of patient with severe hyperkalemia.

cardiopulmonary resuscitation (ECPR) with veno-arterial


extracorporeal membrane oxygenation (VA-ECMO). He
• Non-accidental trauma: scald burns to the feet with clear
demarcation, right buttock wound with skin sloughing,
is given additional IV doses of calcium chloride, sodium possible head trauma (nausea, emesis), possible abdom-
bicarbonate, and insulin with 25% dextrose for the hyperka- inal trauma (nausea, emesis, liver and pancreatic injury).
lemia. Despite these treatments, his potassium level
increases to 10.8 mEq/L, and his cardiac rhythm changes
• Ingestion of unknown drug causing anion-gap metabolic
acidosis and subsequent multiorgan dysfunction. This is
from pulseless electrical activity to wide complex tachycar- less likely given the history but should always be consid-
dia (sinusoidal waveform pattern) for which he is defibril- ered, especially in a developmentally delayed non-verbal
lated four times (Figure 73.1). He is also given amiodarone child.
and lidocaine for the ventricular tachycardia.
Sepsis is defined as the presence of the systemic inflam-
7 What other therapies would be recommended at matory response syndrome (SIRS) with a known or sus-
this time? pected infection. SIRS is the presence of at least two of
the following: altered temperature, heart rate, respiratory
rate, or leukocyte count; however, either an altered temper-
Rationale and Evidence-based Practice ature or an altered leukocyte count should be present for
Explanation the patient to have SIRS. In this case, the child has tachy-
cardia, tachypnea, and granulocytosis (SIRS); there is con-
Based on the history and presented data, what are cern for an infection given his history of inappropriately
the differential diagnoses and the most likely treated burns (sepsis); and he has signs of MODS.
diagnosis for this child?
The child has multiple vital sign and laboratory
derangements: tachycardia, tachypnea, leukocytosis with What, if anything, is concerning about his basic
granulocytosis (75% neutrophils, 4% bands), mild thrombo- metabolic panel?
cytopenia, acute kidney injury (AKI) with hyperkalemia
He has hyperkalemia due to sepsis, MODS, and AKI. He is
and uremia, pancreatic injury, and liver dysfunction with
at high risk for worsening hyperkalemia if these conditions
coagulopathy concerning for disseminated intravascular
are not managed promptly and appropriately. Patients with
coagulation. Looking closer at the electrolytes, he has an
sepsis, MODS, and burns have a high risk for developing
anion-gap metabolic acidosis.
hyperkalemia due to potassium spillage from the damaged
Because of the multiple organ systems involved, the dif-
cells into the extracellular space. AKI also increases the risk
ferential diagnoses for this child include the following.
for hyperkalemia due to a decreased ability to excrete potas-

• Severe sepsis with multiorgan dysfunction syndrome


(MODS). Possible causes of sepsis include:
sium through the urine, especially if the patient is oliguric
or anuric.
• Skin: 8-day history of inappropriately treated scald Hyperkalemia is defined as an elevation of serum potas-
burns to the feet, untreated wounds to right buttocks sium levels. The normal range of potassium varies with age:

••
and right knee.
• Gut: gastroenteritis (nausea, emesis), bowel obstruction preterm 3.0–6.0 mEq/L
newborn 3.7–5.9 mEq/L

••
with bacterial translocation (nausea, bilious emesis).
• Pancreas: pancreatitis (nausea, emesis, elevated infant 4.1–5.3 mEq/L
child 3.4-4.7 mEq/L


lipase).
• Other unknown viral or bacterial etiology. adult 3.5-5.1 mEq/L.
Downloaded from https://onlinelibrary.wiley.com/doi/ by National Institute Of Standard, Wiley Online Library on [06/03/2024]. See the Terms and Conditions (https://onlinelibrary.wiley.com/terms-and-conditions) on Wiley Online Library for rules of use; OA articles are governed by the applicable Creative Commons License
310 Cases in Pediatric Acute Care

Hyperkalemia is frequently seen in patients with renal In addition to following the PALS guidelines for manage-
disease, excess exogenous potassium administration, criti- ment of septic shock and cardiopulmonary arrest, the child
cal illness (e.g. sepsis, septic shock, MODS), or in patients should be aggressively treated for hyperkalemia, a part of
taking a medication that raises the serum potassium level the H’s of the reversible causes of cardiac arrest described
(e.g. angiotensin-converting enzyme inhibitors, spironolac- in the PALS guidelines. All patients with hyperkalemia
tone). Hyperkalemia alters the cardiomyocyte action should have any potassium supplementation discontinued
potential, leading to cardiac arrhythmias. Even though and should have an EKG to evaluate for cardiac arrhyth-
electrocardiogram (EKG) changes do not correlate with mias. Emergent management of severe hyperkalemia
the severity of hyperkalemia, there is usually a progression requires stabilization of the cardiomyocyte membrane
of EKG findings as the potassium level rises. Peaked potential with IV calcium (calcium chloride is preferred
T waves and PR prolongation are initially seen, followed over calcium gluconate, although either can be used).
by widened QRS and P-wave loss. Eventually, the widened Potassium is shifted intracellularly with IV sodium bicarbo-
QRS complexes fuse with the wide and tall ST–T segments, nate, nebulized β-adrenergic agonists such as albuterol, and
forming a sine-wave pattern (Figure 73.2) that can lead to IV insulin (bolus is recommended over other routes of
ventricular fibrillation if not promptly treated. Even though administration due to faster onset of action). Insulin should
severe hyperkalemia can be life-threatening, the type of always be administered with dextrose to avoid causing
EKG alteration is more predictive of life-threatening hyper- hypoglycemia. Other therapies to remove potassium from
kalemia than the level itself. In contrast to peaked T waves, the body include administration of diuretics that increase
QRS widening heralds a life-threatening hyperkalemia that urinary potassium excretion (e.g. furosemide); cation
should be emergently treated. exchange resins such as enteral sodium polystyrene sulfo-
nate (Kayexalate), which bind potassium in the large intes-
tine in exchange for sodium; and hemodialysis, which
Would you continue the current maintenance IVF directly removes potassium from the blood. The effect of
at this time? diuretics and cation exchange resins on serum potassium
With AKI and sepsis, the patient’s hyperkalemia could takes time, and these medications are not effective in
worsen, especially if he is given potassium-containing IVF. patients with oliguria/anuria or bowel pathology. They
His urine output should be monitored closely and the IVF should not be used independently for emergent manage-
rate should be decreased or stopped if he becomes oliguric ment of hyperkalemia. Hemodialysis is an effective way
or anuric to prevent fluid overload. If the decision is made to quickly lower the serum potassium level and is used in
to continue IVF administration, KCl should be removed. patients with acute, persistent, or life-threatening
hyperkalemia.
If a patient remains in cardiac arrest despite standard
In addition to following the PALS guidelines for resuscitation measures, ECPR can be instituted in some
cardiac arrest, what is the best way to manage the
centers. ECPR is early placement of a child with cardiac
patient’s condition?
arrest on extracorporeal life support such as extracorporeal
The patient had multiple potential causes of cardiac arrest: membrane oxygenation (ECMO). ECMO provides cardiac
multiorgan failure associated with septic shock, hyperkale- and/or respiratory support in patients refractory to conven-
mia-induced arrhythmia with subsequent cardiogenic shock, tional therapy, but with a potentially reversible cause of ill-
and aspiration of emesis. His hyperkalemia may be exacer- ness, such as hyperkalemia-induced arrhythmia or septic
bated by the administration of potassium-containing IVF in shock. Despite its life-saving potential, there is considerable
the setting of AKI and ongoing cellular injury from MODS. morbidity and mortality associated with ECMO.

Figure 73.2 Peaked T waves, widened QRS complex, and loss of P waves seen on EKG.
Downloaded from https://onlinelibrary.wiley.com/doi/ by National Institute Of Standard, Wiley Online Library on [06/03/2024]. See the Terms and Conditions (https://onlinelibrary.wiley.com/terms-and-conditions) on Wiley Online Library for rules of use; OA articles are governed by the applicable Creative Commons License
Chapter 73 Child with Scald Burn Injury 311

Would you obtain any other diagnostic studies? arrhythmias have subsided or have been controlled.
Frequent physical examinations and assessments of the
A bacterial blood culture should be obtained as soon as
vital signs are imperative to determine the patient’s hemo-
possible. Broad-spectrum antibiotics should be started
dynamic status. Evaluating the child’s temperature, respi-
immediately in patients with suspected sepsis, even if
ratory rate, oxygen saturation, pulse, heart rate, blood
blood culture is delayed. If clinically indicated, other spe-
pressure, capillary refill time, urine output, and fluid status
cific microorganisms (bacteria, viral, fungal) can be iden-
will give evidence of the child’s response to therapy. Blood
tified via culture, polymerase chain reaction (PCR),
gas analysis may help assess adequacy of therapy and
antigen, or antibody detection from sources such as respi-
whether the patient’s shock is improving.
ratory secretions, blood, urine, stool, vesicles, or cerebro-
spinal fluid if feasible. Antivirals or antifungals should be
started as soon as possible if there is high suspicion for What other therapies would be recommended at
such microorganisms, even if obtaining diagnostic studies this time?
is delayed.
The child should be started on hemodialysis as soon as pos-
In addition to assessing and managing infection (sepsis,
sible. Dialysis decreases serum potassium levels relatively
septic shock), other causes of cardiac arrest should be eval-
quickly and is indicated in acute, persistent, or life-
uated and treated if present. Reversible causes of cardiac
threatening hyperkalemia. Hemofiltration and hemodialy-
arrest include the H’s and T’s described in the PALS guide-
sis can be performed through the ECMO circuit. Otherwise,
lines. Hypovolemia can be reversed with IVF boluses; vas-
a hemodialysis catheter is required.
oactive medications can help with contractility and
Once the child stabilizes, further imaging may be
vasoconstriction to further improve cardiac output.
obtained to assess for head trauma, abdominal trauma,
Hypoxia can be treated with supplemental oxygen and
and fractures. If there is suspicion of non-accidental trauma
assessed via pulse oximetry or blood gas testing. A blood
or neglect, social work should be consulted and a referral
gas can also help assess the severity of respiratory and met-
made to child protective services. Given the delay in seek-
abolic acidosis. A point-of-care blood sugar check can
ing medical care for severe burns, a child protective services
determine if hypoglycemia is present, which should be
referral is appropriate in the case presented.
promptly treated with IV dextrose. The patient should
Application of topical antimicrobials such as silver
be warmed to normal body temperature if a temperature
sulfadiazine (Silvadene) and daily dressing changes to
check demonstrates hypothermia. Auscultating for breath
his wounds are also important to prevent infection and
sounds bilaterally and, if in doubt, performing needle
other wound complications. However, they are not a sub-
decompression can be used to evaluate for and treat a ten-
stitute for systemic antibiotics and these should be contin-
sion pneumothorax. A targeted bedside echocardiogram
ued and tailored to the organism isolated from culture
can be done to assess for cardiac tamponade. Blind peri-
results.
cardiocentesis is rarely performed, but may be considered
with high suspicion of cardiac tamponade. Blood and
urine testing can be sent to assess for toxins. Pulmonary
embolism or myocardial ischemia, although rare in this Case Resolution
child’s age group, can be suggested via EKG and
confirmed via chest computed tomography or cardiac He is started on an insulin and dextrose infusion and con-
catheterization. Arrhythmias can be assessed with EKG tinuous renal replacement therapy via the ECMO circuit.
or telemetry and treated with antiarrhythmics. Amiodar- As his potassium level decreases, his EKG findings change
one or lidocaine is recommended for unstable patients from sinusoidal wave appearance to atrioventricular block,
with ventricular fibrillation or pulseless ventricular during which time he has return of spontaneous circula-
tachycardia. tion. His EKG eventually changes to sinus rhythm as the
potassium level completely normalizes. The patient is con-
tinued on broad-spectrum antibiotics until his blood cul-
How would you determine if the therapy was
ture isolates Klebsiella pneumoniae, for which he finishes
effective?
a full course of IV cefepime. He is successfully decannu-
Frequent repeated evaluations of both serum potassium lated from ECMO 4 days following the cardiac arrest, but
(via serial blood gases or serum electrolyte panel) and continues to require mechanical ventilation and renal
telemetry, defibrillator monitor, or EKG are needed to replacement therapy in the pediatric intensive care unit
trend the potassium level and to assess whether the for several weeks.
Downloaded from https://onlinelibrary.wiley.com/doi/ by National Institute Of Standard, Wiley Online Library on [06/03/2024]. See the Terms and Conditions (https://onlinelibrary.wiley.com/terms-and-conditions) on Wiley Online Library for rules of use; OA articles are governed by the applicable Creative Commons License
312 Cases in Pediatric Acute Care

Further Reading
Arnholt, A.M., Duval-Arnould, J.M., McNamara, L.M., et al. cardiopulmonary resuscitation and emergency
(2015). Comparatively evaluating medication preparation cardiovascular care. Circulation 122(Suppl 3): S876–S908.
sequences for treatment of hyperkalemia in pediatric Lee, J. and Moffett, B.S. (2016). Treatment of pediatric
cardiac arrest: a prospective, randomized, simulation-based hyperkalemia with sodium polystyrene sulfonate. Pediatric
study. Pediatric Critical Care Medicine 16: e224–e230. Nephrology 31: 2113–2117.
Duff, J.P., Topjian, A., Berg, M.D., et al. (2018). 2018 American Littmann, L. and Gibbs, M.A. (2018). Electrocardiographic
Heart Association focused update on pediatric advanced life manifestations of severe hyperkalemia. Journal of
support: an update to the American Heart Association Electrocardiology 51: 814–817.
guidelines for cardiopulmonary resuscitation and Petrov, D.B. (2012). An electrocardiographic sine wave in
emergency cardiovascular care. Circulation 138: e731–e739. hyperkalemia. New England Journal of Medicine 366
Durfey, N., Lehnhof, B., and Bergeson, A. (2017). Severe (19): 1824.
hyperkalemia: can the electrocardiogram risk stratify for Prusakowski, M.K. and Chen, A.P. (2017). Pediatric sepsis.
short-term adverse events? West Journal of Emergency Emergency Medicine Clinics of North America 35: 123–138.
Medicine 18(5): 963–971. Tijssen, J.A. and Filler, G. (2017). When CRRT on ECMO is not
Hughes, H.K. and Kahl, L.K. (ed.) (2018). The Johns Hopkins enough for potassium clearance: a case report. Canadian
Hospital The Harriet Lane Handbook, 21st edn. Philadelphia, Journal of Kidney Health and Disease 4: 2054358117722559.
PA: Elsevier. doi: 10.1177/2054358117722559.
Kleinman, M.E., Chameides, L., Scheznayder, S.M., et al. Wooten, J.M., Kupferman, F.E., and Kupferman, J.C. (2019). A
(2010). Part 14: Pediatric Advanced Life Support: 2010 brief review of the pharmacology of hyperkalemia: causes
American Heart Association guidelines for and treatment. Southern Medical Journal 112(4): 228–233.
Downloaded from https://onlinelibrary.wiley.com/doi/ by National Institute Of Standard, Wiley Online Library on [06/03/2024]. See the Terms and Conditions (https://onlinelibrary.wiley.com/terms-and-conditions) on Wiley Online Library for rules of use; OA articles are governed by the applicable Creative Commons License
313

74

Parvovirus Requiring Blood Transfusion


Ashley Thibodeau
Seattle Children’s Hospital, Seattle, WA, USA

A 2-year-old male presents to urgent care with viral symp- Social History
toms, lethargy, and pale appearance.
He lives at home with mother, father, and 5-year-old sister.
No pets. No daycare.
History of Present Illness

Last evening, this 2-year-old male presented to urgent care Current Status
with viral symptoms, lethargy, and pale appearance. He has
a recent history of parvovirus, diagnosed by his primary The child is admitted to the acute care service and you are the
care provider (PCP) 5 days prior to presentation. He has provider covering his management. You receive a call from
been acting tired and looking pale, so his mother brought the acute care nurse who reports that the child is itching
him to urgent care. A complete blood count (CBC) was his arms and squirming around in the bed. You meet the
drawn and demonstrated a hemoglobin of 6.5 g/dL and child, his mother, and the nurse at the bedside and they report
hematocrit of 18%. Previous iron deficiency anemia screen- the itching began approximately 5 minutes ago. The red blood
ings at 12 and 19 months by his PCP had been normal. The cell transfusion has been infusing for 45 minutes. The child
child eats a regular diet and normal amount of milk for his had no prior rashes, skin infections, or history of eczema.
age. He was admitted to an acute care unit for slow blood The vital signs at the time of your assessment (45 minutes
transfusion for symptomatic relief of anemia. Following into transfusion) are temperature 37.0 C (98.6 F), heart rate
transfusion therapy, intravenous immunoglobulin admin- 102 bpm, respiratory rate 34 breaths per minute, blood pres-
istration was planned to treat red cell aplasia caused by sure 96/52 mmHg, and oxygen saturation 100% on room air.
parvovirus. The vital signs at the start of packed red blood cell infu-
sion were temperature 36.5 C (97.7 F), heart rate 98 bpm,
respiratory rate 32 breaths per minute, blood pressure
Past Medical History 92/50 mmHg, and oxygen saturation 100% on room air.
An urgent complete blood count is ordered with the
No significant past medical or surgical history. Normal results shown in Table 74.1.
term delivery with no pregnancy complications. He was
discharged from the hospital on day of life 2 with mother.
No medication or food allergies. The child is taking vitamin
Questions
D and children’s multivitamin.
Answer the following questions using the details provided.
1 Based on the data given, what are the differential diag-
Family History noses and the most likely diagnosis for the acute change
in this child?
No significant history. No history of anemia, no blood dis- 2 What is your immediate response for this patient?
orders or clotting disorders. Describe your assessment of each body system.

Cases in Pediatric Acute Care: Strengthening Clinical Decision Making, First Edition. Edited by Andrea M. Kline-Tilford and Catherine M. Haut.
© 2020 John Wiley & Sons Ltd. Published 2020 by John Wiley & Sons Ltd.
Downloaded from https://onlinelibrary.wiley.com/doi/ by National Institute Of Standard, Wiley Online Library on [06/03/2024]. See the Terms and Conditions (https://onlinelibrary.wiley.com/terms-and-conditions) on Wiley Online Library for rules of use; OA articles are governed by the applicable Creative Commons License
314 Cases in Pediatric Acute Care

Table 74.1 Complete blood count and reticulocyte count. reaction, febrile non-hemolytic reaction, allergic reaction,
anaphylactoid/anaphylactic reaction, and transfusion-
White blood cell count 9000/mm3 Normal
related acute lung injury. Non-immunologic complica-
Hemoglobin 7 g/dL Low tions include transmission of infection including
Hematocrit 19.8% Low cytomegalovirus and bacterial sepsis, transfusion-
Mean corpuscular volume 78 fL Normal associated circulatory overload, hypothermia, and other
Mean corpuscular hemoglobin 26 pg Normal metabolic derangements. The most common type of
Mean corpuscular hemoglobin 14.0% Normal transfusion reaction during red blood cell or platelet
concentration transfusion is an allergic reaction. There are additional,
Red blood cell count 4.1 × 106/ Normal less-common potential transfusion reactions that should
mm3 be considered when caring for a patient receiving a blood
Red cell distribution width 13.0% Normal product. In order of prevalence they are: febrile non-
Platelet count 250 000/ Normal hemolytic reaction, circulatory volume overload, acute
mm3 hemolysis, hypotensive reaction, bacterial contamination,
Reticulocyte count 2.2% High sepsis, acute lung injury, and air embolism.
Corrected reticulocyte index 3.6% Elevated
What is your immediate response for this patient?
Describe your assessment of each body system
Continuation of case: The child appears calm, pale, and is It is essential to verify that the blood transfusion has been
itching his arms. He is interactive with his mother, has no stopped and 0.9% saline is infusing intravenously. The
change in voice quality, and denies pain or stomach ache. patient should immediately be assessed for evidence of air-
His face is symmetric and pale. His oropharynx is pink way, breathing, or circulatory compromise. As you evaluate
without exudate or edema with tonsils 1+ bilaterally, ton- the patient, it is important to obtain any history of previous
gue protruded midline. His breath sounds are clear bilater- allergic reactions or exposures, any complaints of pain,
ally with no cough, wheeze, crackles, or increased work unusual sensation in the throat, change in voice, or gastro-
of breathing. He has a normal cardiac rate and rhythm intestinal symptoms including nausea and stomach ache.
at apex, with warm and well-perfused extremities with Vital signs should be obtained.
2-second capillary refill. His abdomen is protuberant, soft, Your evaluation by system should include the following.


non-tender, with normoactive bowel sounds. He has three
General: mental status, ability to respond to questions,
urticarial lesions on his neck and scratches on his arms
anxiety, perception of impending doom.


from itching. No other skin changes on lower extremities
Head, eyes, ear, nose and throat: facial color, nasal con-
or trunk are noted.
gestion, patency of the throat, mucosal edema.
3 Based on the additional information, what differential
diagnoses will you continue to consider? • Cardiovascular: chest pain, central and peripheral
perfusion.
4 What pharmacologic interventions may be necessary
for this patient? •• Lungs: breath sounds, work of breathing.
Integument: presence of urticarial or other rash,
5 Would you resume the blood transfusion? skin color.

It is necessary to verify the label on the blood transfusion


bag and also notify the blood bank of a suspected transfu-
Rationale and Evidence-based Practice sion reaction. Additional laboratory studies may be
Explanation required as directed by the blood bank.

Based on the data given, what are the differential Based on the additional information, what
diagnoses and the most likely diagnosis for the differential diagnoses will you continue to
acute change in this child? consider?
The onset of the child’s symptoms suggest that he is experi- The child is not having respiratory symptoms, such as tach-
encing a reaction to the blood transfusion. Blood transfusion ypnea, hypoxemia, bronchospasm, or dyspnea, so acute
reactions are divided into immediate immunologic compli- lung injury and circulatory volume overload are less likely
cations and non-immunologic complications. Immediate at this time. The most common signs/symptoms of a trans-
immunologic complications include hemolytic transfusion fusion reaction are fever, chills, pruritis, and urticaria. Any
Downloaded from https://onlinelibrary.wiley.com/doi/ by National Institute Of Standard, Wiley Online Library on [06/03/2024]. See the Terms and Conditions (https://onlinelibrary.wiley.com/terms-and-conditions) on Wiley Online Library for rules of use; OA articles are governed by the applicable Creative Commons License
Chapter 74 Parvovirus Requiring Blood Transfusion 315

increase in temperature with chills, rigors, hypotension, distress, hypotension, and gastrointestinal upset. The
nausea or vomiting should lead you to consider a hemolytic first-line intervention for anaphylaxis is intramuscular epi-
reaction or reaction due to bacterial contamination. Since nephrine (1:1000) at a dose of 0.01 mg/kg, with a maximum
he has not had an increase in temperature of 1 C or more dose 0.3 mg. In addition to the H1 antihistamine, an H2
and did not have a temperature above 38.0 C, febrile trans- antihistamine such as ranitidine may also be prescribed
fusion reaction and possible sepsis could be excluded from for cutaneous manifestations. Corticosteroids may also be
the differential diagnosis. However, new signs could still indicated. A normal saline bolus of 20 mL/kg may be admi-
arise and transfusion reactions can develop up to 24 hours nistered for hemodynamic instability. In the setting of
following the transfusion. One should always be aware of wheezing, albuterol, a β2-adrenergic receptor agonist bron-
the potential for a more severe reaction to develop and chodilator, may also be administered via inhaler or
for the signs which indicate a severe transfusion reaction. nebulizer.
Other findings of more severe transfusion reactions include
chills, rigors, nausea, vomiting, hypotension, respiratory
distress, hemoglobinuria, loss of consciousness, back pain, Would you resume the blood transfusion?
jaundice, abnormal bleeding, and oliguria. This child was
If the patient’s only symptom of blood product transfusion
experiencing an allergic blood transfusion that was limited
reaction was urticaria that resolved after diphenhydramine
to urticaria, though he required ongoing evaluation for
administration, the transfusion with the same unit can be
more extensive signs and symptoms. The CBC obtained
restarted at a slower rate with close observation. If addi-
during the transfusion demonstrated a slight rise in the
tional signs of allergic reaction or new signs of fever or other
hemoglobin and hematocrit, appropriate for the volume
type of reaction occur, the unit should be returned to the
of red cells that had been administered at the time the sam-
blood bank accompanied by appropriate transfusion reac-
ple was drawn.
tion laboratory sampling.

What pharmacologic interventions may be


necessary for this patient?
An H1 antihistamine is recommended to treat the symptom Case Resolution
of hives and itching. The pediatric dosing recommendation
is 1–2 mg/kg. If the reaction progressed and was consistent This child tolerated resumption of the packed cell transfu-
with anaphylaxis, additional intervention would be sion at a slower rate. The urticarial rash resolved and the
required according to Pediatric Advanced Life Support. follow-up CBC demonstrated an appropriate rise in hemo-
Signs of anaphylaxis include bronchospasm, respiratory globin and hematocrit.

Further Reading
American Academy of Blood Banks (2017). Circular of Lexicomp Online. Diphenhydramine. Updated 8 June 2019;
information for the use of human blood and blood accessed June 10 2019.
components. https://www.aabb.org/tm/coi/Documents/ Oakley, F.D., Woods, M., Arnold, S., et al. (2015) Transfusion
coi1017.pdf reactions in pediatric compared with adult patients.
American Academy of Pediatrics and American Heart Transfusion 55(3): 563–570.
Association (2016). Pediatric Advanced Life Support. Dallas,
TX: American Heart Association.
Delaney, M., Wendel, S., Bercovitz, R.S., et al. (2016).
Transfusion reactions: prevention, diagnosis, and treatment.
Lancet 388: 2825–2836.
Downloaded from https://onlinelibrary.wiley.com/doi/ by National Institute Of Standard, Wiley Online Library on [06/03/2024]. See the Terms and Conditions (https://onlinelibrary.wiley.com/terms-and-conditions) on Wiley Online Library for rules of use; OA articles are governed by the applicable Creative Commons License
317

75

A Teen With High Fever, Rash and Lethargy


Jessica Strohm Farber
Children’s Hospital of Philadelphia, Philadelphia, PA, USA

A 16-year-old, previously healthy adolescent female pre- Family History


sents to the emergency department (ED) with fever, weak-
ness, malaise, lethargy, and rash. The patient’s mother has a history of migraine headaches.
Her father has hypertension that developed in adulthood
and had childhood asthma. She has one sibling (10-year-
old female) who is healthy.
History of Present Illness

This 16-year-old became ill 2 days ago with fatigue,


headache, and abdominal pain. Her mother administered Current Status
ibuprofen for pain, increased fluids, and encouraged rest
believing the teen had a viral illness. Last night, she On arrival to the ED, the patient is febrile to 39.4 C
developed fever to 40.1 C (104.1 F), chills, vomiting, and (102.9 F) orally, pale, toxic appearing, tachypneic with a
weakness. She was not tolerating oral intake and remained respiratory rate of 28 breaths per minute, has sinus tachy-
febrile this morning with lethargy. She developed a rash cardia with a heart rate of 145 bpm, and blood pressure of
this afternoon and has not voided in 18 hours so was 78/42 mmHg. She is oriented but lethargic and weak with
brought to the ED. cool hands and feet and peripheral capillary refill of 3–4
seconds. Peripheral pulses are somewhat weak, but femoral
and jugular pulses are strong. She has a generalized
erythematous macular rash (Figures 75.1 and 75.2) and
Past Medical History oropharyngeal, tongue (Figure 75.3), and conjunctival
hyperemia. Her abdomen is soft without focal tenderness.
The patient was born at term and was a healthy infant. All Bowel sounds are hypoactive.
vaccines are up to date except for influenza vaccine that she Investigations including a blood culture, complete blood
refused this year. Her weight is at the 50th percentile and count, and basic metabolic panel are obtained with results
height at 60th percentile for gender and age. She has never as noted in Tables 75.1 and 75.2.
been hospitalized, but has had two prior ED visits in the last
5 years for sports-related injuries.
Questions

Answer the following questions using the details provided.


Past Surgical History
1 Based on the data given, what are the differential diag-
The teen had myringotomy tubes placed for recurrent otitis noses and what is the most likely diagnosis for this
media in early childhood. adolescent?

Cases in Pediatric Acute Care: Strengthening Clinical Decision Making, First Edition. Edited by Andrea M. Kline-Tilford and Catherine M. Haut.
© 2020 John Wiley & Sons Ltd. Published 2020 by John Wiley & Sons Ltd.
Downloaded from https://onlinelibrary.wiley.com/doi/ by National Institute Of Standard, Wiley Online Library on [06/03/2024]. See the Terms and Conditions (https://onlinelibrary.wiley.com/terms-and-conditions) on Wiley Online Library for rules of use; OA articles are governed by the applicable Creative Commons License
318 Cases in Pediatric Acute Care

Figure 75.1 Erythematous macular rash. Source: Courtesy of The


Centers for Disease Control and Prevention (CDC).

Figure 75.3 Oropharyngeal findings. Source: Courtesy of The


Centers for Disease Control and Prevention (CDC).

Table 75.1 Basic metabolic profile.

Sodium 136 mEq/L


Potassium 4.2 mEq/L
Chloride 101 mEq/L
Carbon dioxide 17 mEq/L
Blood urea nitrogen 28 mg/dL
Creatinine 2.2 mg/dL
Glucose 164 mg/dL
Calcium 7.8 mg/dL

Table 75.2 Complete blood count.


Figure 75.2 Erythematous macular rash. Source: Courtesy of The
White blood cell count 28 000/mm3
Centers for Disease Control and Prevention (CDC).
Hemoglobin 10.2 g/dL
Hematocrit 30.6%
2 What other historical information would be Platelet count 72 000/mm3
beneficial? Neutrophils 80%
3 What additional laboratory studies would be helpful to Lymphocytes 13%
understand the severity of the disease process and help Monocytes 5%
guide therapy? Eosinophils 2%
4 What are initial treatment priorities?
Downloaded from https://onlinelibrary.wiley.com/doi/ by National Institute Of Standard, Wiley Online Library on [06/03/2024]. See the Terms and Conditions (https://onlinelibrary.wiley.com/terms-and-conditions) on Wiley Online Library for rules of use; OA articles are governed by the applicable Creative Commons License
Chapter 75 A Teen With High Fever, Rash and Lethargy 319

Rationale and Evidence-based Practice (Wilkins et al., 2017). A serum lactate to evaluate for evi-
Explanation dence of metabolic acidosis and shock should be obtained,
and a blood gas can also support diagnosis.
Based on the data given, what are the differential Serial laboratory studies should be obtained to evaluate
diagnoses and what is the most likely diagnosis for the severity of shock and organ dysfunction (such as blood
this adolescent? gas and lactate) as well as to evaluate for disseminated
intravascular coagulation, anemia, thrombocytopenia,
Differential diagnoses for this patient include septic shock, and electrolyte abnormalities.
toxic shock syndrome, acute abdominal process, Kawasaki Lastly, the identification of infection source and any inter-
disease shock syndrome, viral syndrome, and pneumonia. ventions such as surgical debridement or others as necessary
The most likely diagnosis is toxic shock syndrome (TSS). to remove the infection source are indicated (Wilkins et al.,
Symptoms in TSS are associated with both the infectious 2017). Cerebrospinal fluid infectious studies should be con-
process and toxin secretion and include fever, rash, and sidered based on presenting symptoms (LeRiche et al., 2012).
hypotension. Other symptoms are those associated with In this case, a history of tampon use is the likely etiology of
serious acute bacterial infection, such as general malaise, the TSS in this adolescent female.
weakness, and chills. Headache, sore throat, and gastroin-
testinal symptoms such as emesis, nausea, diarrhea, and
abdominal pain may also occur (Gottlieb et al., 2018). What are initial treatment priorities?
The clinical case definition of Staphylococcus aureus TSS Fluid resuscitation, hemodynamic stabilization, and rever-
includes initial clinical findings of fever, a rash that is sal of shock are a significant priority. Following current
typically diffuse macular erythroderma, hypotension, and guidelines for the timely recognition and management of
multisystem involvement (Kimberlin et al., 2018). The child shock and monitoring for reversal of shock to ensure ade-
in this case has evidence of multisystem involvement quate end-organ perfusion is vital. Monitoring capillary
including emesis, elevated serum creatinine, and hemato- refill and pulses, urine output, mental status, and estab-
logic effects along with fever, rash, and hypotension, thus lished blood pressure targets are recommended. Treatment
meeting this case definition. priorities include the identification and control of fever
source, appropriate antimicrobial therapy, as well as possi-
What other historical information would be ble surgical evaluation for instances of soft tissue involve-
beneficial? ment. As with any case of septic shock, the timely
administration of appropriate broad-spectrum antibiotics
TSS may be caused by strains of group A Streptococcus or,
is imperative. In suspected TSS, coverage for group
more commonly, toxin-producing strains of Staphylococcus
A Streptococcus, Staphylococcus aureus species, as well as
aureus. TSS is typically subdivided into two categories:
methicillin-resistant Staphylococcus aureus (MRSA) is indi-
menstrual TSS and non-menstrual TSS (LeRiche et al.,
cated (Wilkins et al., 2017).
2012). While the majority of menstrual toxic shock cases
are associated with tampon use, TSS can be related to Continuation of case: The patient receives a total of
bacterial infections such as pneumonia, pharyngitis, soft 60 mL/kg crystalloid fluids followed by an epinephrine infu-
tissue infections, or viral infections including influenza sion at 0.05 μg/kg per min for hemodynamic support. She
(Gottlieb et al., 2018). Eliciting the history for tampon receives vancomycin and ceftriaxone for presumed sepsis
use in females may reveal contributory factors. and is admitted to the pediatric intensive care unit (PICU).
Her blood pressure and work of breathing improve after fluid
What additional laboratory studies would be resuscitation and initiation of the epinephrine infusion. The
helpful to understand the severity of the disease blood culture is found to be positive for Staphylococcus
process and help guide therapy? aureus at 6 hours. Infectious disease specialty is consulted.

Given the presenting symptom of tachypnea, a chest radi-


ograph may be obtained. However, tachycardia, tachypnea,
and hypotension are likely clinical manifestations of septic Additional Questions and Evidence-
shock. Decreased oral intake may indicate a component of based Rationale
hypovolemic shock.
Additional laboratory evaluation could include creatine 5 What antibiotics are appropriate at this time?
phosphokinase with muscular complaints, hepatic function 6 How would you determine if the interventions were
tests, and additional cultures to identify fever source effective?
Downloaded from https://onlinelibrary.wiley.com/doi/ by National Institute Of Standard, Wiley Online Library on [06/03/2024]. See the Terms and Conditions (https://onlinelibrary.wiley.com/terms-and-conditions) on Wiley Online Library for rules of use; OA articles are governed by the applicable Creative Commons License
320 Cases in Pediatric Acute Care

What antibiotics are appropriate at this time? How would you determine if the interventions
were effective?
The rapid administration of broad-spectrum antibiotics is
indicated, including coverage for MRSA. This patient has Appropriate shock resuscitation should result in improved
received ceftriaxone and vancomycin. However, clinda- hemodynamics with resolution of hypotension and tachy-
mycin is indicated as well in the treatment of TSS, largely cardia. Metabolic acidosis and any elevation in serum lac-
to inhibit toxin production but also to potentiate phagocy- tate should also resolve with adequate resuscitation. The
tosis and other beneficial effects (Wilkins et al., 2017). fever curve should improve as well, and the patient should
Other antimicrobials may be used instead of clindamycin become afebrile by approximately day 4–5 of illness.
to reduce toxin release including erythromycin, Gastrointestinal symptoms such as abdominal pain and
fluoroquinolones, linezolid, or rifampin (Gottlieb diarrhea may persist for 7–10 days. Skin changes may prog-
et al., 2017). ress or develop into desquamation at approximately day
7–10, then begin to resolve (LeRiche et al., 2012).

References
Gottlieb, M., Long, B., and Koyfman, A. (2018). The evaluation LeRiche, T., Black, A., and Fleming, N.A. (2012). Toxic shock
and management of toxic shock syndrome in the emergency syndrome of a probable gynecologic source in an adolescent:
department: a review of the literature. Journal of Emergency a case report and review of the literature. Journal of Pediatric
Medicine 54(6): 807–814. doi: 10.1016/j. and Adolescent Gynecology 25(6): e133–e137. doi: 10.1016/j.
jemermed.2017.12.048. jpag.2012.08.011.
Kimberlin, D.W., Brady, M.T., Jackson, M.A., and Long, S.S. Wilkins, A.L., Steer, A.C., Smeesters, P.R., and Curtis, N.
(ed.) (2018) Red Book 2018: Report of the Committee on (2017). Toxic shock syndrome: the seven Rs of management
Infectious Diseases, 31st edn. Itasca, IL: American Academy and treatment. Journal of Infection 74(Suppl 1): S147–S152.
of Pediatrics. doi: 10.1016/S0163-4453(17)30206-2.

Further Reading
Centers for Disease Control and Prevention (CDC) (2011). neonatal septic shock. Critical Care Medicine 45: 1061–1093.
Toxic shock syndrome (other than streptococcal) (TSS) 2011 doi: 10.1097/CCM.0000000000002425.
case definition. Available at https://wwwn.cdc.gov/nndss/ Lin, Y., Cheng, M., Lo, M., and Chien, S. (2015). Early
conditions/toxic-shock-syndrome-other-than- differentiation of Kawasaki disease shock syndrome and
streptococcal/case-definition/2011/ toxic shock syndrome in a pediatric intensive care unit.
Davis, A.L., Carcillo, J.A., Aneja, R.K., et al. (2017). American Pediatric Infectious Disease Journal 34(11): 1163–1167. doi:
College of Critical Care Medicine clinical practice 10.1097/INF.0000000000000852.
parameters for hemodynamic support of pediatric and
Downloaded from https://onlinelibrary.wiley.com/doi/ by National Institute Of Standard, Wiley Online Library on [06/03/2024]. See the Terms and Conditions (https://onlinelibrary.wiley.com/terms-and-conditions) on Wiley Online Library for rules of use; OA articles are governed by the applicable Creative Commons License
321

76

A 9-Year-Old With Scoliosis


Alicia McCarthy
Nemours/Alfred I. duPont Hospital for Children, Wilmington, DE, USA

A 9-year-old female presents to the outpatient orthopedic Social History


clinic for evaluation of scoliosis.
The patient lives at home with her parents and brother and
is in the 4th grade. She is a competitive gymnast, with
History of Present Illness practice and competitions taking up 15 hours per week.

This 9-year-old girl was seen recently by the primary care


provider (PCP) for annual well child check. PCP noticed left
thoracic prominence on Adams forward bend test. There- Current Status
fore, she was referred to orthopedics for further evaluation.
Her family had not previously noticed asymmetry of her This patient is an alert and cooperative female who is in no
back and the patient denies having pain. acute distress, and with age-appropriate affect and mood.
She has no dysmorphic features and walks and changes
positions independently. Neck range of motion (ROM)
Past Medical History shows loss of 10 of left lateral rotation compared to right.
Also loss of 20 of left head tilt compared to right. No
The patient was born at 36 weeks’ gestation, mother with increased work of breathing. Standing evaluation reveals
premature rupture of membranes, delivered vaginally, with her head is level over the pelvis, left shoulder higher than
no neonatal complications. Development included rolling right. No trunk shift. Iliac crest height is symmetric. No
over at 3 months and walking at 14 months. She has had pain on palpation of the spine, with forward bending or
all required immunizations and received annual influenza hyperextension. Adams forward best test reveals left
vaccine. She has never been hospitalized, but did require thoracic prominence. Skin shows no café au lait spots,
physical therapy for an ankle sprain 2 years ago. There hemangiomas, hairy patches, or skin dimpling. Bilateral
are no chronic health conditions. Her current weight is upper extremities are grossly symmetrical with normal
30 kg, height 115 cm. alignment and full ROM. Bilateral lower extremities show
left calf circumference is 1.5 cm smaller than right, left leg
also 1 cm shorter than right, with bilaterally normal
Past Surgical History alignment and full ROM.

No previous surgical procedures.

Questions
Family History Answer the following question using the details provided.
Negative for scoliosis or other orthopedic issues. No neuro- 1 Based on history and physical examination, what are
muscular conditions in the family. the differential diagnoses for this child?

Cases in Pediatric Acute Care: Strengthening Clinical Decision Making, First Edition. Edited by Andrea M. Kline-Tilford and Catherine M. Haut.
© 2020 John Wiley & Sons Ltd. Published 2020 by John Wiley & Sons Ltd.
Downloaded from https://onlinelibrary.wiley.com/doi/ by National Institute Of Standard, Wiley Online Library on [06/03/2024]. See the Terms and Conditions (https://onlinelibrary.wiley.com/terms-and-conditions) on Wiley Online Library for rules of use; OA articles are governed by the applicable Creative Commons License
322 Cases in Pediatric Acute Care

Figure 76.1 Scoliosis radiographs.

Continuation of case: Further examination of the upper refers to patients for whom there is no specific cause; how-
extremities reveals 5/5 strength in all major muscle groups. ever, there is a genetic component. Secondary scoliosis
Sensation intact to light touch bilaterally. Patellar deep ten- refers to patients who have another diagnosis that is caus-
don reflexes are 3+ on left, 1+ on right. Achilles deep ten- ing the scoliosis. Given this child’s age, unusual curve pat-
don reflexes are 3+ on left, 1+ on right. Abdominal reflexes tern, and abnormalities seen on physical and neurologic
are asymmetric. Babinski negative bilaterally. Ankle clonus examination, further work-up is necessary to determine a
of three beats on left, zero beats on right. The patient walks possible secondary cause.
with a heel–toe gait and bears equal weight through both
lower extremities. She can walk on her heels and toes
independently with normal strength and coordination. Based on these additional findings, what are the
Patient was sent to radiology for erect posteroanterior/ next steps in management?
lateral scoliosis radiographs (entire spine on single cassette) After initial spine radiographs, this patient should receive
(Figure 76.1). a full spine (cervical, thoracic and lumbar) MRI to inves-
2 Based on these additional findings, what are the next tigate any intraspinal pathology that may be contributing
steps in management? to her curve. Most common findings include Chiari
3 What time frame would be appropriate to obtain I malformation, syrinx and/or tethered cord. Approxi-
additional imaging? mately 20% of patients with scoliosis under the age of
10 have one of these findings. The pathophysiology of
Chiari I malformation is poorly understood, although
some have suggested that the formation of a syrinx, a
Rationale and Evidence-based Practice
fluid-filled cavity located within the spinal cord or in
Explanation the brainstem, may cause anterior horn cell dysfunction,
resulting in scoliosis (Kelly et al., 2015). If imaging is pos-
Based on history and physical examination, what
itive, referral to neurosurgery would be appropriate. If
are the differential diagnoses for this child?
imaging is negative, referral to neurology for evaluation
The differential diagnoses for this child include idiopathic of other etiologies would be the next step.
scoliosis or secondary scoliosis. Scoliosis is defined as a lat- For idiopathic scoliosis, treatment with a trunk brace is
eral curve in the spine of 10 or more. Idiopathic scoliosis recommended if a curve reaches 25 while the child has
Downloaded from https://onlinelibrary.wiley.com/doi/ by National Institute Of Standard, Wiley Online Library on [06/03/2024]. See the Terms and Conditions (https://onlinelibrary.wiley.com/terms-and-conditions) on Wiley Online Library for rules of use; OA articles are governed by the applicable Creative Commons License
Chapter 76 A 9-Year-Old With Scoliosis 323

growth remaining and surgical intervention may be neces- What time frame would be appropriate to obtain
sary if the scoliosis progresses to the severe range, additional imaging?
usually 50 . For secondary scoliosis, treatment recommen-
The patient has abnormal reflexes and calf size, but
dations depend on the etiology and the location of the
strength, sensation and gait are unaffected by the scoliosis.
curve. If the child has a Chiari I malformation, syrinx
Therefore, MRI should be obtained urgently, but not emer-
and/or tethered cord, surgical correction may result in
gently. Waiting up to several weeks, as long as the patient
improvement or lack of progression of the scoliosis. At
has no change in neurologic status, would be appropriate.
minimum, this patient would require ongoing monitoring
of her spine to evaluate for any changes to the curve.

Reference
Kelly, M.P., Guillaume, J.T., and Lenke, L.G. (2015). Spinal Neurosurgical Clinics of North America 26(4): 579–585. doi:
deformity associated with Chiari malformation. 10.1016/j.nec.2015.06.005.

Further Reading
Chotai, S., Basem, J., Gannon, S., et al. (2018). Effect of neural axis anomalies in idiopathic scoliosis: a systematic
posterior fossa decompression for Chiari malformation I on review. Spine Journal 18(7): 1261–1271. doi: 10.1016/j.
scoliosis. Pediatric Neurosurgery 53(2): 108–115. doi: spinee.2018.02.013.
10.1159/000485254.
Heemskerk, J.L., Kruyt, M.C., Colo, D., Castelein, R.M., and
Kempen, D.H.R. (2018). Prevalence and risk factors for
Downloaded from https://onlinelibrary.wiley.com/doi/ by National Institute Of Standard, Wiley Online Library on [06/03/2024]. See the Terms and Conditions (https://onlinelibrary.wiley.com/terms-and-conditions) on Wiley Online Library for rules of use; OA articles are governed by the applicable Creative Commons License
325

77

An Unresponsive Infant
Charlene Pringle
University of Florida, Gainesville, FL, USA

A 9-month-old infant boy with no significant past medical Past Medical History
history was admitted to the pediatric intensive care unit
(PICU) after cardiopulmonary arrest at home. The baby was born at term and went home with his
mother. He has had all required vaccines up to 6 months
and his height and weight recorded on a growth chart
noted weight in the 16th percentile, length in the 8th
History of Present Illness percentile, weight for length in the 50th percentile, and
head circumference well above the 99th percentile.
This 9-month-old baby who was born at term is being
Newborn metabolic screening was reported as normal.
transferred from a referring hospital following cardiopul-
His parents state they discussed the large head with the
monary arrest with roughly 1 hour without cardiac or
primary care provider at a well child visit early in life.
respiratory function prior to return of spontaneous circu-
He was also spitting up after every meal. They never
lation (ROSC). Parents report that he has been “sick” and
noticed any abnormal movements and do not report any
has seen his primary care provider several times since
known trauma. The infant is home with parents, does
birth for respiratory illnesses and vomiting, but has never
not attend daycare, and there are no other caregivers.
required hospital admission. He has not been febrile
Social services have been involved with this family and
recently. The father of the infant reports that he put the
there is currently an open case for another child in the
baby in his bouncy seat to sleep. He heard him coughing,
home per the emergency department (ED) social worker.
and went to check on him. After finding him limp and not
breathing, he attempted stimulation without success.
Emergency medical service (EMS) was called and arrived
5–10 minutes after the call. No cardiopulmonary resusci-
tation was performed while awaiting EMS. On arrival of Past Surgical History
EMS, baby was noted to be in full arrest. Compressions
and one dose of epinephrine was given en route to the hos- The infant had a circumcision at age 1 day of life, with no
pital. Compressions continued with four additional doses bleeding or complications.
of epinephrine administered at referring hospital with
ROSC roughly 1 hour after initial call. The first blood
gas obtained indicated a pH of 6.8. The patient was intu-
bated and transferred for further care. Initial arterial blood Family History
gas analysis on arrival of transport team confirmed severe
metabolic acidosis with pH 6.7 and lactate 11.7 mmol/L. Parents are both healthy, but the father is being treated
The infant received two 20 mL/kg isotonic fluid boluses with methadone for substance use. There is a 3-year-old sib-
and was started on an epinephrine infusion for ling who has been treated in the ED several times for viral
hypotension. illnesses.

Cases in Pediatric Acute Care: Strengthening Clinical Decision Making, First Edition. Edited by Andrea M. Kline-Tilford and Catherine M. Haut.
© 2020 John Wiley & Sons Ltd. Published 2020 by John Wiley & Sons Ltd.
Downloaded from https://onlinelibrary.wiley.com/doi/ by National Institute Of Standard, Wiley Online Library on [06/03/2024]. See the Terms and Conditions (https://onlinelibrary.wiley.com/terms-and-conditions) on Wiley Online Library for rules of use; OA articles are governed by the applicable Creative Commons License
326 Cases in Pediatric Acute Care

Current Status Table 77.3 Arterial blood gas analysis.

pH 7.1
The baby is intubated and mechanically ventilated, receiv-
PCO2 58 mmHg
ing epinephrine infusion at 0.1 μg/kg per min. His exami-
nation indicates macrocephaly and a bulging and tense PO2 234 mmHg
anterior fontanelle. Pupils are unequal in size, with reactiv- Bicarbonate 15 mEq/L
ity as follows; left pupil mid-position fixed and 7 mm; right Base excess +3
pupil 2–3 mm and sluggishly reactive to light. Corneal
reflexes are absent. Heart sounds are normal, with strong
peripheral pulses and tachycardia. There is clear nasal dis-
Table 77.4 Coagulation studies.
charge, and lungs are clear bilaterally to auscultation. The
infant is breathing over ventilator breaths with retractions Prothrombin time (PT) 14.8 s
with some agonal breaths. Abdomen is soft and non- Partial thromboplastin time (PTT) 46 s
distended. No response to deep pain, nor gag reflex noted
International normalized ratio 2.0
with suctioning. Skin is mottled with prolonged capillary
Fibrinogen 185 mg/dL
refill time, cyanosis to feet. No bruising or other skin lesions

Table 77.1 Complete metabolic panel.

Sodium 140 mEq/L


Potassium 5.1 mEq/L
Chloride 109 mEq/L
Carbon dioxide 10 mEq/L
Blood urea nitrogen 14 mg/dL
Creatinine 0.32 mg/dL
Glucose 100 mg/dL
Calcium 8.1 mg/dL
Albumin 2.8 mg/dL
Total protein 5.4 g/dL
Aspartate aminotransferase 230 U/L
Alanine aminotransferase 568 U/L
Alkaline phosphatase 156 U/L
Total bilirubin 0.3 mg/dL

Table 77.2 Complete blood count and inflammatory markers.

White blood cell count 14 000/mm3


Hemoglobin 8 g/dL Figure 77.1 Non-contrast CT brain, image 1.

Hematocrit 25.6%
observed. A complete metabolic panel, complete blood
Platelet count 340 000/mm3 count, coagulation studies, arterial blood gas analysis,
Neutrophils 51% and inflammatory markers were obtained, with results as
Lymphocytes 45% shown in Tables 77.1–77.4. A chest radiograph is unre-
Monocytes 1% markable and shows endotracheal tube position 1 cm above
Eosinophils 3% the carina. Non-contrast computed tomography (CT) of the
Erythrocyte sedimentation rate 42 mm/hour head reveals large bilateral acute-on-chronic subdural
hematomas, as well as early diffuse cerebral edema
C-reactive protein 304 mg/L
(Figures 77.1–77.3).
Downloaded from https://onlinelibrary.wiley.com/doi/ by National Institute Of Standard, Wiley Online Library on [06/03/2024]. See the Terms and Conditions (https://onlinelibrary.wiley.com/terms-and-conditions) on Wiley Online Library for rules of use; OA articles are governed by the applicable Creative Commons License
Chapter 77 An Unresponsive Infant 327

Questions

Answer the following questions using the details provided.


1 Based on the data given, what are the differential diag-
noses and the most likely diagnosis for this child?
2 What other studies are indicated to confirm your
diagnosis?

Rationale and Evidence-based Practice


Explanation

Based on the data given, what are the differential


diagnoses and the most likely diagnosis for
this child?
There could be many potential diagnoses for this child, but
the primary differential diagnoses include abusive head
trauma, accidental injury, sudden infant death syndrome,
underlying bleeding disorder, and toxic ingestion. After
resuscitation, determining the cause of unresponsiveness
is the next challenge. The results of the CT scan obviously
offer information, but the eye examination is a quicker eval-
Figure 77.2 Non-contrast CT brain, image 2. uation and can be telling. The history does not suggest an
accidental injury nor an ingestion, but if the infant is crawl-
ing he may easily find something on the floor to ingest.
The most likely diagnosis for this infant is abusive head
trauma. The acute-on-chronic nature of the subdural hema-
tomas noted on CT scan demonstrate repeat bleeding events.
The denial of traumatic injury as well as the severity of injury
limits the likelihood of accidental injury. An underlying
bleeding disorder is extremely rare, but should be considered
if a pertinent patient or family history is obtained.

What other studies are indicated to confirm your


diagnosis?
An additional diagnosis for this infant is hypoxic ischemic
encephalopathy due to his prolonged cardiopulmonary
arrest. The early global cerebral edema on the initial head
CT is characteristic of hypoxic ischemic encephalopathy
and the elevated lactic acid, liver enzymes and metabolic
acidosis are all evidence of global ischemic insult. Addi-
tional studies should also include a skeletal survey and
indirect ophthalmoscopy evaluation to determine addi-
tional injury. If the child is sufficiently stable, brain mag-
netic resonance imaging (MRI) can be more precise than
CT, and there is no exposure to radiation. Serum
amylase/lipase, liver enzymes and urinalysis should be
Figure 77.3 Non-contrast CT brain, image 3. obtained to screen for abdominal trauma.
Downloaded from https://onlinelibrary.wiley.com/doi/ by National Institute Of Standard, Wiley Online Library on [06/03/2024]. See the Terms and Conditions (https://onlinelibrary.wiley.com/terms-and-conditions) on Wiley Online Library for rules of use; OA articles are governed by the applicable Creative Commons License
328 Cases in Pediatric Acute Care

Table 77.5 Brain Trauma Foundation: pediatric traumatic brain


injury guidelines.

Monitoring
Intracranial pressure (ICP) monitoring
Advanced neuromonitoring
Specific guidelines for CT imaging

Thresholds
Treatment of ICP to <20 mmHg
Treatment for cerebral perfusion pressure target between 40 and
50 mmHg

Treatment
Hyperosmolar therapy: hypertonic saline, initial 3%, specific use
for 23.4%
Specific uses for analgesics, sedatives and neuromuscular
blockade to avoid brain hypoperfusion
Seizure prophylaxis
Normoventilation
Normothermia, hypothermia to control increasing ICP
Cerebrospinal fluid drainage
Use of barbiturates for hemodynamically stable patients with
refractory intracranial hypertension
Figure 77.4 Retinal hemorrhage.
Decompressive craniotomy for neurologic deterioration,
increased ICP not responsive to medical therapy, or herniation
Retinal hemorrhages are found in 65–85% of patients
Initiation of early enteral feedings for nutrition
with abusive head trauma (Figure 77.4). It is hypothesized
that the eye is also subjected to the acceleration/decelera- Use of corticosteroids is not recommended
tion forces that the brain is subjected to during abusive Source: Kochanek et al. (2019).
traumatic injury. These hemorrhages are typically bilateral,
multilayered, and extend to the periphery. Rib fractures are
commonly encountered when the mechanism of injury is 4 What determines effective therapy?
shaking; other bony fractures can also be associated. 5 What are some risk factors for non-accidental trauma?

Continuation of case: The infant is admitted to the PICU


and standard care for hypoxic ischemic encephalopathy What does supportive care for hypoxic ischemic
and traumatic brain injury is instituted. Pediatric neurosur- encephalopathy and traumatic brain injury
gery is consulted, emergently performs a subdural tap include?
through the fontanelle and drains 100 mL of blood, after
The ultimate goal in treating traumatic brain injury is to
which the pupils transiently become equal in size at
reduce secondary brain injury by limiting cerebral ischemia
3 mm and sluggishly reactive bilaterally. Despite standard
and optimizing cerebral blood flow. The Brain Trauma
supportive care including seizure prophylaxis, head eleva-
Foundation has recently published the third edition of evi-
tion and midline, normoxia, euglycemia, normothermia,
dence-based guidelines for management of traumatic brain
normoventilation and adequate blood pressure, the infant
injury in children, which includes the recommendations
progresses to brain death on day 4 of hospitalization.
shown in Table 77.5.

Additional Questions and Evidence- What determines effective therapy?


based Rationale In the case of hypoxic ischemic encephalopathy and abu-
sive head trauma, the only mechanisms available are sup-
3 What does supportive care for hypoxic ischemic portive care and surgical intervention if indicated. In this
encephalopathy and traumatic brain injury include? case, drainage of the subdural hematoma proved to be a
Downloaded from https://onlinelibrary.wiley.com/doi/ by National Institute Of Standard, Wiley Online Library on [06/03/2024]. See the Terms and Conditions (https://onlinelibrary.wiley.com/terms-and-conditions) on Wiley Online Library for rules of use; OA articles are governed by the applicable Creative Commons License
Chapter 77 An Unresponsive Infant 329

temporizing measure. Overall supportive care includes sei- Continuous electroencephalography may be useful in
zure prophylaxis and maintenance of normonatremia, ensuring seizure control.
euglycemia, normothermia, normoventilation, and normo-
tension. The head should be elevated and kept in the mid-
What are some risk factors for non-accidental
line to optimize cerebral venous return. Avoidance of
trauma?
hypoxia is paramount. Avoiding hyponatremia lessens risk
of increase in cerebral edema. Avoiding hypoglycemia or Identified risk factors for non-accidental traumatic brain
hyperglycemia optimizes cerebral metabolic demand. injury fit into two categories: household risk factors and
Avoiding hypoventilation or hyperventilation optimizes risk factors in the child. Household risk factors include sub-
cerebral blood flow. Ensuring normotension is important stance abuse, mental illness, domestic violence, single par-
for maintaining cerebral perfusion pressure. While thera- enting, poverty, increased family size, and social isolation.
peutic hypothermia has proven successful in other popula- While traditionally it was thought that non-related male
tions, there is not sufficient evidence in children to caregivers were typical perpetrators, recent demographic
recommend this therapy as routine at this time. Fever evidence demonstrates that the majority of perpetrators
should be avoided to optimize cerebral metabolic demand. are the parents of the child. Risk factors in the child include
Frequent, repeated physical examinations including younger age (<3 years), prematurity, low birthweight,
focused neurologic assessment are needed to determine chronic or recurrent illness, and either a developmental
the effectiveness of therapy or progression to brain death. or physical disability.

Reference
Kochanek, P., Tasker, R.C., Carney, N., et al. (2019) Guidelines Foundation guidelines, executive Summary. Neurosurgery
for the management of pediatric severe traumatic brain 84(6): 1169–1178. doi: 10.1093/neuros/nyz051.
injury, third edition: update of the Brain Trauma

Further Reading
Armijo, V. (2016). Child abuse and neglect. In: Comprehensive Care, 5th edn (ed. B.P. Fuhrman and J.J. Zimmerman), 929–
Critical Care: Pediatric (ed. J. Stockwell and M. Kutko), 951. Philadelphia, PA: Elsevier.
763–774. Mount Prospect, IL: Society for Critical Care O’Lynnger, T.M., Shannon, C.N., and Le, T.M. (2016).
Medicine. Standardizing ICU management of pediatric traumatic
Fink, E., Clark, R., Bayir, H., et al. (2017). Hypoxic-ischemic brain injury is associated with improved outcomes at
encephalopathy: pathobiology and therapy of the post- discharge. Journal of Neurosurgery Pediatrics 17: 19–26. doi:
resuscitation syndrome in children. In: Pediatric Critical 10.3171/2015.5.PEDS1544.
Downloaded from https://onlinelibrary.wiley.com/doi/ by National Institute Of Standard, Wiley Online Library on [06/03/2024]. See the Terms and Conditions (https://onlinelibrary.wiley.com/terms-and-conditions) on Wiley Online Library for rules of use; OA articles are governed by the applicable Creative Commons License
331

78

A Toddler With Abdominal Pain, Diarrhea and Lethargy


Jessica Pech
Ann & Robert H. Lurie Children’s Hospital of Chicago, Chicago, IL, USA

An 18-month-old male presents to the emergency depart- circumference. Developmental milestones reached: rolled
ment (ED) with irritability, suspected abdominal pain, over at 5 months, crawled at 9 months, walked at
and lethargy. 13 months, and is saying about six words at present.

History of Present Illness Past Surgical History


An afebrile 18-month-old male presents to the ED with irri- Uncomplicated circumcision the first day of life.
tability, non-bloody diarrhea, and lethargy for the past
24 hours. His mother describes the pain as intermittent,
as he plays in between. Each episode is lasting about
10–15 minutes followed by a period of fatigue. He has
Family History
had decreased oral intake and increased lethargy over the
Mother has migraine headaches, father with elevated
past 12 hours and had one episode of non-bilious emesis
cholesterol, maternal grandmother with a history of breast
this morning and his last wet diaper was 12 hours ago.
cancer, treated and in remission, and maternal grandfather
A week ago, the child had symptoms of an upper
has hypertension, type 2 diabetes mellitus, coronary artery
respiratory infection and a low-grade fever but these symp-
disease, s/p coronary stent placement.
toms have resolved. His 4-year-old sister is currently sick
with symptoms of a cold. The child attends daycare and
there has been no recent travel.
Social History

He lives with his parents and 4-year-old sister (healthy),


Past Medical History
and attends daycare.
The patient was born full term via uncomplicated vaginal
delivery. Mother had prenatal care starting at 10 weeks’
gestation and her laboratory testing was negative for Current Status
sexually transmitted infection, HIV, rubella and group
B Streptococcus. The baby required phototherapy for hyper- Initial vital signs in the ED are temperature 38.5 C
bilirubinemia and was discharged home on the third day of (101.3 F), heart rate 124 bpm, respiratory rate 28 breaths
life. He has received required immunizations up to current per minute, and blood pressure 96/48 mmHg. The child
age, including MMR (measles, mumps, rubella) and first is awake and alert and appears comfortable. The only
varicella and first and second influenza vaccinations. abnormal physical finding is mild diffuse abdominal
Growth and development has been as targeted. He is at tenderness with palpation. Laboratory results are shown
the 50% for weight, 60% for height, and 85% for head in Tables 78.1 and 78.2. Abdominal X-ray showed no

Cases in Pediatric Acute Care: Strengthening Clinical Decision Making, First Edition. Edited by Andrea M. Kline-Tilford and Catherine M. Haut.
© 2020 John Wiley & Sons Ltd. Published 2020 by John Wiley & Sons Ltd.
Downloaded from https://onlinelibrary.wiley.com/doi/ by National Institute Of Standard, Wiley Online Library on [06/03/2024]. See the Terms and Conditions (https://onlinelibrary.wiley.com/terms-and-conditions) on Wiley Online Library for rules of use; OA articles are governed by the applicable Creative Commons License
332 Cases in Pediatric Acute Care

Table 78.1 Basic metabolic panel. Meckel’s diverticulum. Reviewing the primary symptoms
of each of these problems would not support a diagnosis
Sodium 140 mEq/L
for this child. Gastroenteritis would entail more persistent
Potassium 3.6 mEq/L vomiting and diarrhea, malrotation with or without volvu-
Chloride 95 mEq/L lus would most likely present with bilious emesis, Meck-
Carbon dioxide 20.5 mEq/L el’s diverticulum would present with rectal bleeding, and
Blood urea nitrogen 12 mg/dL appendicitis is very unusual in this age child.
Creatinine 0.5 mg/dL
Glucose 82 mg/dL
Calcium 9.8 mg/dL
Additional Questions and Evidence-
based Rationale
Table 78.2 Complete blood count. 2 Describe the incidence, etiology and common risk fac-
tors associated with intussusception.
White blood cell count 13 500/mm3
3 What are the most common presenting symptoms asso-
Hemoglobin 11.8 g/dL ciated with intussusception?
Hematocrit 33% 4 What is the preferred method of radiologic imaging
Platelet count 306 000/mm3 used to diagnose intussusception and what other meth-
Neutrophils 78% ods of imaging may be considered?
Lymphocytes 12% 5 What is the preferred method of non-operative manage-
Bands 2% ment of intussusception?
6 What symptoms and risk factors are associated with
Eosinophils 2%
recurrent intussusception after enema reduction?
7 When is surgical intervention indicated for
intussusception?
obstruction and moderate stool burden. Abdominal ultra-
sound is shown in Figure 78.1.

Describe the incidence, etiology and common risk


factors associated with intussusception
Question
Intussusception occurs in an estimated 3.4 in 10 000 chil-
Answer the following question using the details provided. dren each year and is most prevalent in children less than
3 years of age. Approximately 90% of intussusception cases
1 Based on the available data, what are the possible differ- are classified as ileocolic, involving both the ileum and
ential diagnoses and the most likely diagnosis for colon at the region of the ileocecal valve. The remainder
this child? of intussusception cases are isolated to the small bowel or
colon. An estimated 90% of pediatric intussusception cases
are idiopathic and attributed to a recent respiratory or gas-
trointestinal viral infection with no identifiable or patholo-
Rationale and Evidence-based Practice
gic lead point (Gluckman et al., 2017). Viral infections such
Explanation as adenovirus, norovirus, and rotavirus have been linked to
intussusception as a result of hypertrophic lymphatic tissue
Based on the available data, what are the possible
within the terminal ileum, known as Peyer’s patches, that
differential diagnoses and the most likely
acts as a lead point (Gluckman et al., 2017). Other cases of
diagnosis for this child?
intussusception, occurring in children over the age of
With regard to the presentation of lethargy, decreased 5 years, can result from other pathologic problems includ-
appetite, and colicky intermittent pain, the most likely ing intestinal duplication cysts, polyps, Meckel’s diverticu-
diagnosis for this child is intussusception. In many cases, lum, and lymphoma (Gluckman et al., 2017). Further risk
there is blood in the stool, noted as “currant jelly” in factors include a history of cystic fibrosis complicated by
appearance. Differential diagnoses include, but are not distal intestinal obstruction syndrome, the presence of prior
limited to, viral gastroenteritis, malrotation, mesenteric abdominal surgical adhesions, or Henoch–Schönlein
lymphadenitis, intussusception, acute appendicitis, or purpura.
Downloaded from https://onlinelibrary.wiley.com/doi/ by National Institute Of Standard, Wiley Online Library on [06/03/2024]. See the Terms and Conditions (https://onlinelibrary.wiley.com/terms-and-conditions) on Wiley Online Library for rules of use; OA articles are governed by the applicable Creative Commons License
Chapter 78 A Toddler With Abdominal Pain, Diarrhea and Lethargy 333

Figure 78.1 Abdominal ultrasound: target sign noted superior to umbilicus, no lead point or free fluid.

What are the most common presenting symptoms What is the preferred method of non-operative
associated with intussusception? management of intussusception?
The presenting symptoms of intussusception may be non- The presence of suspected or confirmed intussusception
specific or associated with the classic triad of symptoms that requires urgent evaluation and treatment. Initial manage-
includes vomiting, colicky abdominal pain, and hemato- ment includes intravenous fluid resuscitation, serial
chezia or “currant jelly” stools (Gluckman et al., 2017). abdominal examinations, review of diagnostic imaging,
A typical pain episode includes intermittent colicky abdom- and pediatric surgery consultation to evaluate for enema
inal pain with legs drawn followed by periods of fatigue or reduction versus surgical intervention. Enema reduction
lethargy. Additional symptoms may include fever, diarrhea, is the standard of care for non-operative management of
or anorexia, with the presence of “currant jelly” stools often uncomplicated intussusception, with a success rate of
a late or unnoticed sign of intussusception. 80–95%. Enema reduction should be avoided in patients
with peritonitis, bowel obstruction, or bowel perforation.
What is the preferred method of radiologic Successful enema reduction is performed by radiology
imaging used to diagnose intussusception and using instillation of air, saline or water-soluble contrast
what other methods of imaging may be per rectum in an attempt to increase intraluminal bowel
considered? pressure (Gluckman et al., 2017). There are data suggesting
that air enema reduction is more beneficial than contrast
Abdominal ultrasound is currently the standard of care for
enema reduction due to speed, safety and lower perforation
diagnosis of intussusception. The presence of intussuscep-
rates when compared with liquid enema reduction.
tion on ultrasound may be described as a “bull’s eye” or
“target” sign demonstrating the affected area of telescoped
What symptoms and risk factors are associated
bowel (see Figure 78.1). Additional methods of imaging
with recurrent intussusception after enema
include abdominal radiography or computed tomography
reduction?
(CT) (Smith and Fox, 2016). The presence of a “crescent”
sign on abdominal radiograph may be indicative of intus- After successful enema reduction, the child will be
susception, although the sensitivity for accurate diagnosis admitted to the hospital for observation or monitored in
of intussusception is only 62–86% (Gluckman et al., the ED for evidence of recurrent intussusception prior to
2017). CT scan may be indicated with concern for bowel discharge. Despite successful enema reduction, the pres-
obstruction, perforation, or in cases of recurrent intussus- ence of abdominal pain, vomiting, hematochezia, or diet
ception, but CT imaging poses risk of radiation exposure intolerance may indicate recurrence and require repeat
and should not be routinely utilized. imaging or enema reduction (Smith and Fox, 2016). The
Downloaded from https://onlinelibrary.wiley.com/doi/ by National Institute Of Standard, Wiley Online Library on [06/03/2024]. See the Terms and Conditions (https://onlinelibrary.wiley.com/terms-and-conditions) on Wiley Online Library for rules of use; OA articles are governed by the applicable Creative Commons License
334 Cases in Pediatric Acute Care

recurrence rate after successful enema reduction is approx- lead point, acute peritonitis, or bowel perforation. In
imately 5–10% at 24 hours, but decreasing by 48 hours. Risk uncomplicated cases of intussusception. surgical or manual
factors include children over 2 years of age, duration of reduction is performed by a minimally invasive or laparo-
symptoms greater than 48 hours, rectal bleeding, or the scopic approach. The presence of bowel necrosis, perfora-
presence of a palpable mass or pathologic lead point (Xie tion, or a pathologic lead point may indicate the need for
et al., 2018). an exploratory laparoscopy or laparotomy with excision
of mass and possible bowel resection.
When is surgical intervention indicated for
intussusception?
Surgical intervention may be indicated with failed attempts
at enema reduction or with the presence of a pathologic

References
Gluckman, S., Karpelowsky, J., Webster, A., and McGee, R.G. Xie, X., Wu, Y., Wang, Q., Zhao, Y., and Xiang, B. (2018). Risk
(2017). Management of intussusception in children (review). factors for recurrence of Intussusception in pediatric
Cochrane Database of Systematic Reviews (6): CD006476. doi: patients: a retrospective study. Journal of Pediatric Surgery
10.1002/14651858.CD006476.pub3 53(11): 2307–2311. doi: 10.1016/j.jpedsurg.2018.03.023.
Smith, J. and Fox, S. (2016). Pediatric abdominal pain: an
emergency medicine perspective. Emergency Medical Clinics of
North America 34: 341–361. doi: 10.1016/j.emc.2015.12.010.

Further Reading
Mallicote, M., Isani, M., Roberts, A., et al. (2017). Hospital Wong, C., Jin, S., Chen, J., et al. (2016). Predictors for bowel
admission unnecessary for successful uncomplicated resection and the presence of a pathological lead point for
radiographic reduction of pediatric intussusception. operated childhood intussusception: a multi-center study.
American Journal of Surgery 214(6): 1203–1207. doi: Journal of Pediatric Surgery 51(12): 1998–2000.
10.1016/j.amjsurg.2017.08.040.
Downloaded from https://onlinelibrary.wiley.com/doi/ by National Institute Of Standard, Wiley Online Library on [06/03/2024]. See the Terms and Conditions (https://onlinelibrary.wiley.com/terms-and-conditions) on Wiley Online Library for rules of use; OA articles are governed by the applicable Creative Commons License
335

79

Sore Throat and Shortness of Breath


Amy Manion
Rush University College of Nursing, Chicago, IL, USA

An 11-year-old, well-nourished boy presents to the emer- He was discharged home at 1 week of age weighing 2268 g.
gency department (ED) with sore throat and shortness of He is up to date with all immunizations including his
breath. annual influenza vaccine. His weight has been above the
99th percentile for gender and age since age 6. Currently
he weighs 68.18 kg and his height is 145 cm. His body mass
History of Present Illness index (BMI) is 32.5, which places him in the 99th percentile
for his age and gender. He has a history of nocturnal enu-
This 11-year-old boy became ill 3 days ago when he woke resis and takes desmopressin 0.4 mg 1 hour before bedtime.
up with sore throat and fever of 38.3 C (101 F). His mother He is followed by a pediatric urologist. His renal ultrasound
brought him to see his primary care provider on his first day is normal. No history of urinary reflux or urinary tract
of illness. A rapid group A Streptococcus (GAS) test was infections. He was diagnosed with attention deficit/hyper-
positive in the office. He was diagnosed with GAS pharyn- activity disorder (ADHD) 2 years ago by his primary care
gitis and started on a 10-day course of 800 mg amoxicillin provider. He takes methylphenidate extended-release
(400 mg/5 mL) orally twice daily. He has not had a fever 18 mg tablets once daily. No history of asthma. No prior
since starting the antibiotics. On day 3 of illness, he was hospitalizations.
sleeping for approximately 2 hours when he awoke sud-
denly and started gasping for air and coughing, at which
time his mother decided to bring him to the emergency Past Surgical History
department (ED) for further evaluation.
His mother reports that he is often tired during the day He had tympanostomy tubes placed at 28 months of age for
and falls asleep at school. He has had prominent night-time frequent otitis media with effusion.
snoring for the past 3 years and often complains of head-
aches. Over the last three nights the snoring has gotten lou-
der. He was recently seen by otolaryngology (ENT) and Family History
diagnosed with enlarged tonsils and adenoids. He is sched-
uled for polysomnography next week. His father has type 2 diabetes. His paternal grandfather also
has type 2 diabetes and had a stroke at 55 years of age. He
does not have any siblings. Both of his parents are obese.
Past Medical History

Born at 38 weeks. He was small for gestational age (SGA) Current Status
and weighed 2041 g. He spent 7 days in the neonatal inten-
sive care unit (NICU) because of SGA concerns and tachyp- On examination in the ED, he has moderate inspiratory
nea. He required oxygen support by nasal cannula for stridor while awake with an oxygen saturation of 96% on
2 days and then was weaned off oxygen by day 3 of life. room air. His respiratory rate is 22 breaths per minute with

Cases in Pediatric Acute Care: Strengthening Clinical Decision Making, First Edition. Edited by Andrea M. Kline-Tilford and Catherine M. Haut.
© 2020 John Wiley & Sons Ltd. Published 2020 by John Wiley & Sons Ltd.
Downloaded from https://onlinelibrary.wiley.com/doi/ by National Institute Of Standard, Wiley Online Library on [06/03/2024]. See the Terms and Conditions (https://onlinelibrary.wiley.com/terms-and-conditions) on Wiley Online Library for rules of use; OA articles are governed by the applicable Creative Commons License
336 Cases in Pediatric Acute Care

moderate work of breathing. His lungs are clear, but with Table 79.1 Complete blood count.
referred upper airway noise to auscultation. Mouth and
White blood cell count 8000/mm3
throat examination reveal enlarged tonsils, grade 3+ with
erythema. No head or neck lymphadenopathy is noted. Hemoglobin 13 g/dL
Hematocrit 39%
Platelet count 310 000/mm3

Question

Answer the following question using the details provided. oximetry. After completion of the nebulized racemic epi-
nephrine, he is resting comfortably with no audible stridor
1 Based on the data given, what are the differential diag-
with 98% oxygen saturation on room air.
noses and the most likely diagnosis for this child?
After 2 hours of observation in the ED, he develops mod-
erately audible stridor and his oxygen saturation decreases
to 92% on room air. He is given a second dose of nebulized
Rationale and Evidence-based Practice racemic epinephrine and started on 30% humidified oxygen
Explanation by facemask. On humidified oxygen, his oxygen saturation
quickly increases to 98%. A chest radiograph is obtained
Based on the data given, what are the differential and is normal. A complete blood count is obtained and
diagnoses and the most likely diagnosis for the results are shown in Table 79.1.
this child? He is admitted to the pediatric general care floor for fur-
ther monitoring and management due to his continued
Differential diagnoses for this child include obstructive
need for humidified oxygen support, risk for rebound effect
sleep apnea, GAS pharyngitis, viral infection, foreign body
after racemic epinephrine administration, and history of
aspiration, allergic reaction, asthma, epiglottitis, peritonsil-
enlarged adenoids. His oxygen saturation is 100% on
lar abscess, retropharyngeal abscess, and panic attack.
oxygen 5 L/min by facemask rebreather. When he falls
On further evaluation, the patient has normal complete
asleep his oxygen saturation decreases to 90% and he is
blood count results and lack of fever, moving epiglottitis,
noted to be snoring loudly, with increased respiratory effort
peritonsillar abscess, and retropharyngeal abscess lower
and respiratory rate of 20 breaths per minute. The amoxi-
on the differential diagnosis. The recent acute episode of
cillin treatment is continued to complete the 10-day course.
streptococcal pharyngitis is being adequately treated with
the amoxicillin, but may be associated with residual tonsil-
lar edema. Panic attacks caused by stress and anxiety can
occur in the school-aged child. Symptoms seen with panic Additional Questions and
attacks such as shortness of breath and hyperventilation Evidence-based Rationale
can present acutely. Although this child awoke suddenly
with shortness of breath and gasping for air, his symptoms 2 What symptoms does this patient have that would sug-
did improve with racemic epinephrine, which is used to gest obstructive sleep apnea (OSA) as a comorbidity?
treat upper airway edema. A diagnosis of asthma exacerba- 3 Would you obtain any other diagnostic studies?
tion and/or allergic reaction is less likely due to normal 4 What birth defects or genetic disorders are linked
chest radiograph and lack of expiratory wheezing, forced to OSA?
exhalation pattern, hives, or facial swelling. 5 What medical therapy would you recommend for this
The most likely diagnosis for this child is OSA exacer- patient?
bated by streptococcal pharyngitis infection. This child is 6 What surgical therapy would be beneficial for this
at risk for OSA due to his history of enlarged adenoids patient after resolution of his infectious symptoms?
and tonsils and obesity (BMI >95th percentile), further
exacerbated by an infection.
What symptoms does this patient have that would
Continuation of case: He is treated with nebulized race-
suggest OSA as a comorbidity?
mic epinephrine 2.25% solution, 0.5 mL diluted in 3 mL
normal saline (0.05 mL/kg per dose; maximum 0.5 mL). Obesity can contribute to OSA due to the excess soft tissue
In addition, he receives oral dexamethasone 12 mg (0.6 in the neck area, which obstructs the airway when the
mg/kg; maximum 12 mg). He is placed on continuous pulse patient’s muscles relax during sleep. This child is in the
Downloaded from https://onlinelibrary.wiley.com/doi/ by National Institute Of Standard, Wiley Online Library on [06/03/2024]. See the Terms and Conditions (https://onlinelibrary.wiley.com/terms-and-conditions) on Wiley Online Library for rules of use; OA articles are governed by the applicable Creative Commons License
Chapter 79 Sore Throat and Shortness of Breath 337

99th percentile for BMI which puts him at risk for OSA.
Furthermore, he has been diagnosed by ENT with enlarged
tonsils and adenoids. Enlarged tonsils and adenoids can
occlude the airway during sleep and are a major contributor
to OSA in children. Symptoms of headache, hyperactivity
and nocturnal enuresis are often seen in children with
OSA. Headaches can be caused by short episodes of hypoxia
that frequently occur during the night due to temporary air-
way obstruction and apnea. This chronic lack of oxygen
during the night causes restless sleep and the child wakes
up tired despite having “slept” a full night. During the
day, the child is tired which can cause them to be irritable
and/or have difficulty concentrating. This chronic lack of
restful sleep can cause a patient with OSA to be diagnosed
with ADHD. The ADHD symptoms will often resolve once
the OSA is treated. Nocturnal enuresis is another comorbid-
ity associated with OSA that will resolve once the OSA is
properly treated.

Would you obtain any other diagnostic studies?


No further testing is warranted at this time due to acute ill-
ness. Although a physical examination is important for
diagnosis, large tonsils and adenoids do not directly indi-
cate the presence of OSA. Nocturnal polysomnography
(PSG) should be completed as scheduled on discharge in
order to confirm suspected diagnosis of OSA. Nocturnal Figure 79.1 Patient undergoing polysomnography.
PSG (sleep study) is the gold standard for identification
of OSA in children (see Figure 79.1). The PSG diagnostic
criteria for OSA differ for adults versus children. The breathing so the patient can have a more normal sleep cycle
PSG uses the apnea–hypopnea index to measures the num- and wake up less tired. CPAP delivers positive pressure
ber of apneas or hypopneas per hour and this determines throughout the entire respiratory cycle, whereas BiPAP
OSA severity: mild, more than one but less than five events delivers more positive pressure during inspiration than
per hour; moderate, more than five events per hour; and expiration. Additionally, positive-pressure ventilation
severe, more than 10 events per hour. reduces the episodes of hypopnea and hypoxia, and this
reduces the risk of right heart failure. For children with
What birth defects or genetic disorders are linked severe OSA, supplemental oxygen may also be necessary.
to OSA? For treatment of mild OSA, intranasal corticosteroids are
Children with Pierre Robin syndrome are at higher risk for an option to decrease inflammation in the airway especially
OSA due to micrognathia (small lower jaw) which causes a in children with seasonal allergies
crowded tongue. Children with Down syndrome are also at
increased risk of OSA due to their associated enlarged
tongue, adenoids and tonsils and decreased muscle tone. What surgical therapy would be beneficial for this
patient once his infectious symptoms resolve?
What medical therapy would you recommend for
For adenotonsillar hypertrophy, an adenotonsillectomy or
this patient?
adenoidectomy is considered first-line treatment for chil-
Positive airway pressure is a non-surgical option for man- dren with moderate to severe OSA. PSG should be repeated
agement of OSA in children. Both continuous positive air- if the patient is having persistent symptoms postopera-
way pressure (CPAP) and bilevel positive airway pressure tively. CPAP can be continued for persistent OSA despite
(BiPAP) are the types of therapy often used during the night adenotonsillectomy intervention. Enuresis symptoms will
to prevent airway obstruction and decrease work of often improve or resolve after surgical intervention.
Downloaded from https://onlinelibrary.wiley.com/doi/ by National Institute Of Standard, Wiley Online Library on [06/03/2024]. See the Terms and Conditions (https://onlinelibrary.wiley.com/terms-and-conditions) on Wiley Online Library for rules of use; OA articles are governed by the applicable Creative Commons License
338 Cases in Pediatric Acute Care

Further Reading
Ahn, Y.M. (2010). Treatment of obstructive sleep apnea in Barone, J.B., Hanson, C., DaJusta, D.G., et al. (2009). Nocturnal
children. Korean Journal of Pediatrics 53(10): 872–879. doi: enuresis and overweight are associated with obstructive
10.3345/kjp.2010.53.10.872. sleep apnea. Pediatrics 124(1): e53–e59. doi: 10.1542/
Alexopoulos, E., Malakasioti, G., Varlami, V., et al. (2014). peds.2008-2805
Nocturnal enuresis is associated with moderate-to-severe Marcus, C.L., Brooks, L.J., Ward, S.D., et al. (2012). Diagnosis
obstructive sleep apnea in children with snoring. Pediatric and management of childhood obstructive sleep apnea
Research 76(6): 555–559. syndrome. Pediatrics 130(3): e714–e755.
Alsubie, H.S. and BaHammam, A.S. (2017). Obstructive sleep
apnoea: children are not little adults. Paediatric Respiratory
Reviews 21: 72–79. doi: 10.1016/j.prrv.2016.02.003.
Downloaded from https://onlinelibrary.wiley.com/doi/ by National Institute Of Standard, Wiley Online Library on [06/03/2024]. See the Terms and Conditions (https://onlinelibrary.wiley.com/terms-and-conditions) on Wiley Online Library for rules of use; OA articles are governed by the applicable Creative Commons License
339

80

Oral Presentation and Communication


Jill Siegrist Thomas
University of Maryland Children’s Hospital, University of Maryland Medical Center, Baltimore, MD, USA

Effective and comprehensive communication is an essen- Vaccines are up to date for age and he has no known aller-
tial skill of every healthcare provider for optimizing patient gies, and is not taking any prescribed or over-the-counter
safety and quality patient-centered care. This case presen- medications. No recent fever, illness or sick contacts. No
tation will follow the same patient from the inpatient unit, rhinorrhea or congestion.
to the operating room, and to the pediatric intensive care
unit (PICU) to highlight various types of oral presentations
that may be required. The case progression will focus on
key information to present/handoff to other medical provi- Past Surgical History
ders to prevent communication gaps potentially leading to
medication or other therapy errors and adverse events. None. Elective repair of his AVSD is scheduled to occur in
1 month.

Case Presentation

A 4-month-old male with trisomy 21 and atrioventricular Family History


septal defect (AVSD) is admitted to the inpatient pediatric
unit due to congestive heart failure. Parents and grandparents are reported to be healthy. There
is no familial history of congenital heart disease, trisomy 21,
or sudden or unexplained death.
History of Present Illness

The patient was doing well at home until 1 week prior to


admission when he developed tachypnea at rest with Current Status
decreased oral intake and slowed weight gain. His parents
report he sweats and takes frequent breaks during oral feed- Vital signs are as follows: weight 5.2 kg, afebrile, heart rate
ing with worsening of respiratory effort, but does not 112 bpm, tachypnea with respiratory rate 72 breaths per
exhibit cyanosis. He consumes approximately 57–113 g minute and mild subcostal retractions, blood pressure
(2–4 ounces) of breast milk by bottle every 4 hours. He is 74/48 mmHg, and oxygen saturation 77% on room air.
admitted from cardiology clinic for medical management He has crackles bilaterally upon auscultation. A grade II/VI
of congestive heart failure. holosystolic murmur is auscultated at the left lower sternal
border. Liver is palpable 2 cm below the right costal margin
and he is warm and well perfused. A peripheral intravenous
Past Medical History line has been placed and he receives a one-time intravenous
dose of furosemide 1 mg/kg and is ordered furosemide
Baby was born at term and discharged to home with his 1 mg/kg per dose daily PO and enalapril 0.1 mg/kg per dose
mother. His weight has tracked along the 25th percentile PO every 12 hours. A chest radiograph is significant for
on the trisomy 21 growth curve for gender and age. mild bilateral pulmonary edema.

Cases in Pediatric Acute Care: Strengthening Clinical Decision Making, First Edition. Edited by Andrea M. Kline-Tilford and Catherine M. Haut.
© 2020 John Wiley & Sons Ltd. Published 2020 by John Wiley & Sons Ltd.
Downloaded from https://onlinelibrary.wiley.com/doi/ by National Institute Of Standard, Wiley Online Library on [06/03/2024]. See the Terms and Conditions (https://onlinelibrary.wiley.com/terms-and-conditions) on Wiley Online Library for rules of use; OA articles are governed by the applicable Creative Commons License
340 Cases in Pediatric Acute Care

Questions •• Birth history


Past medical and surgical history
Answer the following questions using the details provided.
•• Allergies and medications
Diet
1 What are the key components of an oral presentation
for a new admission? •• Well child, developmental history and immunizations
Family history
2 Describe a comprehensive oral presentation for this
patient. •• Social history and determinants of health
Review of systems
Continuation of case: The same patient has now under- D) Vital statistics and physical examination: targeted phys-
gone elective surgery, and is being admitted to the PICU ical examination highlighting pertinent positive, nega-
from the operating room. tive or abnormal findings
3 What information is important to obtain during patient E) Laboratory and diagnostic data
handoff from the operating room to the PICU? F) Assessment statement: one-sentence summary that
4 What barriers impact ideal communication systems includes the patient’s age, sex, pertinent past medical
during patient handoff? history, active issues and trajectory of illness
G) Differential diagnosis list
Continuation of case: Consider the next four questions H) Plan: can be presented in a system or problem-based
and then continue to read to determine the answers and format.
view examples of the handoff processes.
5 How can a structured or standardized handoff tool
improve communication and patient care? Describe a comprehensive oral presentation for
6 How do shift-to-shift handoffs differ from oral presenta- this patient
tions during multidisciplinary rounds?
W.D. is a 4-month-old male with trisomy 21 and AVSD pre-
7 What are the key components of a handoff
senting with tachypnea and poor weight gain.
communication?
8 How do oral presentations during multidisciplinary
rounds differ from the presentation of a newly admitted History of present illness/pertinent past medical history
patient? He was born full term, is bottle fed, and has been doing well
at home with fair weight gain. One week prior to admission
he developed tachypnea at rest with decreased oral intake
and slowed weight gain. His parents report he sweats, takes
Rationale and Evidence-based Practice frequent breaks, during oral feeding with worsening respi-
Explanation ratory effort. His oral intake has recently decreased, cur-
rently consuming 2–4 ounces of breast milk by bottle
Oral presentation of a new admission closely follows the every 4 hours. No concerns of cyanosis. He was admitted
written documentation of the history and physical exami- from cardiology clinic due to concern for congestive heart
nation and note performed on admission. Organization of failure and optimization of medication management. He
the data strives to set the context for the listener, presenting takes no medications and is scheduled to have a complete
the information in a concise, logical and story-like fashion. repair of his AVSD in 1 month. Of note, W.D. has had no
This formatting facilitates processing of information and recent fever, illness, sick contacts, congestion or rhinor-
ultimately leads listeners to derive similar diagnostic rhea. Prior to this admission he was growing well, trending
conclusions. in the 25% percentile on his adjusted growth curve.

What are the key components of an oral


Vital signs/physical examination
presentation for a new admission?
Upon admission, his vital signs were significant for tachyp-
Standard framework for oral presentation for a new nea to the 70s in room air and oxygen saturation of 77%.
admission On examination he has crackles bilaterally. He has a
A) Chief complaint and identifying information grade II/VI holosystolic murmur. He is warm and well
B) History of present illness perfused. His liver is palpable 2 cm below the right costal
C) Pertinent past medical history (pertinent positive and margin. Chest X-ray is significant for mild bilateral
negative information only) pulmonary edema.
Downloaded from https://onlinelibrary.wiley.com/doi/ by National Institute Of Standard, Wiley Online Library on [06/03/2024]. See the Terms and Conditions (https://onlinelibrary.wiley.com/terms-and-conditions) on Wiley Online Library for rules of use; OA articles are governed by the applicable Creative Commons License
Chapter 80 Oral Presentation and Communication 341

Assessment statement What information is important to obtain during


W.D. is a 4-month-old male with trisomy 21 and AVSD patient handoff from the operating room to
admitted with respiratory insufficiency, feeding difficulties the PICU?
and poor weight gain due to congestive heart failure.
Multidisciplinary handoff includes the cardiac surgeon,
anesthesiologist, PICU attending, advanced practice provider
Differential diagnosis (APP), respiratory therapist, and nurse at patient bedside.
The differential diagnosis list for this acute presentation Institutional factors and use of a structured handoff tool
includes congestive heart failure, viral respiratory illness will dictate the content of information included during
and anemia. He has most likely developed congestive heart patient handoff from the operating room. The content
failure due to his known structural heart disease. Fatigue needs to include information from the sender (surgical
and associated feeding difficulties are likely due to insuffi- team representative and anesthesiologist) to the ICU team
cient cardiac output to meet his higher metabolic demands needed to provide safe, complete and continuous care.
while feeding. A viral infectious etiology for his tachypnea Key information includes the following (Chatterjee
and feeding difficulties is less likely since he is afebrile, and et al., 2019).
has no sick contacts, rhinorrhea or congestion. Given his
history, anemia is also a less likely cause of congestive heart 1) Patient characteristics
failure in W.D. a) Name, medical record number or date of birth
b) Age, weight, and sex
c) Preoperative diagnoses
Management plan by system
Cardiovascular: Administer a single dosage of IV furose- 2) Surgical sign-out
mide 1 mg/kg due to his tachypnea and chest X-ray findings a) Operative procedure performed
of pulmonary edema. Will initiate furosemide 1 mg/kg per b) Surgical findings and intraoperative complications
dose daily PO and enalapril 0.1 mg/kg per dose PO every c) Cardiopulmonary bypass, aortic cross clamp, and cir-
12 hours. W.D. is scheduled for AVSD repair in 1 month. culatory arrest times as applicable
I will notify surgery of his admission and discuss further d) Bleeding, rhythm or bypass separation issues
management with the cardiologist. e) Presence of drains or pacing wires
f) Special instructions, surgical concerns, and antici-
Respiratory: Monitor tachypnea. Initiate oxygen support
pated adverse events
and/or consider high-flow nasal cannula (HFNC) if respira-
tory insufficiency worsens. Goal oxygen saturation is >80%. 3) Anesthesia sign-out
Fluids/electrolytes/nutrition: Continue bottle feeds of a) Pertinent past medical history, preoperative medica-
breast milk and monitor for increased work of breathing tions, and allergies
with feeds. Send a basic metabolic panel to evaluate for b) Ease of laryngoscopy and airway issues
electrolyte anomalies. c) Ventilator and oxygenation issues with current ven-
Hematology: Send a complete blood count (CBC) to eval- tilator settings
uate for anemia. d) Administration of fluid and blood products
e) Medications administered: includes vasopressors,
Infectious disease: He has no signs or symptoms of an
inotropes, analgesics, sedation, muscle relaxants
infectious etiology so I will not send a respiratory viral
and antibiotics
panel at this time.
f) Recent laboratory data prior to ICU arrival (hemoglo-
Social: I have updated W.D.’s family on his overall status bin and ABG)
and plan of care. g) Desired hemodynamic goals
In addition to using a standard framework for oral pre- h) Special instructions, concerns or anticipated con-
sentations, the use of a semi-structured rounding tool cerns with extubation.
and daily goal sheets can facilitate communication and
exchange of information between providers, patients and
What barriers impact ideal communication
families during multidisciplinary rounds and ensure team
systems during patient handoff?
member awareness of goals of care (Ganesan et al., 2017).
Consistent utilization of a safety checklist is associated with Handoff communication is an integral high-risk compo-
decreased invasive device usage and patient care costs nent of patient care associated with an increased risk of
(Tarrago et al., 2014). clinical instability and sentinel events (Ong and Coiera,
Downloaded from https://onlinelibrary.wiley.com/doi/ by National Institute Of Standard, Wiley Online Library on [06/03/2024]. See the Terms and Conditions (https://onlinelibrary.wiley.com/terms-and-conditions) on Wiley Online Library for rules of use; OA articles are governed by the applicable Creative Commons License
342 Cases in Pediatric Acute Care

2011). Failure to communicate or receive the important Respiratory: Remains intubated and mechanically venti-
information is multifactorial in nature. Disorganization lated on pressure regulated volume control. ABG 7.41/38/
and miscommunication of information occurs due to 194/24/1/98% on 40% FiO2. Chest X-ray shows mild pulmo-
informal or unstructured sign-out processes and inconsist- nary edema without presence of pleural effusions. The
ent, incomplete or inaccurate transfer of information endotracheal tube and all lines are in good position.
between service lines. Human factors, including lack of
Neurology: Pain is well controlled with scheduled aceta-
team focus, parallel conversations and number of indivi-
minophen and fentanyl PRNs for moderate pain. He is
duals present during transfer of care, also contribute to
sedated with dexmedetomidine at 1 μg/kg per hour.
information miscommunication (Vergales et al., 2015;
Chatterjee et al., 2019). Infectious disease: Receiving cefazolin for postoperative
prophylaxis.
Fluids/electrolytes/nutrition: Receiving D5 ½NS at 75%
How can a structured or standardized handoff tool maintenance. Electrolytes have been stable. Last electrolyte
improve communication and patient care?
panel is sodium 146, potassium 3.9, chloride 111, bicarbo-
Effective handoff processes enhance interdisciplinary nate 23, BUN 8, Creatinine 0.18, glucose 195, calcium
team communication and patient safety. Utilization of for- 8.7, ionized calcium 1.31, magnesium 2.1 and phosphorus
malized handoff processes such as standardized proce- 4.9. No electrolyte supplementation given.
dures, scripts and checklists can improve transmission
Hematology: After cardiopulmonary bypass he received
of key data (Dixon et al., 2015). Use of a dedicated hand-
packed red blood cells (PRBCs), cell saver, platelets and
over process during transfer of care from the operating
fresh frozen plasma. His hemoglobin and hematocrit are
room to the PICU after cardiothoracic surgery is also
11.9 and 35. Platelet count 142. PT 18.4, INR 1.4, PTT 38,
associated with improved handover completeness, few
and fibrinogen 210. He has not required any blood products
early postoperative events, improved compliances with
since admission to the PICU. He has three chest tubes
process measures, and provider satisfaction (Chatterjee
draining serosanguineous fluid and output is approxi-
et al., 2019).
mately 1 mL/kg per hour.
The following is an example of a PICU sign-out presen-
tation of a new postoperative patient. Social: Parents are at bedside and updated on plan of care.
W.D. is a 5-month-old male with trisomy 21, AVSD and Overnight tasks: Monitor for dysrhythmias and bleeding.
medically controlled heart failure who is admitted follow- Initiate diuresis with intermittent furosemide and chloro-
ing single patch AVSD repair with bovine pericardium. thiazide. Goal extubation is tomorrow morning – wean
Cardiovascular: There were no intraoperative complica- the ventilator, attempt a pressure support trial and check
tions. His cardiopulmonary bypass time was 132 minutes for an airway leak overnight. Lab schedule is ABGs and
and he had a cross clamp time of 56 minutes. Before coming electrolyte panel every 6 hours, CBC and coagulation stud-
off cardiopulmonary bypass the patient underwent modi- ies every 12 hours. He is ordered for a CXR in the morning.
fied ultrafiltration for 100 mL. He arrived to PICU on epi-
nephrine and milrinone infusions. Since admission he
How do shift-to-shift handoffs differ from oral
developed junctional ectopic tachycardia (JET) which
presentations during multidisciplinary rounds?
resolved with controlled normothermia, sedation and dis-
continued inotropic support. He is now in normal sinus Communication during a shift-to-shift handoff is brief and
rhythm with a heart rate of approximately 120 bpm. Blood concise, reporting only the necessary information needed to
pressures are 75/48 to 95/55 mmHg and renal near-infrared assume care for the patient. Length of presentation will
spectroscopy (NIRS) are in the 60s. Examination is signifi- vary based on the complexity of the patient. Structured
cant for a grade II/VI mid-frequency systolic murmur at the handoff communication tools assist in ensuring necessary
left lower sternal border. No gallop or rub. He is warm and information is both given and received by healthcare
well perfused on a milrinone infusion of 0.5 μg/kg per min. providers.
Lines include a right internal jugular central venous line, a
right radial arterial line, a left hand peripheral IV, and atrial
What are the key components of a handoff
and ventricular pacing wires. The lactate is 1.1. Postopera-
communication?
tive transesophageal echocardiogram shows trivial residual
ventricular septal defect with left-to-right ventricular Patient handoffs are high-risk times associated with
shunt. No residual atrial septal defect. Normal biventricular adverse and sentinel events. Use of the I-PASS Handoff Pro-
function. gram has been shown to facilitate efficient transition of
Downloaded from https://onlinelibrary.wiley.com/doi/ by National Institute Of Standard, Wiley Online Library on [06/03/2024]. See the Terms and Conditions (https://onlinelibrary.wiley.com/terms-and-conditions) on Wiley Online Library for rules of use; OA articles are governed by the applicable Creative Commons License
Chapter 80 Oral Presentation and Communication 343

Table 80.1 IPASS format example.

I: Illness W.D. is a critically ill infant s/p cardiothoracic surgery. He is a full code
severity
P: Patient W.D. is a 4-month-old male with trisomy 21, AVSD, and medically controlled heart failure who was admitted
summary today following single-patch AVSD repair with bovine pericardium. There were no intraoperative complications
or surgical concerns. He remains intubated and sedated with adequate hemodynamics. After returning from the
OR he had JET which resolved with controlled normothermia, sedation and discontinuation of inotropic support
Pertinent review by systems:
CV: He is now in normal sinus rhythm after controlling his JET. His vital signs are stable with renal NIRS in the
60s. He is warm and well perfused on a milrinone infusion of 0.5 μg/kg per min. He has atrial and ventricular
pacing wires
RESP: He is mechanically ventilated without issues
FEN/GIGU: He is NPO on intravenous fluids. He has no electrolyte abnormalities
ID: He is receiving cefazolin for postoperative prophylaxis
HEME: His hemoglobin and hematocrit are 11.9 and 35.2. His chest tube output is approximately 1 mL/kg per hour
NEURO: He is receiving dexmedetomidine at 1 μg/kg per hour. His pain is well controlled with scheduled
acetaminophen and PRN fentanyl
A: Action Continue to assess for hemodynamic instability, dysrhythmias, and bleeding. Initiate diuresis with furosemide and
items chlorothiazide this evening if he is hemodynamically stable. Goal extubation is tomorrow morning – wean the
ventilator, attempt a pressure support trial and check for an airway leak overnight
S: Situation If he develops JET again, consider over-drive pacing. Please start dexamethasone to reduce airway swelling if he
awareness does not have a leak around the endotracheal tube
and
contingency
planning
S: Synthesis Okay, thanks. It sounds like he is doing well despite the JET. Overnight tasks will include monitoring for
by receiver dysrhythmia and bleeding, monitoring for hemodynamic stability, initiating diuresis with furosemide and
chlorothiazide and preparing W.D. for extubation in the morning. I don’t have any additional questions

AVSD, atrioventricular septal defect; OR, operating room; JET, junctional ectopic tachycardia; NIRS, near infrared spectroscopy.
Source: Data from Starmer et al. (2014).

information and decrease medical errors and adverse D) System or problem-based presentation of medications
events (Starmer et al., 2014). The cornerstone mnemonic with interpretation of laboratory and diagnostic data
for the key elements of the handoff process are I, illness E) Assessment statement: one-sentence summary which
severity; P, patient summary; A, action items; S, situation includes the patient’s age, sex, pertinent past medical
awareness and contingency planning; and S, synthesis by history, active issues and trajectory of illness
receiver. Table 80.1 presents the IPASS format for shift- F) Plan

••
to-shift handoff between W.D.’s providers.
Communicate overarching goals as well as daily plan
How do oral presentations during multidisciplinary Can be presented in a systems or problem-based
rounds differ from the presentation of a newly admitted format.
patient?
The following example includes an assessment and man-
Oral presentations during daily multidisciplinary rounds agement plan for postoperative day 1 (POD #1) presented in
concisely review the data from the previous 24 hours and multidisciplinary rounds.
do not contain the depth of information presented during
an oral presentation of a patient who was recently admitted. Assessment statement: W.D. is a 5 month old male with
The standard framework for oral presentation during trisomy 21 and AVSD now POD #1 s/p repair via a single
multidisciplinary rounds is as follows. patch technique with resolved JET and adequate hemody-
namics who is being prepared for extubation.
A) Introductory statement: one-sentence summary which
includes the patient’s age, sex, pertinent past medical Overnight events: W.D. remained intubated, sedated and
history, reason for admission and active diagnosis ventilated overnight. His cardiac output remained ade-
B) Pertinent overnight events and current active issues quate and he has had no further episodes of JET. He
C) Vital statistics and physical examination: targeted is warm and well perfused on a milrinone infusion of
physical examination highlighting pertinent positive, 0.5 μg/kg per hour. His chest tube output decreased to
negative or abnormal findings 0.5 mL/kg per hour and diuresis was initiated with
Downloaded from https://onlinelibrary.wiley.com/doi/ by National Institute Of Standard, Wiley Online Library on [06/03/2024]. See the Terms and Conditions (https://onlinelibrary.wiley.com/terms-and-conditions) on Wiley Online Library for rules of use; OA articles are governed by the applicable Creative Commons License
344 Cases in Pediatric Acute Care

intermittent furosemide and chlorothiazide. The ventila- promote chest tube drainage. Obtain a chest X-ray in the
tor has been weaned overnight and he has successfully morning to evaluate lung fields, chest tube placement and
completed a pressure support trial. for the presence of pleural or pericardial effusions.
Plan: Our major goals for the day will be to focus on extu- Fluids/electrolytes/nutrition: Continue IVF. May start
bation and diuresis: oral feeds of expressed breast milk 4–6 hours after extuba-
Cardiovascular: Continue milrinone at the current dose tion. Send a basic metabolic panel to evaluate for electrolyte
through extubation for afterload reduction. Continue furo- anomalies every 12 hours.
semide 1 mg/kg per dose every 6 hours and chlorothiazide Neurology: Discontinue dexmedetomidine infusion and
2.5 mg/kg every 6 hours for diuresis. His goal fluid balance PRN fentanyl now. Postoperative pain regimen is acetamin-
for the day is negative 100 mL. Monitor chest tube output. ophen 15 mg/kg PO every 6 hour and ketorolac 0.5 mg/kg
Monitor cardiac rhythm and cardiovascular endpoints per dose every 6 hours.
including vital sign trends, perfusion, NIRS trends, urine Hematology: Send a CBC daily to evaluate for anemia or
output and neurologic status. elevation in white blood cell count and coagulation studies
Respiratory: Extubate to 2 L of oxygen via nasal can- daily while the chest tubes are in place.
nula, with goal oxygen saturation of >92%. Obtain an Infectious disease: Continue cefazolin for 48 hours for
ABG 30 minutes after extubation. Include physical ther- postoperative prophylaxis.
apy to help patient out of bed to a chair or held by care- Social: W.D.’s parents remain at his bedside and updated
giver two to three times today for pulmonary toilet and to frequently.

References
Chatterjee, S., Shake, J.G., Arora, R.C., et al. (2019). Handoffs Joint Commission Journal on Quality and Patient Safety 37
from the operating room to the intensive care unit after (6): 274–284. doi: 10.1016/S1553-7250(11)37035-3.
cardiothoracic surgery: From the Society of Thoracic Starmer, A.J., Spector, N.D., Srivastava, R., et al. (2014).
Surgeons workforce on critical care. Annals of Thoracic Changes in medical errors after implementation of a handoff
Surgery 107(2): 619–630. doi: 10.1016/j. program. New England Journal of Medicine 371(19): 1803–
athoracsur.2018.11.010. 1812. doi: 10.1056/NEJMsa1405556.
Dixon, J.L., Stagg, H.W., Wehbe-Janek, H., et al. (2015). A Tarrago, R., Nowak, J.E., Leonard, C.S., and Payne, N.R.
standard handoff improves cardiac surgical patient transfer: (2014). Reductions in invasive devise use and care costs after
operating room to intensive care unit. Journal for Healthcare institution of a daily safety checklist in a pediatric critical
Quality 37(1): 22–32. doi: 10.1097/01. care unit. Joint Commission Journal on Quality and
JHQ.0000460123.91061.b3. Patient Safety 40(6): 270–278. doi: 10.1016/S1553-7250(14)
Ganesan, R., Rajakumar, P., Fogg, L., Silvestri, J., and Kane, J. 40036-9.
M. (2017). WE CARE 4 KIDS: use of a rounding tool in the Vergales, J., Addison, N., Vendittelli, A., et al. (2015). Face-to-
pediatric intensive care unit. Pediatric Quality and Safety 6 face handoff: improving transfer to the pediatric
(2): e044. doi: 10.1097/pq9.0000000000000044. intensive care unit after cardiac surgery. American Journal
Ong, M.S. and Coiera, E. (2011). A systematic review of failures of Medical Quality 30(2): 119–125. doi: 10.1177/
in handoff communication during intrahospital transfers. 1062860613518419.
Downloaded from https://onlinelibrary.wiley.com/doi/ by National Institute Of Standard, Wiley Online Library on [06/03/2024]. See the Terms and Conditions (https://onlinelibrary.wiley.com/terms-and-conditions) on Wiley Online Library for rules of use; OA articles are governed by the applicable Creative Commons License
345

81

A 7-Year-Old with Migratory Joint Pain


Christin Diller1,2
1
Cincinnati Children’s Hospital Medical Center, Cincinnati, OH, USA
2
Vanderbilt University College of Nursing, Nashville, TN, USA

A 7-year-old male presents to the emergency department pulse oximetry 89% on room air. On examination he is
(ED) with new-onset joint pain. awake, alert and oriented. He has coarse crackles through-
out all lung fields and is slightly tachypneic with mild
retractions. Normal S1 and S2 with a regurgitant systolic
History of Present Illness murmur and gallop. He has warm hands and feet with cap-
illary refill of 2 seconds and 2+ peripheral pulses. Liver
Mother reports that he has been complaining of migratory 2 cm below right costal margin. No abdominal tenderness
pain in knees, elbows, and ankles over last 2 days. She appreciated and active bowel sounds. He is placed on
reports he is a generally well and active child who is now oxygen 2 L/min via nasal cannula with improvement in
complaining of frequent pain, is more irritable, and is less oxygen saturations to 93%. During examination, no com-
cooperative and today appeared to be short of breath when plaints of joint pain. A chest radiograph, complete blood
playing after school. No fever in last 24 hours, unsure if any count, and basic metabolic profile are ordered and the
fever in the last 7 days. results are shown in Tables 81.1 and 81.2. The chest radio-
graph shows evidence of pulmonary edema.

Past Medical History


Questions
No known drug allergies. No previous hospitalizations. No
previous surgeries. Up to date on immunizations. Has not Answer the following questions using the details provided.
visited primary care provider in past 6 months. Mother states
he did stay home from school approximately 2 weeks ago for 1 What are the differential diagnoses?
2 days with fever and sore throat, but recovered quickly. 2 What additional laboratory studies would be useful in
making a diagnosis?
3 What additional studies would you order?
Family History

No family history of heart disease, cancer, or unexpected


death. Maternal grandmother has type 2 diabetes.
Rationale and Evidence-based Practice
Explanation

What are the differential diagnoses?


Current Status
Differential diagnoses for this patient would include myo-
On arrival to ED, vital signs are noted to be temperature carditis, endocarditis, and rheumatic heart disease based on
38.3 C (100.9 F) orally, heart rate 132 bpm, respiratory rate history and physical examination findings of gallop, tachy-
38 breaths per minute, blood pressure 110/75 mmHg, and cardia, crackles, and new-onset murmur, which suggest a

Cases in Pediatric Acute Care: Strengthening Clinical Decision Making, First Edition. Edited by Andrea M. Kline-Tilford and Catherine M. Haut.
© 2020 John Wiley & Sons Ltd. Published 2020 by John Wiley & Sons Ltd.
Downloaded from https://onlinelibrary.wiley.com/doi/ by National Institute Of Standard, Wiley Online Library on [06/03/2024]. See the Terms and Conditions (https://onlinelibrary.wiley.com/terms-and-conditions) on Wiley Online Library for rules of use; OA articles are governed by the applicable Creative Commons License
346 Cases in Pediatric Acute Care

Table 81.1 Basic metabolic profile. changes, vegetation, and ventricular dysfunction. Most
changes involve the mitral valve.
Sodium 139 mEq/L
Potassium 3.9 mEq/L Continuation of case: Patient is transferred to the pediat-
ric intensive care unit (PICU) for further monitoring. After
Chloride 101 mEql/L
transfer, he continues to have increased work of breathing
Carbon dioxide 24 mEq/L
with desaturations and was placed on bilevel positive air-
Blood urea nitrogen 25 mg/dL
way pressure (BiPAP) with improvement in oxygen satura-
Creatinine 0.9 mg/dL tions and work of breathing. An echocardiogram shows
Glucose 115 mg/dL moderate to severe mitral regurgitation with abnormal
coaptation of the mitral valve leaflets, and mild to moderate
aortic valve regurgitation. Left ventricular function is mod-
Table 81.2 Complete blood count. erately diminished while right ventricular function is nor-
mal. His EKG is significant for sinus tachycardia with
White blood cell count 18 000/mm3 prolonged PR interval.
Hemoglobin 9 g/dL
Hematocrit 28%
Platelets 105 000/mm3
Additional Questions and Evidence-
based Rationale
cardiac etiology. Differential diagnoses for the migrating
4 What is the most likely diagnosis?
joint pain include septic arthritis, Lyme disease, autoim-
5 What additional therapies would you initiate?
mune connective tissue disease, and leukemia/lymphoma.
Bacterial and viral etiologies should also be considered with
increased work of breathing and mild elevation in
temperature.
What is the most likely diagnosis?
What additional laboratory studies would be Rheumatic heart disease with carditis is the most likely
useful in making a diagnosis? diagnosis. Rheumatic fever is an inflammatory illness,
Anti-streptolysin titer can confirm the previous presence of likely with an autoimmune cause, that can occur after
a group A β-hemolytic Streptococcus infection. In addition, group A β-hemolytic streptococcal pharyngitis. Rheumatic
acute-phase reactants should be ordered and trended if ele- heart disease can be diagnosed using the Jones criteria.
vated. Generally, erythrocyte sedimentation rate (ESR) and Diagnosis is confirmed with presence of two major criteria
C-reactive protein are elevated in cases of rheumatic fever. or one major and two minor criteria (Table 81.3). Rheu-
Blood cultures should be evaluated for primary bacterial matic carditis does not appear in all rheumatic fever
infection and if positive can guide antimicrobial therapy. patients but if present presents with varying severity. This
Viral studies can be used to evaluate for viral infection carditis can cause permanent changes in the endocardium.
and liver function tests can be useful to evaluate for end- Early signs of carditis present as new-onset regurgitant
organ damage. A urinalysis is helpful to evaluate for strep- murmur and tachycardia progressing to signs and symp-
tococcal glomerulonephritis. toms of congestive heart failure.
To meet criteria for rheumatic heart disease, the patient
must show evidence of the presence of preceding group
What additional studies would you order?
A streptoccocal infection (e.g. positive throat culture for
An electrocardiograph (EKG) should be completed to look group A β-hemolytic streptococci or positive/rising ASO
for signs of cardiac strain, ST-segment changes, and other titer). The updated Jones criteria (Gewitz et al., 2015) iden-
arrhythmia or other cardiac irritability. A prolonged PR tified low-, medium- and high-risk populations. Low-risk
interval is one of the minor Jones criteria for diagnosis of populations are identified as those in which acute rheumatic
rheumatic heart disease. Echocardiogram will confirm fever occurs in two or less cases per 100 000 school-age chil-
the presence of intracardiac changes consistent with rheu- dren or an all-age rheumatic heart disease prevalence of
matic fever, including mitral valve and/or aortic valve ≤1 per 1000 population per year. Medium- to high-risk
Downloaded from https://onlinelibrary.wiley.com/doi/ by National Institute Of Standard, Wiley Online Library on [06/03/2024]. See the Terms and Conditions (https://onlinelibrary.wiley.com/terms-and-conditions) on Wiley Online Library for rules of use; OA articles are governed by the applicable Creative Commons License
Chapter 81 A 7-Year-Old with Migratory Joint Pain 347

Table 81.3 Jones criteria.a

Low risk High risk

Minor criteria Polyarthralgia ≥38.0 C (100.4 F)


Fever ≥38.5 C (101.3 F) Monoarthralgia
ESR >60 mm/h in the first hour and/or CRP >3 mg/dL Prolonged PR interval
Prolonged PR interval (for patient age) ESR >30 mm/h in the first hour and/or CRP >3 mg/dL
Major criteria Carditis (clinical or subclinical) Carditis (clinical or subclinical)
Polyarthritis Monoarthritis, polyarthritis, and/or polyarthralgia
Chorea Chorea
Erythema marginatum Erythema marginatum
Subcutaneous nodules Subcutaneous nodules
a
Must meet either two major criteria or one major and two minor criteria.
ESR, erythrocyte sedimentation rate; CRP, C-reactive protein.
Source: Gewitz et al. (2015). Reproduced with permission of Wolters Kluwer Health, Inc.

populations are those that do not meet the criteria for low of carditis is present, in the form of aspirin for mild cases
risk (Table 81.3). and prednisone or methylprednisolone for severe cases.
Anti-inflammatory medications will also improve the joint
pain associated with polyarthritis. Antibiotic therapy
What additional therapies would you initiate? should be initiated in the form of penicillin or, if allergy,
erythromycin to eradicate group A Streptococcus. After
Treatment of carditis should be managed with conven-
initial therapy, these patients will need continued
tional heart failure therapy (e.g. with diuretic and fluid
endocarditis prophylaxis on ongoing evaluation of cardiac
management) depending on severity of carditis. Bedrest
function. Household contacts should be tested for group
and restricted activity is generally advised, though there
A Streptococcus if symptomatic and treated for positive
is lack of recent data to support this practice. Initiation of
results.
anti-inflammatory medications is indicated when evidence

Reference
Gewitz, M., Baltimore, R., Tani, L., et al. (2015). Revision of the from the American Heart Association. Circulation 131:
Jones criteria for the diagnosis of acute rheumatic fever in 1806–1818.
the era of Doppler echocardiography: a scientific statement

Further Reading
Allen, H.D., Driscoll, D.J., Shaddy, R.E., and Feltes, T.F. (ed.) Reuter-Rice, K. and Bolick, B. (2012) Pediatric Acute Care:
(2013). Moss and Adams’ Heart Disease in Infants, Children, A Guide for Interprofessional Practice. Burlington, MA:
and Adolescents, 8th edn. Philadelphia, PA: Lippincott Jones & Bartlett Learning.
Williams & Wilkins.
Downloaded from https://onlinelibrary.wiley.com/doi/ by National Institute Of Standard, Wiley Online Library on [06/03/2024]. See the Terms and Conditions (https://onlinelibrary.wiley.com/terms-and-conditions) on Wiley Online Library for rules of use; OA articles are governed by the applicable Creative Commons License
349

82

Adolescent with Murmur and Chest Flutter


Natalie Hecht
C.S. Mott Children’s Hospital, Ann Arbor, MI, USA

An adolescent presents to a pediatric cardiology clinic after Current Status


referral from his primary care provider (PCP) for a murmur
and palpitations. Vital signs are as follows: heart rate 92 bpm, blood pressure
90/52 mmHg, respiratory rate 20 breaths per minute, and
oxygen saturation of 99% on room air.
He is an alert adolescent in no acute distress. Normal S1,
History of Present Illness
normally split S2. III/VI systolic ejection murmur at left
mid-sternal border. The point of maximal impulse was at
The adolescent was noted to have a new murmur on a well-
the left anterior axillary line. Clear breath sounds noted
child examination. He also complained that his heart was
bilaterally. Mild shortness of breath after walking from
“flipping around in my chest” a couple of times a day,
the waiting room to examination room. His abdomen
which has been going on for several months. When this
was soft and his liver border was palpable 2 cm below
happens, it lasts a few seconds, and is accompanied by dif-
the right costal margin.
ficulty breathing. The adolescent complained of shortness
of breath when climbing stairs and exercising, as well as
fatigue requiring daily naps after school. He denied
chest pain. Questions

Answer the following questions using the details provided.

Past Medical History 1 What are the differential diagnoses for this child?
2 Which diagnostic tests are indicated?
Frequent otitis media as an infant. Obesity. His immuniza-
tions are up to date.

Rationale and Evidence-based Practice


Explanation
Past Surgical History
What are the differential diagnoses for this child?
Tympanostomy tubes placed at 18 months of age.
The differential diagnoses include hypertrophic cardiomy-
opathy (HCM), dilated cardiomyopathy, restrictive cardio-
myopathy, arrhythmias, structural heart disease,
Family History mediastinal mass, and aortic stenosis.

Father has atrial fibrillation and hyperlipidemia. His


Which diagnostic tests are indicated?
mother has irritable bowel syndrome. His younger sister
has mild intermittent asthma. His mother’s cousin died Indicated diagnostic testing for this adolescent includes an
suddenly at age 30 of unknown cause. electrocardiogram (EKG) to evaluate for arrhythmias, an
Cases in Pediatric Acute Care: Strengthening Clinical Decision Making, First Edition. Edited by Andrea M. Kline-Tilford and Catherine M. Haut.
© 2020 John Wiley & Sons Ltd. Published 2020 by John Wiley & Sons Ltd.
Downloaded from https://onlinelibrary.wiley.com/doi/ by National Institute Of Standard, Wiley Online Library on [06/03/2024]. See the Terms and Conditions (https://onlinelibrary.wiley.com/terms-and-conditions) on Wiley Online Library for rules of use; OA articles are governed by the applicable Creative Commons License
350 Cases in Pediatric Acute Care

echocardiogram to evaluate heart structure and function, did not have a significant arrhythmia on EKG. These thera-
and a chest radiograph to evaluate lung fields and pies may be indicated in the future if his status deteriorates.
heart size. HCM has a strong genetic component. The patient under-
went cardiomyopathy genetic testing, which is indicated in
Continuation of case: The results of the diagnostic studies
all patients with HCM. If this screening is positive, all at-
are as follows. EKG showed ST–T changes, deep Q waves,
risk and first-degree relatives should have this testing per-
missing R waves, and left ventricular (LV) hypertrophy.
formed as well. If the patient’s testing is negative, all at-risk
Echocardiogram demonstrated moderate LV hypertrophy,
and first-degree relatives should undergo cardiology eva-
moderate to severe mid-ventricular and outflow LV
luations with an EKG and echocardiogram to evaluate
obstruction, moderate mitral regurgitation, and asymmet-
for HCM. Guidelines are needed to standardize use of AICD
ric septal hypertrophy of interventricular septum, with sep-
in pediatric patients. Indications for AICD placement are
tal wall thicker than posterior LV wall. The chest
increased risk of sudden death, including ventricular
radiograph showed lung fields clear bilaterally with prom-
arrhythmias, syncope, sudden death in a first-degree rela-
inent cardiomegaly.
tive, and/or massive hypertrophy. Based on his presenta-
tion, this patient does not qualify, though his outpatient
Holter results may prompt further intervention.
Additional Questions and Evidence- Treatment for HCM is dependent on severity of symp-
based Rationale toms and presence of LV outflow obstruction. Patients with
non-obstructive HCM may not require pharmacologic
3 What is the most likely diagnosis? treatment. Given his diagnosis of obstructive HCM and
4 Which therapies should be used to medically manage symptomatic presentation, this adolescent will likely bene-
this patient? fit from a beta-adrenergic blocking agent (e.g. labetalol).
5 What anticipatory guidance should you provide to the Beta-blockers are recommended for patients who have
patient? LV outflow tract obstruction or who are symptomatic with
angina or dyspnea. Beta-blockers decrease heart rate and
prolong diastole, thereby promoting more LV filling and
What is the most likely diagnosis? reducing LV outflow obstruction. Beta-blockers are also
HCM is the most likely diagnosis, given the classic echocar- beneficial due to their antiarrhythmic properties.
diogram results, characteristic EKG and murmur, and his- For patients who do not tolerate or respond to beta-
tory of shortness of breath with activity. Given the level of blockers, calcium channel blockers (CCBs) may be consid-
obstruction to LV outflow, HCM can be further classified as ered. Similar to beta-blockers, CCBs are negative inotropes
obstructive. HCM is characterized by a hypertrophied left and reduce heart rate. CCBs are thought to improve dias-
ventricle and decreased ventricular cavity with impaired tolic performance by decreasing myocardial ischemia.
ventricular filling. A thickened septal wall beyond two to However, CCBs should be used with caution, as a decrease
three times the standard deviation for age is clinically sig- in blood pressure may worsen outflow tract obstruction,
nificant for HCM. HCM is associated with symptomatic leading to pulmonary edema. Of note, verapamil has been
heart failure and is the most common cause of cardiac linked to increased risk of death in pediatric patients with
death in teenagers due to fatal arrhythmias. advanced LV hypertrophy.
Diuretics should be carefully considered. Patients with
HCM need adequate preload to allow for sufficient ventric-
Which therapies should be used to medically
ular filling. Thus, diuretics are generally contraindicated, as
manage this patient?
they may worsen symptoms. However, patients with con-
This adolescent was started on labetalol 20 mg three times gestive symptoms may benefit from careful diuretic usage.
daily. Pharmacologic therapy with beta-blockers or calcium Diuretics were not indicated in this patient since he did not
channel blockers helps to relax the hypertrophied myocar- demonstrate congestive symptoms.
dial tissue and reduces the heart rate to improve filling and
ejection time, thereby improving cardiac function. A Holter
What anticipatory guidance should you provide to
monitor was ordered to further evaluate for arrhythmia as
the patient?
an outpatient. He was instructed to follow up with pediatric
cardiology in 1 month. Hospital admission and automatic Strenuous exercise, such as basketball, sprinting, and
implantable cardioverter defibrillator (AICD) were not soccer, increases the risk of sudden cardiac death and is
indicated as his heart failure symptoms were mild and he not recommended for patients with HCM. However,
Downloaded from https://onlinelibrary.wiley.com/doi/ by National Institute Of Standard, Wiley Online Library on [06/03/2024]. See the Terms and Conditions (https://onlinelibrary.wiley.com/terms-and-conditions) on Wiley Online Library for rules of use; OA articles are governed by the applicable Creative Commons License
Chapter 82 Adolescent with Murmur and Chest Flutter 351

lower-intensity exercises, such as golf, walking, biking, and myectomy is typically recommended for patients who are
bowling, are likely safe for patients with HCM. Exercise refractory to oral medication therapy, have worsening
recommendations should be individualized for each mitral regurgitation, or a LV outflow gradient greater than
patient. 60 or 70 mmHg. A myectomy was not indicated for this
Patients with obstructive HCM may require a myectomy. patient based on his presentation, but he should have care-
This is a surgical reduction of the ventricular septum, ful outpatient follow-up with echocardiograms to identify
which decreases the LV outflow tract obstruction. Surgical any changes.

Further Reading
Gersh, B., Maron, B., Bonow, R., et al. (2011). 2011 ACCF/AHA Park, M.K. (2014). Park’s Pediatric Cardiology for Practitioners.
guideline for the diagnosis and treatment of hypertrophic Philadelphia, PA: Elsevier.
cardiomyopathy. Circulation 124: e783–e831. doi: 10.1161/
CIR.0b013e318223e2bd.
Lee, T., Hsu, D., Kantor, P., et al. (2017). Pediatric
cardiomyopathies. Circulation Research 121: 855–873. doi:
10.1161/CIRCRESAHA.116.309386.
Downloaded from https://onlinelibrary.wiley.com/doi/ by National Institute Of Standard, Wiley Online Library on [06/03/2024]. See the Terms and Conditions (https://onlinelibrary.wiley.com/terms-and-conditions) on Wiley Online Library for rules of use; OA articles are governed by the applicable Creative Commons License
353

83

Outpatient Management of Asthma


Jolene Dickerson
Nemours/Alfred I. duPont Hospital for Children, Wilmington, DE, USA

A 7-year-old boy with a history of mild persistent asthma occasions for asthma exacerbations. His past medical his-
diagnosed at age 6 presents to his primary care office with tory is significant for seasonal allergies, eczema, failure to
wheezing and acute respiratory distress. thrive, and autism.

History of Present Illness Past Surgical History

This 7-year-old boy became ill 2 days ago with cough and This child has not had any surgical procedures.
wheezing. He has been treated with albuterol via nebulizer
and/or metered-dose inhaler every 4 hours since the onset
of symptoms yesterday with no improvement. His last albu- Family History
terol treatment was given 3 hours ago. He had problems
with cough and wheezing in school, so was taken to the pri- The patient’s mother and maternal grandfather both have
mary care provider to be seen urgently. There is no history asthma. His mother also has a history of seasonal allergies.
of fever, no recent changes in the weather, nor does the No other family history of asthma, allergies, eczema, or
patient have cold symptoms. He has not had any recent lung disease.
environmental exposures such as cigarette smoke. There
are no known food or drug allergies, but he does have sea-
sonal environmental allergies and today he was playing
outside when his symptoms began. His appetite is also
Current Status
decreased.
On arrival at the primary care office, the patient is wheez-
ing throughout all lung fields and has decreased breath
sounds with poor air exchange. He is hypoxic, with an
Past Medical History oxygen saturation of 88–90% on room air. His initial respi-
ratory rate is 34 breaths per minute with severe intercostal,
This boy was born at term and went home with his mother. suprasternal, and supraclavicular retractions. He is sitting
He is up to date on his routine immunizations, but did not up on the examination table with forward posturing.
have his annual influenza vaccine this year. His weight is
less than the first percentile for his age and gender and
his current body mass index (BMI) is 11.5, which is also less
than the first percentile for age and gender. At age 6 years,
Questions
the patient was diagnosed with asthma after experiencing
Answer the following questions using the details provided.
recurrent episodes of wheezing, treated in outpatient care.
He has no history of hospitalizations for asthma, although 1 Based on the data given, what are the differential diag-
he has been to the emergency department (ED) on multiple noses and the most likely diagnosis for this child?

Cases in Pediatric Acute Care: Strengthening Clinical Decision Making, First Edition. Edited by Andrea M. Kline-Tilford and Catherine M. Haut.
© 2020 John Wiley & Sons Ltd. Published 2020 by John Wiley & Sons Ltd.
Downloaded from https://onlinelibrary.wiley.com/doi/ by National Institute Of Standard, Wiley Online Library on [06/03/2024]. See the Terms and Conditions (https://onlinelibrary.wiley.com/terms-and-conditions) on Wiley Online Library for rules of use; OA articles are governed by the applicable Creative Commons License
354 Cases in Pediatric Acute Care

2 What is the best way to increase this patient’s fatigued muscles while also preventing airway collapse
oxygenation? and supporting oxygenation.
3 What medications are recommended to be given in the
outpatient areas? What medications are recommended to be given in
4 What are the next steps in managing this case? How the outpatient areas?
would you provide a formal sign-out to the ED using
The pathogenesis of asthma involves underlying inflamma-
the SBAR format?
tion, which is variable and based on the individual patient
and the severity of disease. In an acute exacerbation,
bronchoconstriction occurs with exposure to a trigger or ill-
Rationale and Evidence-based Practice ness resulting in airway obstruction. Inhaled short-acting
Explanation beta-agonists (SABAs) such as albuterol and levalbuterol
combined with ipratropium bromide are first-line therapy
Based on the data given, what are the differential for an acute outpatient asthma exacerbation. SABAs have
diagnoses and the most likely diagnosis for a rapid onset of action (within 15 minutes), and because
this child? they are bronchodilators, airflow is improved by relieving
smooth muscle constriction and relaxing the airway. Ipra-
Differential diagnoses for this child include severe asthma tropium bromide (Atrovent) also provides bronchodilation
exacerbation associated with viral illness, exposure to aller- through smooth muscle relaxation in combination with
gens, exercise, or other inciting factors. Other potential albuterol (Link, 2014). These therapies should be adminis-
diagnoses would include foreign body aspiration and pneu- tered up to three times back-to-back or at 20–30 minute
monia. It would be unlikely that the boy had a viral illness intervals. Continuous albuterol therapy may be needed if
or pneumonia, as he has not had any other symptoms there is no significant improvement in the patient’s respira-
except for wheezing and cough and these symptoms were tory status. Because of the risk for side effects from SABAs
only present for 2 days. (hypertension, tachycardia, or tremors), patients must be
A severe asthma exacerbation is the most likely cause of continuously monitored in an acute care setting.
the child’s symptoms, related to the history and findings of Systemic corticosteroids decrease airway inflammation
difficulty breathing, wheezing, cough, and hypoxia. This in patients who experience acute asthma exacerbation.
child has also reported inciting factors of exercise and envi- Their onset of action is within 2–6 hours after administra-
ronmental allergy exposure that may have triggered the tion. It is best to administer systemic corticosteroids as soon
exacerbation (Link, 2014). as possible after arrival in the office or ED. Early adminis-
Moderate to severe asthma exacerbations are typically trig- tration has been shown to reduce hospital admission rates
gered acute episodes of respiratory distress, cough, and for asthma exacerbations. Prednisone or prednisolone are
wheezing in a patient with a history of asthma. This patient typically first-line steroid treatments, but there is limited
requires prompt treatment with beta-agonists, systemic cor- documentation of the benefits of using dexamethasone
ticosteroids, and oxygen. Close monitoring for clinical dete- due to its longer-lasting effect.
rioration and ongoing clinical assessments are important. In
the primary care office, initial therapies can be provided,
What are the next steps in managing this case?
especially if there will be a wait for emergency medical serv-
How would you provide a formal sign-out to the
ice (EMS) to arrive. However, without monitoring equip- ED using the SBAR format?
ment, it is important to observe and assess the patient
frequently until appropriate monitoring is available. This patient continues to be in respiratory distress and he
will therefore require immediate transport to the nearest
ED for continued support and additional intervention. The
What is the best way to increase this patient’s
local EMS should be notified to respond to the pediatric
oxygenation?
office, as opposed to asking family to drive him to the ED.
Oxygen therapy is recommended for children with asthma It is then important for the provider to notify the ED and pro-
who cannot maintain oxygenation above 92% on room air. vide a sign-out regarding the patient. SBAR (situation, back-
For patients who are spontaneously breathing, oxygenation ground, assessment, recommendation) is a standardized
often begins with delivery via a facemask or nasal cannula. method of providing comprehensive information using a
If the patient fails to respond to interventions and requires mnemonic to assist in remembering important information
additional oxygen support, non-invasive positive pressure to provide between healthcare providers in patient handoffs.
ventilation may be beneficial. This may provide relief for Failed communication has been identified as a primary
Downloaded from https://onlinelibrary.wiley.com/doi/ by National Institute Of Standard, Wiley Online Library on [06/03/2024]. See the Terms and Conditions (https://onlinelibrary.wiley.com/terms-and-conditions) on Wiley Online Library for rules of use; OA articles are governed by the applicable Creative Commons License
Chapter 83 Outpatient Management of Asthma 355

Table 83.1 Example of SBAR handoff.

Explanation of section Handoff information to relay

S Current status and situation This is the primary care office on ABC street. We are sending a child who is experiencing status
of the patient asthmaticus by EMS to the ED. He needs continued oxygen and potentially hospital admission.
This is a 7-year-old male who presented to our office today in status asthmaticus with oxygen
saturation of 88% on room air, requiring oxygen and continuing to wheeze despite receiving
albuterol nebs and dexamethasone. Vital signs are temperature 99, HR 136, RR 38, BP 92/46, oxygen
saturation is 94% on 15 L/min oxygen. Weight 19 kg, height 114 cm
B Background information to This boy was diagnosed with mild, intermittent asthma a year ago and has had two ED visits since
include the history of the then, but is currently only taking albuterol nebs PRN for wheezing. He began with a cough
present illness, allergies, and wheezing 2 days ago without any associated illness or fever. Wheezing worsened in school
current medications today and his mother brought him to us this afternoon. His other history includes a diagnosis
of failure to thrive and seasonal allergies. He has no known food or drug allergies and albuterol is
his only current medication
A Assessment of the patient, I think this child is in status asthmaticus and needs more aggressive treatment
with potential diagnoses
R Recommendation for care I do not think this child needs any diagnostic testing, unless he needs to be NPO for a long period
of time, develops fever or becomes more tachypneic, indicating the possibility of pneumonia

reason for sentinel and adverse events as patients are cared back-to-back treatments. Intravenous magnesium sulfate
for across healthcare settings. The Joint Commission and the (25–75 mg/kg, maximum 2.5 g administered over 20 min-
Agency for Healthcare Research and Quality, among other utes) has been shown to be beneficial in children with
organizations, recommend using a standardized approach severe asthma exacerbation and is considered for those
when communicating patient information. Using the SBAR who have failed traditional first-line therapies of albu-
approach, the handoff would result in a phone call to the ED terol and glucocorticoids. It is part of asthma pathways
in order to speak with a pediatric provider. See Table 83.1 for in the ED. It is thought that magnesium relaxes smooth
an example of this call in SBAR format. muscle by enhancing calcium uptake in the sarcoplasmic
reticulum and/or as a calcium antagonist, but the exact
Continuation of case: The boy is treated with oxygen via
mechanism of action is not known. Another benefit of
facemask, and administered oral dexamethasone 0.6 mg/kg
magnesium sulfate in asthma is that it reduces release
and a nebulized treatment of ipratropium bromide 250 μg
of histamine from mast cells and may act as an anticho-
and albuterol 2.5 mg per 3 mL. Oxygen is administered at
linergic agent. Advanced oxygenation therapy such as
10 L/min initially, but requires an increase to 15 L/min
non-invasive positive pressure ventilation may be used
to maintain his oxygen saturations above 96%. Because of
if the patient has poor response to other interventions
the level of hypoxia, respiratory distress, and oxygen
and remains hypoxic. Subcutaneous or intramuscular
requirement of this patient, staff calls 911 for EMS transport
epinephrine or terbutaline (systemic beta-agonists) may
to the nearest ED. Two additional doses of albuterol are
be better than inhaled therapies in children who have
administered at continuous intervals while awaiting the
poor inspiratory flow, are anxious or uncooperative, or
arrival of EMS.
who have poor response to initial inhaled therapies. It
is also important to assess hydration and, if needed,
Additional Questions and Evidence- administer intravenous fluid boluses and maintenance
fluid. Children with asthma can become dehydrated
based Rationale
quickly due to poor oral intake, insensible fluid losses,
and increased metabolic demand.
5 What other therapies would be initiated in the ED?
6 Should diagnostic studies be obtained for this patient in
the ED?
Should diagnostic studies be obtained for this
patient in the ED?
What other therapies would be initiated in the ED?
Arterial blood gases are not as useful in the child who is
Continuous albuterol therapy should be considered if breathing spontaneously without severe distress and are
the patient has limited response to the initial three painful to obtain, which could worsen respiratory distress.
Downloaded from https://onlinelibrary.wiley.com/doi/ by National Institute Of Standard, Wiley Online Library on [06/03/2024]. See the Terms and Conditions (https://onlinelibrary.wiley.com/terms-and-conditions) on Wiley Online Library for rules of use; OA articles are governed by the applicable Creative Commons License
356 Cases in Pediatric Acute Care

With rapid deterioration, a blood gas can assist in making a As the primary care provider for this child, it is important
decision for intubation and ventilation. Chest radiographs to stay in contact with the ED and inpatient unit if the child
only need to be considered with focal findings of crackles or is admitted. With this type of exacerbation and/or hospital-
decreased breath sounds, along with fever and/or severe ization, prevention of future acute illness is the goal. Since
disease. Chest radiographs can help to rule out pneumonia, this boy was not previously taking daily medications, the
atelectasis, air leak, and foreign body in certain severe cases National Institutes of Health Asthma Guidelines recom-
that do not respond to conventional therapies. Overall, mend starting a daily medication such as an inhaled steroid.
there is evidence supporting the view that radiographs The patient should also return to his primary care provider
and laboratory studies are not indicated if the diagnosis within 1–3 days following discharge. Monitoring in the office
is isolated as an asthma exacerbation without other con- on a regular basis is also recommended to monitor symp-
founding factors. toms and need for escalation in maintenance therapies.

Reference
Link, H. (2014). Pediatric asthma in a nutshell. Pediatrics in
Review 35(7): 287–298. doi: 10.1542/pir.35-7-287.

Further Reading
Anderson, W.C., Gleason, M., Miyazawa, N., and Szefler, S.J. improvement project. Journal of Emergency Nursing 45(2):
(2017). Approaching current and new drug therapies for 149–154.
pediatric asthma. Pediatric Clinics of North America 64(6): Schwarz, E.S. and Cohn, B.G. (2015). Is dexamethasone as
1197–1207. doi: 10.1016/j.pcl.2017.08.001. effective as prednisone or prednisolone in the management
Campbell, D. and Dontie, K. (2019) Implementing bedside of pediatric asthma. Annals of Emergency Medicine 65(1):
handoff in the emergency department: a practice 81–82. doi: 10.1016/j.annemergmed.2014.05.023.
Downloaded from https://onlinelibrary.wiley.com/doi/ by National Institute Of Standard, Wiley Online Library on [06/03/2024]. See the Terms and Conditions (https://onlinelibrary.wiley.com/terms-and-conditions) on Wiley Online Library for rules of use; OA articles are governed by the applicable Creative Commons License
357

84

A Newborn with Abnormal Movements


Maura Bernardo
Nemours/Alfred I. duPont Hospital for Children, Wilmington, DE, USA

A 12-day-old female presents to the neurology outpatient had hearing screen and cardiac testing which were both
clinic for evaluation of tremors, with a concern that the normal. She has been seen twice by the primary care pro-
child is having seizures. The infant had been evaluated in vider and has an umbilical hernia which will be followed
the pediatric office and again in the emergency department with the possibility of surgery in the future. She is taking
(ED) as parents were very worried about this infant’s ranitidine for hiccups after feeding.
movement.

Family History
History of Present Illness
Parents and siblings of this infant are healthy. There is no
Since birth, this infant has been having jerking/tremulous family history of seizures, cerebral palsy, multiple sclerosis,
episodes of both lower and upper extremities that occur autism, mental retardation, developmental delay, stroke, or
during sleep only. Initially, she was doing these jerks at deep vein thrombosis in the family. There are no reported
least once a day but for the past 3 days parents report only genetic disorders in the family. Maternal grandmother has
having seen one episode. They deny any breathing disorder, rheumatoid arthritis and there are headaches and prostate
color changing, or vomiting after feedings. No facial droop cancer reported on the maternal side. Paternal family his-
or weakness after episodes has been noted. Parents are able tory is positive for hypertension in the great grandfather.
to move her out of the jerking episodes and deny lethargy or
sleeping after episodes. Her eyes remain closed during epi-
sodes as she is asleep for each one. Parents state she does
hiccup a lot. This is their fourth child; no other child has
Current Status
had these types of episodes and there have been no recent
The infant is alert, awake, and has no birthmarks or other
illnesses, such as cold symptoms or fever.
abnormal pigmentation. Overall physical examination is
within normal limits. The neurologic examination is as
follows.
Past Medical History

The infant was born at 39 weeks’ gestation to a gravida 4,


• Cranial nerve (CN) II: pupils equal, round and reactive to
light and accommodation.
para 4, 35-year-old female with prenatal care beginning
early in pregnancy. She was born by planned cesarean
• CN III, IV, and VI: all extraocular movements intact, no
nystagmus noted.
section after an uneventful pregnancy. No complications
after birth. The infant was discharged home with mother
•• CN V: facial sensation normal and symmetrical.
CN VII: facial contours and strength symmetrical.
at 3 days of age. Blood glucose at birth was normal and
the first metabolic screen was normal, second is pending.
•• CN X: good cry.
CN XII: tongue protrusion midline, no fascicula-
Infant received hepatitis B vaccine in the nursery and tions noted.

Cases in Pediatric Acute Care: Strengthening Clinical Decision Making, First Edition. Edited by Andrea M. Kline-Tilford and Catherine M. Haut.
© 2020 John Wiley & Sons Ltd. Published 2020 by John Wiley & Sons Ltd.
Downloaded from https://onlinelibrary.wiley.com/doi/ by National Institute Of Standard, Wiley Online Library on [06/03/2024]. See the Terms and Conditions (https://onlinelibrary.wiley.com/terms-and-conditions) on Wiley Online Library for rules of use; OA articles are governed by the applicable Creative Commons License
358 Cases in Pediatric Acute Care

Figure 84.1 Electroencephalogram (EEG) is normal.

• Motor examination: normal strength, muscle mass, and


tone in all extremities.
Rationale and Evidence-based Practice


Explanation
Reflexes: brisk, 2+ bilaterally, (+) plantar flexor
bilaterally. Based on the information given, what are the
differential diagnoses and which one is the most
Other results as follows.
likely for this infant?

• Skin: no birthmarks or other abnormal pigmentation,


no edema.
Differential diagnoses for this infant include seizure


disorder, metabolic disorder (inborn error of metabolism),
Head: atraumatic/normocephalic, normal hair newborn tremor, benign neonatal sleep myoclonus,
distribution.


opsoclonus–myoclonus disorder, benign familial neonatal
Extremities: no deformity or asymmetry, normal strength seizures, movement disorder, infantile spasms, and neona-
and tone bilateral, full range of motion


tal abstinence syndrome. Differentiating between these dis-
Back: no scoliosis, full range of motion, no tufts orders can be difficult at the age of 2 weeks, so careful
consideration and understanding of each diagnosis is nec-
The infant was evaluated with a short electroencephalo-
essary. The metabolic disorder is least likely as the first
gram (EEG) with results as shown in Figure 84.1.
screen was determined to be normal, but a second screen
is still pending. Seizure disorder or benign familial neonatal
seizures would also be less likely related to the presentation
Questions and normal EEG.
The most likely diagnosis would be benign neonatal sleep
Answer the following questions using the details provided.
myoclonus due to the history, symptoms, and normal EEG
1 Based on the information given, what are the differen- result. Sleep myoclonus is an underdiagnosed disorder of
tial diagnoses and which one is the most likely for this sleep that occurs from birth to age 3 and consists of rhyth-
infant? mic jerking when drowsy or asleep. It occurs particularly
2 What other tests should be considered/ordered? frequently during quiet sleep but may occur in any sleep
3 What treatment/management is warranted for this state. The jerking movements can be focal, multifocal,
infant? marching, or generalized, involving both upper and lower
Downloaded from https://onlinelibrary.wiley.com/doi/ by National Institute Of Standard, Wiley Online Library on [06/03/2024]. See the Terms and Conditions (https://onlinelibrary.wiley.com/terms-and-conditions) on Wiley Online Library for rules of use; OA articles are governed by the applicable Creative Commons License
Chapter 84 A Newborn with Abnormal Movements 359

limbs. Movements are usually symmetric and occur bilater- ordered, with ammonia, serum amino acids and urine
ally and can range in severity from mild to severe, which organic acids and vitamin B12 levels. If the EEG was abnor-
can make diagnosis difficult. These jerks stop when the mal, then sedated magnetic resonance imaging (MRI)
child is woken up and the patient maintains a normal would be indicated.
EEG and neurologic examination. These movements are
also not usually associated with any other more ominous
seizure feature such as crying, change of color, change in What treatment/management is warranted for
breathing pattern, facial movements, or eye movements. this infant?
Some familial cases have been reported.
Sleep myoclonus does not require medication management
but should be followed to ensure it is not increasing in fre-
What other tests should be considered/ordered?
quency or occurring during wakefulness. Developmental
Other testing that should be done includes serum electro- milestones should also be assessed as a plateau or regres-
lytes, including calcium, magnesium and phosphorus to sion would suggest a more malignant disorder or process.
determine normalcy. A metabolic work-up should also be No medication is necessary.

Further Reading
Afawi, Z., Bassan, H., Heron, S., et al. (2012). Benign neonatal Goraya, J.S. (2015). Acute movement disorders in children:
sleep myoclonus: an autosomal dominant form not allelic to experience from a developing country. Journal of Child
KCNQ2 or KCNQ3. Journal of Child Neurology 27(10): Neurology 30(4): 406–411.
1260–1263.
Fernandez-Alvarez, E. (2018). Transient benign paroxysmal
movement disorders in infancy. European Journal of
Paediatric Neurology 22(2): 230–237.
Downloaded from https://onlinelibrary.wiley.com/doi/ by National Institute Of Standard, Wiley Online Library on [06/03/2024]. See the Terms and Conditions (https://onlinelibrary.wiley.com/terms-and-conditions) on Wiley Online Library for rules of use; OA articles are governed by the applicable Creative Commons License
361

85

Postoperative Infant with Tachycardia


Cathy Woodward
Long School of Medicine, University of Texas Health, San Antonio, TX, USA

An 8-month-old infant is admitted to the intensive care unit and epinephrine infusions. Temporary transcutaneous
after complete repair of tetralogy of Fallot (TOF). atrial and ventricular pacing wires are wrapped and secure
on chest. Initial postoperative laboratory results are shown
in Tables 85.1 and 85.2.
History of Present Illness Vital signs are as follows: temperature 37.3 C (99.2 F)
rectal, heart rate 144 bpm in sinus rhythm, blood pressure
The infant was diagnosed prenatally with TOF and has 90/45 mmHg, and oxygen saturation 94% on FiO2 0.6.
been monitored closely since birth. Increasing cyanotic Approximately 1 hour after admission, the registered nurse
spells, failure to thrive, and worsening right heart failure calls you to the bedside because the heart rate is now 188 bpm.
led to a decision to proceed with surgery at this time. Other vital signs include a temperature of 38 C (100.4 F),
blood pressure 68/35 mmHg, and oxygen saturation 92% on
FiO2 0.6 via ventilator circuit. An electrocardiogram (EKG)
Past Medical History is recorded and the results are shown in Figure 85.1.

Born at term and has received all vaccines as well as influ-


enza and palivizumab. Birthweight was 3.2 kg and current Questions
weight is 6 kg. She has been breast-fed primarily with occa-
sional small amounts of rice cereal and fruits. Home med- Answer the following questions using the details provided.
ications include vitamin D supplements every day and oral
propranolol 5 mg three times daily. 1 What is your interpretation of the rhythm strip shown
in Figure 85.1?
2 What would be your initial interventions and what
Family History additional studies would you order?
3 What additional interventions and medications can be
No family history of congenital heart disease, arrhythmias, considered if the rhythm does not convert to sinus
or sudden cardiac death. She has a 3-year-old sibling who is rhythm spontaneously?
healthy.

Rationale and Evidence-based Practice


Current Status
Explanation
The infant is admitted to the intensive care unit immedi-
What is your interpretation of the rhythm strip
ately after complete repair of TOF (i.e. closure of ventricu-
shown in Figure 85.1
lar septal defect, narrowing of the pulmonary valve and
right ventricular outflow tract is augmented by a combina- This infant is tachycardic with a regular rate. Differential
tion of resection of obstructive muscle tissue in the right diagnoses prior to detailed review of the rhythm include
ventricle and enlargement with a patch). The operative sinus tachycardia, atrial tachycardia, and junctional ectopic
course was uneventful. She is intubated and is on milrinone tachycardia.
Cases in Pediatric Acute Care: Strengthening Clinical Decision Making, First Edition. Edited by Andrea M. Kline-Tilford and Catherine M. Haut.
© 2020 John Wiley & Sons Ltd. Published 2020 by John Wiley & Sons Ltd.
Downloaded from https://onlinelibrary.wiley.com/doi/ by National Institute Of Standard, Wiley Online Library on [06/03/2024]. See the Terms and Conditions (https://onlinelibrary.wiley.com/terms-and-conditions) on Wiley Online Library for rules of use; OA articles are governed by the applicable Creative Commons License
362 Cases in Pediatric Acute Care

What would be your initial interventions and what intravenous piggy back (IVPB) over 60 minutes, which
additional studies would you order? can be repeated up to maximum total dose of 15 mg/kg.
Reducing the inotropic epinephrine infusion support may
A 10 mL/kg bolus of normal saline is administered with no
be considered as it can contribute to tachycardia and
change in rhythm but the blood pressure increases to
arrhythmias. Consider using pacing wires for overdrive
74/40 mmHg. The magnesium level was low immediately
pacing.
postoperatively (0.8 mEq/L) and a dose of magnesium is
JET is a serious and potentially life-threatening rhythm
administered. A 12-lead EKG is ordered with findings
in the postoperative period after surgery to repair TOF. It
of narrow-complex tachycardia, inverted P waves, and
can be the result of edema or injury to the tissue surround-
ventricular–atrial dissociation These findings are consist-
ing the electrical conducting atrioventricular (AV) node
ent with junctional ectopic tachycardia (JET).
and occurs in up to 10% of children after cardiac surgery.
Risk factors include surgeries occurring near the AV node
What additional interventions and medications
such as TOF and atrioventricular septal defect repair. Elec-
can be considered if the rhythm does not convert
trolyte imbalance, particularly low magnesium levels, can
to sinus rhythm spontaneously?
precipitate JET in the postoperative period as can low intra-
Decreasing body temperature should be treated with aceta- vascular volume, fever, or high inotropic support. Success-
minophen and cooling blankets to maintain body temper- ful treatment of JET is difficult and several steps may be
ature at 35.5 C (96 F). Administer amiodarone 5 mg/kg needed, including antiarrhythmics, pacing, cooling, and
electrolyte replacement in addition to maintaining normal
Table 85.1 Blood gas analysis. intravascular volume. The use of amiodarone is considered
by some to be a first-line therapy, and when considered it
pH 7.36
can be done concomitantly with other cooling and replace-
PaCO2 48 mmHg
ment strategies. The rhythm, if a result of edema or inflam-
Pao2 130 mmHg
mation, usually converts 24–48 hours after surgery.
Bicarbonate 24 mEq/L
Base excess –1
Lactate 1.2 mmol/L Further Reading
Ismail, M.F., Arafat, A.A., Hamouda, T.E., et al. (2018).
Table 85.2 Pertinent laboratory findings. Junctional ectopic tachycardia following tetralogy of Fallot
repair in children under two years. Journal of Cardiothoracic
Sodium 140 mEq/L
Surgery 13: 60. doi: 10.1186/s13019-018-0749-y.
Potassium 3.6 mEq/L Kovacikova, L., Hakacova N., Dobos, D., Skrak, P., and
Magnesium 0.8 mEq/L Zahorec, M. (2009). Amiodarone as a first-line therapy for
Hematocrit 33% postoperative junctional ectopic tachycardia. Annals of
Thoracic Surgery 88(2): 616–622.

Figure 85.1 EKG rhythm strip.


Downloaded from https://onlinelibrary.wiley.com/doi/ by National Institute Of Standard, Wiley Online Library on [06/03/2024]. See the Terms and Conditions (https://onlinelibrary.wiley.com/terms-and-conditions) on Wiley Online Library for rules of use; OA articles are governed by the applicable Creative Commons License
363

86

Brain Death
Maria C. Woosley
Long School of Medicine, University of Texas Health, San Antonio, TX, USA

A 3-month-old infant is admitted with respiratory failure, heart rate 114 bpm, respiratory rate 30 breaths per minute,
traumatic brain injury, and concern for child abuse. which was the set ventilator rate, blood pressure BP
73/41 mmHg, and oxygen saturation (SpO2) 100%, with
ventilator oxygen set at 100%. Laboratory studies included
History of Present Illness a complete blood count, coagulation panel, and a compre-
hensive metabolic profile. The results are basically within
The patient is a 3-month-old male who was taken to an out- normal limits, except for an elevated glucose (182 mg/dL)
side facility emergency department (ED) by his parents after and potassium (5.8 mEq/L), and carbon dioxide level of
he was found unresponsive in the crib. The child was trans- 12 mg/dL. The results of arterial blood gas analysis are
ported by private vehicle approximately 15 minutes to the shown in Table 86.1.
hospital, during which time he remained unresponsive. The results of the head CT show diffuse hypoattenuation
On arrival to the ED, cardiopulmonary resuscitation (CPR) throughout the brain parenchyma with sulcal effacement
was initiated, the patient was intubated, and CPR was con- and slit-like lateral ventricles and near-complete efface-
tinued for approximately 6 minutes until return of spontane- ment of the basal cisterns. These findings are consistent
ous circulation was attained. He had a head computed with brain edema likely from hypoxic brain injury. No
tomography (CT) completed which indicated a possible right intracranial bleed was appreciated.
subdural hemorrhage, slit ventricles, and near complete The chest and abdominal CT results show:

••
effacement of the basal cisterns. He was transferred by
helicopter to the pediatric level I trauma center for grade I right mid pole renal laceration;
displaced posterior left 10th rib fracture;


continued care.
non-displaced fracture involving the medial third of the
left clavicle;
Past Medical and Surgical History • thin periostial reaction about the bilateral humeri seen
best on the coronal reformats, which suggests healing
of prior trauma that may be non-accidental;


The patient’s medical history was unobtainable due to lack
of family presence on arrival to the trauma center. transverse fracture through the right hallux proximal
phalanx;

Current Status
• bilateral lung consolidations, likely due to aspiration and
atelectasis.

On initial assessment at the trauma center, the patient has a


Glasgow Coma Scale (GCS) score of 3T (intubated), fixed
Questions
and dilated pupils, and no cough or gag reflexes. He has
Answer the following questions using the details provided.
a repeat head CT and a chest, abdomen and pelvic CT. Ini-
tial laboratory studies are obtained, and the patient is then 1 Based on the data presented, what are the differential
transferred to the pediatric intensive care unit (PICU). Vital diagnoses and what is the most likely diagnosis for this
signs in PICU are as follows: temperature 33.9 C (93.0 F), patient?
Cases in Pediatric Acute Care: Strengthening Clinical Decision Making, First Edition. Edited by Andrea M. Kline-Tilford and Catherine M. Haut.
© 2020 John Wiley & Sons Ltd. Published 2020 by John Wiley & Sons Ltd.
Downloaded from https://onlinelibrary.wiley.com/doi/ by National Institute Of Standard, Wiley Online Library on [06/03/2024]. See the Terms and Conditions (https://onlinelibrary.wiley.com/terms-and-conditions) on Wiley Online Library for rules of use; OA articles are governed by the applicable Creative Commons License
364 Cases in Pediatric Acute Care

Table 86.1 Arterial blood gas analysis. The pathology involved with this type of brain injury is
easily explained. The intracranial vault is composed of a
pH 7.28
fixed volume of brain, cerebrospinal fluid, and blood. The
PCO2 28 mmHg Monro–Kellie doctrine states that an increase of one of
PaO2 307 mmHg these fixed volumes must be offset by a decrease in another
Bicarbonate 14 mEq/L (Saceleanu et al., 2018, p. 48). Secondary cerebral lesions
Base deficit –8 such as subdural hematomas or other intracranial hemor-
rhages can cause cerebral edema. A small increase in cere-
bral edema or intracranial volume can cause a serious
2 What are the important considerations when determin- increase in intracranial pressure which can compromise
ing whether a brain death examination is warranted? cerebral perfusion, resulting in brain ischemia, cerebral
3 What are the relevant clinical findings and laboratory edema, brain herniation, and death (Saceleanu et al., 2018).
and imaging results, presented or missing in reported
data, that help determine the possible outcomes and What are the important considerations when
plans for this patient? determining whether a brain death examination is
warranted?
Continuation of case: The infant continued on mechani-
cal ventilation with no sedation required. A blood gas anal- Brain death is a clinical diagnosis and is defined as the irre-
ysis indicated hypocapnia, so the ventilator rate was versible loss of all brain functions including the brainstem.
decreased. Neurologic examinations remained the same, Coma, apnea, and the absence of brainstem reflexes are
with no responses in pupillary reaction, cough or gag reflex. essential findings in brain death. A task force consisting
of representatives including the American Academy of
4 What disciplines should be included in the care of this
Pediatrics, the Society of Critical Care Medicine, and the
patient?
Child Neurology Society established guidelines for deter-
mining brain death in children initially in 1987, and these
were revised in 2011 (Nakagawa et al., 2011) and reaffirmed
Rationale and Evidence-based Practice in 2015. In summary, to determine brain death in children:
Explanation after hypotension, hypothermia and metabolic distur-
bances are treated and corrected and current medications
Based on the data presented, what are the that can interfere with neurologic examination are discon-
differential diagnoses and what is the most likely tinued and cleared, two examinations including apnea test-
diagnosis for this patient? ing with each examination are completed. Prerequisite
conditions should be met; an etiology of the coma should
The differential diagnoses for this patient are non-
be known, and potentially reversible causes of coma should
accidental trauma to the brain, osteogenesis imperfecta,
be excluded.
ingestion of a toxic substance, bleeding disorder, or cardiac
failure leading to respiratory failure. The most likely diag-
nosis for this patient is traumatic brain injury with acute •• Systolic blood pressure should be normalized for age.
Core body temperature should be >35 C.
respiratory failure. The diagnosis of brain death is also
appropriate in this case because of the apparent lack of • Sedative and narcotic elimination should be considered,
but there is no definite recommendation for the number
brainstem function and reflexes. of half-lives that must pass, as in the adult criteria.
This patient’s lack of brainstem reflexes is an important
clinical finding that helps determine the likely outcome, • The PaCO2 must be at least 60 mmHg and must be
20 mmHg greater than baseline for an apnea test to be
along with the period of unresponsiveness and consistency considered positive.
of the poor neurologic examinations. Relevant imaging
results in this case are the findings on the head CT. The • Ancillary testing may be done if there is uncertainty
about the results of the examintion or if medications
patient’s CT performed at the outside hospital revealed a may affect evaluation of the patient (Nelson and
subdural hemorrhage and subsequently showed hypoxic Lewis, 2017).
ischemic encephalopathy. Both of these injuries are com-
mon results of traumatic brain injury that can lead to brain The brain death examintion is a clinical investigation of
death. The near effacement of the basal cisterns also sig- the brainstem reflexes. It includes pupillary response to
nifies pending brain herniation. These findings are poor light; gag, cough, sucking and rooting reflexes; corneal
prognostic indicators for this patient. reflexes; oculovestibular and oculocephalic reflexes; and
Downloaded from https://onlinelibrary.wiley.com/doi/ by National Institute Of Standard, Wiley Online Library on [06/03/2024]. See the Terms and Conditions (https://onlinelibrary.wiley.com/terms-and-conditions) on Wiley Online Library for rules of use; OA articles are governed by the applicable Creative Commons License
Chapter 86 Brain Death 365

movement of bulbar musculature including facial and oro- antiepileptics, and neuromuscular blockers can all affect
pharyngeal muscles. The pupillary response to light tests the neurologic examination. Knowledge of administration
cranial nerves II and III. When the pupils are not respon- of any of these and other medications is imperative in order
sive to light, it confirms that these nerves are not intact. to determine whether the medication is affecting the
Cranial nerves III, IV, VI, and VIII are responsible for ocu- patient’s clinical examination. The original mechanism of
lar movements and are tested by performing oculovestibu- injury is also unknown.
lar testing or what is commonly called “cold caloric”
testing. If no eye movement is noted after ice water is What are the relevant clinical findings and
injected into each ear 5 minutes apart, this indicates a lack laboratory and imaging results, presented or
of response. Oculocephalic (doll’s eyes) testing is performed missing in reported data, that help determine the
by briskly rotating the head horizontally and vertically and possible outcomes and plans for this patient?
if no eye movement is noted it confirms lack of response.
The patient’s temperature is a vital component when deter-
Corneal reflex is controlled by cranial nerves V and VII,
mining plans for this patient and must be normalized to
and involves the lack of eyelid movement when the cornea
complete the brain death testing. Hypothermia may alter
is touched with a cotton swab. Cranial nerves IX and X are
the metabolism and clearance of medications that can
responsible for the cough and gag reflexes, and they are
interfere with brain death testing, and it is a known cause
absent when a patient is brain dead (Nelson and
of depression of central nervous system functions
Lewis, 2017).
(Nakagawa et al., 2011). In pediatrics, a core body temper-
The apnea test is also part of the brain death examination.
ature of greater than 35 C (95 F) should be achieved when
The main objective of the apnea test is to prove that there is
performing a brain death examination (Nelson and
absence of respiratory system reflexes in the brainstem,
Lewis, 2017).
indicating that there is no physiologic stimulation to
Electrolyte abnormalities can alter a patient’s physical
breathe. There must be documentation of a rise in PaCO2
ability, and can be the cause of major organ failure, so cor-
of 20 mmHg above the baseline value, and the reading must
rection of the abnormal value is imperative to optimize the
be more than 60 mmHg with absence of respiratory effort
patient’s outcome or contribute to the decision to declare
during the test (Nelson and Lewis, 2017).
brain death. Reversible electrolyte disturbances such as
Two examinations must be completed separated by an
severe electrolyte imbalances, hyperglycemia or hypoglyce-
observation period, and must be performed by different
mia, severe pH disturbances, and hepatic or renal dysfunc-
attending physicians (although the apnea test may be per-
tion can all lead to coma in the pediatric patient.
formed by the same physician). The recommended observa-
Sometimes the ancillary tests are not congruent with the
tion period is 24 hours for neonates (37 weeks’ gestation to
clinical examination. For example, a patient with normal
term infants 30 days of age) and 12 hours for infants and
cranial nerve ability may have an isoelectric EEG in the set-
children (>30 days to 18 years) (Nakagawa et al., 2011).
ting of hypothermia or metabolic encephalopathy; another
There are no data available to document brain death in
example is an EEG that shows electrical activity, but this
infants less than 37 weeks’ gestation. Teens may also fall into
may possibly be due to electrical artifact in an intensive care
either pediatric or adult guidelines, based on where they are
environment (pumps, ventilators) (Lewis et al., 2017). It is
cared for. If in the PICU, the child is determined to be brain
important to recognize that there are limitations and poten-
dead based on pediatric guidelines. Ancillary tests are used
tial incongruences with ancillary tests. In pediatric brain
in addition to clinical brain death examinations; for infants
death, ancillary tests are done if there is uncertainty about
and children, they include the nuclear blood flow or perfu-
brain death examination results or if medication effects
sion scan, and electroencephalogram (EEG) only. Reduction
interfere with evaluation of the patient (Nelson and Lewis,
of the observation periods is acceptable using an accepted
2017). They may also be utilized if components of the exam-
ancillary study, but when an ancillary study is used to
ination or apnea testing cannot be completed safely, or to
decrease the observation interval, two examinations and
reduce the observation period between the two examina-
two apnea tests are recommended. One examination and
tions (Nakagawa et al., 2011).
an apnea test should be completed before the ancillary
study, and the second examination and an apnea test should
What disciplines should be included in the care of
be completed after the ancillary study The conditions above
this patient?
must be persistent when examination is repeated.
Missing or unclear information relevant to this patient’s A multidisciplinary approach is important because of
current status and potential outcome is the use of any nar- the mechanism of injury and the poor prognosis of this
cotic or brain-altering medication. Sedatives, analgesics, patient. The medical team, neurosurgery, trauma
Downloaded from https://onlinelibrary.wiley.com/doi/ by National Institute Of Standard, Wiley Online Library on [06/03/2024]. See the Terms and Conditions (https://onlinelibrary.wiley.com/terms-and-conditions) on Wiley Online Library for rules of use; OA articles are governed by the applicable Creative Commons License
366 Cases in Pediatric Acute Care

surgery, and pediatric critical care providers will all be with the medical providers, is important as literature
involved in the care of this patient. The bedside pediatric indicates that this aspect of care provision greatly
nurses are important contributors to discussions invol- impacts the decision for parents to agree to organ dona-
ving the potential brain death examinations because of tion (Basak et al., 2018).
their proximity to the bedside and the patient’s family. For this particular case, the child abuse specialists should
Palliative care and social work are also an important also be involved due to the unknown cause of injury and
resource for the patient’s family as well as with the obvious non-accidental head injury. Child life specialists
staff’s emotional needs. Organ donation teams may also can assist with grief and bereavement support, as well as
be involved, along with the palliative care team because provide handprints, molds, and other special gifts for the
they can help with supporting the bereaved family mem- family. Pastoral services are also an important addition to
bers if they decide to donate their child’s organs. The the team as they can provide spiritual guidance to the fam-
involvement of the palliative care team, in collaboration ily and care providers.

References
Basak, R., Louis, S., Shin, N., et al. (2018). Organ donation in infants and children: An update of the 1987 task force
pediatric patients with severe anoxic brain injury. Indian recommendations. Pediatrics 128: e720–e740. doi: 10.1542/
Journal of Palliative Care 24: 362–364. peds.2011-1511.
Lewis, A., Adams, N., Chopra, A., et al. (2017). Use of ancillary Nelson, A. and Lewis, A. (2017). Determining brain death:
test when determining brain death in pediatric patients in basic approach and controversial issues. American Journal
the United States. Journal of Child Neurology 32(12): of Critical Care 26(6): 496–500. doi: 10.4037/ajcc2017540.
975–980. Saceleanu, C., Cindea, C., and Saceleanu, A. (2018). Pediatric
Nakagawa, T.A., Ashwal, S., Mathur, M., et al. (2011). Clinical traumatic brain injury: pathophysiology, treatment and case
Report: Guidelines for the determination of brain death in reports. Acta Medica Transilvanica 23(3): 48–51.
Downloaded from https://onlinelibrary.wiley.com/doi/ by National Institute Of Standard, Wiley Online Library on [06/03/2024]. See the Terms and Conditions (https://onlinelibrary.wiley.com/terms-and-conditions) on Wiley Online Library for rules of use; OA articles are governed by the applicable Creative Commons License
367

87

Postoperative Spinal Fusion with Complications


Jaime Panton1 and Melissa C. Varrone2,3
1
Columbia University School of Nursing, New York, NY, USA
2
Morgan Stanley Children’s Hospital of New York, New York, NY, USA
3
New York Presbyterian Hospital, Columbia University Irving Medical Center, New York, NY, USA

A 14-year-old female who weighs 50 kg was diagnosed with Past Medical History
adolescent idiopathic scoliosis and is being admitted to the
pediatric intensive care unit (PICU) after undergoing pos- This adolescent female was diagnosed with adolescent idi-
terior spinal fusion. opathic scoliosis at age 12 during a routine sports physical.
She has had a rapidly progressive spinal curve that contin-
ued despite bracing, requiring surgical intervention. She
History of Present Illness
had recurrent acute otitis media during early childhood
but otherwise has had no significant past medical history.
The adolescent was diagnosed with idiopathic scoliosis at
12 years of age. Bracing was attempted but due to her rap-
idly progressing spinal curve, the orthopedic team elected
to perform a posterior spinal fusion, from T2 to L4. The Past Surgical History
patient was in her usual state of good health when she pre-
sented to the operating room today. She had an easy mask Bilateral myringotomy tubes placed at 18 months of age.
induction with oxygen and sevoflurane. She was given a She is currently status post posterior spinal fusion (postop-
dose of rocuronium, fentanyl, and ondansetron and was erative day 1).
intubated with a 7.5 cuffed endotracheal tube. End-tidal
CO2 recording was obtained and maintained throughout
the procedure. Bilateral breath sounds were auscultated. Family History
Once asleep she had two peripheral intravenous catheters
placed in her left and right hand as well as a right radial Mother has hypertension and paternal grandfather has
20-gauge arterial line placed. Anesthesia was maintained hypercholesterolemia.
throughout the case with sevoflurane, air, and intermittent
doses of fentanyl and rocuronium. She was given cefazolin
1 g at 11:40 at the start of the case. Vital signs remained Current Status
within accepted parameters and she tolerated the proce-
dure well. At the conclusion of the case, she was given Postoperative day 1 vital signs as follows: heart rate 78–95
glycopyrrolate and neostigmine and was reversed and bpm, arterial blood pressure 100/52 to 118/71 mmHg, res-
extubated without difficulty. An autologous blood recovery piratory rate 14–26 breaths per minute, and pulse oximetry
system was used. Urine output was 120 mL. She received with oxygen saturations of 95–98% on room air. Her pain is
2000 mL of crystalloid fluids and was then transferred to rated 1–2/10 on a hydromorphone patient-controlled
PICU in satisfactory condition. analgesia pump.
Per the orthopedic surgery team report, the surgery was The 24-hour fluid intake and output totals are shown in
successful. There was no documented loss of motor or Table 87.1. She is currently on maintenance intravenous
sensory function during the procedure. fluid of 5% dextrose with 0.9% normal saline infusing at

Cases in Pediatric Acute Care: Strengthening Clinical Decision Making, First Edition. Edited by Andrea M. Kline-Tilford and Catherine M. Haut.
© 2020 John Wiley & Sons Ltd. Published 2020 by John Wiley & Sons Ltd.
Downloaded from https://onlinelibrary.wiley.com/doi/ by National Institute Of Standard, Wiley Online Library on [06/03/2024]. See the Terms and Conditions (https://onlinelibrary.wiley.com/terms-and-conditions) on Wiley Online Library for rules of use; OA articles are governed by the applicable Creative Commons License
368 Cases in Pediatric Acute Care

Table 87.1 Fluid intake and output totals (mL). Continuation of case: The PICU team is concerned about
the oliguria and orders the following studies:

••
Intake Intravenous fluids 3160 Total 3310
Oral 0 urine specific gravity 1.025
Intravenous medications 150
urine osmolarity 300 mosmol/kg
Output Urine output 480
Drains 100
Operating room blood loss 120
Total 700
•• urine sodium 30 mEq/L
serum osmolarity 267 mosmol/L.
Net +2610 3 Based on this additional information, what is the most
likely diagnosis?
4 What is the appropriate management for this case?
Table 87.2 Complete blood count.

White blood cell count 8000/mm3


Hemoglobin 9 g/dL
Rationale and Evidence-based Practice
Hematocrit 27%
Explanation
Platelet count 188 000/mm3
Based on the information given, what are the most
Neutrophils 69%
likely differential diagnoses?
Lymphocytes 19%
Monocytes 1%
Hyponatremia is one of the most common electrolyte dis-
turbances seen in critically ill children. The most common
Eosinophils 1%
causes of severe hyponatremia are syndrome of inappropri-
ate antidiuretic hormone (SIADH) and cerebral–renal salt
wasting, although iatrogenic causes should be considered
Table 87.3 Basic metabolic profile.
as well, such as the administration of excessive free water
Sodium 129 mEq/L via either intravenous or enteral route. The differential
Potassium 4.4 mEq/L for hyponatremia can be narrowed down by evaluating
the relationship of total body water to sodium levels.
Chloride 98 mEq/L
Hypovolemic hyponatremia is seen in patients who have
Carbon dioxide 23 mEq/L
losses with diuretic use or significant gastrointestinal losses
Blood urea nitrogen 8 mg/dL
of sodium and water, which in turn leads to osmotic anti-
Creatinine 8 mg/dL diuretic hormone (ADH) control resulting in the body
Glucose 122 mg/dL retaining water and increasing the degree of hyponatremia.
Calcium 8.5 mg/dL Other types of decreased total body sodium are secondary to
extrarenal losses, such as with burns and rhabdomyolysis
or through renal losses, as seen in patients with sodium-
90 mL/hour with only sips of water and two episodes of losing renal disease or in patients receiving thiazide-type
non-bloody non-bilious emesis. Urine output is 0.4 mL/kg diuretics. Adrenal insufficiency is another cause and is usu-
per hour with the oliguria occurring over the last 4 hours. ally associated with hyperkalemia. Euvolemic hyponatre-
The physical examination is unremarkable; her surgical site mia is most often seen with excessive water intake, or
(T2–L4) is covered with a surgical dressing that has slight associated with some drugs that potentiate decreased water
shadowing noted, but the dressing is otherwise intact. Two excretion, or with SIADH. The overproduction of ADH acts
drains (one shallow, one deep) are in place, with 100 mL on the collecting ducts of the kidneys resulting in inappro-
of serous drainage output for 24 hours. Complete blood priate retention of water. Excessive ADH may be the result
count and basic metabolic profile are obtained and the of changes in autonomic signals stemming from factors
results are shown in Tables 87.2 and 87.3. common in postoperative patients, such as pain, stress,
hypoxia, and volume depletion. Congestive heart failure,
nephrotic syndrome, or renal failure are common causes
Questions
for hypervolemic hyponatremia. The stimulation of aortic
and carotid vasoreceptors and receptors within the heart
Answer the following questions using the details provided.
may increase sympathetic activity and stimulate the
1 Based on the information given, what are the most renin–angiotensin–aldosterone axis, in turn increasing
likely differential diagnoses? renal vascular resistance resulting in decreased urinary
2 What additional laboratory testing may be helpful? sodium excretion and increased retention of water.
Downloaded from https://onlinelibrary.wiley.com/doi/ by National Institute Of Standard, Wiley Online Library on [06/03/2024]. See the Terms and Conditions (https://onlinelibrary.wiley.com/terms-and-conditions) on Wiley Online Library for rules of use; OA articles are governed by the applicable Creative Commons License
Chapter 87 Postoperative Spinal Fusion with Complications 369

What additional laboratory testing may be swelling/edema and, in worst-case scenarios, herniation of
helpful? the brainstem. Low serum sodium causes water to shift
from extracellular fluid to intracellular fluid, resulting in
Assuming that preoperative laboratory work was obtained
cellular edema.
and the initial basic metabolic panel was unremarkable,
The goal is to correct and normalize sodium levels.
urine studies and serum osmolarity should be obtained.
Sodium levels should be increased by no more than
The hyponatremia found in SIADH is accompanied by
0.5 mEq/L per hour in patients with chronic hyponatremia,
low serum osmolarity as well as increased urine specific
and no faster than 1 mEq/L per hour in patients with acute
gravity, urine osmolarity, and urine sodium.
hyponatremia. More rapid corrections are generally better
tolerated if the hyponatremia has developed within the last
Based on this additional information, what is the
4 hours.
most likely diagnosis?
The use of isotonic intravenous fluids is always indicated
Based on the laboratory findings and clinical picture, this while avoiding hypotonic solutions. Patients who exhibit
patient is diagnosed with SIADH related to surgical poste- symptoms of increased intracranial pressure or seizures
rior spinal fusion. due to hyponatremia should be given a bolus of 3% hyper-
tonic saline to correct serum sodium to at least 125 mEq/L.
What is the appropriate management for SIADH due to spinal surgery is generally self-limiting
this case? and resolves quickly. However, care should be taken to pre-
vent sequelae from the dilutional hyponatremia and poten-
Sudden and severe hyponatremia can be life-threatening
tial fluid overload.
and requires very careful management to prevent cerebral

Further Reading
Ball, S.G. and Iqbal, Z. (2015). Diagnosis and treatment of Jones, D. (2018). Syndrome of inappropriate secretion of
hyponatremia. Best Practice and Research in Clinical antidiuretic hormone and hyponatremia. Pediatrics in
Endocrinology and Metabolism 30: 161–173. doi: 10.1016/j. Review 39: 27–35. doi: 10.1542/pir.2016-0165.
beem.2015.12.001.
Fuhrman, B. and Zimmerman, J. (2011). Pediatric Critical
Care, 4th edn. Philadelphia, PA: Elsevier Saunders.
Downloaded from https://onlinelibrary.wiley.com/doi/ by National Institute Of Standard, Wiley Online Library on [06/03/2024]. See the Terms and Conditions (https://onlinelibrary.wiley.com/terms-and-conditions) on Wiley Online Library for rules of use; OA articles are governed by the applicable Creative Commons License
371

88

Orthopnea in a 10-Year-Old
Max Pizzo
C.S. Mott Children’s Hospital, Ann Arbor, MI, USA

A 10-year-old female presents to her primary care provider colitis. No known family history of cancer or
(PCP) with complaints of worsening orthopnea for 1 week, immunodeficiency.
stating she has required additional pillows to sleep
comfortably.
Current Status
History of Present Illness Patient denies fever and night sweats. Clinical examination
is notable for cervical lymphadenopathy. Her PCP orders
Over the preceding month she has noted an intermittent an initial complete blood count (Table 88.1) with differen-
non-productive cough and exertional dyspnea. In addition, tial, peripheral smear, and a chest radiograph. The chest
she has complained of abdominal pain and nausea, and has radiograph is notable for a widened mediastinum, so a
noted a 2.7 kg weight loss. The abdominal pain and nausea follow-up computed tomography (CT) scan of the chest is
with vomiting have become worse over the past 5 days. urgently scheduled. A referral is placed for follow-up with
pediatric hematology and oncology.
Chest CT results are reviewed by pediatric hematology/
Past Medical History oncology. Imaging is concerning for a large anterior medi-
astinal mass, mild tracheal narrowing, a moderate pericar-
History of reactive airway disease. Her current medications dial effusion, and a moderate right pleural effusion
include budesonide 0.5 mg daily via nebulization and albu- (Figures 88.1 and 88.2). The family is contacted and direc-
terol 0.63 mg every 4–6 hours as needed for cough/wheez- ted to immediately present to the emergency department
ing via nebulization. She has noted no improvement in (ED) for evaluation and admission.
cough, dyspnea, and orthopnea with inhalation treatments.
Currently up-to-date immunizations for age excluding the
seasonal influenza vaccination. Questions

Answer the following questions using the details provided.


Past Surgical History
1 Based on the data given, what are the differential
diagnoses?
No past surgical history.
2 What laboratory work or tests would be helpful in
determining a diagnosis?
Family History Continuation of case: On arrival at the ED, vital signs are
as follows: temperature 36.9 C (98.4 F), heart rate 120
Father, brother, maternal grandmother, maternal aunt, bpm, respiratory rate 22 breaths per minute, blood pres-
and maternal uncle are positive for a history of kidney sure 124/75 mmHg, and oxygen saturation 98% in room
stones. Brother is also positive for a history of ulcerative air. Complete blood count with differential, peripheral

Cases in Pediatric Acute Care: Strengthening Clinical Decision Making, First Edition. Edited by Andrea M. Kline-Tilford and Catherine M. Haut.
© 2020 John Wiley & Sons Ltd. Published 2020 by John Wiley & Sons Ltd.
Downloaded from https://onlinelibrary.wiley.com/doi/ by National Institute Of Standard, Wiley Online Library on [06/03/2024]. See the Terms and Conditions (https://onlinelibrary.wiley.com/terms-and-conditions) on Wiley Online Library for rules of use; OA articles are governed by the applicable Creative Commons License
372 Cases in Pediatric Acute Care

Table 88.1 Initial complete blood count. Table 88.2 Complete metabolic profile.

White blood cell count 10 600/mm3 Sodium 140 mEq/L


Hemoglobin 9.2 g/dL Potassium 3.7 mEq/L
Hematocrit 27.6% Chloride 102 mEq/L
Platelet count 103 000/mm3 Carbon dioxide 19 mEq/L
Atypical lymphocytes 48% Blood urea nitrogen 17 mg/dL
Creatinine 1.19 mg/dL
Glucose 73 mg/dL
Calcium 8.3 mg/dL
Phosphorus 5.5 mg/dL
Lactate dehydrogenase 650 U/L
Uric acid 24.5 mg/dL

Table 88.3 Complete blood count.

White blood cell count 5500/mm3


Hemoglobin 12.1 g/dL
Hematocrit 33.6%
Figure 88.1 Chest CT scan.
Platelet count 65 000/mm3
Neutrophils 5%
Eosinophils 0%
Monocytes 2%
Lymphocytes 6%
Bands 0%
Blasts 86%

She is transferred to the pediatric intensive care unit


(PICU) on completion of the procedures.
3 What are the complications associated with this
Figure 88.2 Chest CT scan. diagnosis?
4 What is the complication associated with a pericardial
effusion?
blood smear and flow cytometry, comprehensive meta- 5 What is the concern, if any, with the use of sedation in a
bolic panel and phosphorus, lactate dehydrogenase patient with this diagnosis?
(LDH), and uric acid are obtained and the results are
Continuation of case: Intravenous fluid at twice the cal-
shown in Tables 88.2 and 88.3. Peripheral smear and flow
culated maintenance rate is continued on admission to
cytometry are consistent with T-cell acute lymphoblastic
the PICU. Rasburicase is administered to treat a uric
leukemia (ALL).
acid level greater than 8 mg/dL and prevention of tumor
An echocardiogram is obtained with results significant
lysis syndrome. Renal function panel, LDH, and uric
for a large global pericardial effusion with right atrial col-
acid continue to be assessed every 6 hours. Fluid that
lapse and intermittent right ventricle collapse. Pediatric
has been removed via the pericardial drain is replaced
cardiology is consulted. She is taken for placement of a per-
1:1 with intravenous fluid for volume repletion. Cefazo-
icardial drain, left upper extremity peripherally inserted
lin is initiated as prophylaxis while the pericardial drain
central catheter, lumbar puncture, and bone marrow
remains in place.
biopsy. Midazolam, ketamine, and fentanyl are used for
sedation and analgesia during the procedure. Cytarabine 6 What are the complications this patient might face
is administered intrathecally during the lumbar puncture. given the current data provided?
Downloaded from https://onlinelibrary.wiley.com/doi/ by National Institute Of Standard, Wiley Online Library on [06/03/2024]. See the Terms and Conditions (https://onlinelibrary.wiley.com/terms-and-conditions) on Wiley Online Library for rules of use; OA articles are governed by the applicable Creative Commons License
Chapter 88 Orthopnea in a 10-Year-Old 373

Rationale and Evidence-based Practice filling and precipitously to decreased left ventricular
Explanation output. Hemodynamic instability is the end result.
The definitive treatment for cardiac tamponade requires
Based on the data given, what are the differential decompression of the pericardium.
diagnoses?
What is the concern, if any, with the use of
Of the data provided, the most concerning symptom is wor- sedation in a patient with this diagnosis?
sening orthopnea over the past week. The differential diag-
nosis should include reactive airway disease, infection, Sedation should be used with caution in those patients with
gastroesophageal reflux disease, and anterior mediastinal an anterior mediastinal mass. These patients are at risk for
mass. The most common anterior mediastinal masses cardiopulmonary collapse when sedated; patients with
include Hodgkin lymphoma, non-Hodgkin lymphoma, orthopnea have an increased risk of airway collapse in
thymomas, and teratomas. Kikuchi–Fujimoto disease is a the setting of anesthesia. The smooth muscle in airways
benign self-limiting histiocytic necrotizing lymphadenitis and vasculature that maintain patency are relaxed with
that can result in an anterior mediastinal mass. Orthopnea sedation, allowing for compression, or further compres-
can also be the result of a pleural or pericardial effusion. sion, by the anterior mediastinal mass. Negative pressure
created by spontaneous inspiration is compromised with
sedation, leading to an increase in intrathoracic pressure
What laboratory work or tests would be helpful in
that results in greater potential for obstruction from an
determining a diagnosis?
anterior mediastinal mass.
A complete blood count with differential and peripheral In addition, the positioning of a patient with an anterior
smear would be helpful in determining if there is malig- mediastinal mass during a procedure can impact ventilation
nancy. A chest radiograph would be beneficial in evalua- and perfusion. Supine positioning increases the collapse cre-
tion for an anterior mediastinal mass or pleural effusion. ated by the anterior mediastinal mass due to the effect of
Pericardial effusions can be appreciated with an gravity, worsening ventilation and perfusion mismatch.
echocardiogram. The use of sedation resulting in the need for positive-
pressure ventilation is also not an ideal situation in a patient
What are the complications associated with this with an anterior mediastinal mass. The point of obstruction
diagnosis? may be beyond the reach of an endotracheal tube, limiting
the ability to stent open the airways. Positive-pressure venti-
Several cardiorespiratory complications are associated with lation can increase intrathoracic pressure. As mentioned
an anterior mediastinal mass, many of which are specifi- already, an increase in intrathoracic pressure can result in
cally related to the position of the mass and the airways greater potential for obstruction from an anterior mediasti-
and vasculature that are obstructed. Hypoxemia can result nal mass. Emergent intervention with steroid or radiation
from compression of airways. Pulmonary edema and therapy may be required to treat the tumor and maintain
decreased pulmonary perfusion can result from com- a patent airway, as the most common tumors causing this
pressed pulmonary vasculature. Hypoventilation may presentation are responsive to steroids as first-line therapy
result from air trapping and elevated carbon dioxide due and radiation as a second-line intervention.
to airway collapse.
Superior vena cava syndrome, a consequence of superior What are the complications this patient might face
vena cava compression, is another complication of an anterior given the current data provided?
mediastinal mass. Compression of the superior vena cava
leads to venous congestion and edema of superior tissues. The diagnosis of T-cell ALL with a mediastinal mass and
Superior mediastinal syndrome describes superior vena cava hyperuricemia places this patient at high risk for the devel-
syndrome with accompanying compression of the trachea. opment of tumor lysis syndrome. Tumor lysis syndrome is
the result of cell breakdown and the release of intracellular
potassium, phosphate, and nucleic acid into the blood-
What is the complication associated with a
stream. It is defined by two or more metabolic abnormal-
pericardial effusion?
ities (hyperuricemia, hyperkalemia, hyperphosphatemia,
The complication associated with a pericardial effusion is and hypocalcemia) 3 days before or up to 7 days after the
cardiac tamponade. Fluid collection in the pericardium initiation of therapy. Tumor lysis syndrome can result in
results in increased intrapericardial pressure that can even- acute kidney injury due to hyperphosphatemia and hyper-
tually exceed transmural myocardial pressure. Compres- uricemia. Hydration at generally two times maintenance is
sion of the myocardium results in decreased diastolic the initial treatment for tumor lysis syndrome and the
Downloaded from https://onlinelibrary.wiley.com/doi/ by National Institute Of Standard, Wiley Online Library on [06/03/2024]. See the Terms and Conditions (https://onlinelibrary.wiley.com/terms-and-conditions) on Wiley Online Library for rules of use; OA articles are governed by the applicable Creative Commons License
374 Cases in Pediatric Acute Care

resultant acute kidney injury. Further treatment directed at Case Resolution


specific metabolic abnormalities may be necessitated.
Worsening of the moderate right pleural effusion The patient responded well to induction therapy and was
requires frequent assessment. This patient is receiving a able to transfer to the hematology/oncology ward after
significant amount of fluid and is at risk for further fluid her metabolic derangements were stabilized and her peri-
accumulation. The pericardial drain will prevent reaccu- cardial effusion resolved and the pericardial drain was
mulation in the pericardium. removed.

Further Reading
Acker, S.N., Linton, J., Tan, G.M., et al. (2015). A Mejia, R., Rodriguez, N.J., Cortes, J.A., et al. (2016). Oncologic
multidisciplinary approach to the management of anterior emergencies and complications. In: Rodgers’ Textbook of
mediastinal masses in children. Journal of Pediatric Surgery Pediatric Intensive Care (ed. D.G. Nichols and D.H.
50: 875–878. Shaffner), 1872–1893. Philadelphia, PA: Lippincott
Henry, M. and Sung, L. (2015). Supportive care in pediatric Williams & Wilkins.
oncology. Pediatric Clinics of North America 62: 27–46. Prusakowski, M.K. and Cannone, D. (2017). Pediatric
Kang, H.M., Kim, J.Y., Choi, E.H., et al. (2018). Clinical oncologic emergencies. Hematology Oncology Clinics of
characteristics of severe histiocytic necrotizing North America 31: 959–980.
lymphadenitis (Kikuchi–Fujimoto disease) in children. Stephanos, K. and Picard, L. (2018). Pediatric oncologic
Journal of Pediatrics 171: 208–212. doi: 10.1016/j. emergencies. Emergency Medicine Clinics of North America
jpeds.2015.12.064. 36(3): 527–535. doi: 10.1016/j.emc.2018.04.007.
Downloaded from https://onlinelibrary.wiley.com/doi/ by National Institute Of Standard, Wiley Online Library on [06/03/2024]. See the Terms and Conditions (https://onlinelibrary.wiley.com/terms-and-conditions) on Wiley Online Library for rules of use; OA articles are governed by the applicable Creative Commons License
375

89

Complication of Traumatic Brain Injury


Christine Schindler1,2
1
Marquette University College of Nursing, Milwaukee, WI, USA
2
Medical College of WI/Children’s Hospital of WI, Milwaukee, WI, USA

A previously healthy 14-year-old boy is admitted to the 12-year-old sister with well-controlled atopic dermatitis
pediatric intensive care unit (PICU) with severe traumatic and asthma and a healthy 10-year-old brother.
brain injury (TBI).

History of Present Illness Current Status

The teen was an unrestrained passenger in a high-speed The patient is being cared for in the PICU for manage-
motor vehicle collision 3 days ago. He presented to the chil- ment of severe TBI. He is currently intubated and
dren’s hospital trauma center with a Glasgow Coma Scale mechanically ventilated with tight carbon dioxide con-
(GCS) score of 7 and initial head computed tomography trol, is deeply sedated, and on seizure prophylaxis with
(CT) demonstrated evidence of diffuse axonal injury with levetiracetam. He has an extraventricular drain in place
cerebral edema and slit-like ventricles (Figure 89.1). He monitoring intracranial pressures ranging from 15 to
was emergently intubated, and the severe TBI protocol 18 mmHg. Norepinephrine is infusing at 0.05 μg/kg
initiated. per min to maintain cerebral perfusion pressure of
65 mmHg. The teen has a urinary drainage catheter in
place and on day 3 post injury he develops an acute
Past Medical History increase in urine output. For the last 3 hours, urine out-
put has been 7 mL/kg per hour.
The patient was born at term and discharged home with his The results of a basic metabolic profile are shown in
mother at 2 days of life. He has no medical problems and Table 89.1. The results of serum and urine osmolality are
has received all required vaccines along with annual influ- as follows:

••
enza immunization. His weight is at 40% for gender and age
and current body mass index (BMI) is 19.5. He has been serum osmolality 332 mosmol/kg
urine osmolality 190 mosmol/kg


well throughout his life with the only notable injury a
non-comminuted right radial fracture at 5-year-old. urine specific gravity 1.005

Past Surgical History

No surgical history, except for circumcision at birth.


Questions

Answer the following questions using the details provided.


Family History 1 Based on the data given, what are the differential diag-
noses and the most likely diagnosis for this child?
The patient’s father has hypertension and type 2 diabetes 2 Why are children with severe traumatic brain injury at
and his mother has Crohn’s disease. The patient has a risk for diabetes insipidus?

Cases in Pediatric Acute Care: Strengthening Clinical Decision Making, First Edition. Edited by Andrea M. Kline-Tilford and Catherine M. Haut.
© 2020 John Wiley & Sons Ltd. Published 2020 by John Wiley & Sons Ltd.
Downloaded from https://onlinelibrary.wiley.com/doi/ by National Institute Of Standard, Wiley Online Library on [06/03/2024]. See the Terms and Conditions (https://onlinelibrary.wiley.com/terms-and-conditions) on Wiley Online Library for rules of use; OA articles are governed by the applicable Creative Commons License
376 Cases in Pediatric Acute Care

Rationale and Evidence-based Practice


Explanation

Based on the data given, what are the differential


diagnoses and the most likely diagnosis for
this child?
Differential diagnoses for this child include syndrome of
inappropriate antidiuretic hormone secretion (SIADH),
cerebral salt wasting (CSW), nephrogenic diabetes insipi-
dus, and central diabetes insipidus.
The most likely diagnosis for this child is central
diabetes insipidus. Diabetes insipidus is a disorder resulting
in decreased secretion (central/neurogenic) or action
(nephrogenic) of antidiuretic hormone (ADH). ADH acts
on specific vasopressin receptors and its main physiologic
actions are to increase water absorption in the distal neph-
ron and promote the expression of specific water channel
proteins on the luminal surface of the collecting duct. Dia-
betes insipidus manifests with loss of large volumes (>5
mL/kg per hour) of dilute urine (specific gravity <1.010)
Figure 89.1 Head CT. in the presence of high plasma osmolality (>295 mos-
mol/kg). Diabetes insipidus is distinguishable with polyuria
and polydipsia in uncontrolled diabetes mellitus or other
forms of solute diuresis by the absence of glucosuria and
Table 89.1 Basic metabolic profile.
a relatively normal rate of total urinary solute excretion.
Sodium 158 mEq/L
Potassium 2.9 mEq/L
Chloride 102 mEq/L Why are children with severe traumatic brain
Carbon dioxide 22 meq/L
injury at risk for diabetes insipidus?
Blood urea nitrogen 28 mg/dL Neuroendocrine dysfunction can be a consequence of TBI
Creatinine 0.8 mg/dL and is a result of both anterior and posterior pituitary insuf-
Glucose 110 mg/dL ficiency. TBI can directly and indirectly lead to damage of
Calcium 8.2 mg/dL
the ADH-producing neurons, their axons, or the posterior
pituitary resulting in post-traumatic diabetes insipidus.
The pituitary gland and stalk are at particular risk of direct
injury in high-speed acceleration–deceleration shear-type
injuries, because the stalk is surrounded anatomically by
Continuation of case: This 14-year-old is diagnosed with
the rigid diaphragma sellae and the gland is encased within
post-traumatic diabetes insipidus. His central venous pres-
the rigid bony sella. The rotational velocity of the head may
sure (CVP) decreases from 7 to 4 mmHg. In children with
result in stretching or tearing of small vessels or neuronal
altered consciousness, hypotonic polyuria can rapidly lead
structures in the hypothalamic–pituitary unit. Indirect
to hypovolemia and hypernatremia. Rapid and accurate
injury to the hypothalamic–pituitary unit can result from
assessment of the patient’s volume status is a crucial com-
small vessel damage, inflammatory edema, and hypoxia.
ponent of clinical management. Hypovolemia is associated
The incidence of acute post-traumatic hypopituitarism of
with adverse outcomes in the child with severe TBI, and
any anterior pituitary hormonal axis is 50–76%. The inci-
therefore initial therapy includes rapid volume expansion
dence of posterior pituitary dysfunction (diabetes insipidus)
with 20 mL/kg of 0.9% normal saline administered over
in the acute state following moderate or severe TBI is
10 minutes.
approximately 20% and permanent diabetes insipidus
3 What additional therapies would be recommended at develops in approximately 7% of patients.
this time? It is often difficult to remember differences between
4 What determines if the therapy was effective? SIADH, CSW and diabetes insipidus (see Table 89.2).
5 Should any other diagnostic studies be obtained? SIADH occurs for a variety of reasons, including TBI,
Downloaded from https://onlinelibrary.wiley.com/doi/ by National Institute Of Standard, Wiley Online Library on [06/03/2024]. See the Terms and Conditions (https://onlinelibrary.wiley.com/terms-and-conditions) on Wiley Online Library for rules of use; OA articles are governed by the applicable Creative Commons License
Chapter 89 Complication of Traumatic Brain Injury 377

Table 89.2 Characteristics of diabetes insipidus, SIADH and CSW.

Diabetes insipidus SIADH CSW

Serum sodium High Low Low


Serum osmolarity (mosmol/kg) >295 <280 <280
Urine sodium Low High High
Urine osmolarity Low High High
Urine specific gravity <1.010 >1.020 >1.010
Urine output ≥4 mL/hour ≤1 mL/hour 2–3 mL/kg per hour

meningitis, and space-occupying brain lesions. SIADH be administered for evidence of persistent posterior pitui-
occurs as a result of excessive release of ADH from the pos- tary dysfunction with derangements in fluid balance,
terior pituitary gland and causes findings of hyponatremia serum osmolality, and serum sodium.
and decreased urine output. CSW is also related to an intra-
cranial disease process, but the pathophysiology is not as What determines if the therapy was effective?
well understood. CSW results in renal loss of sodium with
hyponatremia and volume depletion. Diabetes insipidus is Frequent electrolyte laboratory evaluation is indicated as
a deficiency in ADH, resulting in polyuria with low specific caution must be taken to avoid rapid correction of serum
gravity. Diabetes insipidus can be either central or nephro- sodium. Cerebral tissue is very sensitive to quick osmotic
genic; the nephrogenic form can be either congenital or changes, which can lead to worsening cerebral edema.
acquired. Clinicians should target a serum sodium correction rate
of no more than 0.5 mmol/hour or 10–12 mmol/L over
What additional therapies would be recommended 24 hours. Clinicians should titrate therapy for a urine out-
at this time? put of approximately 1 mL/kg per hour and urine specific
gravity of 1.010–1.030.
Ongoing fluid replacement should be guided by constant
clinical monitoring and CVP measurements to avoid under-
Should any other diagnostic studies be obtained?
hydration leading to hypovolemia and potentially
decreased cerebral perfusion pressure, as well as overhydra- Pituitary dysfunction following TBI can be significant and
tion which can exacerbate cerebral edema, increase intra- the incidence of anterior pituitary dysfunction after TBI is
cranial pressure, or precipitate pulmonary edema. around 30% in the acute time frame and remains about 20%
Additionally, hormonal replacement with desmopressin at 1 year post injury. Consensus guidelines recommend
(exogenous ADH) is indicated. 1-Deamino-8-D-arginine screening all patients with moderate to severe TBI for pitu-
vasopressin (DDAVP), or desmopressin, is a synthetic ana- itary axis dysfunction. Basal hormone testing should be
log of vasopressin with minimal pressor effects and is the completed on any hospitalized patient with hypotension
drug of choice for the management of diabetes insipidus. or hyponatremia and testing at 3 and 12 months regardless
Desmopressin can be administered orally, intranasally, of severity of TBI should be performed. Basal testing
sublingually, or parenterally. In the intensive care setting, includes morning cortisol, free T3, free T4, thyroid-
the parenteral desmopressin formulation is preferred. stimulating hormone (TSH), insulin-like growth factor
There is significant individual variability in the duration (IGF)-1, follicle-stimulating hormone (FSH), luteinizing
and magnitude of effect. The dose of desmopressin should hormone (LH), testosterone (males), estradiol (females),
be titrated to normalize urine output and provide the prolactin, and a 24-hour urine collection for urinary free
required rate of plasma sodium reduction to avoid risks cortisol. Patients with polyuria should also have serum
of overtreatment. Additional doses of desmopressin should sodium and serum osmolality checked.

Further Reading
Capatina, C., Paluzzi, A., Mitchell, R., et al. (2015). Idowu, O.E., Obafunwa, J.O., and Soyemi, S.O. (2017).
Diabetes insipidus after traumatic brain injury. Journal Pituitary gland trauma in fatal nonsurgical closed traumatic
of Clinical Medicine 4: 1448–1462. doi: 10.3390/ brain injury. Brain Injury 31(3): 359–362. doi: 10.1080/
jcm4071448. 02699052.2016.1257823.
Downloaded from https://onlinelibrary.wiley.com/doi/ by National Institute Of Standard, Wiley Online Library on [06/03/2024]. See the Terms and Conditions (https://onlinelibrary.wiley.com/terms-and-conditions) on Wiley Online Library for rules of use; OA articles are governed by the applicable Creative Commons License
378 Cases in Pediatric Acute Care

Lal, R.A. and Hoffman, A.R. (2019). Pituitary response to Scranton, R.A. and Baskin, D.S. (2015). Impaired pituitary axes
traumatic brain injury. In: Pituitary Disorders of Childhood following traumatic brain injury. Journal of Clinical
(ed. B. Kohn). Cham, Switzerland: Humana Press. Medicine 4: 1463–1479. doi: 10.3390/jcm4071463.
Robertson, G.L. (2016). Diabetes insipidus: differential Silva, P.P.B., Bhatnagar, S., Herman, S.D., et al. (2015).
diagnosis and management. Best Practice and Research in Predictors of hypopituitarism in patients with traumatic
Clinical Endocrinology and Metabolism 30: 2015–218. doi: brain injury. Journal of Neurotrauma 32: 1789–1795. doi:
10.1016/j.beem.2016.02.007. 10.1089/neu.2015.3998.
Downloaded from https://onlinelibrary.wiley.com/doi/ by National Institute Of Standard, Wiley Online Library on [06/03/2024]. See the Terms and Conditions (https://onlinelibrary.wiley.com/terms-and-conditions) on Wiley Online Library for rules of use; OA articles are governed by the applicable Creative Commons License
379

90

Newborn with Hypothermia


Marshay James1,2
1
Vanderbilt University School of Nursing, Nashville, TN, USA
2
Department of Critical Care Medicine, St. Jude Children’s Research Hospital, Memphis, TN, USA

A 9-day-old female arrived at her local community emer- Current Status


gency department (ED) with lethargy and dehydration.
The infant was fussy the past few days, would not take a On arrival to the ED, the infant is extremely lethargic,
bottle, and this morning was difficult to wake. The infant with pediatric Glasgow Coma Scale (GCS) score of 6
is under custody of the grandmother who is unsure of (Table 90.1). Her initial vital signs include temperature
her perinatal course. 35.1 C (95.2 F), heart rate 68 bpm, respiratory rate
14 breaths per minute which are very shallow, blood
History of Present Illness pressure 65/32 mmHg, and oxygen saturation with use
of non-rebreather mask 88%. The infant is intubated
This 9-day-old infant became ill-appearing within the last and mechanically ventilated due to mental status and
24 hours, was initially irritable and poorly consoled, pro- poor respiratory effort. Complete blood count (CBC),
gressing to poor oral intake, and inciting concern for the basic metabolic panel (BMP), C-reactive protein (CRP),
caregiver. Temperature was not taken at home. There is cerebrospinal fluid (CSF) studies including cell count,
no report of cough, runny nose, difficulty breathing, wheez- Gram stain, and blood, urine and CSF cultures are
ing or other respiratory symptoms. Grandmother feels the obtained, with herpes simplex virus (HSV) PCR. Results
infant looks pale. The infant is having soft bowel move- for CBC, BMP and CSF analysis are shown in
ments and no excess spitting. Tables 90.2–90.4. CRP level is 234 mg/dL and initial lac-
tate level 7.5 mmol/L.
Head computed tomography (CT) is obtained prior to
Past Medical History lumbar puncture to assess the presence of increased intra-
cranial pressure. A chest radiograph demonstrates endotra-
Previous hospital records reveal that this infant was born at cheal tube just proximal to the carina and no acute
36.4 weeks’ gestation. At birth, she was appropriate for ges- intrapulmonary findings.
tational age, in the 40th percentile for weight and length,
with birthweight of 2.86 kg. She was delivered via uncom-
plicated vaginal delivery but admitted to the neonatal Questions
intensive care unit because of the mother’s known sub-
stance abuse and concern for neonatal abstinence syn- Answer the following questions using the details provided.
drome. She has had first hepatitis B vaccine, vitamin 1 Based on the data, what are the differential diagnoses
K injection, and first metabolic screening completed. and the most likely diagnosis for this child?
2 Given the most likely diagnosis, what is the initial phar-
Family History macologic therapy?
3 What medications to treat herpes simplex virus would
This infant was discharged home to the maternal grand- be indicated?
mother; mother incarcerated; father’s history is unknown 4 What additional diagnostic studies would be warranted
and he is not involved with the mother or infant. at this time?

Cases in Pediatric Acute Care: Strengthening Clinical Decision Making, First Edition. Edited by Andrea M. Kline-Tilford and Catherine M. Haut.
© 2020 John Wiley & Sons Ltd. Published 2020 by John Wiley & Sons Ltd.
Downloaded from https://onlinelibrary.wiley.com/doi/ by National Institute Of Standard, Wiley Online Library on [06/03/2024]. See the Terms and Conditions (https://onlinelibrary.wiley.com/terms-and-conditions) on Wiley Online Library for rules of use; OA articles are governed by the applicable Creative Commons License
380 Cases in Pediatric Acute Care

Table 90.1 Pediatric Glasgow Coma Scale (GCS) score. she had ampicillin and cefotaxime administered in the ED
and was given a 10 mL/kg fluid bolus.
Best eye opening No response, score 1
Best verbal response Moans to pain, score 2
Best motor response Abnormal flexion to pain, score 3
Total 6/15 Rationale and Evidence-based Practice
Explanation

Based on the data, what are the differential


Table 90.2 Complete blood count.
diagnoses and the most likely diagnosis for
White blood cell count 2500/mm3 this child?
Hemoglobin 11.0 g/dL Differential diagnoses for this child include non-accidental
Hematocrit 33.6% trauma, inborn errors of metabolism, and infectious etiol-
Platelet count 85 000/mm3 ogy including but not limited to sepsis, bacteremia and
Neutrophils 36% meningitis. The most likely diagnosis for this child is neo-
natal sepsis due to the child’s history and presenting
Lymphocytes 42%
symptoms of hypothermia, hypoglycemia, irritability pro-
Monocytes 13%
gressing to lethargy, decreased oral intake, and initial lab-
Eosinophils 1% oratory findings. Meningitis is unlikely with normal CSF
results, but cultures are still pending. This child also has
an unclear perinatal history and the maternal health status
Table 90.3 Basic metabolic panel. at the time of birth is unknown, so maternal infection may
be a possibility and typical neonatal organisms include
Sodium 145 mEq/L group B Streptococcus, Escherichia coli, Streptococcus pneu-
Potassium 3.9 mEq/L moniae, and Listeria monocytogenes. In viral pathogenesis,
Chloride 115 mEq/L herpes simplex virus (HSV)-1 and HSV-2 can cause neona-
Carbon dioxide 12 mEq/L tal meningitis, although HSV-2 accounts for 70% of cases.
Blood urea nitrogen 18 mg/dL
Late-onset neonatal sepsis or bacteremia occurs in infants
greater than 72 hours old and can still reflect bacterial organ-
Creatinine <0.1 mg/dL
isms present in early-onset neonatal infection. Organisms
Glucose 53 mg/dL
can be transferred through an ascending pathway from
Calcium 8.5 mg/dL mother to infant or through ruptured membranes. Neonatal
sepsis is associated with high risk of morbidity and mortality.
The risk of sepsis increases in a neonate whose mother had
fever, lack of prenatal care, premature labor, premature rup-
Table 90.4 CSF initial results. ture of membranes, group B Streptococcus colonization, uri-
nary tract infection, and others.
Appearance Clear
Sepsis is invasion by microbial pathogens, characterized
Opening pressure 15 cmH2O
by a proinflammatory response that can lead to severe sep-
White cell count 4500/mm3 sis and septic shock. In neonates, cytokines are regulators
Predominant cell type Lymphocytes of sepsis. Interleukin (IL)-6 is a cytokine which is evident
Protein 62 mg/dL in the neonatal population, and shows early response to
Glucose 46 mg/dL infection, preceding the increase in CRP. IL-6 acts as a sig-
Red cell count 0 nal in the activation of T cells, and induces the secretion of
antibodies by B cells and the differentiation of T cells. It sti-
mulates the release of other cytokines, increasing several
hours before the increase in CRP. These regulators
Continuation of case: This 9-day-old infant is transferred have high sensitivity and are capable of effective defense
to the pediatric intensive care unit (PICU) and placed on against pathogens; however, cytokines can dysregulate
minimal ventilatory support. After cultures were obtained, the immune response and ultimately promote tissue
Downloaded from https://onlinelibrary.wiley.com/doi/ by National Institute Of Standard, Wiley Online Library on [06/03/2024]. See the Terms and Conditions (https://onlinelibrary.wiley.com/terms-and-conditions) on Wiley Online Library for rules of use; OA articles are governed by the applicable Creative Commons License
Chapter 90 Newborn with Hypothermia 381

damage. There is a fine line here, between repair and encephalitis or focal findings on examination, imaging,
improvement and further demise within the infectious or electroencephalography. However, a body of research
cascade. strongly suggests that an infant who is treated for sepsis
should also receive acyclovir until the CSF PCR for HSV
Given the most likely diagnosis, what is the initial returns negative. Clinical features suggestive of HSV infec-
pharmacologic therapy? tion include mucocutaneous vesicles, seizures, lethargy,
respiratory distress, thrombocytopenia, coagulopathy,
Neonates admitted from the community are at lower risk
blood oozing from intravascular catheter sites, hypother-
for infection caused by multidrug-resistant pathogens than
mia, sepsis-like illness, hepatomegaly, ascites, or elevated
infants who have been hospitalized since birth. The combi-
transaminases.
nation of ampicillin plus gentamicin or ampicillin plus a
third-generation cephalosporin (e.g. cefotaxime) are regi-
mens for empiric treatment of sepsis without an apparent What additional diagnostic studies would be
focus of infection, as in this setting. It is also recommended warranted at this time?
to treat for HSV with acyclovir when the patient is a neo-
Obtaining coagulation studies is important to assist in
nate or young infant and recent maternal screens are not
determining if the infant may be experiencing disseminated
available, or there is fever present. This patient will also
intravascular coagulation, which is a concern for any
require fluid resuscitation and electrolyte supplementation.
patient who is septic. If the infant remains stable and is able
Placement of a central venous line to administer inotropic/
to wean and tolerate extubation without any alterations in
vasopressor infusion and fluid boluses may be necessary
hemodynamic status, routine monitoring of electrolytes
and an arterial line may be needed to monitor blood pres-
would continue until feedings are on board. It is also impor-
sure and obtain blood gases.
tant to monitor lactate and CRP levels to ensure that they
are decreasing. Evidence regarding mortality in neonates
What medications to treat HSV would be
with sepsis indicates that high CRP and lactate levels are
indicated?
associated with higher risks of death, especially if levels
Empiric therapy with acyclovir is definitely recommended do not decrease. Following cultures and identification of
for children with CSF abnormalities and who have organisms will document definitive treatment.

Further Reading
Dong, Y. and Speer, C.P. (2015). Late onset neonatal sepsis: Thomson, J., Sucharew, H., Cruz, A.T., et al. (2018).
recent developments. Archives of Disease in Childhood, Fetal Cerebrospinal fluid reference values for young infants
and Neonatal Edition 100: F257–F263. doi: 10.1136/ undergoing lumbar puncture. Pediatrics 141(3): e20173405.
archdischild-2014-306213. doi: 10.1542/peds.2017-3405.
Good, P.I. and Hooven, T.A. (2019). Evaluating infants at risk
for early onset sepsis. Pediatric Clinics of North America 66:
321–331. doi: 10.1016/j.pcl.2018.12.003.
Downloaded from https://onlinelibrary.wiley.com/doi/ by National Institute Of Standard, Wiley Online Library on [06/03/2024]. See the Terms and Conditions (https://onlinelibrary.wiley.com/terms-and-conditions) on Wiley Online Library for rules of use; OA articles are governed by the applicable Creative Commons License
383

91

Back Pain in an Active Teenager


Alicia McCarthy
Nemours/Alfred I. duPont Hospital for Children, Wilmington, DE, USA

A 14-year-old male presents to the outpatient orthopedic Family History


clinic for evaluation of back pain.
Negative for osteogenesis imperfecta or skeletal dysplasias.
Father with degenerative disk disease and disk herniation
History of Present Illness requiring single-level spinal fusion at age 52.

This 14-year-old male fell jumping off a friend’s deck yes-


terday; fall height approximately 1.83 m (6 feet). He landed
Current Status
on his feet and then fell onto the lower part of his back. He
reports immediate pain after the injury, but was able to
The patient is able to ask and answer questions appropri-
walk. Pain is focused in his midline upper lumbar region,
ately. He walks and changes positions independently but
worsens with movement, and improves with lying supine.
with obvious signs of pain. No dysmorphic features, sclera
He reports pain is 8/10 in office today with movement, 5/10
are white. Full neck range of motion without pain. Chest rise
when sitting. Family has given him intermittent
is symmetric on inspiration, and there is no increased work
acetaminophen which has not provided much relief. Used
of breathing. Chest is without deformity. Head is level over
a wheelchair to get into clinic today.
pelvis, sagittal balance is neutral. Focal pain with palpation
of the upper lumbar spine. Significant pain with mild for-
ward bending and hyperextension of the spine in this same
Past Medical History region. Skin is intact with no abrasions, bruises or rashes.
Extremities are pink, warm, no swelling or edema. Radial
This teen was born full term, with no prenatal or neonatal and posterior tibial pulses are +2. No pain with palpation
complications. Parents cannot recall specific developmen- of bilateral upper and lower extremities. There is 5/5
tal milestones but feel they were “normal” and similar to strength in all major muscle groups of upper and lower
those of his siblings. He was previously treated for low extremities. Patellar and Achilles reflexes are at 2+, one beat
vitamin D levels by his primary care provider but stopped of clonus bilaterally. Babinski test is negative.
taking the supplements 1 year ago. Orthopedic history is
positive for distal non-displaced radius fracture at age 7,
after falling in a soccer game. No previous hospitalizations
or chronic health conditions. He weighs 65.2 kg, Questions
height 176 cm.
Answer the following questions using the details provided.
1 Based on history and physical examination, what are
Past Surgical History the differential diagnoses and the most likely diagnosis
for this child?
There is no past surgical history. 2 What testing is required for definitive diagnosis?

Cases in Pediatric Acute Care: Strengthening Clinical Decision Making, First Edition. Edited by Andrea M. Kline-Tilford and Catherine M. Haut.
© 2020 John Wiley & Sons Ltd. Published 2020 by John Wiley & Sons Ltd.
Downloaded from https://onlinelibrary.wiley.com/doi/ by National Institute Of Standard, Wiley Online Library on [06/03/2024]. See the Terms and Conditions (https://onlinelibrary.wiley.com/terms-and-conditions) on Wiley Online Library for rules of use; OA articles are governed by the applicable Creative Commons License
384 Cases in Pediatric Acute Care

Continuation of case: The patient is sent for anteroposte- Rationale and Evidence-based Practice
rior/lateral erect spine radiographs (entire spine on single Explanation
cassette, see Figure 91.1), and a total 25-hydroxyvitamin
D level is obtained in the laboratory (result 19 ng/mL). Based on history and physical examination, what
3 What treatment is recommended to stabilize the spine? are the differential diagnoses and the most likely
4 What pain relief interventions should be diagnosis for this child?
recommended? Initial differential diagnoses include muscular pain, frac-
5 Are other interventions required and what follow-up is ture of the spine, and disk herniation or bulge. Fracture
needed? of the spine is more likely due to focal nature of pain and

Figure 91.1 Erect spine radiographs.


Downloaded from https://onlinelibrary.wiley.com/doi/ by National Institute Of Standard, Wiley Online Library on [06/03/2024]. See the Terms and Conditions (https://onlinelibrary.wiley.com/terms-and-conditions) on Wiley Online Library for rules of use; OA articles are governed by the applicable Creative Commons License
Chapter 91 Back Pain in an Active Teenager 385

midline location. Disk herniation or bulge is less likely due may return to school, but should rest from activities that
to absence of radicular and neurologic symptoms and nor- require prolonged standing and cannot participate in
mal neuromotor testing. The patient was at increased risk gym or sports.
for low vitamin D levels due to his history of this condition,
and the fact that he had stopped supplementation.
What pain relief interventions should be
recommended?
What testing is required for definitive diagnosis?
External bracing will provide some amount of pain relief. In
Radiographs should be obtained as a first-line evaluation
addition, he may take non-steroidal anti-inflammatory
for back pain after trauma and for chronic back pain and
drugs (NSAIDs) for pain relief. Other possible pain relief
should be taken in the erect position. Although radiographs
methods include heat or ice application as needed, and life-
should generally be limited to the area of concern, in the
style modifications (such as sitting in a supportive chair).
case of possible compression fractures, imaging of the
entire spine is important to evaluate a possible impact on
overall alignment. Are other interventions required and what
The radiographs for this patient indicate compression follow-up is needed?
fractures of L1 and L2, with less than 25% loss of vertebral
It is also important to supplement vitamin D. Recom-
height of each vertebra. There is no overall change in his
mended dosing should be 2000 IU of vitamin D daily, with
sagittal alignment. Because of physical examination find-
levels rechecked in 12 weeks. If the level is normal at that
ings, mild loss of vertebral height and no change in align-
time, the patient should continue on maintenance dosing of
ment on X-ray, no additional imaging (such as MRI or CT)
600–1000 IU daily as a long-term medication.
is required for diagnosis and treatment.
The patient should return in 6 weeks for repeat antero-
Laboratory work confirms deficiency of vitamin D, which
posterior/lateral erect spine radiographs (entire spine on
places the patient at increased risk for fracture.
single cassette). At that visit, if symptoms are improved,
he may discontinue his orthosis and start a course of phys-
What treatment is recommended to stabilize
ical therapy to work on postural training, core strengthen-
the spine?
ing, back and lower extremity stretching. He could resume
External bracing (in this case lumbosacral orthosis) should sports to tolerance once cleared by the physical therapist
be used at all times when not bathing or sleeping. The teen (likely 4–6 weeks).

Further Reading
Booth, T.N., Iyer, R.S., Falcone, R.A., et al. (2017). Thompson, R.M., Dean, D.M., Goldberg, S., et al. (2017).
ACR Appropriateness Criteria® Back Pain-Child. Vitamin D insufficiency and fracture risk in urban children.
Journal of the American College of Radiology 14(5 Suppl): Journal of Pediatric Orthopedics 37(6): 368–373. doi: 10.1097/
S13–S24. BPO.0000000000000697.
Downloaded from https://onlinelibrary.wiley.com/doi/ by National Institute Of Standard, Wiley Online Library on [06/03/2024]. See the Terms and Conditions (https://onlinelibrary.wiley.com/terms-and-conditions) on Wiley Online Library for rules of use; OA articles are governed by the applicable Creative Commons License
387

92

Teen with Fever and Abdominal Pain


Sarah Martin
Department of Pediatric Surgery, Ann & Robert H. Lurie Children’s Hospital, Chicago, IL, USA

A 13-year-old-female with a 2-day history of fever and Family History


abdominal pain presents to the emergency department
(ED) for evaluation. The family history is unknown due to her adoption.

History of Present Illness Current Status


This 13-year-old female reports a 2-day history of low-grade On arrival to the ED, the patient reports she has abdominal
fever and abdominal pain that started at her umbilicus and pain of 7/10. Her vital signs are as follows: temperature
now localizes to the right lower quadrant. Her mother 38 C (100.4 F) orally, heart rate 116 bpm, respiratory rate
reports a tactile fever and states her daughter has not been 15 breaths per minute, blood pressure 108/66 mmHg,
eating or drinking very well. The child has been treated and oxygen saturation 100% on room air.
with acetaminophen every 6 hours as needed for her tactile Anthropometric measurements reflect normal growth
fever and Maalox for her abdominal pain. The child has not parameters and physical examination reveals an alert teen-
had emesis or diarrhea and last voided 10 hours ago. Her ager watching a movie on her phone. Her head, ears, nose,
last reported oral intake was a popsicle that she ate 4 hours throat, and chest examination are unremarkable. Abdomi-
prior to her ED presentation. Her last menstrual period was nal examination is significant for right lower quadrant pain
1 week ago. Her parents deny any recent gastrointestinal with rebound tenderness; there are no masses or hernias
illnesses in the family. present. Her extremities are pink, warm, dry with brisk cap-
illary refill time. A basic metabolic panel, complete blood
count, and urinalysis are obtained (see Tables 92.1–92.3).
An abdominal ultrasound is performed done over concern
Past Medical History for appendicitis. A peripheral intravenous catheter is
inserted and intravenous fluids administered at mainte-
This teen was adopted and her birth history is unknown. nance rate after she receives a 20 mL/kg normal saline fluid
Her immunizations are up to date and she takes no bolus for oliguria. The child receives a dose of morphine for
prescription or over-the-counter medications regularly. her abdominal pain. She is ordered nil by mouth until her
She was diagnosed with strep throat 6 months ago. She diagnosis and treatment plan can be determined.
has never been to an ED or required hospitalization. Abdominal ultrasound identified dilated non-
compressible tubular structure in the right lower quadrant
consistent with an appendix measuring up to 1.1 cm in
maximum transverse diameter. There was adjacent inflam-
Past Surgical History matory fat stranding and increased vascularity. Sono-
graphic findings are consistent with acute simple or
There is no past surgical history. uncomplicated acute appendicitis.

Cases in Pediatric Acute Care: Strengthening Clinical Decision Making, First Edition. Edited by Andrea M. Kline-Tilford and Catherine M. Haut.
© 2020 John Wiley & Sons Ltd. Published 2020 by John Wiley & Sons Ltd.
Downloaded from https://onlinelibrary.wiley.com/doi/ by National Institute Of Standard, Wiley Online Library on [06/03/2024]. See the Terms and Conditions (https://onlinelibrary.wiley.com/terms-and-conditions) on Wiley Online Library for rules of use; OA articles are governed by the applicable Creative Commons License
388 Cases in Pediatric Acute Care

Table 92.1 Complete blood count. Continuation of case: The child undergoes a laparoscopic
appendectomy for category 1 appendicitis, which can be
White blood cell count 18 000/mm3
defined as an appendix with acute appendicitis, localized
Hemoglobin 12 g/dL appendicitis, without gangrene, perforation, or abscess
Platelet count 234 000/mm3 (Baxter et al., 2018).
Neutrophils 85%
4 What is appropriate care in the immediate postopera-
Lymphocytes 7.4%
tive period for a child who has undergone a laparo-
Monocytes 7% scopic appendectomy?
Eosinophils 0.2% 5 What would be the plan for hospital discharge and fol-
Basophils 0.2% low-up?

Table 92.2 Basic metabolic panel.


Rationale and Evidence-based Practice
Sodium 137 mEq/L
Explanation
Potassium 3.9 mEq/L
Chloride 102 mEq/L
Based on the data given, what are the differential
Carbon dioxide 18 mEq/L diagnoses that should be considered and the most
Blood urea nitrogen 20 mg/dL likely diagnosis for this teen?
Creatinine 0.56 mg/dL
Differential diagnoses to consider for this child include a
Glucose 98 mg/dL urinary tract infection, ovarian torsion, ovarian cyst, gastro-
enteritis, and appendicitis. Based on this child’s elevated
white blood cell count with a left shift and her ultrasound
Table 92.3 Urinalysis. findings, she most likely has appendicitis. Appendicitis is
the most common emergent indication for abdominal
Specific gravity 1.017 surgery in children. The peak incidence of diagnosis is
Color Yellow 10–19 years of age (Wagner et al., 2018). Although nearly
Nitrite Negative half of the children diagnosed with appendicitis will have
Blood Negative an atypical presentation, this patient had many of the clas-
Glucose Negative sic symptoms including dull periumbilical pain migrating
to the right lower quadrant, fever, and anorexia.

Questions What is the most appropriate testing for this


diagnosis?
Answer the following questions using the details provided.
There are several appendicitis scores (e.g. Alvarado, Appen-
1 Based on the data given, what are the differential diag- dicitis Inflammatory Response Score, Pediatric Appendici-
noses that should be considered and the most likely tis Score) that can aid clinicians when making the
diagnosis for this teen? diagnosis. Depending on the score, a white blood cell count
2 What is the most appropriate testing for this diagnosis? of greater than 10 000–15 000/mm3 with a neutrophil count
Continuation of case: With the confirmation of appendi- above 75% are laboratory findings that contribute to the
citis based on imaging findings, the child is started on intra- likelihood of appendicitis, along with clinical findings of
venous antibiotics and prepared for a laparoscopic right lower quadrant pain, anorexia, and guarding
appendectomy. Ceftriaxone and Flagyl were prescribed (Wagner et al., 2018). Radiologic evaluation includes the
with 24-hour dosing. A human chorionic gonadotropin use of ultrasound, computed tomography (CT), and mag-
point-of-care urine test was obtained and was negative. netic resonance imaging (MRI). CT scan is known to be
the most sensitive but does expose children to ionizing radi-
3 What are appropriate interventions for the treatment of ation. Ultrasound is the most commonly performed radio-
simple acute appendicitis? logic test and can be done in the awake toddler and child
Downloaded from https://onlinelibrary.wiley.com/doi/ by National Institute Of Standard, Wiley Online Library on [06/03/2024]. See the Terms and Conditions (https://onlinelibrary.wiley.com/terms-and-conditions) on Wiley Online Library for rules of use; OA articles are governed by the applicable Creative Commons License
Chapter 92 Teen with Fever and Abdominal Pain 389

without exposure to ionizing radiation. Ultrasound has Table 92.4 Candidate inclusion criteria for non-operative
high specificity but low sensitivity because operator experi- management of acute appendicitis.
ence is variable (Mittal et al., 2013). The findings of
7–17 years old
acute appendicitis include an inflamed fluid-filled appen-
Abdominal pain less than 48 hours with no history of chronic
dix, a non-compressible blind-ending structure measuring abdominal pain
greater than 6 mm in maximal diameter; an appendicolith
White blood cell count 5000–18 000/mm3
may or may not be present. Negative ultrasound findings
Ultrasound imaging: hyperemia, <1.1 cm in diameter,
include the organ not visualized with no secondary findings
compressible or non-compressible, no abscess, no fecalith/
suggestive of appendicitis. MRI is gaining popularity as it appendicolith, no phlegmon
does not employ ionizing radiation and contrast is not No preoperative concern for perforation
needed. Most centers can perform the test without sedation
No diffuse peritonitis
in children older than 5 years (Mittal et al., 2013). Findings
Not pregnant
suggestive of appendicitis on MRI include an appendiceal
diameter greater than 7 mm, periappendiceal fat infiltra- No malignancy
tion, and restricted diffusion of the appendiceal wall Not currently taking antibiotics
(Mittal et al., 2013). Availability of this modality and high
cost are factors that are making use of this terchnique
variable. What is appropriate care in the immediate
postoperative period for a child who has
undergone a laparoscopic appendectomy?
What are appropriate interventions for the
Care in the immediate postoperative period includes intra-
treatment of simple acute appendicitis?
venous and oral pain medications, diet progression as toler-
Treatment approaches for appendicitis most often include ated, and early ambulation with no additional antibiotics
operative management with preoperative intravenous anti- (Hornor et al., 2018). Ambulation and preparation for dis-
biotics, intravenous fluids, and intravenous pain manage- charge are also included in the postoperative plans.
ment. Laparoscopic appendectomy is the most common
operative intervention, with these procedures converted What would be the plan for hospital discharge and
to open operations as indicated. Non-operative intervention follow-up?
has been recently studied and is an option in some cases of
Same-day discharge is the evolving benchmark for children
appendicitis (Gonzalez et al., 2016). Candidates for non-
undergoing appendectomy for simple acute appendicitis
operative intervention are summarized in Table 92.4.
(Benedict et al., 2018). Strategies to achieve this include pre-
Non-operative candidates in a hospitalized setting are
operative education of the patient and family of planned
administered antibiotics and diet is progressed based on
care trajectory. This patient would be discharged once afe-
clinical examination findings. A success rate for non-
brile, tolerating fluids with minimal nausea, voiding,
operative management is cited as 70–85% for children at
ambulating, no bleeding, non-distended abdomen and pain
1 year (Gonzalez et al., 2016).
controlled.

References
Baxter, K.J., Short, H.L., Travers, C.D., et al. (2018). Gonzalez, D.O., Deans, K.J., and Minneci, P.C. (2016). Role of
Implementing a surgeon-reported categorization of non-operative management in pediatric appendicitis.
pediatric appendicitis severity. Pediatric Surgery Seminars in Pediatric Surgery 25: 204–207. doi: 10.1053/j.
International 34: 1281–1286. doi: 10.1007/s00383-018- sempedsurg.2016.05.002.
4364-8. Hornor, M.A., Liu, J.Y., Hu, Q.L., et al. (2018). Surgical
Benedict, L.A., Sujka, J., Sobrino, J., et al. (2018). Same-day technical evidence review for acute appendectomy
discharge for nonperforated appendicitis in children: an conducted for the Agency for Healthcare Research
updated institutional protocol. Journal of Surgical Research and Quality Safety Program for improving surgical
232: 346–350. doi: 10.1016/j.jss.2018.06.057. care and recovery. Journal of the American College of
Downloaded from https://onlinelibrary.wiley.com/doi/ by National Institute Of Standard, Wiley Online Library on [06/03/2024]. See the Terms and Conditions (https://onlinelibrary.wiley.com/terms-and-conditions) on Wiley Online Library for rules of use; OA articles are governed by the applicable Creative Commons License
390 Cases in Pediatric Acute Care

Surgeons 227: 605–617. doi: 10.1016/j. Wagner, M., Tubre, D.J., Asensio, J.A., et al. (2018). Evolution
jamcollsurg.2018.09.024. and current trends in the management of acute appendicitis.
Mittal, M.K., Dayan, P.S., Macias, C.G., et al. (2013). Surgical Clinics of North America 98: 1005–1023. doi:
Performance of ultrasound in the diagnosis of appendicitis 10.1016/j.suc.2018.05.006.
in children in a multicenter cohort. Academic Emergency
Medicine 20(7): 697–702. doi: 10.1111/acem.12161.
Downloaded from https://onlinelibrary.wiley.com/doi/ by National Institute Of Standard, Wiley Online Library on [06/03/2024]. See the Terms and Conditions (https://onlinelibrary.wiley.com/terms-and-conditions) on Wiley Online Library for rules of use; OA articles are governed by the applicable Creative Commons License
391

93

Complication in a 6-Year-Old with a Kidney Transplant


Alison Hewitt Torres
Johns Hopkins Children’s Center, Baltimore, MD, USA

A 6-year-old boy with a history of end-stage renal disease Past Surgical History
(ESRD) due to obstructive uropathy status post living
related kidney transplant from his father 3 months ago pre- The child is status post two fetal surgeries, both for vesico-
sents to the clinic with 1-day history of diarrhea and emesis. peritoneal shunt placement; and is status post suprapubic
catheter placement in the neonatal period. Bladder aug-
mentation with continent stoma creation was done prior
History of Present Illness to transplant in order to facilitate optimal drainage of urine
for transplant. He also had a living related kidney trans-
This 6-year-old boy presented to the nephrology clinic for plant from his father; surgical course was uncomplicated.
evaluation of dehydration, with two episodes of diarrhea
and one episode of vomiting overnight. He is 3 months sta-
tus post living related kidney transplant.
Family History
Past Medical History No family history of kidney problems or anyone requiring
dialysis. Paternal grandmother with history of cancer.
This patient was diagnosed with presumed posterior ure-
thral valves at 20 weeks’ gestation in the setting of anhy-
dramnios, echogenic kidneys, and dilated bladder. Fetally
placed vesicoperitoneal shunt was attempted and subse- Current Status
quently replaced. The shunt again became dislodged and
elective cesarean section was planned for 31 weeks. The On arrival to clinic, the patient is lethargic, pale, and ill
infant then spent 3 months in the neonatal intensive care appearing. Vital signs are as follows: temporal temperature
unit and was discharged home with parents with an 37.8 C (100 F), heart rate 145 bpm, respiratory rate
indwelling suprapubic drain. He continued to have bilat- 40 breaths per minute, automated blood pressure
eral grade 4–5 reflux despite multiple urologic surgical 65/34 mmHg, and oxygen saturation 95% on room air.
interventions and his chronic kidney disease continued to The lips are dry and cracked, capillary refill is delayed at
worsen. By age 5 this boy had reached ESRD and under- 3–4 seconds. Central pulses are easily palpable, but periph-
went evaluation for transplant. Urology performed bladder eral pulses are thready. Abdomen is distended with Foley
augmentation with continent stoma creation to optimize catheter in continent stoma. Basic metabolic panel and
bladder for transplant and received living related kidney complete blood count are obtained at an outside facility
from his father 8 weeks later. Frequent viral infections, prior to coming to clinic, and results are shown in
neutropenia, and low-level donor-specific antibodies have Tables 93.1 and 93.2.
complicated his transplant course. His baseline creatinine A blood culture, urine culture and urinalysis are obtained
is 0.4 mg/dL. and sent to the inpatient laboratory.

Cases in Pediatric Acute Care: Strengthening Clinical Decision Making, First Edition. Edited by Andrea M. Kline-Tilford and Catherine M. Haut.
© 2020 John Wiley & Sons Ltd. Published 2020 by John Wiley & Sons Ltd.
Downloaded from https://onlinelibrary.wiley.com/doi/ by National Institute Of Standard, Wiley Online Library on [06/03/2024]. See the Terms and Conditions (https://onlinelibrary.wiley.com/terms-and-conditions) on Wiley Online Library for rules of use; OA articles are governed by the applicable Creative Commons License
392 Cases in Pediatric Acute Care

Table 93.1 Basic metabolic panel. normotensive, shock and non-compensated, or hypoten-
sive, shock. This patient’s clinical presentation is consistent
Sodium 138 mEq/L
with non-compensated shock as he is lethargic and ill
Potassium 4.8 mEq/L appearing, with delayed capillary refill and hypotension.
Chloride 100 mEq/L Patients presenting in shock require immediate interven-
Carbon dioxide 9 mEq/L tion. Pediatric patients presenting with hypotensive shock
Blood urea nitrogen 38 mg/dL are in imminent danger of death.
Creatinine 2.2 mg/dL Given this 6-year-old’s recent diarrheal episodes in com-
Glucose 121 mg/dL
bination with his increased urine output due to his history
of obstructive uropathy, hypovolemic shock remains high
Calcium 7.8 mg/dL
on the differential. Another significant consideration in
determining the diagnosis for the patient is his current
immunosuppression regimen as well as his induction reg-
Table 93.2 Complete blood count.
imen related to kidney transplant 3 months ago. Pediatric
White blood cell count 2800/mm3 kidney transplant guidelines recommend that recipients
Hemoglobin 8 g/dL receive a biologic, either an interleukin 2 receptor antago-
nist or a lymphocyte-depleting agent for induction. This
Hematocrit 29.3%
child received a lymphocyte-depleting agent, thymoglobu-
Platelet count 493 000/mm3
lin, for induction. Thymoglobulin will deplete lymphocytes
Neutrophils 17% for 2 months but up to 6 months. Most pediatric kidney
Lymphocytes 59% transplant patients require triple immunosuppression, with
Monocytes 23% a calcineurin inhibitor, an antimetabolite, and a steroid;
Eosinophils 1% however, some patients are on a steroid-sparing protocol.
Absolute neutrophil count 500/mm3 Management goals include balancing sufficient immune
suppression to prevent donor-specific antibody formation
and rejection, while also allowing the immune system to
Questions fight off infection. The patient is neutropenic at presenta-
tion and in addition to his immunocompromised state,
Answer the following questions using the details provided. he has other risk factors for development of urinary tract
infections including a complex urologic system requiring
1 Based on the data, what are the differential diagnoses
intermittent catheterization to avoid urinary stasis.
and the most likely diagnosis for this child?
2 What is the initial management for this child?
3 Which antibiotics would be most appropriate and why? What is the initial management for this child?
Early identification of shock and early interventions are
critical. Because the patient is in the clinic but unstable,
a plan for transfer to a higher level of care should be imme-
Rationale and Evidence-based Practice
diately implemented by activating the code team or calling
Explanation 911. While awaiting help, this 6-year-old needs oxygen
even if he is not having respiratory symptoms. Oxygen
Based on the data, what are the differential
via a non-rebreather mask is recommended. A peripheral
diagnoses and the most likely diagnosis for
intravenous catheter or intraosseous access should be
this child?
obtained and isotonic crystalloid 20 mL/kg bolus should
Based on the presentation and laboratory results, the differ- be rapidly administered using a push–pull method. Reas-
ential diagnoses for this child include viral illness, hypovo- sessment following the fluid administration is important
lemic shock, and septic shock with acute kidney injury. as fluid can worsen clinical status in some cases, such
The most likely diagnosis for this child is hypovolemic as cardiogenic shock and some instances of distribu-
and septic shock due to urosepsis. tive shock.
The American Heart Association defines shock as a The patient will be seen briefly in the emergency depart-
condition in which peripheral tissues and end organs ment, where broad-spectrum antibiotics should be admi-
do not receive adequate oxygenation and nutrients. Pedia- nistered without delay. If shock is not improving after
tric shock is further delineated into compensated, or fluid administration, inotropic support should be initiated.
Downloaded from https://onlinelibrary.wiley.com/doi/ by National Institute Of Standard, Wiley Online Library on [06/03/2024]. See the Terms and Conditions (https://onlinelibrary.wiley.com/terms-and-conditions) on Wiley Online Library for rules of use; OA articles are governed by the applicable Creative Commons License
Chapter 93 Complication in a 6-Year-Old with a Kidney Transplant 393

Based on the previous history, and current sepsis picture, coverage with methicillin-resistant organism coverage
this child would also be admitted to the critical care unit and Gram-negative coverage for potential Pseudomonas
for continuous monitoring, especially of cardiorespiratory or E. coli. The patient appropriately received vancomycin
status. Because of his chronic steroid requirement, this 15 mg/kg and cefepime 50 mg/kg while awaiting culture
child is also at risk for adrenal insufficiency and should results. Vancomycin should be dosed by levels to avoid
be given stress-dose steroids comprising hydrocortisone nephrotoxicity, with first level obtained prior to the third
2 mg/kg per dose (100 mg maximum). dose. Cefepime is dosed every 8 hours.
In the pediatric intensive care unit, a collaborative
approach to care is important. Consult with infectious dis-
Which antibiotics would be most appropriate
ease and continued follow-up with the nephrology team
and why?
are indicated to determine continued management. Based
As the patient is toxic in appearance on arrival to the on pediatric sepsis guidelines, laboratory studies include a
clinic, broad-spectrum parenteral antibiotics are indicated. complete blood count, serum electrolytes with glucose,
Escherichia coli is the most common organism seen in blood culture, blood gas, serum lactate, liver function tests,
urinary tract infections, followed by Klebsiella, Proteus coagulation studies, C-reactive protein along with urinal-
spp., Staphylococcus saprophyticus, and Staphylococcus ysis with culture and chest radiograph. Serum lactate is
aureus. Children with complex urologic anatomy and those most important as an indicator of inadequate oxygen deliv-
requiring frequent catheterization experience Enterococcus ery, which is suggestive of septic shock. For this patient, a
and Pseudomonas urinary tract infections more commonly. renal ultrasound should be obtained, although stabiliza-
To cover the potential infectious organisms for this tion of the patient is the most important priority at
patient, best practice would include Gram-positive this point.

Further Reading
Bilgutay, A.N., Roth, D.R., Gonzales, E.T., et al. (2016). Porat, A. and Kesler, S. (2018). Urosepsis. In: StatPearls.
Posterior urethral valves: risk factors for progression to renal Treasure Island, FL: StatPearls Publishing. Available
failure. Journal of Pediatric Urology 12(3): 179e1–179e7. doi: from https://www.ncbi.nlm.nih.gov/books/
10.1016/j.jpurol.2015.10.009. NBK482344/
Plunket, A. and Tong, J. (2015). Sepsis in children. British
Medical Journal 350: h3017. doi: 10.1136/bmj.h3017.
Downloaded from https://onlinelibrary.wiley.com/doi/ by National Institute Of Standard, Wiley Online Library on [06/03/2024]. See the Terms and Conditions (https://onlinelibrary.wiley.com/terms-and-conditions) on Wiley Online Library for rules of use; OA articles are governed by the applicable Creative Commons License
395

94

Acute Hypertension and Seizures in a 17-Year-Old Male


Alison Hewitt Torres
Johns Hopkins Children’s Center, Baltimore, MD, USA

A 17-year-old male with no significant medical problems Family History


presents with hypertension, new-onset seizure, and left vis-
ual field abnormality for the past week. No family history of neurologic problems including sei-
zures or hypertension. Both mother and maternal grand-
mother have histories of UTI and the maternal
grandmother also has renal failure secondary to liver fail-
History of Present Illness ure. Mother has a history of kidney stones and malignant
hyperthermia with general anesthesia.
This teen presented to the emergency department (ED) via
the emergency medical services after having his first seizure
while standing in line at the grocery store. When parame-
dics arrived, his blood pressure ranged in the 250s mmHg Current Status
systolic and 150s mmHg diastolic. The patient has no his-
tory of seizures and no history of elevated blood pressure. On arrival to ED, the patient is alert and reporting a head-
He is a competitive swimmer, and has noticed muscle ache, which he attributes to hitting his head during seizure,
cramps and soreness over the last few weeks. He reports and does have a hematoma on his left scalp. He is alert,
that he has a white spot in the left visual field of his left oriented, and cooperative with the physical examination.
eye for the last week. Pupils are equal, round, reactive to light, and with extrao-
cular movements intact. Cranial nerves III–XII intact, nor-
mal gait, 5/5 strength in all four extremities. Mucous
membranes are moist. Heart rate is regular with normal
Past Medical History sinus rhythm. No murmur on auscultation. Lungs are clear
to auscultation bilaterally with no adventitious sounds
The patient’s past history includes failure to thrive (FTT) as appreciated. Abdomen is soft, non-tender, and non-
a toddler, attributed to caloric deficiency, urinary tract distended with no hepatosplenomegaly. Extremities are
infection (UTI) or kidney infection as a toddler, for which without edema. Peripheral pulses are easily palpable and
mother reports he was treated with antibiotics. He had no capillary refill is less than 2 seconds.
further problems. Vital signs are as follows: heart rate 76 bpm, respiratory
rate 20 breaths per minute, blood pressure 254/157 mmHg
right arm, and oxygen saturation 100% on room air.
Results of complete blood count, basic metabolic panel,
Past Surgical History and urinalysis are shown in Tables 94.1–94.3.
Chest X-ray shows no focal consolidation, pleural effu-
No prior surgeries. sion, or pneumothorax. There is left basilar subsegmental

Cases in Pediatric Acute Care: Strengthening Clinical Decision Making, First Edition. Edited by Andrea M. Kline-Tilford and Catherine M. Haut.
© 2020 John Wiley & Sons Ltd. Published 2020 by John Wiley & Sons Ltd.
Downloaded from https://onlinelibrary.wiley.com/doi/ by National Institute Of Standard, Wiley Online Library on [06/03/2024]. See the Terms and Conditions (https://onlinelibrary.wiley.com/terms-and-conditions) on Wiley Online Library for rules of use; OA articles are governed by the applicable Creative Commons License
396 Cases in Pediatric Acute Care

Table 94.1 Basic metabolic panel. Questions


Sodium 139 mEq/L
Answer the following questions using the details provided.
Potassium 3.8 mEq/L
Chloride 100 mEq/L 1 Based on the data given, what are the differential diag-
Carbon dioxide 16 mEq/L noses and the most likely diagnosis for this child?
2 Would any other diagnostic studies be recommended?
Blood urea nitrogen 102 mg/dL
3 What specialties should be consulted?
Creatinine 8 mg/dL
4 What is the best way to manage emergent hypertension
Glucose 122 mg/dL and what dosing would be used?
Calcium 7.6 mg/dL

Table 94.2 Complete blood count.


Rationale and Evidence-based Practice
White blood cell count 8650/mm 3 Explanation
Hemoglobin 8.3 g/dL
Based on the data given, what are the differential
Hematocrit 24.2%
diagnoses and the most likely diagnosis for
Platelet count 167 000/mm3
this child?
Neutrophils 71.1%
Lymphocytes 10.5%
This teenage patient is presenting with a hypertensive
emergency. The differential diagnoses include essential
Monocytes 7.5%
hypertension and secondary hypertension, although sec-
Eosinophils 2.7%
ondary hypertension is more likely given the severity of
blood pressure elevation. The differential diagnosis of sec-
ondary hypertension in children more commonly includes
Table 94.3 Urinalysis. renovascular disease or renal parenchymal disease. Other
less prevalent causes of hypertensive emergency in adoles-
Color Clear, yellow cence include secondary hypertension due to aortic coarc-
Specific gravity 1.008 tation, endocrine conditions such as pheochromocytoma or
pH 6.0 Cushing disease, or secondary hypertension related to rec-
Nitrite Negative reational drugs (see Table 94.4).
Ketones Negative The most likely diagnosis is hypertensive emergency sec-
Urobilirubin Negative ondary to renal parenchymal disease given the significantly
elevated serum creatinine level and distant history of UTI
Leukocytes Negative
and FTT. The American Academy of Pediatrics (AAP)
Protein 3+
defines hypertension as a blood pressure ≥95th percentile
Glucose Negative
for age, height, and sex, or above 130/80 mmHg for children
Blood Small 13 years and older. Primary or essential hypertension is
White blood cells 51 per high power field hypertension without a known etiology and is the most
Red blood cells 21 per high power field prevalent cause of hypertension is children over 6 years
Hyaline casts 0 old. Secondary hypertension means that there is an under-
Granular casts 0 lying cause for the hypertension (cardiac, renal, endocrine,
Bacteria Occasional
neurologic, vascular, ingestions, etc.). Hypertensive emer-
gency is defined as hypertension with end-organ damage
(abnormalities in the heart, eyes, kidneys and/or brain).
Prompt recognition and treatment of hypertensive emer-
atelectasis, mild cardiomegaly, and no acute osseous abnor- gency is critical in order to avoid further end-organ damage.
mality. Head computed tomography (CT) shows no evi- Treatment of hypertensive emergency should be targeted at
dence of acute intracranial process; left parietal scalp gradually reducing mean arterial pressure by no more than
swelling noted. 25% in the first hour and then gradually over the next
Toxicology screen is negative. 24–48 hours.
Downloaded from https://onlinelibrary.wiley.com/doi/ by National Institute Of Standard, Wiley Online Library on [06/03/2024]. See the Terms and Conditions (https://onlinelibrary.wiley.com/terms-and-conditions) on Wiley Online Library for rules of use; OA articles are governed by the applicable Creative Commons License
Chapter 94 Acute Hypertension and Seizures in a 17-Year-Old Male 397

Table 94.4 Causes of hypertension related to age.

Age group Cause of hypertension Associated findings

Newborn Renal: anomalies of the kidneys and urinary tract, vascular Elevated serum creatinine, abnormalities on
stenosis, thrombosis ultrasound
Cardiac: coarctation of the aorta Discrepant four-extremity blood pressures/perfusion
Endocrine: primary hyperaldosteronism, congenital adrenal Tachycardia, palpitations, sweating, elevated serum
hyperplasia, pheochromocytoma, Cushing disease cortisol levels
Preschool Renal: congenital anomalies of kidneys and urinary tract, Elevated serum creatinine, abnormalities on
through age renal parenchymal disease, vascular stenosis, thrombosis ultrasound
6 years
Cardiac: coarctation of the aorta Discrepant four-extremity blood pressures/perfusion
Enodcrine: pheochromocytoma, Cushing disease Tachycardia, palpitations, sweating, elevated serum
cortisol levels
School age Essential hypertension Obesity + family history
(6–10 years)
Renal: congenital anomalies of kidneys and urinary tract, Elevated serum creatinine, abnormalities on
renal parenchymal disease, vascular stenosis, thrombosis ultrasound
Enodcrine: pheochromocytoma, Cushing disease Tachycardia, palpitations, sweating, elevated serum
cortisol levels
Adolescence Essential hypertension Obesity + family history
Renal: congenital anomalies of kidneys and urinary tract, Elevated serum creatinine, abnormalities on
renal parenchymal disease, vascular stenosis, thrombosis ultrasound
Medications and recreational drugs: stimulants, oral Symptoms are related to medication or drug of
contraceptives, corticosteroids, cocaine, amphetamines exposure
Endocrine: pheochromocytoma, Cushing disease Tachycardia, palpitations, sweating, elevated serum
cortisol levels

Obtaining a detailed history and physical examination count, thyroid studies, metanephrines, additional imaging
will help guide the hypertension evaluation. Physical exam- of the renal vasculature such as CT or magnetic resonance
ination should include height, weight, calculated BMI, and angiogram, and sleep study based on the individual patient.
percentiles for age. Children with chronic illness often have An eye examination and echocardiogram can help to eval-
impaired growth, while obesity is associated with primary uate for end-organ damage from hypertension. In this case
hypertension. With stage II hypertension, or systolic blood study, renal/bladder ultrasound was obtained later and
pressure ≥95th percentile + 12 mmHg or ≥140/90 mmHg confirmed the diagnosis of hypertensive emergency second-
(whichever is lower), physical examination should include ary to renal parenchymal disease as it showed small echo-
both upper extremity blood pressure measurements and genic kidneys with bilateral hydroureteronephrosis and
one leg blood pressure measurement to evaluate for aortic thickened bladder wall. He was diagnosed with end-stage
coarctation. kidney disease due to congenital anomalies of the kidney
and urinary tract.
The AAP 2017 high blood pressure guidelines recom-
Would any other diagnostic studies be
mend obtaining a renal ultrasound for children less than
recommended?
6 years of age or those (regardless of age) with abnormal
Work-up for hypertension includes tests to evaluate for sec- urinalysis or renal function. Renal parenchymal disease
ondary causes as well as to look for target organ damage. combined with renal vascular disease account for 63–74%
The work-up should be guided by the age of the patient, of secondary hypertension in children and adolescents.
the severity of hypertension, associated comorbid condi-
tions, and findings on history and physical examination.
What specialties should be consulted?
General work-up typically includes a urinalysis, compre-
hensive metabolic panel, and renal ultrasound with Dop- Specialty consultation is recommended for pediatric
pler. Additional work-up may include complete blood patients presenting with stage II hypertension. Nephrology
Downloaded from https://onlinelibrary.wiley.com/doi/ by National Institute Of Standard, Wiley Online Library on [06/03/2024]. See the Terms and Conditions (https://onlinelibrary.wiley.com/terms-and-conditions) on Wiley Online Library for rules of use; OA articles are governed by the applicable Creative Commons License
398 Cases in Pediatric Acute Care

and cardiology are both specialty services that manage plan for supportive care for potential life-threatening com-
hypertension. Depending on the suspected etiology, one plications such as seizures.
service may be consulted preferentially over the other. With Medications used in a hypertensive crisis include the
vision disturbance and concern for target organ damage, an following.

••
ophthalmology consultation is also recommended.
Calcium channel blocker: nicardipine 1–3 μg/kg per min.
What is the best way to manage emergent Beta-blocker: esmolol 125–500 μg/kg per min or labetalol
0.25–3 mg/kg per hour.


hypertension and what dosing would be used?
Direct vasodilator: diazoxide 1–3 mg/kg every 5–15 min-
Recommended management for hypertensive emergency is utes or hydralazine 0.2–0.6 mg/kg bolus.
aimed at reducing further end-organ damage by decreasing
blood pressure by no more than 25% in the first hour, with Longer-term initial management of hypertension
gradual reduction over the next 24–48 hours. Lowering includes pharmacologic therapy with an angiotensin-
blood pressure is done gradually as the vascular beds have converting enzyme (ACE) inhibitor, angiotensin receptor
accommodated to the current high pressure and lowering blocker, long-acting calcium channel blocker, or a thiazide
too quickly will cause hypoperfusion and ischemic injury. diuretic. Beta-blockers are not recommended as first-line
This can best be accomplished in the pediatric intensive treatment based on adverse effects and adult data that
care unit by a continuous infusion of a calcium channel there is a lack of effect with these agents. There are other
blocker or beta-blocker with close observation often requir- recommendations by the AAP for managing children
ing intra-arterial blood pressure monitoring. In addition to with other associated problems, such as diabetes and
the goal of reducing blood pressure, it is also important to proteinuria.

Further Reading
Flynn, J.T., Kaelber, D.C., Baker-Smith, C.M., et al. (2017). Nephrology 34(12): 2523–2537. doi: 10.1007//s00467-018-
Clinical practice guideline for screening and management of 4092-2.
high blood pressure in children and adolescents. Pediatrics Singh, D., Akingbola, O., Yosypiv, I., and El-Dahr, S. (2012).
140(3): e20171904. doi: 10.1542/peds.2017-1904. Emergency management of hypertension in children.
Seeman, T., Hamdani, G., and Mitsnefes, M. (2018). International Journal of Nephrology 2012: 420247. doi:
Hypertensive crisis in children and adolescents. Pediatric 10.1155/2012/420247.
Downloaded from https://onlinelibrary.wiley.com/doi/ by National Institute Of Standard, Wiley Online Library on [06/03/2024]. See the Terms and Conditions (https://onlinelibrary.wiley.com/terms-and-conditions) on Wiley Online Library for rules of use; OA articles are governed by the applicable Creative Commons License
399

95

Jittery 2-Week-Old Infant


Christopher D. Newman
Children’s Hospital Colorado, Aurora, CO, USA

A 2-week-old infant female is brought into your emergency Current Status


department because she is “not acting right.”
In the examination room, she is lying in mother’s arms. She
is pale, diaphoretic, and tachypneic with a respiratory rate
of 55 breaths per minute. Her remaining vital signs are
History of Present Illness
temperature 35.4 C (95.7 F), heart rate 190 bpm, blood
pressure 80/40 mmHg, and weight 3.5 kg. The patient’s
The infant was initially healthy on coming home from the
fontanelle is slightly sunken, and mucous membranes are
hospital following her birth, but over the last several days
dry. Lungs are clear to auscultation; cardiac examination
has been increasingly “jittery,” with difficulty consoling,
is normal except for tachycardia and 1+ peripheral pulses.
rapid breathing and sweating. Until yesterday, she was
Abdomen is soft, non-tender, non-distended without hepa-
taking bottles well, but today she has not been latching
tosplenomegaly. Extremities are cool, but without defect or
on to the bottle. Approximately an hour ago, her eyes
tenderness. Her deep tendon reflexes are 2+ bilaterally. As
became unfocused and the patient’s mother decided to
the physical examination is concluding, her eyes deviate to
bring her to the emergency department (ED).
the left and she has a series of tonic–clonic jerks in both
upper and lower extremities. The movements resolve after
15 seconds. A blood glucose level is obtained immediately
Past Medical History with a result of 34 mg/dL.

She was born at 39 weeks to a healthy mother and had an


uncomplicated delivery. She was discharged after 24 hours Questions
and received her hepatitis vaccine prior to discharge. She is
scheduled for her 2-week well-child visit tomorrow. Answer the following questions using the details provided.
1 Based on the presentation, what are the differential
diagnoses for this patient’s hypoglycemia?
Past Surgical History 2 What, if any, additional laboratory tests should be
obtained before initiating treatment?
She has had no surgeries. 3 What is the appropriate initial treatment for this
patient?
Continuation of case: Following glucose administration,
Family History the infant is more awake and active. Her repeat blood
glucose is 110 mg/dL. The laboratory results shown in
The infant’s maternal grandmother had gestational diabe- Table 95.1 are also obtained. Additionally, a glucagon stim-
tes, otherwise there is no significant family history of pre- ulation test is completed, which demonstrates a rise of more
natal or childhood illness. than 30 mg/dL in serum glucose in the first 30 minutes.

Cases in Pediatric Acute Care: Strengthening Clinical Decision Making, First Edition. Edited by Andrea M. Kline-Tilford and Catherine M. Haut.
© 2020 John Wiley & Sons Ltd. Published 2020 by John Wiley & Sons Ltd.
Downloaded from https://onlinelibrary.wiley.com/doi/ by National Institute Of Standard, Wiley Online Library on [06/03/2024]. See the Terms and Conditions (https://onlinelibrary.wiley.com/terms-and-conditions) on Wiley Online Library for rules of use; OA articles are governed by the applicable Creative Commons License
400 Cases in Pediatric Acute Care

Table 95.1 Laboratory studies. free fatty acids, the absence of ketone bodies as an energy
source may play a role. Disorders of carbohydrate metabo-
Insulin 26 μIU/mL
lism can impact metabolism of glycogen, galactose and
C-peptide 5 ng/mL fructose. This group includes the glycogen storage diseases
β-Hydroxybutyrate 0.3 mmol/L (such as glycogen synthetase deficiency), disorders of glyco-
Free fatty acids 0.3 mmol/L sylation (such as phosphoglucomutase 1 deficiency) and
Acylcarnitine profile Normal disorders of gluconeogenesis (such as pyruvate carboxylase
Lactate 1 mmol/L deficiency). Although disorders of amino acid metabolism,
Ammonia 80 μg/dL
such as methylmalonic acidemia and propionic acidemia,
typically present with an episode of profound metabolic aci-
Urine organic acids Normal
dosis, hypoglycemia may be present due to associated liver
dysfunction or protein malnutrition.

4 What is the most likely cause of the patient’s


What, if any, additional laboratory tests should be
hypoglycemia?
obtained before initiating treatment?
5 What is the first-line therapy for this patient’s etiology
of hypoglycemia? It is essential for diagnosing the cause of hypoglycemia that
laboratory testing be done on specimens obtained during
the state of hypoglycemia. In an unstable patient, the
patient’s safety comes first. However, ideally, the following
Rationale and Evidence-based Practice serum studies will be obtained prior to treatment of the
Explanation hypoglycemia: plasma glucose, C-peptide, β-hydroxybuty-
rate, free fatty acids, acylcarnitine profile, lactate, and
Based on the presentation, what are the ammonia. Urine organic acids should also be sent. These
differential diagnoses for this patient’s laboratory values can then be used to identify the most
hypoglycemia? likely cause of hypoglycemia from the differential
Hypoglycemia in infancy can generally be divided into diagnosis.
broad categories: inadequate glucose supply (either
through inadequate intake or due to defects in fatty acid What is the appropriate initial treatment for this
oxidation, carbohydrate metabolism or amino acid metab- patient?
olism) or increased glucose utilization (such as hyperinsu- In older children who are alert enough to swallow safely,
linism or some ingestions). Infants will rapidly exhaust 10–20 g of rapidly absorbed glucose, such as honey, table
their glycogen stores without glucose intake and are there- sugar, glucose tablets or glucose gel, may be administered.
fore less tolerant of fasting. Additionally, illness that In infants such as this patient, the ideal treatment is intra-
increases metabolic demand can exhaust glucose supply venous administration of dextrose 0.25 g/kg (maximum
even in a feeding infant if the infant cannot keep up with 25 g). In an unstable patient when intravenous access can-
the increased metabolic demand. Common ingestions asso- not be achieved, administer glucagon 0.03 mg/kg intramus-
ciated with hypoglycemia can include ethanol, salicylates, cularly. Glucagon will effectively treat hypoglycemia due to
sulfonylureas, and beta-blockers. increased glucose utilization, but may not treat other causes
Hyperinsulinism often presents in the first few weeks of and the effect may be transient. Efforts should continue to
life. The most common cause is congenital hyperinsulin- establish intravenous access and administer intravenous
ism, although insulinomas and exogenous insulin adminis- dextrose.
tration are also possibilities. In these conditions, insulin is
secreted out of proportion to glucose level and the usual
What is the most likely cause of the patient’s
negative feedback loop that disrupts insulin production
hypoglycemia?
when blood glucose levels fall below 60 mg/dL is inacti-
vated. Fatty acid oxidation disorders can include carnitine The elevated insulin level and appropriate response to glu-
deficiency, fatty acid transport defects, and defects of beta- cagon stimulation test are most consistent with hyperinsu-
oxidation enzymes. The most common disorder is medium- linemia. Low ketone production (as evidenced by lower
chain acyl-CoA dehydrogenase deficiency. The exact mech- β-hydroxybutyrate) also supports this diagnosis. The time
anism of hypoglycemia is not understood, but since ketone of presentation and the presence of seizure are most con-
bodies cannot be manufactured without mobilization of sistent with a focal hyperinsulinemia. In one study, 50%
Downloaded from https://onlinelibrary.wiley.com/doi/ by National Institute Of Standard, Wiley Online Library on [06/03/2024]. See the Terms and Conditions (https://onlinelibrary.wiley.com/terms-and-conditions) on Wiley Online Library for rules of use; OA articles are governed by the applicable Creative Commons License
Chapter 95 Jittery 2-Week-Old Infant 401

of patients with focal hyperinsulinemia presented with be low. Additionally, in most circumstances these defects
seizure. are associated with hepatomegaly, which this patient does
Because C-peptide and insulin are secreted in equal pro- not have. Disorders of organic or amino acid metabolism
portions by the beta cells, the presence of an elevated usually have ketosis associated with hypoglycemia. Addi-
C-peptide is most consistent with endogenous insulin pro- tionally, these disorders are commonly associated with hep-
duction. A low C-peptide would raise concern for exoge- atomegaly and alterations in the urine organic acid profile.
nous insulin administration, either accidental or
intentional. Although ketone bodies would also be low in
fatty acid oxidation defects, free fatty acid levels should What is the first-line therapy for this patient’s
be high and there should be minimal or no response to etiology of hypoglycemia?
the glucagon stimulation test. Disorders of carbohydrate
The first-line therapy for congenital hyperinsulinemia is
metabolism may present with different values based on
diazoxide, which blocks sulfonylurea receptors in the beta
the specific defect. For example, glucose 6-phosphate dehy-
cells. This in turn opens ATP-dependent potassium chan-
drogenase is often associated with a lactic acidosis. How-
nels, thus decreasing insulin release. Diazoxide 5–15 mg/kg
ever, all will have minimal response to the glucagon
daily is administered in three divided doses.
stimulation test and both insulin and C-peptide levels will

Further Reading
Gandhi, K. (2017). Approach to hypoglycemia in infants and 223 cases. Journal of Clinical Endocrinology and Metabolism
children. Translational Pediatrics 6(4): 408–420. 98(11): E1786–E1789.
Hoe, F.M. (2008). Hypoglycemia in infants and children. Thornton, P.S., Stanley, C.A., De Leon, D.D., et al. (2015).
Advances in Pediatrics 55: 367–384. Recommendations from the Pediatric Endocrine Society for
Lord, K., Dzata, E., Snider, K.E., Gallagher, P.R., and De León, evaluation and management of persistent hypoglycemia in
D.D. (2013). Clinical presentation and management of neonates, infants, and children. Journal of Pediatrics 167(2):
children with diffuse and focal hyperinsulinism: a review of 238–245.
Downloaded from https://onlinelibrary.wiley.com/doi/ by National Institute Of Standard, Wiley Online Library on [06/03/2024]. See the Terms and Conditions (https://onlinelibrary.wiley.com/terms-and-conditions) on Wiley Online Library for rules of use; OA articles are governed by the applicable Creative Commons License
403

96

Child with Leg and Back Pain


Jill Marks1,2
1
University of Colorado Pediatric Neurology, Boulder, CO, USA
2
Faculty Acute Care Pediatric Nurse Practitioner Program, Rush University College of Nursing, Chicago, IL, USA

A 12-year-old male with no past medical history is admitted Family History


to the emergency department (ED) with a 3-day history of
leg and back pain. Mother has a history of anxiety and a thyroid disorder.
Maternal grandfather died of stroke at age 55 years.

History of Present Illness Current Status


He was in his usual state of health until 3 days prior to pres- Arrives to the ED by wheelchair. He is able to stand up and
entation, when he noted bilateral leg pain and fatigue. He move on to the gurney on his own with some assistance. He
complained to his parents who administered ibuprofen and is a pleasant alert child in no respiratory distress, with lungs
advised he drink extra water. The morning prior to presen- clear and equal. There is a normal S1 and S2 without appre-
tation, he was able to go to school, but complained of ciable murmur. His abdomen is soft and non-distended.
feeling tired so came home early. Today, he continues to Skin is without rashes. No joint abnormalities are noted.
complain of leg and lower back pain with fatigue. He Neurologic examination reveals cranial nerves II–X with-
was able to get up and walk to the car, but needed out deficits. There is no dysmetria or dysdiadochokinesia.
assistance. Caregivers and patient deny fever, vomiting, His strength is 5/5 in the neck, head and upper extremities
diarrhea, or cold symptoms. He did have a cough and runny and 3/5 in the lower extremities bilaterally. Reflexes are 1+
nose a few weeks ago, as did the rest of the family. He bilaterally at biceps, triceps, and brachioradialis, and absent
received an influenza vaccination 1 month ago. in the patella and Achilles. Sensory examination is unre-
markable. He has an absent Babinski. Gait is not ataxic.
Rectal tone is normal.
Results of basic metabolic profile and cerebrospinal fluid
Past Medical History
(CSF) analysis are shown in Tables 96.1 and 96.2.
He was born at 37 weeks and was discharged 3 days after
delivery with parents. He has a history of anxiety and
depression. He takes sertraline 25 mg daily and sees a psy- Questions
chologist twice a month. His development has been
typical. Answer the following questions using the details provided.
1 Based on the information given, what are the differential
diagnoses and the most likely diagnosis for this child?
Past Surgical History 2 Would you obtain any other diagnostic studies?
3 What is the optimal treatment for this neurologic con-
He has had no prior surgeries or hospitalizations. dition and how should it be ordered?

Cases in Pediatric Acute Care: Strengthening Clinical Decision Making, First Edition. Edited by Andrea M. Kline-Tilford and Catherine M. Haut.
© 2020 John Wiley & Sons Ltd. Published 2020 by John Wiley & Sons Ltd.
Downloaded from https://onlinelibrary.wiley.com/doi/ by National Institute Of Standard, Wiley Online Library on [06/03/2024]. See the Terms and Conditions (https://onlinelibrary.wiley.com/terms-and-conditions) on Wiley Online Library for rules of use; OA articles are governed by the applicable Creative Commons License
404 Cases in Pediatric Acute Care

Table 96.1 Basic metabolic profile. (AIDP), acute motor axonal neuropathy and acute motor
and sensory axonal neuropathy (AMAN/AMSAN), multiple
Sodium 140 mEq/L
sclerosis, acute disseminated encephalomyelitis, and trans-
Potassium 4 mEq/L verse myelitis; (ii) spinal injury or compression (vascular
Chloride 100 mEq/L accident, mass); or (iii) infection (encephalitis, meningitis,
Carbon dioxide 23 mEq/L radiculitis, myelitis, epidural abscess). Tick paralysis and
Blood urea nitrogen 10 mg/dL myasthenia gravis can mimic GBS, as can a variety of met-
Creatinine 0.5 mg/dL abolic (thyroid) and electrolyte abnormalities. Myopathy
Glucose 80 mg/dL
and functional neurologic disorder are less likely but should
be considered. Infantile botulism should be considered in
Calcium 10.1 mg/dL
appropriate age groups.
Albumin 4.2 mg/dL
Protein 7.9 mg/dL
Aspartate aminotransferase 40 U/L Would you obtain any other diagnostic studies?
Alanine aminotransferase 42 U/L Further diagnostic studies should be completed to narrow
Bilirubin 0.1 mg/dL the differential. A lumbar puncture should be performed
Alkaline phosphatase 110 U/L and studies sent, including cell counts, glucose, protein
and culture. In patients with concern for elevated intracra-
nial pressure (ICP), head computed tomography (CT)
should be performed first. MRI of the brain and spinal cord
Table 96.2 Cerebrospinal fluid analysis. with and without contrast should be performed as well.
The most likely diagnosis for this child is GBS. CSF
White blood cell count 2/mm3
studies can help confirm the diagnosis of GBS but GBS is
Neutrophils 20%
a diagnosis based on clinical findings so these will be the
Lymphocytes 70% most important. In GBS, CSF will display a classic albumi-
Red blood cell count 100/mm3 nocytologic dissociation (elevated protein level), and nor-
Protein 80 mg/dL mal cell count with lymphocytic predominance. Early in
Glucose 68 mg/dL the course, the CSF profile may be normal, but by the sec-
Opening pressure 14 cmH2O ond week of symptoms more than 90% of patients will have
elevated CSF protein. MRI with gadolinium may show
enhancement of the peripheral nerve roots and cauda
equina, although it is notable that this is not specific and
4 What education would you provide to the family can also be seen following lumbar puncture and with
regarding disease time course? chronic inflammatory neuropathies. Specific anti-
ganglioside antibody testing can be performed as an
adjunct to diagnosis, but is only identifiable in approxi-
mately 50% of children. GBS is a clinical diagnosis and
treatment should be initiated immediately when there is
Rationale and Evidence-based Practice
a high degree of clinical suspicion. Further diagnostic stud-
Explanation ies should be completed to narrow the differential.
GBS is an acute inflammatory polyneuropathy of the
Based on the information given, what are the
peripheral nervous system and the most common cause
differential diagnoses and the most likely
of acute flaccid paralysis in children. Initial symptoms
diagnosis for this child?
include vague, poorly localized pain in the neck, back, but-
Differential diagnoses for patients with nervous system dis- tocks, and lower extremities (up to 50% of patients) fol-
order should include disease localization (in this case can lowed by rapid progression of symmetric ascending
broadly include pathology of the central nervous system, flaccid paralysis. Although absent (or very minimal)
peripheral nervous system, neuromuscular junction, nerve, reflexes are often thought of as a hallmark of GBS, reflexes
and muscle). The differential can be thought of in a may be preserved early in the course. Likewise, although
systematic manner: (i) demyelinating and post-infectious often described as symmetric, clinical cases can have some
conditions including Guillain–Barré syndrome (GBS), degree of asymmetry though not unilaterally. Children are
acute inflammatory demyelinating polyradiculoneuropathy generally afebrile and well appearing. Symptoms usually
Downloaded from https://onlinelibrary.wiley.com/doi/ by National Institute Of Standard, Wiley Online Library on [06/03/2024]. See the Terms and Conditions (https://onlinelibrary.wiley.com/terms-and-conditions) on Wiley Online Library for rules of use; OA articles are governed by the applicable Creative Commons License
Chapter 96 Child with Leg and Back Pain 405

peak or plateau around 7–14 days, although for some immune-mediated response to a trigger (infection, surgery,
patients it can be up to 4 weeks, and recovery occurs immunization) secondary to molecular mimicry. B and
2–4 weeks later. Symptoms may progress to include bulbar T cells produce antibodies directed at antigenic proteins
weakness, respiratory failure, and autonomic nervous of peripheral nerves. Peripheral nerve components are
system abnormalities (blood pressure instability, tachycar- attacked by self, secondary to molecular mimicry. Several
dia, pupillary abnormalities, incontinence, and altered subtypes of GBS exist differentiated by the pattern of neu-
sweating). rologic deficits that occur, secondary to the variable gangli-
oside expression in different neural tissues. AIDP is the
Continuation of case: Magnetic resonance imaging (MRI)
form classically thought of and accounts for about 75% of
of the brain with and without contrast is normal. MRI of the
cases. AMAN is most often related to cases associated with
spine with and without contrast shows enhancement of the
Campylobacter jejuni, thought to be related to the resem-
ventral and dorsal nerve roots in the thoracic and lumbar
blance of of C. jejuni expressed lipo-oligosaccharides to
spine and cauda equina.
the carbohydrate component of gangliosides. Miller–Fisher
syndrome is a subtype characterized by ataxia, areflexia,
What is the optimal treatment for this neurologic
and ophthalmoplegia without weakness, though it can
condition and how should it be ordered?
progress to involve multiple cranial nerves leading to signif-
Optimal treatment for GBS includes intravenous gamma- icant facial paralysis, bulbar weakness and include limb
globulin (IVIG), typically 2 g/kg divided over 2–5 days, and truncal weakness at times as well.
and plasma exchange, both of which shorten the disease Although any virus or infection may trigger GBS, specific
course and time to independent ambulation. They are most infections implicated in this disease include Mycoplasma,
effective when given within 2 weeks of symptom onset. C. jejuni, pneumococcus, and Haemophilus influenzae.
Although the use of these treatments shortens disease Post-vaccination cases have been known to occur, particu-
course, they are not associated with overall difference in larly related to the influenza vaccine, although they are rare
long-term outcome. Patients should be carefully monitored with risk of influenza morbidity outweighing risk of post-
for bulbar weakness, respiratory muscle weakness, and vaccination GBS.
autonomic complications. Hospital admission for continu-
ous cardiorespiratory and pulse-oximeter monitoring is Continuation of case: Following evaluation in the ED, he
indicated. Patients with neuromuscular weakness will is admitted to the hospital. Blood gas testing reveals normal
not demonstrate respiratory distress, and respiratory failure carbon dioxide levels. Negative inspiratory force (NIF) and
in GBS can result from deteriorating diaphragmatic muscle vital capacity (VC) testing are in the normal range. Post-
strength. Measures of diaphragm function and gas void residual reveals more than 300 mL of urine remaining
exchange including NIF, VC, and exhaled carbon dioxide in the bladder. During the first week of hospitalization, his
should be monitored for patients with GBS. Hypoxia is a examination is notable for increased weakness of all extre-
late sign of respiratory failure in patients with GBS. Up mities, hypophonia, present but weak cough, dyspnea, and
to 20% of patients may require mechanical ventilation dur- continued areflexia. Although he does not have respiratory
ing the course of their illness. Indications for admission to distress, his blood gas evaluation reveals hyperacarbia and
the intensive care unit include rapid progression of weak- he demonstrates a steady decline in NIF testing. He is trans-
ness, flaccid quadriparesis, VC less than 20 mL/kg, bulbar ferred to the pediatric intensive care unit (PICU) for closer
palsy, and evidence of autonomic dysfunction. A swallow monitoring. In the PICU he requires frequent oral suction-
study should be obtained with any concern for bulbar ing for secretion management and is started on a cough
weakness. Other complications of GBS include constipa- assist device to promote pulmonary toileting. Despite air-
tion, urinary retention, pain, skin breakdown, and fatigue. way clearance efforts, on day 7 of hospitalization he
Rehabilitation service should be consulted and use of phys- requires intubation for respiratory failure and ventilator
ical, occupational, and speech therapy services should be support for 1 week. Clean intermittent catheterization
initiated. Effective pain management should be a priority, and a bowel management routine are required for effective
and psychosocial support for the patient and caregivers elimination patterns. Vital signs are notable for orthostatic
through social work and child life services is imperative hypotension, intermittent hypertension as well as alternat-
as loss of function for any duration is traumatic for the ing bradycardia and tachycardia. During this time, he has
entire family. periods of being withdrawn while other times he is very
GBS is generally thought of as a monophasic process with angry and not adherent with treatment plans. Sleep
low recurrence rate (2–5%). The mechanism for GBS hygiene is initiated and with the assistance of psychiatry
has not been clearly identified, but is most likely an his maintenance sertraline dose is increased. Consulting
Downloaded from https://onlinelibrary.wiley.com/doi/ by National Institute Of Standard, Wiley Online Library on [06/03/2024]. See the Terms and Conditions (https://onlinelibrary.wiley.com/terms-and-conditions) on Wiley Online Library for rules of use; OA articles are governed by the applicable Creative Commons License
406 Cases in Pediatric Acute Care

services included neurology, rehabilitation medicine, majority of patients have excellent neurologic outcomes,
speech therapy, physical therapy, psychiatry, occupational although a small number of patients will have residual neu-
therapy, child life, wound care team, and case manage- rologic deficits including weakness or persistent loss of
ment. He requires a 5-week hospital stay, including 2 weeks reflexes. Children may be transferred to a rehabilitation
on the inpatient rehabilitation service. center for continuing care. Once the child is home, it is
important to immunize with influenza vaccine annually.
What education would you provide to the family If GBS occurs within six weeks of a vaccination, it is impor-
regarding disease time course? tant to discuss with caregivers the risks and benefits of
future vaccination. The Centers for Disease Control Advi-
Disease course for GBS can be variable and the recovery
sory Council on Immunization Practices recommends pre-
phase can be prolonged. Complete recovery can take sev-
caution rather than absolute contraindication in this
eral months, especially if the patient has been intubated
clinical scenario.
and ventilated for a long period in the PICU. The vast

Further Reading
Centers for Disease Control. (2019). Vaccine Safety. Retrieved GBS/AIDP Foundation International (2012). GBS: An Acute
from https://www.cdc.gov/vaccinesafety/concerns/guillain- Care Guide For Medical Professionals. Narberth, PA: GBS/
barre-syndrome.html. AIDP Foundation International.
Centers for Disease Control. (2019). Vaccine Karalok, Z.S., Taskin, B.D., Yanginlar, Z.B., et al. (2018).
Recommendations and Guidelines of the ACIP: Guillain–Barré syndrome in children: subtypes and
Contraindications and Precautions. Retrieved from https:// outcome. Childs Nervous System 34(11): 2291–2297.
www.cdc.gov/vaccines/hcp/acip-recs/general-recs/ Ryan, M. (2013). Pediatric Guillian–Barré syndrome. Current
contraindications.html. Opinion in Pediatrics 25(6): 689–693. doi: 10.1097/
Chung, A. and Deimling, M. (2018). Guillain–Barré syndrome. MOP.0b013e328365ad3f.
Pediatrics in Review 39(1): 53–54. doi: 10.1542/pir.2017-0189.
Downloaded from https://onlinelibrary.wiley.com/doi/ by National Institute Of Standard, Wiley Online Library on [06/03/2024]. See the Terms and Conditions (https://onlinelibrary.wiley.com/terms-and-conditions) on Wiley Online Library for rules of use; OA articles are governed by the applicable Creative Commons License
407

97

Fussy, Tired Infant


Charlene Pringle
University of Florida, Gainesville, FL, USA

A 6-week-old infant is admitted to the emergency department Diagnostic studies and interventions are indicated emer-
(ED) with excessive crying, fatigue and staring episodes. gently. This critically ill infant is provided with high-flow
oxygen immediately and a basic metabolic panel, complete
blood count, arterial blood gases, and lactate level are
History of Present Illness obtained when intravenous access is gained. A chest radio-
graph is also obtained (Figure 97.1). The results for basic
Infant woke up “screaming” the prior evening and was
metabolic panel, complete blood count, and arterial blood
fussy, crying, grunting, and “staring off into space” prompt-
gas analysis are shown in Tables 97.1–97.3.
ing ED visit. Caregivers deny fever, emesis and diarrhea.
Over the past 2 weeks, the infant has been too tired to finish
its feedings, and the infant’s hands and feet have become Questions
cool when crying.
Answer the following questions using the details provided.

Past Medical History 1 Based on the data given, what are the differential diag-
noses and the most likely diagnosis for this infant?
Term infant, discharged with mother after birth. Immuni- 2 What additional diagnostic study is needed to confirm
zations are up to date. The infant has been followed by the your suspected diagnoses?
primary care provider and has gained weight appropriately. 3 What is the best way to support cardiac function and
increase cardiac output in this infant?
4 How would you determine if the therapy was effective?
Family History 5 What other therapies would be recommended at
this time?
Non-contributory. Lives with healthy parents and two older 6 Which additional diagnostic studies and consults are
half-siblings with no significant medical problems. needed?

Current Status
Rationale and Evidence-based Practice
On arrival to the ED, the infant is noted to be grunting, and
is bradypneic, lethargic and responding to pain by opening
Explanation
eyes only. The infant is mottled and gray with very weak
Based on the data given, what are the differential
femoral pulses. Her liver is palpable 2 cm below the right
diagnoses and the most likely diagnosis for this
costal margin.
infant?
Vital signs on arrival are temperature 35 C (95 F), heart
rate 150 bpm, blood pressure 35/25 mmHg, respiratory rate The differential diagnoses would include cardiomyopathy,
22 breaths per minute, weight 4.8 kg, and oxygen saturation myocarditis, structural heart disease, sepsis, inborn error of
100% on room air. metabolism, and liver failure. The most likely diagnosis for

Cases in Pediatric Acute Care: Strengthening Clinical Decision Making, First Edition. Edited by Andrea M. Kline-Tilford and Catherine M. Haut.
© 2020 John Wiley & Sons Ltd. Published 2020 by John Wiley & Sons Ltd.
Downloaded from https://onlinelibrary.wiley.com/doi/ by National Institute Of Standard, Wiley Online Library on [06/03/2024]. See the Terms and Conditions (https://onlinelibrary.wiley.com/terms-and-conditions) on Wiley Online Library for rules of use; OA articles are governed by the applicable Creative Commons License
408 Cases in Pediatric Acute Care

this infant is cardiomyopathy due to the child’s history, pre-


senting symptoms, and initial imaging.
Cardiomyopathy is defined as an abnormality of the ven-
tricular myocardium that cannot be explained by congeni-
tal heart disease or abnormal loading conditions.
Presentation can be highly variable, ranging from an inci-
dental finding to cardiogenic shock. Cardiogenic shock is a
medical emergency requiring prompt recognition, therapy,
and hospitalization. Management of cardiomyopathy with
cardiogenic shock involves medical therapies to treat the
acutely decompensated heart failure, including potential
mechanical support and/or heart transplantation. Subse-
quent therapy aims to reverse the chronic effects of ventric-
ular remodeling.
There are two main morphologic types of pediatric cardi-
omyopathy: hypertrophic and dilated. Other less common
types include restrictive, non-compaction, mixed, arrhyth-
Figure 97.1 Chest radiograph.
mia-induced, pacemaker induced, and post-exposure to
Table 97.1 Basic metabolic profile. drugs or toxins, including chemotherapy. Morbidity and
mortality are high: only about 20% of children with dilated
Sodium 130 mEq/L cardiomyopathy regain normal function within 2 years,
Potassium 7.8 mEq/L and 40% undergo transplantation or die. Infants with
Chloride 103 mEq/L hypertrophic cardiomyopathy have a 2-year mortality rate
Carbon dioxide 11 mEq/L
of 30%. Cardiomyopathy may present at any age; in infants,
the presentation is often similar to this case. Infants typi-
Blood urea nitrogen 12 mg/dL
cally compensate for heart failure for some time before
Creatinine 0.5 mg/dL
their condition requires emergent treatment.
Glucose 88 mg/dL
What additional diagnostic study is needed to
Table 97.2 Complete blood count. confirm your suspected diagnoses?

White blood cell count 12 000/mm3 Cultures and viral polymerase chain reaction (PCR) were
Hemoglobin 9.9 g/dL obtained and empiric antibiotics are started. An echocardi-
ogram is necessary based on the history, presentation, and
Hematocrit 34.8%
chest radiograph findings. Pediatric cardiology performed a
Platelet count 203 000/mm3
bedside echocardiogram that demonstrated a structurally
Neutrophils 25% normal heart with decreased biventricular function, ven-
Eosinophils 7% tricular hypertrophy, and moderate regurgitation across
Monocytes 18% multiple valves. The ejection fraction was assessed to be
Lymphocytes 41% severely decreased at 14%.
Bands 2%
Metamyelocytes 7% What is the best way to support cardiac function
and increase cardiac output in this infant?

Table 97.3 Arterial blood gas analysis. In addition to increasing cardiac output, the immediate sta-
bilization strategy also includes decreasing the workload of
pH 6.6 the heart. This will include intubation and mechanical ven-
PCO2 75 mmHg tilation, oxygen, and inotropic support aimed at increasing
PaO2 74 mmHg cardiac output, improving diastolic filling, and decreasing
Bicarbonate 8 mEq/L ventricular workload (i.e. afterload reduction). This infant
Base excess –30 was started on milrinone and epinephrine infusions. Milri-
none is a phosphodiesterase inhibitor that results in
Lactate 18 mmol/L
increased inotropy (improved contractility and stroke
Downloaded from https://onlinelibrary.wiley.com/doi/ by National Institute Of Standard, Wiley Online Library on [06/03/2024]. See the Terms and Conditions (https://onlinelibrary.wiley.com/terms-and-conditions) on Wiley Online Library for rules of use; OA articles are governed by the applicable Creative Commons License
Chapter 97 Fussy, Tired Infant 409

volume), lusitropy (improved diastolic relaxation and Which additional diagnostic studies and consults
decreased diastolic pressure to allow for improved filling) are needed?
and vasodilation (afterload reduction). Low-dose epineph-
A 12-lead electrocardiogram (EKG) is indicated and
rine is inotropic, as well as a modest vasodilator.
multiple abnormalities may be present, including biventri-
cular hypertrophy, arrhythmias, and first-degree atrioven-
How would you determine if the therapy tricular block. Infants with hypertrophic cardiomyopathy
was effective? should be evaluated for inborn errors of metabolism or
Frequent invasive and non-invasive hemodynamic moni- the infantile form of Pompe disease. In the case of Pompe
toring is indicated. Normalizing heart rate, increased blood disease, the 12-lead EKG will show very large QRS voltages
pressure and pulse pressure, and improvement in meta- as well as a very short PR interval. This infant should have a
bolic acidosis would be key markers of efficacy. genetics consultation. Additionally, this infant presented
with severe liver dysfunction due to acute heart failure
and cardiogenic shock; an abdominal ultrasound with Dop-
What other therapies would be recommended
pler flow would be recommended to ensure there was no
at this time?
thrombus from the low-flow state resulting in liver
Beta-blockade (if hemodynamically tolerated) may reduce dysfunction.
outflow tract obstruction and allow the heart more time
to fill and empty. Diuretics may be used if there is clinical
volume overload. If medical therapy is insufficient, Case Resolution
mechanical support may be needed. Options could include
veno-arterial extracorporeal membrane oxygenation or a This infant ultimately recovered her end-organ function
ventricular assist device, which is a pneumatically driven while being supported with a ventricular assist device while
device that can be used in infants (e.g. Berlin heart). awaiting heart transplantation.

Further Reading
Lee, T., Hsu, D., Kantor, P., et al. (2017). Pediatric Park, M.K. (ed.) (2014). Park’s Pediatric Cardiology for
cardiomyopathies. Circulation Research 121(7): 855–873. Practitioners. Philadelphia, PA: Elsevier Saunders.
Downloaded from https://onlinelibrary.wiley.com/doi/ by National Institute Of Standard, Wiley Online Library on [06/03/2024]. See the Terms and Conditions (https://onlinelibrary.wiley.com/terms-and-conditions) on Wiley Online Library for rules of use; OA articles are governed by the applicable Creative Commons License
411

98

Tics and Something More


Judy Adelizzi-DeLany
Nemours/Alfred I. duPont Hospital for Children, Wilmington, DE, USA

A 6-year 7-month-old male with mild to moderate severity Past Surgical History
simple tics diagnosed at age 3.5 years is followed every
10–12 months in neurology clinic. He presented to the Aside from circumcision at birth, there is no past surgical
emergency department (ED) yesterday after 2–3 weeks of history.
worsening tics with headaches, left eye pain, and an event
this morning described as his “gaze stuck up and to the left”
with right eye drifting medially and right arm weakness. Family History

Mother is positive for gestational diabetes and has factor


V Leiden deficiency. History of migraine in mother and
History of Present Illness maternal grandmother, hypothyroidism in paternal grand-
mother, and Tourette syndrome in paternal aunt and half-
For the last 3 weeks this child has been noted to have wor- brother (same biological father). Father has obsessive com-
sening photophobia, left eye pain, and head pain in the left pulsive disorder and global mental health issues (bipolar);
occipital area. When he originally presented to neurology he also has mild hand tremors related to his medication.
clinic, he had complaints of left eye motor tics and head-
ache which resolved by the second visit at age 5 years
Current Status
9 months. His tics have “come and gone” by parental
report. He recently requested “needing sunglasses” both
In the ED, the patient is noted as an alert, mildly hyperac-
at school and at home for bright lights. On the morning
tive, slender male standing without difficulty with obvious
of his presentation to the ED, he was unable to hold his
left eye motor tic consisting of rhythmic opening and clos-
spoon when eating his breakfast, and parents were
ing. All systems are reported negative, except for left eye.
concerned.
Headache and photophobia are present. Skin is positive
for hyperpigmentation between shoulder blades.

Past Medical History • Physical examination: temperature 37.1 C (98.8 F), heart
rate 86 bpm, blood pressure 95/46 mmHg, and respira-
tory rate 24 breaths per minute. Height is 1.2 m (22%
This patient was born at 37 weeks’ gestation by planned for age and gender), weight 21 kg (35% for age and gen-
cesarean section due to maternal placenta previa and ges- der), and body mass index (BMI) 15.74.
tational diabetes. His birthweight was 2.78 kg. He went
home with mother 4 days later with no need for any oxygen
• Ophthamologic examination: conjunctivae and sclera are
clear with equal pupils, left eye with constant tic, fundo-
or respiratory support and no antibiotics or phototherapy. scopic examination is normal.
He fed well and was bottle-fed regular infant formula. He
had reported normal development without any evidence
• Musculoskeletal examination: No defects are noted, gait
is appropriate without ataxia. Motor: normal strength,
of developmental delay. muscle mass, and tone in all extremities.

Cases in Pediatric Acute Care: Strengthening Clinical Decision Making, First Edition. Edited by Andrea M. Kline-Tilford and Catherine M. Haut.
© 2020 John Wiley & Sons Ltd. Published 2020 by John Wiley & Sons Ltd.
Downloaded from https://onlinelibrary.wiley.com/doi/ by National Institute Of Standard, Wiley Online Library on [06/03/2024]. See the Terms and Conditions (https://onlinelibrary.wiley.com/terms-and-conditions) on Wiley Online Library for rules of use; OA articles are governed by the applicable Creative Commons License
412 Cases in Pediatric Acute Care

• Neurologic examination: alert and awake, cognition,


communication and socialization appropriate for age,
can exacerbate headache. The morning occurrence of right
arm weakness is a red flag. The best initial approach from
happy and contented child. Cranial nerves as follows: the ED perspective is to order a CT scan to rule out any
III, IV and VI, all extraocular movements are intact; V, structural reason for this event. A second choice, or in addi-
facial sensation normal and symmetrical; VII, facial con- tion to a CT scan, is to obtain an EEG to rule out brain elec-
tours and strength symmetrical; VIII, vibratory stimulus trical abnormality as the event is suspicious for a possible
response normal bilaterally; IX and X, uvula midline seizure given the parental report of abnormal eye and
with normal soft palate movement; XI, neck rotation arm movement.
against resistance showed normal sternocleidomastoid
strength bilaterally; XII, tongue protrusion midline, no
What are next steps given these unexpected
fasciculation noted. Sensory: normal to light touch,
findings?
reflexes 2+ bilaterally.

• Developmental examination: normal for age, no cogni-


tive or motor delay identified.
Admit the patient and schedule a sedated magnetic reso-
nance imaging (MRI) scan to obtain the best picture of

• Audiologic examination: hearing intact. the Chiari malformation and consult neurosurgery for eval-
uation of the Chiari malformation.
As for the CT scan, MRI reveals the Chiari I malformation
Questions with marked reduction of CSF at the craniocervical junction,
with crowding and/or compression of structures in this
Answer the following questions using the details provided. region. There does appear to be edema within the upper cer-
vical cord at C2 at the level of crowding/compression. Par-
1 Given the history and new-onset photophobia, are there
tially empty sella is noted, which is non-specific but could
any initial diagnostic studies that might be useful?
suggest increased intracranial pressure.
Continuation of case: The routine video electroencepha- Neurosurgery was consulted, and given overall baseline
logram (EEG) was normal in wakefulness. There were no normal neurologic and general examinations, with the
focal or epileptiform abnormalities. Numerous typical exception of eye tic, patient was discharged to follow-up
patient events were recorded characterized by frequent within the week for a scheduled MRI of the cervicothoraco-
left/right eye twitching or winking movements, left-sided lumbar junction to rule out syrinx.
facial scrunching, upward eye deviation, eye deviation to Chiari malformation I is a condition in which brain tis-
the right side followed by downward movement, and eye sue extends into the spinal canal. It is a structural defect
deviation to left side. These events were not seizures but in the cerebellum, with displacement of one or both cer-
were consistent with motor/ocular tics. ebellar tonsils through the opening in the base of the
Computed tomography (CT) of the head/brain revealed skull known as the foramen magnum. This condition
no acute intracranial abnormality on non-contrast study, occurs in about 1% of the pediatric population and
but a Chiari I malformation with marked reduction of patients typically present in adolescence or early adult-
the cerebrospinal fluid (CSF) was observed at the craniocer- hood, although the problem is often found incidentally
vical junction. with imaging at younger ages. The pathogenesis of Chiari
I malformation is poorly understood, but it is a heteroge-
2 What are the next steps given these unexpected
neous condition and there is no other causative pathol-
findings?
ogy causing the cerebellar herniation. Congestion at
the foramen magnum and posterior fossa can cause dis-
ruption to flow of CSF that can result in increased intra-
Rationale and Evidence-based Practice
cranial pressure. The defect may also be present with
Explanation syrinx (or cyst/cavity) in the spinal canal. There are cur-
rently no specific guidelines in the treatment or decision
Given the history and new-onset photophobia, are
for surgery, but most children do eventually have a sur-
there any initial diagnostic studies that might be
gical procedure, especially if increased cranial pressure is
useful?
present. In this case, tics were unrelated to the findings of
Tic evaluation and work-up is primarily by symptom report the Chiari malformation I.
and observation. A sudden and dramatic increase in photo- Headache is the most common symptom of Chiari
phobia accompanied by headache can be explained by a I malformation, although the frequency and patterns are
virus; alternatively, worsening tics causing eye muscle pain not consistent, especially in children. The headache is
Downloaded from https://onlinelibrary.wiley.com/doi/ by National Institute Of Standard, Wiley Online Library on [06/03/2024]. See the Terms and Conditions (https://onlinelibrary.wiley.com/terms-and-conditions) on Wiley Online Library for rules of use; OA articles are governed by the applicable Creative Commons License
Chapter 98 Tics and Something More 413

typically described as suboccipital and occurring with Val- to tics but no single gene or genes have been identified.
salva maneuvers such as coughing or straining (Piper et al., Diagnosis of tics or Tourette syndrome is made on history
2019). Other symptoms include limb weakness and symp- and clinical examination. Features that should be docu-
toms of lower cranial nerve palsy such as dysphagia and mented include onset, progression (e.g. motor tics may
dysphonia. Other findings include non-radicular pain, limb manifest in the head and face then migrate distally), fre-
paresthesia, dissociative loss of pain and temperature sen- quency, severity or impairment, and family history. The fre-
sation, and spasticity. quency and description of what is witnessed or occurred
Around 25% of children have transient tics, while during the clinic visit should also be documented. Transient
0.03–1% have Tourette syndrome. Tics typically begin tics have been present for 4 weeks up to a year. Chronic tics
between 4 and 8 years of age, most before the teenage years. present for more than a year. Tourette syndrome is a neu-
The male/female ratio is 4 : 1. Tic symptoms fluctuate, wax- rodevelopmental disorder characterized by spontaneous
ing and waning over time. Tic severity may peak between involuntary movements and vocalizations that occur for
10 and 12 years of age. There may be a genetic component at least a year.

Reference
Piper, R.J., Pike, M., Harrington, R., and Magdum, S.A. (2019). management. British Medical Journal 365: l1159. doi:
Chiari malformations: principles of diagnosis and 10.1136/bmj.l1159.

Further Reading
Piper, R.J. and Magdum, S.A. (2019). Chiari I malformation Toldo, I., Tangari, M., Mardari, R., et al. (2014). Headache
and raised intracranial pressure. Child’s Nervous System in children with Chiari I malformation. Headache 54(5):
35(10): 1719–1725. doi: 10.1007/s00381-019-04232-x. 899–908. doi: 10.1111/head.12341.
Downloaded from https://onlinelibrary.wiley.com/doi/ by National Institute Of Standard, Wiley Online Library on [06/03/2024]. See the Terms and Conditions (https://onlinelibrary.wiley.com/terms-and-conditions) on Wiley Online Library for rules of use; OA articles are governed by the applicable Creative Commons License
415

99

Lightheaded Adolescent
Cathy Woodward
Long School of Medicine, University of Texas Health, San Antonio, TX, USA

A 15-year-old female transferred by emergency medical activity, and her periods are normal. She reports she is due
services to the emergency department (ED) complains of to start menstruation any day.
lightheadedness and near syncope after standing to move
between classes at school.

Family History
History of Present Illness
Her parents are alive and well with no chronic disease. Both
The lightheadedness began within a few moments of stand- siblings, a 16-year-old old sister and 12-year-old brother,
ing from a desk and taking a few steps. A friend helped her are healthy with no medical issues. Maternal grandmother
to the floor. When she got up to go to the school nurse’s has type 2 diabetes. Fraternal grandparents both have
office, her lightheadedness worsened and says she “almost hypertension.
fainted.” Associated symptoms are a mild headache today
and feeling tired most days for over 6 months. She reports
she used to be more physically active, but now does not Current Status
have the energy for exercise. She has had this same light-
headedness at least a few times a week for several months On arrival to the ED she was awake and alert with no acute
and they are more frequent around the time of her distress. She reports a mild posterior headache with almost
menstruation. complete resolution of her lightheadedness. Her physical
examination is normal. Her initial vital signs include tem-
perature 37 C (98.6 F), heart rate 92 bpm, blood pressure
Past Medical History 108/66 mmHg, and oxygen saturation 99% on room air.
She denies chest pain and shortness of breath. The monitor
The patient has not had any surgeries or trauma requiring demonstrates normal sinus rhythm and her 12-lead electro-
ED or medical visits. She has no chronic medical illnesses. cardiogram (EKG) is read as normal. A pregnancy test is
She has seen her primary care provider twice in the past negative.
6 months for similar symptoms and was initially told she A basic metabolic profile is obtained to evaluate for elec-
was dehydrated from not drinking enough water. During trolyte disturbances and dehydration. A complete blood
the second medical visit her mother was advised to seek count is obtained to evaluate for anemia and signs of infec-
psychotherapy for a possible anxiety disorder. Her immuni- tion. Orthostatic blood pressure measurements are
zations are up to date for age along with annual influenza obtained to evaluate for change in blood pressure in differ-
vaccines. She has a weight at the 60th percentile for gender ent positions. The results of these studies are shown in
and age. She takes no medication or vitamins, denies sexual Tables 99.1–99.3.

Cases in Pediatric Acute Care: Strengthening Clinical Decision Making, First Edition. Edited by Andrea M. Kline-Tilford and Catherine M. Haut.
© 2020 John Wiley & Sons Ltd. Published 2020 by John Wiley & Sons Ltd.
Downloaded from https://onlinelibrary.wiley.com/doi/ by National Institute Of Standard, Wiley Online Library on [06/03/2024]. See the Terms and Conditions (https://onlinelibrary.wiley.com/terms-and-conditions) on Wiley Online Library for rules of use; OA articles are governed by the applicable Creative Commons License
416 Cases in Pediatric Acute Care

Table 99.1 Basic metabolic profile. Table 99.3 Orthostatic vital signs.

Sodium 141 mEq/L Position Vital signs


Potassium 4.1 mEq/L
Chloride 101 mEq/L Lying down Blood pressure 102/60 mmHg
Heart rate 90 bpm
Carbon dioxide 24 mEq/L
Sitting up with legs dangling Blood pressure 106/62 mmHg
Blood urea nitrogen 12 mg/dL Heart rate 96 bpm
Creatinine 0.6 mg/dL Standing Blood pressure 104/58 mmHg
Heart rate 135 bpm
Glucose 92 mg/dL

Table 99.2 Complete blood count. 6 months. It is more common in female adolescents, with
a median onset of symptoms of 13 years of age. The phys-
White blood cell count 10 000/mm3 iology of POTS is understood but the etiology remains
Hemoglobin 14 g/dL unknown. The pathophysiology of POTS is associated with
Hematocrit 42% a change in vascular tone when standing, with chronotro-
Platelet count 346 000/mm3 pic sympathetic stimulation and increased heart rate and
resultant impaired cerebral perfusion initiating lighthead-
edness and occasionally syncope.
After standing, she complains of lightheadedness and
nausea. There is no change in the cardiac rhythm, other
than rate, during the procedure.
What other therapies or diagnostic studies would
be recommended at this time?
Questions
A formal head-up tilt table test and consideration of Holter
Answer the following questions using the details provided. monitor to exclude cardiac dysrhythmia. Plasma norepi-
nephrine levels in supine and standing positions may aid
1 Based on the information given, what diagnoses are in the diagnosis. Supine norepinephrine levels can be
included in your differential diagnosis? high-normal in POTS patients, but will usually be elevated
2 Based on the data, what is the most likely diagnosis for when upright.
this adolescent?
3 What other therapies or diagnostic studies would be
recommended at this time? What treatment can be started after diagnosis
4 What treatment can be started after diagnosis is is confirmed?
confirmed?
Therapy should be multimodal and include both non-
pharmacologic approaches and possibly medications. Ini-
tially, good sleep hygiene with regular bedtimes and elim-
Rationale and Evidence-based Practice
ination of electronic devices at night must be ensured.
Explanation A formal exercise regimen has been shown to reduce ortho-
static tachycardia and improve quality of life. There are no
Based on the information given, what diagnoses
formal guidelines for pharmacotherapy of these patients,
are included in your differential diagnosis?
with only case reports and anecdotal accounts to support
The differential diagnoses include orthostatic hypotension, the use of medication for the desired response of maintain-
dehydration, cardiac dysrhythmia, postural tachycardia ing vascular tone and heart rate and decreasing sympa-
syndrome, hyperventilation syndrome, hypoglycemia, anx- thetic response. Hypovolemia may be associated with
iety disorder. POTS and increasing fluid administration during the day
should be advised. Patient education should encourage
Based on the data, what is the most likely
avoidance of aggravating factors like heat and dehydration,
diagnosis for this adolescent?
and promote the use of support hose (30–40 mmHg pres-
The most likely diagnosis is postural tachycardia syndrome sure). A referral to a cardiologist or center for POTS patients
(POTS). POTS is characterized by orthostatic tachycardia may be warranted if non-pharmacologic approaches are
without associated hypotension intermittently for at least unsuccessful.
Downloaded from https://onlinelibrary.wiley.com/doi/ by National Institute Of Standard, Wiley Online Library on [06/03/2024]. See the Terms and Conditions (https://onlinelibrary.wiley.com/terms-and-conditions) on Wiley Online Library for rules of use; OA articles are governed by the applicable Creative Commons License
417
Chapter 99 Lightheaded Adolescent

Raj, S. (2013). Postural tachycardia syndrome (POTS).


Circulation 127: 2336–2342.
Boris, J. (2018). Postural orthostatic tachycardia syndrome in
children and adolescents. Autonomic Neuroscience: Basic
and Clinical 215: 97–101.
Further Reading
Downloaded from https://onlinelibrary.wiley.com/doi/ by National Institute Of Standard, Wiley Online Library on [06/03/2024]. See the Terms and Conditions (https://onlinelibrary.wiley.com/terms-and-conditions) on Wiley Online Library for rules of use; OA articles are governed by the applicable Creative Commons License
419

100

Infant with Hyponatremia


Sarah Pihl
Children’s Hospital of Colorado, Aurora, CO, USA

A 5-month-old female with no prior medical history is Family History


admitted to the pediatric intensive care unit (PICU) with
new-onset seizures. No significant family history. No history of seizures.

History of Present Illness Current Status


The patient was in her usual state of health until the day
On arrival at the PICU she has a generalized tonic–clonic
prior to presentation when she developed increased
seizure lasting 35 seconds with apnea and hypoxia, associ-
malaise. There are no sick contacts. She had adequate oral
ated with desaturation to 74% on room air. Lorazepam
intake and no change in the number of wet diapers. Her
0.1 mg/kg is given to treat the seizure and a nasal cannula
parents report she had a “shaking episode” and her lips
is placed. Her physical examination is notable for an ante-
turned blue. Emergency medical services were called and
rior fontanelle that is soft, flat and open. Pupils are brisk
on arrival patient was no longer having signs of seizure
bilaterally. General hypotonia is noted. Plasma osmolality
activity. She was transported to the emergency department
study is sent. Review of history included a feeding history.
(ED) for evaluation. Laboratory studies were obtained and
On review of the formula mixing technique, it was discov-
were notable for a sodium level of 117 mEq/L.
ered that during preparation a low amount of formula pow-
der was used for the amount of water that which it
was mixed.
Past Medical History The results of a basic metabolic profile are shown in
Table 100.1.
She was born full term via vaginal delivery and discharged
home on day of life 2 her with parents. Weight checks at the
primary care provider (PCP) office indicate appropriate
weight gain. She is at the 55th percentile for weight and Questions
the 60th percentile for height. Her mother recently stopped
breastfeeding due to decreased supply after returning to Answer the following questions using the details provided.
work. Milestones are developmentally appropriate and 1 Based on the data given, what are the differential diag-
include a social smile, rolling onto back, and bringing noses and the most likely diagnosis for this child?
hands to mouth. Vaccinations are up to date for her age. 2 What is the best way to treat her seizures?
3 What is the best way to correct this electrolyte
derangement?
Past Surgical History 4 How would you order this intervention?
5 How would you prevent future recurrence of hypona-
None. tremia in this infant?

Cases in Pediatric Acute Care: Strengthening Clinical Decision Making, First Edition. Edited by Andrea M. Kline-Tilford and Catherine M. Haut.
© 2020 John Wiley & Sons Ltd. Published 2020 by John Wiley & Sons Ltd.
Downloaded from https://onlinelibrary.wiley.com/doi/ by National Institute Of Standard, Wiley Online Library on [06/03/2024]. See the Terms and Conditions (https://onlinelibrary.wiley.com/terms-and-conditions) on Wiley Online Library for rules of use; OA articles are governed by the applicable Creative Commons License
420 Cases in Pediatric Acute Care

Table 100.1 Basic metabolic profile. to cross when the osmolarity of extracellular and intracel-
lular fluid is not equal. Sodium is the largest determinant of
Sodium 117 mEq/L
osmolarity, so in acute hyponatremia the change in osmo-
Potassium 4.1 mEq/L larity causes increased fluid volume in the brain, known as
Chloride 94 mEq/L cerebral edema. Other factors, such as hypoxia, can also
Carbon dioxide 24 mEq/L contribute to the incidence of seizures in the presence of
Blood urea 12 mg/dL hyponatremia and can worsen clinical outcomes in hypo-
nitrogen natremic encephalopathy as the active transport of sodium
Creatinine 0.8 mg/dL impairs the brain’s adaptive response. Hypoxia can result
Glucose 81 mg/dL from a seizure or can be an effect of hyponatremia.
Serum 289 mosmol/kg (normal range 285–295
osmolality mosmol/kg)
What is the best way to correct this electrolyte
derangement?
Care must be taken to slowly correct hyponatremia due to
Rationale and Evidence-based Practice the risk of central pontine myelinolysis. Rapid correction of
Explanation serum sodium can result in the loss of some of the myelin
sheath of neural cells and significant neurologic symptoms
Based on the data given, what are the differential including spastic quadriparesis. If seizure activity is pres-
diagnoses and the most likely diagnosis for ent, increasing the initial sodium by 5 mEq/L is appropriate
this child? in order to decrease the risk of additional seizures. Serial
This child has hyponatremia, defined as a serum sodium serum sodium measurements must be obtained to ensure
less than 135 mEq/L; this child’s serum sodium is correction does not exceed an increase of greater than
117 mEq/L. Differential diagnoses of hyponatremia include 10 mEq/L in a 24-hour period. Hyponatremia due to excess
syndrome of inapproriate antidiuretic hormone (SIADH) water intake resulting from incorrectly mixed formula is
secretion, glucocorticoid deficiency, hypothyroidism, water not uncommon, so it is important to educate parents on
intoxication, and fluid loss such as gastrointestinal or renal the proper mixing of powder or concentrated formula with
losses. The most likely diagnosis for this infant is water appropriate volumes of water, thus eliminating this source
intoxication from incorrectly mixed formula. The acute of free water. Infants are at a much higher risk of hypona-
presentation of low serum sodium with a normal plasma tremic seizures due to the higher brain to intracranial vol-
osmolality supports this conclusion. ume ratio and immature kidney function, which can
prevent water excretion at adequate levels when serum
sodium is low.

What is the best way to treat her seizures?


How would you order this intervention?
Seizures caused by hyponatremia should be treated emer-
gently with hypertonic saline, which corrects the electrolyte Use of oral hydration is recommended with mild hypona-
abnormality. This type of seizure is not as responsive to tremia caused by excess water ingestion. This infant has
antiepileptic medications as other etiologies of seizure severe hyponatremia and careful isotonic fluid manage-
activity. The administration of hypertonic saline results ment with measurement of serum sodium levels every
in a small increase in serum sodium and decreases the risk 2 hours will allow careful correction of sodium levels.
of additional seizures. It is estimated that each 1 mL/kg of
3% hypertonic saline increases serum sodium by 1 mEq/L.
How would you prevent future recurrence
As an example, administration of 5 mL/kg of 3% saline to a
of hyponatremia in this infant?
child with a serum sodium of 120 mEq/L would result in an
estimated increase in serum sodium to 125 mEq/L. Educating the caregivers on proper formula preparation is
The clinical effects of hyponatremia involve the regula- key to preventing recurrence. Providing this information in
tion of the blood–brain barrier to allow fluid as free water a non-judgmental manner is essential. Engagement of a
Downloaded from https://onlinelibrary.wiley.com/doi/ by National Institute Of Standard, Wiley Online Library on [06/03/2024]. See the Terms and Conditions (https://onlinelibrary.wiley.com/terms-and-conditions) on Wiley Online Library for rules of use; OA articles are governed by the applicable Creative Commons License
Chapter 100 Infant with Hyponatremia 421

dietitian can also be helpful. A social work consultation and A detailed neurologic examination will be needed prior to
connection with community resources may be needed to discharge as well as developmental milestone monitoring
ensure the family has access to adequate formula/nutrition. by the PCP.

Further Reading
Greenbaum, L. (2019). Sodium. In: Nelson Textbook of Reid-Adam, J. (2013). Hyponatremia. Pediatrics in Review
Pediatrics, 21st edn (ed. R.M. Kliegman, J.W. St Geme, N.J. 34(9): 417–419.
Blum, S.S. Shah, R.C. Tasker, K.M. Wilson, and R.E.
Behrman), 389–425. Philadelphia, PA: Elsevier Saunders.
Downloaded from https://onlinelibrary.wiley.com/doi/ by National Institute Of Standard, Wiley Online Library on [06/03/2024]. See the Terms and Conditions (https://onlinelibrary.wiley.com/terms-and-conditions) on Wiley Online Library for rules of use; OA articles are governed by the applicable Creative Commons License
423

101

Bone Marrow Transplant in Sickle Cell Disease


J. Paige Rieckhoff
C.S. Mott Children’s Hospital, Ann Arbor, MI, USA

A 17-year-old female who has undergone bone marrow managed with chronic exchange transfusion. In addition,
transplant (BMT) for sickle cell disease (SCD) presents with she suffers from chronic pain, liver disease due to iron over-
fever and 3 days of cough, rhinorrhea, and wheezing. load from chronic transfusion therapy, obesity, and asthma.
Her immunizations are up to date for age.

History of Present Illness

A 17-year-old female with history of SCD was admitted to


Past Surgical History
the hematology/oncology unit in preparation for a haploi-
She had a tonsillectomy and adenoidectomy at 12 years of
dentical 4/8 match BMT. A preparative regimen including
age for obstructive sleep apnea and a left total hip replace-
thymoglobulin, fludarabine, cyclophosphamide, and thio-
ment at 15 years of age due to damage secondary to avascu-
tepa was administered. The donor, the adolescent’s mother,
lar necrosis.
is a 48-year-old female. A regimen to prevent graft-versus-
host disease (GVHD) including cyclophosphamide on days
+3 and +4 following BMT, sirolimus titrated with a goal
serum trough level of 5–15 ng/mL, and mycophenolate Family History
mofetil 15 mg/kg by mouth three times daily from days
+5 to +35 was administered. She engrafted neutrophils Her brother also has SCD. Her mother has sickle cell trait,
on day +16. On day +21, an erythematous maculopapular hypertension and asthma. Her father’s history is unknown.
rash was noted on her bilateral forearms with associated
erythema of the palms and soles. Topical triamcinolone
cream was prescribed twice daily and the rash resolved.
She was discharged to home on day +23. She presents to Current Status
the emergency department on day +37 post transplant
for fever with 3 days of cough, rhinorrhea, and wheezing. On day +39, an erythematous, pruritic, maculopapular
She is positive for adenovirus on a respiratory virus panel. rash is noted over her face and neck. She is additionally
Serum adenovirus testing reveals an initial level of noted to have palmar and plantar erythema.
11 800 copies/mL. Intravenous cidofovir 5 mg/kg weekly
is initiated.

Questions
Past Medical History Answer the following questions using the details provided.
The adolescent’s SCD has been complicated by multiple 1 What differential diagnoses would you consider?
episodes of acute pain crisis, acute chest syndrome, dacty- 2 Based on her history, what factors increase this adoles-
litis, and avascular necrosis. Prior to BMT, her SCD was cent’s risk of developing this condition?

Cases in Pediatric Acute Care: Strengthening Clinical Decision Making, First Edition. Edited by Andrea M. Kline-Tilford and Catherine M. Haut.
© 2020 John Wiley & Sons Ltd. Published 2020 by John Wiley & Sons Ltd.
Downloaded from https://onlinelibrary.wiley.com/doi/ by National Institute Of Standard, Wiley Online Library on [06/03/2024]. See the Terms and Conditions (https://onlinelibrary.wiley.com/terms-and-conditions) on Wiley Online Library for rules of use; OA articles are governed by the applicable Creative Commons License
424 Cases in Pediatric Acute Care

Rationale and Evidence-based Practice How would you grade this child’s GVHD?
Explanation Using the rule of 9s, her skin GVHD is estimated at approx-
imately 10% (3.5% for her face, 1.5% each for hands and
What differential diagnoses would you consider? feet). She has no symptoms of gastrointestinal or hepatic
Conditions to consider for this child include GVHD, drug GVHD. Disease is staged as follows: skin, 1; gastrointesti-
eruption, and viral exanthem. With her history of BMT, nal, 0; liver, 0; overall grade, 1.
GVHD is the most likely diagnosis.
What treatment is recommended for GVHD?
Corticosteroid therapy is recommended as first line for all
Based on her history, what factors increase this forms of acute GVHD. For less than 50% body surface area
adolescent’s risk of developing this condition? without other organ involvement, the topical route alone is
In this case, human leukocyte antigen (HLA) disparity, sufficient.
increased donor parity, and the advanced age of her donor Continuation of case: On day +43, she develops large-
add increased risk for developing acute GVHD. Other risk volume diarrhea (1.5–2 L/day) without associated crampy
factors include sex mismatching, peripheral blood stem abdominal pain. A colonoscopy with biopsies is performed.
cells as a source of donor cells, and pre-transplant total Her bilirubin is closely monitored and remains below
body irradiation. 2 mg/dL.

What are your differential diagnoses for her


new-onset diarrhea?
Additional Questions and Evidence-based
Rationale With her initial presentation of fevers and positive serum and
respiratory adenovirus, an infection must be excluded as the
3 What three organ systems are targeted by acute GVHD primary cause of diarrhea. Stomach biopsy and stool sample
and what are the manifestations of GVHD in were positive for adenovirus. Based on the history of trans-
each organ? plant and history of skin GVHD, the diarrhea may also be
4 How would you grade this child’s GVHD? caused by lower gastrointestinal GVHD. Although GVHD is
5 What treatment is recommended for GVHD? a clinical diagnosis, it may be confirmed by biopsy. Her biop-
6 What are your differential diagnoses for her new-onset sies were also positive for GVHD in the colon and stomach.
diarrhea?
How would you grade her GVHD?
7 How would you grade her GVHD?
8 What is first-line medication management for grade II– Her skin score remained at approximately 10%. She now
IV GVHD including dose? has diarrhea of 1.5–2 L/day with biopsy confirming gastro-
9 What side effects of high-dose steroid therapy should be intestinal GVHD, but there are no signs of liver GVHD,
monitored for? with bilirubin below 2 mg/dL. Disease is staged as follows:
skin, 1; gastrointestinal, 3; liver, 0; overall grade, 3.

What is first-line medication management for


What three organ systems are targeted by acute
grade II–IV GVHD including dose?
GVHD and what are the manifestations of GVHD in
each organ? Methylprednisolone 1–2 mg/kg daily is the standard for
first-line systemic therapy. There are several approaches
Skin is the most common organ involved, affecting 75% of
regarding such regimens, many with goals of tapering ster-
patients with acute GVHD. Manifests as a maculopapular
oids as tolerated due to associated side effects.
rash or generalized erythema, which can be pruritic or
painful. It usually involves the palms and soles. Gastroin-
What side effects of high-dose steroid therapy
testinal GVHD is the second most common form, affecting
should be monitored for?
50% of patients with acute GVHD. Upper gastrointestinal
GVHD manifests as anorexia, nausea, and dyspepsia. High-dose steroid therapy can cause hypertension,
Lower gastrointestinal manifestations include profuse Cushing syndrome, adrenal axis suppression, hyperglyce-
watery diarrhea and crampy abdominal pain. Hepatic mia, and gastrointestinal bleeding. This additional
GVHD affects only about 20% of patients with acute GHVD immunosuppression also further increases the risk of infec-
and is manifested as hyperbilirubinemia with jaundice. tious complications.
Downloaded from https://onlinelibrary.wiley.com/doi/ by National Institute Of Standard, Wiley Online Library on [06/03/2024]. See the Terms and Conditions (https://onlinelibrary.wiley.com/terms-and-conditions) on Wiley Online Library for rules of use; OA articles are governed by the applicable Creative Commons License
Chapter 101 Bone Marrow Transplant in Sickle Cell Disease 425

Further Reading
Choi, S.W., Levine, J.E., and Ferrara, J.L.M. (2010). Jacobsohn, D.A. (2008). Acute graft-versus-host disease in
Pathogenesis and management of graft-versus-host disease. children. Bone Marrow Transplantation 41: 215–222.
Immunology and Allergy Clinics of North America 30: Martin, P.J., Rizzo, D., Wingard, J.R., et al. (2012). First and
75–101. second-line systemic treatment of acute graft-versus-host
Flowers, M.E.D., Inamoto, Y., Carpenter, P.A., et al. (2011). disease: recommendations of the American Society of Blood
Comparative analysis of risk factors for acute graft-versus- and Marrow Transplantation. Biology of Blood and Marrow
host disease and for chronic graft-versus-host disease Transplantation 18: 1150–1163.
according to National Institutes of Health consensus Taketomo, C., Hodding, J., and Kraus, D. (2017). Pediatric
criteria. Blood 117: 3214–3219. Dosage Handbook. Hudson, OH: Lexi-Comp.
Holtan, S.G., Pasquini, M., and Weisdorf, D.J. (2014). Acute Zeiser, R. and Blazar, B.R. (2017). Acute graft-versus-host
graft-versus-host disease: a bench-to-bedside update. Blood disease: biologic process, prevention, and therapy. New
124(3): 363–373. England Journal of Medicine 377: 2167–2179.
Downloaded from https://onlinelibrary.wiley.com/doi/ by National Institute Of Standard, Wiley Online Library on [06/03/2024]. See the Terms and Conditions (https://onlinelibrary.wiley.com/terms-and-conditions) on Wiley Online Library for rules of use; OA articles are governed by the applicable Creative Commons License
427

102

Child with Respiratory Failure and Transaminitis


Kim Steanson1,2
1
Monroe Carell Jr. Children’s Hospital at Vanderbilt, Nashville, TN, USA
2
Vanderbilt University College of Nursing, Nashville, TN, USA

A 13-year-old previously healthy girl is admitted to the Past Surgical History


pediatric intensive care unit (PICU) with respiratory fail-
ure, hypotension, acute kidney injury, transaminitis, and None.
anemia.
Family History
History of Present Illness Maternal grandmother with hypertension, paternal grandfa-
ther with cardiovascular disease. Two older siblings and
This girl recently returned from a trip to the Dominican mother are healthy, father currently with flu-like symptoms.
Republic with her family. She stayed with her family at
an all-inclusive resort on the beach. She was able to spend Current Status
time on the beach, playing in the ocean and in the pool at
the resort. She and her family left the resort to be involved On arrival to the local ED, she is obtunded with respiratory
in a service project with their church group. distress. She receives midazolam and etomidate for intuba-
Approximately 1 week after returning to the United tion and 1 L of lactated Ringer’s fluid is started. She is being
States, she developed fever to 38.3 C (101 F) and body transferred to the pediatric intensive care unit (PICU) at a
and muscle aches. On day 2 of the illness, she developed sig- children’s hospital. She arrives at the PICU intubated and
nificant back pain and chills. She was treated at home with sedated. A complete metabolic panel and complete blood
acetaminophen and ibuprofen for the fever and body aches. count are obtained and the results are shown in Tables 102.1
Her parents took her to the emergency department (ED) at and 102.2. Influenza (rapid), respiratory viral panel, and
a local hospital for further care on day 3 of the illness. blood cultures are sent. A chest radiograph was obtained
In the ED, she received a diagnosis of probable flu or viral at the outside hospital after intubation and showed
illness. She received antiviral medication and was endotracheal tube at level of T1–T2 and bilateral diffuse
instructed to take the medication for 5 days. After 5 days, pulmonary edema with loss of hemidiaphragms bilaterally.
the symptoms began to worsen and abdominal pain and
headache began. Her parents decided to bring her back to
Questions
the ED and she fainted while attempting to walk to the car.
Answer the following questions using the details provided.
Past Medical History 1 What other historical information would it be impor-
tant to obtain for this patient?
She was born at term and was discharged with her mother 2 What are the differential diagnoses and the most likely
on day of life 2. She has received all immunizations along diagnosis for this child?
with the influenza vaccine this year. She has never been hos- 3 What therapies are necessary to treat the current labo-
pitalized or had surgery. She is active and in the 7th grade. ratory abnormalities?

Cases in Pediatric Acute Care: Strengthening Clinical Decision Making, First Edition. Edited by Andrea M. Kline-Tilford and Catherine M. Haut.
© 2020 John Wiley & Sons Ltd. Published 2020 by John Wiley & Sons Ltd.
Downloaded from https://onlinelibrary.wiley.com/doi/ by National Institute Of Standard, Wiley Online Library on [06/03/2024]. See the Terms and Conditions (https://onlinelibrary.wiley.com/terms-and-conditions) on Wiley Online Library for rules of use; OA articles are governed by the applicable Creative Commons License
428 Cases in Pediatric Acute Care

Table 102.1 Basic metabolic profile. symptoms of fever, body and muscle aches, headache,
and abdominal pain, now progressed to respiratory failure,
Sodium 145 mEq/L
anemia, acute kidney injury, transaminitis, and hypoten-
Potassium 3.5 mEq/L sion. The clinical features of malaria are non-specific and
Chloride 90 mEq/L overlap significantly with those of other febrile illnesses.
Carbon dioxide 22 mEq/L The World Health Organization recommends that all
Blood urea nitrogen 82 mg/dL patients have a parasite-specific laboratory test performed
Creatinine 0.75 mg/dL to confirm the clinical impression (Figure 102.1).
Glucose 60 mg/dL
Malaria results from infection with one of the four species
of Plasmodium (P. falciparum, P. vivax, P. ovale, P. malar-
Magnesium 2.8 mg/dL
iae). The most typical means of transmission of malaria is
Aspartate aminotransferase 70 U/L
via the bite of the Anopheles mosquito. Malaria typically
Alanine aminotransferase 50 U/L presents as fever, chills, headache, fatigue, dizziness, and
Alkaline phosphatase 400 U/L malaise. In pediatric patients, fever and headache may be
Total bilirubin 0.3 mg/dL the only presenting symptoms. Initial management of
Direct bilirubin 0.5 mg/dL patients with malaria includes treating acute symptoms
and treatment of the malaria, based on the knowledge of
resistance to chloroquine in the area in which malaria
was contracted (Figure 102.2).
Table 102.2 Complete blood count.

White blood cell count 9000/mm3


What therapies are necessary to treat the current
Hemoglobin 5 g/dL laboratory abnormalities?
Hematocrit 15%
Support and correction of respiratory failure, anemia, and
Platelet count 7900/mm3
hypotension are of the highest importance. This child
was intubated at the outside hospital and will be main-
Rationale and Evidence-based Practice tained on ventilator settings to maintain appropriate oxy-
genation and ventilation. She received a liter of fluid
Explanation

What other historical information would it be


important to obtain for this patient?
Travelers to areas where malaria is endemic may need che-
moprophylaxis. The Centers for Disease Control and Pre-
vention (CDC) provides information about each
destination including the prevailing type of malaria and
the appropriate regimens based on the epidemiology of
the area. Other malaria prevention interventions are also
encouraged, including using mosquito nets and repellants.
In this situation, it is important to know where the
patient traveled and if chemoprophylaxis was prescribed
and administered appropriately.

What are the differential diagnoses and the most


likely diagnosis for this child?
Differential diagnoses for this child include malaria, den-
gue fever, hepatitis A, influenza, measles, leptospirosis,
viral illnesses, and pneumonia.
The most likely diagnosis for this child is malaria because
of the child’s history, travel to a malaria endemic area with-
out prophylaxis, laboratory findings, and presenting Figure 102.1 Rapid diagnostic test (CDC).
Downloaded from https://onlinelibrary.wiley.com/doi/ by National Institute Of Standard, Wiley Online Library on [06/03/2024]. See the Terms and Conditions (https://onlinelibrary.wiley.com/terms-and-conditions) on Wiley Online Library for rules of use; OA articles are governed by the applicable Creative Commons License
Chapter 102 Child with Respiratory Failure and Transaminitis 429

Figure 102.2 Malaria transmission areas. Source: Courtesy of CDC.

while being transported to correct the hypotension. Treat- Additional Questions and Evidence-
ment of anemia will be completed with a packed red blood based Rationale
cell infusion. The metabolic acidosis and renal function
should also be carefully monitored. 4 What additional therapies would be recommended at
this time?
Continuation of case: Upon further discussion, the par- 5 Should other family members be treated as well?
ents did not think that any prophylaxis was needed when
traveling to the Dominican Republic. She is maintained on
the ventilator in pressure-regulated, volume control mode
with the following settings: pressure support 8 cmH2O, pos- What additional therapies would be recommended
at this time?
itive end-expiratory pressure (PEEP) 5 cmH2O, rate
18 breaths per minute, and tidal volume 6 mL/kg. She is Malaria causes anemia due to the high parasite load,
transfused with 15 mL/kg of packed red blood cells. Dextrose sequestration, and hemolysis associated with hypersplen-
5% with 0.45% normal saline is infused at 80 mL/hour. The ism. The choice of malaria medications is based on the
infectious disease team is consulted and requests a malaria infecting species of Plasmodium, possible drug resistance,
rapid diagnostic test (RDT), malaria smear (thick and thin), and the severity of the disease. For travel to the Dominican
and dengue fever polymerase chain reaction (PCR), with Republic, where malaria is chloroquine sensitive, travelers
results as follows: malaria RDT is positive; malaria smear are usually treated with oral chloroquine. However, this
shows 2.6% parasite count. patient’s infection is considered severe because of
Downloaded from https://onlinelibrary.wiley.com/doi/ by National Institute Of Standard, Wiley Online Library on [06/03/2024]. See the Terms and Conditions (https://onlinelibrary.wiley.com/terms-and-conditions) on Wiley Online Library for rules of use; OA articles are governed by the applicable Creative Commons License
430 Cases in Pediatric Acute Care

significant anemia and respiratory failure, so treatment is be carefully monitored for signs and symptoms consistent
started with intravenous quinidine and doxycycline. with malaria for up to 4 weeks after returning from travel.

Should other family members be treated as well?


If other family members who also traveled to the Dominican Case Resolution
Republic are experiencing symptoms such as fever and mus-
cle and body aches, a malaria RDT and thick and thin smears The patient received treatment with quinidine and doxycy-
should be completed and therapy should commence. Oral cline and parasite counts continued to decline. She was
therapy with chloroquine is appropriate in this case after extubated on hospital day 3 and remained hospitalized
travel to the Dominican Republic. Family members should for 10 days prior to being discharged home.

Further Reading
Ashley, E.A., Phyo, A.P., and Woodrow, C.J. (2018). Malaria. Olivero, R.M. and Barnett, E.D. (2018). Malaria. In: Feigin and
Lancet 391(10130): 1608–1621. Cherry’s Textbook of Pediatric Infectious Disease, 8th edn (ed.
Centers for Disease Control and Prevention (2018). Malaria. J.D. Cherry, G.J. Harrison, S.L. Kaplan et al.), 2156–2178.
https://www.cdc.gov/parasites/malaria/index.html Philadelphia, PA: Elsevier.
(accessed 28 March 2019). Taylor, W.R.J., Hanson, J., Turner, G.D.H., White, N.J., and
Kimberlin, D.W., Brady, M.T., and Jackson, M.A. (ed.) (2018). Dondorp, A.M. (2012). Respiratory manifestations of
Malaria. In: Red Book: 2018 Report of the Committee on malaria. Chest 142(2): 492–505.
Infectious Diseases, 31st edn, 527–537. Elk Grove Village, IL: White, N.J. (2014). Malaria. In: Manson’s Tropical Diseases,
American Academy of Pediatrics Committee on Infectious 23rd edn (ed. J. Farrar, P.J. Hotez, T. Junghanss et al.),
Diseases. 532–600. London: Elsevier.
Mathison, B.A. and Pritt, B.S. (2017). Update on malaria World Health Organization (2018). World malaria report 2018.
diagnostics and test utilization. Journal of Clinical https://www.who.int/malaria/publications/world-malaria-
Microbiology 55(7): 2009–2017. report-2018/en/ (accessed 28 March 2019).
Downloaded from https://onlinelibrary.wiley.com/doi/ by National Institute Of Standard, Wiley Online Library on [06/03/2024]. See the Terms and Conditions (https://onlinelibrary.wiley.com/terms-and-conditions) on Wiley Online Library for rules of use; OA articles are governed by the applicable Creative Commons License
431

103

A Serious Football Injury


Julie Resler
Riley Hospital for Children, Indianapolis, IN, USA

A 12-year-old male is admitted to the general surgery/ Current Status


trauma unit for a Grade 3 splenic laceration.
On arrival at the emergency department (ED), a primary
survey is completed showing an intact airway, breath
History of Present Illness sounds clear and equal bilaterally, strong pulses in arms
and legs, Glasgow Coma Scale (GCS) score 15, and football
This 12-year-old was playing in a football game with full
clothing in place with pads removed. He has left-sided
contact and padding, with helmet. He was carrying the ball
abdominal pain with inspiration and positive Kehr’s sign.
and running down the field when he sustained a hit to his
Vital signs are as follows: temperature 37.2 C (99 F), heart
abdomen with the helmet of a defensive player. He initially
rate 120 bpm, respiratory rate 18 breaths per minute, blood
had difficulty catching his breath but was able to participate
pressure 86/50 mmHg, and oxygen saturation 97% on room
in another couple of plays. He then developed significant
air. Peripheral intravenous catheter is placed and complete
abdominal pain and nausea.
blood count, comprehensive metabolic profile, amylase,
and lipase are obtained, and a urinalysis is done to assess
Past Medical History for blood in the urine (see Tables 103.1–103.4). A 20 mL/kg
fluid bolus of normal saline is given to the patient with
The patient’s birth history includes full-term delivery and increase in blood pressure to 110/70 mmHg. A chest radi-
discharge home with his mother. His height and weight ograph is negative. Computed tomography (CT) scan of
are at the 50th percentile and he recently received age the abdomen and pelvis with intravenous contrast is
11 vaccinations and seasonal influenza vaccine. He has a obtained and indicates a Grade 3 splenic laceration
history of attention deficit hyperactivity disorder and takes (Figure 103.1). He is admitted to the pediatric trauma serv-
Vyvanse 20 mg daily. He has never been hospitalized. ice on the general ward, placed on bedrest and is nil by
mouth with intravenous fluids running at a mainte-
nance rate.
Past Surgical History

The patient had an uncomplicated circumcision 2 days after


birth and a tonsillectomy and adenoidectomy at age 5 for Questions
recurrent pharyngitis.
Answer the following questions using the details provided.

Family History 1 Based on the data given, what are the differential diag-
noses for this child?
The parents of this patient are both healthy. He has a 5- 2 What is the best treatment plan for this child?
year-old brother who has a history of mild intermittent 3 What signs would prompt escalation of intervention for
asthma. this child and would any other imaging be needed?

Cases in Pediatric Acute Care: Strengthening Clinical Decision Making, First Edition. Edited by Andrea M. Kline-Tilford and Catherine M. Haut.
© 2020 John Wiley & Sons Ltd. Published 2020 by John Wiley & Sons Ltd.
Downloaded from https://onlinelibrary.wiley.com/doi/ by National Institute Of Standard, Wiley Online Library on [06/03/2024]. See the Terms and Conditions (https://onlinelibrary.wiley.com/terms-and-conditions) on Wiley Online Library for rules of use; OA articles are governed by the applicable Creative Commons License
432 Cases in Pediatric Acute Care

Table 103.1 Comprehensive metabolic profile.

Sodium 140 mEq/L


Potassium 4.1 mEq/L
Chloride 100 mEq/L
Carbon dioxide 24 mEq/L
Blood urea nitrogen 14 mg/dL
Creatinine 0.6 mg/dL
Glucose 148 mg/dL
Calcium 9.5 mg/dL
Albumin 3.5 mg/dL
Total protein 5.0 g/dL
Aspartate aminotransferase 25 U/L
Alanine aminotransferase 43 U/L
Alkaline phosphatase 140 U/L
Total bilirubin 0.1 mg/dL
Magnesium 1.8 mEq/L Figure 103.1 CT of abdomen/pelvis.
Phosphorus 4.2 mg/dL

Rationale and Evidence-based Practice


Explanation
Table 103.2 Complete blood count. Based on the data given, what are the differential
3
diagnoses for this child?
White blood cell count 9500/mm
Hemoglobin 10.8 g/dL The differential diagnoses for this child include splenic lac-
Hematocrit 32.6% eration (noted on CT imaging), additional abdominal
injury/trauma, or hypotension from infectious source not
Platelet count 260 000/mm3
yet identified.
The most likely diagnosis is a splenic laceration as noted
on the CT imaging. Splenic lacerations are graded accord-
ing to a scale from the American Association for the Sur-
Table 103.3 Amylase and lipase. gery of Trauma (AAST) and range from a low grade of 1
to a high grade of 5. A grade 3 injury is defined as a subcap-
Amylase 43 U/L
sular hematoma of more than 50% of the surface area of the
Lipase 56 U/L
spleen, ruptured subcapsular or intraparenchymal hema-
toma greater than 5 cm, or parenchymal laceration more
than 3 cm in depth.

Table 103.4 Urinalysis. What is the best treatment plan for this child?
Splenic lacerations rarely require intervention beyond non-
Color Clear, yellow
surgical monitoring and management. A period of bedrest
pH 7.0
followed by limited activity is generally advised. Pulmonary
Specific gravity 1.010
toilet and judicious pain control should be initiated. Avoid-
White blood cells 0 ance of another direct blow to the abdomen is necessary to
Nitrite Negative allow reabsorption of the blood and to allow healing of the
Protein Negative spleen. Serial abdominal examinations should be per-
Glucose Negative formed. Hemoglobin and hematocrit should be checked
Bilirubin Negative until the counts have stabilized and there is no concern
for ongoing bleeding. A diet can be started early and
Downloaded from https://onlinelibrary.wiley.com/doi/ by National Institute Of Standard, Wiley Online Library on [06/03/2024]. See the Terms and Conditions (https://onlinelibrary.wiley.com/terms-and-conditions) on Wiley Online Library for rules of use; OA articles are governed by the applicable Creative Commons License
Chapter 103 A Serious Football Injury 433

advanced with toleration. Many institutions have protocols to embolize any ongoing bleeding in the spleen. Rarely or
designed to determine intervals for laboratory draws and if the patient is unstable, surgical intervention up to and
activity restrictions. The newest guidelines, published by including a splenectomy may be necessary. This is usually
the Pediatric Trauma Society, utilize the ATOMAC only required in high-grade (Grade 4 and 5) injuries. Stud-
(Arizona–Texas–Oklahoma–Memphis–Arkansas Consor- ies indicate that 96–98% of splenic injuries in children can
tium), which consists of a group of Level I pediatric trauma be managed with non-operative intervention.
centers from across the United States who have participated Additional follow-up imaging is not necessary if the
in research to improve outcomes for children who suffer patient is progressing and shows no signs of ongoing bleed-
trauma injuries. The publication in 2015 offers a practice ing. Continued radiation exposure, such as that with CT
guideline with four highlights. Guidelines include manag- imaging, can increase the lifetime risk of cancer in pediatric
ing liver and spleen trauma based on hemodynamic stabil- patients.
ity versus grade of injury, a period of as little as 24 hours
bedrest, and limiting transfusions to those who have hemo-
globin levels below 7 g/dL.
Case Progression and Resolution
What signs would prompt escalation of intervention
for this child and would any other imaging be
The patient is admitted to the general ward. His hemoglo-
needed?
bin and hematocrit are checked every 12 hours until stable.
For this patient, follow-up with the trauma surgeon may be He is kept on bedrest until hospital day 3 and then allowed
necessary depending on the grade of injury. Ongoing signs to ambulate. He is started on incentive spirometry while on
of bleeding (tachycardia, hypotension not responsive to bedrest and is allowed a clear liquid diet on hospital day 1
fluid bolus or blood administration, decreasing hemoglobin and then advanced to a regular diet. He is given intravenous
and hematocrit) would prompt one to escalate intervention. opiate medication while nil by mouth and changed to
A “blush” on the CT imaging may indicate ongoing and/or enteral opiate medication when tolerating a diet. His hemo-
arterial bleeding and may necessitate intervention. If the globin and hematocrit remain stable. He is discharged to
patient is stable, interventional radiology can be utilized home on hospital day 4.

Further Reading
Linnaus, M.E., Langlais, C.S., Garcia, N.M., et al. (2017). GRADE. Journal of Trauma and Acute Care Surgery
Failure of nonoperative management of pediatric blunt liver 79(4): 683–693.
and spleen injuries. Journal of Trauma and Acute Care Wisner, D.H., Kuppermann, N., Cooper, A., et al. (2015).
Surgery 82(4): 672–679. Management of children with solid organ injuries after
Notrica, D.M., Eubanks, J.W., Tuggle, D.W., et al. (2015). blunt torso trauma. Journal of Trauma and Acute Care
Nonoperative management of blunt liver and spleen injury Surgery 79(2): 206–214.
in children: evaluation of the ATOMAC guideline using
Downloaded from https://onlinelibrary.wiley.com/doi/ by National Institute Of Standard, Wiley Online Library on [06/03/2024]. See the Terms and Conditions (https://onlinelibrary.wiley.com/terms-and-conditions) on Wiley Online Library for rules of use; OA articles are governed by the applicable Creative Commons License
435

104

Infant with Generalized Weakness


Blythe Pollack
C.S. Mott Children’s Hospital, Ann Arbor, MI, USA

A 5-month-old female with no significant medical history is Past Surgical History


admitted to the pediatric intensive care unit (PICU) with
respiratory failure and generalized weakness of unclear No past surgical history.
etiology.

Family History
History of Present Illness
No significant medical history. No neuromuscular diseases.
This 5-month-old female presented to the emergency Two older siblings, both currently well, one of which
department (ED) with her mother due to increased work attends public school. Both without medical disease, but
of breathing and poor oral intake. In the hours preceding do suffer from the occasional virus.
the visit, she had poor latch with breastfeeding and
decreased intake. Bottle feeds were attempted, but unsuc-
cessful. She was taken to the urgent care in the morning
Current Status
hours for concern of constipation, and sent home with a
prescription of polyethylene glycol. Hours after she
On presentation to the ED at an outside hospital, her vital
returned home, the infant had a sudden increase in her
signs are stable and she is afebrile. Influenza and respira-
work of breathing, audible chest congestion, weak cry,
tory syncytial virus swab are both negative. Peripheral
and lethargy.
intravenous catheter is placed and blood work obtained.
Secondary to her respiratory compromise, she is presumed
to have bronchiolitis and receives nebulized albuterol. Dur-
Past Medical History ing her albuterol inhalation, she desaturates to 80% but
quickly recovers. Because of her poor oral intake and gen-
She was a full-term infant, born via non-complicated vagi- eral malaise, she is admitted to the general pediatric floor
nal delivery. She did not require airway interventions at for observation and intravenous fluids.
birth. She did have coughing/choking episodes with initial Results of laboratory tests are shown in Tables 104.1 and
feeds, was diagnosed with reflux, and was repositioned 104.2. Virus studies were negative for influenza A and
more upright for feedings. She had no reoccurrence of these B and respiratory syncytial virus
episodes. She was discharged home with her mother fol-
lowing mothers’ postpartum stay. No other hospital admis-
sions or urgent care/ED visits. No preceding illnesses. She is
primarily breast-fed, only taking from a bottle as needed.
Questions
She was started on fruit purees a couple of days prior to
Answer the following questions using the details provided.
presentation. She did not ingest honey. There is some con-
struction happening across the street from their apartment 1 Based on the data given, formulate a differential diag-
complex. There are many farms in the surrounding area. nosis for this child?

Cases in Pediatric Acute Care: Strengthening Clinical Decision Making, First Edition. Edited by Andrea M. Kline-Tilford and Catherine M. Haut.
© 2020 John Wiley & Sons Ltd. Published 2020 by John Wiley & Sons Ltd.
Downloaded from https://onlinelibrary.wiley.com/doi/ by National Institute Of Standard, Wiley Online Library on [06/03/2024]. See the Terms and Conditions (https://onlinelibrary.wiley.com/terms-and-conditions) on Wiley Online Library for rules of use; OA articles are governed by the applicable Creative Commons License
436 Cases in Pediatric Acute Care

Table 104.1 Basic metabolic profile. What other diagnostic studies can be completed to
help differentiate diagnosis? How can they change
Sodium 139 mEq/L management?
Potassium 5.4 mEq/L
To gain clarity on her diagnosis a multitude of additional
Chloride 108 mEq/L
diagnostic studies can be completed. Procalcitonin
Carbon dioxide 18 mEq/L
(marker of inflammation) can help differentiate from a
Blood urea nitrogen 12 mg/dL bacterial infection and better elucidate the need for anti-
Creatinine <0.3 mg/dL biotics. A chest radiograph would be helpful to evaluate
Glucose 86 mg/dL for airspace disease, aspiration, consolidation, and ate-
Calcium 10.4 mg/dL lectasis. Continued monitoring of electrolytes will lead
Alkaline phosphatase 137 U/L to supplementation or identification of disturbances.
Albumin 4.3 g/dL Laboratory values can be beneficial in the identification
of fatigue related to hyponatremia or hypoglycemia in
the setting of poor oral intake. A blood gas can be used
for trending in the setting of impending respiratory
Table 104.2 Complete blood count.
failure.
White blood cell count 13 700/mm3 Continuation of case: Within the first 12 hours of admis-
Hemoglobin 12.2 g/dL sion, she continues to have worsening weakness, proximal
Hematocrit 36.8% greater than distal, and intermittent hypoxia. A chest radi-
Platelet count 441 000/mm3 ograph is completed with concern for bilateral airspace dis-
Neutrophils 29% ease, and head computed tomography (CT) is completed
Eosinophils 1%
and interpreted as normal. High-flow nasal cannula is
initiated and she is transferred to the intensive care unit
Monocytes 8%
(ICU). Following transfer to the ICU, she has increased fre-
Lymphocytes 62%
quency of oxygen desaturations, declining mental status,
and hypotonia. Subsequently, her trachea is orally intu-
bated for respiratory support in the setting of neurologic
2 What other diagnostic studies can be completed to decompensation.
help differentiate diagnosis? How can they change
management?
Additional Questions and Evidence-
based Rationale
Rationale and Evidence-based Practice 3 What differential diagnosis are you considering with
Explanation the changes in patient status?
4 What other diagnostic studies can be completed to
Based on the data given, formulate a differential help differentiate diagnosis? How can they change
diagnosis for this child? management?
Based on the presenting symptoms of increased work of
breathing and poor oral intake, it is reasonable to consider
What differential diagnosis are you considering
a primary respiratory issue. Her poor oral intake can be
with the changes in patient status?
related to her work of breathing or other possibilities.
The differential diagnoses include viral bronchiolitis, viral The clinical decompensation in addition to hypotonia
pneumonia, or upper respiratory infection. When consider- expands the diagnosis to include, but not limited to, hypo-
ing her general malaise, constipation, and increased work thyroidism, inborn errors of metabolism, encephalitis, poli-
of breathing, her differential diagnosis should widen. Addi- omyelitis, ingestion/poisoning, neuromuscular junction
tionally, dehydration, failure to thrive, spinal muscular disorders (botulism, myasthenia gravis), demyelinating
atrophy, suspected sepsis, or meningitis should be consid- disorders (Guillain–Barré syndrome/Miller–Fisher syn-
ered. With such vague symptomology the differential drome), acute flaccid myelitis, axonal neuropathy, and
should be expanded and narrowed through the hospi- myopathy/myositis. Symptoms were consistent with bulbar
tal stay. paralysis. Referring to lower brainstem area or medulla
Downloaded from https://onlinelibrary.wiley.com/doi/ by National Institute Of Standard, Wiley Online Library on [06/03/2024]. See the Terms and Conditions (https://onlinelibrary.wiley.com/terms-and-conditions) on Wiley Online Library for rules of use; OA articles are governed by the applicable Creative Commons License
Chapter 104 Infant with Generalized Weakness 437

oblongata, control of cranial nerves VII–XII. Term bulbar Table 104.3 Initial CSF analysis.
can be used for lesions at any level (muscle, neuromuscular
Color Colorless
junction, peripheral nerves, motor nuclei).
Clarity Clear
What other diagnostic studies can be completed to Xanthochromia Absent
help differentiate diagnosis? How can they change Red blood cells 1/mm3
management? Protein 15 g/L

Any patient with unknown disease etiology involving the Glucose 126 mg/dL
respiratory system and neurologic decompensation war-
rants a full septic work-up. Blood, urine and respiratory cul-
tures, along with a chest radiograph, should be completed
for further evaluation. Lumbar puncture should be per- change. A muscle biopsy is completed. Viral studies are
formed to evaluate for meningitis, and viral studies from sent for evaluation, including human herpesvirus 7, cyto-
cerebrospinal fluid (CSF) should be sent for evaluation. megalovirus (CMV), enterovirus, Epstein–Barr virus, her-
There are a number of adjunctive tests that can be used pes simplex virus, and West Nile virus. She is treated with
to help determine the cause of a neurologic insult. EEG ganciclovir for 5 days for a CMV viral load above 1400
should be completed to assess for seizure activity. EMG copies/mL. She receives 48 hours of vancomycin and cef-
can be completed to assess for acute flaccid paralysis or bot- triaxone for initial concern for sepsis which is discontin-
ulism but results are not always definitive and can cause ued following 48 hours of negative cultures and absence
discomfort, so the benefits should be weighed. A nerve con- of fever. Enterovirus sent to the state laboratory for further
duction study can typically be completed at the same time subtyping.
as EMG and is used to evaluate the speed of the electrical The current focused neurologic examination produces
impulse and can eliminate the Miller–Fisher variant. The the following results.


Tensilon test, which involves the injection of edrophonium
chloride, can be sensitive for myasthenia gravis. The Tensi- Mental status: she is encephalopathic with no spontane-
lon test is influenced by patient response and may produce ous eye opening. She has spontaneous, but not vigorous
false-positive results, with diagnoses such as botulism, movements of the bilateral hand and feet, with deep
stimulation.


Guillain–Barré syndrome, motor neuron diseases, and
brainstem lesions. Cranial nerves: pupils are 3 mm with briskly reactive
Each diagnostic study by itself is not definitive, though a pupil on the left, 5 mm sluggishly reactive pupil on the
diagnosis can be made with additional information from right. Her facial structures appear symmetric. She has
CSF studies, laboratory results, and clinical presentation. midline tongue movement without fasciculations. No
cough or gag reflex is noted. She has weak dolls eye
Continuation of case: During her first few days of ICU and absent corneal reflexes.
admission, she has decline in many body systems. Routine
blood work is completed, along with blood, urine and spu- • Motor: diffuse peripheral hypotonia with normal bulk
throughout. She has 2+ strength of the fingers, wrists
tum cultures. She requires moderate to high ventilator set- and elbows of the upper extremities. She has 1+ strength
tings, with increased atelectasis and large mucous plugs. in the bilateral shoulders, 2+ strength in bilateral ankles
Cardiac echocardiogram is completed with normal func- and knees, and 1+ strength in her bilateral hips.
tion and anatomy, despite persistent hypertension. She
no longer voids independently, and requires intermittent The patient is transferred to a children’s hospital in the
straight catheterization. Iatrogenic electrolyte disturbances setting of unknown etiology of her neurologic and respira-
are corrected with initiation of continuous feeds. A second tory decompensation. On arrival to the center of higher
viral swab is completed, which is positive for rhinovirus/ care, multiple services are consulted, including neurology,
enterovirus. Lumbar puncture is completed twice, with infectious disease, and pulmonology. The EMG, EEG, brain
the results shown in Table 104.3. Electroencephalography and spine MRI, and lumbar puncture are repeated. Blood
(EEG) shows stage 2 sleep, no wakefulness, and no sei- and stool specimens for botulism are sent for evaluation.
zures. Magnetic resonance imaging (MRI) of her brain Medications are avoided which could interfere with neuro-
and spine is completed without acute findings. Electromy- muscular junction functions. Antibiotics are not initiated,
ography (EMG) shows active myopathy or primarily as the etiology of illness did not appear infectious. BabyBIG
axonal motor neuropathy. A 48-hour course of intrave- (human botulism immunoglobulin) is administered prior
nous gamma globulin (IVIG) is completed without clinical to any results.
Downloaded from https://onlinelibrary.wiley.com/doi/ by National Institute Of Standard, Wiley Online Library on [06/03/2024]. See the Terms and Conditions (https://onlinelibrary.wiley.com/terms-and-conditions) on Wiley Online Library for rules of use; OA articles are governed by the applicable Creative Commons License
438 Cases in Pediatric Acute Care

The radiology repeat imaging and stool sampling produce Additional Questions and Evidence-
the following results. based Rationale

• MRI: negative for intracranial mass, midline shift, extra-


axial fluid collection or hemorrhage. The ventricles, sulci
5 What is the difference between an EEG and EMG? In
what situation would you order either or both studies?
and cisterns are reported as normal. There are no abnor-
6 Should the provider be concerned for a toxic ingestion
mal areas of altered signal intensity or enhancement.
this far into her admission and what could still be eval-
There is no sign of infarction. The visualized aspects of
uated for?
the orbits, paranasal sinuses, and the mastoid air cells
7 What is infantile botulism and how can it be prevented?
are normal. MRI brain and spine: susceptibility signals
8 What is BabyBIG and how does it work?
in the left cerebellar hemispheres may represent a sta-
ble-appearing small cavernoma. No evidence of acute
ischemia or acute parenchymal/extraparenchymal hem- What is the difference between an EEG and EMG?
orrhage present in the study. No convincing evidence of In what situation would you order either or both
spinal cord compression present. No abnormal enhance- studies?
ment of the spinal cord. No abnormal thickening of the
cauda equina or the exiting nerve roots noted. No abnor- EEG is the process of attaching electrodes to the scalp to
mal enhancement of the structures. evaluate the electrical activity of the brain. The activity is

• EMG: abnormal examination. There is electrodiagnostic


evidence of a severe motor axonal polyneuropathy with
received from the neurons on the outermost layers of the
cerebral cortex. EEG can be used to observe and identify
active denervation, with the differential diagnoses refractory status epilepticus, achievement of cerebral
including acute motor axonal neuropathy (AMAN) and silence, treatment monitoring of traumatic brain injury,
anterior horn cell disorder. There is electrodiagnostic evi- metabolic encephalopathies, and neurologic conditions
dence of a severe sensory neuropathy or a defect in neu- that inhibit patient responses. EEG continues to be a con-
romuscular junction transmission. venient evaluation of continuous, real-time monitoring of

• EEG on day 1 of admission: normal EEG demonstrates


slowing, encephalopathic features. There is also noted
brain function and is supplemental to the clinical
examination.
to be normal features of sleep seen intermittently as well. EMG is a test that measures electrical signals. The proc-
The background is reactive. No seizures and no epilepti- ess consists of inserting into a muscle a small needle that
form features are recorded. sends an electrical signal causing the muscle to contract.

• EEG on day 4 of admission: abnormal tracing for age.


There is normal sleep with no wakefulness despite nox-
The contraction is recorded in as a number, wave or sound,
which is interpreted by a specialist. EMG can be used to
ious stimulation. Intermittent focal slowing is noted over detect nerve or muscle dysfunction, or a dysfunction in
the left parietal region. their communication process. In this case, the EMG did

• Muscle biopsy: there is no evidence of abnormal central


nuclei, cytoplasmic inclusions, or ragged red fibers. Focal
not support a diagnosis of botulism.

mild endomysial inflammation is non-specific but is con- Should the provider be concerned for a toxic
sistent with immune reactive changes that could be due ingestion this far into her admission and what
to viral infection or an autoimmune process. Clinical cor- could still be evaluated for?
relation is warranted.

••
With an unknown neurologic decompensation, toxic inges-
CSF: results shown in Table 104.4. tions should be considered early and urine and/or blood
Stool specimen: Clostridium botulinum type A confirmed samples sent for evaluation. A week into her inpatient stay,
by mouse inoculation. evaluating for several toxic ingestions will not have much
utility in reaching a diagnosis. Evaluating for alcohol is
Table 104.4 Repeat CSF analysis. not useful, as it would have left her system this far into
her course. Acetaminophen and salicylate levels are easily
Color Colorless evaluated with a serum sample but this far into the course
Clarity Clear the levels would be decreased to non-existent. Cholinergic
Xanthochromia Absent toxidromes can have nicotinic effects with symptoms of
Red blood cells 2/mm3 weakness, paralysis, agitation and tachycardia; agents such
Protein 16 g/L as organophosphate pesticides present this way. Typical
Glucose 73 mg/dL presentation following inhalation is respiratory distress,
whereas emesis and drooling are typical clinical signs with
Downloaded from https://onlinelibrary.wiley.com/doi/ by National Institute Of Standard, Wiley Online Library on [06/03/2024]. See the Terms and Conditions (https://onlinelibrary.wiley.com/terms-and-conditions) on Wiley Online Library for rules of use; OA articles are governed by the applicable Creative Commons License
Chapter 104 Infant with Generalized Weakness 439

oral ingestion. Airway management, atropine to reverse What is BabyBIG and how does it work?
muscarinic effects, and benzodiazepines to control seizures
BabyBIG is the human botulism immunoglobulin used for
are typical management strategies. Gas chromatography–
treatment of botulism. It is administered as soon as there is
mass spectrometry detects compounds even in small quan-
suspicion of infant botulism. It is an intravenous medica-
tities in the urine but hold low utility at this time. Consid-
tion that helps reverse the toxemia by neutralizing the cir-
ering further evaluation of organophosphates, lead,
culating toxins and preventing further disruption of the
cadmium, and arsenic could narrow the differential and
neuromuscular junction. BabyBIG has immediate onset,
help direct further treatment.
with an average half-life of 28 days. The most common
adverse reaction is skin rash, occurring in less than 5% of
What is infantile botulism and how can it be
patients. As with any immunoglobulin administration,
prevented?
infusion should be initiated slowly with close monitoring
In the United States, 99.8% of cases of infantile botulism of vital signs for the first hour; with stability, rate of infusion
have occurred in infants and children under 1-year-old, can increase. There is risk of anaphylaxis, and if this occurs
with 91.2% found in children under 6 months old. Illness the infusion should be discontinued, followed by adminis-
occurs equally across ethnic and racial groups, with equal tration of epinephrine. If patients have a known history of
instances in males and females. Clostridium botulinum IgA deficiency, they can develop further antibodies to IgA,
toxin is subclassified into types A–F, the most common which would put them at higher risk for anaphylaxis to any
being A and B. Infantile botulism occurs when the C. bot- subsequent blood product transfusions.
ulinum toxin is ingested/inhaled and the spores enter the
digestive system. The spores will then grow and release
the toxin. The botulinum toxin binds at the presynaptic seg-
ment of the neuromuscular junction and prevents acetyl- Case Resolution
choline from being released into the neuromuscular
junction. Ten days following admission to the higher level of care,
The commonly discussed causative agent of infant botu- her stool sample resulted positive for botulism. She demon-
lism is honey, even though it is uncommon for the ingestion strated improvement over the next weeks as an inpatient.
of honey to be linked with reported cases of infant botulism. She was successfully extubated 23 days following her initial
Avoidance of honey in infants under 12 months of age is a presentation and intubation. Four months following her
preventative strategy, but does not guarantee the develop- admission she was discharged home requiring non-invasive
ment of botulism from a different source. Clostridium bot- positive pressure for nocturnal hypoventilation, and naso-
ulinum is a bacterium that inhabits soil and dust and is gastric feeds for oral aversion. She was able to tolerate sup-
therefore found inside and outside the home. The distribu- ported sitting, roll from side to side, though was not yet
tion of confirmed cases nationally has been linked to the supporting her own weight to crawl. She continued occupa-
same botulism gene in the soil of a particular area. tional and physical therapy as an outpatient.

Further Reading
Carrillo-Marquez, M. (2016). Botulism. Pediatrics in Review children: a systematic review. Clinical Infectious Diseases
37(5): 183–192. doi: 10.1542/pir.2015-0018. 66(Suppl 1): S17–S29. doi: 10.1093/cid/cix812.
Centers for Disease Control and Prevention (2019). Kinds of Opila, T., George, A., El-Ghanem, M., and Souayah, N. (2017).
botulism. Available at https://www.cdc.gov/botulism/ Trends in outcomes and hospitalization charges of infant
definition.html (accessed 11 March 2019). botulism in the United States: a comparative analysis
Choi, J. and McCarthy, M. (2017). Novel applications for serum between kids’ inpatient database and national inpatient
procalcitonin testing in clinical practice. Expert Review of sample. Pediatric Neurology 67: 53–58. doi: 10.1016/j.
Molecular Diagnostics 18(1): 27–34. doi: 10.1080/ pediatrneurol.2016.10.009.
14737159.2018.1407244. Shaffner, D. and Nichols, D. (ed.) (2016). Rogers’ Textbook of
Griese, S., Kisselburgh, H., Bartenfeld, M., et al. (2017). Pediatric Intensive Care, 5th edn. Philadelphia, PA: Wolters
Pediatric botulism and use of equine botulinum antitoxin in Kluwer.
Downloaded from https://onlinelibrary.wiley.com/doi/ by National Institute Of Standard, Wiley Online Library on [06/03/2024]. See the Terms and Conditions (https://onlinelibrary.wiley.com/terms-and-conditions) on Wiley Online Library for rules of use; OA articles are governed by the applicable Creative Commons License
441

105

A 3-Day-Old with Irritability


Christopher D. Newman
Children’s Hospital Colorado, Aurora, CO, USA

A 3-day-old female infant is brought to the emergency who are both healthy. Both were born at home. Mother
department (ED) by her mother due to irritability and ina- reports several cousins that died in infancy, but she is not
bility to feed. sure about the cause of death. The mother has no health
problems, but she believes that the paternal grandfather
had “liver issues.”
History of Present Illness

She was delivered at home with a birth coach via vaginal


delivery. She was initially “a bit floppy” but this resolved Current Status
after her first feed. Over the next 48 hours, she appeared
normal and active, but would become irritable, “jittery” The infant is examined in her mother’s arms. She is pale,
and sweaty as feeding time approached. These symptoms diaphoretic and listless. Her fontanelle is mildly sunken.
would all resolve after feeding. Early this morning, the Her pupils are reactive but she does not fix and follow.
infant slept for several hours and when her mother Her lips are dry. She is tachypneic with a respiratory rate
attempted to wake her, she was pale, sweaty, listless and of 60 breaths per minute, though her lung fields are clear
tremulous. Mother attempted to feed her, but she was list- to auscultation. She is tachycardic with a heart rate of
less and would not latch or suck consistently. Her mother 180 bpm with a regular rhythm and no murmurs, gallops
reports that the infant’s “heart was racing” and after an or rubs. She has 1+ peripheral pulses and 2-second capil-
hour of attempting to feed without success, she became lary refill peripherally. The abdomen is soft, with the liver
“floppy” and unresponsive, and was brought to the ED edge 1 cm below the costal margin. The spleen is not pal-
for assistance. pated and bowel sounds are sparse but present. There are
The infant’s mother denies fevers, cough, congestion, or no rashes or extremity deformities.
eye discharge. The child has not vomited nor had diarrhea. You request that a peripheral intravenous catheter be
Nobody in the family has been ill recently. She has not tra- placed, laboratory samples obtained, and maintenance
velled and there are no medications in the home. intravenous fluids are started. You order a bedside blood
glucose check, complete blood count, comprehensive met-
abolic panel, blood culture, coagulation studies and procal-
Past Surgical Hisotry citonin for initial evaluation. The nurse notifies you that the
glucometer blood glucose is 25 mg/dL.
No surgical history.

Family History Questions

Answer the following questions using the details provided.


The family are recent immigrants to this country and the
mother admits that she does not have complete details of 1 Based on the history, clinical presentation and labora-
family health history. The infant has two older siblings tory values, what is your differential diagnosis?

Cases in Pediatric Acute Care: Strengthening Clinical Decision Making, First Edition. Edited by Andrea M. Kline-Tilford and Catherine M. Haut.
© 2020 John Wiley & Sons Ltd. Published 2020 by John Wiley & Sons Ltd.
Downloaded from https://onlinelibrary.wiley.com/doi/ by National Institute Of Standard, Wiley Online Library on [06/03/2024]. See the Terms and Conditions (https://onlinelibrary.wiley.com/terms-and-conditions) on Wiley Online Library for rules of use; OA articles are governed by the applicable Creative Commons License
442 Cases in Pediatric Acute Care

2 What, if any, additional laboratory testing would acidosis, hypoglycemia may be present due to associated
you order? liver dysfunction or protein malnutrition.
3 What, if any, treatment would you administer?
What, if any, additional laboratory testing would
you order?
Rationale and Evidence-based Practice It is essential in diagnosing the cause of hypoglycemia that
Explanation laboratory testing be done on specimens obtained during
the state of hypoglycemia. In an unstable patient, the
Based on the history, clinical presentation and patient’s safety comes first. However, ideally, the following
laboratory values, what is your differential studies will be obtained prior to treatment of the hypogly-
diagnosis? cemia: plasma glucose, C-peptide, β-hydroxybutyrate, free
fatty acids, acylcarnitine profile, lactate, and ammonia.
In any neonate presenting with hypoglycemia, sepsis
Urine organic acids should also be sent. These laboratory
should be considered. If there are any infectious symptoms,
values can then be used to identify the most likely cause
treatment should begin immediately while an evaluation is
of hypoglycemia from the differential diagnosis.
performed. In this case, there is no clinical evidence of
infection, so laboratory testing for evidence of sepsis is
What, if any, treatment would you administer?
appropriate. Hypoglycemia in infancy can generally be
divided into broad categories: inadequate glucose supply In older children who are alert enough to swallow safely,
(either through inadequate intake or due to defects in fatty 10–20 g of rapidly absorbed glucose, such as honey, table
acid oxidation, carbohydrate metabolism or amino acid sugar, glucose tablets or glucose gel, may be administered.
metabolism) or increased glucose utilization (such as In the case of infants such as this patient, the ideal treat-
hyperinsulinism or some ingestions). Infants will rapidly ment is intravenous administration of dextrose 0.25 g/kg
exhaust their glycogen stores without glucose intake and (maximum 25 g). In an unstable patient where intravenous
are therefore less tolerant of fasting. Additionally, illness access cannot be achieved, administer glucagon 0.03 mg/kg
that increases metabolic demand can exhaust glucose intramuscularly. Glucagon will effectively treat hypoglyce-
supply even in a feeding infant if the infant cannot keep mia due to increased glucose utilization, but may not treat
up with the increased metabolic demand. Common inges- other causes and the effect may be transient. Efforts should
tions associated with hypoglycemia can include ethanol, continue to establish intravenous access and administer
salicylates, sulfonylureas, and beta-blockers. intravenous dextrose.
Hyperinsulinism often presents in the first few weeks of
Continuation of case: The results for basic metabolic pro-
life. The most common cause is congenital hyperinsulin-
file and complete blood count and coagulation are shown in
ism, although insulinomas and exogenous insulin adminis-
Tables 105.1 and 105.2. Additional laboratory test results
tration are also possibilities. In these conditions, insulin is
are as follows:
secreted out of proportion to glucose level and the usual
negative feedback loop that disrupts insulin production
when blood glucose levels fall below 60 mg/dL is inacti- •• blood culture results pending, Gram stain negative
insulin <2 μIU/mL
vated. Fatty acid oxidation disorders can include carnitine
deficiency, fatty acid transport defects, and defects of beta- •• C-peptide 0.4 ng/mL
β-hydroxybutyrate 0.3 mmol/L
oxidation enzymes. The most common disorder is medium-
chain acyl-CoA dehydrogenase deficiency. The exact mech- •• free fatty acids 0.3 mmol/L
acylcarnitine profile normal
anism of hypoglycemia is not understood, but since ketone
bodies cannot be manufactured without mobilization of •• lactate 4 mmol/L
ammonia 80 μg/dL
free fatty acids, the absence of ketone bodies as an energy
source may play a role. Disorders of carbohydrate metabo- • urine organic acids normal.

lism can impact metabolism of glycogen, galactose and


fructose. This group includes the glycogen storage diseases
(such as glycogen synthetase deficiency), disorders of glyco- Additional Questions and Evidence-
sylation (such as phosphoglucomutase 1 deficiency) and based Rationale
disorders of gluconeogenesis (such as pyruvate carboxylase
deficiency). Although disorders of amino acid metabolism, 4 Given the laboratory values, what is the most likely
such as methylmalonic acidemia and propionic acidemia, cause of this patient’s hypoglycemia?
typically present with an episode of profound metabolic 5 How can this diagnosis be confirmed?
Downloaded from https://onlinelibrary.wiley.com/doi/ by National Institute Of Standard, Wiley Online Library on [06/03/2024]. See the Terms and Conditions (https://onlinelibrary.wiley.com/terms-and-conditions) on Wiley Online Library for rules of use; OA articles are governed by the applicable Creative Commons License
Chapter 105 A 3-Day-Old with Irritability 443

Table 105.1 Basic metabolic profile. they are generally associated with facial dysmorphisms,
developmental delay, and multiorgan impacts and are gen-
Sodium 142 mEq/L
erally not seen in the neonatal period. Given the age at pres-
Potassium 5.2 mEq/L entation, association with fasting, resolution with glucose
Chloride 102 mEq/L administration, presence of lactic acid, and absence of
Carbon dioxide 16 mEq/L ketone bodies, the most likely diagnosis is a glycogen
Blood urea nitrogen 11 mg/dL storage disease. While there are over 20 identified glycogen
Creatinine 0.6 mg/dL storage diseases, only five are associated with
Calcium 2 mmol/L
hypoglycemia.
Glycogen synthetase deficiency (GSD 0) typically pre-
Phosphorus 2 mmol/L
sents with fasting ketosis, postprandial hyperglycemia,
Magnesium 1 mmol/L
and absence of hepatomegaly.
Aspartate aminotransferase 50 U/L Glucose 6-phosphatase deficiency (GSD I) is the most
Alanine aminotransferase 35 U/L common deficiency (accounting for over 80% of cases)
Alkaline phosphatase 200 U/L and has two subtypes, a and b. Typical presentation is hypo-
Total bilirubin 160 μmol/L glycemia with lactic acidemia and hepatomegaly. Mild to
Albumin 3 g/dL moderate ketosis may be present or absent. When patients
Procalcitonin 0.2 μg/L present later in life, additional findings can include growth
retardation, delayed puberty, hyperuricemia, platelet dys-
function, and hyperlipidemia. GSD Ib is similar to GSD
Table 105.2 Complete blood count and coagulation. Ia, but often includes neutropenia. This is the most likely
diagnosis for this patient. Debrancher deficiency (GSD
White blood cell count 10 000/mm3 III) may present with hypoglycemia if there is liver involve-
Hemoglobin 17 g/dL ment. Hypoglycemia is generally milder than in GSD
Hematocrit 55% I because gluconeogenesis still functions. This is a ketotic
Platelet count 310 000/mm3 hypoglycemia and ketone bodies are typically markedly ele-
Prothrombin time (PT) 12 s vated. Hepatic phosphorylase deficiency may present with
Partial thromboplastin time (PTT) 35 s
hypoglycemia, but lactic acid level is usually normal,
ketones are elevated, while transaminases are usually
International normalized ratio 1.0
mildly elevated. Hepatic phosphorylase b kinase deficiency
can present with hypoglycemia, but the disease itself is typ-
ically mild and a presentation this early would be unusual.
Given the laboratory values, what is the most
The hypoglycemia itself is usually mild and there is usually
likely cause of this patient’s hypoglycemia?
an elevated ketone level associated with the hypoglycemia.
A normal white blood cell count and a procalcitonin below
0.5 μg/L in the absence of infectious symptoms make sepsis How can this diagnosis be confirmed?
extremely unlikely. Accidental ingestion in a neonate is
also very unlikely and there are no associated agents in First-line testing is DNA assessment for known mutations.
the home. Salicylate ingestion could explain the hypoglyce- If none of the common mutations are identified, whole
mia and acidosis. If suspicion were higher, a salicylate level exome sequencing can be done to look for less common
could be obtained to exclude this diagnosis. A suppressed mutations. In the event that no mutation can be identified,
insulin level and C-peptide level can exclude hyperinsulin- a liver biopsy may be necessary to confirm the functional
ism as a cause of the hypoglycemia. Similarly, a normal free defect.
fatty acid level and normal organic acid levels can exclude
fatty acid oxidation or amino acid metabolism defects.
The resolution of symptoms following glucose adminis- Case Progression and Resolution
tration, the pattern of symptoms following fasting, the find-
ing of hepatomegaly, and the presence of lactic acidemia Following intravenous administration of dextrose
are all most consistent with a carbohydrate metabolism dis- 0.25 g/kg, the patient is more alert and active. She is less
order. Disorders of gluconeogenesis generally cause ketotic diaphoretic, but continues to be tachycardic. Her repeat
hypoglycemia. glucometer check is 82 mg/dL. Avoidance of fasting, fre-
While there are over 130 recognized glycosylation disor- quent glucose monitoring, and a genetics team consult
ders and many can include hypoglycemia at presentation, are initiated.
Downloaded from https://onlinelibrary.wiley.com/doi/ by National Institute Of Standard, Wiley Online Library on [06/03/2024]. See the Terms and Conditions (https://onlinelibrary.wiley.com/terms-and-conditions) on Wiley Online Library for rules of use; OA articles are governed by the applicable Creative Commons License
444 Cases in Pediatric Acute Care

Further Reading
Bhattacharya, K. (2015). Investigation and management of the Thornton, P.S., Stanley, C.A., De Leon, D.D., et al. (2015).
hepatic glycogen storage diseases. Translational Pediatrics Recommendations from the Pediatric Endocrine Society for
4(3): 240–248. evaluation and management of persistent hypoglycemia in
Chang, I.J., He, M., and Lam, C.T. (2018). Congenital disorders neonates, infants, and children. Journal of Pediatrics 167(2):
of glycosylation. Annals of Translational Medicine 238–245.
6(24): 477.
Downloaded from https://onlinelibrary.wiley.com/doi/ by National Institute Of Standard, Wiley Online Library on [06/03/2024]. See the Terms and Conditions (https://onlinelibrary.wiley.com/terms-and-conditions) on Wiley Online Library for rules of use; OA articles are governed by the applicable Creative Commons License
445

106

Child with Fever and Rash


Misty Evans
Vanderbilt College of Nursing, Nashville, TN, USA

A 7-year-old girl presents with her mother to the emergency Past Medical History
department (ED) with a 3-day history of nausea, vomiting,
stomach ache, fever, and skin rash. Born at term and went home from the hospital with her
mother. She is a healthy child with no significant past med-
ical history. She is up to date on all required vaccines along
with annual influenza vaccine. She weighs 25 kg and height
History of Present Illness is 125 cm, both at 75th percentile for gender and age. No
known allergies.
This 7-year-old girl initially became ill 2 days ago with
fever of 38.3 C (101 F), stomach ache, nausea, vomiting,
and decreased appetite. Mother reports that several chil-
Past Surgical History
dren in the child’s school were recently sick with a “stom-
ach virus.” The child was provided symptomatic care at
She has no prior surgical history.
home with oral fluids, acetaminophen for fever, and rest.
Overnight, the child’s fever increased to 39.3 C (102.7 F).
Her mother administered acetaminophen every 6 hours
and the last dose was given 2 hours ago. This morning,
Family History
a red rash was noted on her wrists and ankles. Worsening
No significant family medical history. Mother, father, and
fever and the new rash prompted her mother to bring the
9-year-old brother are alive and healthy.
child to the ED for further evaluation. The child also com-
plains of intermittent headache and generalized body
aches which are relieved with the acetaminophen given
for fever. She reports abdominal pain with no associated Social History
factors that make the pain better or worse. She vomited
three times yesterday and once this morning. Emesis is She is a first-grade student and lives in a house with her
not bloody or bilious. She is not eating or drinking as usual mother, father, and 9-year-old brother. There is no tobacco
and is having decreased urinary output. She has had reg- exposure in the home. They have one dog in the home. No
ular soft bowel movements with no diarrhea or constipa- other pets.
tion. She has had no sick contacts in the home. She has not
had any recent travel. She has not started any new medi-
cations. She has not used any new detergent, soap or Current Status
lotion. Prior to current illness, she is an active child
who spends a great deal of time playing outdoors. Mother On arrival at the ED, she is pale and lethargic. Physical
reports that she removed a tick from the child’s lower back examination is significant for dry mucous membranes
5 days ago. and tachycardia, with a heart rate of 136 bpm. She has

Cases in Pediatric Acute Care: Strengthening Clinical Decision Making, First Edition. Edited by Andrea M. Kline-Tilford and Catherine M. Haut.
© 2020 John Wiley & Sons Ltd. Published 2020 by John Wiley & Sons Ltd.
Downloaded from https://onlinelibrary.wiley.com/doi/ by National Institute Of Standard, Wiley Online Library on [06/03/2024]. See the Terms and Conditions (https://onlinelibrary.wiley.com/terms-and-conditions) on Wiley Online Library for rules of use; OA articles are governed by the applicable Creative Commons License
446 Cases in Pediatric Acute Care

generalized abdominal pain that is tender on light palpa- Questions


tion. She is afebrile on presentation. Periorbital edema is
noted. A blanching erythematous rash with macules Answer the following questions using the details provided.
2 mm in size is noted on her feet, ankles, hands, wrists,
and forearms bilaterally (Figure 106.1). Blood cultures, uri- 1 Based on the data given, what are the differential diag-
nalysis, complete blood count, and complete metabolic noses and the most likely diagnosis for this child?
panel are obtained, with results as shown in Tables 106.1 2 Would you obtain any other diagnostic studies based on
and 106.2. the working differential diagnoses?
3 What treatment would be recommended at this time?
4 Would you obtain any additional diagnostic studies?
5 What are other potential complications of the illness
that may require an escalation in care?

Rationale and Evidence-based Practice


Explanation

Based on the data given, what are the differential


diagnoses and the most likely diagnosis for
this child?
Differential diagnoses for this child include Rocky Moun-
Figure 106.1 Rash. Source: www.cdc.gov.
tain spotted fever (RMSF), Lyme disease, ehrlichiosis and
granulocytic anaplasmosis, meningitis, meningococcemia,
Table 106.1 Basic metabolic profile. streptococcal infection, gastroenteritis, and acute abdo-
men. Other illnesses such as rubella, measles, and non-
Sodium 140 mEq/L
specific viral illness may be included in the differential
Potassium 3.5 mEq/L
diagnoses.
Chloride 100 mEq/L The most likely diagnosis for this child is RMSF due to
Carbon dioxide 24 mEq/L the child’s history and presenting symptoms of nausea,
Blood urea nitrogen 24 mg/dL vomiting, abdominal pain, headache, fever, and rash fol-
Creatinine 0.7 mg/dL lowing the fever, and known tick bite. Edema around the
Glucose 85 mg/dL eyes is common in early illness. Approximately half of
Alanine aminotransferase 65 U/L patients with RMSF infection do not recall tick exposure,
which can make clinical diagnosis difficult.
Aspartate aminotransferase 50 U/L
RMSF is an infectious tick-borne illness that is curable
but which can be fatal without proper treatment. It is
caused by the intracellular bacterium Rickettsia rickettsii.
Table 106.2 Complete blood count. RMSF is considered one of the most severe tick-borne rick-
ettsial infections, with pre-antibiotic case-fatality rates as
White blood cell count 4500/mm3
high as 80% in some case reports. The disease causes fever,
Hemoglobin 12 g/dL
headache, abdominal pain, and rash in the majority of
Hematocrit 36% patients. The hallmark of RMSF is a blanching erythema-
Platelet count 120 000/mm3 tous rash with macules (1–4 mm in size) that become pete-
Neutrophils 66% chial over time. Although the evolution of skin rash may
Eosinophils 2% vary among patients, it usually begins on the ankles and
Monocytes 2% wrists and spreads to the trunk. A rash that appears on
Lymphocytes 30%
the palms and soles is highly suggestive of RMSF, but usu-
ally occurs in later-stage disease.
Downloaded from https://onlinelibrary.wiley.com/doi/ by National Institute Of Standard, Wiley Online Library on [06/03/2024]. See the Terms and Conditions (https://onlinelibrary.wiley.com/terms-and-conditions) on Wiley Online Library for rules of use; OA articles are governed by the applicable Creative Commons License
Chapter 106 Child with Fever and Rash 447

Would you obtain any other diagnostic studies or less is 2.2 mg/kg twice daily (maximum daily dose 200
based on the working differential diagnoses? mg). Children who weigh over 45 kg should receive the
adult dose of 100 mg twice daily. Fever generally resolves
RMSF is difficult to diagnose because there is no rapid
within 24–48 hours after initiation of doxycycline. Doxycy-
point-of-care testing available during an acute infection.
cline resistance in RMSF has not been reported. Patients
Therefore, the clinical diagnosis must be confirmed retro-
should be treated with doxycycline for at least 3 days after
spectively by the use of serologic testing.
fever resolves and clinical improvement has been noted,
The standard serologic test for diagnosis of RMSF is the
and thus minimum duration of treatment is 5–7 days. Tet-
indirect immunofluorescence antibody (IFA) assay for IgG
racyclines can cause dental staining when administered to
using R. rickettsii antigen. IgM and IgG antibodies typically
children younger than 8 years. However, studies have
appear 7–10 days after onset of the illness, and the optimal
shown that the risk of dental staining with doxycycline is
time to obtain a convalescent antibody titer is at 14–21 days
minimal if a short course is administered.
after the onset of symptoms. A fourfold rise in IgG titers
Chloramphenicol is typically used in the rare setting
between the acute and convalescent stage is diagnostic of
when an individual has a history of severe adverse reaction
seroconversion and recent illness. Therefore, IgG IFA
to doxycycline (e.g. toxic epidermal necrolysis, severe hepa-
assays should be performed on paired acute and convales-
toxicity). The dose of chloramphenicol is 50 mg/kg daily in
cent serum samples collected 2–4 weeks apart. Because
four divided doses (maximum 4 g/day); dose reductions are
antibody titers are frequently negative in the first week of
required if used to treat neonates.
illness, RMSF cannot be confirmed using single acute anti-
body results. IgM IFA assays are available through some
reference laboratories, but results might be less specific Would you obtain any additional diagnostic
than IgG IFA assays for diagnosing a recent infection. studies?
Another diagnostic option in the evaluation of RMSF is A lumbar puncture may be indicated in the setting of severe
polymerase chain reaction (PCR) amplification performed headache in order to evaluate for meningococcal disease. If
on DNA extracted from peripheral blood and a swab of the a lumbar puncture is performed in a patient with RMSF,
skin rash. Rickettsia rickettsii infects the endothelial cells cerebrospinal fluid (CSF) results typically show a white
that line blood vessels and may not circulate in large num- blood count of less than 100 cells/mm3 (with lymphocytic
bers in the blood until the disease has progressed to a severe or monocytic predominance), moderately elevated protein
phase of infection. Although a positive PCR result is help- (100–200 mg/dL), and a normal glucose level. Abnormal
ful, a negative result does not exclude the diagnosis, and coagulation studies may be present in the setting of RMSF.
treatment should not be withheld due to a negative result. Coagulation studies including prothrombin time (PT) and
activated partial thromboplastin time (PTT) should be
What treatment would be recommended assessed.
at this time?
What are other potential complications of the
The patient is dehydrated based on the history, physical
illness that may require an escalation in care?
examination with tachycardia, and elevated serum creati-
nine and blood urea nitrogen demonstrated on laboratory Cardiopulmonary manifestations may include myocarditis,
findings. The dehydration is secondary to vomiting, congestive heart failure, dysrhythmia, and pneumonia, but
decreased oral intake, and insensible fluid losses from fever. these are uncommon in patients who are treated early.
A 20 mL/kg normal saline bolus is indicated with reevalua- Conjunctivitis is the ocular abnormality that is most com-
tion after the bolus is complete. Administration of an anti- monly present, but optic edema, retinal arterial occlusion,
emetic, such as ondansetron, is appropriate given the child retinal hemorrhage, and retinal vein engorgement can also
has experienced ongoing nausea and vomiting. occur. Neurologic complications can include lethargy,
The most important aspect to timely and effective treat- meningismus, cortical blindness, seizures, cranial nerve
ment of a rickettsial illness is clinical recognition. When palsies, confusion, photophobia, or psychosis. Hyponatre-
a rickettsial illness is suspected, rapid empiric treatment mia is a particularly common finding in patients with
with oral or intravenous doxycycline is the treatment of central nervous system involvement. Azotemia may be seen
choice for both adults and children. Delay in treatment and is usually due to hypovolemia. Critical illness is
can lead to severe disease and death or long-term complica- marked by disseminated intravascular coagulation and
tions. The dose of doxycycline for children weighing 45 kg acute respiratory distress syndrome.
Downloaded from https://onlinelibrary.wiley.com/doi/ by National Institute Of Standard, Wiley Online Library on [06/03/2024]. See the Terms and Conditions (https://onlinelibrary.wiley.com/terms-and-conditions) on Wiley Online Library for rules of use; OA articles are governed by the applicable Creative Commons License
448 Cases in Pediatric Acute Care

Children are at high risk of death from RMSF, and survi- to severe myocarditis, encephalitis, and multiorgan failure.
vors may have severe complications, including limb ampu- Early antibiotics are crucial in this population, and reduce
tation, deafness, and cognitive deficits. Pediatric patients the risk of morbidity and mortality.
must be monitored closely as they can quickly progress

Further Reading
Blanton, L. (2019). The rickettsioses. Infectious Disease Clinics Gottlieb, M., Long, B., and Koyfman, A. (2018). The evaluation
of North America 33(1): 213–229. and management of Rocky Mountain spotted fever in the
Centers for Disease Control and Prevention. Rocky Mountain emergency department: a review of the literature. Journal of
spotted fever. https://www.cdc.gov/rmsf/ (accessed Emergency Medicine 55(1): 42–50.
29 March 2019).
Downloaded from https://onlinelibrary.wiley.com/doi/ by National Institute Of Standard, Wiley Online Library on [06/03/2024]. See the Terms and Conditions (https://onlinelibrary.wiley.com/terms-and-conditions) on Wiley Online Library for rules of use; OA articles are governed by the applicable Creative Commons License
449

107

A Teen With Vomiting and Diarrhea


Christyne Kyper
Alaska Native Tribal Health Consortium, Anchorage, AK, USA

A 15-year-old female is admitted to the pediatric floor with Past Surgical History
abdominal pain, vomiting, diarrhea, and an elevated
lipase level. The patient had a tonsillectomy and adenoidectomy at
age 7.
Family history includes both parents are healthy. Pater-
nal grandmother has a history of depression and anxiety.
History of Present Illness Paternal grandfather is deceased. Maternal grandparents
are alive and well.
This 15-year-old female presents with a 3-day history of
abdominal pain and vomiting with occasional loose stools.
She is nauseated any time she tries to eat or drink. Her Social History
abdominal pain is diffuse, and she points to her entire abdo-
men when asked where it hurts, but with persistent ques- Parents are separated and live in different cities. Patient
tioning will say it is mostly the upper area. Her pain is reports having multiple altercations with mother and
exacerbated by eating, lying flat, or walking. She describes spends most of her time living with either the father or
emesis as “orange liquid with chunks in it.” She reports paternal grandmother.
drinking approximately two cups of hand sanitizer per
day (about 226 g), which she gets during gym class when
no one is watching. She is currently in a residential treat-
ment program for depression and suicidal ideation.
Current Status

On presentation to the emergency department (ED), her


vital signs are as follows: temperature 38.4 C (101.2 F),
heart rate 112 bpm, respiratory rate 20 breaths per minute,
Past Medical History blood pressure 151/87 mmHg, and oxygen saturation 95%.
Her weight is 132 kg.
This teen’s medical history includes type 2 diabetes, depres- A basic metabolic panel, complete blood count, pan-
sion, suicide attempts, self-mutilation, obesity, and creatic enzymes, lipid panel, and urinalysis are obtained
bulimia. She admits to using marijuana and alcohol since and the results are shown in Tables 107.1–107.5. Hemo-
12 years of age. globin A1c level is 5.2 mg/dL. Urine and blood cultures
Her current medications include aripiprazole, Miralax, are obtained. Pregnancy and influenza screens are
fluoxetine, quetiapine, escitalopram, doxycycline, olanza- negative.
pine, metformin, Colace, famotidine, potassium supple- Imaging results are as follows.
ments, omega 3, vitamin B complex, vitamin C, ferrous
sulfate, and diphenhydramine. She is allergic to shellfish
and reports a weight gain of 4.5 kg in the past week.
• Abdominal ultrasound: enlarged fatty liver, large amount
of non-mobile sludge within the gallbladder lumen, mild

Cases in Pediatric Acute Care: Strengthening Clinical Decision Making, First Edition. Edited by Andrea M. Kline-Tilford and Catherine M. Haut.
© 2020 John Wiley & Sons Ltd. Published 2020 by John Wiley & Sons Ltd.
Downloaded from https://onlinelibrary.wiley.com/doi/ by National Institute Of Standard, Wiley Online Library on [06/03/2024]. See the Terms and Conditions (https://onlinelibrary.wiley.com/terms-and-conditions) on Wiley Online Library for rules of use; OA articles are governed by the applicable Creative Commons License
450 Cases in Pediatric Acute Care

Table 107.1 Basic metabolic panel. gallbladder wall thickening. Positive sonographic Mur-
phy sign. Visualized portions of pancreas within normal
Sodium 143 mEq/L
limits.
Potassium
Chloride
3.8 mEq/L
105 mEq/L • Abdominal computed tomography (CT): pancreas is dif-
fusely edematous with surrounding peripancreatic fat
Carbon dioxide 30 mEq/L stranding and peripancreatic fluid. No organized pseudo-
Blood urea nitrogen 5 mg/dL cyst. Mild abdominal ascites, no calcifications in gallblad-
der or biliary tree. Small pleural effusions noted bilaterally.


Creatinine 0.6 mg/dL
Glucose 116 mg/dL
Magnetic resonance cholangiopancreatography (MRCP):
findings same as CT.
Calcium 8.5 mg/dL
The patient had another elevation in her lipase 5 days
later when resuming a regular diet, which gradually came
Table 107.2 Complete blood count. down over the next 5 days.
White blood cell count 16 500/mm3
Hemoglobin 11 g/dL
Questions
Hematocrit 35.9%
Platelet count 159 000/mm3 Answer the following questions using the details provided.
Neutrophils 38%
1 What would be included in the differential diagnosis?
Lymphocytes 45%
2 How is the diagnosis of pancreatitis made?
Monocytes 1.3%
3 What is the initial management of acute pancreatitis?
Eosinophils 4% 4 When can you begin to feed the patient?
5 Is any other management needed for this patient?

Table 107.3 Pancreatic enzymes.


Rationale and Evidence-based Practice
Amylase 191 U/L Explanation
Lipase 236 U/L
What would be included in the differential
diagnosis?

Table 107.4 Lipid panel. There are many differential diagnoses for abdominal pain,
including gastroenteritis, cholecystitis, pancreatitis, intra-
Cholesterol 83 mg/dL abdominal infection, gastritis, pelvic inflammatory disease
Lipid 53 mg/dL (PID), ovarian torsion, appendicitis, and constipation. Each
High-density lipoprotein 34 mg/dL of these diagnoses presents with similar symptoms, but
Low-density lipoprotein 35 mg/dL physical findings and diagnostic testing results will be dif-
ferent. For this patient, pancreatitis is a consideration due
to her elevated lipase. It may be associated with her medi-
Table 107.5 Urinalysis. cations or from drinking hand sanitizer and alcohol.
Because her pain is primarily in the upper region, PID,
Color Clear, yellow ovarian torsion, and appendicitis are less likely.
pH 6.5
Specific gravity 1.009 How is the diagnosis of pancreatitis made?
White blood cells 0 The diagnosis of pancreatitis is defined by two of the follow-
Nitrite Negative ing: acute onset of persistent severe epigastric pain often
Protein Negative radiating to the back, elevation in serum lipase or amylase
Glucose Negative to three times or greater the upper limit of normal, or char-
Bilirubin Negative acteristic findings of acute pancreatitis on imaging with
contrast CT, magnetic resonance imaging (MRI) or
Downloaded from https://onlinelibrary.wiley.com/doi/ by National Institute Of Standard, Wiley Online Library on [06/03/2024]. See the Terms and Conditions (https://onlinelibrary.wiley.com/terms-and-conditions) on Wiley Online Library for rules of use; OA articles are governed by the applicable Creative Commons License
Chapter 107 A Teen With Vomiting and Diarrhea 451

transabdominal ultrasound. The diagnosis can be made Patients with acute pancreatitis should be monitored
without imaging if the first two criteria are met. closely for the first 24–48 hours. Vital signs including oxy-
gen saturations, urine output, electrolytes, and serum glu-
What is the initial management of acute cose should all be assessed frequently. Hyperglycemia
pancreatitis? should be treated because it can increase the risk of second-
ary pancreatic infections.
Initial management of acute pancreatitis consists of fluid
resuscitation, pain control, and monitoring. Fluid status
must be assessed first. If patients present with hypotension When can you begin to feed the patient?
and tachycardia, fluid boluses of 20 mL/kg of isotonic fluid
should be administered until stable. This patient received Oral feedings can be initiated within 24 hours if there is no
40 mL/kg of normal saline soon after initial evaluation in ileus, nausea, or vomiting, the pain is decreasing, and
the ED. Therapy should be goal-directed and reassessed inflammatory markers are improving. The suggested diet
at frequent intervals. Adequate fluid replacement should is low residue, low fat, and soft. If the patient cannot toler-
be based on clinical assessment of vital signs, urine output, ate oral feedings within 5 days, it is recommended that
and reduction in hematocrit and blood urea nitrogen enteral feeds with high-protein, low-fat, semi-elemental
(BUN) over the first 24 hours. Persistent hemoconcentra- formula be started.
tion at 24 hours has been associated with development of
necrotizing pancreatitis.
Is any other management needed for this patient?
Pain control is achieved with opioids. Intravenous hydro-
morphone or fentanyl are typically used, with fentanyl This patient has an extensive psychiatric history. Because
becoming more widely used due to its better safety profile, she has previous attempts at self-harm she should have sui-
especially related to renal impairment. Morphine can cause cide precautions. She has admitted to drinking hand sani-
an increase in sphincter of Oddi pressure, but there are no tizer so this needs to be carefully monitored while she is in
clinical studies to suggest that morphine can aggravate or the hospital. Consults to behavioral health, nutrition, and
cause pancreatitis. endocrinology would also be warranted.

Further Reading
Tenner, S., Baillie, J., DeWitt, J., and Vege, S.S. (2013). Vege, S.S., DiMagno, M.J., and Forsmark, C.E. (2018). Initial
American College of Gastroenterology Guideline: medical treatment of acute pancreatitis: American
management of acute pancreatitis. American Journal of Gastroenterological Association Institute Technical Review.
Gastroenterology 108(9): 1400–1415. doi: 10.1038/ Gastroenterology 154(4): 1103–1139.
ajg.2013.218.
Downloaded from https://onlinelibrary.wiley.com/doi/ by National Institute Of Standard, Wiley Online Library on [06/03/2024]. See the Terms and Conditions (https://onlinelibrary.wiley.com/terms-and-conditions) on Wiley Online Library for rules of use; OA articles are governed by the applicable Creative Commons License
453

108

Acute Jaundice in a 7-Year-Old Child


Roseann Hausmann
Seattle Children’s Hospital, Seattle, WA, USA

A 7-year-old female presents to the emergency department infection, but did not see a medical provider or require
(ED) from her primary care provider’s office with a treatment.
complaint of fever and abdominal pain.

Current Status
History of Present Illness
The patient is evaluated in the ED and initially noted to be
Parents report that 2 weeks ago the patient had a fever to sleepy but rousable. She is able to answer questions with
38.8 C (101.8 F), cough, vomiting, and diarrhea which prompting from her mother. Vital signs are stable: temper-
resolved without treatment. Today she began to have ature 38.9 C (102 F), heart rate 120 bpm, respiratory rate
dark-colored urine and the primary care provider noted 18 breaths per minute, and blood pressure 100/65 mmHg.
jaundice of her skin and sclera, and a 5-kg weight gain com- She complains of itching in her arms and legs and right
pared to her last appointment a month ago. She was then upper quadrant abdominal pain. She is noted to have scle-
referred to the ED for further evaluation. ral icterus. Laboratory studies are obtained and the results
are shown in Tables 108.1–108.3. The ammonia level is
found to be 86 μg/dL.
Past Medical History

The child was born at term by spontaneous vaginal delivery Questions


with no medical complications and discharged home with
her mother. She has received all immunizations to date, but Answer the following questions using the details provided.
did not get influenza vaccine this year. She has never been
1 Based on the data given, what are the differential diag-
hospitalized and does not have any chronic illness.
noses for this child?
2 What additional testing would be warranted?

Past Surgical History

The patient had pressure equalizer tubes placed at 2 years of Rationale and Evidence-based Practice
age for recurrent otitis media. Explanation

Based on the data given, what are the differential


diagnoses for this child?
Family History
This patient presents with signs of acute liver failure.
Parents are both alive and healthy. Her 14-year-old brother Elevation of the liver function enzymes alanine amino-
was recently sick with a an acute upper respiratory transferase (ALT) and aspartate aminotransferase (AST)

Cases in Pediatric Acute Care: Strengthening Clinical Decision Making, First Edition. Edited by Andrea M. Kline-Tilford and Catherine M. Haut.
© 2020 John Wiley & Sons Ltd. Published 2020 by John Wiley & Sons Ltd.
Downloaded from https://onlinelibrary.wiley.com/doi/ by National Institute Of Standard, Wiley Online Library on [06/03/2024]. See the Terms and Conditions (https://onlinelibrary.wiley.com/terms-and-conditions) on Wiley Online Library for rules of use; OA articles are governed by the applicable Creative Commons License
454 Cases in Pediatric Acute Care

Table 108.1 Complete blood count. which acetaminophen was given during her acute illness.
It is also important to question the patient and parents
White blood cell count 26 000/mm3
about other potential drugs which could cause liver failure,
Hemoglobin 10 g/dL like some antibiotics, antifungals and ibuprofen, among
Platelet count 85 000/mm3 others.
Neutrophils 8% Immune system dysfunction is another category to con-
Lymphocytes 22% sider within the differential diagnosis. The two forms that
Monocytes 1% are often seen in children are autoimmune hepatitis and
Bands 60%
hemophagocytic lymphohistiocytosis (HLH). While auto-
immune hepatitis presents equally across all ages of pediat-
ric patients, HLH is most typically seen in children under
5 years of age. HLH has been reported in older children
Table 108.2 Complete metabolic profile and liver function test
results. and in adolescence, so while less likely, should be
considered.
Sodium 146 mEq/L Another cause of acute liver failure can be hypoperfusion
Potassium 3.5 mEq/L due to cardiac dysfunction, dehydration, or decreased blood
Chloride 100 mEq/L flow due to sepsis or hepatic artery clot. As the patient has
Carbon dioxide 24 mEq/L
been without symptoms other than fatigue in recent days,
and she has been eating and drinking normally, this cause
Blood urea nitrogen 26 mg/dL
may not be high on the differential list.
Creatinine 1.4 mg/dL
Metabolic conditions or processes are the final concern
Glucose 90 mg/dL for cause of liver failure, which in the school-aged child
Albumin 2.9 g/dL would be Wilson disease or mitochondrial diseases
Alanine aminotransferase 980 U/L (Squires et al., 2019). Screening for Wilson disease should
Aspartate aminotransferase 765 U/L include serum ceruloplasmin, urine copper, Coombs test,
Bilirubin 6.1 mg/dL and ophthalmology examination for Kayser–Fleischer
rings (Squires et al., 2019).

Table 108.3 Coagulation studies.


What additional testing would be warranted?
Prothrombin time (PT) 18 s
In order to consider the list of differential diagnoses and to
Partial thromboplastin time (PTT) 40 s finalize a working diagnosis, additional laboratory studies
INR 2.5 are indicated. An infectious evaluation should be per-
Fibrinogen 250 mg/dL formed to determine a potential viral cause of acute liver
failure. Depending on facility availability of testing, the
tests can include IgG and IgM to determine current and
indicates inflammation of the liver, while the elevated past infectious exposures, and quantitative polymerase
international normalized ratio (INR), bilirubin and ammo- chain reaction (PCR) to determine exact viral loads.
nia, and clinical findings of encephalopathy indicate that A respiratory viral PCR should also be obtained for a broad
she has progressed to liver failure. The differential diagno- screening of viruses prevalent in the local area. It is impor-
sis for cause of pediatric acute liver failure is broad and will tant to note that many of these viruses have no treatment,
require a detailed evaluation, but some of the potential dif- so the information provides prognostic indication but may
ferentials would include drug toxicity, infection, immune not change the course of treatment.
system dysfunction, hypoperfusion, and metabolic disease. Autoimmune hepatitis may be suggested by positive
Given the history of viral symptoms, an infectious disease immune markers, such as anti-nuclear antibody (ANA)
evaluation should be performed to establish this as a poten- and anti-smooth muscle antibody (SMA), so obtaining this
tial cause of liver failure. Some infectious causes that are blood work is important. The treatment for autoimmune
known to result in acute liver failure in pediatric patients hepatitis will include steroids, so it is important to confirm
include hepatitis A, B and C, adenovirus, parvovirus, the absence of an infectious agent, since an infection will be
Epstein–Barr virus, cytomegalovirus, and enterovirus. Ace- made worse by the addition of steroids. HLH can be diag-
taminophen overdose is another diagnosis to consider, and nosed with cytopenia in two or more cell lines, and by an
it is important to assess the amount and frequency with elevated ferritin level. To exclude hypoperfusion injury,
Downloaded from https://onlinelibrary.wiley.com/doi/ by National Institute Of Standard, Wiley Online Library on [06/03/2024]. See the Terms and Conditions (https://onlinelibrary.wiley.com/terms-and-conditions) on Wiley Online Library for rules of use; OA articles are governed by the applicable Creative Commons License
Chapter 108 Acute Jaundice in a 7-Year-Old Child 455

an abdominal ultrasound with Doppler can be performed to Additional Questions and Evidence-based
monitor blood flow into the liver. Rationale
Continuation of case: The patient is admitted to the pedi-
atric intensive care unit (PICU) for acute liver failure of 3 Given the diagnostic evaluation, what is the most likely
unknown etiology. Over the next 36 hours, she becomes diagnosis for this patient?
unresponsive and requires intubation and is maintained 4 What consults would be recommended at this time?
on a mechanical ventilator. She is given vitamin K with
no change to her INR. On abdominal ultrasound she is
noted to have a large-volume ascites. An ophthalmology Given the diagnostic evaluation, what is the most
examination is completed which shows Kayser–Fleischer likely diagnosis for this patient?
rings. Additional diagnostic laboratory results based on Based on laboratory findings and the ophthalmology exam-
the differential diagnoses are shown in Table 108.4. Liver ination, the most likely diagnosis is Wilson disease. The
function tests have also been trended every 8 hours since American Association for the Study of Liver Diseases
admission and the results are shown in Table 108.5. criteria for the diagnosis of Wilson disease include a low
serum ceruloplasmin (<20 mg/dL), the presence of
Kayser–Fleischer rings on eye examination, and a 24-hour
urine copper level of greater than 40 μg/day.
Table 108.4 Additional diagnostic laboratory results based on Wilson disease is an autosomal recessive disorder that
differential diagnoses. can be associated with liver disease, degenerative changes
in the brain, and the presence of Kayser–Fleischer rings in
Viral respiratory panel Negative
the cornea. The pathologic mechanism is a defect in
Coombs test Negative ATP7B, a copper-transporting ATPase mainly expressed
Hepatitis A Negative in hepatocytes (Schilsky, 2017). When the liver’s capacity
Hepatitis B Negative for copper storage is exceeded or liver cells are damaged,
Hepatitis C Negative there is resulting accumulation of copper in the liver
Epstein–Barr virus PCR Not detected which then spills over into the circulation, and potentially
Cytomegalovirus PCR Not detected into other organs such as the brain, cornea and kidneys.
Wilson disease can present as acute liver failure, progres-
Parvovirus PCR Not detected
sive neurologic disorder, or psychological illness. Most
Adenovirus PCR Not detected
patients with liver disease present in childhood, within
Herpes simplex virus Negative the first 20 years of life, and about 5% of these patients
Ferritin 100 ng/mL develop acute liver failure. If there is a family history of
Anti-nuclear antibody Negative psychological disorders, neurologic disorders that occurred
Anti-smooth muscle antibody Negative in the latter part of life, consideration for Wilson disease
Urine copper 200 μg per 24 hours should be made. There is also a scoring system that
Ceruloplasmin 7 mg/dL includes different laboratory tests, including ceruloplas-
min, urinary copper and chromosome analyses, which
Acetaminophen Not detected
can be used for diagnosis.

Table 108.5 Laboratory trends.

AST (U/L) ALT (U/L) Bilirubin (mg/dL) Albumin (g/dL) Ammonia (μg/L) INR

Monday 10:00 (ED) 895 765 8 2 86 2.6


Monday 18:00 1900 1500 8.5 2.2 90 2.6
Tuesday 02:00 3800 3000 9 2 98 (started 1 (given factor VII
lactulose) for procedure)
Tuesday 10:00 6400 5100 11 1.8 85 1.6
Tuesday 18:00 5800 4800 11.5 1.8 95 2
Wednesday 02:00 3800 3000 14 1.6 100 2.5
Downloaded from https://onlinelibrary.wiley.com/doi/ by National Institute Of Standard, Wiley Online Library on [06/03/2024]. See the Terms and Conditions (https://onlinelibrary.wiley.com/terms-and-conditions) on Wiley Online Library for rules of use; OA articles are governed by the applicable Creative Commons License
456 Cases in Pediatric Acute Care

If identified early, Wilson disease can be managed with a with a rising bilirubin, ammonia, and INR. These results
low-copper diet and chelating agents, such as D-penicilla- are concerning, indicating that the liver does not have suf-
mine, trientine or zinc salts, which increase the urinary ficient viable tissue to continue producing the enzymes,
excretion of copper. Foods that are high in copper include nor is it able to filter toxins. Prognosis in Wilson disease
mushrooms, chocolate, liver, and nuts. Patients should also with liver failure and hemolysis is poor without liver trans-
be monitored with laboratory testing, including complete plant. Acute decompensated Wilson disease qualifies a
blood count, liver panel, serum copper and ceruloplasmin, patient to be listed as a Status 1A on the United Network
and 24-hour urine collection for copper and zinc. for Organ Sharing (UNOS) waiting list for deceased donor
transplants. Liver transplant is curative for Wilson disease,
What consults would be recommended at but transplant in itself is a risk and requires lifelong
this time? follow-up.
A consult with the genetics team would also be indicated.
The most important and urgent consult is with a pediatric
All siblings and first-degree relatives of the patient should
liver transplant team. When a patient experiences decom-
be screened for Wilson disease. If the disease is identified,
pensated Wilson disease causing fulminant liver failure,
treatment with chelating agents can be begun to slow the
transplant is often the only treatment option. Current lab-
progression and hopefully prevent liver failure.
oratory studies in this case demonstrate a falling AST/ALT

References
Schilsky, M.L. (2017). Wilson disease: diagnosis, treatment and meeting report from the Starzl Network for Excellence in
follow-up. Liver: the Clinics 21: 755–767. doi: 10.1016/j. Pediatric Transplantation. Pediatric Transplantation 23(6):
cld.2017.06.011. e13528. doi: 10.1111/petr.13528.
Squires, J.E., Logan, B., Lorts, A., et al. (2019). A learning
health network for pediatric liver transplantation: inaugural

Further Reading
Bandmann, O., Weiss, K.H., and Kaler, S.G. (2015). Wilson’s Neurology 14(1): 102–113. doi: 10.1016/S1474-4422(14)
disease and other neurological copper disorders. Lancet 70190-5.
Downloaded from https://onlinelibrary.wiley.com/doi/ by National Institute Of Standard, Wiley Online Library on [06/03/2024]. See the Terms and Conditions (https://onlinelibrary.wiley.com/terms-and-conditions) on Wiley Online Library for rules of use; OA articles are governed by the applicable Creative Commons License
457

109

Idiopathic Nephrotic Syndrome


Tamara Hill
University of Maryland Medical Center, Baltimore, MD, USA

A 3-year-old male is brought to the emergency department Family History


(ED) with complaints of weight gain/puffiness, abdominal
pain, and rapid breathing and is admitted to the pediatric The patient’s father and paternal grandparents have a his-
intensive care unit. tory of hypertension. Mother is healthy. There are no
siblings.
History of Present Illness

This 3-year-old child became ill 3 weeks ago when his


mother noticed swelling around his eyes when he woke Current Status
up in the morning. After several days of these symptoms,
he was seen by his pediatrician who thought he might have On arrival to the ED, the patient has mild tachypnea with
seasonal allergies and prescribed loratadine. Two weeks respiratory rate of 28 breaths per minute. His lung sounds
ago, he developed three to four loose watery stools per are clear. His blood pressure is elevated (122/82 mmHg)
day (non-bilious, non-bloody) and has complained inter- and his weight is up by 1.8 kg from his baseline weight.
mittently of mild abdominal pain. He was evaluated again He has normal S1 and S2 heart sounds with heart rate of
by another provider in the pediatric office and was diag- 74 bpm. He has facial, periorbital and scrotal edema with
nosed with gastroenteritis. Over the past week his mother 1–2+ pitting edema up to the knees and ankles; no rashes
noticed that, despite his poor appetite, the waistband of or ecchymosis noted. His abdomen is rounded and full with
his pants are getting tighter and his face and legs seem ascites 2+ noted. He has no organomegaly but he has
puffy. He is normally a very energetic and happy child abdominal pain 8 on Wong–Baker Faces Scale.
but for a week now he has been fussy and tired. A complete metabolic panel, complete blood count and
urinalysis are obtained, with results as shown in
Tables 109.1–109.3.
Past Medical History

This child was born by spontaneous vaginal delivery at


term and went home with his mother. No complications. Questions
He is current on all immunizations. He has a history of fre-
quent episodes of otitis media. His weight is 99% for gender Answer the following questions using the details provided.
and he has a current body mass index (BMI) of 28.5.
1 Based on the data given, what are the differential diag-
noses and the most likely diagnosis for this child?
Past Surgical History 2 What is the best initial management for this condition?
3 What other therapies would be recommended for fre-
History of uncomplicated circumcision, but no other surgi- quent relapses of this condition?
cal procedures. 4 Would you obtain any other diagnostic studies?

Cases in Pediatric Acute Care: Strengthening Clinical Decision Making, First Edition. Edited by Andrea M. Kline-Tilford and Catherine M. Haut.
© 2020 John Wiley & Sons Ltd. Published 2020 by John Wiley & Sons Ltd.
Downloaded from https://onlinelibrary.wiley.com/doi/ by National Institute Of Standard, Wiley Online Library on [06/03/2024]. See the Terms and Conditions (https://onlinelibrary.wiley.com/terms-and-conditions) on Wiley Online Library for rules of use; OA articles are governed by the applicable Creative Commons License
458 Cases in Pediatric Acute Care

Table 109.1 Complete metabolic panel. Rationale and Evidence-based Practice


Sodium 132 mEq/L
Explanation
Potassium 4.9 mEq/L
Based on the data given, what are the differential
Chloride 105 mEq/L
diagnoses and the most likely diagnosis for
Carbon dioxide 26 mEq/L this child?
Blood urea nitrogen 14 mg/dL
This child is presenting with idiopathic nephrotic syn-
Creatinine 0.36 mg/dL
drome (NS) based on laboratory results and presentation.
Glucose 71 mg/dL
Nephrotic syndrome is characterized by heavy proteinuria,
Calcium 9.8 mg/dL hypoalbuminemia (serum albumin <2.5 g/dL), hyperlipi-
Albumin 2.0 mg/dL demia (serum cholesterol >200 mg/dL), and edema.
Total protein 4.8 g/dL Nephrotic range proteinuria is present if early morning
Aspartate aminotransferase 37 U/L urine protein is 3+/4+ (on dipstick), spot protein/creati-
Alanine aminotransferase 20 U/L nine ratio is above 2 mg/mg, or urine albumin excretion
Alkaline phosphatase 252 U/L
exceeds 40 mg/m2 per hour (on a timed sample). Precise
quantitative assessment of proteinuria, including 24-hour
Total bilirubin 0.6 mg/dL
urine protein measurement, is seldom necessary.
Minimal change nephrotic syndrome (MCNS), focal seg-
mental glomerulosclerosis (FSGS), membranoproliferative
Table 109.2 Complete blood count. glomerulonephritis (MPGN), and IgA nephropathy are
detected via renal biopsy. MCNS is the most common histo-
White blood cell count 8000/mm3
logic form of NS, with males affected more frequently than
Hemoglobin 13.1 g/dL
females. FSGS may present like MCNS or with less profound
Hematocrit 38% proteinuria. MPGN is characterized by hypocomplementemia
Platelet count 275 000/mm3 and glomerular renal disease. IgA nephropathy is commonly
Neutrophils 64% seen in adolescents and children with systemic infections.
Lymphocytes 4%
Monocytes 3.4%
What is the best initial management
Eosinophils 3.4%
for this condition?
The standard medication for treatment of NS is predniso-
lone or prednisone. The initial episode of NS should be trea-
Table 109.3 Urinalysis. ted at a dose of 2 mg/kg daily (maximum 60 mg in single or
divided doses) for 6 weeks, followed by 1.5 mg/kg (maxi-
Color Dark yellow mum 40 mg) as a single morning dose on alternate days
pH 6.0 for the next 6 weeks. The duration of initial prednisolone
Specific gravity 1.008 therapy should be for a minimum of 12 weeks. The benefits
Protein 3+ of sustained remission and reduction in relapse rates are
Nitrites Negative superior if alternate-day treatment is not stopped abruptly
Urobilirubin 1+ at 12 weeks, but tapered over the next 2–4 months.
Blood Small
Leukocytes Negative What other therapies would be recommended
Ketones Negative for frequent relapses of this condition?
Glucose Negative
Treatment for infection is recommended before initiating
White blood cells 11–25 per high power field steroid therapy. Prednisolone is administered at a dose of
Red blood cells 0–2 per high power field 2 mg/kg daily (single or divided doses) until urine protein
Hyaline casts None is trace or nil for three consecutive days. Subsequently, pre-
Bacteria Occasional dnisolone is given in a single morning dose of 1.5 mg/kg on
Crystals None alternate days for 4 weeks and then discontinued. The usual
duration of treatment for a relapse is thus 5–6 weeks.
Downloaded from https://onlinelibrary.wiley.com/doi/ by National Institute Of Standard, Wiley Online Library on [06/03/2024]. See the Terms and Conditions (https://onlinelibrary.wiley.com/terms-and-conditions) on Wiley Online Library for rules of use; OA articles are governed by the applicable Creative Commons License
Chapter 109 Idiopathic Nephrotic Syndrome 459

In case the patient is not in remission despite 2 weeks of hematuria. Appropriate tests are performed, if necessary,
treatment with daily prednisolone, the treatment is for associated conditions (e.g. chest X-ray and tuberculin
extended for a further 2 weeks. If the patient is steroid test, hepatitis B surface antigen, and anti-nuclear antibo-
sensitive, he will respond to steroids. If there is steroid dies). Urine culture is not necessary unless the patient
resistance, other immunosuppressive therapy (e.g. myco- has clinical features suggestive of a urinary tract
phenolate mofetil or tacrolimus) will need to be considered. infection.
Therapy also includes nutrition education: a low-sodium Children with idiopathic NS who do not have hematuria,
and low-protein diet, and fluid restriction. There should be hypertension, or impaired renal function are treated with
close monitoring for hypertension and recording of weight. corticosteroids without requiring a kidney biopsy.
A biopsy is usually not necessary in patients with frequent
Would you obtain any other diagnostic studies? relapses or steroid dependence before starting treatment
with levamisole, cyclophosphamide, or mycophenolate
Diagnostic studies during the initial episode should
mofetil, but should be performed before therapy with calci-
include urinalysis, complete blood count, and blood
neurin inhibitors. A biopsy is required to identify the
levels of albumin, cholesterol, urea, and creatinine. Esti-
underlying renal disease in certain cases (MCNS, FSGS,
mation of blood levels of antistreptolysin O and C3 is
MPGN and IgA nephropathy).
required in patients with gross or persistent microscopic

Further Reading
Marcdante, K.J. and Kliegman, R. (ed.) (2019). Nelson 2014. Clinical and Experimental Nephrology 20(3): 342–370.
Essentials of Pediatrics, 8th edn. Philadelphia, PA: Elsevier. doi: 10.1007/s10157-015-1216-x.
Nishi, S., Ubara, Y., Utsunomiya, Y., et al. (2016). Evidence-
based clinical practice guidelines for nephrotic syndrome
Downloaded from https://onlinelibrary.wiley.com/doi/ by National Institute Of Standard, Wiley Online Library on [06/03/2024]. See the Terms and Conditions (https://onlinelibrary.wiley.com/terms-and-conditions) on Wiley Online Library for rules of use; OA articles are governed by the applicable Creative Commons License
461

110

School-age Child with Hives and Lip Swelling


Jay M. Hunter
UCSF Benioff Children’s Hospital and UCSF School of Nursing, San Francisco, CA, USA

A 10-year-old male presents to the emergency department Current Status


(ED) with hives and lip swelling.
On arrival to the ED, the patient has facial angioedema,
scattered hives, mild respiratory distress, and appears
History of Present Illness scared. His respiratory rate is 38 breaths per minute with
moderate subcostal retractions. He is wheezing and his oxy-
The boy was outside playing with his friends this afternoon
gen saturations are 92% on 100% oxygen via a non-
when he was stung by a bee. The patient’s mother reports
rebreather mask. His blood pressure is 78/42 mmHg.
that he was crying, screaming, and tearful when she noticed
that his lips were swollen. The patient began to have diffi-
culty breathing, so she called emergency medical services
(EMS). On arrival to the scene, the patient had marked Questions
facial and lip edema and was in significant respiratory dis-
tress, so EMS administered intramuscular epinephrine to Answer the following questions using the details provided.
the patient and transported him to the ED. 1 Based on the currently available data, what are the dif-
ferential diagnoses and the most likely diagnosis for
this child?
Past Medical and Surgical History 2 What is the best way to treat this patient’s current signs
and symptoms on arrival at the ED?
The patient was born full term via vaginal delivery to a
3 How would you determine if the treatment was
healthy mother without complications. The patient has
effective?
completed all vaccinations to date, including an influenza
4 What other treatments would you recommend at
vaccine earlier this year. He has no known medical pro-
this time?
blems. The patient has never had surgery, nor has been
5 What is the patient’s disposition?
admitted to the hospital. He does not take any medications
and has no known allergies to medications or foods.

Rationale and Evidence-based Practice


Family and Social History
Explanation
The patient’s parents are married. His mother is 34 years old,
Based on the currently available data, what are the
healthy, does not report any medical problems, and works as
differential diagnoses and the most likely
a lawyer. His father is 36 years old, reports a history of hyper-
diagnosis for this child?
tension, no other medical problems, and works as a regis-
tered nurse. The patient has one sister who is 12 years old, Differential diagnoses for this child would include
healthy, and does not have any medical problems. The anaphylaxis, acute asthma exacerbation, severe croup,
patient is in the 4th grade and he does well in school. urticaria, foreign body aspiration, isolated angioedema,

Cases in Pediatric Acute Care: Strengthening Clinical Decision Making, First Edition. Edited by Andrea M. Kline-Tilford and Catherine M. Haut.
© 2020 John Wiley & Sons Ltd. Published 2020 by John Wiley & Sons Ltd.
Downloaded from https://onlinelibrary.wiley.com/doi/ by National Institute Of Standard, Wiley Online Library on [06/03/2024]. See the Terms and Conditions (https://onlinelibrary.wiley.com/terms-and-conditions) on Wiley Online Library for rules of use; OA articles are governed by the applicable Creative Commons License
462 Cases in Pediatric Acute Care

food poisoning, vasovagal episode, idiopathic capillary leak until signs and symptoms are resolved are important in
syndrome, and other causes of shock. Anaphylaxis is the determining if other signs of anaphylaxis and shock are
most likely diagnosis due to the presentation of facial present.
angioedema, urticaria, and respiratory compromise follow-
ing an insect sting. Anaphylaxis is commonly misdiagnosed
and underdiagnosed when cutaneous signs are not present, What other treatments would you recommend
so it is important to recognize the other signs and symptoms at this time?
of anaphylaxis, including, but not limited to, dizziness,
Additional systemic symptoms should be treated with sec-
hypotension, cough, stridor, dyspnea, vomiting, diarrhea,
ond-line agents following epinephrine administration, as
and wheezing.
appropriate, with particular consideration of biphasic ana-
phylactic reactions. Histamine H1 and H2 receptor antago-
What is the best way to treat this patient’s current
nists, such as diphenhydramine and ranitidine, may be
signs and symptoms on arrival at the ED?
administered for cutaneous and gastrointestinal symptoms,
Epinephrine continues to be the mainstay of treatment for and inhaled beta-agonists, such as albuterol, may be admi-
anaphylaxis, with the greatest benefit on administration nistered to treat reactive airway. Intravenous fluid replace-
prior to ED presentation. For pediatric patients, epineph- ment and oxygen therapies should be provided for patients
rine should be administered intramuscularly at a dose of with hypoxia and hypotension.
0.01 mg/kg of 1:1000 concentration in the vastus lateralis
muscle, with a maximum dose of 0.3 mg but up to
0.5 mg in prepubertal and teenage children. All other med- What is the patient’s disposition?
ications, including antihistamines, inhaled beta-agonists
For those patients with concern for protracted or biphasic
and glucocorticoids, are adjuvant medications that should
reactions, they should be monitored in the ED for approx-
be administered only in addition to, but not to replace,
imately 3–4 hours after administration of epinephrine to
epinephrine.
assess for recurrence of symptoms. If symptoms recur, then
medications should be re-dosed for a 24-hour period and
How would you determine if the treatment
hospitalization is warranted.
was effective?
Consideration should be made about a referral to an
Close monitoring for improvement of presenting signs and allergy and immunology specialist for treatment with
symptoms are paramount in determining the effectiveness venom immunotherapy for insect sting-induced anaphy-
of treatment. In addition, close and continuous monitoring laxis. This treatment can reduce the risk of subsequent ana-
of vital signs, including blood pressure, heart rate, respira- phylaxis to less than 5% if a full course of therapy is
tory rate, and oxygen saturations, during treatment and completed.

Further Reading
Farbman, K.S. and Michelson, K.A. (2016). Anaphylaxis in Simons, F.E.R., Ebisawa, M., Sanchez-Borges, M., et al. (2015).
children. Current Opinion in Pediatrics 28(3): 294–297. 2015 update of the evidence base: World Allergy
doi: 10.1097/MOP.0000000000000340. Organization anaphylaxis guidelines. World Allergy
Langley, E.W. and Gigante, J. (2013). Anaphylaxis, urticaria, Organization Journal 8(1): 32. doi: 10.1186/s40413-
and angioedema. Pediatrics in Review 34(5): 247–257. doi: 015-0080-1.
10.1542/pir.34-6-247.
Sicherer, S.H. and Simons, F.E.R. (2017). Epinephrine
for first-aid management of anaphylaxis. Pediatrics
139(3): e20164006. doi: 10.1542/peds.2016-4006.
Downloaded from https://onlinelibrary.wiley.com/doi/ by National Institute Of Standard, Wiley Online Library on [06/03/2024]. See the Terms and Conditions (https://onlinelibrary.wiley.com/terms-and-conditions) on Wiley Online Library for rules of use; OA articles are governed by the applicable Creative Commons License
463

111

A 10-Year-Old with Asthma in the PICU


Catherine M. Haut1,2
1
Nemours/Alfred I. duPont Hospital for Children, Wilmington, DE, USA
2
Mednax-Pediatrix Medical Group Richardson, TX, USA

A 10-year-old girl with moderate intermittent asthma diag- hypoxemia with wheezing. Current medications include
nosed at age 4 is admitted to the pediatric intensive care Proair MDI two puffs as needed for wheezing, montelukast
unit (PICU) with status asthmaticus. 5 mg daily, and Advair (fluticasone propionate/salmeterol
100 μg/50 μg) Diskus one puff twice daily. Parents report
compliance with medications. Despite being previously
History of Present Illness referred to pulmonology, she has not had an appointment.
Triggers for wheezing include cigarette smoke, drastic
This 10-year-old girl became ill 2 days ago with cough and changes in weather, and viral illness such as a cold.
upper respiratory congestion. She was determined by the
family to be wheezing, so she was given albuterol three times
today at home and twice yesterday with no real improve- Past Surgical History
ment of symptoms. She was also treated with over-the-
counter cough preparation which was also ineffective. There The girl had a tonsillectomy and adenoidectomy at age 6 for
is no history of fever, but the child spent a week at a cousin’s obstructive sleep apnea.
house where family members were smoking. She is not eat-
ing as usual and has complained of a stomach ache.
Family History

The father of the patient had asthma as a child, but has not
Past Medical History had any wheezing for many years. The patient has a 5-year-
old brother who had one episode of wheezing at age 2.
This patient was born at term and went home with her There is no known family history of allergies or eczema.
mother. She has had all required immunizations along with Neither parent smokes.
annual influenza vaccine. Her weight has been at 99% for
gender and age since age 3 and her current body mass index
(BMI) is 28.5. Diagnosis of asthma was made at age 4 after a Current Status
2-week inpatient hospitalization for wheezing and pneu-
monia. She required intubation and ventilation during On arrival to the emergency department (ED), the patient is
the hospitalization for respiratory failure, unresponsive to wheezing and hypoxic, with an oxygen saturation of 88% on
usual asthma medications, including continuous albuterol, room air. Her initial respiratory rate was 45 breaths per
methylprednisolone, and magnesium sulfate administered minute with mild subclavicular retractions. She is treated
intravenously. She was admitted again to the PICU at age 6, with oxygen, oral prednisone, and continuous albuterol
when she was managed with bilevel positive airway pres- administered by facemask. Oxygen is provided at 50% ini-
sure (BiPAP) and was discharged after 4 days in the hospi- tially and as she is admitted to the PICU, when she has
tal. She has had other episodes of wheezing, with two an increased oxygen requirement to 75% to maintain
additional admissions to the general pediatric unit due to saturations greater than 91%. A complete metabolic panel

Cases in Pediatric Acute Care: Strengthening Clinical Decision Making, First Edition. Edited by Andrea M. Kline-Tilford and Catherine M. Haut.
© 2020 John Wiley & Sons Ltd. Published 2020 by John Wiley & Sons Ltd.
Downloaded from https://onlinelibrary.wiley.com/doi/ by National Institute Of Standard, Wiley Online Library on [06/03/2024]. See the Terms and Conditions (https://onlinelibrary.wiley.com/terms-and-conditions) on Wiley Online Library for rules of use; OA articles are governed by the applicable Creative Commons License
464 Cases in Pediatric Acute Care

Table 111.1 Basic metabolic panel.

Sodium 141 mEq/L


Potassium 3.8 mEq/L
Chloride 101 mEq/L
Carbon dioxide 24 mEq/L
Blood urea nitrogen 12 mg/dL
Creatinine 0.6 mg/dL
Glucose 171 mg/dL
Calcium 8.6 mg/dL

Table 111.2 Complete blood count.

White blood cell count 18 000/mm3


Hematocrit 12 g/dL
Hemoglobin 36%
Platelet count 346 000/mm3
Neutrophils 79%
Lymphocytes 9%
Monocytes 6%
Eosinophils 6%

Figure 111.1 Admission chest radiograph showing


and complete blood count are obtained, with results as hyperinflation in status asthmaticus.
shown in Tables 111.1 and 111.2. A chest X-ray is obtained
and shows hyperinflation with concern for right upper and due to the child’s history and presenting symptoms of
middle lobe infiltrates (Figure 111.1). wheezing, cough, and hypoxia. This child also has reported
inciting factors of tobacco exposure and viral symptoms
that may have triggered the exacerbation.
Questions Status asthmaticus is defined as a severe asthma exacer-
bation not responsive to standard initial treatment doses of
Answer the following questions using the details provided. beta-agonists (e.g. albuterol, levalbuterol). Status asthmati-
cus is a medical emergency requiring prompt recognition,
1 Based on the data given, what are the differential diag- therapy, and hospitalization. Initial management of status
noses and the most likely diagnosis for this child? asthmaticus includes oxygen, beta-agonist therapy (e.g.
2 What is the best way to increase oxygenation and how continuous albuterol inhalation), and systemic steroids.
would you order this?
3 How would you determine if the therapy was effective?
What is the best way to increase oxygenation and
how would you order this?
Oxygen therapy often begins with delivery via a facemask;
Rationale and Evidence-based Practice
however, this is frequently not sufficient to support the oxy-
Explanation genation needs of these children and some will require the
application of non-invasive ventilation support to assist in
Based on the data given, what are the differential
gas exchange and distending pressure to improve oxygena-
diagnoses and the most likely diagnosis for
tion. Non-invasive ventilation such as continuous positive
this child?
airway pressure (CPAP) or bilevel positive airway pressure
Differential diagnoses for this child include status asthma- (BiPAP) is frequently delivered via facemask or nasal
ticus (associated with viral illness, exposure to allergens, or pillows/prongs, but there is little evidence that documents
other inciting factors), pneumonia, and aspiration. The the effectiveness of high-flow nasal cannula. When order-
most likely diagnosis for this child is status asthmaticus ing this therapy, the following settings will be needed:
Downloaded from https://onlinelibrary.wiley.com/doi/ by National Institute Of Standard, Wiley Online Library on [06/03/2024]. See the Terms and Conditions (https://onlinelibrary.wiley.com/terms-and-conditions) on Wiley Online Library for rules of use; OA articles are governed by the applicable Creative Commons License
Chapter 111 A 10-Year-Old with Asthma in the PICU 465

amount of oxygen, inspiratory positive airway pressure ventilation circuit. Additional therapies for this child
(IPAP, similar to pressure support on a conventional venti- include continuation of systemic steroids and consideration
lator), and expiratory positive airway pressure (EPAP, sim- of adjuvant pharmacologic therapies including additional
ilar to positive end-expiratory pressure on a conventional bronchodilators, anticholinergic medications (e.g. ipratro-
ventilator). pium bromide) and/or intravenous magnesium sulfate. In
some cases, aminophylline could be considered. A small
How would you determine if the therapy was population of children with status asthmaticus will require
effective? invasive and/or mechanical ventilation if other initial man-
agement strategies do not improve clinical status. Risks
Frequent, repeated physical examinations are needed in this
associated with the invasive positive-pressure ventilation
child to determine the effectiveness of therapy. Evaluating
of such children, who have air trapping related to their sta-
the child’s respiratory rate, work of breathing, inspiratory
tus asthmaticus, include the air leak syndromes (e.g. pneu-
to exhalation ratio, presence of wheezing, aeration, ability
mothorax, pneumomediastinum). In order to avoid such air
to speak, overall level of comfort (e.g. reduction or absence
leak syndromes, children with status asthmaticus requiring
of anxiousness), and oxygenation status are all needed to
intubation and mechanical ventilation often need liberal
assess the child’s response to therapy.
sedation/analgesia and paralytics. Other strategies that
Continuation of case: The girl is admitted to the PICU can be considered include administration of Heliox, a
and placed on BiPAP with settings of IPAP +12 cmH2O helium/oxygen gas mixture, or in severe cases, general
and EPAP +6 cmH2O, plus inhaled continuous albuterol anesthesia, although there is limited evidence to support
15 mg/hour. She is continued on intravenous methylpred- these therapies.
nisolone 2 mg/kg daily and started on ceftriaxone for pre- An assessment of the child’s hydration status is also
sumed bacterial pneumonia. She is breathing at a rate of needed. Children with respiratory distress have high insen-
30 breaths per minute with minimal subclavicular retrac- sible losses and may have had reduced oral intake prior
tions and is still found to have inspiratory and expiratory to presentation. These children may require fluid boluses
wheezing present. After 3 hours, oxygen has been and/or intravenous hydration.
decreased to 40%, her respiratory rate is 28 breaths per Evaluation for an infectious etiology is also needed.
minute, and she is still wheezing. Asthma exacerbations are most commonly associated with
viral illnesses, though may be associated with a bacterial
superinfection. A close evaluation of the complete blood
Additional Questions and Evidence- count and fever history/fever curve can assist in determin-
based Rationale ing the need for antibiotics.

4 Should albuterol be continued and what other therapies


are recommended at this time? Would you obtain any other diagnostic studies?
5 Would you obtain any other diagnostic studies? If the child has a history of a viral prodrome, viral studies
can be sent to assist in identifying the trigger of the exacer-
Should albuterol be continued and what other bation and to appropriately isolate the child if positive.
therapies are recommended at this time? Blood culture and/or C-reactive protein may be considered
if the history suggests a bacterial illness.
Beta-agonists are the mainstay of therapy for status asthma-
ticus and will be continued through the non-invasive

Further Reading
Henderson, M.B., Schunk, J.E., and Henderson, J.L. (2018). Kaiser, S.V., Rodean, J., and Bekmezian, A. (2018).
An assessment of asthma therapy in the pediatric ICU. Effectiveness of pediatric asthma pathways for hospitalized
Hospital Pediatrics 8(6): 361–367. doi: 10.1542/ children: a multicenter national analysis. Journal of
hpeds2017-0003. Pediatrics 197: 165–171.e2. doi: 10.1016/j.jpeds.2018.01.084.
Downloaded from https://onlinelibrary.wiley.com/doi/ by National Institute Of Standard, Wiley Online Library on [06/03/2024]. See the Terms and Conditions (https://onlinelibrary.wiley.com/terms-and-conditions) on Wiley Online Library for rules of use; OA articles are governed by the applicable Creative Commons License
467

112

Skin Wound in Postoperative Neonate


Hayley Redmond
Norton Children’s Hospital, Louisville, KY, USA

You are called to the bedside of a pediatric cardiac intensive arteriosus. He was treated empirically with antibiotics for
care unit (CICU) patient to evaluate a new skin wound in a 48 hours after birth. The remainder of his NICU course
neonate who has recently undergone heart surgery. was uneventful. He was taken to the operating room for
cardiac repair when he was 3 days old.

History of Present Illness


Past Surgical History
A 10-day-old infant with a past medical history of complex
congenital heart disease was admitted postoperatively to Underwent arterial switch procedure at 3 days of age.
the pediatric CICU 1 week ago following cardiac surgery
in which he underwent an arterial switch procedure. He
Family History
had a difficult operative course, but has been recovering
well and is hemodynamically stable. He remains endotra-
Parents and 1-year-old sibling are healthy.
cheally intubated and mechanically ventilated but is toler-
ating a slow wean of support. Five days ago he had an
arterial catheter placed in the right femoral artery and also
has a peripherally inserted central catheter in the right
Current Status
lower extremity which ends in the femoral vein. Yesterday,
Vital signs are as follows: temperature is elevated to 38 C
during a dressing change, the nurse noted that the area
(100.4 F), heart rate is 121 bpm, respiratory rate 32 breaths
around the arterial catheter in the right groin was swollen
per minute, blood pressure 70/40 mmHg, and oxygen satu-
and had purple discoloration to the skin, but blood return
ration 98% on 30% FiO2 on invasive mechanical ventilatory
from the cannula continued (Figure 112.1). The arterial
support. The daily basic metabolic panel and complete
catheter was removed, but today the area of discoloration
blood count are obtained with results as noted in
is starting to ulcerate and there are new areas of dark
Tables 112.1 and 112.2. A chest X-ray shows good position
and necrotic skin (Figure 112.2).
of endotracheal tube with no infiltrates. An arterial blood
gas analysis shows a pH of 7.43, PaCO2 of 51 mmHg, PaO2
of 74 mmHg, and a bicarbonate of 28 mmol/L. Lactic acid
Past Medical History is 0.8 mmol/L. C-reactive protein is 4.2 mg/L.

The patient was born at 39 weeks’ gestation via cesarean


section. There were no pregnancy complications and his
mother was healthy with negative prenatal laboratory
Questions
results. His cardiac abnormalities were discovered prena-
Answer the following questions using the details provided.
tally on ultrasound. After birth, he was admitted to the neo-
natal intensive care unit (NICU) and started on 1 Based on the data given, what are your differential diag-
prostaglandin infusion to maintain patency of his ductus noses and the most likely diagnosis for this infant?

Cases in Pediatric Acute Care: Strengthening Clinical Decision Making, First Edition. Edited by Andrea M. Kline-Tilford and Catherine M. Haut.
© 2020 John Wiley & Sons Ltd. Published 2020 by John Wiley & Sons Ltd.
Downloaded from https://onlinelibrary.wiley.com/doi/ by National Institute Of Standard, Wiley Online Library on [06/03/2024]. See the Terms and Conditions (https://onlinelibrary.wiley.com/terms-and-conditions) on Wiley Online Library for rules of use; OA articles are governed by the applicable Creative Commons License
468 Cases in Pediatric Acute Care

Figure 112.1 Discoloration of skin. Source: Courtesy of Megan Figure 112.2 Areas of ulceration. Source: Courtesy of Megan
Boone, MSN, RN. Boone, MSN, RN.

2 What is the best next step in the management of this Table 112.1 Basic metabolic profile.
infant?
3 Would you obtain any other diagnostic studies at Sodium 131 mEq/L
this time? Potassium 3.7 mEq/L
Chloride 96 mEq/L
Carbon dioxide 28 mEq/L
Blood urea nitrogen 26 mg/dL
Rationale and Evidence-based Practice Creatinine 0.5 mg/dL
Explanation Glucose 92 mg/dL

Based on the data given, what are your differential Table 112.2 Complete blood count.
diagnoses and the most likely diagnosis for this
infant? White blood cell count 20 400/mm3
Hemoglobin 11.6 g/dL
Differential diagnoses for this infant include cellulitis,
necrotizing fasciitis, pyoderma gangrenosum, Fournier’s Hematocrit 36%
gangrene, intravenous extravasation, and deep venous Platelet count 334 000/mm3
thrombosis. The most likely diagnosis is necrotizing Neutrophils 62%
fasciitis given the clinical picture and rapid progression. Eosinophils 1%
Necrotizing fasciitis is an infection of the deep soft tissues Monocytes 8%
which results in necrosis of muscle fascia and overlying Lymphocytes 30%
subcutaneous tissue. The infection typically begins with
Bands 0%
trauma to, or a break in, the skin and rapidly spreads to
Downloaded from https://onlinelibrary.wiley.com/doi/ by National Institute Of Standard, Wiley Online Library on [06/03/2024]. See the Terms and Conditions (https://onlinelibrary.wiley.com/terms-and-conditions) on Wiley Online Library for rules of use; OA articles are governed by the applicable Creative Commons License
Chapter 112 Skin Wound in Postoperative Neonate 469

the muscle fascia. Necrotizing fasciitis is considered a med-


ical and surgical emergency.
The initial presentation consists of pain, edema, and ery-
thema of the soft tissue. The area will feel firm and indu-
rated. These skin changes are similar to those of cellulitis
or abscess, making early diagnosis difficult. However, as
necrotizing fasciitis progresses, a purple discoloration of
the skin may be noted due to bleeding from damaged
blood vessels, bullae formation, ulceration, or presence
of gas in the tissue which results in crepitus. Patients
may or may not develop fever. This condition can rapidly
progress and result in sepsis and septic shock despite anti-
biotic therapy.
Necrotizing fasciitis is caused by bacterial infection, most
commonly with a mixture of various types of bacteria, spe-
cifically Gram-positive cocci including Streptococcus, Figure 112.3 Erythematous spread to scrotum and right lower
Gram-negative rods including Escherichia coli, and anae- extremity. Source: Courtesy of Megan Boone, MSN, RN.
robes such as Clostridium species.

What is the best next step in the management of


this infant?
The diagnosis of necrotizing fasciitis is confirmed with
surgical exploration of the tissues in the operating room.
It is very important not to delay surgical intervention when
there is a clinical suspicion for necrotizing fasciitis, as
delays in surgery are associated with a higher risk of
mortality. The infection is treated with surgical debride-
ment of necrotic tissue and intravenous antibiotics. Antibi-
otic therapy should include activity against Gram-positive,
Gram-negative, and anaerobic organisms.

Would you obtain any other diagnostic studies at


this time?
When necrotizing fasciitis is suspected, laboratory test-
ing is reasonable including inflammatory markers of
infection. Blood cultures should be obtained prior to
Figure 112.4 Erythematous spread to scrotum and right lower
starting antibiotics, if possible. Computed tomography
extremity. Source: Courtesy of Megan Boone, MSN, RN.
(CT) may be able to help determine whether a necrotiz-
ing infection is present, but should not delay surgical
intervention. Additional Question and
Evidence-based Rationale
Continuation of case: Over the next several hours, the
surrounding tissue becomes erythematous and the redness 4 What other therapies should be considered at this time?
spreads rapidly to the scrotum and right lower extremity
(Figures 112.3 and 112.4). The patient is empirically started
What other therapies should be considered at
on broad-spectrum antibiotics including clindamycin, cef-
this time?
triaxone, and vancomycin. The plastic surgery service is
emergently consulted and the patient is taken to the oper- With progression of infection despite antibiotic therapy, the
ating room for surgical exploration and wound debride- patient may show signs of sepsis or septic shock. Fluid
ment. Blood cultures and cultures from the right groin resuscitation and careful monitoring of vital signs, blood
wound are obtained. pressure, and perfusion are important.
Downloaded from https://onlinelibrary.wiley.com/doi/ by National Institute Of Standard, Wiley Online Library on [06/03/2024]. See the Terms and Conditions (https://onlinelibrary.wiley.com/terms-and-conditions) on Wiley Online Library for rules of use; OA articles are governed by the applicable Creative Commons License
470 Cases in Pediatric Acute Care

Further Reading
Hakkarainen, T.W., Kopari, N.M., Pham, T.N., and Evans, H.L. Stevens, D.L., Bisno, A.L., Chambers, H.F., et. al. (2014).
(2014). Necrotizing soft tissue infections: review and current Practice guidelines for the diagnosis and management of
concepts in treatment, systems of care, and outcomes. skin and soft tissue infections: 2014 update by the Infectious
Current Problems in Surgery 51(8): 344–362. Diseases Society of America. Clinical Infectious Diseases 59
Stevens, D.L. and Bryant, A.E. (2017). Necrotizing soft-tissue (2): e10–e52.
infections. New England Journal of Medicine 377(23):
2253–2265.
Downloaded from https://onlinelibrary.wiley.com/doi/ by National Institute Of Standard, Wiley Online Library on [06/03/2024]. See the Terms and Conditions (https://onlinelibrary.wiley.com/terms-and-conditions) on Wiley Online Library for rules of use; OA articles are governed by the applicable Creative Commons License
471

113

Provider Burnout and Resiliency


Jill Siegrist Thomas
University of Maryland Children’s Hospital, University of Maryland Medical Center, Baltimore, MD, USA

A novice acute-care provider is working in an intermediate- how to perform a dressing change. She again feels her
level pediatric burn unit. She completed a structured orien- colleagues are questioning her abilities and she is underap-
tation 3 months ago and works a rotating day and night preciated. She leaves work feeling defeated and questions if
schedule. Clinical responsibilities include the comprehen- she’s made the right career change.
sive diagnosis, assessment and management of infants
2 What are the symptoms of burnout syndrome?
and children status post second- and third-degree thermal
3 What interventions potentially prevent burnout syn-
and chemical burns. She is active in multidisciplinary
drome and enhance personal well-being?
rounds, frequently participates in the admission, transfer
4 What strategies can this provider employ to foster
and discharge of patients, and performs moderate sedation
resiliency?
and/or sterile dressing changes daily for assigned patients.
She often works long shifts to complete the required written
documentation and research the literature related to the
patient population. While she states she enjoys her job, Rationale and Evidence-based Practice
she feels her new occupation consumes her life and has lit-
Explanation
tle time outside of work to participate in activities she for-
merly enjoyed.
What concerns or identified problems are included
in this scenario?
Personal characteristics, organizational factors, quality of
Questions working relationships, and exposure to end-of-life issues
place individuals at risk for developing “burnout syn-
Answer the following questions using the details provided. drome.” Personal risk factors associated with burnout syn-
drome include work–life imbalance, engagement in
1 What concerns or identified problems are included in
unhelpful coping strategies, sleep deprivation, and being
this scenario?
self-critical, perfectionistic and over-committed. Organiza-
Continuation of case: During multidisciplinary rounds tional factors associated with burnout syndrome include a
the novice provider presents her plan to moderately sedate highly pressured or chaotic work environment, increasing
a patient with ketamine and midazolam. The patient’s workload, loss of control of the work environment, process
nurse quickly interjects to state her preference of using pro- or schedule, insufficient rewards, and breakdown of the
pofol for moderate sedation due to institutional “norm.” work community. These risk factors can lead to unclear
The provider educates the nurse that propofol is contrain- or overly demanding occupational responsibilities, perfor-
dicated in this patient due to an egg allergy, but feels frus- mance of menial tasks, and poor alignment of personal
trated and undermined and interprets the interaction as not values with organizational leadership (Moss et al., 2016;
being trusted by the nursing staff. During the dressing Pastores et al., 2019). Poor working relationships can lead
change the surgical resident does not know she independ- to conflict and foster ineffective communication. Strained
ently performs dressing changes and tries to educate her on relationships can occur between colleagues, supervisors,

Cases in Pediatric Acute Care: Strengthening Clinical Decision Making, First Edition. Edited by Andrea M. Kline-Tilford and Catherine M. Haut.
© 2020 John Wiley & Sons Ltd. Published 2020 by John Wiley & Sons Ltd.
Downloaded from https://onlinelibrary.wiley.com/doi/ by National Institute Of Standard, Wiley Online Library on [06/03/2024]. See the Terms and Conditions (https://onlinelibrary.wiley.com/terms-and-conditions) on Wiley Online Library for rules of use; OA articles are governed by the applicable Creative Commons License
472 Cases in Pediatric Acute Care

Table 113.1 Key resiliency competencies and development strategies.

Competency Associated attributes Development strategies

Reflective Recognize own strengths and limitations


•• Journaling: narrative writing

••
ability Debriefing
Utilize a coach or mentor
Find meaning in work and setting realistic short-term
and long-term goals

Emotional Self-awareness: recognize own emotions and their effects,


• Experiential learning, e.g. simulated practice, case


intelligence accurate self-assessment, have self-confidence (sure about studies to facilitate dialogue and reflection
self-worth) Attend professional conferences, workshops, etc. to


Self-management: manage disruptive emotions and build communication, leadership skills
impulses, conscientious, self-motivated demonstrating Exercise skills/strategies to reduce stress and frustration
commitment, initiative and optimism in work environment (e.g. “step away”, take a deep


Social awareness: empathy and culturally sensitive patient- breath, “act rather than react,” establish boundaries)
centered care Maintain physical health (e.g. exercise, obtain adequate


Social skills: collaborative, cooperative, skilled sleep, practice good nutrition)
communicator Practice gratitude
Social Social confidence, assertiveness, and well-developed
• Exercise mindfulness and relaxation techniques

••
competency communication and conflict resolution skills (e.g. yoga, meditation, deep breathing activities)
Find time for yourself every day

••
Celebrate successes
Develop rituals to release tension


Maintain sense of humor
Ask for help when needed
Social
support
Positive interactions with others with whom there is a
mutual trust and respect •• Mentoring and peer coaching
Foster professional relationships and create a caring
community: connect with peers, friends and family on a


regular basis
Participate in a novice network (e.g. organizational NP


committee)
Develop hobbies outside of work

and patients and their families and is a prominent risk includes communication training to reduce interpersonal
factor for burnout. conflict, appropriate staffing, and meaningful recognition
for completion of occupational duties. Providers may
What are the symptoms of burnout syndrome? benefit from self-scheduling practices and time off. Support
groups and cognitive therapy can address the cause of
Heathcare providers need to recognize the symptoms of
moral distress and provide coping skills to increase perso-
burnout syndrome and seek personal assistance and sup-
nal resilience and emotional intelligence.
port. Symptoms are vague, develop gradually, and can be
Team-based interventions to optimize team culture
both psychological and physical in nature. Psychological
include use of structured communication tools, team
symptoms include feelings of job-related disillusionment,
building, interpersonal skills training activities, and team
feeling insufficient at work, anger, frustration, anxiety, dis-
debriefings. Healthcare providers are encouraged to partici-
content, and the inability to feel happiness or pleasure.
pate in stress reduction training, meditation or relaxation
Physical symptoms include fatigue, exhaustion, insomnia,
techniques to increase coping skills. Self-care measures
headache, muscle tension, and gastrointestinal problems
include provision of adequate rest, exercise, and assurance
(Moss et al., 2016).
of work–life balance through time management and
engagement in social activities (Pastores et al., 2019). The
What interventions potentially prevent burnout
development of personal coping skills emphasizing the
syndrome and enhance personal well-being?
core principles of emotional intelligence (see Table 113.1) –
Interventions focused on preventing burnout target the self-awareness, self-management, social awareness and
organizational and personal drivers. Social context and inter- social skills – is essential to optimize personal wellness,
action with colleagues play a pivotal role in the development improve stress management skills, and prevent burnout
of burnout. Promotion of a healthy work environment (Shahid et al., 2018).
Downloaded from https://onlinelibrary.wiley.com/doi/ by National Institute Of Standard, Wiley Online Library on [06/03/2024]. See the Terms and Conditions (https://onlinelibrary.wiley.com/terms-and-conditions) on Wiley Online Library for rules of use; OA articles are governed by the applicable Creative Commons License
Chapter 113 Provider Burnout and Resiliency 473

What strategies can this provider employ to foster rather than merely survive. The foundation for building
resiliency? resiliency is multidimensional and focuses on self-care
and self-awareness. Table 113.1 outlines key resiliency
Resiliency is defined as the capacity to recover after a stress-
competencies and associated developmental strategies.
ful situation and this attribute allows individuals to thrive
and succeed in the face of adversity and stressful situations

References
Moss, M., Good, V.S., Gozal, D., Kleinpell, R., and Sessler, C.N. Care Medicine 47(4): 550–557. doi: 10.1097/
(2016). An official critical care societies collaborative CCM.0000000000003637.
statement: burnout syndrome in critical care healthcare Shahid, R., Stirling, J., and Adams, W. (2018). Promoting
professionals: a call for action. Chest 150(1): 17–26. doi: wellness and stress management in residents through
10.1016/j.chest.2016.02.649. emotional intelligence training. Advances in Medical
Pastores, S., Kvetan, V., Coopersmith, C., et al. (2019). Education and Practice 9: 681–686. doi: 10.2147/AMEP.
Workforce, workload, and burnout among intensivists and S175299.
advanced practice providers: a narrative review. Critical
Downloaded from https://onlinelibrary.wiley.com/doi/ by National Institute Of Standard, Wiley Online Library on [06/03/2024]. See the Terms and Conditions (https://onlinelibrary.wiley.com/terms-and-conditions) on Wiley Online Library for rules of use; OA articles are governed by the applicable Creative Commons License
475

114

Kidney Transplant Complication


Cathy C. McAdams
Nemours/Alfred I. duPont Hospital for Children, Wilmington, DE, USA

A 17-year-old African American female with a history of received a deceased donor transplant. At the time of trans-
focal segmental glomerulosclerosis (FSGS) is now 1 year plant, the patient was EBV (Epstein–Barr virus) and CMV
post successful deceased donor kidney transplant. She (cytomegalovirus) negative and the donor was both EBV
was admitted to the hospital for lethargy, diarrhea, fever, and CMV positive. Creatinine at discharge was 0.6 mg/
decreased urine output, and an elevated creatinine. dL, on intravenous fluids; her creatinine post discharge
has been stable at 0.6–0.8 mg/dL depending on oral intake.
Her blood pressure on twice daily amlodipine is 118/
68 mmHg. Her medication regimen is tacrolimus 2 mg
History of Present Illness
twice daily, mycophenolate mofetil 750 mg twice daily,
prednisone 5 mg daily, amolodipine 5 mg twice daily,
This teen was well up until 2 days ago when she started
and vitamin D 1000 mg daily.
feeling tired. She denies any cold symptoms, headaches,
dizziness, vomiting or edema. She does report loose stools
(four to six times over the last 48 hours), decreased oral
fluid intake, decreased urine output, and mild burning with Past Surgical History
urination. She has not been checking her temperature at
home because she reports not being able to find the Deceased donor kidney transplant and central dialysis cath-
thermometer, but she states that she feels hot (especially eter removal after transplant.
at night) and has been having chills.

Family History
Past Medical History
Family history is negative for kidney disease or kidney
The patient was born full term with no known complica- stones.
tions. Prior to presentation of her renal disease, her health
history consisted of “several ear infections as a child” and
extraction of molars 3 years ago. Two years ago, the teen Current Status
went to the dentist and was found to be hypertensive with
blood pressure 180/110 mmHg She was sent to the local The teen presents to the emergency room with diarrhea and
emergency department where hypertension was confirmed malaise. Her temperature is 39 C (102.3 F), heart rate 135
and laboratory studies obtained. At that time, her creati- bpm, and blood pressure 90/42 mmHg. Her weight is 48 kg,
nine was 7.6 mg/dL, and she had large amounts of protein down about 2 kg from baseline and she reports feeling light-
and blood in her urine. She was sent to a pediatric nephrol- headed. A chest X-ray, transplant ultrasound, urine culture,
ogist and underwent a percutaneous kidney biopsy and and blood cultures are obtained. Blood work on admission
subsequently was diagnosed with FSGS and started on reveals a serum creatinine of 1.4 mg/dL, about a 40% reduc-
hemodialysis three times a week. After 12 months, she tion in her glomerular filtration rate (GFR). The patient is

Cases in Pediatric Acute Care: Strengthening Clinical Decision Making, First Edition. Edited by Andrea M. Kline-Tilford and Catherine M. Haut.
© 2020 John Wiley & Sons Ltd. Published 2020 by John Wiley & Sons Ltd.
Downloaded from https://onlinelibrary.wiley.com/doi/ by National Institute Of Standard, Wiley Online Library on [06/03/2024]. See the Terms and Conditions (https://onlinelibrary.wiley.com/terms-and-conditions) on Wiley Online Library for rules of use; OA articles are governed by the applicable Creative Commons License
476 Cases in Pediatric Acute Care

admitted and a percutaneous kidney biopsy is scheduled for Table 114.3 Urinalysis.
the next day.
Color Dark yellow
Results of a comprehensive metabolic panel, complete
blood count, and urinalysis are shown in Tables 114.1–114.3. pH 6.5
The chest X-ray image is normal, while transplant ultra- Specific gravity 1.030
sound shows no hydronephrosis, although the kidney Protein 2+
may have slightly increased echodensity, thought to be Nitrites Positive
artifactual. Urobilirubin 1+
Blood Large
Leukocytes Positive
Questions Ketones Negative
Glucose Negative
Answer the following questions using the details provided.
1 Based on the data given, what are the most likely differ-
ential diagnoses for this patient?
2 What therapy would you consider starting to try to 3 How would you determine if the therapy is effective?
improve the patient’s kidney function? 4 What additional studies should be ordered?
5 What therapies could be offered if the creatinine does
not improve with rehydration?
Table 114.1 Comprehensive metabolic panel.

Sodium 138 mEq/L


Potassium 5.4 mEq/L Rationale and Evidence-based Practice
Chloride 110 mEq/L Explanation
Carbon dioxide 16 mEq/L
Blood urea nitrogen 42 mg/dL Based on the data given, what are the most likely
Creatinine 1.4 mg/dL differential diagnoses for this patient?
Glucose 126 mg/dL For a stable transplant patient who has had an acute
Calcium 9.8 mg/dL increase in a previously stable creatinine, the differential
Albumin 3.0 mg/dL diagnosis should include:
Total protein
Aspartate aminotransferase
6.0 g/dL
24 U/L • a pre-renal state secondary to poor perfusion related to
diarrheal dehydration and hypotension;
Alanine aminotransferase
Alkaline phosphatase
24 U/L
60 U/L
• intrarenal injury related to drug toxicity (tacrolimus),
rejection, recurrence of her FSGS in the transplanted kid-
ney, or viral/bacterial-related infection;


Total bilirubin 0.5 mg/dL
Magnesium 1.6 mEq/L post-renal obstruction related to a kidney stone or
blood clot.
Phosphorus 4.2 mg/dL

What therapy would you consider starting to try


Table 114.2 Complete blood count. to improve the patient’s kidney function?
White blood cell count 10 500/mm3 Because of the patient’s diarrhea and dehydration along
Hemoglobin 12.2 g/dL with weight loss, initial therapy would be administration
Hematocrit 36.6% of an intravenous fluid bolus of normal saline followed
Platelet count 195 000/mm3 by an increased maintenance rate of 1.5 times normal.
Neutrophils 36%
How would you determine if the therapy is
Lymphocytes 42%
effective?
Monocytes 13%
Eosinophils 1% An improvement in the serum creatinine, with return to
baseline values, on rehydration would indicate that the
Downloaded from https://onlinelibrary.wiley.com/doi/ by National Institute Of Standard, Wiley Online Library on [06/03/2024]. See the Terms and Conditions (https://onlinelibrary.wiley.com/terms-and-conditions) on Wiley Online Library for rules of use; OA articles are governed by the applicable Creative Commons License
Chapter 114 Kidney Transplant Complication 477

elevated serum creatinine was due to something that was 30% chance of recurrence in the transplanted kidney.
either transient or reversible and related to dehydration. Although this is more common immediately after trans-
The clinician would also be reassured if the patient plant, it can occur at any point and should remain in
became normotensive, and had a clean urine (free of the differential, especially if the patient is noted to have
protein, blood and/or leukocytes) and near-normal serum proteinuria and a lower serum albumin.
albumin. Dehydration secondary to diarrhea and decreased fluids
is the most likely cause of the patient’s increased serum cre-
What additional studies should be ordered? atinine. She has experienced several days of frequent loose
stools and a decrease in her urine output from baseline.
In addition to the laboratory and imaging studies already
Tacrolimus toxicity (or an elevated tacrolimus level) can
carried out, it would be important to obtain a tacrolimus
be the result of persistent diarrhea and can also result in
level, viral PCR for EBV, CMV and BK polyoma virus, as
an increase in the serum creatinine. Diarrhea and dehydra-
well as donor-specific antibodies. A random urine pro-
tion, especially if the tacrolimus level is increased, can also
tein/creatinine ratio should be sent as well; this would
lead to acute renal injury.
ideally be a first morning sample.
Rejection of the kidney transplant can occur at any time.
Although the cause of this in the early months can be
What therapies could be offered if the creatinine
purely immunologic, non-compliance should be consid-
does not improve with rehydration?
ered in the patient who is further out from transplant. Teen-
Intravenous fluids should be infused at 1.5 times mainte- agers in particular are at increased risk of losing kidneys
nance if the diarrhea continues. Antidiarrheal agents could because of skipping or not taking their immunosuppressant
be administered in the short term to prevent loss of electro- medications.
lytes and tacrolimus toxicity. Laboratory work should be Another thing to consider in this patient who has previ-
repeated 8–12 hours after starting intravenous fluids to ously had no evidence of CMV or EBV would be a change in
determine a trend in the creatinine and to evaluate electro- viral status.
lytes. Because the urine is positive for blood, protein, and Finally, the increase in the creatinine could be the result
leukocytes, and the patient had urinary tract symptoms, of an infection, and in this case where the urine is positive
antibiotics should be started after the blood and urine cul- for protein, blood and leukocytes, there should be higher
tures are obtained and continued until the 24-hour culture suspicion of a urinary tract infection until the urine culture
results are obtained. results show otherwise. In the presence of a fever and
urinary tract symptoms, initiation of antibiotics is war-
Continuation of case: Over the first 12 hours, the
ranted while waiting on the urine culture results. Urinary
serum creatinine improves to 1.0 mg/dL (still above
tract infections (especially pyelonephritis) can damage
baseline), and her blood pressure and hydration status
the transplanted kidney causing acute or chronic injury.
normalized. Her pre-renal state is reversed and she has
no evidence of obstruction on ultrasound. Her differen-
tial diagnoses at this point still include recurrence of
disease, resolving dehydration effect, tacrolimus toxicity, Case Resolution
acute renal injury, acute rejection, or urinary tract
infection. At 24 hours post admission, the creatinine has returned to
baseline (0.8 mg/dL), and blood and urine cultures are
negative. The tacrolimus level was elevated to 15 ng/mL
Additional Question and
on admission and returned to 5 ng/mL with improvement
Evidence-based Rationale of diarrhea and hydration. The patient’s urine has
improved, with only trace blood and protein with negative
6 Based on these supplementary data, what are now the
leukocytes. Blood and urine cultures were negative at
most likely differential diagnoses for this patient?
24 hours and antibiotics were discontinued. At this time,
with a baseline creatinine, no proteinuria, and negative
Based on these supplementary data, what are now
cultures, recurrence, rejection, infection, and acute kidney
the most likely differential diagnoses for this
injury related to dehydration or tacrolimus toxicity can be
patient?
excluded. The decision was made to cancel her transplant
FSGS is an illness of the kidney that causes scarring in the biopsy and the patient was discharged home on her rou-
glomeruli and can lead to kidney failure. There is about a tine medications.
Downloaded from https://onlinelibrary.wiley.com/doi/ by National Institute Of Standard, Wiley Online Library on [06/03/2024]. See the Terms and Conditions (https://onlinelibrary.wiley.com/terms-and-conditions) on Wiley Online Library for rules of use; OA articles are governed by the applicable Creative Commons License
478 Cases in Pediatric Acute Care

Further Reading
Avery, R.K. and Fishman, J.A. (2011). Infectious diseases in Rodig, N.M., Vakili, K., and Harmon, W.E. (2016). Pediatric
transplantation. In: Primer on Transplantation, 3rd edn renal transplantation. In: Pediatric Nephrology, 7th edn
(ed. D. Hricik), 51–72. Chichester: Wiley-Blackwell. (ed. E.D. Avner, W.E. Harmon, P. Niaudet et al.), 2519–2534.
Dharnidharka, V.R. and Araya, C.E. (2016). Complications New York: Springer.
of pediatric renal transplantation. In: Pediatric Nephrology,
7th edn (ed. E.D. Avner, W.E. Harmon, P. Niaudet et al.),
2574–2578. New York: Springer.
Downloaded from https://onlinelibrary.wiley.com/doi/ by National Institute Of Standard, Wiley Online Library on [06/03/2024]. See the Terms and Conditions (https://onlinelibrary.wiley.com/terms-and-conditions) on Wiley Online Library for rules of use; OA articles are governed by the applicable Creative Commons License
479

115

Infant with Influenza A and New Fever


Lauren Flagg
Yale New Haven Hospital, New Haven, CT, USA

A 4-month-old female, born at 28 weeks, is admitted to the discharge. A small patent ductus arteriosus was noted
pediatric intensive care unit (PICU) with respiratory failure shortly after delivery, for which no intervention has been
secondary to influenza A infection, now with new onset required. She is followed by ophthalmology for retinopathy
of fever. of prematurity. Her immunizations are up to date.

History of Present Illness Past Surgical History

The infant presented to the emergency department (ED) None.


14 days prior with fever, respiratory distress, lethargy,
and poor oral intake. Her older brother had tested positive
for influenza A several days earlier with similar symptoms. Family History
She was initially admitted to the regular nursing floor for
management. Shortly after arrival, her work of breathing Has a 4-year-old brother who is otherwise healthy, as are
worsened and she required transfer to the PICU for escala- both parents.
tion of monitoring and management. She required intuba-
tion for respiratory failure, with subsequent placement of a
femoral central venous catheter and radial arterial line. Current Status
Since PICU admission she has slowly defervesced, with
hemodynamic stability and improved ventilation and oxy- The infant is intubated, sedated, mechanically ventilated,
genation on weaning ventilator settings. Over the past and receiving inotropic support. Mean arterial pressure is
6 hours, however, she has developed fever to 39.5 C maintained above 45 mmHg on epinephrine 0.1 μg/kg
(103.1 F) with hypotension requiring fluid resuscitation per min. Over the past 6 hours she has received a total of
and initiation of inotropic support. 40 mL/kg in fluid resuscitation. Her current temperature
is 38.5 C (101.3 F) rectally after 15 mg/kg of intravenous
acetaminophen. Lung sounds are clear with good chest wall
Past Medical History excursion bilaterally. She is tachycardic without cardiac
murmurs. Capillary refill is 5 seconds. There are no rashes.
The child was born at 28 weeks’ gestation to a gravida 3 Central line and peripheral intravenous catheter sites are
para 2 mother with incompetent cervix. Maternal screening without signs of inflammation. A basic metabolic panel
for transmittable infections was unrevealing. She was ini- and a complete blood count with differential are obtained.
tially admitted to the neonatal intensive care unit A chest radiograph from earlier in the day demonstrated
(NICU), requiring intubation for 2 days, but was in room stable, patchy perihilar infiltrates. Respiratory secretions
air prior to discharge at age 10 weeks of life. She is fed a for- are unchanged, thin and white. Urine output is decreased
tified infant formula by mouth and prior to this admission from baseline, but clear and without odor.
had been gaining weight appropriately since NICU Laboratory results are shown in Tables 115.1 and 115.2.

Cases in Pediatric Acute Care: Strengthening Clinical Decision Making, First Edition. Edited by Andrea M. Kline-Tilford and Catherine M. Haut.
© 2020 John Wiley & Sons Ltd. Published 2020 by John Wiley & Sons Ltd.
Downloaded from https://onlinelibrary.wiley.com/doi/ by National Institute Of Standard, Wiley Online Library on [06/03/2024]. See the Terms and Conditions (https://onlinelibrary.wiley.com/terms-and-conditions) on Wiley Online Library for rules of use; OA articles are governed by the applicable Creative Commons License
480 Cases in Pediatric Acute Care

Table 115.1 Basic metabolic profile. Tracheitis should also be considered due to previous viral
illness and prolonged intubation, though is unlikely in
Sodium 140 mEq/L
the setting of unchanged respiratory secretions. Because
Potassium 3.9 mEq/L of her young age, meningitis should also be considered;
Chloride 108 mEq/L however, this infant is not displaying any meningitic signs
Carbon dioxide 18 mEq/L of changes in mental status, so this would be low on the dif-
Blood urea nitrogen 16 mg/dL ferential diagnosis. Another consideration for the differen-
Creatinine 0.4 mg/dL tial diagnosis includes urinary tract infection (UTI), which
Glucose 101 mg/dL
is the second most common infection in the intensive care
unit. However, most UTIs are associated with indwelling
urinary catheters, not present in this infant. Finally, a
new respiratory virus should also be considered on the dif-
Table 115.2 Complete blood count.
ferential diagnosis. While this is possible, it is less likely due
White blood cell count 30 000/mm3 to the unchanged respiratory examination and the promi-
Hemoglobin 11 g/dL nent hemodynamic instability.
Hematocrit 33%
What additional studies would you order?
Platelet count 425 000/mm3
Neutrophils 88% Blood cultures should be obtained in a patient with new
Monocytes 6% fever when the history and clinical evaluation do not indi-
Lymphocytes 6% cate a non-infectious cause. Guidelines recommend that in
patients with indwelling vascular catheters, at least two
blood cultures should be obtained, one from peripheral
venipuncture and one from the intravascular catheter with
Questions suspected infection. Obtaining acute-phase reactants, such
as procalcitonin, may help to evaluate the risk of bacterial
Answer the following questions using the details provided.
infection as the cause of fever. Procalcitonin is specific to
1 Based on these data, what differential diagnoses would bacterial infections. Viral infection, surgery, and chronic
you consider as the cause of the new fever? inflammation are not associated with increased procalcito-
2 What additional studies would you order? nin levels. A procalcitonin level of less than 0.5 ng/mL indi-
3 What empiric management would you begin and why? cates that bacterial infection is not present; levels of 0.6–2
ng/mL are consistent with systemic inflammatory response
syndrome; levels of 2–10 ng/mL are indicative of severe sep-
sis; and levels above 10 ng/mL indicate septic shock. Addi-
Rationale and Evidence-based Practice tionally, to exclude other foci of infection, respiratory and
Explanation urinary cultures are valuable to evaluate for ventilator-
associated pneumonia, tracheitis, and UTI.
Based on these data, what differential diagnoses
would you consider as the cause of the new fever? What empiric management would you begin
and why?
In this case, the combination of leukocytosis and hemody-
namic instability requiring volume resuscitation and ino- Antibiotic coverage should begin after cultures are
tropic support would suggest an infectious process. obtained. However, inability to obtain cultures (such as a
Leading considerations include the following: lumbar puncture) should not delay initiation of antibiotic
Central line-associated bloodstream infection (CLABSI) administration. In this case, the primary concern is the
is high on the differential diagnosis as she has an existing development of CLABSI due to the severity of the clinical
central line and hemodynamic instability. This is the most picture and shock-like state. Empiric antibiotic therapy
likely cause, but other sources of severe infection should should be selected to cover the most common CLABSI
also be considered. organisms, with consideration to common facility-specific
Pneumonia should be considered due to previous respira- nosocomial pathogens as well as institutional protocols.
tory illness with ongoing intubation. It is less likely in the The most common CLABSI pathogens are staphylococcal
setting of normal respiratory examination, normal chest species (including Staphylococcus aureus and the coagu-
radiograph, and unchanged ventilator requirements. lase-negative staphylococci) and enteric Gram-negative
Downloaded from https://onlinelibrary.wiley.com/doi/ by National Institute Of Standard, Wiley Online Library on [06/03/2024]. See the Terms and Conditions (https://onlinelibrary.wiley.com/terms-and-conditions) on Wiley Online Library for rules of use; OA articles are governed by the applicable Creative Commons License
Chapter 115 Infant with Influenza A and New Fever 481

bacilli (such as Escherichia coli and Klebsiella spp.). In Additional Questions and
settings with significant rates of methicillin-resistant Evidence-based Rationale
S. aureus (MRSA), as well as in patients with high suspicion
for infection (such as known colonization), vancomycin 4 Based on the blood culture results, what pathogens are
should be included in the initial therapy. When deter- most likely?
mining coverage for Gram-negative bacilli, both local 5 What additional management changes would you
susceptibility data and the severity of infection should prioritize?
be considered. Coverage typically should include a
fourth-generation cephalosporin such as carbapenem, or
a β-lactam/β-lactamase inhibitor combination, with or Based on the blood culture results, what
without the addition of an aminoglycoside. Patients in pathogens are most likely?
whom Pseudomonas aeruginosa is a concern (i.e. those with
immunocompromise, severe sepsis, or known colonization) Staphylococcal species, specifically S. aureus due to the
should receive an antibiotic regimen with appropriate severity of the disease. Consider the likelihood for MRSA
coverage. Concern for CLABSI in patients with femoral based on facility and patient risks.
vascular access lines should include coverage for Gram-
negative bacilli, with consideration given to Candida spp. What additional management changes would you
prioritize?
Continuation of case: The patient is empirically started
on vancomycin and cefotaxime after obtaining peripheral Removal of central lines in cases of severe CLABSI is a
and central blood cultures, urine culture, and respiratory priority in management. Because of hemodynamic instabil-
cultures. Lumbar puncture is deferred due to hemo- ity, she will require a new central line to facilitate manage-
dynamic instability. Procalcitonin resulted at 4.5 ng/mL. ment. Placement of a new central venous catheter in a new
Twelve hours later, preliminary results from both blood location is preferable to exchanging the existing line over a
cultures are positive for Gram-positive cocci in clusters. guidewire. Consultation with infectious diseases is recom-
Perfusion remains decreased, and she continues to receive mended for ongoing therapy. Empiric antibiotics should be
epinephrine drip at 0.1 μg/kg per min without further continued pending speciation and susceptibilities of the
advancement. bacteria, after which coverage may be narrowed.

Further Reading
American Academy of Pediatrics (2011). Urinary tract Infectious Diseases Society of America. Clinical Infectious
infection: clinical practice guideline for the diagnosis and Diseases 49(1): 1–45. doi: 10.1086/599376.
management of the initial UTI in febrile infants and O’Grady, N.P., Barie, P.S., Bartlett, J.G., et al. (2008).
children 2 to 24 months. Pediatrics 128(3): 595–610. doi: Guidelines for evaluation of new fever in critically ill adult
10.1542/eds.2011-1330. patients: 2008 update from the American College of Critical
Mermel, L.A., Allon, M., Bouza, E., et al. (2009). Clinical Care Medicine and the Infectious Diseases Society of
practice guidelines for the diagnosis and management of America. Critical Care Medicine 36(4): 1330–1349. doi:
intravascular catheter-related infection: 2009 update by the 10.1097/CCM.0b013e318169eda9.
Downloaded from https://onlinelibrary.wiley.com/doi/ by National Institute Of Standard, Wiley Online Library on [06/03/2024]. See the Terms and Conditions (https://onlinelibrary.wiley.com/terms-and-conditions) on Wiley Online Library for rules of use; OA articles are governed by the applicable Creative Commons License
483

116

A 3-Year-Old with Fever and Painful Urination


Roseann Hausmann
Seattle Children’s Hospital, Seattle, WA, USA

A 3-year-old male presents to the emergency department Family History


(ED) with a fever 38.9 C and painful urination.
The patient’s father has a history of chronic kidney disease
of unknown etiology that requires dialysis.

History of Present Illness


Current Status
This 3-year-old male began with fever 24 hours ago which
has been responsive to acetaminophen and ibuprofen at On arrival at the ED the child appeared ill with a rectal
home. He was potty-trained at 22 months of age, and has temperature of 39.3 C (102.7 F). He was tachycardic
recently begun to have episodes of incontinence. This after- with heart rate 150 bpm, blood pressure 80/50 mmHg,
noon he began complaining that it hurt to go to the bath- and respiratory rate of 40 breaths per minute. He
room, prompting parents to bring him to the ED. received intravenous acetaminophen and a normal
saline fluid bolus of 20 mL/kg. His vital signs have
now stabilized, with temperature 38 C (100.4 F), heart
rate 110 bpm, blood pressure 90/60 mmHg, and respira-
Past Medical History tory rate 25 breaths per minute. Results of complete
blood count, basic metabolic panel, and urinalysis are
The child was born at term and went home with his mother.
shown in Tables 116.1–116.3; results of urine culture
He was not circumcised. His immunization status is up to
and blood culture are pending.
date and he has continued to grow along the curve for both
height (50th percentile) and weight (40th percentile). He
had two urinary tract infections (UTIs) as an infant, and
Questions
was diagnosed with grade 3 vesicoureteral reflux (VUR)
and is taking prophylactic sulfamethoxazole trimethoprim.
Answer the following questions using the details
The parents report he has not missed any doses of medica-
provided.
tion in the previous month. He was hospitalized at 6 months
for a UTI with urosepsis, but has had no other hospital 1 What is the differential diagnosis and the most likely
admissions. diagnosis for this child?
2 What additional testing would be warranted?
3 What medications would you anticipate starting for this
patient?
Past Surgical History 4 Is surgical intervention indicated for this child?
5 What anticipatory guidance should be provided to the
None. family?

Cases in Pediatric Acute Care: Strengthening Clinical Decision Making, First Edition. Edited by Andrea M. Kline-Tilford and Catherine M. Haut.
© 2020 John Wiley & Sons Ltd. Published 2020 by John Wiley & Sons Ltd.
Downloaded from https://onlinelibrary.wiley.com/doi/ by National Institute Of Standard, Wiley Online Library on [06/03/2024]. See the Terms and Conditions (https://onlinelibrary.wiley.com/terms-and-conditions) on Wiley Online Library for rules of use; OA articles are governed by the applicable Creative Commons License
484 Cases in Pediatric Acute Care

Table 116.1 Complete blood count. VUR occurs when there is passage of urine from the blad-
der into the upper urinary tract. This predisposes patients to
White blood cell count 5400/mm3
recurrent UTIs as well as acute pyelonephritis due to the
Hemoglobin 11.4 g/dL transportation of bacteria from the bladder into the kidney.
Hematocrit 36% Left untreated, recurrent UTIs may result in renal scarring,
Platelet count 172 000/mm3 hypertension, and eventually end-stage renal disease. VUR
can be primary, due to inadequate closure of the ureterove-
sical junction (UVJ) or to a congenitally short intravesical
Table 116.2 Complete metabolic profile. ureter; or secondary, caused by high pressure in the bladder
that prevents the UVJ from closing during contraction,
Sodium 138 mEq/L
often due to anatomic or functional bladder obstruction.
Potassium 3.9 mEq/L Patients with primary VUR may have spontaneous resolu-
Chloride 100 mEq/L tion, with lower grades having the highest likelihood of this
Carbon dioxide 29 mEq/L response.
Blood urea nitrogen 12 mg/dL
Creatinine 0.24 mg/dL What additionally testing would be warranted?
Glucose 270 mg/dL
A renal and bladder ultrasound is recommended to assess
for pyelonephritis, or perirenal abscess. Close monitoring of
Table 116.3 Urinalysis. vital signs and blood pressure is also warranted. In a patient
who had not previously had a voiding cystourethrogram
Color Dark yellow (VCUG), this would be indicated given that it is a second
Clarity Cloudy febrile UTI and occurred while on prophylactic antibiotics.
pH 5 VCUG is recommended to be completed after resolution of
Specific gravity 1.022 symptoms, while still on antibiotics, or immediately after
the discontinuation of antibiotics.
Glucose 100 mg/dL
Nitrites Positive
Ketones Negative What medications would you anticipate starting
Leukocyte esterase Positive for this patient?
Bilirubin Negative Escherichia coli is the most common cause of UTI in chil-
Protein 100 mg/dL dren, so coverage of Gram-negative bacteria is impera-
Red blood cell count 1 /hpf tive. The patient will require parenteral inpatient
White blod cell count 1 /hpf antibiotic therapy, since he is ill-appearing on presenta-
tion. Antibiotic choice should be influenced by the resist-
ance pattern of E. coli in the local area; in this case the
Rationale and Evidence-based Practice agent of choice was ceftriaxone. Other choices would
include antibiotics in the cephalosporin category, includ-
Explanation
ing cefdinir, ceftibuten, and cefotaxime for best coverage
of Gram-negative organisms, or gentamicin. Once the
What is the differential diagnosis and the most
child has clinically improved and is able to tolerate oral
likely diagnosis for this child?
antibiotics, the agent can be switched based on culture
The differential diagnoses for this patient include UTI with susceptibilities.
urosepsis secondary to VUR, acute viral infection, or sepsis
of unknown origin.
Is surgical intervention indicated for this child?
The most likely diagnosis in this case would be UTI with
urosepsis secondary to VUR. The history of being uncir- Watchful waiting can be successful in the treatment of
cumcised and having VUR, as well as the increased epi- patients who have VUR and this decision was made for
sodes of incontinence in a previously potty-trained child this patient after his initial diagnosis of VUR. He was
and pain with voiding, provide a compelling history for this placed on antibiotic prophylaxis with sulfamethoxazole
diagnosis. Urinalysis demonstrating positive nitrites and trimethoprim during this time. Surgery is needed in
leukocyte esterase also supports the diagnosis. patients who persist with significant reflux past 3 years
Downloaded from https://onlinelibrary.wiley.com/doi/ by National Institute Of Standard, Wiley Online Library on [06/03/2024]. See the Terms and Conditions (https://onlinelibrary.wiley.com/terms-and-conditions) on Wiley Online Library for rules of use; OA articles are governed by the applicable Creative Commons License
Chapter 116 A 3-Year-Old with Fever and Painful Urination 485

of age, as the likelihood of self-resolution decreases at this and proteinuria. Constipation and bowel dysfunction can
point. It is also indicated in patients with higher grade increase the occurrence of UTIs, so it is important to ensure
reflux (>3 years old) who have breakthrough infections patients are having a soft bowel movement daily and may
on prophylactic therapy. require stool softeners to achieve this.

What anticipatory guidance should be provided to


Continuation of case: The patient is admitted to the
the family?
general acute care floor for management of his febrile
If surgery is performed endoscopically, there is an increased UTI and started on ceftriaxone and intravenous fluids.
risk of recurrent VUR compared with patients who have an Renal and bladder ultrasound demonstrates pyelonephri-
open surgical repair. Patients who have undergone treat- tis. Urology is consulted who plan to take the child to the
ment for VUR have an increased risk of renal scarring, operating room when afebrile for 48 hours for manage-
which can be associated with chronic kidney disease, and ment of VUR. After 48 hours, the urine culture taken in
as such should be followed closely for signs of impaired the ED demonstrates Escherichia coli that is sensitive to
renal functioning, including hypertension, poor growth, cefdinir.

Further Reading
American Academy of Pediatrics, Subcommittee on Urinary Shaikh, N., Borrell, J.L., Evron, J., and Leeflang, M.M. (2015).
Tract Infection, Committee on Quality Improvement, Procalcitonin, C-reactive protein, and erythrocyte
Subcommittee on Urinary Tract Infection (2016). sedimentation rate for the diagnosis of acute pyelonephritis
Reaffirmation of the Clinical Practice Guideline: the in children. Cochrane Database of Systematic Reviews (1):
diagnosis and management of initial urinary tract infection CD009185.
in febrile infants and young children, 2 to 24 months of age. Spencer, J.D., Schwaderer, A., McHugh, K., and Hains, D.S.
Pediatrics 138(6): 843–852. (2010). Pediatric urinary tract infections: an analysis of
Hoberman, A., Greenfeld, S.P., Mattoo, T.K., et al. (2014). hospitalizations, charges and costs in the USA. Pediatric
Antimicrobial prophylaxis for children with vesicoureteral Nephrology 25(2): 2469–2475.
reflux. New England Journal of Medicine 370(25): Tillman, B. and Wunsch, H. (2018). Epidemiology and
2367–2376. outcomes. Critical Care Clinics 34: 15–27.
ShaikhN.HobermanA.In: Comprehensive Pediatric Hospital White, B. (2011). Diagnosis and management of urinary tract
Medicine, 2nd edn (ed. L.B. Zaoutis and V.W. Chiang), infection. American Family Physician 83(4): 409–415.
559–565. Philadelphia: Mosby/Elsevier.
Downloaded from https://onlinelibrary.wiley.com/doi/ by National Institute Of Standard, Wiley Online Library on [06/03/2024]. See the Terms and Conditions (https://onlinelibrary.wiley.com/terms-and-conditions) on Wiley Online Library for rules of use; OA articles are governed by the applicable Creative Commons License
487

Index

Page locators in bold indicate tables. Page locators in italics indicate figures. This index uses letter-by-letter alphabetization.

a adenosine 70–72
abdominal pain adrenocorticotropic hormone (ACTH) stimulation test 9
acute jaundice 453 airway, breathing, and circulation (ABC) 213
after cardiac surgery 55–56, 56 albuterol 74, 354–355
diarrhea and lethargy 331–334, 332, 333 allergic reactions
fever 387–389, 388–389 billing and coding 38–40
fever and rash 446 hives and lip swelling 461–462
idiopathic nephrotic syndrome 457 vomiting 58
nausea and vomiting 371 allopurinol 63
splenic laceration 431–433, 432, 432 altered mental status 23–27, 24, 25
vomiting and diarrhea 449–451 ambiguous genitalia 261–263, 262
abdominal trauma 29–31, 30, 30 American Academy of Pediatrics (AAP)
abducens nerve palsy 204 acute hypertension and seizures 396–398
abnormal movements 357–359, 358 jaundice 162
acetaminophen otitis media 106
acetaminophen ingestion 207–210, 209–210 quality improvement 134
head injury 255–256, 258–259 respiratory syncytial virus and respiratory failure 52
acetazolamide 229 American College of Critical Care Medicine (ACCM) 179
activated charcoal 208 amiodarone 362
acute abdomen 215–217 amniotic membrane transplantation (AMT) 167
acute hyperglycemia 191–194, 192 amputation 154
acute hypertension and seizures 395–398, 396–397 amylase 432
acute inflammatory demyelinating polyradiculoneuropathy anaphylaxis/anaphylactic shock 212–213, 461–462
(AIDP) 404–405 anemia 265–267, 266, 266
acute jaundice 453–456, 454–455 anesthesia 58–59
acute kidney injury (AKI) 309–310 angiography 122, 122, 130, 131
acute liver failure 453–456, 454–455 angiotensin-converting enzyme (ACE) inhibitors 6
acute lymphoblastic leukemia (ALL) 266, 372–374 anion gap metabolic acidosis 197–198
acute mastoiditis 278–279 anterior mediastinal mass 373–374
acute motor axonal neuropathy/acute motor and sensory antiarrhythmics 117
axonal neuropathy (AMAN/AMSAN) 404–405 antibiotics
acute myeloid leukemia (AML) 266 complication of pneumonia 1–2
acute otitis media (AOM) 277–278, 291 fever and cough 271
acute pancreatitis 450–451 fever and rash 447–448
acute peritonitis 334 fever and redness behind ear 279
acute respiratory distress syndrome (ARDS) 157–159 fever and sore throat 171
acute respiratory failure/distress 251–254, 252, 252, 353 fever, rash and lethargy 320

Cases in Pediatric Acute Care: Strengthening Clinical Decision Making, First Edition. Edited by Andrea M. Kline-Tilford and Catherine M. Haut.
© 2020 John Wiley & Sons Ltd. Published 2020 by John Wiley & Sons Ltd.
Downloaded from https://onlinelibrary.wiley.com/doi/ by National Institute Of Standard, Wiley Online Library on [06/03/2024]. See the Terms and Conditions (https://onlinelibrary.wiley.com/terms-and-conditions) on Wiley Online Library for rules of use; OA articles are governed by the applicable Creative Commons License
488 Index

antibiotics (cont’d) benzodiazepines 82–84


fussiness and decreased oral intake 156, 157, 159 beta-agonists 465
fussy, tired infant 408 beta-blockers
generalized weakness 437 acute hypertension and seizures 398
influenza A and new fever 480–481 fussy, tired infant 409
kidney transplant complication 392–393, 477 murmur and chest flutter 350
left eye swelling 35 bilateral tympanostomy tubes 105–106
liver transplantation and fever 93–94 bilevel positive airway pressure (BiPAP)
lower respiratory tract infection 112 asthma 463–465
neonatal tachypnea 201–202 fever and cough 271
osteomyelitis 90–91 respiratory syncytial virus and respiratory failure 52–53
scald burn injury 311 sore throat and shortness of breath 337
shunted hydrocephalus 66–67 spinal muscular atrophy 299
skin wound in postoperative neonate 467, 469 biliary atresia 162
upper respiratory infection, headache and fever 237 bilirubin 93
antiemetics 12 billing and coding 37–45
antiepileptic drugs (AED) 82–84 ambulatory and inpatient billing encounters 40–41
antihistamines 315 bundled services 42, 43, 44
antihypertensives 221 charge generating procedure 38
anti-inflammatory medications 347 codes used to bill for services 38, 39
antinuclear antibody (ANA) 244–245 critical care management 41–44, 41
antipsychotics 297 documentation elements 38
antipyretics 17 documentation for critical care services 42
antiretroviral therapy (ART) 176 outpatient case documentation 38–40
apnea PICU case study 42–44
brain death 365 requirements for new providers 37
progressive lethargy and poor feeding 195 services not meriting critical care codes 42
sore throat and shortness of breath 336–337, 337 time-based codes 41–42, 42, 43–44
appendicitis 387–389, 388–389 transfers 41
arterial blood gas analysis see blood gas analysis biobanking 275
arteriovenous malformation (AVM) 121–123, 121–122 biopsy
aspirin 130–131 bone marrow transplant in sickle cell disease 424
asthma generalized weakness 438
acute respiratory failure 252–253 idiopathic nephrotic syndrome 459
outpatient management of asthma 353–356, 355 knee pain 153–154
status asthmaticus 463–465, 464, 464 tachypnea and cool extremities 225
atrioventricular block 140 blisters/bullae 165, 166
atrioventricular septal defect (AVSD) blood gas analysis
oral presentation and communication 339–344, 343 acute hyperglycemia 192
postoperative heart block 285–286, 286 altered mental status 24
autism spectrum disorders (ASD) 48 brain death 364
autoimmune hepatitis 454–455 desaturation in postoperative cardiac surgery infant 78
automatic implantable cardioverter defibrillator (AICD) 350 fussiness and decreased oral intake 156, 157
fussy, tired infant 408
b headache and seizure 120
back pain 385, 403–406, 404 neonatal tachypnea 200
bacteremia 380 outpatient management of asthma 355–356
bacterial meningitis 236–237 postoperative tachycardia 362
basal hormone testing 377 progressive lethargy and poor feeding 196
basic metabolic profile see metabolic profile scald burn injury 308
Beckwith–Weidemann syndrome 102–103 stridor 292
benign neonatal sleep myoclonus 357–359, 358 tachypnea and cool extremities 224
Downloaded from https://onlinelibrary.wiley.com/doi/ by National Institute Of Standard, Wiley Online Library on [06/03/2024]. See the Terms and Conditions (https://onlinelibrary.wiley.com/terms-and-conditions) on Wiley Online Library for rules of use; OA articles are governed by the applicable Creative Commons License
Index 489

unresponsive patient 326 carditis 346–347


weaning preterm infant from ventilation 248 ceftriaxone 252
blood in stool 85–87, 86 Centers for Disease Control and Prevention (CDC)
blood transfusion 313–315, 314 head injury 257–258
bloody diarrhea 125–126, 126 respiratory failure and transaminitis 428, 428
bone marrow biopsy/aspirate 62 sickle cell disease and fever 175
bone marrow transplant (BMT) 19–21, 20, 423–425 Centers for Medicare and Medicaid Services (CMS) 37–45
botulism 437–439 central line-associated bloodstream infection (CLABSI)
bowel necrosis 334 480–481
bowel perforation 334 central nervous system (CNS) tumors 204–206
brain death 363–366, 364 central venous pressure (CVP) 376–377
Brain Trauma Foundation 328, 328 cerebral angiography 122, 122
breastfeeding cerebral edema 193
neonatal hypoglycemia 101 cerebral palsy 177–181
neonatal tachypnea 202 cerebral salt wasting (CSW) 67, 376–377, 377
neurologic changes and respiratory failure 183 cerebrospinal fluid (CSF)
progressive lethargy and poor feeding 195 fever and rash 447
vomiting 59 generalized weakness 437–438, 437–438
breathlessness headache and seizure 121
sore throat and shortness of breath 335–338, 336, 337 hypothermia 380–381, 380
sudden onset of breathing difficulty 211–213 intractable headache 228, 228
bronchiolitis leg and back pain 404, 405
cough 73–75 neurologic changes and respiratory failure 187
fever and cold symptoms 15–17 progressive lethargy and poor feeding 195–196
fever and cough 270 shunted hydrocephalus 66–67, 66
respiratory syncytial virus and respiratory failure 52–53 tic disorders 412
bronchodilators 16 upper respiratory infection, headache and fever
bronchopulmonary dysplasia (BPD) 248–249 236–237, 236
Brudzinski sign 185 cerebrovascular accident (CVA) 282–283, 282, 284
bruising 61–63, 62, 62–63 cervicalgia 257
bundled services 42, 43, 44 CHARGE syndrome 211–213
burnout and resiliency 471–473, 472 chemotherapy
diplopia and esotropia 206
c knee pain 154
calcium channel blockers (CCB) pineoblastoma and bone marrow transplant 19
acute hypertension and seizures 398 chest flutter 349–351
altered mental status 24–27 chest X-ray
murmur and chest flutter 350 acute hypertension and seizures 395–396
calcium homeostasis 23–26 acute respiratory failure 252, 253
Campylobacter jejuni 405 altered mental status 23, 25
capillary blood gas analysis see blood gas analysis anemia 266
cardiac arrest 311 asthma 464
cardiac catheterization 130, 131 bruising 61–63, 62
cardiac output complication of pneumonia 1–2, 2
desaturation in postoperative cardiac surgery infant 78–79 cough 74–75
fussy, tired infant 408–409 fever and cold symptoms 15, 16, 17
tachypnea and cool extremities 224 fever and cough 270
cardiac surgery 55–56, 56 fussiness and decreased oral intake 157, 158, 158
cardiac tamponade 311, 373 fussy, tired infant 408
cardiomyopathy 407–409 generalized weakness 436
cardiopulmonary arrest 325, 327 headache and seizure 120
cardiopulmonary resuscitation (CPR) 308–310, 363 kidney transplant complication 476
Downloaded from https://onlinelibrary.wiley.com/doi/ by National Institute Of Standard, Wiley Online Library on [06/03/2024]. See the Terms and Conditions (https://onlinelibrary.wiley.com/terms-and-conditions) on Wiley Online Library for rules of use; OA articles are governed by the applicable Creative Commons License
490 Index

chest X-ray (cont’d) neurologic changes and respiratory failure 183–186,


lower respiratory tract infection 110 185, 186, 187
neonatal tachypnea 200 orthopnea 371–372, 372
neurologic changes and respiratory failure 184 pineoblastoma and bone marrow transplant 19
newborn immediately post delivery 141 right-sided weakness 283, 284
orthopnea 373 seizures 84
outpatient management of asthma 356 shunted hydrocephalus 65, 66
racing heart 116, 117 splenic laceration 431, 432, 433
respiratory decompensation in trached infant 98, 98 sudden onset of breathing difficulty 212
sickle cell disease and fever 174, 174 tic disorders 412
weaning preterm infant from ventilation 248–249 traumatic brain injury 376
Chiari I malformation 322–323, 412 unresponsive patient 326–327, 326–327
child abuse see physical abuse; sexual abuse upper respiratory infection, headache and fever 237
child assent 274–275 vomiting and diarrhea 450
chloramphenicol 447 concussion 257
chloroquine 429–430 confidentiality 275
chronic otitis media 105–107 congenital adrenal hyperplasia (CAH) 262–263
clonazepam 147 congenital hearing loss 48
Clostridium botulinum 438–439 congenital heart disease (CHD) 97–100, 98–99, 200–201
coagulation profile congenital hyperinsulinemia 442
acute jaundice 454 congested cough 15–16
bloody diarrhea 126 conjunctivitis 165, 167
fever and rash 447 continuous positive airway pressure (CPAP)
fussiness and decreased oral intake 158, 158 asthma 464
irritability in 3-day-old 443 desaturation in postoperative cardiac surgery infant 77
progressive lethargy and poor feeding 196 diaphoresis during feedings 6
scald burn injury 308 fever and cough 269, 271
unresponsive patient 326 neonatal tachypnea 201
upper respiratory infection, headache and fever 236 respiratory syncytial virus and respiratory failure 52–53
cognitive impairment 48 sore throat and shortness of breath 337
Collaborative Institutional Training Initiative (CITI) stridor 291, 293
program 274 cool extremities 223–225, 224
communication see oral presentation and communication corneal reflex 364–365, 437
community-acquired pneumonia (CAP) 270 corticosteroids
complex chronic condition 147–150 acute respiratory failure 253
complication of pneumonia 1–4, 2, 2–4 ambiguous genitalia 262–263
compression fractures 385 bone marrow transplant in sickle cell disease 424
computed tomography (CT) cough 74
abdominal pain, diarrhea and lethargy 333 Henoch–Schönlein purpura nephritis 220–221
abdominal trauma 29, 30, 31 outpatient management of asthma 354–355
acute respiratory failure 253 stridor 293
anemia 267 cough 73–75
brain death 363–364 acute respiratory failure 252–253
diplopia and esotropia 205–206 congested cough 15–16
fever and abdominal pain 387 fever and cough 269–272, 270, 270
fever and redness behind ear 278, 279 stridor 291
fever and sore throat 170, 170 C-peptide 401
headache and seizure 120–121, 121 crackles
head injury 256–258 cough 73–74
hypothermia 379 migratory joint pain 345
knee pain 153–154, 154 tachypnea and cool extremities 223
left eye swelling 34, 35 C-reactive protein (CRP)
Downloaded from https://onlinelibrary.wiley.com/doi/ by National Institute Of Standard, Wiley Online Library on [06/03/2024]. See the Terms and Conditions (https://onlinelibrary.wiley.com/terms-and-conditions) on Wiley Online Library for rules of use; OA articles are governed by the applicable Creative Commons License
Index 491

fever and redness behind ear 278 diazepam 147


hypothermia 380–381 diazoxide 401
migratory joint pain 346 diffuse midline gliomas 204–206
neonatal tachypnea 201 digoxin 6
osteomyelitis 90 dilated cardiomyopathy 408
persistent fever 130 diplopia and esotropia 203–206, 204, 205
critical blood sample 102, 103 disseminated intravascular coagulation (DIC) 159, 236–237
critical care management 41–44, 41 diuretics
croup 292–293 diaphoresis during feedings 6
C-spine injury 257 fussy, tired infant 409
current procedural terminology (CPT) codes 38, 41 murmur and chest flutter 350
Cutibacterium acnes 67 neonatal tachypnea 202
cytomegalovirus (CMV) 174–175, 174, 437 postoperative spinal fusion with complications 368
dizziness 255
d DMAIC process 133–134
Debrancher deficiency 443 domestic minor sex trafficking (DMST) 288–289
deep tendon reflex (DTR) dopamine 7
intractable headache 228 Down syndrome 337
neurologic changes and respiratory failure 185 doxycycline 430, 447
right-sided weakness 282 drooling 165, 167
dehydration duodenal obstruction 216
acute hyperglycemia 192–193
hypothermia 379 e
kidney transplant complication 391, 475–477 early transitional adaptive hypoglycemia 102
neurologic changes and respiratory failure 183, 184 echocardiography
osteomyelitis, electrolyte disturbance and headaches altered mental status 24
303–305 desaturation in postoperative cardiac surgery infant 77
poor weight gain and dehydration 143–145 diaphoresis during feedings 6
vomiting and diarrhea 12–13, 13 fussy, tired infant 408
see also electrolytes/fluid therapy migratory joint pain 346
delirium 295–298 newborn immediately post delivery 139, 141
contributing factors 296 orthopnea 372
environmental versus pharmacologic therapy 297 persistent fever 130–131, 131
pathophysiology 296 poor oral intake and irritability 70–71
screening tools 296–297 tachypnea and cool extremities 224
desaturation in postoperative cardiac surgery infant 77–79, 78 upper respiratory infection, headache and fever 237
desmopressin 377 weaning preterm infant from ventilation 248–249
dexamethasone 204–205, 263 electrocardiogram (EKG)
diabetes insipidus 67, 376–377, 377 altered mental status 23–24, 25
diabetes mellitus (DM) 143–145, 191–194, 192 anemia 267
diabetic ketoacidosis (DKA) 192–193 bruising 63
Diagnostic and Statistical Manual of Mental Disorders (DSM) diaphoresis during feedings 5–6
296–297 fussy, tired infant 409
dialysis 127 lightheadedness 415
diaphoresis during feedings 5–6 migratory joint pain 346
diarrhea murmur and chest flutter 349–351
abdominal pain, diarrhea and lethargy 331–334, 332, 333 newborn immediately post delivery 139–141, 140
bloody diarrhea 125–126, 126 persistent fever 130
idiopathic nephrotic syndrome 457 poor oral intake and irritability 69–72, 71
kidney transplant complication 391–392, 475–477 postoperative heart block 285–286, 286
osteomyelitis, electrolyte disturbance and headaches 303 postoperative tachycardia 361–362, 362
vomiting and diarrhea 11–14, 13, 449–451, 450 racing heart 115–118, 116–117
Downloaded from https://onlinelibrary.wiley.com/doi/ by National Institute Of Standard, Wiley Online Library on [06/03/2024]. See the Terms and Conditions (https://onlinelibrary.wiley.com/terms-and-conditions) on Wiley Online Library for rules of use; OA articles are governed by the applicable Creative Commons License
492 Index

electrocardiogram (EKG) (cont’d) gaining consensus on plan of care 178–179, 179


scald burn injury 308, 309–310, 310–311 palliative care 180
tachypnea and cool extremities 224 research review 273–275
electroencephalogram (EEG) script of family meeting 178
abnormal movements 358–359, 358 successful processes utilized in family meeting 179–180
brain death 365 evaluation and management (E/M) documentation 38,
generalized weakness 437–438 39, 40–41
headache and seizure 121 exercise/physical activity
neurologic changes and respiratory failure 186 lightheadedness 416
seizures 81–83, 83 murmur and chest flutter 350–351
tic disorders 412 poor weight gain and dehydration 145
electromyelogram (EMG) 437–438 external bracing 385
electronic medical records (EMR) 38, 135 external ventricular drain (EVD) 121
emesis see vomiting extracorporeal cardiopulmonary resuscitation (ECPR) 308–310
emotional intelligence 472 extracorporeal membrane oxygenation (ECMO) 23, 25–27,
empyema 2–4, 4 309–311
encephalitis 187–188, 188 exudative effusion see empyema
end of life 149 eye swelling 33–35, 34–35, 34–35
endoscopic retrograde cholangiopancreatography (ERCP)
31, 94, 162 f
endotracheal tubes 187 facial nerve palsy 204
end-stage renal disease (ESRD) 391–393, 392 failure mode and effects analysis (FMEA) 133–134
enema reduction 333–334 failure to thrive 144
enterovirus 111 febrile neutropenia 20–21
environmental modifications 297 feeding/nutritional support
epididymitis 241 acute jaundice 456
epilepsy blood in stool 87
complex chronic condition 147–149 diaphoresis during feedings 5–6
seizures 81–84, 82, 83 fussiness and decreased oral intake 155–159, 156, 158, 158
seizures in school-age child 231–233, 232 hyponatremia 419–421
epinephrine irritability in 3-day-old 441–443
cough 74 lower respiratory tract infection 111–112
hives and lip swelling 462 neonatal tachypnea 201–202
hypotension in the operating room 9 neurologic changes and respiratory failure 189
stridor 291–293 poor oral intake and irritability 69–72, 70, 71
sudden onset of breathing difficulty 213 poor weight gain and dehydration 143–145
tachypnea and cool extremities 225 progressive lethargy and poor feeding 195–198, 196
unresponsive patient 325 rash and drooling 167
epistaxis 255 spinal muscular atrophy 299–301, 300
Epstein–Barr virus (EBV) 174–175 splenic laceration 432–433
erythema 469 vomiting 59
erythematous rash with macules 318, 446, 446 fentanyl 7
erythrocyte sedimentation rate (ESR) fever
fever and rash 244–245 acute jaundice 453
knee pain 153 and abdominal pain 387–389, 388–389
migratory joint pain 346 and cold symptoms 15–17, 16
Escherichia coli 86–87, 126–127, 393, 484 and cough 269–272, 270, 270
esotropia see diplopia and esotropia and painful urination 483–485, 484
ethics 177–181 and rash 243–246, 244–245, 445–448, 446, 446
case notes and history 177–178 and redness behind ear 277–279, 278, 279
consulting an ethics committee 181 and sore throat 169–171, 170, 171
essential components of family meeting 179, 180 liver transplantation and fever 93–95, 94
Downloaded from https://onlinelibrary.wiley.com/doi/ by National Institute Of Standard, Wiley Online Library on [06/03/2024]. See the Terms and Conditions (https://onlinelibrary.wiley.com/terms-and-conditions) on Wiley Online Library for rules of use; OA articles are governed by the applicable Creative Commons License
Index 493

osteomyelitis 89 osteomyelitis, electrolyte disturbance and headaches


persistent fever 129–131, 130, 131 303–305, 304
rash and lethargy 317–320, 318, 318 sore throat and shortness of breath 337
sickle cell disease and fever 173–176, 174, 174–175 tic disorders 411–413
upper respiratory infection, headache and fever upper respiratory infection, headache and fever
235–238, 236 235–238, 236
flaccid myelitis 188 head injury 255–259, 256
flexible bronchoscopy 253 Health Insurance Portability and Accountability Act
fludrocortisone 263 (HIPAA) 44
fluorescence in situ hybridization (FISH) 263 hearing loss 48
fluoroscopy 31 heart block 141, 285–286, 286
focal segmental glomerulosclerosis (FSGS) 458, 475–478, Heliox therapy 293
476 hemodialysis
football injury 431–433, 432, 432 altered mental status 26–27
foreign body aspiration 252–253 progressive lethargy and poor feeding 198
fussiness and decreased oral intake 155–159, 156, 158, 158 scald burn injury 311
fussy, tired infant 407–409, 408, 408 hemoglobin/hematocrit 78
[beta-]hemolytic streptococcal pharyngitis 346–347
g hemolytic uremic syndrome (HUS) 86–87, 126–127
gamma-glutamyltransferase (GGT) 93–94 hemophagocytic lymphohistiocytosis (HLH) 454–455
gastroenteritis Henoch–Schönlein purpura (HSP) nephritis
abdominal pain, diarrhea and lethargy 332 219–221, 220
acute abdomen 216 hepatic panel 94
vomiting and diarrhea 12–13 hepatic phosphorylase deficiency 443
gastroesophageal reflux disease (GERD) 58, 216 herpes simplex virus (HSV) 380–381
generalized periodic epileptiform discharges (GPED) 82 high-flow nasal cannula (HFNC)
generalized weakness 435–439, 436–438 complication of pneumonia 1–2
genetic testing 262 cough 74–75
gestational diabetes 101 respiratory syncytial virus and respiratory failure 51–53
Glasgow Coma Scale (GCS) spinal muscular atrophy 299
abdominal trauma 29 stridor 291, 293
brain death 363 history component codes 38, 39
head injury 255–256 HIV/AIDS 174–176, 175
hypothermia 380 hives and lip swelling 461–462
splenic laceration 431 hoarseness 165, 167
traumatic brain injury 375 human leukocyte antigen (HLA) 424
glucose 6-phosphatase deficiency 443 human trafficking 287–289
glucose tests 101–102, 102 screening and identification 289
glycogen synthetase deficiency 443 signs and common complaints 288–289
good clinical practice (GCP) 274 types and prevalence 288
graft-versus-host disease (GVHD) 423–424 hydrocephalus 65–68, 66, 66
grunting 1 hydrocortisone 263
Guillain–Barré syndrome (GBS) 404–406 β-hydroxybutyrate (BOHB) 193
hyperactive delirium 296
h hyperammonemia 197–198
Haemophilus influenzae 35, 236 hyperbilirubinemia 162
headache hyperglycemia 191–194, 192
acute hyperglycemia 193 hyperinsulinemia 400–401, 442
acute hypertension and seizures 395 hyperinsulinemic euglycemia (HIE) therapy 25–26
and seizure 119–123, 120, 121–122 hyperkalemia 62, 63, 309–310
head injury 257–259 hyperleukocytosis 266–267
intractable headache 227–229, 228, 228 hypernatremia 304–305, 376
Downloaded from https://onlinelibrary.wiley.com/doi/ by National Institute Of Standard, Wiley Online Library on [06/03/2024]. See the Terms and Conditions (https://onlinelibrary.wiley.com/terms-and-conditions) on Wiley Online Library for rules of use; OA articles are governed by the applicable Creative Commons License
494 Index

hyperoxia test 201 immunoglobulin A vasculitis see Henoch–Schönlein purpura


hyperparathyroidism 232 nephritis
hypersensitivity reactions 208 immunosuppressive therapy
hypertension fever and rash 246
acute hypertension and seizures 395–398, Henoch–Schönlein purpura nephritis 221
396–397 idiopathic nephrotic syndrome 458–459
fever and rash 244–245 inborn error of metabolism (IEM) 196–198
hypertrophic cardiomyopathy (HCM) 349–351, 409 indomethacin 248
hypertrophic pyloric stenosis (HPS) 57–58 inflammatory markers 326
hypoactive delirium 296 influenza A and new fever 479–481, 480
hypocalcemia 232–233 influenza vaccine 405–406
hypocomplementemia 245 informed consent 274–275
hypoglycemia inotropes 392–393, 409
irritability in 3-day-old 441–443 insect stings 461–462
jittery 2-week-old infant 399–401, 400 Institutional Review Board (IRB) 136–137, 136, 274
neonatal hypoglycemia 101–103, 102–103 insulin-dependent diabetes mellitus (IDDM) see type 1 diabetes
progressive lethargy and poor feeding 197–198 mellitus
hyponatremia 419–421, 420 insulin therapy 102, 193
acute hyperglycemia 193 intellectual disability 48, 177–181
postoperative spinal fusion with complications 368–369 interleukin (IL)-6 380
shunted hydrocephalus 67 intracranial pressure (ICP) 120–122, 205
traumatic brain injury 377 intractable headache 227–229, 228, 228
hypoparathyroidism 232–233 intravenous immunoglobulin (IVIG)
hypotension leg and back pain 405
altered mental status 23, 26 neurologic changes and respiratory failure 188
in the operating room 7–9, 8 persistent fever 130–131
hypothermia 365, 379–381, 380 rash and drooling 167
hypotonia 177–181, 436–437 intraventricular hemorrhage (IVH) 248, 249
hypovolemia/hypovolemic shock intussusception 216, 332–334, 333
hyponatremia 368 IPASS Handoff program 342–344, 343
kidney transplant complication 392 irritability
lightheadedness 416 in 3-day-old 441–444, 443
scald burn injury 311 poor oral intake and irritability 69–72, 70, 71
traumatic brain injury 376
vomiting and diarrhea 12–13 j
hypoxia jaundice 161–163, 162, 453–456, 454–455
acute respiratory failure 251–253 jittery 2-week-old infant 399–401, 400
asthma 463 jittery 3-day-old 441
complication of pneumonia 1–2 joint pain 220–221
fever and cough 269–271 Jones criteria 346–347, 347
hyponatremia 419–420 junctional ectopic tachycardia (JET) 361–362, 362, 362
outpatient management of asthma 353
sore throat and shortness of breath 337 k
sudden onset of breathing difficulty 212 Kawasaki disease 130, 166
hypoxic ischemic encephalopathy 81, 327–329 Kayser–Fleischer rings 454–455
Kernig sign 185
i ketorolac 258
iatrogenic withdrawal syndrome (IWS) 296 kidney transplant complication 391–393, 392, 475–478, 476
ibuprofen 248, 256, 259 knee pain 151–154, 152–154, 152
idiopathic nephrotic syndrome 457–459, 458
idiopathic scoliosis 367 l
immunocompromised patient 19–21, 392 lactic acidosis 401
immunoglobulin A nephropathy 458 Ladd procedure 216–217
Downloaded from https://onlinelibrary.wiley.com/doi/ by National Institute Of Standard, Wiley Online Library on [06/03/2024]. See the Terms and Conditions (https://onlinelibrary.wiley.com/terms-and-conditions) on Wiley Online Library for rules of use; OA articles are governed by the applicable Creative Commons License
Index 495

language delay 47–49 malaria 427–430, 428–429, 428


laparoscopic appendectomy 389 malnutrition 299–301, 300
left eye swelling 33–35, 34–35, 34–35 malrotation 215–217
leg and back pain 403–406, 404 mechanical ventilation see ventilation
lethargy meconium aspiration syndrome (MAS) 200–201
abdominal pain, diarrhea and lethargy 331–334, 332, 333 Medicare/Medicaid see Centers for Medicare and Medicaid
fever and rash 445–446 Services
fever, rash and lethargy 317–320, 318, 318 membranoproliferative glomerulonephritis (MPGN) 458
hypothermia 379 meningismus 155
progressive lethargy and poor feeding 195–198, 196 meningitis
vomiting and diarrhea 11 fussiness and decreased oral intake 156, 157, 159
leukemia 62–63, 266–267 hypothermia 380
leukocytosis 62 neurologic changes and respiratory failure 184
lightheadedness 415–415, 416 upper respiratory infection, headache and fever 236–237
lipase 432 meningococcemia 156, 158, 159
lipid emulsion therapy 26–27 mental status
lipid profile 450 altered mental status 23–27, 24, 25
lip swelling 461–462 cognitive impairment 48
literature review 135 generalized weakness 437
liver failure 208, 210, 210 metabolic acidosis 325
liver function test 453–454, 454 metastatic disease 19–21, 153
liver transplantation and fever 93–95, 94 metered-dose inhalers (MDI) 133–137, 136, 143
lorazepam 119 methicillin-resistant Staphylococcus aureus (MRSA)
loss of consciousness (LOC) 255 fever and sore throat 171
lower respiratory tract infection 109–113, 110, 110, 112 fever, rash and lethargy 319–320
lumbar puncture influenza A and new fever 481
fussiness and decreased oral intake 157 osteomyelitis 90
intractable headache 229 shunted hydrocephalus 67
neurologic changes and respiratory failure 186 methicillin-sensitive Staphylococcus aureus (MSSA) 67
Lyme meningitis 228 methylprednisolone 424
lymph nodes 61, 143–144 middle ear ventilation disorders 48
migraine
m headache and seizure 119
magnesium sulfate 355 head injury 258
magnetic resonance cholangiogram (MRCP) 94 intractable headache 227–229, 228, 228
magnetic resonance imaging (MRI) migratory joint pain 345–347, 346–347
diplopia and esotropia 203–206, 205 milk-protein allergy 58
fever and abdominal pain 387–389 milrinone 225
fever and redness behind ear 278 minimal change nephrotic syndrome (MCNS) 458
generalized weakness 437–438 mononucleosis 173
head injury 258 multiorgan dysfunction syndrome (MODS) 309–310
intractable headache 228, 228 murmur 69
knee pain 153, 153 murmur and chest flutter 349–351
left eye swelling 34 myectomy 351
leg and back pain 403, 405 myocarditis 223–225, 224, 447–448
neurologic changes and respiratory failure 186, 187, 187,
188, 189 n
osteomyelitis 90–91, 91 N-acetylcysteine (NAC) 207–208
right-sided weakness 282–283, 282, 284 narrow-complex tachycardia 69–72
scoliosis 322–323 nasal flaring 1
seizures 84 nasogastric (NG) tubes
tic disorders 412 blood in stool 87
unresponsive patient 327 shunted hydrocephalus 65
Downloaded from https://onlinelibrary.wiley.com/doi/ by National Institute Of Standard, Wiley Online Library on [06/03/2024]. See the Terms and Conditions (https://onlinelibrary.wiley.com/terms-and-conditions) on Wiley Online Library for rules of use; OA articles are governed by the applicable Creative Commons License
496 Index

nasogastric (NG) tubes (cont’d) otitis media (OM)


spinal muscular atrophy 301 acute otitis media 277–278, 291
National Provider Identifier (NPI) 37 chronic otitis media 105–107
nausea and vomiting language delay 48
headache and seizure 119 outpatient management of asthma 353–356, 355
orthopnea 371 oxygen therapy/support
seizures 231 altered mental status 23, 25–27
necrotizing enterocolitis (NEC) 248–249 anemia 266–267
necrotizing fasciitis 467–470, 468–469, 468 asthma 463–465
negative inspiratory force (NIF) 404–405 complication of pneumonia 1–2
Neisseria meningitidis 156, 236–237 cough 74–75
neonatal hypoglycemia 101–103, 102–103, 441–444, 443 diaphoresis during feedings 6
neonatal tachypnea 199–202, 200, 200 fever and cold symptoms 16
nephritis 244–246 fever and cough 269, 271
nephrotic syndrome (NS) 457–459, 458 fussy, tired infant 408–409
neurologic disorders lower respiratory tract infection 111–112
and respiratory failure 183–189, 184, 186, 185–187, 188 neonatal tachypnea 199, 201
headache and seizure 119–123, 120, 121–122 outpatient management of asthma 354–355
seizures 81–84, 82, 83 poor oral intake and irritability 72
newborn immediately post delivery 139–141, 140–141 respiratory decompensation in trached infant 97–99
nocturnal polysomnography 337, 337 respiratory failure and transaminitis 428–429
non-accidental trauma see physical abuse respiratory syncytial virus and respiratory failure 51–53
non-invasive positive pressure ventilation (NIPPV) 271 scald burn injury 309–311
nonsteroidal anti-inflammatory drugs (NSAID) sore throat and shortness of breath 335–337
back pain 385 stridor 291–293
fever and rash 246 tachypnea and cool extremities 224–225
head injury 255–256, 258
Henoch–Schönlein purpura nephritis 220–221 p
norepinephrine 9 pacemakers
nutritional support see feeding/nutritional support newborn immediately post delivery 139–141, 140–141
postoperative heart block 285–286, 286
o packed red blood cell (PRBC) transfusion 158–159
obesity 101 painful urination 483–485, 484
obstructive sleep apnea (OSA) 336–337, 337 palivizumab 75
opioid analgesics 451 palliative care
oral presentation and communication 339–344 brain death 366
barriers to ideal communication during handoff 341–342 ethical case study 180
describing a comprehensive oral presentation 340–341 palpitations 349–351
key components for new admission 340 pancreatic enzymes 450
key components of handoff communication 342–344, 343 pancreatitis 450–451
obtaining important information during handoff 341 panic attacks 212
shift-to-shift handoffs 342 parathyroid hormone (PTH) 232–233
structured/standardized handoff tools 342 parvovirus requiring blood transfusion 313–315, 314
orbital cellulitis 34–35 patent ductus arteriosus (PDA) 247–249
orchiectomy 241 patent foramen ovale (PFO) 70
orthopnea 371–374, 372, 372 pathologic lead point 334
orthostatic blood pressure 415, 416 Pediatric Emergency Care Applied Research Network
Osgood–Schlatter disease 151–152 (PECARN) 257–258
osteomyelitis Pediatric Evaluation of Developmental Status (PEDS)
case study 89–91, 90, 91 tool 47–49
electrolyte disturbance and headaches 303–305, 304 pediatric nurse practitioner (PNP) 37–38, 40–41, 43–44
osteosarcoma 153–154 pericardial effusion 371–374
Downloaded from https://onlinelibrary.wiley.com/doi/ by National Institute Of Standard, Wiley Online Library on [06/03/2024]. See the Terms and Conditions (https://onlinelibrary.wiley.com/terms-and-conditions) on Wiley Online Library for rules of use; OA articles are governed by the applicable Creative Commons License
Index 497

periodic lateralizing epileptiform discharges (PLED) 82 respiratory decompensation in trached infant 98


periorbital edema 33, 34 respiratory failure and transaminitis 429
peripheral blood smear 63 stridor 291, 293
peripherally inserted central catheter (PICC) 67 post-coarctectomy syndrome 56
peripheral pulmonic stenosis (PPS) 70 postoperative heart block 285–286, 286
peritonsillar abscess 170 postoperative spinal fusion with complications 367–369, 368
persistent emesis 58–59 postoperative tachycardia 78–79, 361–362, 362, 362
persistent fever 129–131, 130, 131 postural tachycardia syndrome (POTS) 415–416, 416
persistent hemodynamic instability 23 prednisolone 263, 458–459
persistent pulmonary hypertension of the newborn (PPHN) prednisone 458
200–201 prenatal laboratory tests 102
petechiae 155–156 preseptal cellulitis 34
petechial rash 61 pressure-equalizing (PE) tubes 105–106
pharyngitis 169 progressive lethargy and poor feeding 195–198, 196
phenobarbital 83 provider burnout and resiliency 471–473, 472
phonophobia 227–228 Pseudomonas aeruginosa 112, 279, 481
photophobia 227–228, 411 pseudotumor cerebri syndrome (PTCS) 228–229
physical abuse pulmonary edema
brain death 363–366, 364 altered mental status 26
unresponsive patient 327–329, 328 desaturation in postoperative cardiac surgery infant 78
Pierre Robin syndrome 337 migratory joint pain 345–347
pineoblastoma and bone marrow transplant 19–21, 20 pulmonary hypertension 78–79
Plan, Do, Study, Act (PDSA) 133–134 pulse oximetry 199
plaques 166 pyloric regimen 59
Plasmodium spp. 428–430 pyloric stenosis (PS) 57–59, 216
pleural effusion 3, 371–374 pyloromyotomy 58–59
pneumonia
acute respiratory failure 252–253 q
billing and coding 42–44 quality improvement (QI) 133–137, 136
complication of pneumonia 1–4, 2, 2–4 appropriate goals for the problem 135
fever and cough 270 appropriate question for case scenario 135
influenza A and new fever 480 barriers to implementation 135–136
upper respiratory infection, headache and fever 236 defining research versus QI 136
point-of-care test (POCT) glucose 191, 193 final steps 137
point-of-care ultrasound (POCUS) 116 Institutional Review Board 136–137, 136
poisoning and toxic events key members of QI team 133–134
acetaminophen ingestion 207–210, 209–210 literature review 135
altered mental status 23–27 measurement of outcomes 136
generalized weakness 438–439 quinidine 430
poliovirus 188
polymerase chain reaction (PCR) r
acute jaundice 454, 455 racing heart 115–118, 116–118, 117
fever and rash 447 radiography
fussy, tired infant 408 abdominal pain after cardiac surgery 56
sickle cell disease and fever 174–175 acute abdomen 216
upper respiratory infection, headache and fever 237 back pain 385
polysomnography (PSG) 337, 337 head injury 256–258, 256
poor oral intake and irritability 69–72, 70, 71 jaundice 162
poor weight gain and dehydration 143–145 knee pain 151–153, 152
positive end-expiratory pressure (PEEP) osteomyelitis 89–90
desaturation in postoperative cardiac surgery infant 77, 79 scoliosis 322
fussiness and decreased oral intake 158 stridor 293
Downloaded from https://onlinelibrary.wiley.com/doi/ by National Institute Of Standard, Wiley Online Library on [06/03/2024]. See the Terms and Conditions (https://onlinelibrary.wiley.com/terms-and-conditions) on Wiley Online Library for rules of use; OA articles are governed by the applicable Creative Commons License
498 Index

radiography (cont’d) scald burn injury 307–312, 308, 309–310


see also chest X-ray scoliosis 321–323, 322, 367
radiotherapy 206 sedation
range of motion (ROM) 321 delirium 295–296
rapid diagnostic test (RDT) 428, 429 desaturation in postoperative cardiac surgery infant 77–79
rasburicase 63 orthopnea 373
rash seizures 81–84, 82, 83
and drooling 165–168, 166–167, 166 acute hypertension and seizures 395–398, 396–397
bone marrow transplant in sickle cell disease 423 complex chronic condition 147–149
fever and rash 243–246, 244–245, 445–448, 446, 446 headache and seizure 119–123, 120, 121–122
fever, rash and lethargy 317–320, 318, 318 hyponatremia 419–420
redness behind ear 277–279, 278, 279 in school-age child 231–233, 232
refeeding syndrome 300–301 sepsis/septic shock
referral 48 delirium 295
reflective ability 472 fussiness and decreased oral intake 155–156
renal parenchymal disease 396–397 hypothermia 380–381
research review 273–275 irritability in 3-day-old 442
consents and assents 274–275 kidney transplant complication 392
participants 273 parvovirus requiring blood transfusion 315
protection of human subjects 273–274 pineoblastoma and bone marrow transplant 21
responsibilities for data collection 275 scald burn injury 309–310
training programs 274 shunted hydrocephalus 66
resiliency see burnout and resiliency skin wound in postoperative neonate 469
respiratory decompensation in trached infant 97–100, 98–99 upper respiratory infection, headache and fever 236–237
respiratory distress syndrome (RDS) 200–201 septic arthritis 89–90, 151–152
respiratory failure serology 447
generalized weakness 435 serum creatinine 476–477
neurologic changes and respiratory failure 183–189, sexual abuse 287–289
184, 186, 185–187, 188 sexually transmitted infections (STI) 287–288
respiratory syncytial virus 51–53 shiga-like toxin-producing Escherichia coli hemolytic uremic
transaminitis 427–430, 428–429, 428 syndrome (STEC-HUS) 86–87, 126–127
respiratory infections 20–21, 110, 111 short-acting beta-agonists (SABA) 354–355
respiratory syncytial virus (RSV) 51–53, 129 shunted hydrocephalus 65–68, 66, 66
reticulocyte count 314 sickle cell disease (SCD)
retinal hemorrhage 328, 328 and fever 173–176, 174, 174–175
retropharyngeal abscess (RPA) 170–171, 170 bone marrow transplant 423–425
return of spontaneous circulation (ROSC) 325 right-sided weakness 281–284, 282, 282, 284
rheumatic heart disease 346–347, 347 sinus bradycardia 23, 25, 26–27
rhinorrhea 15–16, 195 sinusitis 34
rhinosinusitis 34 sinus tachycardia 5–6
rhinovirus 111 Six Sigma DMAIC process 133–134
rhombencephalitis 187–188 skin wound in postoperative neonate 467–470, 468–469, 468
rhonchi 73 sleep myoclonus 357–359, 358
Rickettsia rickettsii 446–447 SMART objectives 135
right-sided weakness 281–284, 282, 282, 284 social competency 472
Rocky Mountain spotted fever (RMSF) 445–448, 446, 446 social support 472
root cause analysis (RCA) 133–134 sore throat
Rumack–Matthew nomogram 208, 209 and shortness of breath 335–338, 336, 337
fever and 169–171, 170, 171
s speech therapy 49
salbutamol 202 spinal fracture 383
SBAR handoff 354–355, 355 spinal muscular atrophy (SMA) 299–301, 300
Downloaded from https://onlinelibrary.wiley.com/doi/ by National Institute Of Standard, Wiley Online Library on [06/03/2024]. See the Terms and Conditions (https://onlinelibrary.wiley.com/terms-and-conditions) on Wiley Online Library for rules of use; OA articles are governed by the applicable Creative Commons License
Index 499

splenic laceration 431–433, 432, 432 T-cell acute lymphoblastic leukemia 372–374
sputum culture 112, 112 team culture 472
Staphylococcus aureus testicular torsion 239–241, 240, 240
fever and redness behind ear 279 tethered cord 322–323
fever, rash and lethargy 319 tetralogy of Fallot (TOF) 211–213, 361
influenza A and new fever 480–481 thrombocytopenia 62
left eye swelling 35 thyroid-stimulating hormone (TSH) 116
osteomyelitis 90 tic disorders 411–413
see also methicillin-resistant Staphylococcus aureus time-based codes 41–42, 42, 43–44
status asthmaticus 463–465, 464, 464 tonic–clonic seizures 119
status epilepticus 81, 84, 147 Tourette syndrome 411–413
stay sutures 97, 98, 99 toxic shock syndrome (TSS) 319–320
Stenotrophomonas maltophilia 112 toxic synovitis 89–90
steroid-induced hyperglycemia 144 tracheitis 480
steroids tracheobronchoscopy 99, 99, 100
ambiguous genitalia 262 tracheostomy 97–100, 177–181, 189
neurologic changes and respiratory failure 188 transaminitis 427–430, 428–429, 428
sudden onset of breathing difficulty 213 transcranial Doppler (TCD) 281, 283
see also corticosteroids transfers 41
Stevens–Johnson syndrome and toxic epidermal necrolysis transient hyperinsulinemic hypoglycemia 102
(SJS/TEN) 166–167 transient tachypnea of the newborn (TTN) 199–202, 200, 200
Streptococcus spp. trauma
migratory joint pain 346–347 abdominal trauma 29–31, 30, 30
osteomyelitis 90 splenic laceration 431–433, 432, 432
sore throat and shortness of breath 336 traumatic brain injury (TBI)
upper respiratory infection, headache and fever 236–237 brain death 363–366, 364
stridor 291–294, 292, 335–336 complication of 375–378, 376, 376–377
substance use/abuse 287–289, 449–451 head injury 257–258
sudden cardiac death 350–351 unresponsive patient 327–329, 328
sudden onset of breathing difficulty 211–213 triple flexion reflex 187
sumatriptan 258 trisomy 21 339–344, 343
superior vena cava syndrome 373 tumor lysis syndrome (TLS) 62, 266–267, 373–374
supraventricular tachycardia (SVT) 70–72, 117–118 type 1 diabetes mellitus 143–145, 192–193
swollen eye 33–35, 34–35, 34–35
swollen lip 461–462 u
synchronized cardioversion 72 ulceration 468
synchronized intermittent mandatory ventilation (SIMV) 252 ultrasonography
syndrome of inappropriate antidiuretic hormone (SIADH) 67, abdominal pain, diarrhea and lethargy 333, 333
368–369, 376–377, 377 acute abdomen 216
syrinx 322–323 fever and abdominal pain 387–389
systemic inflammatory response syndrome (SIRS) 20, 309 Henoch–Schönlein purpura nephritis 221
systemic lupus erythematosus (SLE) 139, 244–246, 245 jaundice 162
liver transplantation and fever 94
t neurologic changes and respiratory failure 183
tachypnea racing heart 116
and cool extremities 223–225, 224 testicular torsion 240–241
complication of pneumonia 1 vomiting 58
fever and cough 269–271 vomiting and diarrhea 449–450
migratory joint pain 345 weaning preterm infant from ventilation 249
neonatal tachypnea 199–202, 200, 200 unresponsive patient 325–329, 326–328, 326, 328, 441
oral presentation and communication 339 upper respiratory infection (URI)
respiratory syncytial virus and respiratory failure 51–52 fever and sore throat 169–170
Downloaded from https://onlinelibrary.wiley.com/doi/ by National Institute Of Standard, Wiley Online Library on [06/03/2024]. See the Terms and Conditions (https://onlinelibrary.wiley.com/terms-and-conditions) on Wiley Online Library for rules of use; OA articles are governed by the applicable Creative Commons License
500 Index

upper respiratory infection (URI) (cont’d) complex chronic condition 148–149


seizures 81 delirium 295
with headache and fever 235–238, 236 fever and cough 271
uric acid 63 fussiness and decreased oral intake 156–158
urinalysis fussy, tired infant 408–409
acute hyperglycemia 192 leg and back pain 404
acute hypertension and seizures 396 lower respiratory tract infection 111–112
blood in stool 86 neurologic changes and respiratory failure 185, 189
bloody diarrhea 126 orthopnea 373
fever and abdominal pain 388 progressive lethargy and poor feeding 198
fever and painful urination 484 respiratory failure and transaminitis 428–429
fever and rash 244 weaning preterm infant from mechanical ventilation
Henoch–Schönlein purpura nephritis 220 247–249, 248–249, 248
idiopathic nephrotic syndrome 458, 459 ventricular septal defect (VSD) 6
kidney transplant complication 476, 477 ventriculoperitoneal (VP) shunt 65–68
neurologic changes and respiratory failure 184 vesicoureteral reflux (VUR) 483–485, 484
pineoblastoma and bone marrow transplant 21 viral encephalitis 228–229
postoperative spinal fusion with complications 368 viral meningitis 236
progressive lethargy and poor feeding 196 vital capacity (VC) 405–406
splenic laceration 432 vitamin D deficiency 232, 385
testicular torsion 240, 241 vomiting 57–59, 59
traumatic brain injury 375 abdominal pain after cardiac surgery 55–56
vomiting and diarrhea 450 acute abdomen 215
urinary tract infection (UTI) and diarrhea 449–451, 450
fever and painful urination 483–485 vomiting and diarrhea 11–14, 13
influenza A and new fever 480 see also nausea and vomiting
kidney transplant complication 392–393, 477
pineoblastoma and bone marrow transplant 20–21 w
wheezing
v acute respiratory failure 252–253
VALUE acronym 179 asthma 463
vancomycin 271 cough 73–74
vasodilators 78–79, 398 fever and cold symptoms 16
vasopressors 157 hives and lip swelling 461
venous–arterial extracorporeal membrane oxygenation (VA- outpatient management of asthma 353
ECMO) 97 respiratory syncytial virus and respiratory failure 51–53
venous blood gas analysis see blood gas analysis sudden onset of breathing difficulty 212
ventilation white blood cell (WBC) count 90
acute respiratory failure 251–253 Wilson disease 454–456, 455
asthma 463–465 Wolff–Parkinson–White (WPW) syndrome 71–72,
brain death 363–364 117–118

You might also like